30510870 Cost Accounting and Financial Management

921
PROFESSIONAL COMPETENCE COURSE STUDY MATERIAL COST ACCOUNTING AND FINANCIAL MANAGEMENT

Transcript of 30510870 Cost Accounting and Financial Management

Page 1: 30510870 Cost Accounting and Financial Management

PROFESSIONAL COMPETENCE COURSE

STUDY MATERIAL

COST ACCOUNTING AND FINANCIAL MANAGEMENT

Page 2: 30510870 Cost Accounting and Financial Management

PAPER 4

Cost Accounting and Financial Management

BOARD OF STUDIES THE INSTITUTE OF CHARTERED ACCOUNTANTS OF INDIA

Page 3: 30510870 Cost Accounting and Financial Management

This study material has been prepared by the faculty of the Board of Studies. The objective of the study material is to provide teaching material to the students to enable them to obtain knowledge and skills in the subject. Students should also supplement their study by reference to the recommended text books. In case students need any clarifications or have any suggestions to make for further improvement of the material contained herein, they may write to the Director of Studies.

All care has been taken to provide interpretations and discussions in a manner useful for the students. However, the study material has not been specifically discussed by the Council of the Institute or any of its Committees and the views expressed herein may not be taken to necessarily represent the views of the Council or any of its Committees.

Permission of the Institute is essential for reproduction of any portion of this material.

© The Institute of Chartered Accountants of India

All rights reserved. No part of this book may be reproduced, stored in a retrieval system, or transmitted, in any form, or by any means, electronic, mechanical, photocopying, recording, or otherwise, without prior permission, in writing, from the publisher.

Website : www.icai.org E-mail : [email protected]

Published by Dr. T.P. Ghosh, Director of Studies, ICAI, C-1, Sector-1, NOIDA-201301

Typeset and designed at Board of Studies, The Institute of Chartered Accountants of India.

Printed at VPS Engineering Impex Pvt. Ltd. Phase – II Noida. August, 2006, 25,000 copies

Page 4: 30510870 Cost Accounting and Financial Management

PREFACE

The recent surge in globalisation and the massive cross border flow of capital has increased the significance of Cost Accounting and Financial Management for management and control purposes. The study of these two important subjects opens new opportunities for Chartered Accountancy students. It provides them with an opportunity to draw upon previous experiences and education to apply various business concepts and analytical tools to complex problems and issues in organizational settings. This study material provides the basic concepts, theories and techniques relating to Cost Accounting and Financial Management and aims to develop the students’ ability in understanding the different concepts and their application in the real life situations. The study material is divided into two parts. Part I relates to Cost Accounting and Part II deals with Financial Management. The Cost Accounting portion has ten chapters having an in depth analysis of concepts relating to Material, Labour, Overheads and other important costing techniques. Standard Costing, Marginal Costing and Budgeting have been included in the syllabus at an introductory level. The syllabus has been designed in such a way that it helps students understand the traditional concepts, their applications, advantages and disadvantages. Contemporary changes in the subject shall be dealt with in the Final stage . The portion on Financial Management is divided into seven chapters. Chapter 1 describes the scope, objectives and importance of financial management and its relationship with other disciplines. Time value of money is discussed in Chapter 2. Chapter 3 explains various tools and techniques of financial management namely Ratio Analysis and Cash Flow Analysis and their application in practical situations. Chapters 4, 6 and 7 deal with the theories, concepts and assumptions underlying financial decisions, namely, Financing, Investment and Working Capital Management. Chapter 5 describes the various Sources of Finance available to business enterprises to cater their different types of requirements. The entire study material has been written in a simple language. A number of self-examination questions are given at the end of each chapter for practice by students. There are also a number of illustrations in each chapter to help students to have a better grasp of the subjects.

Page 5: 30510870 Cost Accounting and Financial Management

SYLLABUS

PAPER – 4 : COST ACCOUNTING AND FINANCIAL MANAGEMENT (One paper ─ Three hours – 100 Marks)

Level of Knowledge: Working knowledge

Part I – Cost Accounting (50 Marks)

Objectives: (a) To understand the basic concepts and processes used to determine product costs, (b) To be able to interpret cost accounting statements, (c) To be able to analyse and evaluate information for cost ascertainment, planning, control

and decision making, and (d) To be able to solve simple cases.

Contents

1. Introduction to Cost Accounting (a) Objectives and scope of Cost Accounting (b) Cost centres and Cost units

(c) Cost classification for stock valuation, Profit measurement, Decision making and control

(d) Coding systems (e) Elements of Cost (f) Cost behaviour pattern, Separating the components of semi-variable costs (g) Installation of a Costing system

(h) Relationship of Cost Accounting, Financial Accounting, Management Accounting and Financial Management

Page 6: 30510870 Cost Accounting and Financial Management

2. Cost Ascertainment (a) Material Cost

(i) Procurement procedures ─ Store procedures and documentation in respect of receipts and issue of stock, Stock verification

(ii) Inventory control ─ Techniques of fixing of minimum, maximum and reorder levels, Economic Order Quantity, ABC classification; Stocktaking and perpetual inventory

(iii) Inventory accounting (iv) Consumption ─ Identification with products of cost centres, Basis for

consumption entries in financial accounts, Monitoring consumption. (b) Employee Cost

(i) Attendance and payroll procedures, Overview of statutory requirements, Overtime, Idle time and Incentives

(ii) Labour turnover (iii) Utilisation of labour, Direct and indirect labour, Charging of labour cost,

Identifying labour hours with work orders or batches or capital jobs (iv) Efficiency rating procedures (v) Remuneration systems and incentive schemes.

(c) Direct Expenses Sub-contracting ─ Control on material movements, Identification with the main product or service.

(d) Overheads (i) Functional analysis ─ Factory, Administration, Selling, Distribution, Research

and Development Behavioural analysis ─ Fixed, Variable, Semi variable and Step cost

(ii) Factory Overheads ─ Primary distribution and secondary distribution, Criteria for choosing suitable basis for allotment, Capacity cost adjustments, Fixed absorption rates for absorbing overheads to products or services

(iii) Administration overheads ─ Method of allocation to cost centres or products (iv) Selling and distribution overheads ─ Analysis and absorption of the expenses

in products/customers, impact of marketing strategies, Cost effectiveness of various methods of sales promotion.

Page 7: 30510870 Cost Accounting and Financial Management

3. Cost Book─ keeping Cost Ledgers ─ Non-integrated accounts, Integrated accounts, Reconciliation of cost and financial accounts.

4. Costing Systems (a) Job Costing Job cost cards and databases, Collecting direct costs of each job, Attributing overhead costs to jobs, Applications of job costing. (b) Batch Costing (c) Contract Costing Progress payments, Retention money, Escalation clause, Contract accounts, Accounting for material, Accounting for plant used in a contract, Contract profit and Balance sheet entries. (d) Process Costing Double entry book keeping, Process loss, Abnormal gains and losses, Equivalent units, Inter-process profit, Joint products and by products. (e) Operating Costing System

5. Introduction to Marginal Costing Marginal costing compared with absorption costing, Contribution, Breakeven analysis and profit volume graph.

6. Introduction to Standard Costing Various types of standards, Setting of standards, Basic concepts of material and Labour standards and variance analysis.

Part II – Financial Management (50 Marks)

Objectives: (a) To develop ability to analyse and interpret various tools of financial analysis and

planning, (b) To gain knowledge of management and financing of working capital, (c) To understand concepts relating to financing and investment decisions, and (d) To be able to solve simple cases.

Page 8: 30510870 Cost Accounting and Financial Management

Contents

1. Scope and Objectives of Financial Management (a) Meaning, Importance and Objectives (b) Conflicts in profit versus value maximisation principle (c) Role of Chief Financial Officer.

2. Time Value of Money Compounding and Discounting techniques─ Concepts of Annuity and Perpetuity.

3. Financial Analysis and Planning (a) Ratio Analysis for performance evaluation and financial health (b) Application of Ratio Analysis in decision making

(c) Analysis of Cash Flow Statement.

4. Financing Decisions (a) Cost of Capital ─ Weighted average cost of capital and Marginal cost of capital (b) Capital Structure decisions ─ Capital structure patterns, Designing optimum capital

structure, Constraints, Various capital structure theories (c) Business Risk and Financial Risk ─ Operating and financial leverage, Trading on

Equity.

5. Types of Financing (a) Different sources of finance (b) Project financing ─ Intermediate and long term financing (c) Negotiating term loans with banks and financial institutions and appraisal thereof (d) Introduction to lease financing (e) Venture capital finance.

6. Investment Decisions (a) Purpose, Objective, Process (b) Understanding different types of projects

Page 9: 30510870 Cost Accounting and Financial Management

(c) Techniques of Decision making: Non-discounted and Discounted Cash flow Approaches ─ Payback Period method, Accounting Rate of Return, Net Present Value, Internal Rate of Return, Modified Internal Rate of Return, Discounted Payback Period and Profitability Index

(d) Ranking of competing projects, Ranking of projects with unequal lives.

7. Management of Working Capital (a) Working capital policies (b) Funds flow analysis (c) Inventory management (d) Receivables management (e) Payables management (f) Management of cash and marketable securities (g) Financing of working capital.

Page 10: 30510870 Cost Accounting and Financial Management

CONTENTS

PART I – COST ACCOUNTING

CHAPTER 1 – BASIC CONCEPTS

1. Evolution of Cost Accounting ......................................................................... 1.1

2. Cost Accounting and Inventory Valuation ....................................................... 1.2

3. Objectives of Cost Accounting ....................................................................... 1.3

4. Importance of Cost Accounting ...................................................................... 1.5

5. Advantages of a Cost Accounting system .... …………………………………………1.7

6. Essential factors for . establishing a Cost Accounting system……………………..1.8

7. Relationship between Cost Accounting, Financial Accounting

Management Accounting and Financial Management .. …………………………. 1.10

8. Cost concepts and terms ...................……………………………………………….1.12

9. Elements of cost..................................................................... ………………..1.17

10. Classification of costs .................................................................................. 1.18

12. Coding systems ........................................................................................... 1.26

13. Types of Costing ......................................................................................... 1.27

14. Methods of Costing...................................................................................... 1.28

15. Direct expenses .......................................................................................... 1.29

16. Self examination questions .......................................................................... 1.31

CHAPTER 2 – MATERIAL

1. Introduction .................................................................................................. 2.1

2. Material control ............................................................................................ 2.2

3. Material procurement procedure ..................................................................... 2.4

4. Material storage .......................................................................................... 2.14

Page 11: 30510870 Cost Accounting and Financial Management

5. Inventory control.......................................................................................... 2.19

6. Valuation of material receipts ....................................................................... 2.44

7. Valuation of material issues ......................................................................... 2.47

8. Valuation of returns and shortages ............................................................... 2.64

9. Treatment of Normal and Abnormal loss of material ...................................... 2.65

10. Accounting and control: waste,scrap,spoilage & defective ............................. 2.65

11. Consumption of material .............................................................................. 2.71

12. Self examination questions .......................................................................... 2.73

CHAPTER 3 – LABOUR

1 Introduction .................................................................................................. 3.1

2 Labour cost control ........................................................................................ 3.1

3 Attendance & payroll procedures ................................................................... 3.3

4 Idle time...................................................................................................... 3.11

5 Overtime ..................................................................................................... 3.13

6 Labour turnover ........................................................................................... 3.18

7. Incentive system ......................................................................................... 3.25

8. Labour utilisation ......................................................................................... 3.28

9. System of wage payment and incentive ...................................................... 3.31

10. Absorption of wages .................................................................................... 3.66

11. Efficiency rating procedures ........................................................................ 3.74

12 Self examination questions .......................................................................... 3.76

CHAPTER 4 – OVERHEADS

1 Introduction .................................................................................................. 4.1

2 Classification of Overheads............................................................................ 4.2

3 Accounting and control of Manufacturing Overheads. ...................................... 4.8

ii

Page 12: 30510870 Cost Accounting and Financial Management

4 Distribution of overheads ............................................................................. 4.10

5 Methods of absorbing Overheads ................................................................. 4.35

6 Treatment of Overheads in Cost Accounting ................................................. 4.46

7. Accounting and control of Administrative Overhead ....................................... 4.59

8 Accounting and control of Selling &Distribution Overhead.............................. 4.64

9. Concepts related to Capacity ....................................................................... 4.68

10 Treatment of certain items in Cost Accounting .............................................. 4.69

11 Self examination questions .......................................................................... 4.73

CHAPTER 5 – NON INTEGRATED ACCOUNTS

1. Introduction ................................................................................................... 5.1

2. Non Integrated Accounting System ................................................................ 5.1

3. Integrated Accounting System ...................................................................... 5.34

4. Reconciliation of Cost and Financial Accounts .............................................. 5.49

5. Self examination questions .......................................................................... 5.61

CHAPTER 6 – METHOD OF COSTING(I)

1. Introduction ................................................................................................... 6.1

2. Job Costing ................................................................................................... 6.2

3. Contract Costing ........................................................................................... 6.9

4. Batch Costing.............................................................................................. 6.31

5. Operating Costing ....................................................................................... 6.33 6. Self examination questions ......................................................................... 6. 41

CHAPTER 7 – METHOD OF COSTING (II)

1 Meaning of Process Costing........................................................................... 7.1

2 Operation Costing ......................................................................................... 7.2

iii

Page 13: 30510870 Cost Accounting and Financial Management

3 Treatment of Process losses .......................................................................... 7.4

7 Costing of Equivalent production units ............................................................ 7.6

8 Inter Process Profits. ................................................................................... 7.13

9 Joint products and by products..................................................................... 7.15

10 Self examination questions .......................................................................... 7.31

CHAPTER 8 – STANDARD COSTING

1. Introduction ................................................................................................... 8.1

2 Definition of standard cost ............................................................................. 8.3

3 Setting up of standard cost ............................................................................ 8.3

4 Types of standards ........................................................................................ 8.8

5 Need for standard cost................................................................................... 8.9

6 Process of standard costing ......................................................................... 8.10

7 Types of variances ...................................................................................... 8.11

8 Accounting procedure for standard cost ........................................................ 8.23

9. Advantages and criticisms ........................................................................... 8.26

10. Self examination questions .......................................................................... 8.29

CHAPTER 9 – MARGINAL COSTING

1 Introduction ................................................................................................... 9.1

2 Theory of Marginal Costing ............................................................................ 9.2

3 Ascertainment of Marginal costing .................................................................. 9.5

4 Distinction between Marginal and Absorption Costing...................................... 9.7

5 Advantages and Limitation of Marginal Costing ............................................. 9.11

6 Cost Volume Profit Analysis ......................................................................... 9.13

7 Self Examination Questions ................................................................................

iv

Page 14: 30510870 Cost Accounting and Financial Management

CHAPTER 10 – BUDGETS AND BUDGETARY CONTROL

1 Introduction ................................................................................................. 10.1

2 Objectives of budgeting ............................................................................... 10.2

3 Budgetary control ........................................................................................ 10.3

4 Different types of budgets ............................................................................ 10.7

5 Preparation of budgets ................................................................................ 10.8

6 Self examination questions .................................................................................

v

Page 15: 30510870 Cost Accounting and Financial Management

CONTENTS

PART II – FINANCIAL MANAGEMENT

CHAPTER 1 – SCOPE AND OBJECTIVES OF FINANCIAL MANAGEMENT

1. Introduction .................................................................................................. 1.1

2. Meaning of Financial Management ................................................................ 1.2

3. Evolution of Financial Management ............................................................... 1.4

4. Importance of Financial Management ............................................................ 1.5

5. Scope of Financial Management ……………………………………………………….1.5

6. Objectives of Financial Management…………………………………………………..1.6

7. Conflicts in Profit versus Value Maximisation Principle …………………………. 1.10

8. Role of Chief Financial Officer (CFO) ……………………………………………….1.12

9. Relationship of Financial Management with Related Disciplines………………..1.13

CHAPTER 2 – TIME VALUE OF MONEY

1. Concept of Time Value of Money ................................................................... 2.1

2. Simple Interest ............................................................................................. 2.2

3. Compound Interest ....................................................................................... 2.3

4. Effective Rate of Interest .............................................................................. 2.8

5. Present Value ............................................................................................... 2.8

6. Annuity ....................................................................................................... 2.10

7. Perpetuity .................................................................................................. 2.12

8. Sinking Fund ............................................................................................... 2.14

9. Techniques of Discounting ........................................................................... 2.14

10. Techniques of Compounding ........................................................................ 2.18

Page 16: 30510870 Cost Accounting and Financial Management

CHAPTER 3 – FINANCIAL ANALYSIS AND PLANNING

UNIT I : APPLICATION OF RATIO ANALYSIS FOR PERFORMANCE EVALUATION, FINANCIAL HEALTH AND DECISION MAKING

1.1 Introduction .................................................................................................. 3.1

1.2 Ratio Analysis .............................................................................................. 3.1

1.3 Types of Ratios ............................................................................................ 3.2

1.4 Application of Ratio Analysis in Financial Decision Making ............................ 3.17

1.5 Limitations of Financial Ratios ...................................................................... 3.18

1.6 Summary of Ratios .................................................................................... 3.19

UNIT II : CASH FLOW ANALYSIS

2.1 Introduction ................................................................................................ 3.42

2.2 Utility of Cash Flow Analysis ...................................................................... 3.42

2.3 Limitations of Cash Flow Analysis .............................................................. 3.43

2.4 Benefits of Cash Flow Information ............................................................... 3.44

2.5 Definitions .................................................................................................. 3.44

2.6 Cash and Cash Equivalents ........................................................................ 3.44

2.7 Presentation of Cash Flow Statement ……………………………………………….3.45

2.8 Procedure in Preparation of Cash Flow Statement ........................................... 3.49

2.9 Funds Flow Statement vs. Cash Flow Statement ……………………………………..3.54

CHAPTER 4 – FINANCING DECISIONS

UNIT I : COST OF CAPITAL

1.1 Introduction .................................................................................................. 4.1

1.2 Definition of Cost of Capital .......................................................................... 4.3

1.3 Measurement of Cost of Capital . ................................................................... 4.4

1.4 Weighted Average Cost of Capital (WACC) ................................................... 4.17

1.5 Marginal Cost of Capital .............................................................................. 4.21

vii

Page 17: 30510870 Cost Accounting and Financial Management

UNIT II : CAPITAL STRUCTURE DECISIONS

2.1 Meaning of Capital Structure ....................................................................... 4.26

2.2 Choice of Capital Structure ......................................................................... 4.26

2.3 Significance of Capital Structure ................................................................. 4.28

2.4 Optimal Capital Structure ............................................................................ 4.30

2.5 EBIT-EPS Analysis ..................................................................................... 4.30

2.6 Cost of Capital, Capital Structure and Market Price of Share ........................ 4.33

2.7 Capital Structure Theories .......................................................................... 4.34

2.8 Capital Structure and Taxation .................................................................... 4.46

UNIT III : BUSINESS RISK AND FINANCIAL RISK

3.1 Introduction ................................................................................................. 4.47

3.2 Debt versus Equity Financing ...................................................................... 4.48

3.3 Types of Leverage ...................................................................................... 4.50

CHAPTER 5 – TYPES OF FINANCING

1. Introduction ................................................................................................... 5.1

2. Financial Needs and Sources of Finance of a Business .................................. 5.1

3. Long Term Sources of Finance ........................................................................ 5.4

4. Venture Capital Financing ........................................................................... 5.12

5. Debt Securitisation ..................................................................................... 5.14

6. Lease Financing ........................................................................................ 5.16

7. Short Term Sources of Finance ................................................................... 5.16

8. Other Sources of Financing ......................................................................... 5.25

9. New Instruments ....................................................................................... 5.27

10. International Financing ................................................................................ 5.29

viii

Page 18: 30510870 Cost Accounting and Financial Management

CHAPTER 6 – INVESTMENT DECISIONS

1. Introduction ................................................................................................... 6.1

2. Purpose of Capital Budgeting………………………………………………………………… 6.2

3. Capital Budgeting Process ............................................................................ 6.2

4. Types of capital Investment Decisions ............................................................ 6.3

5. Project Cash flows......................................................................................... 6.4 6. Basic Principles for Measuring Project Cash Flows ....................................... 6. 5 7. Capital Budgeting Techniques ...................................................................... 6. 9 8. Capital Rationing ......................................................................................... 6.18

CHAPTER 7 – MANAGEMENT OF WORKING CAPITAL

UNIT I : MEANING, CONCEPT AND POLICIES OF WORKING CAPITAL

1.1 Introduction .................................................................................................. 7.1

1.2 Meaning and Concept of Working Capital ...................................................... 7.1

1.3 Management of Working Capital ................................................................... 7.5

1.7 Issues in the Working Capital Management ................................................... 7.6

1.8 Estimating Working Capital Needs ................................................................. 7.9

1.9 Operating or Working Capital Cycle ............................................................... 7.9

UNIT II : TREASURY AND CASH MANAGEMENT

2.1 Treasury Management: Meaning ................................................................. 7.30

2.2 Functions of Treasury Department ............................................................... 7.30

2.3 Management of Cash .................................................................................. 7.31

2.4 Methods of Cash Flow Budgeting ................................................................ 7.34

2.5 Cash Management Models …………………………………………………………….7.48

2.6 Recent Developments in Cash Management ……………………………………….7.50

2.7 Cash Management Services – The ICICI Bank Way ………………………………7.54

2.8 Management of Marketable Securities……………………………………………….7.55

ix

Page 19: 30510870 Cost Accounting and Financial Management

UNIT III : MANAGEMENT OF INVENTORY

3.1 Inventory Management ................................................................................ 7.56

UNIT IV : MANAGEMENT OF RECEIVABLES

4.1 Introduction ................................................................................................ 7.57

4.2 Role to be Played by the Finance Manager …………………………………………7.57

4.3 Aspects of Management of Debtors ............................................................ 7.57

4.4 Factors Determining Credit Policy ............................................................... 7.58

4.5 Factors under the Control of the Finance Manager ....................................... 7.59

4.6 Financing Receivables ................................................................................ 7.67

4.7 Innovations in Receivable Management ...................................................... 7.69

4.8 Monitoring of Receivables .......................................................................... 7.74

UNIT V : FINANCING OF WORKING CAPITAL

5.1 Introduction ................................................................................................ 7.76

5.2 Sources of Finance………………………………………………………………………7.77

5.3 Working Capital Finance from Banks ........................................................... 7.81

5.4 Factors Determining Credit Policy ............................................................... 7.82

x

Page 20: 30510870 Cost Accounting and Financial Management

PART I

COST ACCOUNTING

Page 21: 30510870 Cost Accounting and Financial Management

CHAPTER 1

BASIC CONCEPTS

Learning Objectives When you have finished studying this chapter, you should be able to ♦ Understand the objective and importance of Cost Accounting. ♦ Understand the cost accounting terminology. ♦ Differentiate between cost accounting and financial accounting ♦ Understand the relationship between Cost Accounting, Financial Accounting,

Management Accounting and Financial Management. ♦ Understand the concept of codes and the process of codification. ♦ Understand the various types and methods of cost accounting.

1.1 EVOLUTION OF COST ACCOUNTING Prior to the industrial revolution, businesses were small and characterised by simple market exchanges between individuals and organisations. In those times there was a need of accurate book keeping though not that much of cost accounting. However, by the seventeenth century in France, the Royal Wallpaper Manufactory had a Cost Accounting System. Some iron masters and potters in eighteenth century in England too began to produce Cost Accounting information before the Industrial Revolution. Subsequently, with the advent of the industrial revolution, large sized process industries performing single activities (e.g. textiles , railways etc)came into being. During this period, there was a lack of market for intermediary products because of which cost information gained importance as a tool for measuring efficiency of different processes. The period, 1880 AD – 1925 AD saw the development of complex product designs and the emergence of multi activity diversified corporations like Du Pont, General Motors etc. It was during this period that scientific management was developed which led accountants to convert physical standards into cost standards, the latter being used for variance analysis and control. During World War I and II the social importance of cost accounting grew with the growth of each country’s defence expenditure. In the absence of competitive markets for most of the material required to fight war, the Governments in several countries placed cost-plus contracts

Page 22: 30510870 Cost Accounting and Financial Management

Cost Accounting

under which the price to be paid was the cost of production plus an agreed rate of profit. The reliance on cost information by the parties to defence contracts continued after World War II as well. Even today, most of the government contracts are decided on a cost plus basis.

1.2 COST ACCOUNTING AND INVENTORY VALUATION The spurt in the industrial growth, as mentioned above, also resulted in the increased importance of financial accounting and audit (1900 AD onwards).One of the fundamental issues to be resolved by the accountants during this period was the measurement of the value of inventory while preparing financial statements. The valuation had a deep impact over the projected profitability of a company, which in turn affected the willingness of various stakeholders to inject large amount of capital in the business. The valuation also directly affected the taxes which the company was obliged to pay to the government since higher profits meant higher taxes and vice versa. It was in this context that the need of establishing rules for inventory valuation was felt. It was then decided that inventory should be valued at ‘cost’ or ‘market value’ which ever is lower. The term ‘cost’, being restricted to the money expended in manufacturing the product till the time the product was sold. Hence, expenditure incurred in research and development, distribution, marketing or customer support functions was to be excluded while computing ‘costs’ for inventory valuation. The computation of the cost incidence on different types of inventory with different degrees of completion necessitated the need of accounting for ‘costs’ in order to arrive at the correct values. As you would have understood by now, ‘cost accounting’ was initiated for manufacturing organizations and as a field of practice was limited within the factory premises. However, with the increase in its scope , cost accounting today is equally important to both manufacturing and service organizations and also does not restrict itself to inventory valuation alone. It is used in (1)various decision making scenarios e.g. whether to produce for captive consumption or buy from outside suppliers,(2) supply of information to the government (cost audit), (3)planning and control of expenses(variance analysis) , (4)tracking expenses through a products life cycle (life cycle costing),(5) fixation of selling prices (cost plus and other approaches) etc. The use of information technology has helped companies keep /maintain different cost systems for different purposes. Today, Cost Accounting is popularly known as ‘Cost and Management Accounting’. Before you begin your study of Cost Accounting, you must be clear in your mind that you are going to study a subject, which is immensely useful in all economic activities. It is a natural instinct with all of us to measure the pros and cons of everything. A prudent housewife who goes for shopping considers the quality and price of each product before she buys it. In short, each economic activity, if rationally viewed, has two aspects - firstly, the costs involved in it

1.2

Page 23: 30510870 Cost Accounting and Financial Management

Basic Concept

and secondly, the benefits obtained out of it. This analysis is technically known as cost-benefit analysis. It is very important in industrial and commercial activities. Cost Accounting involves a study of those concepts, tools, and techniques, which help us in ascertaining and analysing costs. 1.2.1 Definition of Costing, Cost Accounting and Cost Accountancy: Costing is defined as “the technique and process of ascertaining costs”. Cost Accounting is defined as "the process of accounting for cost which begins with the recording of income and expenditure or the bases on which they are calculated and ends with the preparation of periodical statements and reports for ascertaining and controlling costs." Cost Accountancy has been defined as “the application of costing and cost accounting principles, methods and techniques to the science, art and practice of cost control and the ascertainment of profitability. It includes the presentation of information derived there from for the purpose of managerial decision making.”

1.3 OBJECTIVES OF COST ACCOUNTING The main objectives of Cost Accounting are as follows : (i) Ascertainment of cost. (ii) Determination of selling price. (iii) Cost control and cost reduction. (iv) Ascertaining the profit of each activity. (v) Assisting management in decision-making. 1.3.1 Ascertainment of Cost: There are two methods of ascertaining costs, viz., Post Costing and Continuous Costing. Post Costing means, analysis of actual information as recorded in financial books. It is accurate and is useful in the case of “Cost plus Contracts” where price is to be determined finally on the basis of actual cost. Continuous Costing, aims at collecting information about cost as and when the activity takes place so that as soon as a job is completed the cost of completion would be known. This involves careful estimates being prepared of overheads. In order to be of any use, costing must be a continuous process. Cost ascertained by the above two methods may be compared with the standard costs which are the target figures already compiled on the basis of experience and experiments.

1.3

Page 24: 30510870 Cost Accounting and Financial Management

Cost Accounting

1.3.2 Determination of selling price: Though the selling price of a product is influenced by market conditions, which are beyond the control of any business, it is still possible to determine the selling price within the market constraints. For this purpose, it is necessary to rely upon cost data supplied by Cost Accountants. 1.3.3 Cost control and cost reduction: as “The guidance and regulation, by executive action of the cost of operating an undertaking”. The word “guidance” indicates a goal or target to be guided; ‘regulation’ indicates taking action where there is a deviation from what is laid down; executive action denotes action to “regulate” must be initiated by executives i.e. persons responsible for carrying out the job or the operation; and all this is to be exercised through modern methods of costing in respect of expenses incurred in operating an undertaking. To exercise cost control, broadly speaking the following steps should be observed: (i) Determine clearly the objective, i.e., pre-determine the desired results; (ii) Measure the actual performance; (iii) Investigate into the causes of failure to perform according to plan; and (iv) Institute corrective action. The target cost and/or targets of performance should be laid down in respect of each department or operation and these targets should be related to individuals who, by their action, control the actual and bring them into line with the targets. Actual cost of performance should be measured in the same manner in which the targets are set up, i.e. if the targets are set up operation-wise, then the actual costs should also be collected operation-wise and not cost centre or department-wise as this would make comparison difficult. Cost Reduction, may be defined "as the achievement of real and permanent reduction in the unit cost of goods manufactured or services rendered without impairing their suitability for the use intended or diminution in the quality of the product." Cost reduction should not be confused with Cost control. Cost saving could be a temporary affair and may be at the cost of quality. Cost reduction implies the retention of the essential characteristics and quality of the product and thus it must be confined to permanent and genuine savings in the cost of manufacture, administration, distribution and selling, brought about by elimination of wasteful and inessential elements from the design of the product and from the techniques carried out in connection therewith. In other words, the essential characteristics and quality of the products are retained through improved methods and techniques and thereby a permanent reduction in unit cost is achieved. The definition of cost reduction does not, however, include reduction in expenditure arising from reduction in taxa-tion or similar Government action or the effect of price agreements.

1.4

Page 25: 30510870 Cost Accounting and Financial Management

Basic Concept

The three-fold assumptions involved in the definition of cost reduction may be summarised as under : (a) There is a saving in unit cost. (b) Such saving is of permanent nature. (c) The utility and quality of the goods and services remain unaffected, if not improved. 1.3.4 Ascertaining the profit of each activity : The profit of any activity can be ascertained by matching cost with the revenue of that activity. The purpose under this step is to determine costing profit or loss of any activity on an objective basis. 1.3.5 Assisting management in decision making : Decision making is defined as a process of selecting a course of action out of two or more alternative courses. For making a choice between different courses of action, it is necessary to make a comparison of the outcomes, which may be arrived under different alternatives. Such a comparison has only been made possible with the help of Cost Accounting information.

1.4 IMPORTANCE OF COST ACCOUNTING TO BUSINESS CONCERNS Management of business concerns expects from Cost Accounting a detailed cost information in respect of its operations to equip their executives with relevant information required for planning, scheduling, controlling and decision making. To be more specific, management expects from cost accounting - information and reports to help them in the discharge of the following functions : (a) Control of material cost : Cost of material usually constitute a substantial portion of the total cost of a product. Therefore, it is necessary to control it as far as possible. Such a control may be exercised by (i) Ensuring un-interrupted supply of material and spares for production. (ii) By avoiding excessive locking up of funds/capital in stocks of materials and stores. (iii) Also by the use of techniques like value analysis, standardisation etc. to control material cost. (b) Control of labour cost : It can be controlled if workers complete their work within the standard time limit. Reduction of labour turnover and idle time too help us, to control labour cost. (c) Control of overheads : Overheads consists of indirect expenses which are incurred in the factory, office and sales department ; they are part of production and sales cost. Such expenses may be controlled by keeping a strict check over them. (d) Measuring efficiency : For measuring efficiency, Cost Accounting department should provide information about standards and actual performance of the concerned activity. (e) Budgeting : Now–a–days detailed estimates in terms of quantities and amounts are drawn up before the start of each activity. This is done to ensure that a practicable course of

1.5

Page 26: 30510870 Cost Accounting and Financial Management

Cost Accounting

action can be chalked out and the actual performance corresponds with the estimated or budgeted performance. The preparation of the budget is the function of Costing Department. (f) Price determination: Cost accounts should provide information, which enables the management to fix remunerative selling prices for various items of products and services in different circumstances. (g) Curtailment of loss during the off-season: Cost Accounting can also provide information, which may enable reduction of overhead, by utilising idle capacity during the off-season or by lengthening the season. (h) Expansion: Cost Accounts may provide estimates of production of various levels on the basis of which the management may be able to formulate its approach to expansion. (i) Arriving at decisions: Most of the decisions in a business undertaking involve correct statements of the likely effect on profits. Cost Accounts are of vital help in this respect. In fact, without proper cost accounting, decision would be like taking a jump in the dark, such as when production of a product is stopped.

1.5 VARIOUS REPORTS PROVIDED BY COST ACCOUNTING DEPARTMENT Following reports may be provided by a Cost Accounting Department for the use of its executives: (a) Cost sheets setting out the total cost, analysed into various elements, giving comparative figures for the previous period and for other plants under the same management. (b) Consumption of material statements, showing total quantity of materials issued for production, materials actually embodied in production and wastage. (c) Labour utilisation statements providing details about the total number of hours paid for, standard hours for the output, idle time (and amount involved) and causes thereof. (d) Overheads incurred compared with budgets; overheads actually charged to production and the difference between the amount actually incurred and the amount so charged. (e) Sales effected compared with budgets, showing the difference between the two because of quality being different from those taken into account while budgeting. (f) Reconciliation of actual profit earned with estimated or budgeted profit. (g) The total cost of abnormally spoiled work in the factory and abnormal losses in the store. (h) The total cost of inventory carried, analysed into raw materials in chief stores and other stores. The number of months for which stocks would be sufficient (on the basis of average consumption being worked out).

1.6

Page 27: 30510870 Cost Accounting and Financial Management

Basic Concept

(i) Labour turnover, and the cost of recruitment and training of new employees. (j) Expenses incurred on Research and Development as compared with the budgeted amount. Reports about particular departments and operations (like transport or power generation) may also be compiled and submitted to the departmental manager concerned.

1.6 ADVANTAGES OF A COST ACCOUNTING SYSTEM Important advantages of a Cost Accounting System may be listed as below : 1. A good Cost Accounting System helps in identifying unprofitable activities, losses or

inefficiencies in any form. 2. The application of cost reduction techniques, operations research techniques and value

analysis technique, helps in achieving the objective of economy in concern’s operations. Continuous efforts are being made by the business organisation for finding new and improved methods for reducing costs.

3. Cost Accounting is useful for identifying the exact causes for decrease or increase in the profit/loss of the business. It also helps in identifying unprofitable products or product lines so that these may be eliminated or alternative measures may be taken.

4. It provides information and data to the management to serve as guides in making decisions involving financial considerations. Guidance may also be given by the Cost Accountant on a host of problems such as, whether to purchase or manufacture a given component, whether to accept orders below cost, which machine to purchase when a number of choices are available.

5. Cost Accounting is quite useful for price fixation. It serves as a guide to test the adequacy of selling prices. The price determined may be useful for preparing estimates or filling tenders.

6. The use of cost accounting technique viz., variance analysis, points out the deviations from the pre-determined level and thus demands suitable action to eliminate such deviations in future.

7. Cost comparison helps in cost control. Such a comparison may be made from period to period by using the figures in respect of the same unit of firms or of several units in an industry by employing uniform costing and inter-firm comparison methods. Comparison may be made in respect of costs of jobs, processes or cost centres.

8. A system of costing provides figures for the use of Government, Wage Tribunals and other bodies for dealing with a variety of problems. Some such problems include price fixation, price control, tariff protection, wage level fixation, etc.

1.7

Page 28: 30510870 Cost Accounting and Financial Management

Cost Accounting

9. The cost of idle capacity can be easily worked out, when a concern is not working to full

capacity. 10. The use of Marginal Costing technique, may help the executives in taking short term

decisions. This technique of costing is highly useful during the period of trade depression, as the orders may have to be accepted during this period at a price less than the total cost.

11. The marginal cost has linear relationship with production volume and hence in formulating and solving “Linear Programming Problems”, marginal cost is useful.

1.7 ESSENTIAL FACTORS FOR INSTALLING A COST ACCOUNTING SYSTEM As in the case of every other form of activity, it should be considered whether it would be profitable to have a cost accounting system. The benefits from such a system must exceed the amount to be spent on it. This would depend upon many factors including the nature of the business and the quality of the management. Management, which is prone to making decisions on the basis of pre-conceived notions without taking into account the information and data placed before it, cannot derive much benefit from a costing system. On the other hand management, which is in the habit of studying information thoroughly before making decisions, would require cost accounting system. Before setting up a system of cost accounting the under mentioned factors should be studied: (i) The objective of costing system, for example whether it is being introduced for fixing

prices or for insisting a system of cost control. (ii) The areas of operation of business wherein the managements’ action will be most

beneficial. For instance, in a concern, which is anxious to expand its operations, increase in production would require maximum attention. On the other hand for a concern, which is not able, to sell the whole of its production the selling effort would require greater attention. The system of costing in each case should be designed to highlight, in signifi-cant areas, factors considered important for improving the efficiency of operations in that area.

(iii) The general organisation of the business, with a view of finding out the manner in which the system of cost control could be introduced without altering or extending the organ-isation appreciably.

(iv) The technical aspects of the concern and the attitude and behaviour that will be successful in winning sympathetic assistance or support of the supervisory staff and workmen.

(v) The manner in which different variable expenses would be affected with expansion or cessation of different operations.

1.8

Page 29: 30510870 Cost Accounting and Financial Management

Basic Concept

(vi) The manner in which Cost and Financial accounts could be inter-locked into a single integral accounting system and in which results of separate sets of accounts, cost and financial, could be reconciled by means of control accounts.

(vii) The maximum amount of information that would be sufficient and how the same should be secured without too much clerical labour, especially the possibility of collection of data on a separate printed form designed for each process; also the possibility of instruction as regards filling up of the forms in writing to ensure that these would be faithfully carried out.

(viii) How the accuracy of the data collected can be verified? Who should be made responsible for making such verification in regard to each operation and the form of certificate that he should give to indicate the verification that he has carried out ?

(ix) The manner in which the benefits of introducing Cost Accounting could be explained to various persons in the concern, specially those in charge of production department and awareness created for the necessity of promptitude, frequency and regularity in collection of costing data.

1.7.1 Essentials of a good Cost Accounting System: The essential features, which a good Cost Accounting System should possess, are as follows: (i) Cost Accounting System should be tailor-made, practical, simple and capable of meeting

the requirements of a business concern. (ii) The data to be used by the Cost Accounting System should be accurate; otherwise it may

distort the output of the system. (iii) Necessary cooperation and participation of executives from various departments of the

concern is essential for developing a good system of Cost Accounting. (iv) The Cost of installing and operating the system should justify the results. (v) The system of costing should not sacrifice the utility by introducing meticulous and

unnecessary details. (vi) A carefully phased programme should be prepared by using network analysis for the

introduction of the system. (vii) Management should have a faith in the Costing System and should also provide a

helping hand for its development and success.

1.9

Page 30: 30510870 Cost Accounting and Financial Management

Cost Accounting

1.8 RELATIONSHIP BETWEEN COST ACCOUNTING, FINANCIAL ACCOUNTING, MANAGEMENT ACCOUNTING AND FINANCIAL MANAGEMENT Cost Accounting is a branch of accounting, which has been developed because of the limitations of Financial Accounting from the point of view of management control and internal reporting. Financial accounting performs admirably, the function of portraying a true and fair overall picture of the results or activities carried on by an enterprise during a period and its financial position at the end of the year. Also, on the basis of financial accounting, effective control can be exercised on the property and assets of the enterprise to ensure that they are not misused or misappropriated. To that extent financial accounting helps to assess the overall progress of a concern, its strength and weaknesses by providing the figures relating to several previous years. Data provided by Cost and Financial Accounting is further used for the management of all processes associated with the efficient acquisition and deployment of short, medium and long term financial resources. Such a process of management is known as Financial Management. The objective of Financial Management is to maximise the wealth of shareholders by taking effective Investment, Financing and Dividend decisions. Investment decisions relate to the effective deployment of scarce resources in terms of funds while the Financing decisions are concerned with acquiring optimum finance for attaining financial objectives. The last and very important ‘Dividend decision’ relates to the determination of the amount and frequency of cash which can be paid out of profits to shareholders. On the other hand, Management Accounting refers to managerial processes and technologies that are focused on adding value to organisations by attaining the effective use of resources, in dynamic and competitive contexts. Hence, Management Accounting is a distinctive form of resource management which facilitates management’s ‘decision making’ by producing information for managers within an organisation. 1.8.1 Limitations of Financial Accounting: There are, however, serious limitations of financial accounting from the point of view of management. These stem from the fact that management must have information properly analysed and continuously flowing to it. Management cannot be satisfied with a broad picture, and that too available at the end of a period. The limitations of financial accounts together with procedures that overcome the limitations are given below :

1.10

Page 31: 30510870 Cost Accounting and Financial Management

Basic Concept

Limitations Procedures that overcome limitations A. Forecasting and Planning Management requires information so that it can make effective plans for the coming year and the period after that. In other words, information about the future is required. Financial Accounts, provide this information.

The technique of budgeting has been evolved. Starting with the assessment of the limiting factors that may be operating, careful estimates can be prepared of all the activities that will be undertaken and these then can cannot be translated in terms of money. (An analysis of cost into fixed and variable is most useful in this respect). These “estimates”, properly coordinated, become budgets and plans of action.

B. Decision making Management requires information daily for making decision of any type. Financial Accounts normally are not able to provide information for this purpose. Information is required to answer the under mentioned questions amongst others :

It is the complete analysis of cost incurred that can help management in making the type of decisions mentioned. The analysis has to be two-fold :

(i) What should be the product under normal circumstances and under special circumstances ?

(i) The total cost of each job, product, process etc. as to be ascertained; and

(ii) Should a part be produced in the factory itself or bought from the market ?

(ii) The cost must be further analysed as fixed and variable.

(iii) Should the production of a product be given up ?

(iii) In other words, management should have information to see what the effect on the revenues and cost (both) will be if a proposed course of action is taken. This is the marginal costing technique and most problems can be handled with the aid of this technique.

(iv) What should be the priority accorded to a product ?

1.11

Page 32: 30510870 Cost Accounting and Financial Management

Cost Accounting

(v) Should investment be made in a new

project ?

(vi) How much should be produced to earn a certain profit, given the selling price?

C. Control and assessment Management requires information to assess the performance of various persons and departments and to see that costs do not exceed a reasonable limit for a given quantum of work of the requisite quality. Financial accounts cannot provide information for this purpose. Apart from the generality of the above, management requires to know :

The techniques of budgeting and standard costing enable management to perform this function which is one of most important one. In this case also the main activity is analysis and comparison. If standard costing is not adopted, simple comparison of figures over the periods for each element of cost in terms of quantity if possible is of great help.

(i) the profitability of each product ; and (ii) the extent of unnecessary material,

labour, facilities etc.

Profitability of each waste product can be easily established by comparing its selling price with the contribution it makes i.e., the difference between the selling price and its variable cost.

It will be seen from the above that the chief limitation of financial accounting is lack of analysis of information and absence of measuring rods. Cost accounting with the aid of budgeting, standard costing and marginal costing has filled the need in this respect.

1.9 COST CONCEPTS AND TERMS

1.9.1 Cost (a) The amount of expenditure (actual or notional) incurred on or attributable to a specified

article, product or activity. (here the word cost is used as a noun) (b) To ascertain the cost of a given thing. (here the word cost is used as a verb) NOTE : The word ‘Cost’ can rarely stand on its own and should be qualified as to its limitation (e.g., historical, variable etc.) and related to a particular thing or ‘object of thought’ e.g., a given quantity or unit of goods made or services performed. 1.9.2 Cost object – Anything for which a separate measurement of cost is desired. Examples of cost objects include a product, a service , a project , a customer , a brand category , an activity , a department , a programme. 1.9.3 Direct costs – Costs that are related to the cost object and can be traced in an economically feasible way.

1.12

Page 33: 30510870 Cost Accounting and Financial Management

Basic Concept

1.9.4 Indirect costs – Costs that are related to the cost object but cannot be traced to it in an economically feasible way. 1.9.5 Pre-determined - A cost which is computed in advance before production or operations start, on the basis of specification of all the factors affecting cost, is known as a pre-determined cost. 1.9.6 Standard Cost - A pre-determined cost, which is calculated from managements ‘expected standard of efficient operation’ and the relevant necessary expenditure. It may be used as a basis for price fixing and for cost control through variance analysis. 1.9.7 Marginal Cost - The amount at any given volume of output by which aggregate costs are changed if the volume of output is increased or decreased by one unit. Note : In this context a unit may be a single article, an order, a stage of production, a process of a department. It relates to change in output in the particular circumstances under consideration within the capacity of the concerned organisation. 1.9.8 Cost of Sales - The cost which is attributable to the sales made. Note: It is not uncommon to use this in a restricted sense as the production cost of goods sold. 1.9.9 Total Cost - The sum of all costs attributable to the cost object under consideration. 1.9.10 Cost Centre - It is defined as a location, person or an item of equipment (or group of these) for which cost may be ascertained and used for the purpose of Cost Control. Cost Centres are of two types, viz., Personal and Impersonal. A Personal cost centre consists of a person or group of persons and an Impersonal cost centre consists of a location or an item of equipment (or group of these). In a manufacturing concern there are two main types of Cost Centres as indicated below : (i) Production Cost Centre : It is a cost centre where raw material is handled for conversion

into finished product. Here both direct and indirect expenses are incurred. Machine shops, welding shops and assembly shops are examples of production Cost Centres.

(ii) Service Cost Centre : It is a cost centre which serves as an ancillary unit to a production cost centre. Power house, gas production shop, material service centres, plant maintenance centres are examples of service cost centres.

1.9.11 Cost unit - It is a unit of product, service or time (or combination of these) in relation to which costs may be ascertained or expressed. We may for instance determine the cost per tonne of steel, per tonne kilometre of a transport service or cost per machine hour. Sometime, a single order or a contract constitutes a cost unit. A batch which consists of a group of

1.13

Page 34: 30510870 Cost Accounting and Financial Management

Cost Accounting

identical items and maintains its identity through one or more stages of production may also be considered as a cost unit. Cost units are usually the units of physical measurement like number, weight, area, volume, length, time and value. A few typical examples of cost units are given below :

Industry or Product Cost Unit Basis

Automobile −Number

Cement −Tonne/per bag etc.

Chemicals −Litre, gallon, kilogram, tonne etc.

Power −Kilo-watt hour

Steel −Tonne

Transport −Passenger kilometre 1.9.12 Responsibility Centre - It is defined as an activity centre of a business organisation entrusted with a special task. Under modern budgeting and control, financial executives tend to develop responsibility centres for the purpose of control. Responsibility centres can broadly be classified into three categories. They are : (a) Cost Centres ; (b) Profit Centres ; and (c) Investment Centres ; 1.9.13 Profit Centres - Centres which have the responsibility of generating and maximising profits are called Profit Centres. 1.9.14 Investment Centres - Those centres which are concerned with earning an adequate return on investment are called Investment Centres. 1.9.15 Cost allocation - It is defined as the assignment of the indirect costs to the chosen cost object. 1.9.16 Cost absorption - It is defined as the process of absorbing all indirect costs allocated to or apportioned over a particular cost centre or production department by the units produced. Hence, while allocating, the relevant cost objects would be the concerned cost centre or the concerned department, while, the process of absorption would consider the units produced as the relevant cost object. For example, the overhead costs of a lathe centre may be absorbed by using a rate per lathe hour. Cost absorption can take place only after cost allocation. In other words, the overhead costs are either allocated or apportioned over different cost centres and afterwards they are absorbed on equitable basis by the output of the same cost centres.

1.14

Page 35: 30510870 Cost Accounting and Financial Management

Basic Concept

1.9.17 Estimated cost - Kohler defines estimated cost as “the expected cost of manufacture, or acquisition, often in terms of a unit of product computed on the basis of information available in advance of actual production or purchase”. Estimated cost are prospective costs since they refer to prediction of costs. 1.9.18 Differential cost - (Incremental and decremental costs). It represents the change (increase or decrease) in total cost (variable as well as fixed) due to change in activity level, technology, process or method of production, etc. For example if any change is proposed in the existing level or in the existing method of production, the increase or decrease in total cost or in specific elements of cost as a result of this decision will be known as incremental cost or decremental cost. 1.9.19 Imputed costs - These costs are notional costs which do not involve any cash outlay. Interest on capital, the payment for which is not actually made, is an example of imputed cost. These costs are similar to opportunity costs. 1.9.20 Capitalised costs – These are costs which are initially recorded as assets and subsequently treated as expenses. 1.9.21 Product costs - These are the costs which are associated with the purchase and sale of goods (in the case of merchandise inventory). In the production scenario, such costs are associated with the acquisition and conversion of materials and all other manufacturing inputs into finished product for sale. Hence, under marginal costing, variable manufacturing costs and under absorption costing, total manufacturing costs (variable and fixed) constitute inventoriable or product costs. Under the Indian GAAP, product costs will be those costs which are allowed to be a part of the value of inventory as per Accounting Standard 2, issued by the Council of the Institute of Chartered Accountants of India. 1.9.22 Opportunity cost - This cost refers to the value of sacrifice made or benefit of opportunity foregone in accepting an alternative course of action. For example, a firm financing its expansion plan by withdrawing money from its bank deposits. In such a case the loss of interest on the bank deposit is the opportunity cost for carrying out the expansion plan. 1.9.23 Out-of-pocket cost - It is that portion of total cost, which involves cash outflow. This cost concept is a short-run concept and is used in decisions relating to fixation of selling price in recession, make or buy, etc. Out–of–pocket costs can be avoided or saved if a particular proposal under consideration is not accepted. 1.9.24 Shut down costs - Those costs, which continue to be, incurred even when a plant is temporarily shutdown, e.g. rent, rates, depreciation, etc. These costs cannot be eliminated with the closure of the plant. In other words, all fixed costs, which cannot be avoided during the temporary closure of a plant, will be known as shut down costs.

1.15

Page 36: 30510870 Cost Accounting and Financial Management

Cost Accounting

1.9.25 Sunk costs - Historical costs incurred in the past are known as sunk costs. They play no role in decision making in the current period. For example, in the case of a decision relating to the replacement of a machine, the written down value of the existing machine is a sunk cost and therefore, not considered. 1.9.26 Absolute cost - These costs refer to the cost of any product, process or unit in its totality. When costs are presented in a statement form, various cost components may be shown in absolute amount or as a percentage of total cost or as per unit cost or all together. Here the costs depicted in absolute amount may be called absolute costs and are base costs on which further analysis and decisions are based. 1.9.27 Discretionary costs – Such costs are not tied to a clear cause and effect relationship between inputs and outputs. They usually arise from periodic decisions regarding the maximum outlay to be incurred. Examples include advertising, public relations, executive training etc. 1.9.28 Period costs - These are the costs, which are not assigned to the products but are charged as expenses against the revenue of the period in which they are incurred. All non-manufacturing costs such as general and administrative expenses, selling and distribution expenses are recognised as period costs. 1.9.29 Engineered costs - These are costs that result specifically from a clear cause and effect relationship between inputs and outputs. The relationship is usually personally observable. Examples of inputs are direct material costs, direct labour costs etc. Examples of output are cars, computers etc. 1.9.30 Explicit Costs - These costs are also known as out of pocket costs and refer to costs involving immediate payment of cash. Salaries, wages, postage and telegram, printing and stationery, interest on loan etc. are some examples of explicit costs involving immediate cash payment. 1.9.31 Implicit Costs - These costs do not involve any immediate cash payment. They are not recorded in the books of account. They are also know as economic costs.

1.16

Page 37: 30510870 Cost Accounting and Financial Management

Basic Concept

1.10 ELEMENTS OF COST A diagram as given below shows the elements of cost described as under :

ELEMENTS OF COST MATERIALS LABOUR OTHER COST COST EXPENSES

DIRECT INDIRECT DIRECT INDIRECT DIRECT INDIRECT MATERIALS MATERIALS LABOUR LABOUR EXPENSES EXPENSES COST COST COST COST

OVERHEADS

PRODUCTION OR ADMINISTRATION SELLING DISTRIBUTION

WORKS OVERHEADS OVERHEADS OVERHEADS OVERHEADS

1.10.1 Direct materials : Materials which are present in the finished product(cost object) or can be economically identified in the product are called direct materials. For example, cloth in dress making; materials purchased for a specific job etc. Note: However in some cases a material may be direct but it is treated as indirect, because it is used in small quantities and it is not economically feasible to identify that quantity. 1.10.2 Direct labour : Labour which can be economically identified or attributed wholly to a cost object is called direct labour. For example, labour engaged on the actual production of the product or in carrying out the necessary operations for converting the raw materials into finished product. 1.10.3 Direct expenses : It includes all expenses other than direct material or direct labour which are specially incurred for a particular cost object and can be identified in an economically feasible way. 1.10.4 Indirect materials : Materials which do not normally form part of the finished product (cost object) are known as indirect materials. These are —

Stores used for maintaining machines and buildings (lubricants, cotton waste, bricks

1.17

Page 38: 30510870 Cost Accounting and Financial Management

Cost Accounting

etc.)

Stores used by service departments like power house, boiler house, canteen etc. 1.10.5 Indirect labour : Labour costs which cannot be allocated but can be apportioned to or absorbed by cost units or cost centres is known as indirect labour. Examples of indirect labour includes - charge hands and supervisors; maintenance workers; etc. 1.10.6 Indirect expenses : Expenses other than direct expenses are known as indirect expenses. Factory rent and rates, insurance of plant and machinery, power, light, heating, repairing, telephone etc., are some examples of indirect expenses. 1.10.7 Overheads : It is the aggregate of indirect material costs, indirect labour costs and indirect expenses. The main groups into which overheads may be subdivided are the following : (i) Production or Works overheads (ii) Administration overheads (iii) Selling overheads (iv) Distribution overheads

1.11 CLASSIFICATION OF COSTS It means the grouping of costs according to their common characteristics. The important ways of classification of costs are : (1) By nature or element (2) By functions (3) As direct and indirect (4) By variability (5) By controllability (6) By normality 1.11.1 By Nature of Element - Under this classification the costs are divided into three categories i.e., materials cost, labour cost and expenses. This type of classification is useful to determine the total cost. 1.11.2 By Functions - Under this classification, costs are divided according to the function for which they have been incurred. Some of the examples are : Production cost - The cost of sequence of operations which begins with supplying materials, labour and services and ends with primary packing of the product. Selling cost - The cost seeking to create and stimulate demand (sometimes termed ‘marketing’) and of securing orders. Distribution cost - The cost of the sequence of operations which begins with making the packed product available for despatch and ends with making the reconditioned returned empty package, if any available for re-use.

1.18

Page 39: 30510870 Cost Accounting and Financial Management

Basic Concept

Note - It also includes expenditure incurred in transporting articles to central or local storage. Distribution costs include expenditure incurred in moving articles to and from prospective customers as in case of goods on sale or return basis. In the gas, electricity and water industry distribution means pipes, mains and services which may be regarded as the equivalent of packing and transportation. Administrative cost - The cost of formulating the policy, directing the organisation and controlling the operations of an undertaking which is not related directly to a production, selling and distribution, research or development activity or function. Research cost - The cost of researching for new or improved products, new applications of materials, or improved methods. Development cost - The cost of the process which begins with the implementation of the decision to produce a new or improved product or to employ a new or improved method and ends with commencement of formal production of that product or by that method. Pre-production cost - The part of development cost incurred in making a trial production run preliminary to formal product. Note - This term is sometimes used to cover all activities prior to production including research and development, but in such cases the usage should be made clear in the context. Conversion cost - The sum of direct wages, direct expenses and overhead cost of converting raw materials to the finished stage or converting a material from one stage of production to the next. Note - In some circumstances this phrase is used to include any excess material cost or loss of material incurred at the particular stage of production. Product costs – Please refer to 1.9.21 Purposes for computing product costs : The three different purposes for computing product costs are as follows : (i) Preparation of financial statements: Here focus is on inventoriable costs for complying

with Accounting Standard –2 , issued by the Council of ICAI. (ii) Product pricing: It is an important purpose for which product costs are used. For this

purpose, the cost of the other areas of the value chain should be included to make the product available to the customer.

(iii) Contracting with government agencies: Normally such contracts are on a cost plus basis. For this purpose government agencies may not allow the contractors to recover research and development and marketing costs under cost plus contracts.

1.11.3 As Direct and Indirect – Please refer to 1.9.3 and 1.9.4

1.19

Page 40: 30510870 Cost Accounting and Financial Management

Cost Accounting

1.11.4 By Variability - According to this classification costs are classified into three groups viz., fixed, variable and semi-variable. (a) Fixed costs - These are the costs which are incurred for a period, and which, within

certain output and turnover limits, tend to be unaffected by fluctuations in the levels of activity (output or turnover). They do not tend to increase or decrease with the changes in output. For example, rent, insurance of factory building etc., remain the same for different levels of production. A fixed cost can be depicted graphically as follows,

Total Fixed Cost Rs. 1,000

Activity Level The graph shows that the cost is constant (Rs.1,000) for all activity levels. However, it should be noted that this is only true for a relevant range of activity. Fixed costs tend to change beyond the relevant range. Such cost behaviour pattern is described as a stepped fixed cost:

Total Fixed Relevant Relevant Relevant Cost (Rs.) Range 1 Range 2 Range 3 Activity Level (b) Variable costs - These costs tend to vary with the volume of activity. Any increase in the

activity results in an increase in the variable cost and vice-versa. For example, cost of direct labour, etc. Variable costs are depicted graphically as follows,

Total variable Cost (Rs.)

Activity Level

1.20

Page 41: 30510870 Cost Accounting and Financial Management

Basic Concept

(c) Semi-variable costs - These costs contain both fixed and variable components and are thus partly affected by fluctuations in the level of activity. Examples of semi variable costs are telephone bills, gas and electricity etc. Such costs are depicted graphically as follows:

Total Cost (Rs.) Variable Cost Fixed Cost

Activity Level Methods of segregating Semi-variable costs into fixed and variable costs - The segregation of semi-variable costs into fixed and variable costs can be carried out by using the following methods: (a) Graphical method (b) High points and low points method (c) Analytical method (d) Comparison by period or level of activity method (e) Least squares method (a) Graphical method: Under this method, a large number of observations regarding the total costs at different levels of output are plotted on a graph with the output on the X-axis and the total cost on the Y-axis. Then, by judgment, a line of “best-fit”, which passes through all or most of the points is drawn. The point at which this line cuts the Y-axis indicates the total fixed cost component in the total cost. If a line is drawn at this point parallel to the X-axis, this indicates the fixed cost. The variable cost, at any level of output, is derived by deducting this fixed cost element from the total cost. The following graph illustrates this:

1.21

Page 42: 30510870 Cost Accounting and Financial Management

Cost Accounting

(b) High points and low points method: - Under this method in the following illustration the difference between the total cost at highest and lowest volume is divided by the difference between the sales value at the highest and lowest volume. The quotient thus obtained gives us the rate of variable cost in relation to sales value. The fixed cost is the remainder. See the following illustration. Illustration : Sales value Total cost Rs. Rs. At the Highest volume 1,40,000 72,000 At the Lowest volume 80,000 60,000 60,000 12,000 Thus, Variable Cost (Rs. 12,000/Rs. 60,000) = 1/5 or 20% of sales value

= Rs. 28,000 (at highest volume) Fixed Cost : Rs. 72,000 – Rs. 28,000 i.e., (20% of Rs. 1,40,000) = Rs. 44,000. Alternatively : Rs. 60,000 – Rs. 16,000 (20% of Rs. 80,000) = Rs. 44,000. (c) Analytical method: Under this method an experienced cost accountant tries to judge empirically what proportion of the semi-variable cost would be variable and what would be fixed. The degree of variability is ascertained for each item of semi-variable expenses. For example, some semi-variable expenses may vary to the extent of 20% while others may vary to the extent of 80%. Although it is very difficult to estimate the extent of variability of an expense, the method is easy to apply. (Go through the following illustration for clarity).

1.22

Page 43: 30510870 Cost Accounting and Financial Management

Basic Concept

Illustration Suppose, last month the total semi-variable expenses amounted to Rs. 3,000. If the degree of variability is assumed to be 70%, then variable cost = 70% of Rs. 3,000 = Rs. 2,100. Fixed cost = Rs. 3,000 – Rs. 2,100 = Rs. 900. Now in the future months, the fixed cost will remain constant, but the variable cost will vary according to the change in production volume. Thus, if in the next month production increases by 50%, the total semi-variable expenses will be: Fixed cost of Rs. 900, plus variable cost viz., Rs. 3,150 i.e., (Rs. 2,100(V.C.) plus 50% increase of V.C. i.e., Rs. 1,050) i.e., Rs. 4,050. (d) Comparison by period or level of activity method - Under this method, the variable overhead may be determined by comparing two levels of output with the amount of expenses at those levels. Since the fixed element does not change, the variable element may be ascertained with the help of the following formula.

output ofquantity the in Changeexpense ofamount the in Change

Suppose the following information is available: Production Units Semi-variable expenses Rs. January 100 260 February 140 300 Difference 40 40 The variable cost :

volume production in Change

expenses variable Semi in Change − =units 40

40 .Rs = Re. 1/unit

Thus, in January, the variable cost will be 100 × Re. 1 = Rs. 100 and the fixed cost element will be (Rs. 260 – Rs. 100) or Rs. 160. In February, the variable cost will be 140 × Re. 1 = Rs. 140 whereas the fixed cost element will remain the same, i.e., Rs. 160. (e) Least squared method : This is the best method to segregate semi-variable costs into its fixed and variable components. This is a statistical method and is based on finding out a line of best fit for a number of observations. The method uses the linear equation y = mx + c, where m represents the variable element of cost per unit, ‘c’ represents the total fixed cost, ‘y’ represents the total cost, ‘x’ represents the volume of output. The total cost is thus split into its

1.23

Page 44: 30510870 Cost Accounting and Financial Management

Cost Accounting

fixed and variable elements by solving this equation. By using this method, the expenditure against an item is determined at various levels of output and values of x and y are fitted in the above formula to find out the values of m and c. The following illustration may be helpful to understand this method. Level of activity Capacity % 60% 80% Volume (Labour hours) x 150 200 Semi-variable expenses (maintenance of plant) y Rs. 1,200 Rs. 1,275 Substituting the values of x and y in the equation, y = mx + c, at both the levels of activity, we get 1,200 = 150 m + c 1,275 = 200 m + c On solving the above equation, we get (c) (Fixed cost) = Rs. 975 and m (Variable cost) = Rs. 1.50 per labour hour. Advantages of Classification of Costs into Fixed and Variable: The primary objective of segregating semi-variable expenses into fixed and variable is to ascertain marginal costs. Besides this, it has the following advantages also. (a) Controlling expenses: The classification of expenses into fixed and variable components helps in controlling expenses. Fixed costs are generally policy costs, which cannot be easily reduced. They are incurred irrespective of the output and hence are more or less non controllable. Variable expenses vary with the volume of activity and the responsibility for incurring such an expenditure is determined in relation to the output. The management can control these costs by giving proper allowances in accordance with the output achieved. (b) Preparation of budget estimates: The segregation of overheads into fixed and variable part helps in the preparation of flexible budget. It enables a firm to estimate costs at different levels of activity and make comparison with the actual expenses incurred. Suppose in October, 2006 the output of a factory was 1,000 units and the expenses were:

Rs. Fixed 5,000 Variable 4,000 Semi-variable (40% fixed) 6,000 15,000

1.24

Page 45: 30510870 Cost Accounting and Financial Management

Basic Concept

In November, 2006 the output was likely to increase to 1,200 units. In that case the budget or estimate of expenses will be :

Rs. Fixed 5,000 Variable 4,800

⎟⎠

⎞⎜⎝

⎛ ×units 1,000

units 1,200Rs.4,000

Semi-variable Fixed, 40% of Rs. 6,000 2,400

Variable: ⎥⎦

⎤⎢⎣

⎡ ×units 1,000

units 1,200Rs.3,600 4,320 6,720

16,520 It would be a mistake to think that with the output going up from 1,000 units to 1,200 units the expenses would increase proportionately to Rs. 18,000. (c) Decision making: The segregation of semi variable cost between fixed and variable overhead also helps the management to take many important decisions. For example, decisions regarding the price to be charged during depression or recession or for export market. Likewise, decisions on make or buy, shut down or continue, etc., are also taken after separating fixed costs from variable costs. In fact, when any change is contemplated, say, increase or decrease in production, change in the process of manufacture or distribution, it is necessary to know the total effect on cost (or revenue) and that would be impossible without a correct segregation of fixed and variable costs. The technique of marginal costing, cost volume profit relationship and break-even analysis are all based on such segregation. 1.11.5 By Controllability - Costs here may be classified into controllable and uncontrollable costs. (a) Controllable costs - These are the costs which can be influenced by the action of a specified member of an undertaking. A business organisation is usually divided into a number of responsibility centres and an executive heads each such centre. Controllable costs incurred in a particular responsibility centre can be influenced by the action of the executive heading that responsibility centre. Direct costs comprising direct labour, direct material, direct expenses and some of the overheads are generally controllable by the shop level management.

1.25

Page 46: 30510870 Cost Accounting and Financial Management

Cost Accounting

(b) Uncontrollable costs - Costs which cannot be influenced by the action of a specified member of an undertaking are known as uncontrollable costs. For example, expenditure incurred by, say, the Tool Room is controllable by the foreman incharge of that section but the share of the tool-room expenditure which is apportioned to a machine shop is not to be controlled by the machine shop foreman. The distinction between controllable and uncontrollable costs is not very sharp and is sometimes left to individual judgement. In fact no cost is uncontrollable; it is only in relation to a particular individual that we may specify a particular cost to be either controllable or uncontrollable. 1.11.6 By Normality - According to this basis cost may be categorised as follows: (a) Normal cost - It is the cost which is normally incurred at a given level of output under the

conditions in which that level of output is normally attained. (b) Abnormal cost - It is the cost which is not normally incurred at a given level of output in

the conditions in which that level of output is normally attained. It is charged to Costing Profit and loss Account.

1.12 CODING SYSTEMS

Codes The Chartered Institute of Management Accountants has defined a code as “ a system of symbols designed to be applied to a classified set of items to give a brief account reference , facilitating entry collation and analysis” Hence cost classification forms the basis of any cost coding. It helps us understand the characteristic of any cost through a short symbolised form. Composite Codes Let us consider the following example A company has devised a system of codification in which the first three digits indicate the nature of the expenditure and the last three digits the cost centre or cost unit to be charged e.g. if the first digit is 1, the system implies that it refers to raw material and if the number is 2 it represents a labour cost. The second and third numbers relating to 1 i.e., raw material, provide details of the type e.g., whether the raw material is an electronic component (number 4), mechanical component (number 1) consumables(number 2) or packing (number 3) and the name respectively. Hence the description of a cost with a code 146.729 shall be understood as follows: ♦ Since the first number is 1 the cost refers to raw material cost

1.26

Page 47: 30510870 Cost Accounting and Financial Management

Basic Concept

♦ The second number being 4 indicates that the raw material is an electronic component. ♦ The third number 6 refers to the description which according to the company’s codification

refers to Diodes. The last three numbers provide details of the cost centre e.g. the first number provides details of the location of the plant, the second number gives detail of the department (machining or assembly or something else) and the third number indicates whether the cost is direct or indirect. Advantages of a coding system The following are some of the advantages of a well-designed coding system : (a) Since the code is, most of the times, briefer than a description, it saves time when

systems are worked upon manually and in case the system is computerised it reduces the data storage capacity. The illustration above demonstrates this advantage very clearly.

(b) A code helps in reducing ambiguity. In case two professionals understand the same item differently a code will help them objectively.

(c) Unlike detailed descriptions, a code facilitates data processing in computerised systems. The requirements for an efficient coding system (a) Every number used in the code should be unique and certain, i.e. it should be easily

identified from the structure of the code. (b) Elasticity and comprehensiveness is an absolute must for a well designed coding system.

It should be possible to identify a code for every item and the coding system should be capable of expanding to accommodate new items.

(c) The code should be brief and meaningful. (d) The maintenance of the coding system should be centrally controlled. It should not be

possible for individuals to independently add new codes to the existing coding system. (e) Codification systems should be of the same length. This makes errors easier to spot and

it assists computerised data processing.

1.13 TYPES OF COSTING For ascertaining cost, following types of costing are usually used. 1.13.1 Uniform Costing: When a number of firms in an industry agree among themselves to follow the same system of costing in detail, adopting common terminology for various items and processes they are said to follow a system of uniform costing. In such a case, a

1.27

Page 48: 30510870 Cost Accounting and Financial Management

Cost Accounting

comparison of the performance of each of the firms can be made with that of another, or with the average performance in the industry. Under such a system it is also possible to determine the cost of production of goods which is true for the industry as a whole. It is found useful when tax-relief or protection is sought from the Government. 1.13.2 Marginal Costing: It is defined as the ascertainment of marginal cost by differentiating between fixed and variable costs. It is used to ascertain effect of changes in volume or type of output on profit. 1.13.3 Standard Costing and variance analysis: It is the name given to the technique whereby standard costs are pre-determined and subsequently compared with the recorded actual costs. It is thus a technique of cost ascertainment and cost control. This technique may be used in conjunction with any method of costing. However, it is especially suitable where the manufacturing method involves production of standardised goods of repetitive nature. 1.13.4 Historical Costing: It is the ascertainment of costs after they have been incurred. This type of costing has limited utility. 1.13.5 Direct Costing: It is the practice of charging all direct costs to operations, processes or products leaving all indirect costs to be written off against profits in which they arise. 1.13.6 Absorption Costing: It is the practice of charging all costs, both variable and fixed to operations, processes or products. This differs from marginal costing where fixed costs are excluded.

1.14 METHODS OF COSTING Different industries follow different methods of costing because of the differences in the nature of their work. The various methods of costing are as follows: 1.14.1 Job Costing: In this case the cost of each job is ascertained separately. It is suitable in all cases where work is undertaken on receiving a customer’s order like a printing press, motor workshop, etc. In case a factory produces a certain quantity of a part at a time, say 5,000 rims of bicycle, the cost can be ascertained like that of a job. The name then given is Batch Costing. 1.14.2 Batch Costing: It is the extension of job costing. A batch may represent a number of small orders passed through the factory in batch. Each batch here is treated as a unit of cost and thus separately costed. Here cost per unit is determined by dividing the cost of the batch by the number of units produced in the batch. 1.14.3 Contract Costing : Here the cost of each contract is ascertained separately. It is suitable for firms engaged in the construction of bridges, roads, buildings etc.

1.28

Page 49: 30510870 Cost Accounting and Financial Management

Basic Concept

1.14.4 Single or Output Costing : Here the cost of a product is ascertained, the product being the only one produced like bricks, coals, etc. 1.14.5 Process Costing : Here the cost of completing each stage of work is ascertained, like cost of making pulp and cost of making paper from pulp. In mechanical operations, the cost of each operation may be ascertained separately ; the name given is operation costing. 1.14.6 Operating Costing : It is used in the case of concerns rendering services like transport, supply of water, retail trade etc. 1.14.7 Multiple Costing : It is a combination of two or more methods of costing outlined above. Suppose a firm manufactures bicycles including its components; the parts will be costed by the system of job or batch costing but the cost of assembling the bicycle will be computed by the Single or output costing method. The whole system of costing is known as multiple costing.

1.15. DIRECT EXPENSES 1.15.1. Meaning of Direct Expenses : Direct Expenses are also termed as ‘Chargeable expenses’. These are the expenses which can be allocated directly to a cost object. Direct expenses are defined as ‘costs other than material and wages which are incurred for a specific product or saleable services’. Examples of direct expenses are : (i) Hire charges of special machinery or plant for a particular production order or job. (ii) Payment of royalties. (iii) Cost of special moulds, designs and patterns. (iv) Experimental costs before undertaking the job concerned. (v) Travelling and conveyance expenses incurred in connection with a particular job. (vi) Sub-contracting expenses or outside work costs if jobs are sent out for special

processing. 1.15.2 Characteristics of Direct Expenses : (i) Direct expenses are those expenses, which are other than the direct materials and direct

labour. (ii) These expenses are either allocated or charged completely to cost centres or work

orders. (iii) These expenses are included in prime cost of a product.

1.29

Page 50: 30510870 Cost Accounting and Financial Management

Cost Accounting

The nature of direct expenses demands a strict control over such expenses. This feature of controlling direct expenses in business houses compels their management to treat some of the direct expenses as indirect expense. Sometime a direct expense is assumed as indirect due to the convenience. Sometimes a concern may treat an expense as direct whereas another may treat the same expense as indirect. Further the amount of these expenses in proportion to the total cost also influences the decision to treat them as direct expense or as an indirect expense. 1.15.3 Sub-contracting : It is a common business practice followed by business concerns, under which operations requiring special processing are sub-contracted. Examples of such operations are painting, cutting, stitching etc. This is done due to following reasons : (i) The operations, which are given to outside sub-contractors, are those operations, which

require the use of special skill, or special equipment, which is not available with the concern.

(ii) If the management of a concern intends to engage available labour hours and machine hours for operations, which require special skill or special facility available with the concern, the operations, which require a lower level of skill or general-purpose machine, are given to outsider.

(iii) If there is temporary increase in demand of product of a concern, some of the operations are given to outsiders to bridge the imbalance between the production capacities available with various work-centres. The payments made to sub-contractors or outsiders are charged as direct expenses of the specific jobs/work orders.

1.15.4. Documentation: The basic document which is used for the charging of direct expenses to products or batches or work order is the invoice received from suppliers of such service. The payment to supplier of service is made on the basis of invoice & expenses are booked in financial accounts. For example, hiring charges of a machine is charged to the product for which it is hired on the basis of invoice received from supplier of machinery. 1.15.5. Identification of direct expenses with main product or service: For the identification of direct expense with main product or service it is required that the code number of product or service should be inscribed on invoice received from supplier of services. For example, if a machine is hired to complete a particular product, then the hiring charging of a machine is direct expense of the particular product. For charging hiring charges of machine to a particular product it is required that the invoice received from supplier of machine should be coded with the product code for ensuring that the hiring charges of machine is charged to the particular product. Alternatively, if cash is paid, then the cash book analysis will show the product code which is to be charged with the cost of hiring machinery.

1.30

Page 51: 30510870 Cost Accounting and Financial Management

Basic Concept

1.16 Self Examination Questions

Multiple Choice Questions (a) The main purpose of cost accounting is to :

(i) maximise profits; (ii) help in inventory valuation; (iii) provide information to management for decision making; (iv) aid in the fixation of selling price.

(b) One of the most important tools in cost planning is : (i) direct cost; (ii) budget; (iii) cost sheet; (iv) marginal costing.

(c) Increase in total variable cost is due to : (i) increase in fixed cost; (ii) increase in sales; (iii) increase in production.

(d) An example of fixed cost is : (i) direct material cost; (ii) works manager’s salary; (iii) depreciation of machinery; (iv) chargeable expenses.

(e) Cost of goods produced includes : (i) production cost and finished goods inventory; (ii) production cost and work-in-progress; (iii) production cost, work-in-progress and finished goods inventory.

(f) Conversion cost is equal to the total of : (i) material cost and direct wages; (ii) material cost and indirect wages;

1.31

Page 52: 30510870 Cost Accounting and Financial Management

Cost Accounting

(iii) direct wages and factory overhead; (iv) material cost and factory overhead.

(g) Costs which are ascertained after they have been incurred are known as (i) imputed costs; (ii) sunk costs; (iii) historical costs; (iv) opportunity costs.

(h) Prime costs plus variable overhead is known as : (i) production cost; (ii) marginal costs; (iii) total cost; (iv) cost of sales.

(i) Indirect costs are known as: (i) Variable costs; (ii) Fixed costs; (iii) Overheads; (iv) None of the above.

(j) Accounting standard 2 recommends the valuation of inventory on: (i) Absorption costing; (ii) Marginal costing; (iii) Activity based costing; (iv) None of the above.

(k) If Rs 10 is spend on producing 10 units and Rs 15 for producing 15, then the fixed cost per unit is: (i) Rs 0; (ii) Rs 1; (iii) Rs 2; (iv) None of the above.

(l) The variable cost per unit is:

1.32

Page 53: 30510870 Cost Accounting and Financial Management

Basic Concept

(i) Variable in nature; (ii) Fixed in nature; (iii) Semivariable in nature; (iv) None of the above.

(m) The total cost for producing 10 items is Rs 15 and that for producing 15 items is Rs 20. What is the fixed cost? (i) Rs 10; (ii) Rs 15; (iii) Rs 5; (iv) None of the above.

(n) A company presently does not utilise its available capacity. In case of full capacity utilisation, the cost per unit shall (i) Increase; (ii) Decrease; (iii) Remain constant; (iv) None of the above.

(o) The following extract is taken from the distribution cost budget of DC Ltd. Volume delivered (units) 8,000 14,000 Distribution cost Rs. 7,200 Rs. 10,500

The budgeted cost allowance for distribution cost for a delivery volume of 12,000 units is (i) Rs. 6,600 (ii) Rs. 9,000 (iii) Rs. 9,400 (iv) Rs. 10,800

Answers to Multiple Choice Questions (a) iii; (b) ii; (c) iii; (d) ii; (e) ii; (f) iii; (g) iii; (h) ii; (i) iii; (j) i; (k) i; (l) ii; (m) i; (n) ii; (o) iii

Short Answer Type Questions 1. What is Cost Accounting? Explain its important objectives.

1.33

Page 54: 30510870 Cost Accounting and Financial Management

Cost Accounting

2. Write short note on:

(i) Conversion cost. (ii) Sunk cost. (iii) Differential cost. (iv) Opportunity cost. (v) Out of pocket cost.

3. Distinguish between: (i) Cost centre and cost unit. (ii) Cost control and cost reduction. (iii) Period cost and product cost. (iv) Controllable and non-controllable costs. (v) Estimated cost and standard cost. (vi) Variable cost and direct cost.

4. Write brief answers to the following : (a) While introducing a cost accounting system, to what should most attention be paid-

production or sales ? (b) Are direct expenses more important than indirect expenses ? (c) Is prompt reporting better than accurate reporting ? (d) If the selling price for a product is not within the control of the firm, why should it

have a system of cost accounting ? 5. Below is given a list of industries and also the methods of costing and cost unit. Give the

correct number of methods of costing and unit of cost against each industry. Industry Methods of Costing Unit of Cost (i) Nursing Home (a) Process 1. Each job (ii) Road Transport (b) Job 2. Bed per week or per day (iii) Steel (owning (c) Multiple 3. Per tonne iron ore mines) (d) Single 4. Each contract (iv) Coal (e) Operating 5. Each unit (v) Bicycle (f) Contract 6. Per tonne-kilo-metre

1.34

Page 55: 30510870 Cost Accounting and Financial Management

Basic Concept

(vi) Bridge construction (vii) Interior decoration (viii) Advertising (ix) Furniture (x) Sugar company having own sugarcane fields 7. A company manufacturing garments. The following costs are incurred by the company.

You are required to group the costs which are listed below into following classifications: (a) direct materials (b) direct labour (c) direct expenses (d) indirect production overhead (e) research and development cost (f) selling & distribution cost (g) administration costs (h) finance costs.

(i) Lubricant for sewing machines (ii) Floppy disk for general office computer (iii) Maintenance contact for office photocopying machine (iv) Road licence for delivery vehicle (v) Market research prior to new product launch (vi) Cost of painting advertising slogans in delivery vans (vii) Trade magazine (viii) Upkeep of delivery vehicles (ix) Wages of operative in cutting department (x) Interest on bank overdraft

Long Answer Type Questions 1. State and explain the differences between Financial Accounting, Cost Accounting and

Management Accounting.

1.35

Page 56: 30510870 Cost Accounting and Financial Management

Cost Accounting

2. What do you understand by direct expenses? What are their characteristics? 3. Discuss the factors which a Cost Accountant should consider before installing a costing

system in a manufacturing concern. 4. Discuss the various methods known to you which may be utilised for segregating a semi

variable cost into its fixed and variable component. 5. How is Cost Accounting important to business concerns? Discuss.

1.36

Page 57: 30510870 Cost Accounting and Financial Management

CHAPTER 2

MATERIAL

Learning Objectives

When you have finished studying this chapter, you should be able to

♦ Understand the need and importance of material control. ♦ Describe the procedures involved in procuring, storing and issuing material. ♦ Differentiate amongst the various methods of valuing material. ♦ Understand the meaning and the accounting treatment for normal and abnormal loss of

material. ♦ Understand the meaning and the accounting treatment of waste, scrap, spoilage and

defectives.

2.1 INTRODUCTION We have acquired a basic knowledge about the concepts, objectives, advantages, methods and elements of cost. We shall now study each element of cost separately. The first element of cost is “Direct Material Cost”. Materials constitute a very significant proportion of total cost of finished product in most of the manufacturing industries. A proper recording and control over the material costs is essential because of the following : (a) The exact quality of specification of materials required should be determined according to

the required quality of the finished product. If too superior quality of material is purchased, it would mean higher cost due to high prices; if the quality of materials purchased is too low, the product will be of inferior quality.

(b) The price paid should be the minimum possible otherwise the higher cost of the finished products would make the product uncompetitive in the market.

(c) There should be no interruption in the production process for want of materials and stores, including small inexpensive items like lubricating oil for a machine. Sometime their out of stock situation may lead to stoppage of machines.

Page 58: 30510870 Cost Accounting and Financial Management

Cost Accounting (d) There should be no over stocking of materials because that would result in loss of

interest charges, higher godown charges, deterioration in quality and losses due to obsolescence (either due to manufacture of certain articles being given up or the material previously required for the production not being required any longer due to a change in methods of production).

(e) Wastage and losses while the materials are in store should be avoided as far as possible; and

(f) Wastage during the process of manufacture should be the minimum possible. It may also be added that information about availability of materials and stores should be continuously available so that production may be planned properly and the required materials purchased in time.

2.2 MATERIAL CONTROL

The publication of the Institute of Cost and Management Accountants on Budgetary Control defines it as “the function of ensuring that sufficient goods are retained in stock to meet all requirements without carrying unnecessarily large stocks”. When the functions of indexing buying, receiving, inspection, storing and paying of the goods are separated it is essential that these should be properly co-ordinated so as to achieve the advantages of specialisation.

2.2.1 Objectives of system of material control : The objectives of a system of material control are the following : (i) Ensuring that no activity, particularly production, suffers from interruption for want of

materials and stores-it should be noted that this requires constant availability of every item that may be needed howsoever small its cost may be. Lubricating oil may cost much less than the main raw material but, from the point of view of uninterrupted production, both have equal importance.

(ii) Seeing to it that all the materials and stores are acquired at the lowest possible price considering the quality that is required and considering other relevant factors like reliability in respect of delivery, etc.

(iii) Minimisation of the total cost involved, both for acquiring stocks (apart from the price paid to the supplier) and for holding them.

(iv) Avoidance of unnecessary losses and wastages that may arise from deterioration in quality due to defective or long storage or from obsolescence. It may be noted that losses and wastages in the process of manufacture, concern the production department.

2.2

Page 59: 30510870 Cost Accounting and Financial Management

Material

(v) Maintenance of proper records to ensure that reliable information is available for all items of materials and stores that not only helps in detecting losses and pilferages but also facilitates proper production planning.

The fulfilment of the objectives mentioned above will require that standard lists of all the materials and stores required for the firm’s work be drawn up with the weekly consumption figures. Also the lead time for each item has to be determined which will then enable the firm to ascertain the minimum quantity for each items. It is also necessary to fix maximum quantity so that capital is not locked up unnecessarily and the risk of obsolescence is minimised. Costs are minimised through the use of ABC analysis (which means classification of the various items on the basis of investment involved into three categories, viz., A, B and C.)

2.2.2 Requirements of material control - Material control requirements are as follows:— 1. Proper co-ordination of all departments involved viz., finance, purchasing, receiving,

inspection, storage, accounting and payment. 2. Determining purchase procedure to see that purchases are made, after making suitable

enquiries, at the most favourable terms to the firm. 3. Use of standard forms for placing the order, noting receipt of goods, authorising issue of

the materials etc. 4. Preparation of budgets concerning materials, supplies and equipment to ensure economy

in purchasing and use of materials. 5. Operation of a system of internal check so that all transactions involving materials,

supplies and equipment purchases are properly approved and automatically checked. 6. Storage of all materials and supplies in a well designated location with proper

safeguards. 7. Operation of a system of perpetual inventory together with continuous stock checking so

that it is possible to determine at any time the amount and value of each kind of material in stock.

8. Operation of a system of stores control and issue so that there will be delivery of materials upon requisition to departments in the right amount at the time they are needed.

9. Development of system of controlling accounts and subsidiary records which exhibit summary and detailed material costs at the stage of material receipt and consumption.

2.3

Page 60: 30510870 Cost Accounting and Financial Management

Cost Accounting 10. Regular reports of materials purchased, issue from stock, inventory balances, obsolete

stock, goods returned to vendors, and spoiled or defective units.

2.3 MATERIALS PROCUREMENT PROCEDURE

If a concern can afford it, there should be a separate purchase department for all purchases to be made on behalf of all other departments. Purchasing should be centralised i.e. all purchases should be done by the purchasing department except for small purchases which may be done by the user’s department. What is needed is that there should be staff wholly devoted to purchasing. Such a staff is bound to become expert in the various matters to be attended to, for examples— units of materials to be purchased and licences to be obtained, transport, sources of supply, probable price etc. The concerned officers in this department keep themselves in constant touch with the markets either by reading various trade magazines or by direct association, to have the latest information.

If a concern has a number of factories requiring the same material and stores, there will be advantage in centralising purchases since important economies in prices and even transport may be obtained by placing large orders at a time. In such a case inspection at a source may also be possible. Of course, there will be a little delay in supply because of clerical processes and sometimes, there may be a misunderstanding resulting in supply of wrong articles.

However, there is no advantage in centralised purchasing if different materials, and stores are required by different plants.

Materials purchase department in a business house is confronted with the following issues: (i) What to purchase ? (ii) When to purchase ? (iii) How much to purchase ? (iv) From where to purchase. (v) At what price to purchase. To overcome the above listed issues, the purchase department follows the procedure involving following steps : 1. Receiving purchase requisitions. 2. Exploring the sources of materials supply and selecting suitable material suppliers.

2.4

Page 61: 30510870 Cost Accounting and Financial Management

Material

3. Preparation and execution of purchase orders. 4. Receipt and inspection of materials. 5. Checking and passing of bills for payment. 2.3.1 Receiving purchase requisitions - Since the materials and stores purchased will be used by the production departments, there should be constant co-ordination between the purchase and production departments.

A purchase requisition is a form used for making a formal request to the purchasing department to purchase materials. This form is usually filled up by the store keeper for regular materials and by the departmental head for special materials (not stocked as regular items). The requisition form is duly signed by either works manager or plant superintendent, in addition to the one originating it.

At the beginning a complete list of materials and stores required should be drawn up, the list should have weekly consumption figures. It should be gone through periodically so that necessary deletion and addition may be made. If there is any change in the rate of consumption per week (say, due to extra shift being worked), the purchase department should be informed about the new figures. Once an item has been included in the standard list, it becomes the duty of the purchase department to arrange for fresh supplies before existing stocks are exhausted. But if the production department requires some new material, it should make out an indent well in time and send it to the purchase department for necessary action.

Control over buying - For control over buying of regular stores materials it is necessary to determine their maximum, minimum, reorder level and economic order quantities. The use of economic order quantities and various levels constitutes an adequate safeguard against improper indenting of regular materials. In respect of special materials, required for a special order or purpose, it is desirable that the technical department concerned should prepare materials specifications list specifying the quantity, size and order specifications of materials to be drawn from the store and those to be specially procured.

In all cases, the starting point in the process for purchasing is the issue of a proper purchase requisition (form is given below). (Students may note that the forms suggested in this booklet as well as in the text books are illustrative in nature. The actual form may differ under different circumstances). It may originate either in the stores department in connection with regular stock of materials or in the production planning or in other technical departments concerned in respect of special materials. Its purpose is to request and authorise the purchase department to order to procure the materials specified in

2.5

Page 62: 30510870 Cost Accounting and Financial Management

Cost Accounting stated quantities. It should be made out in triplicate. The original copy is sent to the purchasing department, the duplicate is kept by the storekeeper or the department which initates the requisition and the triplicate is sent to the authorising executive.

Purchase Requisition (Regular/Special) (Use a separate form for each item)

No.................. Department................................. Date............. Purchase................... Date by which material required............................... Description of Quantity Exact specification Materials required required ........................ Indentor For use in purchase department Firm 1. 2. 3. Order Quotations No. & Date........................... ....................................................................................... Price (including charges) With...................................... ....................................................................................... Price............................... Date of Delivery Date of dly................... ....................................................................................... Remarks Purchase Manager

2.6

Page 63: 30510870 Cost Accounting and Financial Management

Material

A specimen form of purchase requisition for reordering regular stock items. Purchase Requistion

No. Date S.No. Name of Present Minimum Maximum Remarks article stock stock stock Remarks .............................. .............................. .............................. .............................. Purchase manager Stores Ledger Clerk 2.3.2 Exploring the sources of materials supply and selecting suitable material suppliers: A source for the supply of each material may be selected after the receipt of the purchase requisition. Purchase department in each business house usually maintains a list of suppliers for each group of materials, required by their concern. Atleast three quotations are invited from such suppliers. On the receipt of these quotations a comparative statement is prepared. For selecting material suppliers the factors which the purchase department keeps in its mind are—price; quantity; quality offered; time of delivery; mode of transportation; terms of payment; reputation of supplier; etc. In addition to the above listed factors purchase manager obtains the necessary information from the statement of quotations; past records, buyer guides etc. for finally selecting material suppliers.

2.3.3 Preparation and execution of purchase orders : Having decided on the best quotation that should be accepted, the purchase manager or concerned officer proceeds to issue the formal purchase order. It is a written request to the supplier to supply certain specified materials at specified rates and within a specified period.

Copies of purchase order are sent to : (i) The supplier; (ii) Store or the order indenting department; (iii) Receiving department; and (iv) Accounting department. A copy of the purchase order, alongwith relevant purchase requisitions, is held in the file

2.7

Page 64: 30510870 Cost Accounting and Financial Management

Cost Accounting of the department to facilitate the follow-up, of the delivery and also for approving the invoice for payment.

2.3.4 Receipt and inspection of materials: Under every system of stores organisation, a distinction is made between the function of receiving and storing, so that each acts as a check on the other.

The receiving department or section is responsible for taking charge of the incoming materials, checking and verifying their quantities, inspecting them as regards their grade, quality or other technical specifications and if found acceptable, passing them on to the stores (or other departments for which these might have been purchased). In large organisation, a special inspection wing is often attached to the receiving department and, where it is not so, technical appraisal of the incoming supplies is carried out by the general inspecting staff. In case the quality is not the same as ordered, the goods are not accepted. If everything is in order and the supply is considered suitable for acceptance, the Receiving department prepares a Receiving Report or Material Inward Note or Goods Received Note. It is prepared in quadruplicate, the copies being distributed as under : (i) First copy is sent to the Purchase Department for verifying supplier’s bill for payment. (ii) Second copy is sent to the store or the department that indents the material. (iii) Third copy is sent to the stores ledger clerk in the Cost Department. (iv) Fourth and the last copy is retained for use by the receiving department. A good plan would be that the receiving clerk sends all the three copies (meant for others) along with the materials to the store or the department that placed the order. The materials are then physically inspected and the particulars thereof as recorded in the Receiving Report are verified. If the quantity and quality are in order, the delivery of the same is accepted and copies of the report are signed; two copies of the report are forwarded to the Purchase Department and the third is kept on the file as documentary evidence of the quantities of stores received for storage or use, as the case may be. The Purchase Department in turn, enters the purchase price and forwards one copy to the Accounts Department and the second to the Cost Department. A specimen form of the receiving report is given below:

2.8

Page 65: 30510870 Cost Accounting and Financial Management

Material

Goods Received Note Received from.............................. No. .............. Order No. .................. Date............................ Amount Quantity Code Description Amount Charges Total Remarks due to Rs. Rs. supplier Rs.

Inspector....................... Store Keeper.................................. Receiver........................ Store Ledger Clerk...................... Material outward return note - Sometimes materials have to be returned to suppliers after these have been received in the factory. Such returns may occur before or after the preparation of the receiving report. If the return takes place before the preparation of the receiving report, such material obviously would not be included in the report and hence not debited in the stores books and ledgers. In that case, no adjustment in the account books would be necessary. But if the material is returned after its entry in the receiving report, a suitable document must be drawn up in support of the credit entry so as to exclude from the Stores of Material Account the value of the materials returned back. This document usually takes the form of a Material outward return note.

The Material outward return note is drawn up by the Stores or the Despatch Department. Five copies of it are usually prepared; two for the supplier (one of which is to be sent back by the supplier after he has signed the same), one for Store, one for Cost (stores) Ledger and one copy to be retained in the Material outward return book. (Please draw up the form yourself and then see the text book).

2.3.5 Checking and passing of bills for payment : The invoice received from the supplier is sent to the stores accounting section to check authenticity and mathematical accuracy. The quantity and price are also checked with reference to goods received note and the purchase order respectively. The stores accounting section after checking its accuracy finally certifies and passes the invoice for payment. In this way the payment is

2.9

Page 66: 30510870 Cost Accounting and Financial Management

Cost Accounting made to supplier.

2.4 MATERIAL ISSUE PROCEDURE Issue of material must not be made except under properly authorised requisition slip; usually it is the foreman of a department who has the authority to draw materials from the store. Issue of material must be made on the basis of first in first out, that is, out of the earliest lot on hand. If care is not exercised in this regard, quality of earliest lot of material may deteriorate for having been kept for a long period. Material requisition note : It is the voucher of the authority as regards issue of materials for use in the factory or in any of its departments. Where a ‘Materials List’ has been prepared, either the whole of the materials would be withdrawn on its basis or separate materials requisitions would be prepared by the person or department and the material drawn upto the limit specified in the list. The Requisition Notes are made out in triplicate. The copies are distributed in the following manner :

One copy for the Store-keeper. One copy for Cost department. One copy for the Department requiring it.

If no material list has been prepared, it is desirable that the task of the preparation of Material Requisition Notes be left to the Planning Department. If there is no Planning Department, (or although in existence, is unable to undertake this task), the Requisition Notes should be prepared by the person or department that requires the materials. Usually, a foreman’s authority is enough but, in the case of costly materials, it would be desirable to have such requisitions duly approved by some higher authority, like the Superintendent or Works Manager before these are presented to Stores.

A specimen form of the Material Requisition is shown below: Material Requisition Note

Work Order No. ............................ No. .......................................... Department ..................................... Date .......................................

Item No. Particulars Qnty. Rate Amount Rs. Rs. Store-keeper Workman receiving Foreman S.L. Clerk the material

2.10

Page 67: 30510870 Cost Accounting and Financial Management

Material

Bill of material : It is also known as Material Specification List or simply Material List. It is a schedule of standard quantities of materials required for any job or other unit of production. A comprehensive Materials List should rigidly lay down the exact description and specifications of all materials required for a job or other unit of production and also required quantities so that if there is any deviation from the standard list, it can easily be detected. The materials List is prepared by the Engineering or Planning Department in a standard form. The number of copies prepared vary according to the requirement of each business, but four is the minimum number. A copy of it is usually sent to each of the following department : (i) Stores department. (ii) Cost Accounts Department. (iii) Production Control department. (iv) Engineering or Planning department. The advantages of using “bill of material”, by the above departments may be summed up as follows:—

Stores Department : 1. A bill of material serves as an important basis of preparing material purchase requisitions

by stores department. 2. It acts as an authorisation for issuing total material requirement. 3. The clerical activity is reduced as the stores clerk issues the entire/part of the material

requirement to the users if the details of material asked are present in the bill of materials.

Cost Accounts Department : 1. Bill of material, is used by Cost Accounts department for preparing an estimate/budget of

material cost for the job/process/operation, it is meant. 2. It may be used as a device for controlling the excess cost of material used. This is done

after determining material variances and ascertaining the reasons for their occurrence.

Production Control Department : 1. Bill of material, may be used by this department for controlling usage of materials. 2. Its usage saves time which otherwise would have been wasted for preparing separate

requisitions of material.

2.11

Page 68: 30510870 Cost Accounting and Financial Management

Cost Accounting Engineering or Planning department : As stated earlier this department prepares the materials list in a standard form. A copy of list is sent to stores, cost accounts and production control department.

Proforma of Bill of Materials Job No. ............................................. No. ...................................... Department authorised ............... Date ................................... Sl. Code Description Qty. Date of Rate Amount No. No. or issue & Qty. Rs. Rs. issued Date Qty. Authorised by .................................................. Received by ......................... Checked by .......................... Store Keeper’s signature ............................. Cost clerk ............................. Transfer of material: The surplus material arising on a job or other units of production may sometime be unsuitable for transfer to Stores because of its bulk, heavy weight, brittleness or some such reason. It may, however, be possible to find some alternative use for such materials by transferring it to some other job instead of returning it to the Store Room.

It must be stressed that generally transfer of material from one job to another is irregular, if not improper, in so far it is not conducive to correct allocation and control of material cost of jobs or other units of production. It is only in the circumstances envisaged above that such direct transfer should be made, at the time of material transfer a material transfer note should be made in duplicate, the disposition of the copies of this note being are as follows :

One copy for the Cost Department, and One copy for the department making the transfer.

No copy is required for the Store as no entry in the stores records would be called for. The Cost Department would use its copy for the purpose of making the necessary entries

2.12

Page 69: 30510870 Cost Accounting and Financial Management

Material

in the cost ledger accounts for the jobs affected. The form of the Material Transfer Note is shown below:

Material Transfer Note From Job No. .................................. No. .................................... To Job No. ........................................ Date ................................. Item No. Particulars Rate Amount Rs. Rs. Transferred by Received by .................................... Job Ledger Clerk

Return of material: Sometimes, it is not possible before hand to make any precise estimate of the material requirements or units of production. Besides, at times due to some technical or other difficulty, it is not practicable to measure exactly the quantity of material required by a department. In either case, material may have to be issued from stores in bulk, often in excess of the actual quantity required. Where such a condition exists, it is of the utmost importance from the point of view of materials control that any surplus material left over on the completion of a job should be promptly hand over to the storekeeper for safe and proper custody.

Unless this is done, the surplus material may be misappropriated or misapplied to some purpose, other than that for which it was intended. The material cost of the job against which the excess material was originally drawn in that case, would be overstated unless the job is given credit for the surplus arising thereon.

The surplus material, when it is returned to the storeroom, should be accompanied by a document known either as a Shop Credit Note or alternatively as a Stores Debit Note. This document should be made out, by the department returning the surplus material and it should be in triplicate to be used as follows:

One copy for the Store Room; One copy for the Cost Department; and One copy (book copy) for the department returning the surplus material.

2.13

Page 70: 30510870 Cost Accounting and Financial Management

Cost Accounting The form of Shop Credit Note is given below:

Shop Credit Note Job No. ...................................... No. ........................ Department ............................ Date ...................... Item No. Particulars Qnty. Rate Amount

Store-keeper S.L. Clerk Foreman of the Returning Department

2.5 MATERIAL STORAGE

Proper storing of materials is of primary importance. It is not enough only to purchase material of the required quality. If the purchased material subsequently deteriorates in quality because of bad storage, the loss is even more than what might arise from purchase of bad quality materials. Apart from preservation of quality, the store-keeper also must ensure safe custody of the material. It should be the function of store-keeper that the right quantity of materials always should be available in stock.

2.5.1 Duties of store keeper : These can be briefly set out as follows : 1. To exercise general control over all activities in Stores Department 2. To ensure safe keeping both as to quality and quantity of materials. 3. To maintain proper records. 4. To initiate purchase requisitions for the replacement of stock of all regular stores items

whenever the stock level of any item of store approaches the minimum limit fixed in respect thereof.

5. To initiate action for stoppage of further purchasing when the stock level approaches the maximum limit.

6. To check and receive purchased materials forwarded by the receiving department and to arrange for the storage in appropriate places.

7. To reserve a particular material for a specific job when so required. 8. To issue materials only in required quantities against authorised requisition

notes/material lists.

2.14

Page 71: 30510870 Cost Accounting and Financial Management

Material

9. To check the book balances, with the actual physical stock at frequent intervals by way of internal control over wrong issues, pilferage, etc.

2.5.2 Minimising the cost of purchasing and store-keeping: There are two types of costs which are involved in making a purchase and keeping the goods in the store. For placing each order, a certain amount of labour is required and, therefore, it will involve a certain sum of money as cost. It should be noted that the cost of making a purchase not only includes the cost incurred by the purchasing department but it also includes the cost of receiving and inspecting the goods. These costs will naturally increase if the number of order is large; there can be saving if the number of orders is reduced. The other type of cost is concerned with keeping the goods is stock, it comprises the money invested, the loss which is likely to take place if the goods are kept, the expenses incurred on looking after the items etc. Larger the stock, higher will be this type of cost. In order to reduce this cost, it is necessary to bring down level of the stock. It may be noted that the number of orders can be cut down only, if the quantity of each order is increased, but if that is done, the average quantity on hand will increase and, therefore, interest and the cost of store keeping will be higher. It is necessary, therefore to have balance between those two costs and to keep total of the two at the minimum level. With this objective in view, the economic order quantity is worked out. But different items for stock have to be treated differently. The name given to such classification is the ‘ABC’ Analysis, or the Selective Inventory Control.

2.5.3 Different classes of stores: Broadly speaking, there are three classes of stores viz., central or main stores, sub-stores and departmental stores. The central stores are the most common of all and in practice, factories generally have only a central store under the control of one store keeper. Such a store is centrally situated and is easily accessible to all departments. If receipts and issues of different items of stores are not large, and the various departments are close to each other, one central store for all purposes is sufficient.

In big organisations, particularly in the case of collieries, tea gardens, etc., where the work spots are distributed over a large area, sub-stores are created. A sub-store is in fact a branch of the central store. It is generally created to facilitate easy accessibility to the various work spots or consumption centres. Only the essential items, as well as those required urgently, are kept in them. The issues to sub-stores are not treated as con-sumption but only as a transfer, from one store (central) to another sub-store. The control in the matter of ordering or receiving rests with the central stores and the sub-stores do not generally receive any item directly.

2.15

Page 72: 30510870 Cost Accounting and Financial Management

Cost Accounting Departmental stores are created normally to minimise the time spent on drawing from stores. For example, a week’s supply may be drawn at one time and kept in a departmental store at a place marked for the purpose. Such stores, however, are essential where one or more production departments work in multiple shifts and the central store works for only one shift; also for the storage of work in progress and semi-finished components where these are large in number or in bulk. Unlike a sub store in the departmental store, the control rests with the department in charge. The materials are generally issued in bulk to the departmental store and it is the responsibility of the department-in-charge to keep proper accounts as regards issues and stock. If the bulk of materials is required for only one department, it is usually stored near the department under the charge of the super intendent concerned.

2.5.4 Stores location : The location of store should be carefully planned. It should be near to the material receiving department so that transportation charges are minimum. At the same time, it should be easily accessible to all other departments of the factory, railway siding, roads etc. Planned location of the stores department avoids delay in the movement of materials to the departments in which they are needed.

2.5.5 Stores layout : The store should be adequately provided with the necessary racks, drawers and other suitable receptacles for storing materials. Each place (for example, a drawer or a corner) where materials are kept is called a bin. Each bin should be serially numbered and for every item a bin should be allowed. All receipts of the item of the same type should be kept in the bin allotted, for convenience of access. The number of the bin should be entered in the Store Ledger concerned accounts.

2.6 STORE RECORD

The record of stores may be maintained in three forms: (a) Bin Cards (b) Stock Control Cards, (c) Stores Ledger. The first two forms of accounts are records of quantities received, issued and those in balance, but the third one is an account of their cost also. Usually, the account is kept in both the forms, the quantitative in the store and quantitative-cum-financial in the Cost Department.

Bin Cards and Stock Control Cards : These are essentially similar, being only quantitative records of stores. The latter contains further information as regards stock on order. Bin

2.16

Page 73: 30510870 Cost Accounting and Financial Management

Material

cards are kept attached to the bins or receptacles or quite near thereto so that these also assist in the identification of stock. The Stock Control Cards, on the other hand, are kept in cabinets or trays or loose binders.

Advantages of Bin Cards : (i) There would be less chances of mistakes being made as entries will be made at the

same time as goods are received or issued by the person actually handling the materials. (ii) Control over stock can be more effective, in as much as comparison of the actual

quantity in hand at any time with the book balance is possible. (iii) Identification of the different items of materials is facilitated by reference to the Bin Card

the bin or storage receptacle. Disadvantages of Bin Cards :

(i) Store records are dispersed over a wide area. (ii) The cards are liable to be smeared with dirt and grease because of proximity to material

and also because of handling materials. (iii) People handling materials are not ordinarily suitable for the clerical work involved in

writing Bin Cards. Advantages of Stock Control Cards :

(i) Records are kept in a more compact manner so that reference to them is facilitated. (ii) Records can be kept in a neat and clean way by men solely engaged in clerical work so

that a division of labour between record keeping and actual material handling is possible. (iii) As the records are at one place, it is possible to get an overall idea of the stock position

without the necessity of going round the stores. Disadvantages of Stock Control Cards :

(i) On the spot comparison of the physical stock of an item with its book balance is not facilitated.

(ii) Physical identification of materials in stock may not be as easy as in the case of bin cards, as the Stock Control Cards are housed in cabinets or trays.

The specimen forms of these cards may be studied from any text book.

Stores Ledger: A Modern Stores Ledger is a collection of cards or loose leaves specially ruled for maintaining a record of both quantity and cost of stores received, issued and those in stock. It being a subsidiary ledger to the main cost ledger, it is maintained by the Cost Accounts Department. It is posted from Goods Received Notes and Materials

2.17

Page 74: 30510870 Cost Accounting and Financial Management

Cost Accounting requisition. The advantages of writing up Stores Ledger mechanically are: (1) It enables distribution of work among a number of clerks due to which receipts and issues

are posted quickly and regularly. (2) It enables stock records to be centralised in case of an organisation having a number of

depots. (3) The accuracy of posting can be mechanically tested more conveniently. (4) The records are clearer and neater. Also the recurring cost of maintaining them is much

less than those kept manually. (5) If up-to-date records are available, the management will be able to exercise greater

control over quantities held in stock from time to time which may result in a great deal of saving in both the amount of investment in stock and their cost.

Now-a-days, mostly a duplicate record of issues and receipt of materials is kept one on Bin Cards in the Store and the second in the Stores Ledger in the stores. The form of ruling the Store Ledger and Bin Cards should be studied from the text book.

2.6.1 Treatment of shortages in stock taking : At the time of stock taking generally discrepancies are found between physical stock shown in the bin card and stores ledger. These discrepancies are in the form of shortages or losses. The causes for these discrepancies may be classified as unavoidable or avoidable.

Losses arising from unavoidable causes should be taken care of by setting up a standard percentage of loss based on the study of the past data. The issue prices may be inflated to cover the standard loss percentage. Alternatively, issues may be made at the purchase price but the cost of the loss or shortage may be treated as overheads.

Actual losses should be compared with the standard and excess losses should be analysed to see whether they are due to normal or abnormal reasons. If they are attributable to normal causes, an additional charge to overheads should be made on the basis of the value of materials consumed. If they arise from abnormal causes, they should be charged to the Costing Profit and Loss account.

Avoidable losses are generally treated as abnormal losses. These losses should be debited to the Costing Profit and Loss Account.

Losses or surpluses arising from errors in documentation, posting etc., should be corrected through adjustment entries.

2.18

Page 75: 30510870 Cost Accounting and Financial Management

Material

2.7 INVENTORY CONTROL

The main objective of inventory control is to achieve maximum efficiency in production and sales with the minimum investment in inventory.

Inventory comprises of stocks of materials, components, work-in-progress, and finished products and stores and spares. The techniques commonly applied for inventory control are as follows:

Techniques of Inventory control : (i) Setting of various stock levels. (ii) ABC analysis. (iii) Two bin system. (iv) Establishment of system of budgets. (v) Use of perpetual inventory records and continuous stock verification. (vi) Determination of economic order quantity. (vii) Review of slow and non-moving items. (viii) Use of control ratios. 2.7.1 Setting of various stock levels:

Minimum level - It indicates the lowest figure of inventory balance, which must be maintained in hand at all times, so that there is no stoppage of production due to non-availability of inventory.

The main consideration for the fixation of minimum level of inventory are as follows: 1. Information about maximum consumption and maximum delivery period in respect of

each item to determine its re-order level. 2. Average rate of consumption for each inventory item. 3. Average delivery period for each item. This period can be calculated by averaging the

maximum and minimum period. The formula used for its calculation is as follows:

Minimum level of inventory = Re-order level – (Average rate of consumption × average time of inventory delivery)

2.19

Page 76: 30510870 Cost Accounting and Financial Management

Cost Accounting Maximum level :

It indicates the maximum figure of inventory quantity held in stock at any time.

The important considerations which should govern the fixation of maximum level for various inventory items are as follows : 1. The fixation of maximum level of an inventory item requires information about its-re-order

level. The re-order level itself depends upon its maximum rate of consumption and maximum delivery period. It in fact is the product of maximum consumption of inventory item and its maximum delivery period.

2. Knowledge about minimum consumption and minimum delivery period for each inventory item should also be known.

3. The determination of maximum level also requires the figure of economic order quantity. 4. Availability of funds, storage space, nature of items and their price per unit are also

important for the fixation of maximum level. 5. In the case of imported materials due to their irregular supply, the maximum level should

be high. The formula used for its calculation is as follows :

Maximum level of inventory = Re-order-level + Re-order quantity − (Minimum consumption × Minimum re-order period)

Re-order level - This level lies between minimum and the maximum levels in such a way that before the material ordered is received into the stores, there is sufficient quantity on hand to cover both normal and abnormal consumption situations. In other words, it is the level at which fresh order should be placed for replenishment of stock. The formula used for its calculation is as follows :

Re-order level = Maximum re-order period × Maximum Usage (or) = Minimum level + (Average rate of consumption × Average time to obtain fresh supplies). Average Inventory Level - This level of stock may be determined by using the following equal formula : Average inventory level = Minimum level + 1/2 Re-order quantity (or)

= 2

level Minimum level Maximum +

2.20

Page 77: 30510870 Cost Accounting and Financial Management

Material

Danger level - It is the level at which normal issues of the raw material inventory are stopped and emergency issues are only made.

Danger level = Average consumption × Lead time for emergency purchases Illustration - Two components, A and B are used as follows : Normal usage 50 per week each Maximum usage 75 per week each Minimum usage 25 per week each Re-order quantity A : 300; B : 500 Re-order period A : 4 to 6 weeks B : 2 to 4 weeks Calculate for each component (a) Re-ordering level, (b) Minimum level, (c) Maximum level, (d) Average stock level.

Solution : (a) Re-ordering level : Minimum usage per week × Maximum delivery period. Re-ordering level for component A = 75 units× 6 weeks = 450 units Re-ordering level for component B = 75 units× 4 weeks = 300 units (b) Minimum level :

Re-order level – (Normal usage × Average period) Minimum level for component A = 450 units – 50 units × 5 weeks = 200 units Minimum level for component B = 300 units – 50 units × 3 weeks = 150 units

(c) Maximum level : ROL + ROQ – (Min. usage × Minimum period) Maximum level for component A = (450 units + 300 units) – (25 units × 4 weeks) = 650

units Maximum level for component B = (300 units + 500 units) – (25 units × 2 weeks) = 750

units (d) Average stock level : ½ (Minimum + Maximum) stock level

2.21

Page 78: 30510870 Cost Accounting and Financial Management

Cost Accounting

Average stock level for component A = ½ (200 units + 650 units) = 425 units. Average stock level for component B = ½ (150 units + 750 units) = 450 units.

Illustration A Company uses three raw materials A, B and C for a particular product for which the following data apply:

Raw Material

Usage per unit

of Product

(Kgs.)

Re-order quantity

(Kgs.)

Price per Kg.

Delivery period

(in weeks)

Re-order level

(Kgs)

Minimum level

(Kgs.)

Minimum Average Maximum Rs.

A 10 10,000 0.10 1 2 3 8,000 B 4 5,000 0.30 3 4 5 4,750 C 6 10,000 0.15 2 3 4 2,000

Weekly production varies from 175 to 225 units, averaging 200 units of the said product. What would be the following quantities : (i) Minimum stock of A ?

(ii) Maximum stock of B ?

(iii) Re-order level of C ?

(iv) Average stock level of A ?

Solution

(i) Minimum stock of A

Re-order level – (Average rate of consumption × Average time required to obtain fresh delivery)

= 8,000 – (200 × 10 × 2) = 4,000 kgs. (ii) Maximum stock of B

Re-order level – (Minimum consumption × Minimum re-order period) + Re-order quantity

2.22

Page 79: 30510870 Cost Accounting and Financial Management

Material

= 4,750 – (175 × 4 × 3) + 5,000

= 9,750 – 2,100 = 7,650 kgs.

(iii) Re-order level of C

Maximum re-order period × Maximum usage

= 4 × 225 × 6 = 5,400 kgs. OR

Re-order level of C = Minimum stock of C + [Average rate of consumption × Average time required

to obtain fresh delivery]

= 2,000 + [(200 × 6) × 3] kgs.

= 5,600 kgs.

(iv) Average stock level of A

= Minimum stock level of A + ½ Re-order quantity of A

= 4,000 + ½ × 10,000 = 4,000 + 5,000 = 9,000 kgs. OR

Average Stock level of A

2

A of level stock Maximum A of level stock Minimum + (Refer to working note)

2

16,250 4,000+ = 10,125 kgs.

Working note

Maximum stock of A = ROL+ ROQ – (Minimum consumption × Minimum re-order period)

= 8,000 + 10,000 – [(175 × 10) × 1]

= 16,250 kgs.

2.7.2 ABC Analysis: It is a system of inventory control. It exercises discriminating control over different items of stores classified on the basis of the investment involved. Usually the items are divided into three categories according to their importance, namely,

2.23

Page 80: 30510870 Cost Accounting and Financial Management

Cost Accounting their value and frequency of replenishment during a period. (i) ‘A’ Category of items consists of only a small percentage i.e., about 10% of the total

items handled by the stores but require heavy investment about 70% of inventory value, because of their high prices or heavy requirement or both.

(ii) ‘B’ Category of items are relatively less important; they may be 20% of the total items of material handled by stores. The percentage of investment required is about 20% of the total investment in inventories.

(iii) ‘C’ Category of items do not require much investment; it may be about 10% of total inventory value but they are nearly 70% of the total items handled by store.

‘A’ category of items can be controlled effectively by using a regular system which ensures neither over-stocking nor shortage of materials for production. Such a system plans its total material requirements by making budgets. The stocks of materials are controlled by fixing certain levels like maximum level, minimum level and re-order level. A reduction in inventory management costs is achieved by determining economic order quantities after taking into account ordering cost and carrying cost. To avoid shortage and to minimize heavy investment in inventories, the techniques of value analysis, variety reduction, standardisation etc., may be used.

In the case of ‘B’ category of items, as the sum involved is moderate, the same degree of control as applied in ‘A’ category of items is not warranted. The orders for the items, belonging to this category may be placed after reviewing their situation periodically.

For ‘C’ category of items, there is no need of exercising constant control. Orders for items in this group may be placed either after six months or once in a year, after ascertaining consumption requirements. In this case the objective is to economise on ordering and handling costs.

Illustration

A factory uses 4,000 varieties of inventory. In terms of inventory holding and inventory usage, the following information is compiled:

No. of varieties % % value of inventory % of inventory usage of inventory holding (average) (in end-product) 3,875 96.875 20 5 110 2.750 30 10 15 0.375 50 85 4,000 100.000 100 100

Classify the items of inventory as per ABC analysis with reasons.

2.24

Page 81: 30510870 Cost Accounting and Financial Management

Material

Solution

Classification of the items of inventory as per ABC analysis

1. 15 number of varieties of inventory items should be classified as ‘A’ category items because of the following reasons : (i) Constitute 0.375% of total number of varieties of inventory handled by stores of

factory, which is minimum as per given classification in the table.

(ii) 50% of total use value of inventory holding (average) which is maximum according to the given table.

(iii) Highest in consumption about 85% of inventory usage (in end-product). 2. 110 number of varieties of inventory items should be classified as ‘B’ category items

because of the following reasons : (i) Constitute 2.750% of total number of varieties of inventory items handled by stores

of factory.

(ii) Requires moderate investment of about 30% of total use value of inventory holding (average).

(iii) Moderate in consumption about 10% of inventory usage (in end–product). 3. 3,875 number of varieties of inventory items should be classified as ‘C’ category items

because of the following reasons: (i) Constitute 96.875% of total varieties of inventory items handled by stores of factory.

(ii) Requires about 20% of total use value of inventory holding (average).

(iii) Minimum inventory consumption i.e. about 5% of inventory usage (in end-product).

Advantages of ABC analysis : The advantages of ABC analysis are the following : (i) It ensures that, without there being any danger of interruption of production for want of

materials or stores, minimum investment will be made in inventories of stocks of materials or stocks to be carried.

(ii) The cost of placing orders, receiving goods and maintaining stocks is minimised specially if the system is coupled with the determination of proper economic order quantities.

(iii) Management time is saved since attention need be paid only to some of the items rather than all the items as would be the case if the ABC system was not in operation.

2.25

Page 82: 30510870 Cost Accounting and Financial Management

Cost Accounting (iv) With the introduction of the ABC system, much of the work connected with purchases can

be systematized on a routine basis to be handled by subordinate staff. 2.7.3 Two bin system: Under this system each bin is divided into two parts - one, smaller part, should stock the quantity equal to the minimum stock or even the re-ordering level, and the other to keep the remaining quantity. Issues are made out of the larger part; but as soon as it becomes necessary to use quantity out of the smaller part of the bin, fresh order is placed. “Two Bin System” is supplemental to the record of respective quantities on the bin card and the stores ledger card.

2.7.4 Establishment of system of budgets: To control investment in the inventories, it is necessary to know in advance about the inventories requirement during a specific period usually a year. The exact quantity of various type of inventories and the time when they would be required can be known by studying carefully production plans and production schedules. Based on this, inventories requirement budget can be prepared. Such a budget will discourage the unnecessary investment in inventories.

2.7.5 Use of perpetual inventory records and continuous stock verification − Perpetual inventory represents a system of records maintained by the stores department. It in fact comprises: (i) Bin Cards, and (ii) Stores Ledger.

Bin Card maintains a quantitative record of receipts, issues and closing balances of each item of stores. Separate bin cards are maintained for each item. Each card is filled up with the physical movement of goods i.e. on its receipt and issue.

Like bin cards, the Store Ledger is maintained to record all receipt and issue transactions in respect of materials. It is filled up with the help of goods received note and material issue requisitions.

A perpetual inventory is usually checked by a programme of continuous stock taking. Continuous stock taking means the physical checking of those records (which are maintained under perpetual inventory) with actual stock. Perpetual inventory is essential for material control. It incidentally helps continuous stock taking. The success of perpetual inventory depends upon the following:

(a) The Stores Ledger−(showing quantities and amount of each item). (b) Stock Control cards (or Bin Cards). (c) Reconciling the quantity balances shown by (a) & (b) above. (d) Checking the physical balances of a number of items every day systematically and by

rotation.

2.26

Page 83: 30510870 Cost Accounting and Financial Management

Material

(e) Explaining promptly the causes of discrepancies, if any, between physical balances and book figures.

(f) Making corrective entries where called for after step (e) and (g) Removing the causes of the discrepancies referred to in step (e) Advantages − The main advantages of perpetual inventory are as follows : (1) Physical stocks can be counted and book balances adjusted as and when desired

without waiting for the entire stock-taking to be done. (2) Quick compilation of Profit and Loss Account (for interim period) due to prompt

availability of stock figures. (3) Discrepancies are easily located and thus corrective action can be promptly taken to

avoid their recurrence. (4) A systematic review of the perpetual inventory reveals the existence of surplus, dormant,

obsolete and show-moving materials, so that remedial measures may be taken in time. (5) Fixation of the various stock levels and checking of actual balances in hand with these

levels assist the Store keeper in maintaining stocks within limits and in initiating purchase requisitions for correct quantity at the proper time.

Continuous stock verification − The checking of physical inventory is an essential feature of every sound system of material control. Such a checking may be periodical or continuous. Annual stock-taking, however, has certain inherent shortcomings which tend to detract from the usefulness of such physical verification. For instance, since all the items have to be covered in a given number of days, either the production department has to be shut down during those days to enable thorough checking of stock or else the verification must be of limited character. Moreover, in the case of periodical checking there is the problem of finding an adequately trained contingent. It is likely to be drawn from different departments where stock-taking is not the normal work and they are apt to discharge such temporary duties somewhat perfunctorily. The element of surprise, that is essential for effective control is wholly absent in the system. Then if there are stock discrepancies, they remain undetected until the end of the period. It means that the figures of stock during the period continue to be supplied incorrectly. Often, the discrepancies are not corrected.

The system of continuous stock-taking consists of counting and verifying the number of items daily throughout the year so that during the year all items of stores are covered three or four times. The stock verifiers are independent of the stores, and the stores staff have no foreknowledge as to the particular items that would be checked on any particular

2.27

Page 84: 30510870 Cost Accounting and Financial Management

Cost Accounting day. But it must be seen that each item is checked a number of times in a year.

Advantages − The advantages of continuous stock-taking are : 1. Closure of normal functioning is not necessary. 2. Whole time specialised staff can be engaged for the purpose since the work is spread

throughout the year. In smaller concerns, duties may be assigned to various officers of middle rank by rotation to the checking, say, of 20 items. This would be easy because the store ledger card and the bin card will bear the bin number. The officers concerned need only walk up to the particular bin number, count, weigh or measure the article lying there and enter the quantity on the form provided for the purpose. The rest of the work (comparison with book figures) can be done by the stores ledger clerk.

3. Stock discrepancies are likely to be brought to the notice and corrected much earlier than under the annual stock-taking system.

4. The system generally has a sobering influence on the stores staff because of the element of surprise present therein.

5. The movement of stores items can be watched more closely by the stores auditor so that chances of obsolescence buying are reduced.

6. Final Accounts can be ready quickly. Interim accounts are possible quite conveniently. 2.7.6 Economic Order Quantity (EOQ): Purchase department in manufacturing concerns is usually faced with the problem of deciding the ‘quantity of various items’ which they should purchase. If purchases of material are made in bulk then inventory carrying cost will be high. On the other hand if order size is small each time, then the ordering cost will be high. In order to minimise ordering and carrying costs it is necessary to determine the order quantity which minimises these two costs. The size of the order for which both ordering and carrying cost are minimum is known as economic order quantity.

Assumptions underlying E.O.Q.: The calculation of economic order of material to be purchased is subject to the following assumptions: (i) Ordering cost per order and carrying cost per unit per annum are known and they are

fixed. (ii) Anticipated usage of material in units is known. (iii) Cost per unit of the material is constant and is known as well. (iv) The quantity of material ordered is received immediately i.e. the lead time is zero. The famous mathematician Wilson derived the formula which is used for determining the size of order for each of purchases at minimum ordering and carrying costs.

2.28

Page 85: 30510870 Cost Accounting and Financial Management

Material

The formula given by Wilson for calculating economic order quantity is as follows :

EOQ = CAS2

where, A = Annual usage units

S = Ordering cost per order

C = Annual carrying cost of one unit, i.e., carrying cost percentage × cost of one unit.

Illustration

Calculate the Economic Order Quantity from the following information. Also state the number of orders to be placed in a year.

Consumption of materials per annum : 10,000 kg.

Order placing cost per order : Rs. 50

Cost per kg. of raw materials : Rs. 2

Storage costs: 8% on average inventory

Solution

EOQ = CSA2 ×

A = Units consumed during year S = Ordering cost per order C = Inventory carrying cost per unit per annum.

EOQ =

10082

50000,102×

×× = 4

2550000,102 ×××

= 2,500 kg.

No. of orders to be placed in a year = EOQ

annum per materials of nconsumptio Total

2.29

Page 86: 30510870 Cost Accounting and Financial Management

Cost Accounting

= kg. 2,500kg. 10,000 = 4 Orders per year

Illustration

X Ltd. is committed to supply 24,000 bearings per annum to Y Ltd. on steady basis. It is estimated that it costs 10 paise as inventory holding cost per bearing per month and that the set-up cost per run of bearing manufacture is Rs. 324. (a) What would be the optimum run size for bearing manufacture ? (b) Assuming that the company has a policy of manufacturing 6,000 bearings per run, how

much extra costs the company would be incurring as compared to the optimum run suggested in (a) above ?

(c) What is the minimum inventory holding cost ?

Solution

(a) Optimum production run size (Q) = I

UP2

where, U = No. of units to be produced within one year. P = Set-up cost per production run I = Carrying cost per unit per annum.

= I

UP2=

1210.0324.Rs000,242

×××

= 3,600 bearings. (b) Total Cost (of maintaining the inventories) when production run size (Q) are 3,600 and

6,000 bearings respectively Total cost = Total set-up cost + Total carrying cost. (Total set-up cost) Q = 3,600 = (No. of production runs ordered) × (Set-up cost per

production run)

= 600,3000,24 × Rs. 324

= Rs. 2,160 ...(1)

2.30

Page 87: 30510870 Cost Accounting and Financial Management

Material

(Total set–up cost) Q = 6,000 = ×000,6000,24 Rs. 324

= 1,296 ...(2) (Total carrying cost) Q = 3,600 = 1/2 Q × I = 1/2 × 3,600 × 0.10P × Rs. 12 = Rs. 2,160 ...(3) (Total carrying cost) Q = 6,000 = 1/2 × 6,000 × 0.10P × Rs. 12 = Rs. 3,600 ...(4) (Total Cost) Q = 3,600 = Rs. 2,160 + Rs. 2,160 [ (1) + (3)] = Rs. 4,320 ...(5)

(Total Cost) Q = 6,000 = Rs. 1,296 + Rs. 3,600 [(2) + (4)] = Rs. 4,896 ...(6) Extra cost incurred = Rs. 4,896 – Rs. 4,320 [(6) – (5)] = Rs. 576 (c) Minimum inventory holding cost = 1/2 Q × I (when Q = 3,600 bearings) = 1/2 × 3,600 bearings × 0.10P × Rs. 12 = Rs. 2,160

Illustration

Shriram enterprise manufactures a special product “ZED”. The following particulars were collected for the year 2006:

(a) Monthly demand of ZED − 1,000 units (b) Cost of placing an order Rs. 100. (c) Annual carrying cost per unit Rs. 15. (d) Normal usage 50 units per week. (e) Minimum usage 25 units per week. (f) Maximum usage 75 units per week. (g) Re-order period 4 to 6 weeks. Compute from the above

2.31

Page 88: 30510870 Cost Accounting and Financial Management

Cost Accounting (1) Re-order quantity (2) Re-order level (3) Minimum level (4) Maximum level (5) Average stock level.

Solution

1. Re-order quantity of units used = CAS2 =

15.Rs100.Rs600,22 ××

= 186 units (approximately) (Refer to note)

where, A = Annual demand of input units S = Cost of placing an order C = Annual carrying cost per unit 2. Re-order level = Maximum re-order period × maximum usage

= 6 weeks × 75 units = 450 units 3. Minimum Level = Re-order level – (normal usage × average re-order period). = 450 units – 50 units × 5 weeks. = 450 units – 250 units = 200 units. 4. Maximum Level = Re-order level + Re-order quantity – Minimum usage × Minimum order period. = 450 units + 186 units – 25 units × 4 weeks = 536 units 5. Average Stock Level = 1/2 (Minimum stock level + Maximum stock level) = 1/2 (200 units + 536 units) = 368 units. Note: A = Annual demand of input units for 12,000 units of ‘ZED’ = 52 weeks × Normal usage of input units per week

2.32

Page 89: 30510870 Cost Accounting and Financial Management

Material

= 52 weeks × 50 units of input per week = 2,600 units.

Illustration (a) EXE Limited has received an offer of quantity discounts on its order of materials as

under: Price per tonne Tonnes Rs. Nos. 1,200 Less than 500 1,180 500 and less than 1,000 1,160 1,000 and less than 2,000 1,140 2,000 and less than 3,000 1,120 3,000 and above. The annual requirement for the material is 5,000 tonnes. The ordering cost per order is Rs. 1,200 and the stock holding cost is estimated at 20% of material cost per annum. You are required to compute the most economical purchase level. (b) What will be your answer to the above question if there are no discounts offered and the

price per tonne is Rs. 1,500 ?

Solution (a)

Total annual requirement

(S)

Ordersize

(Units)

(q)

No. of orders

S/q

Cost of inventory S×Per unit cost

(Rs.)

Ordering cost

S/q×Rs. 1200

(Rs.)

Carrying cost

p.u.p.a 1/2×q×20% of

cost p.u.

(Rs.)

Total Cost (4+5+6)

(Rs.)

1 2 3 4 5 6 7

500 units 400 12.5 60,00,000 15,000 48,000 60,63,000 (5,000×Rs. 1200) (200 × Rs. 240) 500 10 59,00,000 12,000 59,000 59,71000 (5,000×Rs. 1180) (250 × Rs. 236) 1000 5 58,00,000 6,000 1,16,000 59,22,000

2.33

Page 90: 30510870 Cost Accounting and Financial Management

Cost Accounting

(5,000×Rs. 1160) (500 × Rs. 232) 2000 2.5 57,00,000 3,000 2,28,000 59,31,000 (5,000×Rs. 1140) (1,000×Rs. 228) 3000 1666 56,00,000 2,000 3,36,000 59,38,000 (5,000×Rs. 1120) (1,500×Rs. 224)

The above table shows that the total cost of 5,000 units including ordering and carrying cost is minimum (Rs. 59,22,000) when the order size is 1,000 units. Hence the most economical purchase level is 1,000 units.

(b) EOQ = 1

O

iCSC2

where S is the annual inventory requirement, Co, is the ordering cost per order and iC1 is the carrying cost per unit per annum.

= 500,1.Rs%20

200,1.Rsunits500,22×

××

Illustration

From the details given below, calculate: (i) Re-ordering level (ii) Maximum level (iii) Minimum level (iv) Danger level. Recording quantity is to be calculated on the basis of following information:

Cost of placing a purchase order is Rs. 20

Number of units to be purchased during the year is 5,000

Purchase price per unit inclusive of transportation cost is Rs. 50

Annual cost of storage per units is Rs. 5.

2.34

Page 91: 30510870 Cost Accounting and Financial Management

Material

Details of lead time : Average 10 days, Maximum 15 days, Minimum 6 days.

For emergency purchases 4 days.

Rate of consumption : Average : 15 units per day, Maximum : 20 units per day.

Solution

BASIC DATA :

Co (Ordering cost per order) = Rs. 20

S (Number of units to be purchased annually) = 5,000 units C1 (Purchase price per unit inclusive of transportation cost) = Rs. 50.

iC1 (Annual cost of storage per unit) = Rs. 5

Computations : (i) Re-ordering level = Maximum usage per period × Maximum re-order period (ROL) = 20 units per day × 15 days = 300 units (ii) Maximum level = ROL + ROQ – [Min. rate of consumption × Min. re-order

period] (Refer to working notes1 and 2) = 300 units + 200 units – [10 units per day × 6 days] = 440 units (iii) Minimum level = ROL – Average rate of consumption × Average re-order-

period = 300 units – (15 units per day × 10 days) = 150 units (iv) Danger level = Average consumption × Lead time for emergency

purchases = 15 units per day × 4 days = 60 units Working Notes :

1. ROQ = = 1

0

iCSC2 =

500,1.Rs%20200,1.Rsunits000,52

××× = 200 units

2.35

Page 92: 30510870 Cost Accounting and Financial Management

Cost Accounting

2. 2nconsumptioof rate Maximum

+nconsumptioof rate Minimum

=nconsumptioof rate Av.

15 units per day = 2

day per 20unitsXunits/day+

or X = 10 units per day.

Illustration

About 50 items are required every day for a machine. A fixed cost of Rs. 50 per order is incurred for placing an order. The Inventory carrying cost per item amounts to Rs. 0.02 per day. The lead period is 32 days. Compute : (i) Economic order quantity. (ii) Re-order level.

Solution Annual consumption (S) = 50 items × 365 days

= 18,250 items Fixed cost per order (Co) or Ordering cost = Rs. 50 Inventory carrying cost per item per annum (iC1) = Rs. 0.02 × 365 = Rs. 7.30

(i) Economic order quantity =1

0

iCSC2 =

730.Rs50.Rsunits 250,182 ×× = 500 units

(ii) Re-order level = Maximum usage per day × Maximum lead time = 50 items per day × 32 days = 1,600 items

Illustration

M/s. Tubes Ltd. are the manufacturers of picture tubes for T.V. The following are the details of their operation during 2006:

Average monthly market demand 2,000 Tubes Ordering cost Rs. 100 per order

2.36

Page 93: 30510870 Cost Accounting and Financial Management

Material

Inventory carrying cost 20% per annum Cost of tubes Rs. 500 per tube Normal usage 100 tubes per week Minimum usage 50 tubes per week Maximum usage 200 tubes per week Lead time to supply 6 − 8 weeks Compute from the above:

(1) Economic order quantity. If the supplier is willing to supply quarterly 1,500 units at a discount of 5%, is it worth accepting?

(2) Maximum level of stock. (3) Minimum level of stock. (4) Re-order level

Solution S = Annual usage of tubes = Normal usage per week × 52 weeks = 100 tubes × 52 weeks = 5,200 tubes. Co = Ordering cost per order = Rs. 100/- per order

C1 = Cost per tube = Rs. 500/-

iC1 = Inventory carrying cost per unit per annum

= 20% × Rs. 500 = Rs. 100/- per unit, per annum (1) Economic order quantity

E.O.Q = 1

0

iCSC2 =

00,1.Rs00,1.Rsunits200,52 ×× = 102 tubes (approx.)

If the supplier is willing to supply 1500 units at a discount of 5% is it worth accepting?

Total cost (when order size is 1,500 units) = Cost of 5,200 units + Ordering cost + Carrying cost

= 5,200 units × Rs. 475 + units 1,500units 5,200

× Rs. 100 + + 1,500 units × 20% × Rs. 475

2.37

Page 94: 30510870 Cost Accounting and Financial Management

Cost Accounting

= Rs. 24,70,000 + Rs. 346.67 + Rs. 71,250 = Rs. 25,41,596.67 Total cost (when order size is 102 units)

= 5,200 units × Rs. 500 + units 102units 5,200

× Rs. 100 + × 102 units × 20% × Rs. 500

= Rs. 26,00,000 + Rs. 5,098.03 + Rs. 5,100 = Rs. 26,10,198.03 Since, the total cost under quarterly supply of 1,500 units with 5% discount is lower than that when order size is 102 units, therefore the offer should be accepted. While accepting this offer consideration of capital blocked on order size of 1,500 units per quarter has been ignored.

(2) Maximum level of stock = Re-order level + Re-order quantity – Min. usage × Min. re-order period = 1,600 units + 102 units – 50 units × 6 weeks = 1,402 units.

(3) Minimum level of stock = Re-order level – Normal usage × Average reorder period = 1,600 units – 100 units × 7 weeks = 900 units.

(4) Reorder level = Maximum consumption × Maximum re-order period = 200 units × 8 weeks = 1,600 units.

Illustration

The complete Gardener is deciding on the economic order quantity for two brands of lawn fertilizer: Super Grow and Nature’s Own. The following information is collected:

Fertilizer

Super Grow Nature’s Own Annual Demand 2,000 Bags 1,280 Bags Relevant ordering cost per purchase order Rs. 1,200 Rs. 1,400 Annual relevant carrying cost per bag Rs. 480 Rs. 560

2.38

Page 95: 30510870 Cost Accounting and Financial Management

Material

Required : (i) Compute EOQ for Super Grow and Nature’s Own. (ii) For the EOQ, what is the sum of the total annual relevant ordering costs and total annual

relevant carrying costs for Super Grow and Nature’s Own ? (iii) For the EOQ Compute the number of deliveries per year for Super Grow and Nature’s

Own.

Solution

(i) EOQ = 1

0

iC*SC2

* Here S = Annual demand of fertilizer bags. C1 = Cost per bag.

C0 = Relevant ordering cost per purchase order

iC1 = Annual relevant carrying cost per bag

EOQ for Super Grow Fertilizer EOQ for Nature’s Own Fertilizer

480.Rs

200,1.Rsbags 000,22 ×× = 100 bags 560.Rs

400,1.Rsbags 280,12 ×× = 80 bags

(ii) Total annual relevant costs for Super Grow Fertilizer = Total annual relevant ordering costs + Total annual relevant carrying costs

= EOQ

S × C0 + 21 EOQ × iC1

= bags 100bags 2000

× Rs. 1,200 + 21 × 100 bags × Rs. 480

= Rs. 24,000 + Rs. 24,000 = Rs. 48,000 Total annual relevant costs for Nature’s Own Fertilizer

= bags 80

bags 280,1 × Rs. 1,400 + 21 × 80 bags × Rs. 560

= Rs. 22,400 + Rs. 22,400 = Rs. 44,800

2.39

Page 96: 30510870 Cost Accounting and Financial Management

Cost Accounting (iii) Number of deliveries for Super Grow Fertilizer per year.

EOQ

S = (annual demand of fertiliser bags) = bags 100bags 2000

= 20 orders

Number of deliveries for Nature’s Own fertilizers per year.

= bags 80

bags 1,280 = 16 orders

Illustration

G. Ltd. produces a product which has a monthly demand of 4,000 units. The product requires a component X which is purchased at Rs. 20. For every finished product, one unit of component is required. The ordering cost is Rs. 120 per order and the holding cost is 10% p.a.

You are required to calculate: (i) Economic order quantity. (ii) If the minimum lot size to be supplied is 4,000 units, what is the extra cost, the company

has to incur ? (iii) What is the minimum carrying cost, the company has to incur ?

Solution (a) (i) Economic order quantity : S (Annual requirement or Component ‘X’) = 4,000 units per month × 12 months = 48,000 units C1 (Purchase cost p.u.) = Rs. 20

C0 (Ordering cost per order) = Rs. 120

i (Holding cost) = 10% per annum

E.O.Q. = EOQ

S = Rs.20×10%Rs.120×units 48,000×2

= 2,400 units

(ii) Extra cost incurred by the company Total cost = Total ordering cost + Total carrying cost (when order size is 4,000 units)

2.40

Page 97: 30510870 Cost Accounting and Financial Management

Material

= QS × C0 + q (iC1)

=units 000,4units 000,48 × Rs. 120 +

21 × 4,000 units × 10% × Rs. 20

= Rs. 1,440 + Rs. 4,000 = Rs. 5,440 ...(a)

Total cost = units 400,2units 000,48 × Rs. 120 +

21 × 2,400 units × 10% × Rs. 20

(when order size is 2,400 units) = Rs. 2,400 + Rs. 2,400 = Rs. 4,800 ...(b) Extra cost : (a) – (b) = Rs. 5,440 – Rs. 4,800 = Rs. 640 (incurred by the company) (iii) Minimum carrying cost : Carrying cost depends upon the size of the order. It will be minimum on the least

order size. (In this part of the question the two order sizes are 2,400 units and 4,000 units. Here 2,400 units is the least of the two order sizes. At this order size carrying cost will be minimum.)

The minimum carrying cost in this case can be computed as under :

Minimum carrying cost = 21 × 2,400 units × 10% × Rs. 20 = Rs. 2,400.

Illustration

A Ltd. is committed to supply 24,000 bearings per annum to B Ltd. on a steady basis. It is estimated that it costs 10 paise as inventory holding cost per bearing per month and that the set-up cost per run of bearing manufacture is Rs. 324. (i) What should be the optimum run size for bearing manufacture ? (ii) What would be the interval between two consecutive optimum runs ? (iii) Find out the minimum inventory cost per annum.

2.41

Page 98: 30510870 Cost Accounting and Financial Management

Cost Accounting

Solution (i) Optimum run size for bearing manufacture

= bearing percost holding Annualrun production percost up-Set×bearings ofsupply Annual×2

=0.10P. months 12

324.Rsbearings 000,242×

×× = 3,600 bearings

(ii) Interval between two consecutive optimum runs

= annum per runs nproductioo of Number

months 12 =

⎟⎟⎠

⎞⎜⎜⎝

⎛size run Optimum

production Annualmonths 12

=

⎟⎟⎠

⎞⎜⎜⎝

⎛bearings 3,600bearings 24,000

months 12 = 66.6

months 12 = 1.8 months or 55 days approximately

(iii) Minimum inventory cost per annum = Total production run cost + Total carrying cost per annum

= bearings 3,600bearings 000,24 × Rs. 324 +

21 × 3,600 bearings × 0.10 P × 12 months

= Rs. 2,160 + Rs. 2,160 = Rs. 4,320

2.7.7 Review of slow and non-moving items : Sometimes, due to high value of slow moving and non-moving raw materials, it appears that the concern has blocked huge sum of money unnecessarily in raw materials. To overcome this problem, it is necessary to dispose-off as early as possible, the non-moving items or make arrangements for their exchange with the inventories required by the concern. Besides this no new requisition should be made for the purchase of slow moving items, till the existing stock is exhausted. Computation of inventory turnover ratio may help in identifying slow moving items.

2.7.8 Use of control ratios : (i) Input output ratio : Inventory control can also be exercised by the use of input output ratio analysis. Input-output ratio is the ratio of the quantity of input of material to production and the standard material content of the actual output.

2.42

Page 99: 30510870 Cost Accounting and Financial Management

Material

This type of ratio analysis enables comparison of actual consumption and standard consumption, thus indicating whether the usage of material is favourable or adverse.

(ii) Inventory turnover ratio : Computation of inventory turnover ratios for different items of material and comparison of the turnover rates, provides a useful guidance for measuring inventory performance. High inventory turnover ratio indicates that the material in the question is a fast moving one. A low turnover ratio indicates over-investment and locking up of the working capital in inventories. Inventory turnover ratio may be calculated by using the following formulae:-

Inventory turnover ratio = period the during held stock average ofCost period the durjing consumed materials ofCost

Average stock = 1/2 (opening stock + closing stock) By comparing the number of days in the case of two different materials, it is possible to know which is fast moving and which is slow moving. On this basis, attempt should be made to reduce the amount of capital locked up, and prevent over-stocking of the slow moving items.

Illustration The following data are available in respect of material X for the year ended 31st March, 2006.

Rs. Opening stock 90,000 Purchases during the year 2,70,000 Closing stock 1,10,000

Calculate : (i) Inventory turnover ratio, and (ii) The number of days for which the average inventory is held.

Solution Inventory turnover ratio

(Refer to working note) = material raw of stock Average

consumed material raw of stock ofCost

2.43

Page 100: 30510870 Cost Accounting and Financial Management

Cost Accounting

= 000,00,1.Rs000,50,2.Rs

= 2.5 Average number of days for which

the average inventory is held = ratio turnoverInventory

365 = 5.2

days365

= 146 Working Note :

Rs. Opening stock of raw material 90,000 Add: Material purchases during the year 2,70,000 Less: Closing stock of raw material 1,10,000Cost of stock of raw material consumed 2,50,000

2.8 VALUATION OF MATERIAL RECEIPTS

The invoice of material purchased from the market sometime contain items such as trade discount, quantity discount, freight, duty, insurance, cost of containers, sales tax, excise duty, cash discount etc. Under such a situation, the general principal is that all costs incurred upto the point of procuring and storing materials should constitute the cost of materials purchased. The amount of trade discount, quantity discount and excise duty (under MODVAT credit scheme) being credit items and are thus deducted from the invoice of material purchased. The transport charges (carriage and freight), sales tax, insurance, cost of containers, customs and excise duty (without MODVAT credit) should be included in the invoice cost of material. The cash discount is considered as financial gain, so it is kept outside the domain of material cost. In case the containers are returnable, their resale value should also be taken in the invoice price of material to correctly ascertain the cost of material purchased. The cost of material purchased so determined may be used for the entry of material in the Stores Ledger.

2.44

Page 101: 30510870 Cost Accounting and Financial Management

Material

Illustration

An invoice in respect of a consignment of chemicals A and B provides the following information:

Rs. Chemical A: 10,000 lbs. at Rs. 10 per lb. 1,00,000 Chemical B: 8,000 lbs. at Rs. 13 per lb. 1,04,000 Sales tax @ 10% 20,400 Railway freight 3,840Total cost 2,28,240

A shortage of 500 lbs. in chemical A and 320 lbs. in chemical B is noticed due to normal breakages. You are required to determine the rate per lb. of each chemical, assuming a provision of 2% for further deterioration.

Solution

Statement showing computation of effective quantity of each chemical available for use Chemical A Chemical B lbs. lbs. Quantity purchased 10,000 8,000 Less : Shortage due to normal breakages 500 320 9,500 7,680 Less : Provision for deterioration 2% 190 53.6 Quantity available 9,310 7,526.4

Statement showing the computation of rate per lb. of each chemical Chemical A Chemical B

Rs. Rs.

Purchase price 1,00,000 1,04,000 Add : Sales tax (10%) 10,000 10,400

2.45

Page 102: 30510870 Cost Accounting and Financial Management

Cost Accounting Railway freight (in the ratio of quantity purchased i.e., 5:4) 2,133 1,707Total cost 1,12,133 1,16,107

Rate per 1b. A : bs3101,9133,12,1.Rs = Rs. 12.04

Rate per lb. B : bs341.526,7

107,16,1.Rs = Rs. 15.43

Illustration

At what price per unit would Part No. A 32 be entered in the Stores Ledger, if the following invoice was received from a supplier: Invoice Rs. 200 units Part No. A 32 @ Rs. 5 1,000.00 Less : 20% discount 200.00 800.00 Add : Excise duty @ 15% 120.00 920.00 Add : Packing charges (5 non-returnable boxes) 50.00 970.00 Notes: (i) A 2 per cent discount will be given for payment in 30 days. (ii) Documents substantiating payment of excise duty is enclosed for claiming MODVAT

credit.

Solution 200 units net cost after trade discount Rs. 800 Add : Packing charges Rs. 50 Total cost per 200 units Rs. 850

Cost per unit = 200

850.Rs = Rs. 4.25

2.46

Page 103: 30510870 Cost Accounting and Financial Management

Material

2.9 VALUATION OF MATERIAL ISSUES

Materials issued from stores should be priced at the value at which they are carried in stock. But the value attached to the stock of any item of material, at any particular point of time may be the result, not of one purchase rate or price but of different purchase rate or prices. In other words, the same material may have been acquired at different prices and its stock at any particular time may comprise materials of more than one lot so that there would be a problem of determining the appropriate rate at which to price out the issues of materials.

Several methods of pricing material issues have been evolved in an attempt to satisfactorily answer the problem. These methods may be grouped and explained as follows :

2.9.1 Cost Price Methods:

(a) Specific price method.

(b) First-in First-out method.

(c) Last-in-First-out method.

(d) Base stock method.

2.9.2 Average Price Methods :

(e) Simple average price method.

(f) Weighted average price method.

(g) Periodic simple average price method.

(h) Periodic weighted average price method.

(i) Moving simple average price method.

(j) Moving weighted average price method.

2.9.3 Market Price Methods :

(k) Replacement price method.

(l) Realisable price method.

2.9.4 Notional Price Methods :

(m) Standard price method.

2.47

Page 104: 30510870 Cost Accounting and Financial Management

Cost Accounting

(n) Inflated price methods.

(o) Re-use Price Method.

We may now briefly discuss all the above methods:

(a) Specific Price Method - This method is useful, specially when materials are purchased for a specific job or work order, and as such these materials are issued subsequently to that specific job or work order at the price at which they were purchased. To use this method, it is necessary to store each lot of material separately and maintain its separate account. The advantages and disadvantage of this method are :

Advantages :

1. The cost of materials issued for production purposes to specific jobs represent actual and correct costs.

2. This method is best suited for non-standard and specific products.

Disadvantage : This method is difficult to operate, specially when purchases and issues are numerous.

(b) First-in-First out Method (FIFO) - It is a method of pricing the issues of materials, in the order in which they are purchased. In other words, the materials are issued in the order in which they arrive in the store or the items longest in stock are issued first. Thus each issue of material only recovers the purchase price which does not reflect the current market price. This method is considered suitable in times of falling price because the material cost charged to production will be high while the replacement cost of materials will be low. But, in the case of rising prices, if this method is adopted, the charge to production will be low as compared to the replacement cost of materials. Consequently, it would be difficult to purchase the same volume of material (as in the current period) in future without having additional capital resources. The advantages and disadvantages of the method may be stated as follows :

Advantages :

1. It is simple to understand and easy to operate. 2. Material cost charged to production represents actual cost with which the cost of

production should have been charged. 3. In the case of falling prices, the use of this method gives better results.

4. Closing stock of material will be represented very closely at current market price.

2.48

Page 105: 30510870 Cost Accounting and Financial Management

Material

Disadvantages :

1. If the prices fluctuate frequently, this method may lead to clerical error.

2. Since each issue of material to production is related to a specific purchase price, the costs charged to the same job are likely to show a variation from period to period.

3. In the case of rising prices, the real profits of the concern being low, they may be inadequate to meet the concern’s demand to purchase raw materials at the ruling price.

The application of FIFO method is illustrated below :

Material Received and Issued Lot Date Quantity Lot Rate Amount

No. Kg. No. 1. July 3 600 1.00 600.00 2. July 13 800 1.20 960.00 3. July 23 600 0.90 540.00 4. August 5 400 1.10 440.00

5. August 6 1200 0.80 960.00 July 8 400 Kgs. out of (1) 1.00 400.00 July 12 200 Kgs. out of (1) 1.00 200.00 July 22 600 Kgs. out of (2) 1.20 720.00 July 25 200 Kgs. out of (2) 1.20 240.00 200 Kgs. out of (3) 0.90 180.00 August 8 400 Kgs. out of (3) 0.90 360.00 400 Kgs. out of (4) 1.10 440.00 200 Kgs. out of (5) 0.80 160.00 The stock in hand after 8th August will be 1,000 Kgs. This will be out of lot number (5) and its value will be Rs. 800, i.e., @ Re. 0.80 per Kg.

(c) Last-in-First out method (LIFO) - It is a method of pricing the issues of materials. This method is based on the assumption that the items of the last batch (lot) purchased are the first to be issued. Therefore, under this method the prices of the last batch (lot) is used for pricing the issues, until it is exhausted, and so on. If however, the quantity of

2.49

Page 106: 30510870 Cost Accounting and Financial Management

Cost Accounting issue is more than the quantity of the latest lot than earlier (lot) and its price will also be taken into consideration. During inflationary period or period of rising prices, the use of LIFO would help to ensure that the cost of production determined on the above basis is approximately the current one. This method is also useful specially when there is a feeling that due to the use of FIFO or average methods, the profits shown and tax paid are too high.

The advantages and disadvantages of LIFO method are as follows : Advantages : 1. The cost of materials issued will be either nearer to and or will reflect the current market price. Thus, the cost of goods produced will be related to the trend of the market price of materials. Such a trend in price of materials enables the matching of cost of production with current sales revenues. 2. The use of the method during the period of rising prices does not reflect undue high profit in the income statement as it was under the first-in-first-out or average method. In fact, the profit shown here is relatively lower because the cost of production takes into account the rising trend of material prices. 3. In the case of falling prices profit tends to rise due to lower material cost, yet the finished products appear to be more competitive and are at market price. 4. Over a period, the use of LIFO helps to iron out the fluctuations in profits. 5. In the period of inflation LIFO will tend to show the correct profit and thus avoid paying undue taxes to some extent. Disadvantages :

1. Calculation under LIFO system becomes complicated and cumbersome when frequent purchases are made at highly fluctuating rates. 2. Costs of different similar batches of production carried on at the same time may differ a great deal.

3. In time of falling prices, there will be need for writing off stock value considerably to stick to the principle of stock valuation, i.e., the cost or the market price whichever is lower.

4. This method of valuation of material is not acceptable to the income tax authorities.

(d) Base Stock Method - A minimum quantity of stock under this method is always held at a fixed price as reserve in the stock, to meet a state of emergency, if it arises. This minimum stock is known as base stock and is valued at a price at which the first lot of

2.50

Page 107: 30510870 Cost Accounting and Financial Management

Material

materials is received and remains unaffected by subsequent price fluctuations. Thus, this is more a method of valuing inventory than a method of valuing issues because, with the base of stock valued at the original cost some other method of valuing issues should be adopted. The quantity in excess of the base stock may be valued either on the FIFO or LIFO basis. This method is not an independent method as it uses FIFO or LIFO. Its advantages and disadvantages therefore will depend upon the use of the other method viz., FIFO or LIFO. Illustration :

‘AT’ Ltd. furnishes the following store transactions for September, 2006 : 1-9-06 Opening balance 25 units value Rs. 162.50 4-9-06 Issues Req. No. 85 8 units 6-9-06 Receipts from B & Co. GRN No. 26 50 units @ Rs. 5.75 per unit

7-9-06 Issues Req. No. 97 12 units 10-9-06 Return to B & Co. 10 units 12-9-06 Issues Req. No. 108 15 units 13-9-06 Issues Req. No. 110 20 units 15-9-06 Receipts from M & Co. GRN. No. 33 25 units @ Rs. 6.10 per unit 17-9-06 Issues Req. No. 12 10 units 19-9-06 Received replacement from B & Co. GRN No. 38 10 units 20-9-06 Returned from department, material of M & Co. MRR No. 4 5 units 22-9-06 Transfer from Job 182 to Job 187 in the dept. MTR 6 5 units 26-9-06 Issues Req. No. 146 10 units 29-9-06 Transfer from Dept. “A” to Dept. “B” MTR 10 5 units 30-9-06 Shortage in stock taking 2 units

Write up the priced stores ledger on FIFO method and discuss how would you treat the shortage in stock taking.

2.51

Page 108: 30510870 Cost Accounting and Financial Management

Solution : Stores Ledger of AT Ltd. for the month of September, 2006 (FIFO Method)

RECEIPT ISSUE BALANCE Date GRN No. Qty. Rate Amount Requisi- Qty. Rate Amount Qty. Rate Amount MRR No. Units Rs. P Rs. P tion No. Units Rs. P. Rs. P. Units Rs. P. Rs. P. 1 2 3 4 5 6 7 8 9 10 11 12 1-9-06 — — — — — — — — 25 6.50 162.50 4-9-06 — — — — 85 8 6.50 52 17 6.50 110.50 6-9-06 26 50 5.75 287.50 — — — — 17 6.50 50} 5.75} 398.00 7-9-06 — — — — 97 12 6.50 78 5 6.50 50} 5.75 320.00 10-9-06 — — — — Nil 10 5.75 57.50 5 6.50 40} 5.75 262.00 12-9-06 — — — — 108 5 6.50 10} 5.75} 90 30 5.75 172.50 13-9-06 — — — — 110 20 5.75 115 10 5.75 57.50 15-9-06 33 25 6.10 152.50 — — — — 10 5.75 25} 6.10 210.00 17-9-06 — — — — 121 10 5.75 57.50 25 6.10 152.50 19-9-06 38 10 5.75 57.50 — — — — 25 6.10 10} 5.75} 210.00 5 5.75 20-9-06 4 5 5.75 28.75 — — — — 25 6.10 258.75 10} 7.75} 26-9-06 — — — — 146 5 5.75 20 6.10 5} 6.10} 59.29 10} 5.75} 179.50 30-9-06 — — — — Shortage 2 6.10 12.20 18 6.10 10} 5.75} 167.30

Page 109: 30510870 Cost Accounting and Financial Management

Material

Working Notes : 1. The material received as replacement from vendor is treated as fresh supply.

2. In the absence of information the price of the material received from within on 20-9-06 has been taken as the price of the earlier issue made on 17-9-06. In FIFO method physical flow of the material is irrelevant for pricing the issues.

3. The issue of material on 26-9-06 is made out of the material received from within.

4. The entries for transfer of material from one job and department to other on 22-9-06 and 29-9-06 are book entries for adjusting the cost of respective jobs and as such they have not been shown in the stores ledger account.

5. The material found short as a result of stock taking has been written off. Illustration : The following information is provided by SUNRISE INDUSTRIES for the fortnight of April, 2006 : Material Exe : Stock on 1-4-2006 100 units at Rs. 5 per unit.

Purchases 5-4-06 300 units at Rs. 6 8-4-06 500 units at Rs. 7 12-4-06 600 units at Rs. 8 Issues 6-4-06 250 units 10-4-06 400 units 14-4-06 500 units

Required : (A) Calculate using FIFO and LIFO methods of pricing issues : (a) the value of materials consumed during the period (b) the value of stock of materials on 15-4-06. (B) Explain why the figures in (a) and (b) in part A of this question are different under the two methods of pricing of material issues used. You need not draw up the Stores Ledgers.

2.53

Page 110: 30510870 Cost Accounting and Financial Management

Cost Accounting Solution (A) (a) Value of Material Exe consumed during the period 1-4-06 to 15-4-06 by using FIFO method.

Date Description Units Qty. Rs.

Rate Rs.

Amount

1-4-06 Opening balance 100 5 500 5-4-06 Purchased 300 6 1,800 6-4-06 Issued 100

150 ⎭⎬⎫

65

1,400

8-4-06 Purchased 500 7 3,500 10-4-06 Issued 150

250 ⎭⎬⎫

76

2,650 12-4-06 Purchased 600 8 4,800 14-4-06 Issued 250

250 ⎭⎬⎫

87

3,750

15-4-06 Balance 350 8 2,800

Total value of material Exe consumed during the period under FIFO method comes to (Rs. 1,400 + Rs. 2,650 + Rs. 3,750) Rs. 7,800 and balance on 15-4-06 is of Rs. 2,800.

Value of Material Exe consumed during the period 1-4-06 to 15-4-06 by using LIFO method

Date Description Qty. Rate Amount Units Rs. Rs. 1-4-06 Opening balance 100 5 500 5-4-06 Purchased 300 6 1,800 6-4-06 Issued 250 6 1,500 8-4-06 Purchased 500 7 3,500 10-4-06 Issued 400 7 2,800 12-4-06 Purchased 600 8 4,800 14-4-06 Issued 500 8 4,000 15-4-06 Balance 350 — 2,300*

2.54

Page 111: 30510870 Cost Accounting and Financial Management

Material

Total value of material Exe issued under LIFO method comes to (Rs. 1,500 + Rs. 2,800 + Rs. 4,000) Rs. 8,300.

*The balance 350 units on 15-4-06 of Rs. 2,300, relates to opening balance on 1-4-06 and purchases made on 5-4-06, 8-4-06 and 12-4-06. (100 units @ Rs. 5, 50 units @ Rs. 6, 100 units @ Rs. 7 and 100 units @ Rs. 8).

(b) As shown in (a) above, the value of stock of materials on 15-4-06:

Under FIFO method Rs. 2,800

Under LIFO method Rs. 2,300 (B) Total value of material Exe issued to production under FIFO and LIFO methods comes to

Rs. 7,800 and Rs. 8,300 respectively. The value of closing stock of material Exe on 15-4-06 under FIFO and LIFO methods comes to Rs. 2,800 and Rs. 2,300 respectively. The reasons for the difference of Rs. 500 (Rs. 8,300 – Rs. 7,800) as shown by the following table in the value of material Exe, issued to production under FIFO and LIFO are as follows :

Date Quantity Value Total Value Total Issued FIFO LIFO (Units) Rs. Rs. Rs. Rs. 6-4-06 250 1,400 1,500 10-4-06 400 2,650 2,800 14-4-06 500 3,750 7,800 4,000 8,300

1. On 6-4-06, 250 units were issued to production. Under FIFO their value comes to Rs. 1,400 (100 units × Rs. 5 + 150 units × Rs. 6) and under LIFO Rs. 1,500 (250 × Rs. 6). Hence, Rs. 100 was more charged to production under LIFO.

2. On 10-4-06, 400 units were issued to production. Under FIFO their value comes to Rs. 2,650 (150 × Rs. 6 + 250 × Rs. 7) and under LIFO Rs. 2,800 (400 × Rs. 7). Hence, Rs. 150 was more charged to production under LIFO.

3. On 14-4-06, 500 units were issued to production. Under FIFO their value comes to Rs. 3,750 (250 × Rs. 7 + 250 × Rs. 8) and under LIFO Rs. 4,000 (500 × Rs. 8). Hence, Rs. 250 was more charged to production under LIFO.

Thus the total excess amount charged to production under LIFO comes to Rs. 500.

2.55

Page 112: 30510870 Cost Accounting and Financial Management

Cost Accounting

The reasons for the difference of Rs. 500 (Rs. 2,800 – Rs. 2,300) in the value of 350 units of Closing Stock of material Exe under FIFO and LIFO are as follows : 1. In the case of FIFO, all the 350 units of the closing stock belongs to the purchase of

material made on 12-4-06, whereas under LIFO these units were from opening balance and purchases made on 5-4-06, 8-4-06 and 12-4-06.

2. Due to different purchase price paid by the concern on different days of purchase, the value of closing stock differed under FIFO and LIFO. Under FIFO 350 units of closing stock were valued @ Rs. 8 p.u. Whereas under LIFO first 100 units were valued @ Rs. 5 p.u., next 50 units @ Rs. 6 p.u., next 100 units @ Rs. 7 p.u. and last 100 units @ Rs. 8 p.u.

Thus under FIFO, the value of closing stock increased by Rs. 500.

Illustration

The following transactions in respect of material Y occurred during the six months ended 30th June, 2006:

Month Purchase (units) Price per unit Issued Rs. units January 200 25 Nil February 300 24 250 March 425 26 300 April 475 23 550 May 500 25 800 June 600 20 400

Required : (a) The Chief Accountant argues that the value of closing stock remains the same no matter

which method of pricing of material issues is used. Do you agree? Why or why not? Detailed stores ledgers are not required.

(b) When and why would you recommend the LIFO method of pricing material issues ?

Solution (a) The Closing Stock at the end of six months period i.e., on 30th June, 2006 will be 200 units, whereas up to the end of May 2006, total purchases coincide with the total

2.56

Page 113: 30510870 Cost Accounting and Financial Management

Material

issues i.e., 2,300 units. It means that at the end of May 2006, there was no closing stock. In the month of June 2006, 600 units were purchased out of which 400 units were issued. Since there was only one purchase and one issue in the month of June, 2006 and there was no opening stock on 1st June 2006, the Closing Stock of 200 units is to be valued at Rs. 20 per unit. In view of this, the argument of the Chief Accountant appears to be correct. Where there is only one purchase and one issue in a month with no opening stock, the method of pricing of material issues becomes irrelevant. Therefore, in the given case one should agree with the argument of the Chief Accountant that the value of Closing Stock remains the same no matter which method of pricing the issue is used. It may, however, be noted that the argument of Chief Accountant would not stand if one finds the value of the Closing Stock at the end of each month. (b) LIFO method has an edge over FIFO or any other method of pricing material issues due to the following advantages : (i) The cost of the materials issued will be either nearer or will reflect the current market

price. Thus, the cost of goods produced will be related to the trend of the market price of materials. Such a trend in price of materials enables the matching of cost of production with current sales revenues.

(ii) The use of the method during the period of rising prices does not reflect undue high profit in the income statement, as it was under the first-in-first-out or average method. In fact, the profit shown here is relatively lower because the cost of production takes into account the rising trend of material prices.

(iii) In the case of falling prices, profit tends to rise due to lower material cost, yet the finished products appear to be more competitive and are at market price.

(iv) During the period of inflation, LIFO will tend to show the correct profit and thus, avoid paying undue taxes to some extent.

(c) Simple Average Price Method - Under this method, materials issued are valued at average price, which is calculated by dividing the total of all units rate by the number of unit rate. In other words :

Material issue price = purchases of number Total

purchase each of pricesunit of Total

This method is useful under the following circumstances : 1. When the materials are received in uniform lots of similar quantity, otherwise, it will give

wrong results. 2. When purchase prices do not fluctuate considerably.

2.57

Page 114: 30510870 Cost Accounting and Financial Management

Cost Accounting Advantage : It is simple to understand and easy to operate. Disadvantages : 1. Materials issue cost does not represent actual cost price. Since the materials are issued at a price obtained by averaging cost prices, a profit or loss may arise from such type of pricing. 2. In case the prices of material fluctuate considerably, this method will give incorrect results. 3. The prices of materials issues used are determined by averaging prices of purchases without giving consideration to the quantity. Such a price determination is unscientific. (d) Weighted Average Price Method : This method gives due weights to quantities purchased and the purchase price, while, determining the issue price. The average issue price here is calculated by dividing the total cost of materials in the stock by total quantity of materials prior to each issue. The advantages and disadvantages of this method are : Advantages : 1. It smoothens the price fluctuations if at all it is there due to material purchases. 2. Issue prices need not be calculated for each issue unless new lot of materials is received. Disadvantage : 1. Material cost does not represent actual cost price and therefore, a profit or loss will arise out of such a pricing method. (e) Periodic Simple Average Price Method : This method is similar to Simple Average Price Method except that the average price is calculated at the end of the concerned period. In other words, the price paid during the period for different lots of materials purchased are added up and the total is divided by the number of purchases made during the period. The rate so computed is then used to price all the issues made during the period, and also for valuing the closing inventory of the period. Advantages : 1. It is simple to operate, as it avoids calculation of issue price after every receipt. 2. This method can usefully be employed in costing continuous processes where each individual order is absorbed into the general cost of producing large quantities of articles. Disadvantages : 1. This method cannot be applied in jobbing industry where each individual job order is to be priced at each stage of its completion. 2. This method is unscientific as it does not take into consideration the quantities

2.58

Page 115: 30510870 Cost Accounting and Financial Management

Material

purchased at different prices. 3. This method also suffers from all those disadvantages of simple average cost method. (f) Periodic Weighted Average Price Method : This method is like weighted average price method, except that the calculations of issue prices are made periodically (say, a month). The rate so arrived is used for the issues made during that period and also for valuing the inventory at the end of the period. Advantage : 1. This method is superior to the periodic simple average price method as it takes into account the quantities also. 2. It overcomes or evens out the effect of fluctuations. 3. In addition to above, the method also possesses all the advantages of the simple weighted average price method. Disadvantage : This method is not suitable for job costing because each job is to be priced at each stage of completion. (g) Moving Simple Average Price Method : Under this method, the rate for material issues is determined by dividing the total of the periodic simple average prices of a given number of periods by the numbers of periods. For determining the moving simple average price, it is necessary to fix up first period to be taken for determining the average. Suppose a six monthly period is decided upon and moving, average rate for the month of June is to be calculated. Under such a situation, we have to make a list of the simple average prices from January to June, add them up, and divide the total by six. To calculate the moving average rate for July, we have to omit simple average rate pertaining to January and add the rate relating to July and divide the total by six. Advantage : This method evens out price fluctuations over a longer period, thus stabilising the charges to work-in-progress. Thus the cost of production will be stable to a significant extent. Disadvantage : A profit or loss arises by the use of moving simple average cost. (h) Moving Weighted Average Price Method : Under this method, the issue, rate is calculated by dividing the total of the periodic weighted average price of a given number of periods by the number of periods. (i) Replacement Price Method: Replacement price is defined as the price at which it is possible to purchase an item, identical to that which is being replaced or revalued. Under this method, materials issued are valued at the replacement cost of the items. This method pre-supposes the determination of the replacement cost of materials at the time of each issue; viz., the cost at which identical materials could be currently purchased. The

2.59

Page 116: 30510870 Cost Accounting and Financial Management

Cost Accounting product cost under this method is at current market price, which is the main objective of the replacement price method. This method is useful to determine true cost of production and to value material issues in periods of rising prices, because the cost of material considered in cost of production would be able to replace the materials at the increased price. Advantage: Product cost reflects the current market prices and it can be compared with the selling price. Disadvantage: The use of the method requires the determination of market price of material before each issue of material. Such a requirement creates problems. (j) Realisable Price Method: Realisable price means a price at which the material to be issued can be sold in the market. This price may be more or may be less than the cost price at which it was originally purchased. Like replacement price method, the stores ledger would show profit or loss in this method too. (k) Standard Price Method: Under this method, materials are priced at some predetermined rate or standard price irrespective of the actual purchase cost of the materials. Standard cost is usually fixed after taking into consideration the following factors: (i) Current prices, (ii) Anticipated market trends, and (iii) Discount available and transport charges etc. Standard prices are fixed for each material and the requisitions are priced at the standard price. This method is useful for controlling material cost and determining the efficiency of purchase department. In the case of highly fluctuating prices of materials, it is difficult to fix their standard cost on long-term basis. Advantages: (1) The use of the standard price method simplifies the task of valuing issues of materials. (2) It facilitates the control of material cost and the task of judging the efficiency of purchase

department. (3) It reduces the clerical work. Disadvantages: (1) The use of standard price does not reflect the market price and thus results in a profit or

loss. (2) The fixation of standard price becomes difficult when prices fluctuate frequently.

2.60

Page 117: 30510870 Cost Accounting and Financial Management

Statement of receipts and issues by adopting First-in-First-Out Method Date Particulars Receipts Issues Balance Units Rate Value Units Rate Value Units Rate Value No. Rs. Rs. No. Rs. Rs. No. Rs. Rs.

Jan. 1 Purchase 100 1 100 — — — 100 1 100 Jan. 20 Purchase 100 2 200 — — — 100 1 100 100} 2} 200} Jan. 22 Issue to Job W 16 — — — 60 1 60 40 1 40 100} 2} 200} Jan. 23 Issue to Job W 17 — — — 40 1 40 20 2} 40} 80 2 160

Statement of receipts and issues by adopting Last-In-First-Out method Date Particulars Receipts Issues Balance Units Rate Value Units Rate Value Units Rate Value No. Rs. Rs. No. Rs. Rs. No. Rs. Rs.

Jan. 1 Purchase 100 1 100 — — — 100 1 100 Jan. 20 Purchase 100 2 200 — — — 100 1 100 100} 2} 200} Jan. 22 Issue to — — — 60 2 120 100 1 100 Job W 16 40} 2} 80} Jan. 23 Issue to — — — 40 2 80 80 1 80 Job W 17 20} 1} 20}

Page 118: 30510870 Cost Accounting and Financial Management

Statement of Receipt and Issues by adopting Weighted Average method

Date Particulars Receipts Issues Balance Units Rate Value Units Rate Value Units Rate Value

No. Rs. Rs. No. Rs. Rs. No. Rs. Rs.

Jan. 1 Purchase 100 1 100 — — — 100 1 100

Jan. 20 Purchase 100 2 200 — — — 200 1.50 300

Jan. 22 Issue to Job W 16 — — — 60 1.50 90 140 1.50 210

Jan. 23 Issue to Job W 17 — — — 60 1.50 90 80 1.50 120

Statement of Material Values allocated to Job W 16, Job 17 and Closing Stock, under aforesaid methods

FIFO LIFO Weighted Average Rs. Rs. Rs. Material for Job W 16 60 120 90

Material for Job W 17 80 100 90

Closing Stock 160 80 120

300 300 300

Page 119: 30510870 Cost Accounting and Financial Management

Material

(n) Inflated Price Method - In case material suffers loss in weight due to natural or climatic factors, e.g., evaporation, the issue price of the material is inflated to cover up the losses.

(o) Re-use Price Method - When materials are rejected and returned to the stores or a processed material is put to some other use, then for the purpose it is meant, then such materials are priced at a rate quite different from the price paid for them originally. There is no final procedure for valuing use of material.

Illustration :

The following information is extracted from the Stores Ledger: Material X

Opening Stock Nil Purchases : Jan. 1 100 @ Re. 1 per unit Jan. 20 100 @ Rs. 2 per unit Issues : Jan. 22 60 for Job W 16 Jan. 23 60 for Job W 17

Complete the receipts and issues valuation by adopting the First-In-First-Out, Last-In-First-Out and the Weighted Average Method. Tabulate the values allocated to Job W 16, Job W 17 and the closing stock under the methods aforesaid and discuss from different points of view which method you would prefer.

Solution

From the point of view of cost of material charged to each job, it is minimum under FIFO and maximum under LIFO (Refer to Tables on previous pages). During the period of rising prices, the use of FIFO give rise to high profits and that of LIFO low profits. In the case of weighted average there is no significant adverse or favourable effect on the cost of material as well as on profits.

From the point of view of valuation of closing stock it is apparent from the above statement that it is maximum under FIFO, moderate under weighted average and minimum under LIFO.

2.63

Page 120: 30510870 Cost Accounting and Financial Management

Cost Accounting It is clear from the Tables on previous page that the use of weighted average evens out the fluctuations in the prices. Under this method, the cost of materials issued to the jobs and the cost of material in hands reflects greater uniformity than under FIFO and LIFO. Thus from different points of view, weighted average method is preferred over LIFO and FIFO.

2.10 VALUATION OF RETURNS & SHORTAGES

2.10.1 Valuation of Materials Returned to the Vendor : Materials which do not meet quantity, dimensional and other specifications and are considered to be unfit for production are usually returned to the vendor. These materials can be returned to the vendor before they are sent to the stores. In case materials reach store and are noticed to be of sub-standard quality, then also they can be returned to vendor. The price of the materials to be returned to vendor should include its invoice price plus freight, receiving and handling charges etc. Strictly speaking, the materials returned to vendor should be returned at the stores ledger price and not at invoice price. But in practice invoice price is only considered, the gap between the invoice price and stores ledger price is charged as overhead.

In Stores ledger the defective or sub-standard materials are shown in the issue column at the rate shown in the ledger, and the difference between issue price and invoice cost is debited to an inventory adjustment account.

2.10.2 Valuation of Materials Returned to Stores: When materials requisitioned for a specific job or work-in progress are found to be in excess of the requirement or are unsuitable for the purpose, they are returned to the stores. There are two ways of treating such returns. (1) Such returns are entered in the receipt column at the price at which they were originally

issued, and the materials are kept in suspense, to be issued at the same price against the next requisition.

(2) Include the materials in stock as if they were fresh purchases at the original issue price. 2.10.3 Valuation of Shortages during Physical Verification: Materials found short during physical verification should be entered in the issue column and valued at the rate as per the method adopted, i.e., FIFO or any other.

2.11 SELECTION OF PRICING METHOD

No hard and fast rule or procedure has been laid down to select a method of pricing

2.64

Page 121: 30510870 Cost Accounting and Financial Management

Material

issues of material. However, the ultimate choice of a method of selection may be based on the following considerations. (a) The method of costing used and the policy of management. (b) The frequency of purchases and issues. (c) The extent of price fluctuations. (d) The extent of work involved in recording, issuing and pricing materials. (e) Whether cost of materials used should reflect current or historical conditions?

2.12 TREATMENT OF NORMAL AND ABNORMAL LOSS OF MATERIALS

Whichever method may be adopted for pricing materials, certain differences between the book balance and the value of physical stock are bound to occur. These differences, which may be a gain or loss, should be transferred to Inventory Adjustment Account pending investigation. If, upon investigation, they are regarded as normal, they should be transferred to Overhead Control Account; if abnormal, they should be written off to the Costing Profit and Loss Account.

In the case of normal losses, an alternative method used to price per unit of material so as to cover the normal loss can be understood with the help of the example considered. Suppose 1,000 metres of gunny cloth are purchased at Rs. 2 per metre. It is expected that 1% would be the normal loss due to issues being made in small lots. The inflated price would be Rs. 2.02 p. i.e., (Rs. 2,000 for 990 metres). The rate of Rs. 2.02 per metre of gunny cloth covers the cost a normal loss as well.

2.13 ACCOUNTING AND CONTROL OF WASTE, SCRAP, SPOILAGE AND DEFECTIVES

2.13.1 Waste - It represents the portion of basic raw materials lost in processing having no recoverable value. Waste may be visible - remnants of basic raw materials - or invisible; e.g., disappearance of basic raw materials through evaporation, smoke etc. Shrinkage of material due to natural causes may also be a form of a material wastage.

Normal waste is absorbed in the cost of net output, whereas abnormal waste is transferred to the Costing Profit and Loss Account.

For effective control of waste, normal allowances for yield and waste should be made from past experience, technical factors and special features of the material process and product. Actual yield and waste should be compared with anticipated figures and appropriate actions should be taken where necessary. Responsibility should be fixed on

2.65

Page 122: 30510870 Cost Accounting and Financial Management

Cost Accounting purchasing, storage, maintenance, production and inspection staff to maintain standards. A systematic procedure for feedback of achievement against laid down standards should be established.

2.13.2 Scrap - It has been defined as the incidental residue from certain types of manufacture, usually of small amount and low value, recoverable without further processing. Scrap may be treated in cost accounts in the following ways:- (i) Where the value of scrap is negligible, it may be excluded from costs. In other words, the

cost of scrap is borne by good units and income scrap is treated as other income. (ii) The sales value of scrap net of selling and distribution cost, is deducted from overhead to

reduce the overhead rate. A variation of this method is to deduct the net realisable value from material cost. This method is followed when scraps cannot be aggregated job or process-wise.

(iii) When scrap is identifiable with a particular job or process and its value is significant, the scrap account should be charged with full cost. The credit is given to the job or process concerned. The profit or loss in the scrap account, on realisation, will be transferred to the Costing Profit and Loss Account.

Control of scrap really means the maximum effective utilisation of raw material. Scrap control does not, therefore, start in the production department; it starts from the stage of product designing. Thus the most suitable type of materials, the right type of equipment and personnel would help in getting maximum quantity of finished product from a given raw material.

A standard allowance for scrap should be fixed and actual scrap should be collected, recorded and reported indicating the cost centre responsible for it. A periodical scrap report would serve the purpose where two or more departments or cost centres are responsible for the scrap; the reports should be routed through the departments concerned.

2.13.3 Spoilage - It is the term used for materials which are badly damaged in manufacturing operations, and they cannot be rectified economically and hence taken out of process to be disposed of in some manner without further processing. Spoilage may be either normal or abnormal.

Normal spoilage (i.e., which is inherent in the operation) costs are included in costs either charging the loss due to spoilage to the production order or by charging it to production overhead so that it is spread over all products. Any value realised from spoilage is credited to production order or production overhead account, as the case may be.

2.66

Page 123: 30510870 Cost Accounting and Financial Management

Material

The cost of abnormal spoilage (i.e., arising out of causes not inherent in manufacturing process) is charged to the Costing Profit and Loss Account. When spoiled work is the result of rigid specification, the cost of spoiled work is absorbed by good production while the cost of disposal is charged to production overhead.

To control spoilage, allowance for normal spoilage should be fixed and actual spoilage should be compared with standard set. A systematic procedure of reporting would help control over spoilage. A systematic procedure of reporting would help control over spoilage. A spoilage report should highlight the normal and abnormal spoilage, the department responsible, the causes of spoilage and the corrective action taken, if any.

2.13.4 Defectives - It signifies those units or portions of production which can be rectified and turned out as good units by the application of additional material, labour or other service. For example, some mudguards produced in a bicycle factory may have dents; or there may be duplication of pages or omission of some pages in a book. Defectives arise due to sub-standard materials, bad-supervision, bad-planning, poor workmanship, inadequate-equipment and careless inspection. To some extent, defectives may be unavoidable but usually, with proper care it should be possible to avoid defect in the goods produced.

Reclamation of loss from defective units - In the case of articles that have been spoiled, it is necessary to take steps to reclaim as much of the loss as possible. For this purpose: (i) All defective units should be sent to a place fixed for the purpose; (ii) These should be dismantled; (iii) Goods and serviceable parts should be separated and taken into stock; (iv) Parts which can be made serviceable by further work should be separated and sent to

the workshop for the purpose and taken into stock after the defects have been removed; and

(v) Parts which cannot be made serviceable should be collected in one place for being melted or sold.

Printed forms should be used to record quantities for all purposes aforementioned.

Control - When defectives are found, the Inspector will make out the Defective Work Report, giving particulars of the department, process or job, defective units, normal and abnormal defectives, cost of rectification etc. On receipt of the defective Work Report, it may be decided to rectify the defective work; all costs of rectification are collected against the rectification work order, precaution will be taken to see that number of defectives is

2.67

Page 124: 30510870 Cost Accounting and Financial Management

Cost Accounting within normal limits. Defectives are generally treated in two ways, either they are brought up to the standard by incurring further costs on additional material and labour or where possible, they are sold as inferior production (seconds) at lower prices.

Defectives are generally treated in two ways: either they are brought up to the standard by incurring further costs on additional material and labour or where possible, they are sold as inferior products (seconds) at lower prices. The following illustration is given to explain the accounting procedure followed in either case.

Total expenses of manufacture Rs. 5,000 Output Good: 450 units Defective: 50 units Cost of rectifying defectives Rs. 50

Cost per unit of production = 500

50.Rs000,5.Rs + = Rs. 10.10 per unit

If the defectives are not rectified but sold as ‘second’s say, @ Rs. 8 each then cost of goods produced will be;

= 450

400.Rs000,5.Rs − = Rs. 10.22 per unit.

Distinction between spoilage and defectives :

The difference between spoilage and defectives is that while spoilage cannot be repaired or reconditioned, defectives can be rectified and transferred, either back to standard production or to seconds.

Treatment of spoilage and defectives in Cost Accounting − Under Cost Accounts normal spoilage costs i.e., (which is inherent in the operation) are included in cost either by charging the loss due to spoilage to the production order or charging it to production overhead so that it is spread over all products. Any value realised from the sale of spoilage is credited to production order or production overhead account, as the case may be. The cost of abnormal spoilage (i.e. arising out of causes not inherent in manufacturing process) is charged to the Costing Profit and Loss Account. When spoiled work is the result of rigid specifications the cost of spoiled work is absorbed by good production while the cost of disposal is charged to production overheads.

The problem of accounting for defective work is the problem of accounting of the costs of

2.68

Page 125: 30510870 Cost Accounting and Financial Management

Material

rectification or rework.

The possible ways of treatment are as below: (i) Defectives that are considered inherent in the process and are identified as normal can

be recovered by using the following methods: (a) Charged to good products - The loss is absorbed by good units. This method is

used when ‘seconds’ have a normal value and defectives rectified into ‘seconds’ or ‘first’ are normal;

(b) Charged to general overheads - When the defectives caused in one department are reflected only on further processing, the rework costs are charged to general overheads;

(c) Charged to the department overheads - If the department responsible for defectives can be identified then the rectification costs should be charged to that department;

(d) Charged to Costing Profit and Loss Account - If defectives are abnormal and are due to causes beyond the control of organisation, the rework cost should be charged to Costing Profit and Loss Accounts.

(ii) Where defectives are easily identifiable with specific jobs, the work costs are debited to the job.

Procedure for the control of Spoilage and Defectives - To control spoilage, allowance for a normal spoilage should be fixed up and actual spoilage should be compared with standard set. A systematic procedure of reporting would help control over spoilage. A spoilage report (as below) would highlight the normal and abnormal spoilage, the department responsible, the causes of spoilage and the corrective action taken if any.

Spoilage Report Units/Deptt. No................................. Date......................... Production Order No......................

Units Produced

Units spoiled

Normal Qty.

Spoilage %

Abnormal Qty.

Spoilage %

Cost of abnormal spoilage

Rs.

Reason spoilage

Action taken

2.69

Page 126: 30510870 Cost Accounting and Financial Management

Cost Accounting Control of defectives may cover the following two areas: (a) Control over defectives produced (b) Control over reworking costs. For exercising effective control over defectives produced and the cost of reworking, standards, for normal percentage of defectives and reworking costs should be established.

Actual performance should be compared with the standards set. Defective Work Report (as shown on below page should be fed back to the respective Centres of Control.)

Defective Work Report Dept................. Date................. Causes of defects................. Nature of defects.................

Job/ Defective Detail of work to be done

Re-work Costs Total Rs.

Unit Cost of reworking Rs.

Net good output after re-wroking

Normal Abnormal Materials Rs.

Labour Rs

Overhead Rs.

Losses due to obsolete stores - Obsolescence is defined as “the loss in the intrinsic value of an asset due to its supersession”. Materials may become obsolete under any of the following circumstances: (i) where it is a spare part or a component of a machinery used in manufacture and that

machinery becomes obsolete ; (ii) where it is used in the manufacture of a product which has become obsolete ; (iii) where the material itself is replaced by another material due to either improved quality or

fall in price. In all three cases, the value of the obsolete material held in stock is a total loss and immediate steps should be taken to dispose it off at the best available price. The loss

2.70

Page 127: 30510870 Cost Accounting and Financial Management

Material

arising out of obsolete materials on abnormal loss does not form part of the cost of manufacture. Losses due to obsolescence can be minimised through careful forethought and reduced stocking of spares, etc. Stores records should be continuously gone through to see whether any item is likely to become obsolete. There will be such likelihood if an item has not been used for a long time. (This does not apply to spare parts of machines still in use).

2.14 CONSUMPTION OF MATERIALS

Any product that is manufactured in a firm entails consumption of resources like material, labour etc. The management for planning and control must know the cost of using these resources in manufacturing. The consumption of materials takes place say when the material is used in the manufacture of the product. It is important to note that the amount of materials consumed in a period by a cost object need not be equal to the amount of material available with the concern. For example, during any period the total of raw material stock available for use in production may not be equal to the amount of materials actually consumed and assigned to the cost object of the production. The difference between the material available and material consumed represents the stock of material at the end of the period.

2.14.1 Identification of Materials: For the identification of consumption of materials with products of cost centres the followings points should be noted: 1. It is required that the concern should follow coding system for all materials so that each

material is identified by unique code number. 2. It is required that each product of a cost centre should be given a unique code number so

that the direct material issued for production of particular product of a cost centre can be collected against the code number of that product.

However, it may not be possible to allocate all materials directly to individual product of a cost centre e.g. maintenance materials, inspection and testing materials etc. The consumption of these materials are collected for cost centre and then charged to individual product by adopting suitable overhead absorption rate of cost centre.

Overhead absorption rate of cost centre =

hrs.) machine or hrs. r (e.g.laboucentrecost to relating Base

centrecost forCost

2.71

Page 128: 30510870 Cost Accounting and Financial Management

Cost Accounting 3. Each issue of materials should be recorded. One way of doing this is to use a material

requisition note. This note shows the details of materials issued for product of cost centre and the cost centre which is to be charged with cost of materials.

4. A material return note is required for recording the excess materials returned to the store. This note is required to ensure that original product of cost centre is credited with the cost of material which was not used and that the stock records are updated.

5. A material transfer note is required for recording the transfer of materials from one product of cost centre to other or from one cost centre to other cost centre.

6. The cost of materials issued would be determined according to stock valuation method used.

2.14.2 Monitoring Consumption of Materials: For monitoring consumption of materials a storekeeper should periodically analyse the various material requisitions, material return notes and material transfer notes. Based on this analysis, a material abstract or material issue analysis sheet is prepared, which shows at a glance the value of material consumed in manufacturing each product. This statement is also useful for ascertaining the cost of material issued for each product.

Format of Material Abstract Week Ending............

Product Nos.

101 102 103 104 105 106

Total

for

Product

Overheads (Indirect Material

charged)

Material requisition or Transfer Note or Returned

Note No.

Amount

Rs. Rs. Rs. Rs. Rs. Rs. Rs. Rs.

— — — — — — — — —

Total

The material abstract statement serves a useful purpose. It in fact shows the amount of material to be debited to various products & overheads. The total amount of stores debited to various products & overheads should be the same as the total value of stores issued in any period.

2.14.3 Basis for consumption entries in Financial Accounts: Every manufacturing organisation assigns material costs to products for two purposes. Firstly, for external financial accounting requirements in order to allocate the material costs incurred during

2.72

Page 129: 30510870 Cost Accounting and Financial Management

Material

the period between cost of goods produced and inventories; secondly to provide useful information for managerial decision making requirements. In order to meet external financial accounting requirements, it may not be necessary to accurately trace material costs to individual products. Some products costs may be overstated and others may be understated but this may not matter for financial accounting purposes as long as total of individual materials costs assigned to cost of production and inventories are equal to total cost of materials.

In Financial Accounts the external transactions are recorded i.e. transaction between the firm and other entities are recorded in a manner that facilitates periodical reporting of assets, liabilities, revenue and expenditures for a firm as a whole or for each business segment or geographical segment in which firm operates. In Cost Accounting the internal transactions are recorded i.e., transactions between cost centre within the firm are recorded in a manner that facilitates analysis of costs for assigning them to cost units.

The consumption entries in financial accounts are made on the basis of total cost of purchases of materials after adjustment for opening and closing stock of materials. The stock of materials is taken at cost or net realisable value whichever is less.

Self-Examination Questions

Multiple Choice Questions (a) Direct material is a

(i) Fixed cost (ii) Variable cost (iii) Semi-variable cost.

(b) In most of the industries, the most important element of cost is (i) Material (ii) Labour (iii) Overheads.

(c) Which of the following is considered to be the normal loss of materials? (i) Loss due to accidents (ii) Pilferage (iii) Loss due to breaking the bulk (iv) Loss due to careless handling of materials (v) All of these.

2.73

Page 130: 30510870 Cost Accounting and Financial Management

Cost Accounting (d) In which of following methods of pricing, costs lag behind the current economic values?

(i) Last-in-first out price (ii) First-in-first out price (iii) Replacement price (iv) Weighted average price.

(e) Continuous stock taking is a part of (i) Annual stock taking (ii) Perpetual inventory (iii) ABC analysis.

(f) In which of the following methods, issues of materials are priced at pre-determined rate? (i) Inflated price method (ii) Standard price method (iii) Replacement price method (iv) Specific price method.

(g) When material prices fluctuate widely, the method of pricing that gives absurd results is (i) Simple average price (ii) Weighted average price (iii) Moving average price (iv) Inflated price.

(h) When prices fluctuate widely, the method that will smooth out the effect of fluctuations is (i) Simple average (ii) Weighted average (iii) FIFO (iv) LIFO.

(i) Which of the following is considered to be a normal loss of material? (i) Loss due to accidents (ii) Pilferage (iii) Loss due to careless handling of material. (iv) Loss due to breaking the bulk.

(j) Continuous stock taking is a part of, (i) Annual stock taking (ii) Perpetual inventory

2.74

Page 131: 30510870 Cost Accounting and Financial Management

Material

(iii) ABC analysis (iv) None of the above.

(k) Lead time 5 weeks, average weekly consumption 28 units. What should be the re- ordering level ?

(i) 120 units (ii) 130 units (iii) 140 units (iv) 150 units. (l) Price per unit Rs 150, annual consumption 2,000 units, ordering cost Rs 300 per order

and other charges 20% of cost. What should be the quantity of each order? (i) 150 units (ii) 200 units (iii) 225 units (iv) None of the above. (m) Bin card is maintained by the (i) Accounts department (ii) Costing department (iii) Stores (iv) None of the above. (n) Bin card contains (i) Details of the price of raw material lying in the Bin (ii) Details of the price and quantity of raw material lying in the Bin (iii) Details of quantity of material lying in the Bin (iv) None of the above. (o) Which of the following assumptions hold true for the calculation of Economic Order

Quantity? (i) Anticipated usage of material in units is known (ii) Cost per unit of material is constant and known (iii) Ordering cost per order is fixed (iv) None of the above. Answers to multiple choice questions (a) (ii); (b) (i); (c) (ii); (e) (ii); (f) (ii); (g) (i); (h) (ii);. (i) ii; (j) ii; (k) iii ; (l) ii; (m) iii ; (n) iii ; (o) iv

2.75

Page 132: 30510870 Cost Accounting and Financial Management

Cost Accounting Short Answer Type Questions 1. What are the main objectives of material control ? Explain the important requirements to

attain these objectives. 2. Give specimen form of the following :

(i) Bill of material. (ii) Purchase requisition.

3. State how you would treat the following in cost records : (a) Pricing of materials returned to stores and (b) Pricing of materials returned to suppliers.

4. What do you mean by waste, scrap, spoilage and defectives ? How they are treated in Cost Accounts ?

5. How would you deal the following in Cost Accounts : (i) Carriage inwards on raw materials. (ii) Cost of handling materials.

Long Answer Type Questions 1. Define re-ordering level and explain its relationship to maximum and minimum stock

levels. What factors may be considered in fixing reordering levels? 2. Discuss the use of perpetual inventory records and continuous stock taking. 3. Explain the concept of ‘ABC Analysis’ as a technique of inventory control. 4. Explain FIFO and LIFO methods of valuation of material issue. Discuss the effect of

rising prices and falling prices on these two methods of pricing of material issues. 5. Discuss in detail the procedure followed for procuring material. Numerical Questions 1. The rate of interest is 12%, the price per unit is Rs. 50, the number of units required in a

year is 5,000 and the cost of placing one order and receiving the goods once is Rs. 54. How much should be purchased at a time ?

2. The following is the record of an item in a stores ledger Units Amount Units Amount Rs. Rs. Jan. 1 To Balance b/d 10,000 20,000 Jan. 15 By Issue 14,000 30,000 Jan. 10 To Purchase 5,000 12,000 Feb. 8 By Issue 6,000 15,000 Feb. 3 To Purchase 20,000 55,000 Mar. 15 By Issue 22,000 60,000 Mar. 10 To Purchase 10,000 30,000 Mar. 31 By Bal. C/d 3,000 12,000 45,000 1,17,000 45,000 1,17,000

2.76

Page 133: 30510870 Cost Accounting and Financial Management

Material

Comment upon the method followed to price the issues. Find out the value of closing stock assuming issue to have been made for period on (i) FIFO basis, (ii) LIFO basis, and (iii) Weighted average basis.

3. In a printing press, a form of 8 pages was fitted upside down and this was discovered only after 5,000 sheets had been printed. The total print order was for 10,000 sheets, the cost per 1,000 sheets being Rs. 50. Would it be correct to say that the loss is Rs. 250 ?

4. A company ordered 54 tonnes of coal from a colliery. The invoice was for Rs. 1,000, the freight being Rs. 300. The actual quantity received was 52 tonnes. What should be the price per tonne ? Ten tonnes of coal were destroyed by an accident quantity received was 52 tonnes. How should be the loss be treated ?

5. In a section of ready-made garments factory the monthly wages and overhead respectively amounted to Rs. 5,875 and Rs. 3,650. In a period 10,000 metres of cloth were introduced out of which 600 metres still remained in stock. It takes 2 metres to make the garments but in the month, the total output was 4,500 garments. The cost of the cloth is Rs. 3.50 per metre. Ascertain the cost of garment, assuming cuttings were sold for Rs. 125.

6. The purchase Department of your organisation has received an offer of quantity discounts on its orders of materials as under :

Price per tonne Tonnes Rs. Nos. 1,400 Less than 500 1,380 500 and less than 1,000 1,360 1,000 and less than 2,000 1,340 2,000 and less than 3,000 1,320 3,000 and above.

The annual requirement for the material is 5,000 tonnes. The delivery cost per order is Rs. 1,200 and the annual stock holding cost is estimated at 20% of the average inventory. The Purchase Department wants you to consider the following purchase options and advise which among them will be the most economical ordering quantity, presenting the relevant information in a tabular form. The purchase quantity options to be considered are 400 tonnes, 500 tonnes, 1,000 tonnes, 2,000 tonnes and 3,000 tonnes.

7. Component ‘Pee’ is made entirely in cost centre 100. Material cost is 6 paise per component and each component takes 10 minutes to produce. The machine operator is

2.77

Page 134: 30510870 Cost Accounting and Financial Management

Cost Accounting

paid 72 paise per hour, and the machine hour rate is Rs. 1.50. The setting up of the machine to produce the component ‘Pee’ takes 2 hours 20 minutes. On the basis of this information, prepare a cost sheet showing the production and setting up cost, both in total and per component, assuming that a batch of : (a) 10 components, (b) 100 components, and (c) 1,000 components (d) is produced.

8. From the details given below, calculate : (i) Re-ordering level (ii) Maximum level (iii) Minimum level (iv) Danger level Re-ordering quantity is to be calculated on the basis of following information. - cost of placing a purchase order is Rs. 20 - Number of units to be purchased during the year is 5,000. - Purchase price per unit inclusive of transportation cost is Rs. 50 - Annual cost of storage per unit is Rs. 5 - Details of lead time : Average 10 days, Maximum 15 days, Minimum 6 days. For emergency purchase 4

days. - Rate of consumption Average : 15 units per day. Minimum : 20 units per day.

2.78

Page 135: 30510870 Cost Accounting and Financial Management

CHAPTER 3

LABOUR

Learning objectives When you have finished studying this chapter, you should be able to ♦ Understand the need of labour cost control. ♦ Understand the attendance and the payroll procedure. ♦ Describe the meaning and accounting treatment of idle time and overtime. ♦ Understand the concept of labour turnover and the various methods of computing the

same. ♦ Understand various types of systems of wage payment and incentives. ♦ Describe the efficiency rating procedures.

3.1 INTRODUCTION Labour cost after material cost is another significant element of cost not only because the wage bill in a modern organisation is generally substantial but also because it has certain peculiar characteristics which other elements of cost do not have. A good cost accountant must understand the special features of labour cost, the most important of which is that there is almost no limit to the increase of output of this most important and wonderful factor of production.

3.2 LABOUR COST CONTROL Labour costs are associated with human beings. To control labour costs one has to understand why human beings work and what factors motivate them to give best performance. (Here the students may refer to Chapters 2 and 3 in Organisation and Management booklet). Control over labour costs does not imply control over the size of the wage bill; it also does not imply that wages of each worker should be kept as low as possible. Actually if a policy of low wages is adopted, it may turn out to be expensive since the ill-paid workers will be dissatisfied and turn out low output. Low wages are, therefore, often dear wages. The aim should be to keep the wages cost per unit of output as low as possible. This can only be brought about by giving workers optimum wages and then harnessing their energies to optimise output. A well

Page 136: 30510870 Cost Accounting and Financial Management

Cost Accounting motivated team of workers can bring about wonders. Each concern should, therefore, constantly strive to raise the productivity of labour. The efforts for the control of labour costs should begin from the very beginning. There has to be a concerted effort by all the concerned departments. In a large organisation, generally the following departments are involved in the control of labour costs : 1. Personnel Department - This department is assigned the duty of recruiting workers,

training them and maintaining their record. It is the duty of this department to ensure that the persons recruited possess the qualifications and qualities necessary to perform well the concerned jobs.

2. Engineering and Work Study Department - This department prepares plans and specifications for each job, supervises production activities, conducts time and motion studies, undertakes job analysis, etc.

3. Time-keeping Department - This Department is primarily concerned with the maintenance of attendance records of the employees and the time spent by them on various jobs, etc.

4. Payroll Department - This department is responsible for the preparation of payroll of the employees.

5. Cost Accounting Department - This department is responsible for the accumulation and classification etc. of all type of costs. All such data pertaining to labour costs are also collected, analysed and allocated to various jobs, processes, departments, etc., by this department.

3.2.1 Important Factors for the Control of Labour Cost : To exercise an effective control over the labour costs, the essential requisite is efficient utilisation of labour and allied factors. The main points which need consideration for controlling labour costs are the following : (i) Assessment of manpower requirements. (ii) Control over time-keeping and time-booking. (iii) Time & Motion Study. (iv) Control over idle time and overtime. (v) Control over labour turnover. (vi) Wage systems. (vii) Incentive systems. (viii) Systems of wage payment and incentives. (ix) Control over casual, contract and other workers. (x) Job Evaluation and Merit Rating.

3.2

Page 137: 30510870 Cost Accounting and Financial Management

Labour

(xi) Labour productivity. 3.2.2 Collection of Labour Costs : The task of collecting labour costs is performed by the Cost Accounting Department which record separately wages paid to direct and indirect labour. It is the duty of this department to ascertain the effective wages per hour in each department and to analyse the total payment of wages of each department into : (i) the amount included in the direct cost of goods produced or jobs completed; (ii) the amount treated as indirect labour and thus included in overheads ; and (iii) the amount treated as the cost of idle time and hence loss. Through this process costs of various jobs are ascertained. Naturally, in this the proper recording of time spent by the workers is essential. Labour cost per hour may be collected through the use of the form given below :

A.B.C. Co. Ltd. Department Labour Cost Report Weekended

Productive Time Wages Paid Name of the employee

Section Day Work Hrs.

O/Time Hours

Total Hours

Paid Idle Time Hrs.

Total Time Hrs.

Time Hrs.

O/Time Premium

Bonus Total Cost per Production Hours

Rs. Rs. Rs. Rs. Rs.

3.3 ATTENDENCE & PAYROLL PROCEDURES 3.3.1 Attendance Procedure / Time-keeping : It refers to correct recording of the employees’ attendance time. Students may note the difference between “time keeping” and “time booking”. The latter refers to break up of time on various jobs while the former implies a record of total time spent by the workers in a factory. Objectives of Time-keeping : Correct recording of employees’ attendance time is of utmost importance where payment is made on the basis of time worked. Where payment is made by results viz; straight piece work, it would still be necessary to correctly record attendance for the purpose of ensuring that proper discipline and adequate rate of production are maintained. In fact the various objectives of time-keeping are as follows: (i) For the preparation of payrolls. (ii) For calculating overtime. (iii) For ascertaining and controlling labour cost. (iv) For ascertaining idle time.

3.3

Page 138: 30510870 Cost Accounting and Financial Management

Cost Accounting (v) For disciplinary purposes. (vi) For overhead distribution. Methods of Time-keeping : There are two methods of time-keeping. They are the manual methods and the mechanical methods. The choice of a particular method depends upon the requirements and policy of a firm. But whichever method is followed, it should make a correct record of the time incurring the minimum possible expenditure and should minimise the risk of fraudulent payments of wages. Manual method : The manual methods of time-keeping are as follows : (a) Attendance Register Method, and (b) Metal Disc Method. (a) Attendance Register Method : It is the oldest method of recording time. Under this method, an attendance register is kept in the time office adjacent to the factory gate or in each department for workers employed therein. The attendance register contains such columns as the name of the worker, the worker’s number, the department in which he is working, the rate of wages, the time of arrival and departure, normal time and the overtime. The time of a arrival and departure, may be noted down by an employee know as time-keeper. This method is simple and inexpensive and can be used in small firms where the number of workers is not large. This method may lead to dishonest practice of recording wrong time because there is possibility of collusion between some of the workers and the time-keeper. However, for recording the time of workers who work at customers’ premises and places which are situated at a distance from the factory, this may be the only suitable method. (b) Metal Disc Method : Under this method, each worker is allotted a metal disc or a token with a hole bearing his identification number. A board is kept at the gate with pegs on it and all token are hung on this board. These boards can be maintained separately for each department so that the workers could remove their tokens from the board without undue delay. As the workers enter the factory gate, they remove their respective discs or tokens and place them in a box or tray kept near the board. Immediately after the scheduled time for entering the factory, the box is removed and the late comers will have to give their tokens to the time-keeper personally so that the exact time of their arrival could be recorded. The discs or tokens still left on the board represent the absentee workers. Later the time-keeper records the attendance in a register known as Daily Muster Roll which is subsequently passed on to the Pay Roll Department. This method is simple because illiterate workers can very easily recognize their tokens and put them in the box. This method is better than attendance register method and is useful when the number of employees is not large. But it has certain disadvantages of its own as given below : 1. There are chances that a worker may try to remove his companion’s token from the

3.4

Page 139: 30510870 Cost Accounting and Financial Management

Labour

board in order to get his presence marked when he is absent. 2. There are chances of disputes regarding the exact time of arrival of a worker because the

time-keeper marking the attendance can commit mistakes deliberately or through carelessness. There is no authentic proof of the presence or absence of the workers.

3. There are chances of inclusion of dummy or ghost workers by the time-keeper in the attendance register or Daily Muster Roll.

Mechanical methods : The mechanical methods that are generally used for the recording of time of workers may be as follows : (a) Time Recording Clocks; and (b) Dial Time Records. (a) Time Recording Clocks : The time recording clock is mechanical device which automatically records the time of the workers. This method has been developed to obviate some of the difficulties experienced in case of manual methods and this method is useful when the number of workers is fairly large. Under this method, each worker is given a Time Card usually of one week duration. Time cards are serially arranged in a tray near the factory gate and as the worker enters the gate, he picks up his card from the tray, puts it in the time recording clock which prints the exact time of arrival in the proper space against the particular day. This process is repeated for recording time of departure for lunch, return from lunch and time of leaving the factory in the evening. Late arrivals, early leavings and overtime are printed in red to attract the attention of the management. A time card may also give such particulars as hourly rate, total gross wages, less deductions and net wages payable. If these particulars are included in the time card, it would be known as combined time and pay-roll card divided into two parts, the upper part being the record of time and the lower one serving as the wage ticket. Wages are calculated on the basis of time recorded in the upper portion and are entered in the lower portion by the pay-roll department. The specimen of a combined time and pay-roll card may be as given below :

3.5

Page 140: 30510870 Cost Accounting and Financial Management

Cost Accounting

COMBINED TIME AND PAY ROLL CARD Name of the worker................. Week ending............... No. of the worker............... Department........................ Regular Over Time Total Time Day In Out In Out Normal Overtime Time Monday A.M. P.M. Tuesday A.M. P.M. Wednesday A.M. P.M. Thursday A.M. P.M. Friday A.M. P.M. Saturday A.M. P.M. Sunday A.M. P.M. Normal Hours Rate Amount Deduc- Net Time worked tions Amount Calculation of Wages Over Time Total Time-keeper....................... Pay Roll Clerk..................... Foreman............................. Received the net amount as above............... Worker...............

3.6

Page 141: 30510870 Cost Accounting and Financial Management

Labour

The main advantage of this method is that there are no chances of disputes arising in connection with recording of time of workers because time is recorded by the time recording clock and not by the time-keeper. There is no scope for partiality or carelessness of the time-keeper as it is in case of manual methods. But this method suffers from the following defects : 1. There are chances that a worker may try to get his friend’s time card from the tray in

order to get him marked present in time when he is actually late or get his presence marked when he is absent. This drawback can be removed if the time-keeper does not show carelessness.

2. Sometimes, the time recording clock goes out of order and the work of recording of time is dislocated.

(b) Dial Time Records : The dial time recorder is a machine which has a dial around the clock. This dial has a number of holes (usually about 150) and each hole bears a number corresponding to the identification number of the worker concerned. There is one radial arm at the centre of the dial. As a worker enters the factory gate, he is to press the radial arm after placing it at the hole of his number and his time will automatically be recorded on roll of a paper inside the dial time recorder against the number. The sheet on which the time is recorded provides a running account of the worker’s time. This machine allows greater accuracy and can itself transcribe the number of hours to the wages sheets. This machine can also calculate the wages of the workers and thus avoids much loss of time. However, the high installation cost of the dial time recorder and its use for only a limited of workers are the drawbacks of this method. Requisites of a Good Time-keeping System : A good time-keeping system should have a following requisites : 1. System of time-keeping should be such which should not allow proxy for another worker

under any circumstances. 2. There should also be a provision of recording of time of piece workers so that regular

attendance and discipline may be maintained. This is necessary to maintain uniformity of flow of production.

3. Time of arrival as well as time of departure of workers should be recorded so that total time of workers may be recorded and wages may be calculated accordingly.

4. As far as possible, method of recording of time should be mechanical so that chances of disputes regarding time may not arise between workers and the time-keeper.

5. Late-comers should record late arrivals. Any relaxation by the time-keeper in this regard will encourage indiscipline.

6. The system should be simple, smooth and quick. Unnecessary queuing at the factory gate should be avoided. Sufficient clocks should be installed keeping in view the number

3.7

Page 142: 30510870 Cost Accounting and Financial Management

Cost Accounting

of workers so that workers may not have to wait for a long period for recording their time of arrivals and departures.

7. A responsible officer should pay frequent visits at the factory gate to see that proper method of recording of time is being followed.

Time-Booking - The clock card is required, essentially, for the correct determination of the amount of wages due to a worker on the basis of time he has put in the factory. It merely records day by day and period by period the total time spent by each individual worker in the factory. But it does not show how that time was put to use in the factory—how an individual worker utilised his time in completing jobs entrusted to him and how long he was kept waiting for one reason or another due to lack of work, lack of material and supplies, lack of instructions, machine breakdowns, power failures and the like. These are all vital pieces of information necessary for the proper collection of cost data and for effective controlling of costs. For the collection of all such information, a separate record, generally known as Time (or Job) card, is kept. The time (or job) card can be of two types—one containing analysis of time with reference to each job and the other with reference to each worker. In case of job card made out according to job a separate job card is employed in respect of a job undertaken; where a job involves several operations, a separate entry is made in respect of each operation. Thus the job card would record the total time spent on a particular job or operation. If a number of people are engaged on the same job or operation, the time of all those workers would be booked on the same card. One obvious advantage of this method is that it provides complete data on the labour content of job or operation collectively so that the computation of labour cost is greatly facilitated. But this method has drawbacks as well. Since a worker’s job timing is scattered over a number of job cards the time spent on all these jobs and idle time must be abstracted periodically for finding each worker’s total time spent on different jobs and the time for which he remained idle during the period. The total of these two times (job and idle) must obviously equal his total attendance time, as shown by his clock card or attendance register. Thus, it would be seen that if the job cards are made out according to job or operation a separate summary has to be prepared for reconciling each worker’s job and idle time with his gate time. It would be quite obvious that such a reconciliation is of great importance from the point of view of labour costs. If on the other hand, one job (or time) card were to be issued for each worker, it would greatly facilitate reconciliation of the worker’s job time with his gate time. Under this system, a card would be issued to each worker for each day or for each week and the time which he spends on different jobs (and also any idle time) would be recorded in the same card so that the card would have a complete history on it as to how his time had been spent during the period. Since all the details would be on one card the total time accounted for in the job card would be readily tallied with the total time put in the Gate Card or attendance register. In this case

3.8

Page 143: 30510870 Cost Accounting and Financial Management

Labour

however, a Labour Abstract for different jobs would have to be prepared from the card of individual worker so that total hours (and/or their value) put in by different workers on different jobs during the period could be ascertained and aggregated. It would thus be seen that according to either of the method a process of abstraction and reconciliation is necessary. Specimens of two types of job cards are given below:

JOB CARD (1st type) Description of ................................. Job No. ........................................ Department ..................................... Date.............................................. Worker’s Start Stop Elapsed Actual time Rate Amount No. time taken

Supervisor’s initial JOB (OR TIME) CARD (2nd type)

No. .................... Date.................................... Name of the worker ................................. Ticket No............................... Department........................................... Operation............................... Job Start Stop Time Time Rate Amount No. Elapsed

Supervisor’s initial Reconciliation of gate and job cards - An advantage of the introduction of job card is that it enables a reconciliation to be made of the time spent by the worker in each department with the time paid for as per the attendance record. Reconciliation not only helps in locating wastage of time, but also in preventing dummy workers being put on the payroll of workers paid for time not worked by them. The two sets of records serve separate purposes. Where payment to labour is on the time rate basis, the Gate Card is a record of the hours of work that should be paid for. Since the Gate Card merely records the hours during which the worker has been within the premises of the factory and it does not contain any details as to how those hours have been put to use by the worker in his department, a job card must be prepared to provide the necessary information. As we have already seen, the job card may be prepared either worker-wise or job-wise.

3.9

Page 144: 30510870 Cost Accounting and Financial Management

Cost Accounting Objectives of Time-Booking - Objectives of time-booking are as follows: 1. To ensure that time paid for, according to time keeping, has been properly utilised on

different jobs or work orders. 2. To ascertain the cost of each job or work order. 3. To provide a basis for the apportionment of overhead expenses over various jobs/work

orders when the method for the allocation of overhead depends upon time spent on different jobs.

3.3.2 Payroll procedure : The hours worked by each employee as reflected on the completed clock cards are entered by an accounting department employee (or employees) on the payroll sheet or payroll summary. All employees authorized for employment by the personnel department are first listed on the payroll sheet. Hours and hourly rates are then transferred from the clock cards, and total earnings are computed. Should a clock card for an employee not listed on the payroll sheet be found, investigation of its propriety is required. Likewise, there should be an explanation for any missing clock cards. After the gross earnings (that is, the total amount earned by an employee before any deductions are taken into consideration) have been calculated for every employee, deduction are entered on the payroll sheet, and the net pay of each employee is determined. Under a computerized system, each employee’s payroll data would be input into the computer and it would prepare the entire payroll sheet. Payroll deductions are of two kinds, nontax and tax. Nontax deductions are made at the request of the employee or are required by union contracts. Among the more common examples are union dues, hospitalization insurance, withholding for the purchase of savings bonds, and contributions to charities. Tax deductions are made in compliance with Income Tax Act. Paying the wages : The payroll sheet is the basis for the preparation of a payroll voucher by the accounting department authorizing disbursements for the net amounts payable to employees. If the number of employees is large, payments are usually made from a special payroll account. Using a separate payroll account contributes to good internal control, since the audit trail of payroll activities is easier to follow when a separate account for payroll disbursements is used by a company. In each pay period an amount to cover the net payroll is transferred from the company’s general account to a special payroll account. Cheques payable to the individual employees are then drawn against the payroll account. In a computerized system, the employees’ paycheques would be printed out by the computer based on the information within the computer-prepared payroll sheet. In addition to providing a better means of control, use of a separate account for payroll simplifies record keeping by reducing the number of cheques that will clear through the general account. Precautions to ensure proper payment of the payroll should be taken. Payments should be made only to the employees themselves after proper identification. As a control, payroll

3.10

Page 145: 30510870 Cost Accounting and Financial Management

Labour

cheques should not be given to factory supervisors or department heads for distribution to the employees under their jurisdiction, since they were probably involved in the process of accumulating the hours worked by their employees. Rather, an individual (or individuals) having no record-keeping functions associated with the payroll (such as the time-keeping function and the preparation of payroll function) should be assigned the job of distributing paycheques. Unclaimed paycheques should be investigated to determine why they have not been picked up by employees. 3.3.3 Overview of Statutory requirements : According to the Factories Act, 1948, every worker is required to work not more than 9 hours a day or 48 hours in a week. If, due to the urgency of the work, a worker is required to work for more than 9 hours a day, excess time over 48 hours i.e. overtime is to be paid to the worker at a higher rate, generally at double the normal wage rate. The excess rate over normal wage rate is called overtime premium.

3.4 IDLE TIME It is a time during which no production is carried out because the worker remains idle even though they are paid. Idle time can be normal idle or abnormal idle time. Normal idle time : It is inherent in any work situation and cannot be eliminated. Abnormal idle time : Apart from normal idle time, there may be factors which give rise to abnormal idle time. Machines and men cannot be expected to work continuously. In the midst of operations, a machine may have to be stopped for some adjustments being made. Each morning and on the resumption of work in the post-lunch period, some time will be lost before a job can be started. There may also be some waiting time in between the finishing of one job and the starting of another. Similarly, even if a plant operates with a reasonable degree of efficiency, some allowance may be required for loss of time due to occasional power failure, machines or tool break-down, delay in delivery of material and stores or of tools etc. All such reasonable time losses are normal idle time. Normal idle time of direct workers can be treated as a part of the direct cost. This can be done by inflating the wage rates for costing purposes. Suppose 10 minutes per hour of work are treated as normal idle time. If a worker puts in 3 hours on a job, his wages for 3½ hours can be charged to the job. Alternatively the cost of normal idle time may be treated as a part of factory overheads. Losses in excess of the normal idle time are not properly chargeable as overheads; abnormal losses on account of idle time should be written off by being directly debited to the Costing Profit and Loss Account. It is obvious that for establishing abnormal idle time, the normal time required for each product or job will have to be determined. Accurate recording of the idle time in the departments is of great importance from the point of view of cost finding and cost control. For controlling costs, careful analysis and recording of

3.11

Page 146: 30510870 Cost Accounting and Financial Management

Cost Accounting idle time under significant heads is essential. In order to facilitate identification, the major causes which account for idle time may be grouped under the following two heads : (i) Normal causes : Some idle time is inherent in every situation. The time lost between factory gate and the place of work, the interval between one job and another, the setting up time for the machine, normal fatigue etc. result in normal idle time. (ii) Abnormal causes : Idle time may also arise due to abnormal factors like lack of coordination, power failure, breakdown of machines, non-availability of raw materials, strikes, lockouts, poor supervision, fire, flood etc. The causes for abnormal idle time should be further analysed into controllable and uncontrollable. Controllable abnormal idle time refers to that time which could have been put to productive use had the management been more alert and efficient. All such time which could have been avoided is controllable idle time. However, time lost due to abnormal causes, over which management does not have any control e.g., breakdown of machines, flood etc. may be characterised as uncontrollable idle time. Treatment of idle time in Cost Accounting : Normal idle time is treated as a part of the cost of production. Thus, in the case of direct workers an allowance for normal idle time is built into the labour cost rates. In the case of indirect workers, normal idle time is spread over all the products or jobs through the process of absorption of factory overheads. Abnormal idle time cost is not included as a part of production cost and is shown as a separate item in the Costing Profit and Loss Account so that normal costs are not disturbed. This also helps in drawing the attention of the management towards the exact losses due to abnormal idle time. The cost of abnormal idle time should be further categorised into controllable and uncontrollable. For each category, the break-up of cost due to various factors should be separately shown. This would help the management in fixing responsibility for controlling idle time. Management should aim at eliminating controllable idle time and on a long-term basis reducing even the normal idle time. This would require a detailed analysis of the causes leading to such idle time. Depending upon the particular causes, proper managerial action would be required to reduce the impact of such idle time. Basic control can be exercised through periodical reports on idle time showing a detailed analysis of the causes for the same, the departments where it is occurring and the persons responsible for it, along with a statement of the cost of such idle time. Illustration : ‘X’ an employee of ABC Co. gets the following emoluments and benefits:

(a) Basic pay Rs. 1,000 p.m. (b) Dearness allowance Rs. 200 p.m. (c) Bonus Rs. 20% of salary and D.A. (d) Other allowances Rs. 250 p.m.

3.12

Page 147: 30510870 Cost Accounting and Financial Management

Labour

(e) Employee’s contribution to P.F. 10% of salary and D.A. ‘X’ works for 2,400 hours per annum, out of which 400 hours are non-productive and treated as normal idle time. You are required to find out the effective hourly cost of employee ‘X’. Solution

Statement showing computation of effective hourly cost of employee ‘X’

(i) Earning of Employee ‘X’: Per month Per annum Rs. Rs. Basic pay 1,000 12,000 Dearness Allowance 200 2,400 Bonus 240 2,880 Employees’ contribution to provident fund 120 1,440 Other allowance 250 3,000 1,810 21,720(ii) Effective working hours : Annual working hours 2,400 Less : Normal idle time 400 Effective working hours 2,000 Effective hourly cost of ‘X’ : Rs. 21,720/2,000 10.86

3.5 OVERTIME Work done beyond normal working hours is known as ‘overtime work’. Overtime has to be paid in India at double the rate of wages including dearness allowance and the value of food concession, according to the Factories Act, 1948. This Act as stated earlier also lays down that a worker is entitled to overtime when he works for more than 9 hours on any day or more than 48 hours in a week. Occasional overtime is a healthy sign since it indicates that the firm has the optimum capacity and that the capacity is being fully utilised. But persistent overtime is rather a bad sign because it may indicate either : (a) that the firm needs larger capacity in men and machines, or (b) that men have got into the habit of postponing their ordinary work towards the evening so that they can earn extra money in the form of overtime wages. Overtime work may arise in a department in one of the following circumstances : (1) The customer may agree to bear the entire charge of overtime because of urgency of

3.13

Page 148: 30510870 Cost Accounting and Financial Management

Cost Accounting

work. (2) Overtime may be called for to make up any shortfall in production due to some

unexpected development. (3) Overtime work may be necessary to make up a shortfall in production due to some fault

of management. (4) Overtime work may be resorted to, to secure an out-turn in excess of the normal output

to take advantage of an expanding market or of rising demand. Overtime premium : Overtime payment is the amount of wages paid for working beyond normal working hours. The rate for overtime work is higher than the normal time rate; usually it is at double the normal rates. The extra amount so paid over the normal rate is called overtime premium. Effect of overtime payment on productivity : Overtime work should be resorted to only when it is extremely essential because it involves extra cost. The overtime payment increases the cost of production in the following ways : 1. The overtime premium paid is an extra payment in addition to the normal rate. 2. The efficiency of operators during overtime work may fall and thus output may be less

than normal output. 3. In order to earn more the workers may not concentrate on work during normal time and

thus the output during normal hours may also fall. 4. Reduced output and increased premium of overtime will bring about an increase in costs

of production. Treatment of overtime premium in Cost Accounting : Under Cost Accounting the overtime premium is treated as follows : (1) If overtime is resorted to at the desire of the customer, then overtime premium may be

charged to the job directly. (2) If overtime is required to cope with general production programmes or for meeting urgent

orders, the overtime premium should be treated as overhead cost of the particular department or cost centre which works overtime.

(3) If overtime is worked in a department due to the fault of another department, the overtime premium should be charged to the latter department.

(4) Overtime worked on account of abnormal conditions such as flood, earthquake etc., should not be charged to cost, but to Costing Profit and Loss Account.

Steps for controlling overtime : To keep overtime to its minimum, it is necessary to exercise proper control over the overtime work. The suitable procedure which may be adopted for

3.14

Page 149: 30510870 Cost Accounting and Financial Management

Labour

controlling overtime comprises the following steps : 1. Watch on the output during normal hours should be maintained to ensure that overtime is

not granted when normal output is not obtained during the normal hours, without any special reasons.

2. Statement concerning overtime work be prepared along with justifications, at appropriate places for putting up before the competent authority.

3. Prior sanction about overtime should be obtained from competent authority. 4. Actual rate of output produced during the overtime period should be compared with

normal rate of output. 5. Periodical reports on overtime wages should be sent to top management for taking

corrective action. 6. If possible an upper limit may be fixed for each category of workers in respect of

overtime. Illustration : It is seen from the job card for repair of the customer’s equipment that a total of 154 labour hours have been put in as detailed below :

Worker ‘A’ paid at Worker ‘B’ paid at Supervisory Rs. 2 per day of Re. 1 per day worker ‘C’ 8 hours of 8 hours paid of Rs. 3 per day of 8 hours

Monday 10 - 1/2 hours 8 hours 10 - 1/2 hrs. Tuesday 8 ” 8 ” 8 ” Wednesday 10 - 1/2 hours 8 ” 10 - 1/2 hours Thursday 9 - 1/2 ” 8 ” 9 - 1/2 ” Friday 10 - 1/2 ” 8 ” 10 - 1/2 ” Saturday − 8 ” 8 ”

Total 49 hours 48 hours 57 hours

In terms of an award in a labour conciliation, the workers are to be paid dearness allowance on the basis of cost of living index figures relating to each month which works out @ Rs. 96 for the relevant month. The dearness allowance is payable to all workers irrespective of wage rate if they are present or are on leave with wages on all working days. Sunday is a weekly holiday and each worker has to work for 8 hours on all week days and 4 hours on Saturdays; the workers are however paid full wages for Saturday (8 hours for 4 hours worked).

3.15

Page 150: 30510870 Cost Accounting and Financial Management

Cost Accounting Workers are paid overtime according to the Factories Act for hours worked in excess of normal working hours on each day. Excluding holidays (including 4 hours work to be put in on Saturday) the total number of hours work out to 172 in the relevant month. The company’s contribution to Provident Fund and Employees State Insurance Premium are absorbed into overheads. Work out the wages payable to each worker. Solution (1) Calculation of hours to be paid for worker A :

Normal Extra Overtime Equivalent Total hours hours hours normal normal hours of hours overtime hours

Monday 8 1 1 ½ 3 12 Tuesday 8 - - - 8 Wednesday 8 1 1½ 3 12 Thursday 8 1 ½ 1 10 Friday 8 1 1½ 3 12 Saturday - - - - -

40 4 5 10 54

Worker B will get wages for 52 hours i.e., actually worked plus 4 hours worked extra on Saturday. Worker C will get wages for 66 hours—the same as for A till Friday plus 12 hours for work on Saturday.

(a) Wages payable :

A B C Basic wages per hour (Rs.) 0.250 0.125 0.375 Dearness allowance per hour (Rs.) 0.500 0.500 0.500 Hourly rate (Rs.) 0.750 0.625 0.875 Normal hours 44 52 56 Overtime hours 5 - 5 Normal wages (Rs.) 33.00 32.50 49.00 Overtime wages (Rs.) 7.50 - 8.75Total wages payable : (Rs.) 40.50 32.50 57.75

3.16

Page 151: 30510870 Cost Accounting and Financial Management

Labour

Illustration In a factory, the basic wage rate is Rs. 10 per hour and overtime rates are as follows :

Before and after normal working hours : 175% of basic wage rate Sundays and holidays : 225% of basic wage rate During the previous year, the following hours were worked : Normal time : 1,00,000 hours Overtime before and after working hours : 20,000 hours Overtime on Sundays and holidays : 5,000 hoursTotal : 1,25,000 hoursThe following hours have been worked on job ‘Z’ : Normal : 1000 hours. Overtime before and after working hrs. : 100 hours. Sundays and holidays : 25 hours.

Total : 1125 hours. You are required to calculate the labour cost chargeable to jobs ‘Z’ and overhead in each of the following instances: (a) Where overtime is worked regularly throughout the year as a policy due to the labour

shortage. (b) Where overtime is worked irregularly to meet the requirements of production. (c) Where overtime is worked at the request of the customer to expedite the job. Solution Workings Computation of average inflated wage rate (including overtime premium) :

Basic wage rate : Rs. 10 per hour Overtime wage rate before and after working hours : Rs. 10 × 175% = Rs. 17.50 per hour Overtime wage rate for Sundays and holidays : Rs. 10 × 225% =Rs. 22.50 per hour Annual wages for the previous year for normal time : 1,00,000 hrs. × Rs. 10 = Rs. 10,00,000 Wages for overtime before and after working hours : 20,000 hrs. × Rs. 17.50=Rs. 3,50,000

3.17

Page 152: 30510870 Cost Accounting and Financial Management

Cost Accounting Wages for overtime on Sundays and holidays : 5,000 hrs. × Rs. 22.50 = Rs. 1,12,500 Total wages for 1,25,000 hrs. = Rs. 14,62,500 Average inflated wage rate : Rs. 14,62,500 = Rs. 11.70 per hour. = 1,25,000 hrs. (a) Where overtime is worked regularly as a policy due to labour shortage, the overtime

premium is treated as a part of labour cost and job is charged at an inflated wage rate. Hence, Labour cost chargeable to job Z = Total hours × Inflated wage rate = 1,125 hrs. × Rs. 11.70 = Rs. 13,162.50

(b) Where overtime is worked irregularly to meet the requirements of production, basic wage rate is charged to the job and overtime premium is charged to factory overheads as under :

Labour cost chargeable to Job Z : 1,125 hours @ Rs. 10 per hour = Rs. 11,250.00 Factory overhead : 100 hrs. × Rs. (17.50 – 10) = Rs. 750.00 25 hrs. × Rs. (22.50 – 10) = Rs. 312.50

Total factory overhead Rs. 1,062.50 (c) Where overtime is worked at the request of the customer, overtime premium is also

charged to the job as under :

Rs. Job Z labour cost 1,125 hrs. @ Rs. 10 = 11,250.00 Overtime premium 100 hrs. @ Rs. (17.50 – 10) = 750.00 25 hrs. @ Rs. (22.50 – 10) = 312.50

Total 12,312.50

3.6 LABOUR TURNOVER Labour turnover in an organisation is the rate of change in the composition of labour force during a specified period measured against a suitable index. The standard of usual labour turnover in the industry or locality or the labour turnover rate for a past period may be taken as the index or norm against which actual turnover rate is compared. There are three methods of calculating labour turnover which are given below :

3.18

Page 153: 30510870 Cost Accounting and Financial Management

Labour

(i) Replacement method = 100roll on employees of number Average

replaced employees of Number×

(ii) Separation method = 100period the during rolls on employees of number Average

year the during separated employees of Number×

(iii) Flux method = 100period the during rolls on employees of number Average

replaced employees of number separated employees of Number×

+

Labour turnover due to new recruitment: Workers joining a business concern on account of its expansion do not account for labour turnover. But these newly recruited workers are certainly responsible for a change in the composition of labour force, due to this feature, some cost accountants measure workers to the extent of new (excluding replacements) joining the labour force as follows :

period a in roll the on workersof number Averagets)replacemen (excluding period a in joining workersnew of No.

× 100

The total number of workers joining, including replacements, are called accessions. The labour turnover rate, in such a case, may also be computed in respect of total number of workers joining (accessions) the business concern, during a given period both on account of replacements and because of expansion is as under :

period a in workersof number Averageperiod a in accessions No.of

× 100

When number of accessions are considered for measuring labour turnover, the labour turnover rate by flux method may be computed by using any one of the following expressions :

100 workersof number Average

tsrecruitmen new of No. tsreplacemen of No. sseparation of No. method) (Flux rate turnover Labour ×++

=

OR

100× workersof number Average accessions of No. + sseparation of No.

The above rate of labour turnover indicates the total effect of number of workers separated, number of workers replaced and number of new workers recruited and joined the concern on account of its expansion, etc. If in the above computations, the data given is for a period other than a year, the labour turnover rate so computed may be converted into equivalent annual labour turnover rate by the following formula:

3.19

Page 154: 30510870 Cost Accounting and Financial Management

Cost Accounting

Equivalent annual labour turnover rate = period the in days of Numberperiod the for rate Turnover

× 365

Causes of labour turnover : The main causes of labour turnover in an organisation/industry can be broadly classified under the following three heads : (a) Personal Causes; (b) Unavoidable Causes; and (c) Avoidable Causes. Personal causes are those which induce or compel workers to leave their jobs; such causes include the following : (i) Change of jobs for betterment. (ii) Premature retirement due to ill health or old age. (iii) Domestic problems and family responsibilities. (iv) Discontent over the jobs and working environment. In all the above cases the employee leaves the organisation at his will and, therefore, it is difficult to suggest any possible remedy in the first three cases. But the last one can be overcome by creating conditions leading to a healthy working environment. For this, officers should play a positive role and make sure that their subordinates work under healthy working conditions. Unavoidable causes are those under which it becomes obligatory on the part of management to ask one or more of their employees to leave the organisation; such causes are summed up as listed below: (i) Seasonal nature of the business; (ii) Shortage of raw material, power, slack market for the product etc.; (iii) Change in the plant location; (iv) Disability, making a worker unfit for work; (v) Disciplinary measures; (vi) Marriage (generally in the case of women). Avoidable causes are those which require the attention of management on a continuous basis so as to keep the labour turnover ratio as low as possible. The main causes under this case are indicated below : (1) Dissatisfaction with job, remuneration, hours of work, working conditions, etc.,

3.20

Page 155: 30510870 Cost Accounting and Financial Management

Labour

(2) Strained relationship with management, supervisors or fellow workers; (3) Lack of training facilities and promotional avenues; (4) Lack of recreational and medical facilities; (5) Low wages and allowances. Proper and timely management action can reduce the labour turnover appreciably so far as avoidable causes are concerned. Effects of labour turnover High labour turnover increases the cost of production in the following ways: (i) Even flow of production is disturbed; (ii) Efficiency of new workers is low; productivity of new but experienced workers is low in

the beginning; (iii) There is increased cost of training and induction; (iv) New workers cause increased breakage of tools, wastage of materials, etc. In some companies, the labour turnover rates are as high as 100%; it means that on the average, all the work is being done by new and inexperienced workers. This is bound to reduce efficiency and production and increases the cost of production. Two types of costs which are associated with labour turnover are : (a) Preventive costs : These include costs incurred to keep the labour turnover at a low level, i.e. cost of medical services, welfare schemes and pension schemes. If a company incurs high preventive costs, the rate of labour turnover is usually low. (b) Replacement costs : These are the costs which arise due to high labour turnover. If men leave soon after they acquire the necessary training and experience of good work, additional costs will have to be incurred on new workers, i.e., cost of employment, training and induction, abnormal breakage and scrap and extra wages and overheads due to the inefficiency of new workers. It is obvious that a company will incur very high replacement costs if the rate of labour turnover is high. Similarly, only adequate preventive costs can keep labour turnover at a low level. Each company must, therefore, work out the optimum level of labour turnover keeping in view its personnel policies and the behaviour of replacement cost and preventive costs at various levels of labour turnover rates. Remedial steps to minimise labour turnover - The following steps are useful for minimising labour turnover. 1. Exit interview : An interview be arranged with each outgoing employee to ascertain the

3.21

Page 156: 30510870 Cost Accounting and Financial Management

Cost Accounting

reasons of his leaving the organisation. 2. Job analysis and evaluation : Before recruiting workers, job analysis and evaluation may

be carried out to ascertain the requirements of each job. 3. Scientific system of recruitment, placement and promotion - The organisation should

make use of a scientific system of recruitment, selection, placement and promotion for employees.

4. Enlightened attitude of management - The management should introduce the following steps for creating a healthy working atmosphere : (i) Service rules should be framed, discussed and approved among management and

workers, before their implementation. (ii) Provide facilities for education and training of workers. (iii) Introduce a procedure for settling worker’s grievances.

5. Use of committee : Issues like control over workers, handling their grievances etc., may be dealt by a committee, comprising of members from management and workers.

Illustration The management of Bina and Rina Ltd. are worried about their increasing labour turnover in the factory and before analyzing the causes and taking remedial steps, they want to have an idea of the profit foregone as a result of labour turnover in the last year. Last year sales amounted to Rs. 83,03,300 and P/V ratio was 20 per cent. The total number of actual hours worked by the Direct Labour force was 4.45 Lakhs. As a result of the delays by the Personnel Department in filling vacancies due to labour turnover, 1,00,000 potentially productive hours were lost. The actual direct labour hours included 30,000 hours attributable to training new recruits, out of which half of the hours were unproductive. The costs incurred consequent on labour turnover revealed, on analysis, the following :

Settlement cost due to leaving Rs. 43,820 Recruitment costs Rs. 26,740 Selection costs Rs. 12,750 Training costs Rs. 30,490 Assuming that the potential production lost as a consequence of labour turnover could have been sold at prevailing prices, find the profit foregone last year on account of labour turnover.

3.22

Page 157: 30510870 Cost Accounting and Financial Management

Labour

Solution Determination of contribution foregone

Actual hours worked (given) 4,45,000 Less : Unproductive training hours 15,000Actual productive hours 4,30,000The potentially productive hours lost are 1,00,000

Sales lost for 1,00,000 hours = hrs 000,30,4300,03,83.Rs × 1,00,000 hrs = Rs. 19,31,000

Contribution lost for 1,00,000 hrs.

=100

000,31,19.Rs × 20 = Rs. 3,86,200...........(i)

Statement showing profit foregone last year on account of labour turnover of Bina and Rina Ltd.

Rs.

Contribution foregone as per (i) 3,86,200 Settlement cost due to leaving 43,820 Recruitment cost 26,740 Selection cost 12,750 Training costs 30,490 Profit foregone 5,00,000

Illustration The Cost Accountant of Y Ltd. has computed labour turnover rates for the quarter ended 31st March, 2007 as 10%, 5% and 3% respectively under ‘Flux method’, ‘Replacement method’ and ‘Separation method’ respectively. If the number of workers replaced during that quarter is 30, find out the number of: (1) workers recruited and joined and (2) workers left and discharged.

3.23

Page 158: 30510870 Cost Accounting and Financial Management

Cost Accounting Solution Working Note: Average number of workers on roll:

100×roll on workersof number Averagetsreplacemen of No.

= methodt replacemen under rate turnover Labour

Or 100

5 =roll on workersof number Average

30

Or Average number of workers on roll = 510030× = 600

(1) Number of workers recruited and joined :

Labour turnover rate = roll on workersof number Average

(A)accessions of No.(S)*sseparation of No. + × 100

(Flux method) (Refer to Working Note)

Or 10010 =

600A*18 +

Or A = ⎥⎦⎤

⎢⎣⎡ −18

1006000 = 42

No. of workers recruited and joined 42.

(2) Number of workers left and discharged :

Labour turnover rate = roll on workersof number Averages(S)separation of No.

× 100

(Separation method) (Refer to working note)

100

3 =600S

Or S* S* = 18

3.24

Page 159: 30510870 Cost Accounting and Financial Management

Labour

Hence, number of workers left and discharged comes to 18.

3.7 INCENTIVE SYSTEM 3.7.1 Important factors necessary for introducing an incentive system : An incentive can be defined as the stimulation for effort and effectiveness by offering monetary inducement or enhanced facilities. It may be monetary in the form of a bonus or non-monetary tending to improve living and working conditions. It may be provided individually or collectively. In the first case, the employee gets a reward for his efforts directly and in the second, a group of employees share the reward arising out of their combined effort in equitable production. The main factors that should be taken into account before introducing a scheme of incentives are stated below : (i) The need for producing goods of high quality or those having very good workmanship or

finish and the manner this can be ensured. Only if a system of quality control can be relied upon to maintain the quality of goods of the standard required, an incentive scheme should be introduced; otherwise, workers should be paid on time basis.

(ii) The need to maximise production—thus required incentives to be given to workers. But sometimes workmanship is more important than quantity of output; in such cases, incentive schemes of wage payment are not suitable.

(iii) Where the quantity of work done cannot be measured precisely, incentive schemes cannot be offered.

(iv) The role of management and workers in achieving greater efficiency, if it is unnecessary for the management to constantly plan work for example, when the work is repetitive, workers should be offered good incentives to achieve high efficiency; but in case management is constantly required to plan the work, as in the case of job work, the management should share the fruits of extra efficiency achieved. This factor determines the choice of a particular incentive scheme.

(v) Whether the quantity of output is within the control of the worker and if so, to what extent. Sometimes, as in the case of chain assembly work the output is not dependent on the effort put in by workers; incentive schemes in such cases are not suitable.

(vi) The exactitude with which standards of performance can be laid down. Fixation of standard is necessary for the introduction of a scheme of incentives. When this requires heavy expenditure, incentive schemes may be rather costly.

(vii) The effect of an incentive scheme for one set of workers on other workers. If for instance, an incentive scheme makes it possible for unskilled workers to earn high wages, the wage rates for skilled workers must also be raised (if they are paid on time basis) to avoid dissatisfaction among them. In that event, the incentive scheme may raise

3.25

Page 160: 30510870 Cost Accounting and Financial Management

Cost Accounting

labour cost instead of lowering it. (viii) The system of wage payment prevailing in other areas and industries or similar

occupations. If possible, there should be uniformity. (ix) The attitude of labour and trade unions towards incentive schemes. Workers usually like

to have a certain guaranteed time-basis wage but also like to earn extra through an incentive scheme.

On the whole, the system of wage payment should be such as would increase production without lowering quality. This will increase the surplus and will enable the employer to pay higher wages which, in turn, will lead to higher output. 3.7.2 Main principles for a sound system of wage incentive : The objective of wage incentives is to improve productivity and increase production so as to bring down the unit cost of production. In order to make the incentive scheme effective and useful, the following general principles have to be considered while designing a sound system of wage incentives. (i) The reward for a job should be linked with the effort involved in that job and the scheme

should be just and fair to both employees and employers. This involves the following : (a) The standard required of the workers should be carefully set, if possible through

proper time and motion studies. (b) If the work is of repetitive type, the entire benefit of the time saved should be

available to the worker but, in the case of non-standardised work or where precise standards cannot be set, the benefit of the time saved, if any, should be shared by the employer, the supervisor and the worker.

(ii) The scheme should be clearly defined and be capable of being understood by the employees easily. The standards set should be such that they can be achieved even by average employees. While standards are being set, the workers concerned should be consulted.

(iii) As far as possible, no limit should be placed on the amount of additional earnings, otherwise it will dampen the initiative of the workers. In this regard, what is important is not what actually prevails but what the workers think—if they think, even wrongly, that the employer will stop wages from rising beyond a certain limit, the incentive scheme may not be really effective.

(iv) The scheme should be reasonable and stable, and should not be changed or modified too often without consulting the employees.

(v) The scheme should take care that the employees are not penalised for reasons beyond their control.

(vi) The scheme should provide for inspection of output so that only good pieces qualify for

3.26

Page 161: 30510870 Cost Accounting and Financial Management

Labour

incentives. It would even be better not to introduce any incentive scheme if workmanship is of vital importance in sales.

(vii) The management should ensure that there is no cause for complaint by the workers that they are sitting idle, say for want of tools or materials. Management has to see that there is, as far as practicable, no interruption of production.

(viii) The operation of the scheme should not entail heavy clerical costs. In fact the scheme should facilitate the introduction of budgetary control and standard costing.

(ix) It should be capable of improving the morale of the employees and it should be in conformity with the local trade union agreements and other government regulations.

(x) There should be a guaranteed wage on time basis which generally works as a good psychological boost to incentive scheme.

(xi) Last, but not least, the effect of incentive scheme on those who cannot be covered should be gauged and taken note of. Sometimes, highly skilled workers have perforce to be paid on time basis whereas semiskilled or unskilled workers may be put on incentive scheme. If the latter earn more than former, the incentive schemes on the whole prove harmful.

3.7.3 Essential characteristics of a good incentive system : to recapitulate (i) It should be just both to the employer and to the employee. It should be positive and not

unnecessarily punitive and so operated as to promote confidence. (ii) It should be strong both ways i.e. it should have a standard task and a generous return.

The latter should be in direct proportion to employee’s efforts. It should reflect the employer’s contribution to the success of the company.

(iii) It should be unrestricted as to the amount of the earning. (iv) It should be reasonable, apart from being simple, for employee to figure out his incentive

in relation to his individual performance, as far as practicable. (v) It should be flexible and intimately related to other management controls. (vi) It should automatically assist supervision and, when necessary, aid team work. (vii) It should have employee’s support and in no way should it be paternalistic. (viii) It should have managerial support in so far as production material, quality control,

maintenance and non-financial incentives are concerned. (ix) It should not be used temporarily and dropped in recission times as means of wage

reduction. 3.7.4 Procedure for laying down an incentive system : An incentive is a reward for effort made; hence correct measurement of the effort involved is a prerequisite for any incentive

3.27

Page 162: 30510870 Cost Accounting and Financial Management

Cost Accounting system. Measurement of effort is made by time and motion study by specialists appointed for the purpose. Based on its finding and that of job evaluation, the rates of wages are fixed for different operations. The levels of efficiency that must be attained to qualify for incentives are then fixed on a consideration of the factors mentioned above. Having drawn up the broad outlines of the scheme, the next step is to educate the workers as regards the benefits of the proposed scheme. This is done through joint consultation with the leading employees or with union representatives. The scheme is then publicised extensively with the specimen, calculations of the rewards that would arise under it. After the basic scheme has been accepted by all, a decision on two vital points will have to be made viz. how spoiled work will be treated and the intervals at which payment of wages will be made. An incentive system tends to increase the rate of production and consequently increases spoilage. But the very purpose of the scheme would be defeated if spoilage increases beyond a certain limit. It is, therefore, necessary that the method of treating the spoiled work should be agreed upon in advance. The prevention of spoiled work can be encouraged either by making the worker do the job again in his own time or by paying the worker at the time rate for the period covered by spoiled work not giving him credit at all for the spoiled work; of these, the first method is more commonly employed since it is both equitable and deterrent. The incentive should be paid promptly at short intervals of time. This would give the worker immediate satisfaction of having earned something by the extra effort he had put in. If payment is delayed the effect of the incentive would be greatly diminished.

3.8 LABOUR UTILISATION For identifying utilisation of labour a statement is prepared (generally weekly) for each department / cost centre. This statement should show the actual time paid for, the standard time (inlcuding normal idle time) allowed for production and the abnormal idle time analysed for causes thereof. 3.8.1 Distinction between Direct and Indirect Labour Cost : Any labour cost that is specifically incurred for or can be readily charged to or identified with a specific job, contract,. work order or any other unit of cost, is termed as direct labour cost; it includes : (i) all labour that is engaged in converting raw materials into manufactured articles in the case of manufacturing industries, and (ii) other forms of labour, which although not immediately engaged in converting raw materials into manufactured articles nonetheless is incurred wholly or specifically for any particular unit of production and hence, can be readily identified with the unit of production (Example: A helper attending solely a machine operator in the case of manufacturing industries ; the entire contingent of labour and staff employed on a construction job or project). Any labour that does not meet the above test is indirect, e.g. men generally employed in machine shop such as tool setters, fitters, workers in tool room, stores, etc. Their wages are charged as indirect expenses.

3.28

Page 163: 30510870 Cost Accounting and Financial Management

Labour

Page – 26 on Landscape file Blank page for insertion

3.29

Page 164: 30510870 Cost Accounting and Financial Management

Cost Accounting Thus direct labour is for a specific job or product while indirect labour is for work in general. In a printing press, for example, wages paid to compositors will be direct while wages paid to the cleaners of machines will be indirect. The distinction is one relative to each particular firm or industry. Labour which is direct in one unit may indeed be indirect in another where the work or process or method of manufacture is different in nature. The importance of the distinction lies in the fact that whereas direct labour can be identified with and charged to the job, indirect labour cannot be so charged and has therefore to be treated as part of the factory overheads to be included in the cost of production on some suitable basis of apportionment and absorption. 3.8.2 Identification of utilisation of labour with cost centres : For the identification of utilisation of labour with the cost centre a wage analysis sheet is prepared. Wage analysis sheet is a columnar statement in which total wages paid are analysed according to cost centre, jobs, work orders etc. The data for analysis is provided by wage sheet, time card, piece work cards and job cards. The preparation of such sheet serves the following purposes : (i) It analyse the labour time into direct and indirect labour by cost centres, jobs, work

orders. (ii) It provides details of direct labour cost comprises of wages, overtime to be charged as

production cost of cost centre, jobs or work orders. (iii) It provides information for treatment of indirect labour cost as overhead expenses.

Wage Analysis Sheet No. week- Department/ Total Work in Factory Administration Selling & ending cost centre progress Overhead Overhead control Distribution control A/c A/c overhead control Account

3.30

Page 165: 30510870 Cost Accounting and Financial Management

Labour

ABC & CO. Wages Analysis Sheet

(Sharing break–up of work in progress) Job No. Job No. Job No. Summary Cost 1 2 3 Ledger Folio

Clock No. Hrs. Amount Clock No. Hrs. Amount Clock No. Hrs. Amount Job No. Hrs. Amount Total Total 3.8.3 Identification of labour hours with work order or batches or capital job : For identification of labour hours with work order or batches or capital jobs or overhead work orders the following points are to be noted : (i) The direct labour hours can be identified with the particular work order or batches or

capital job or overhead work orders on the basis of details recorded on source document such as time sheet or job cards.

(ii) The indirect labour hours cannot be directly identified with the particular work order or batches or capital jobs or overhead work orders. Therefore, they are traced to cost centre and then assigned to work order or batches or capital jobs or overhead work orders by using overhead absorption rate.

3.9 SYSTEMS OF WAGE PAYMENT AND INCENTIVES There exist several systems of employee wage payment and incentives, which can be classified under the following heads :

3.31

Page 166: 30510870 Cost Accounting and Financial Management

Cost Accounting

SYSTEMS OF WAGE PAYMENT & INCENTIVES

Time Rate Systems Payment by Results Time High Measured Diff- Piece Combina- Premium Group System of Rate Wage Day eren- Work tion of Bonus System Incentive System Plan Work tial Time and Methods of wage Schemes Time Piece payment for Rate Work Indirect Workers Straight Differential Halsey Rowan Barth Piece Work Piece Work and Halsey System System Weir Systems Taylor Merrick System System Gantt Emerson’s Points Accelerated Task and Efficiency Scheme Premium System Bonus System System

Beadaux Syst. Haynes Syst.

One should remember that Provident Fund, Employees State Insurance Corporation Premium and bonus are payable on the basic wages, dearness allowance and value of food concession. 3.9.1 Time rate system : It is perhaps the oldest system of remunerating labour. It is also known by other names such as time work, day work, day wages and day rates. Under this system, the worker is paid by the hour, day, week, or month. The amount of wages due to a worker are arrived at by multiplying the time worked (as shown by the gate card) by the appropriate time rate. The time rate here is fixed after taking into account the rates relevant in the particular industrial locality for similar trade and skill. The rate may be either fixed or may be a progressive one, starting from a minimum and rising upto a maximum, in stages, through periodical increments.

3.32

Page 167: 30510870 Cost Accounting and Financial Management

Labour

Merits : (i) Simple to understand and to calculate wages. (ii) Reduces temptation on the part of workers to increase the output at the cost of

quality. Demerits: (i) No monetary incentive to raise the level of production. (ii) No distinction between the slow and the efficient worker. (iii) The tendency is for the fall in output; this raises the cost per unit (because both

labour and fixed expenses will be spread over a smaller number of units). (iv) A firm cannot be sure of labour costs per unit under this method and, hence, may

suffer a loss on quotations if already submitted. In the following cases, time rates are to be preferred. (1) Persons whose services cannot be directly or tangibly measured, e.g., general helpers,

supervisory and clerical staff etc. (2) Workers engaged on highly skilled jobs or rendering skilled services, e.g., tool making,

inspection and testing. (3) Where the pace of output is independent of the operator, e.g., automatic chemical plants. 3.9.2 High wage plan : This plan was first introduced by Ford Motor Company (in USA) in order to induce workers to exercise extra effort in their work. Under this plan a worker is paid a wage rate which is substantially higher than the rate prevailing in the area or in the industry. In return, he is expected to maintain a very high level of performance, both quantitative and qualitative. As a result, high rate men are not as costly or expensive as they might appear at first sight. High wage plan is suitable where high quality of work and also increased productivity are required. The advantage which may accrue from the implementation of this plan are : (1) It is simple and inexpensive to operate.

(2) It helps in attracting highly skilled and efficient workers by providing suitable incentive.

(3) It reduces the extent of supervision.

(4) Increased productivity may result in reduction of unit labour cost. 3.9.3 Measured day work : According to this method the hourly rate of the time worker consists of two parts viz, fixed and variable. The fixed element is based on the nature of the job i.e. the rate for this part is fixed on the basis of job requirements. The variable portion varies for each worker depending upon his merit rating and the cost of living index. The aggregate of fixed and variable part for a day is termed as Measured day’s work rate of a worker.

3.33

Page 168: 30510870 Cost Accounting and Financial Management

Cost Accounting As the rate is based on two different elements, there are separate time rates not only for each worker but also for each job. This method does not find much favour with workers due to the following : 1. The rates fixed are not easily understood by the workers. 2. Merit rating tends to be arbitrary and unless changed at rapid intervals, the ratings will

not reflect the correct ranking of the qualities of a worker. 3.9.4 Differential time rate : According to this method, different hourly rates are fixed for different levels of efficiency. Up to a certain level of efficiency the normal time or day rate is paid. Based on efficiency level the hourly rate increases gradually. The following table shows different differential rates :

Up to, say 75% efficiency Normal (say Rs. N per hr.) From 76% to 80% efficiency 1.10 × N From 81% to 90% efficiency 1.20 × N From 91% to 100% efficiency 1.30 × N From 101% to 120% efficiency 1.40 × N

As this method is linked with the output and efficiency of workers, therefore, it cannot be strictly called as a time rate method of wage payment. This method in fact is similar to differential piece work system. 3.9.5 Payment by result : Under this system the payment made has a direct relationship with the output given by a worker. The attendance of the worker or the time taken by him for doing a job has no bearing on the payment. The system of payment by results may be classified into the following four categories : (a) Systems in which the payment of wages is directly proportionate to the output given by

workers. (b) Systems in which the proportion of the wage payment to the worker increases

progressively with increase in production. (c) Systems in which payment rate decreases with the increase in output. (d) Systems with earnings varying in proportions which differ at different levels of production. 3.9.6 Straight piece work system : Under this system of wage payment, each operation, job or unit of production is termed a piece. A rate of payment, known as the piece rate or piece work rate is fixed for each piece. The wages of the worker depend upon his output and rate of each unit of output; it is in fact independent of the time taken by him. The wages paid to a worker are calculated as :

3.34

Page 169: 30510870 Cost Accounting and Financial Management

Labour

Wages = Number of units produced × Piece rate per unit. Considerable care and judgment are called for fixing the piece rate. If the rate fixed is too high or too low, it would operate to the disadvantage of either the employer or the employee. Any attempt on the part of the management to revise a piece rate, erroneously set too high, is likely to lead to friction and conflict with labour. If on the other hand, it is too low, it would fail in its objective. The only way all this may be avoided is by employing scientific methods of job evaluation and time and motion study for the purpose of setting the rates. Advantages : 1. The system is simple to operate and also easy to understand. 2. The incentive provided is quite effective as the workers get the full benefit of any

increase in production and the employer also gains by saving on overhead costs. 3. Labour cost per unit being constant, these can be calculated in advance and quotations

can be confidently submitted. Disadvantages : 1. The quality of output usually suffers. 2. Maintenance of detailed statistics as regards production of individual workers is

necessary. 3. Maintenance of satisfactory discipline in the matter of arrival and departure of workers

becomes somewhat difficult. 4. In the anxiety to produce as large a quantity as possible, workers may damage the

machines and may also increase wastage of materials. 5. Skilled workers and supervisors (who are often paid on time basis) may resent higher

wages to unskilled workers paid on the piece basis. 3.9.7 Differential piece work system : This system provide for higher rewards to more efficient workers. The main feature of all differential piece-work systems is that several piece rates on a slab scale are fixed for a job or operation which is put on piece-work. For different levels of output below and above the standard, different piece rates are applicable. Taylor Dif-ferential Piece Work System and Merrick Differential Piece Rate System are two important differential piece work systems discussed briefly as below: (a) Taylor’s differential piece work system - The Taylor’s Differential Piece Rate System aims at rewarding efficient workers by providing increased piece rate beyond certain level of output. Under this system two widely differing piece-rates are prescribed for each job. The lower rate is 83% of the normal piece rate and the higher rate is 125% of the normal piece rate. In other words the higher rate is 150% of the lower rate. The lower rate is given to a

3.35

Page 170: 30510870 Cost Accounting and Financial Management

Cost Accounting worker when his efficiency level is less than 100%. The higher rate is offered at efficiency level of either 100% or more. Due to the existence of the two piece rates, the system is known as differential piece rate system. Note: Some authors also use 80% and 120% of the piece rates as lower and higher rates respectively at the efficiency levels, as indicated in the above paragraph. Advantages: 1. It is simple to understand and operate. 2. The incentive is very good and attractive for efficient workers. 3. It has a beneficial effect where overheads are high as increased production has the

effect of reducing their incidence per unit of production. This system is quite harsh to workers, as a slight reduction in output may result in a large reduction in the wages earned by them. This system is no longer in use in its original form, though the main idea behind it is used in many wage schemes.

Illustration : Using Taylor’s differential piece rate system, find the earnings of the Amar, Akbar and Ali from the following particulars:

Standard time per piece : 20 minutes Normal rate per hour : Rs. 9.00 In a 8 hour day Amar produced : 23 units Akbar produced : 24 units Ali produced : 30 units

Solution Earnings under Differential piece rate system Workers Amar Akbar Ali Standard output per day (units) 24 24 24 Actual output per day (units) 23 24 30 Efficiency (%) 95.83% 100% 125%

100×output Standaradoutput Actual

100×unit 24unit 23

100×unit 24unit 24

100×unit 24unit 30

3.36

Page 171: 30510870 Cost Accounting and Financial Management

Labour

* Earning rate per unit 83% of 125% of 125% of the piece the piece the piece rate rate rate Earning rate per unit (Rs.) 2.49 3.75 3.75 (Refer to working note) (83% of Rs.3) (125% of Rs.3) (125% of Rs.3) Earnings (Rs.) 57.27 90.00 112.50 (23 units × Rs. 2.49) ( 24 units × Rs. 3.75) (30 units × Rs. 3.75) * Under Taylor’s Differential price rate system, two widely differing price rates are prescribed for each job. The lower rate is 83% of the normal piece rate and is applicable if efficiency of the worker is below 100%. The higher piece rate is 125% of the normal piece rate and is applicable if work completed is at efficiency level of 100% and above. Working Note: Normal rate per hour = Rs. 9.00

Normal rate per unit = hour per production StandardRs.9.00

= units 3

00.9.Rs

= Rs. 3 (b) Merrick differential piece rate system - Under this system three piece rates for a job are fixed. None of the fixed rates is below the normal. These three piece rates are as below: Efficiency Piece rate applicable

Upto 83% Normal rate, Above 83% and upto 100% 10% above normal rate. Above 100% 20% or 30% above normal rate. This system is an improvement over Taylor’s Differential Piece Rate System.

Illustration Refer to the statement of previous Illustration and compute the earnings of workers under Merrick Differential Piece Rate System.

3.37

Page 172: 30510870 Cost Accounting and Financial Management

Cost Accounting

Solution

Workers Amar Akbar Ali * Earning rate per unit 10% above 10% above 20% above (Refer to previous illustration) the normal the normal the normal rate rate rate or 30% above the normal Earning rate per unit (Rs.) 3.30 3.30 3.60 or 3.90 Earnings (Rs.) 75.90 79.20 108 or 117 (23 units × Rs. 3.30)(24 units × Rs. 3.30)(30 units × Rs. 3.60) or (30 units × Rs. 3.90)

3.9.8 Gantt task and bonus system : This system is a combination of time and piece work system. According to this system a high standard or task is set and payment is made at time rate to a worker for production below the set standard. If the standards are achieved or exceeded, the payment to the concerned worker is made at a higher piece rate. The piece rate fixed under this system also includes an element of bonus the extent of 20%. The figure of bonus to such workers is calculated over the time rate of the workers. Thus in its essence, the system consists of paying a worker on time basis if he does not attain the standard and on piece basis if he does. Wages payable to workers under this plan are calculated as under : Output Payment (i) Output below standard Guaranteed time rate. (ii) Output at standard Time rate plus bonus of 20% (usually) of time rate. (iii) Output above standard High piece rate on worker’s whole output. It is so fixed, so as to include a bonus of 20% of the time rate. Advantages : 1. It provides good incentive for efficient workers and at the same time protects the less

efficient by guaranteeing the time rate.

3.38

Page 173: 30510870 Cost Accounting and Financial Management

Labour

2. It is simple to understand and operate. 3. It encourages better supervision and planning. Disadvantage The guaranteed time rate may have the effect of weakening the urge of slower worker to increase his output.

Illustration : In a factory the standard time allowed for completing a given task (50 units), is 8 hours. The guaranteed time wages are Rs. 20 per hour. If a task is completed in less than the standard time, the high rate of Rs. 4 per unit is payable. Calculate the wages of a worker, under the Gantt system, if he completes the task in (i) 10 hours; (ii) 8 hours, and (iii) in 6 hours. Also ascertain the comparative rate of earnings per hour under the three situations.

Solution (i) When the worker performs the task in 10 hours, his earnings will be at the time wage rate

i.e. 10 hours × Rs. 20 per hour = Rs. 200. (ii) When the worker performs the task is standard time i.e. in 8 hours, his earning will be: 8 hours × Rs. 20 = Rs. 160 Bonus @ 20% of time wages = Rs. 32 Total earnings Rs. 192 (iii) When the worker performs the task in less than the standard time his earning will be at

piece rate i.e. 50 units × Rs. 4 per hour = Rs. 200

The comparative rate of earnings per hour under the above three situations is: (i) Rs. 200/10 hrs. = Rs. 20 per hour (ii) Rs. 192/8 hrs. = Rs. 24 per hour (iii) Rs. 200/6 hrs. = Rs. 33.33 per hour

3.9.9 Emerson’s efficiency system : Under this system minimum time wages are guaranteed. But beyond a certain efficiency level, bonus in addition to minimum day wages is given. A worker who is able to attain efficiency, measured by his output equal to 2/3rd of the

3.39

Page 174: 30510870 Cost Accounting and Financial Management

Cost Accounting standard efficiency, or above, is deemed to be an efficient worker deserving encouragement. The scheme thus provides for payment of bonus at a rising scale at various levels of efficiency, ranging from 66.67% to 150%. For a performance below 66.67% only time rate wages without any bonus are paid. Above 66⅔% to 100% efficiency, bonus varies between 0.01% and 20%. Above 100% efficiency bonus of 20% of basic wages plus 1% for each 1% increase in efficiency is admissible. This system is superior to one to the differential piece rate in so far as it encourages the slow worker to do a little better than before. Also it does not pre-suppose a high degree of average performance. Wages on time basis are guaranteed.

Illustration From the following information you are required to calculate the bonus and earnings under Emerson Efficiency System. The relevant information is as under:

Standard working hours : 8 hours a day Standard output per hour in units : 5 Daily wage rate : Rs. 50 Actual output in units

Worker A 25 units Worker B 40 units Worker C 45 units

Solution Statement showing bonus and earnings under Emerson Efficiency System

Workers A B C Actual output in units 25 40 45 Standard output in units 40 40 40 Efficiency level (%) 62.5% 100% 112.50%

⎥⎦

⎤⎢⎣

⎡×100

output Standaradoutput Actual

Rate of bonus No bonus 20% 32.50% (20% + 12.5%) Time wages (Rs.) 50 50 50 Bonus (Rs.) Nil 10 16.25

(20% of Rs. 50) (32.5% of Rs. 50) Total earnings (Rs.) 50 60 66.25

3.40

Page 175: 30510870 Cost Accounting and Financial Management

Labour

3.9.10 Points scheme or Bedeaux system : Under this scheme, firstly the quantum of work that a worker can perform is expressed in Bedaux points or B’s. There points represent the standard time in terms of minutes required to perform the job. The standard number of points in terms of minutes are ascertained after a careful and detailed analysis of each operation or job. Each such minute consists of the time required to complete a fraction of the operation or the job, and also an allowance for rest due to fatigue. Workers who are not able to complete tasks allotted to them within the standard time are paid at the normal daily rate. Those who are able to improve upon the efficiency rate are paid a bonus, equal to the wages for time saved as indicated by excess of B’s earned (standard minutes for work done) over actual time. Workers are paid 75% of the time saved.

Illustration Calculate the earnings of worker from the following information under Bedeaux system : Standard time for a product A-30 seconds plus relaxation allowance of 50%. Standard time for a product B-20 second plus relaxation allowance of 50%.

During 8 hour day for Actual output of product for A 500 units. Actual output of product B 300 units Wage rate Rs. 10 per hour

Solution Bedeaux point per unit of product A :

60seconds 15 seconds 30 + =

6045 = 0.75 B’s

Bedeaux point per unit of product B:

60seconds 10+ seconds 15 =

6030 = 0.50 B’s

Total production in terms of B’s: 500 × 0.75 + 300 × 0.50 = 525 B’s Standard B’s (8 hours × 60) = 480 B’s No. of B’s saved (525 B’s – 480 B’s) = 45 B’s

Earnings = Hrs. worked × rate per hour + 75/100 × 6045 × Rs. 10 = Rs. 80 + Rs. 5.63 = Rs. 85.63

3.41

Page 176: 30510870 Cost Accounting and Financial Management

Cost Accounting 3.9.11 Hayne’s system : Under this system also the standard is set in minutes. The standard time for the job is expressed in terms of the standard man-minutes called as “MANIT”. Manit stands for man-minute. In the case of repetitive work the time saved is shared between the worker and the foreman in the ratio 5 : 1. If the work is of non-repetitive nature, the worker, the employer and the foreman share the value of time saved in the ratio of 5 : 4 : 1. Each worker is paid according to hourly rate for the time spent by him on the job. 3.9.12 Accelerated premium system : Under this system earnings increase with output; the rate of increase of earnings itself increases progressively with output; in fact the earnings in-crease in greater proportion than the increase in production. This system acts as a strong incentive for skilled workers to earn high wages by increasing output and for production beyond standard. 3.9.13 Premium bonus methods : Under these methods, standard time is established for performing a job. The worker is guaranteed his daily wages (except in Barth System), if his output is below and upto standard. In case the task is completed in less than the standard time, the saved time is shared between the employee and the employer. There are two types of time-sharing plans in use viz., constant sharing plans and variable sharing plans. 3.9.14 Halsey and Halsey Weir systems : Under Halsey system a standard time is fixed for each job or process. If there is no saving on this standard time allowance, the worker is paid only his day rate. He gets his time rate even if he exceeds the standard time limit, since his day rate is guaranteed. If, however, he does the job in less than the standard time, he gets a bonus equal to 50 percent of the wages of time saved; the employer benefits by the other 50 percent. The scheme also is sometimes referred to as the Halsey fifty percent plan. Formula for calculating wages under Halsey system = Time taken × Time rate + 50% of time saved × Time rate. The Halsey Weir System is the same as the Halsey System except that the bonus paid to workers is 30% of the time saved. Advantages: 1. Time rate is guaranteed while there is opportunity for increasing earnings by increasing

production. 2. The system is equitable in as much as the employer gets a direct return for his efforts in

improving production methods and providing better equipment. Disadvantages: 1. Incentive is not so strong as with piece rate system. In fact the harder the worker works,

the lesser he gets per piece. 2. The sharing principle may not be liked by employees.

3.42

Page 177: 30510870 Cost Accounting and Financial Management

Labour

Illustration Calculate the earning of a worker under Halsey System. The relevant data is as below :

Time Rate (p.h.) Re. 0.6 Time allowed 8 hours Time taken 6 hours Time saved 2 hours

Solution Calculation of total earnings : 6 hrs. × Re. 0.6 + 1/2 × (2 hrs. × Re. 0.60) or Rs. 3.60 + Re. 0.60 = Rs. 4.20 Of his total earnings, Rs. 3.60 is on account of the time worked and Re. 0.60 is on account of his share of the premium bonus.

3.9.15 Rowan system : According to this system a standard time allowance is fixed for the performance of a job and bonus is paid if time is saved. Under Rowan System the bonus is that proportion of the time wages as time saved bears to the standard time. Formula for calculating wages under Rowan system

= Time taken × Rate per hour +allowed TimeSaved Time × Time taken × Rate per hour

Advantages :

1. It is claimed to be a fool-proof system in asmuch as a worker can never double his earnings even if there is bad rate setting.

2. It is admirably suitable for encouraging moderately efficient workers as it provides a better return for moderate efficiency than under the Halsey Plan.

3. The sharing principle appeals to the employer as being equitable. Disadvantages:

1. The system is a bit complicated.

2. The incentive is weak at a high production level where the time saved is more than 50% of the time allowed.

3. The sharing principle is not generally welcomed by employees.

3.43

Page 178: 30510870 Cost Accounting and Financial Management

Cost Accounting

Illustration Calculate the earnings of a worker under Rowan System. The relevant data is given as below:

Time rate (per Hour) Re. 0.6 Time allowed 8 hours. Time taken 6 hours. Time saved 2 hours.

Solution Calculation of total earnings:

= Time taken × Rate per hour + allowed TimeSaved Time × Time taken × Rate per hour

= 6 hours × Rs. 0.60 + hours 8hours 2 × 6 hours × 0.60

= Rs. 3.60 + Rs. 0.90 = Rs. 4.50 3.9.16 Barth system : The formula used for calculating the remuneration under this system is as follows :

Earnings = Hourly rate × workedHours × hours Standard

The system is particularly suitable for trainees and beginners and also for unskilled workers. The reason is that for low production efficiency, the earnings are higher than in the piece-work system but as the efficiency increases, the rate of increase in the earnings falls.

Illustration A factory having the latest sophisticated machines wants to introduce an incentive scheme for its workers, keeping in view the following: (i) The entire gains of improved production should not go to the workers. (ii) In the name of speed, quality should not suffer. (iii) The rate setting department being newly established are liable to commit mistakes. You are required to devise a suitable incentive scheme and demonstrate by an illustrative numerical example how your scheme answers to all the requirements of the management.

Solution

3.44

Page 179: 30510870 Cost Accounting and Financial Management

Labour

Rowan Scheme of premium bonus (variable sharing plan) is a suitable incentive scheme for the workers of the factory. If this scheme is adopted, the entire gains due to time saved by a worker will not pass to him. Another feature of this scheme is that a worker cannot increase his earnings or bonus by merely increasing its work speed. The reason for this is that the bonus under Rowan Scheme is maximum when the time taken by a worker on a job is half of the time allowed. As this fact is known to the workers, therefore, they work at such a speed which helps them to maintain the quality of output too. Lastly, Rowan System provides a safeguard in the case of any loose fixation of the standards by the rate-setting department. It may be observed from the following illustration that in the Rowan Scheme the bonus paid will be low due to any loose fixation of standards. Workers cannot take undue advantage of such a situation. The above three features of Rowan Plan can be discussed with the help of the following illustration:

(i) Time allowed = 4 hours Time taken = 3 hours Time saved = 1 hour Rate = Rs. 5 per hour

Bonus = allowed Timetaken Time × Time saved × Rate

= hours 4hours 3 × 1 hour × Rs. 5 = Rs. 3.75

In the above illustration time saved is 1 hour and, therefore, total gain is Rs. 5. Out of Rs. 5 according to Rowan Plan only Rs. 3.75 is given to the worker in the form of bonus and the remaining Rs. 1.25 remains with the management. In other words a worker is entitled for 75 percent of the time saved in the form of bonus.

(ii) The figures of bonus in the above illustration when the time taken is 2 hours and 1 hour respectively are as below:

Bonus = allowed Timetaken Time × Time saved × Rate

= hours 4hours 2 × 2 hours × Rs. 5 = Rs. 5

3.45

Page 180: 30510870 Cost Accounting and Financial Management

Cost Accounting

= hours 4hours 1 × 3 hours × Rs. 5 = Rs. 3.75

The above figures of bonus clearly show that when time taken is half of the time allowed, the bonus is maximum. When the time taken is reduced from 2 to 1 hour, the bonus figure fell by Rs. 1.25. Hence, it is quite apparent to workers that it is of no use to increase speed of work. This feature of Rowan Plan thus protects the quality of output.

(iii) If the rate-setting department erroneously sets the time allowed as 10 hours instead of 4 hours, in the above illustration; then the bonus paid will be as follows:

Bonus = hours 10hours 3 × 7 hours × Rs. 5 = Rs. 10.50

The bonus paid for saving 7 hours thus is Rs. 10.50 which is approximately equal to the wages of 2 hours. In other words the bonus paid to the workers is low. Hence workers cannot take undue advantage of any mistake committed by the time setting department of the concern.

Illustration (a) Bonus paid under the Halsey Plan with bonus at 50% for the time saved equals the

bonus paid under the Rowan System. When will this statement hold good ? (Your answer should contain the proof).

(b) The time allowed for a job is 8 hours. The hourly rate is Rs. 8. Prepare a statement showing: (i) The bonus earned (ii) The total earnings of labour and (iii) Hourly earnings.

Under the Halsey System with 50% bonus for time saved and Rowan System for each hour saved progressively.

Solution (a) Bonus under Halsey Plan = Standard wage rate × 50/100 × Time saved ...(i) Bonus under Rowan Plan

= Standard wage rate ×allowed Timetaken Time × Time taken ...(ii)

3.46

Page 181: 30510870 Cost Accounting and Financial Management

Labour

Bonus under Halsey Plan will be equal to the Bonus under Rowan Plan when the following condition holds good = Standard wage rate × 50/100 × Time saved

= Standard wage rate × allowed Timetaken Time × Time saved

or 21 =

allowed Timetaken Time

or Time taken = 21 of time allowed.

Hence, when the time taken is 50% of the time allowed, the bonus under Halsey and Rowan Plans is equal.

(b) Statement of Bonus, Total earnings of Labour and hourly earnings under Halsey and Rowan Systems

Time Time Time Basic Bonus Bonus Total Total Hourly Hourly allowed taken saved Wages under under earning earning earning earning Halsey Rowan under under under under System system Halsey Rowan Halsey Rowan System System System System

B × Rs. 8 C ×10050

× Rs. 8 AC

× B × Rs. 8 D+E

A B C =(A-B) D E F G H I J

hours hours hours Rs. Rs. Rs. Rs. Rs. Rs. Rs. 8 8 - 64 - - 64 64 8.00 8.00 8 7 1 56 4 7 60 63 8.57 9.00 8 6 2 48 8 12 56 60 9.33 10.00 8 5 3 40 12 15 52 55 10.40 11.00 8 4 4 32 16 16 48 48 12.00 12.00 8 3 5 24 20 15 44 39 14.67 13.00 8 2 6 16 24 12 40 28 20.00 14.00 8 1 7 8 28 7 36 15 36.00 15.00

Illustration Mr. A. is working by employing 10 skilled workers. He is considering the introduction of some

3.47

Page 182: 30510870 Cost Accounting and Financial Management

Cost Accounting incentive scheme - either Halsey Scheme (with 50% bonus) or Rowan Scheme - of wage payment for increasing the labour productivity to cope with the increased demand for the product by 25%. He feels that if the proposed incentive scheme could bring about an average 20% increase over the present earnings of the workers, it could act as sufficient incentive for them to produce more and he has accordingly given this assurance to the workers. As a result of the assurance, the increase in productivity has been observed as revealed by the following figures for the current month :

Hourly rate of wages (guaranteed) Rs. 2.00 Average time for producing 1 piece by one worker at the previous performance 2 hours (This may be taken as time allowed) No. of working days in the month 25 No. of working hours per day for each worker 8 Actual production during the month 1,250 units

Required : 1. Calculate effective rate of earnings per hour under Halsey Scheme and Rowan Scheme. 2. Calculate the savings to Mr. A in terms of direct labour cost per piece under the

schemes. 3. Advise Mr. A about the selection of the scheme to fulfil his assurance.

Solution Working Notes: 1. Total time wages of 10 workers per month : = No. of working days in the month × No. of working hours per day of each worker ×

Hourly rate of wages × No. of workers = 25 days × 8 hrs. × Rs. 2 × 10 workers = Rs. 4,000 2. Time saved per month : Time allowed per piece by a worker 2 hours No. of units produced during the month by 10 workers 1,250 pieces Total time allowed to produce 1,250 pieces 2,500 hours (1,250 × 2 hours)

3.48

Page 183: 30510870 Cost Accounting and Financial Management

Labour

Actual time taken to produce 1,250 pieces 2,000 hours Time saved (2,500 hours – 2,000 hours) 500 hours 3. Bonus under Halsey scheme to be paid to 10 workers : Bonus = (50% of time saved) × hourly rate of wages = 50/100 × 500 hours × Rs. 2 = Rs. 500 Total wages to be paid to 10 workers are (Rs. 4,000 + Rs. 500) Rs. 4,500, if Mr. A

considers the introduction of Halsey Incentive Scheme to increase the labour productivity.

4. Bonus under Rowan Scheme to be paid to 10 workers :

Bonus = allowed Timetaken Time × Time wages = × Rs. 4,000 = Rs. 800

Total wages to be paid to 10 workers are (Rs. 4,000 + Rs. 800) Rs. 4,800, if Mr. A considers the introduction of Rowan Incentive Scheme to increase the labour productivity.

1. (i) Effective hourly rate of earnings under Halsey scheme : (Refer to Working Notes 1, 2 and 3)

= workedhours TotalschemeHalsey under bonus Total+ workers10 of wagestime Total

= hours 2,000Rs.500+Rs.4,000

= Rs. 2.25

(ii) Effective hourly rate of earnings under Rowan scheme : (Refer to Working Notes 1, 2 and 4)

= workedhours Totalscheme Rowan under bonus Total+ workers10 of wagestime Total

= hours 2,000Rs.800+Rs.4,000

= Rs. 2.40

2. (i) Saving in terms of direct labour cost per piece under Halsey scheme : (Refer to Working Note 3) Labour cost per piece (under time wage scheme) = 2 hours × Rs. 2 = Rs. 4.

3.49

Page 184: 30510870 Cost Accounting and Financial Management

Cost Accounting Labour cost per piece (under Halsey scheme)

= produced units of number Totalscheme the under paid wagesTotal

= 250,1

500,4.Rs = Rs. 3.60

Saving per piece : (Rs. 4 – Rs. 3.60) = Re. 0.40. (ii) Saving in terms of direct labour cost per piece under Rowan Scheme : (Refer to Working Note 4) Labour cost per piece under Rowan scheme = Rs. 4,800/1,250 units = Rs. 3.84 Saving per piece = Rs. 4 – Rs. 3.84 = Re. 0.16. 3. From the labour cost per piece under Halsey scheme (Rs. 3.60) and Rowan scheme

(Rs. 3.84), it is quite clear that Halsey scheme brings about more saving than Rowan scheme to the concern. But Halsey scheme does not fulfil the assurance given to the workers about 20% increase in their earnings as it secures only 12.5% [500/4,000] × 100 increase.

On the other hand, Rowan scheme secures 20% [800/4,000] × 100 increase in the earnings and it fulfils the assurance. Therefore, Rowan scheme may be adopted. Illustration Wage negotiations are going on with the recognised Labour Union and the Management wants you as the Cost Accountant of the Company to formulate an incentive scheme with a view to increase productivity. The case of three typical workers Achyuta, Ananta and Govida who produce respectively 180, 120 and 100 units of the company’s product in a normal day of 8 hours is taken up for study. Assuming that day wages would be guaranteed at 75 paise per hour and the piece rate would be based on a standard hourly output of 10 units, calculate the earnings of each of the three workers and the labour cost per 100 pieces under (i) Day wages, (ii) Piece rate, (iii) Halsey scheme, and (iv) The Rowan scheme. Also calculate under the above schemes the average cost of labour for the company to produce 100 pieces. Solution Calculation of earnings of each of the three workers and the labour cost per 100 pieces under different wage schemes. (i) Day wages

Name of workers Day Actual Labour wages output cost per (units) 100 pieces

3.50

Page 185: 30510870 Cost Accounting and Financial Management

Labour

Rs. Rs. Achyuta 6.00 180 3.33 Ananta 6.00 120 5.00 Govinda 6.00 100 6.00 Total 18.00 400

Average cost of labour for the company to produce 100 pieces :

output Total

paid wagesTotal × 100 = 400

18.Rs × 100 = Rs. 4.50

(ii) Piece rate

Name of workers Actual Piece Wages Labour output rate earned cost per (units) Rs. Rs. 100 pieces Rs. Achyuta 180 0.075 13.50 7.50 Ananta 120 0.075 9.00 7.50 Govinda 100 0.075 7.50 7.50 Total 400 30.00

Average cost of labour for the company to produce 100 pieces : =400

30.Rs × 100 = Rs.

7.50 (iii) Halsey Scheme

Name of Acutal Std. time Actual Time Bonus Total Labour workers output for time saved Hrs. wages cost per (units) actual for Hrs. (50% including 100 output actual of time Bonus* pieces Hrs. output saved Hrs. Hrs.) Rs. Rs. Achyuta 180 18 8 10 5 9.75 5.42 Ananta 120 12 8 4 2 7.50 6.25 Govinda 100 10 8 2 1 6.75 6.75

3.51

Page 186: 30510870 Cost Accounting and Financial Management

Cost Accounting 24.00

Average cost of labour for the company to produce 100 pieces =400

30.Rs (Rs. 24/400) ×

100 = Rs.6.00 *Total wages = (Actual hours worked + bonus hours) × Rate per hour Hence total wages of Achyuta are: (8 + 5) × Rs. 0.75 = Rs. 9.75 Similarly, the total wages of Ananta and Govinda are Rs. 7.50 and Rs. 6.75 respectively. (iv) Rowan Scheme: Name of Actual Std. time Actual Time Bonus* Wages for Bonus Total Labour workers output for time saved hrs. actual @ 75 P earnings cost (units) actual taken (hours) hrs. per per output hours @ 75 P Bonus 100 (hrs.) per hr. hour pieces Rs. Rs. Rs. Rs

(1) (2) (3) (4) (5) (6) (7) (8) (7)+(8)=(9) (10) Achyuta 180 18 8 10 4.44 6.00 3.33 9.33 5.18 Ananta 120 12 8 4 2.67 6.00 2.00 8.00 6.67 Govinda 100 10 8 2 1.6 6.00 1.20 7.20 7.20 24.53

Average cost of labour to the = 400

53.24.Rs × 100 = Rs. 6.13

company for 100 pieces

*Bonus hours = Time taken × allowed Timesaved Time

Bonus hours of Achyuta = hours 18hours 10×hours 8

= 4.44

Similarly, bonus hours of Ananta and Govinda are 2.67 hours and 1.6 hours respectively. 3.9.17 Group System of wage payment : Certain jobs or operations are required to be performed collectively by a number of workers. Under such cases each man’s work depends on the work performed by one or more of his colleagues and as such it is not possible to measure separately the output of each worker.

3.52

Page 187: 30510870 Cost Accounting and Financial Management

Labour

The workers constituting a group or a team here are considered as a composite unit and the combined output of such a unit is measured for the purpose of wage calculation. The methods usually used for distributing wages to each worker are the following : 1. Equally, if all the workers of the group are of the same grade and skill, same rate of pay

and has worked for same duration. 2. Prorata to the time-rate of each worker where the time spent by the individual worker is

the same. 3. On the basis of the time rates and attendance of each worker. 4. On a specified percentage basis; the percentage applicable to a worker is pre-

determined on the basis of the skill, rate of pay etc. 5. In a group of unskilled and skilled workers, a method of distribution is to pay the unskilled

workers at their time rates. The balance amount remaining out of the total earnings after payment to the unskilled workers is distributed among the skilled workers by any of the methods discussed above.

Group Bonus - Group Bonus refers to the bonus paid for the collective efforts made by a group of workers. The amount of bonus is distributed among the individual members of the group on some agreed basis. Group Bonus Schemes - Under a group bonus scheme, bonus is paid to a team/group of employees working together. Such a scheme is introduced generally where individual efficiency cannot be established for the payment of bonus. For example, in the construction work, it is the team work of masons and labourers which produces results. If any incentive is to be offered, it should be offered to the team as a whole and not to an individual. Group bonus is based on the combined output of the team as a whole. The quantum of bonus is determined on the basis of the productivity of the team and the bonus is shared by individual workers in specified proportions, often in the proportion of wages on time basis. Objectives of Group Bonus Schemes: The objectives of a group bonus scheme are the following :- (i) To create collective interest and team spirit among workers. (ii) To create interest among supervisors to improve performance. (iii) To reduce wastage in materials and idle time. (iv) To achieve optimum output at minimum cost. (v) To encourage individual members of the group, team where only the output of the team

as a whole can be measured. Advantages of Group Bonus Schemes :

3.53

Page 188: 30510870 Cost Accounting and Financial Management

Cost Accounting 1. They create a sense of team spirit since all the workers in a group realise that their

personal incentives are dependent upon group effort. 2. A spirit of healthy competition amongst various groups doing identical jobs is also

created. This results is the elimination of excessive waste of materials. 3. The operators and supervisors also feel interested in raising the production to higher

levels. 4. The scheme is usually easier to understand and entails less costing and accounting

work. It is easier to set up group activity targets, since the performance unit is large. Schemes of group bonus - There are five schemes of group bonus as discussed below : (1) Priestman’s Production Bonus : This method was adopted by Priestman Bros. Ltd., in

1917. According to this method when the actual production in units or points exceeds the standard fixed, a bonus is paid to the workers as additional wages equivalent to the percentage of actual output over the standard output.

(2) Cost Efficiency Bonus : Under this scheme, the amount of bonus is calculated when the cost is reduced below the normal established targets. Targets of cost, for example, material cost, labour cost and overhead cost etc., per unit are fixed. If the measurement of actual performance shows a saving in the total labour and material cost or a reduction in the total cost per unit, a fair percentage of the saving is distributed among the staff. Three popular schemes usually used for calculating the amount to be distributed to workers as Bonus are as below : (i) Nunn-Bush Plan : According to this plan a norm of direct labour cost is fixed and

expressed as a percentage of the sales value. The amount calculated at this percentage is credited to a fund. The actual labour cost is debited to this fund and the balance remaining to the credit of this fund is distributed as bonus to all the workers and employees.

(ii) Scanlon Plan : Here also a fund is created for the normal cost of wages and salaries. This fund is debited with the actual labour costs. Two-thirds to three-fourths of the credit balance, if any, is distributed as bonus, the balance is kept as reserve for future set-backs.

(iii) Rucker Plan : This plan is quite similar to Nunn-Bush Plan except that the percentage for crediting the fund is based on the total value added by manufacturer (i.e. the total cost less the value of the material) and not on total sales value.

(3) Town Gain Sharing Plan : Under this plan bonus is dependent upon a saving in the labour cost as compared to standard. The bonus is calculated at 50% of the saving achieved.

3.54

Page 189: 30510870 Cost Accounting and Financial Management

Labour

(4) Budgeted Expense Bonus : Bonus is determined in advance and paid as a percentage of savings effected in the actual total expenses as compared to the budgeted expense. It is payable to indirect workers also.

(5) Waste Reduction Bonus : Bonus becomes payable under this scheme if the team of workers brings about a reduction in the percentage of material wastages as compared to the standard set. It is applicable to industries where the material cost assumes a greater proportion of the total cost.

Many times group bonus schemes do not enjoy the approval of workers. Some workers tend to feel that their personal incentives are low merely because some members of the group are lazy or inefficient. Such workers believe that it is better to provide incentives on individual basis, if it is possible. 3.9.18 System of Incentive schemes for Indirect Workers: Since the setting of work standards and measurement of output in the case of indirect workers is not an easy task in respect of maintenance, internal transport, inspection, packing and cleaning, therefore the introduction of a system of payment by results for indirect workers is difficult. In spite of the aforesaid difficulty, it has been felt necessary to provide for incentives to indirect workers, due to the following reasons: 1. Payment of incentive bonus to direct workers and time rate to indirect workers leads to

dissatisfaction and labour unrest. 2. Indirect workers are as much entitled to bonus as direct workers. 3. Bonus payment to indirect workers creates team spirit. 4. An incentive system for indirect workers assists in maintaining the efficiency of services

such as plant repairs, stores maintenance, material handling etc. 5. The efficiency of direct workers is reduced where their work is dependent upon the

service rendered by the indirect workers. A few examples of incentive schemes to indirect workers are as below : (i) Incentive to supervisors and foremen : Supervisors and foremen are an important link

between the management and the workers. Incentive payment to these persons would assist in maintaining all round efficiency. Incentive to supervisors and foremen may be provided in the form of non-financial benefits.

Incentive can also be provided to these workers in the form of Bonus. The extent of bonus which will be distributed as incentive will depend on the savings effected over the standards.

(ii) Incentive to maintenance and repairs staff : Under mass production work, repair and maintenance duties can be considered as routine and repetitive for which percentage of

3.55

Page 190: 30510870 Cost Accounting and Financial Management

Cost Accounting

efficiency can be evaluated. In case such an evaluation is not possible or practicable, a group bonus system may be established, on the basis of reduction in breakdown or on the number of complaints.

Illustration A, B and C were engaged on a group task for which a payment of Rs. 725 was to be made. A’s time basis wages are Rs. 8 per day, B’s Rs. 6 per day and C’s Rs. 5 per day. A worked for 25 days; B worked for 30 days; and C for 40 days. Distribute the amount of Rs. 725 among the three workers. Solution Total wages on time basis : Rs.

A 25 @ Rs. 8 200 B 30 @ Rs. 6 180 C 40 @ Rs. 5 200 580 Payment for the task Bonus : (Rs. 725 – Rs. 580) 145 or, 25% of the time-basis wages. 725

Earnings of each worker Worker Wages on time basis Group task Bonus 25% Total Rs. Rs. Rs. A 200 50 250 B 180 45 225 C 200 50 250 580 145 725

Illustration Both direct and indirect labour of a department in a factory are entitled to production bonus in accordance with a group incentive scheme, the outline of which is as follows : (a) For any production in excess of the standard rate fixed at 16,800 tonnes per month (of 28

days) a general incentive of Rs. 15 per tonne is paid in aggregate. The total amount payable to each separate group is determined on the basis of an assumed percentage of such excess production being contributed by it, namely @ 65% by direct labour, @ 15%

3.56

Page 191: 30510870 Cost Accounting and Financial Management

Labour

by inspection staff, @ 12% by maintenance staff and @ 8% by supervisory staff. (b) Moreover, if the excess production is more than 20% above the standard, direct labour

also get a special bonus @ Rs. 5 per tonne for all production in excess of 120% of standard.

(c) Inspection staff are penalized @ Rs. 20 per tonne for rejection by customer in excess of 2% of production.

(d) Maintenance staff are also penalized @ Rs. 20 per hour for breakdown. From the following particulars for a month, work out production bonus earned by each group : (a) Actual working days : 25 (b) Production : 21,000 tonnes (c) Rejection by customer : 500 tonnes (d) Machine breakdown : 40 hours

Solution

1. Standard output per day = month a in days of number Budgeted month peroutput Standard

= days 28 tonnes 16,800

= 600 tonnes

2. Standard output for 25 days = 600 tonnes × 25 days = 15,000 tonnes (a) General Incentive

(i) Standard output : 15,000 tonnes (ii) Actual output : 21,000 tonnes (iii) Excess output over standard : 21,000 – 15,000 = 6,000 tonnes (iv) Percentage of excess : 40%

production to standard output : tonnes 15,000 tonnes 6,00

× 100

(v) Aggregate general incentive : = Excess output × Rs. 15 : = 6,000 tonnes × Rs.15 = Rs. 90,000 (vi) Allocation of general incentive Direct labour : 65% of Rs. 90,000 Rs. 58,500

3.57

Page 192: 30510870 Cost Accounting and Financial Management

Cost Accounting

Inspection staff : 15% of Rs. 90,000 Rs. 13,500 Maintenance staff : 12% of Rs. 90,000 Rs. 10,800 Supervisory staff : 8% of Rs. 90,000 Rs. 7,200 Total Rs. 90,000

(b) Special bonus to direct workers (i) 20% is the excess output over 120% of standard output or 3,000 tonnes (15,000 tonnes × 20%) (ii) Special incentive = 3,000 tonnes × Rs. 5 = Rs. 15,000

(c) Penalty imposed on inspection staff (i) Normal rejection = 2% of production = 2% of 21,000 tonnes = 420 tonnes (ii) Actual rejection = 500 tonnes (iii) Excess rejection over normal rejection = 500 – 420 = 80 tonnes (iv) Penalty = 80 tonnes × Rs. 20 per tonne = Rs. 1,600

(d) Penalty imposed on maintenance staff (i) Breakdown hours = 40 hours (ii) Penalty = 40 hours × Rs. 20 per hour = Rs. 800

Statement of production bonus earned by each group

Particulars Direct Inspection Maintenance Supervisory Total labour staff staff staff (Rs.) (Rs.) (Rs.) (Rs.) (Rs.) 1 2 3 4 5

Aggregate general incentive 58,500 13,500 10,800 7,200 90,000 Special bonus 15,000 - - - 15,000 Penalty - (1,600) (800) - (2,400) Production bonus 73,500 11,900 10,000 7,200 1,02,600

3.9.19 Profit-sharing and Co-partnership schemes - A profit-sharing scheme implies that the net profit of business would be shared between the workers and the shareholders or the partners in a pre-determined ratio. Co-partnership, on the other hand, implies that the workers

3.58

Page 193: 30510870 Cost Accounting and Financial Management

Labour

shall own the business jointly with the shareholders. In this case, usually the workers share of profits is given in the form of shares. Some employers in our country originally introduced profit-sharing schemes with a view of stimulating interest among workers for increasing production. But the schemes have not been successful on account of unwillingness on the part of the management to consult workers. Even a demand for copies of final accounts of the business to be shown to them has been considered by some employers to be an unwarranted interference. The question of bonus has thus been one of the major causes of industrial disputes in recent years. (Payment of compulsory bonus is now governed by the payment of Bonus Act.) Though profit sharing has become a normal feature of the industrial life in this country, co-partnership is comparatively unknown. Nevertheless it must be pointed out that in England and other Western countries, a number of successful concerns have been allotting shares to their workers in proportion to their shares of bonus. Some of them have advanced loans to them to purchase shares. Both these forms of benefit have been quite popular with labour. Advantages : (i) Employees are made to feel that they too have a stake in the well-being of the

undertaking and have a contribution to make in earning of profits by improving production and operations.

(ii) Labour turnover may be reduced, particularly if a minimum period of service is laid down as a condition for participating in such schemes.

Disadvantages : (i) Profit may fluctuate from year to year; there is thus an element of uncertainty in such

schemes. (ii) Profit depends upon many factors of which labour efficiency is only one. Insufficiency of

bonus may lead to dissatisfaction instead of promoting good relations, if the good work done by labour is nullified by other factors.

(iii) The reward may be too remote to sustain continued interest in and zeal for work. (iv) There may be doubt and suspicion about the profit disclosed. (v) Since all are entitled to participate in such schemes, there is no recognition of individual

merit. (vi) The individual share of profit may be too meagre to be appealing. (vii) Since in practice bonus has to be fought in India, so it has become an important cause of

labour disputes. Treatment in Costing : In foreign countries bonus is an ex-gratia payment and hence it is

3.59

Page 194: 30510870 Cost Accounting and Financial Management

Cost Accounting regarded as an appropriation of profit not to be included in costs. In fact trade unions there do not look upon bonus with favour. In India however, payment of bonus is compulsory under the Payment of Bonus Act under which 8.33% of wages earned or Rs. 100 whichever is higher, is the minimum bonus payable, the maximum being 20%. Hence bonus must be treated as part of costs in India. There can be two methods of dealing with bonus. It may be treated as part of overheads; in any case, this must be so for bonus paid to indirect workers. In the case of direct workers the bonus payable may be estimated beforehand and wage rates for costing purposes suitably inflated by including the bonus that would be paid. Suppose, a worker gets Rs. 800 p.m. as wages and it is expected that he will be paid two months’s wages as bonus. His total earning will be Rs. 11,200 (Rs. 9,600 + Rs. 1,600). If the worker works for 2,400 hours in a year the wage rate for costing purposes will be : Rs. 4.67, i.e., Rs. 11,200/2,400 hours.

Illustration A skilled worker in XYZ Ltd. is paid a guaranteed wage rate of Rs. 30 per hour. The standard time per unit for a particular product is 4 hours. P, a machineman, has been paid wages under the Rowan Incentive Plan and he had earned an effective hourly rate of Rs. 37.50 on the manufacture of that particular product. What could have been his total earnings and effective hourly rate, had been put on Halsey Incentive Scheme (50%) ?

Solution Let T hours be the total time worked in hours by the skilled worker (machineman P); Rs. 30/- is the rate per hour; standard time is 4 hours per unit and effective hourly earning rate is Rs. 37.50 then

Earnings = Hours worked × Rage per hour + allowed Timesaved Time

× Time taken × Rate per hour

(Under Rowan incentive plan)

Rs. 37.5 T = T × Rs. 30 + ( )4

T4− × T × Rs. 30

Rs. 37.5 = Rs. 30 + (4 – T) × Rs. 7.5 Or Rs. 7.5 T = Rs. 22.5 Or T = 3 hours Total earnings and effective hourly rate of skilled worker (machineman P) under Halsey Incentive Scheme (50%)

3.60

Page 195: 30510870 Cost Accounting and Financial Management

Labour

Total earnings = Hours worked × Rate per hour + 21 Time saved × Rage per hour

(Under 50% Halsey incentive Scheme)

= 3 hours × Rs. 30 + 21 × 1 hour × Rs. 30 = Rs. 105

Effective hourly rate = allowed Timeearnings Time

= hours 3Rs.105

= Rs. 35

Illustration During audit of account of G Company, your assistant found errors in the calculation of the wages of factory workers and he wants you to verify his work. He has extracted the following information : (i) The contract provides that the minimum wage for a worker is his base rate. It is also paid

for downtimes i.e., the machine is under repair or the worker is without work. The standard work week is 40 hours. For overtime production, workers are paid 150 percent of base rates.

(ii) Straight Piece Work – The worker is paid at the rate of 20 paise per piece. (iii) Percentage Bonus Plan – Standard quantities of production per hour are established by

the engineering department. The workers’ average hourly production, determined from his total hours worked and his production, is divided by the standard quantity of production to determine his efficiency ratio. The efficiency ratio is then applied to his base rate to determine his hourly earnings for the period.

(iv) Emerson Efficiency Plan – A minimum wages is paid for production upto 66-2/3% of standard output or efficiency. When the workers production exceeds 66-2/3% of the standard output, he is paid bonus as per the following table :

Efficiency Level Bonus

Upto 6632 % Nil

Above 6632 % to 79 % 10%

80% – 99% 20% 100% – 125% 45%

3.61

Page 196: 30510870 Cost Accounting and Financial Management

Cost Accounting Your assistant has produced the following schedule pertaining to certain workers of a

weekly pay roll : Workers Wage Incentive Total Down Units Standard Base Gross Plan Hours Time Produced Units Rate Wages Hours as per Book Rs. Rs. Rajesh Straight piece work 40 5 400 — 1.80 85 Mohan* Straight piece work 46 — 455 — 1.80 95 John Straight piece work 44 — 425 — 1.80 85 Harish Percentage bonus plan 40 4 250 200 2.20 120 Mahesh Emerson 40 — 240 300 2.10 93 Anil Emerson 40 — 600 500 2.00 126 (40 hours production) * Total hours of Mohan include 6 overtime hours. Prepare a schedule showing whether the above computation of workers’ wages are

correct or not. Give details. Solution Schedule showing the correct figure of minimum wages, gross wages and wages to be paid Workers Wage Minimum Gross wages Gross wage Wages to be paid incentive wages computed as per as per book are Maximum of : plan Incentive plan minimum and gross computed wages (Rs.) (Rs.) (Rs.) (Rs.)

Rajesh Straight piece work 72.00 80.00 85 80.00 (Refer to W. Note 1) Mohan Straight piece work 88.20 91.00 95 91.00 (Refer to W. Note 2) John Straight piece work 82.80 85.00 85 85.00 (Refer to W. Note 3) Harish Percentage bonus plan 88.00 110.00 120 110.00 (Refer to W. Note 4)

3.62

Page 197: 30510870 Cost Accounting and Financial Management

Labour

Mahesh Emerson 84.00 100.80 93 100.80 (Refer to W. Note 5) Anil Emerson 80.00 116.00 126 116.00 (Refer to W. Note 6)

Working notes : 1. Minimum wages = Total normal hours × rate per hour = 40 hours × Rs. 1.80 = Rs. 72 Gross wages (computed) = No. of units × rate per unit as per incentive plan = 400 units × Rs. 0.20 = Rs. 80 2. Minimum wages = Total normal hours × Rate per hour + Overtime

hours × Overtime rate per hour = 40 hours × Rs. 1.80 + 6 hours × Rs. 2.70 = Rs. 72 + Rs. 16.20 = Rs. 88.20 Gross wages (computed) as per incentive plan = 455 units × Re. 0.20 = Rs. 91.00 3. Minimum wages = 40 hours × Rs. 1.80 + 4 hours × Rs. 2.70 = Rs. 72 + Rs. 10.80 = Rs. 82.80 Gross wages (computed) = 425 units × Rs. 0.20 = Rs. 85 as per incentive plan 4. Minimum wages = 40 hours × Re. 2.20 = Rs. 88

Efficiency of worker = hour per nproductiio Standardhour per production Actual

× 100

= hours) units/40 (200hours) units/40 (250

× 100 = 125%

Hourly rate = Rate per hour × Efficiency of worker = Rs. 2.20 × 125% = Rs. 2.75 Gross wages (computed) as percentage of bonus plan = 40 hours × Rs. 2.75 = Rs. 110/- 5. Minimum wages = 40 hours × Rs. 2.10 = Rs. 84

3.63

Page 198: 30510870 Cost Accounting and Financial Management

Cost Accounting

Efficiency of worker = hours) units/40 (300hours) units/40 (240

× 100 = 80%

Bonus (as per Emerson’s plan) = Total minimum wages × Bonus percentage = Rs. 84 × 20% = Rs. 16.80 Gross wages (computed) as per Emerson’s Efficiency plan = Minimum wages + Bonus = Rs. 84 + Rs. 16.80 = Rs. 100.80 6. Minimum wages = 40 hours × Rs. 2 = Rs. 80

Efficiency of worker = 500600 × 100 = 120%

Bonus (as per Emerson’s plan) = Rs. 80 × 45% = Rs. 36 Gross wages (computed) as per Emerson’s Efficiency plan = Rs. 80 + Rs. 36 = Rs. 116

Illustration The present output details of a manufacturing department are as follows : Average output per week 48,000 units from 160 employees Saleable value of output Rs. 6,00,000 Contribution made by output towards fixed expenses and profit Rs. 2,40,000 The Board of Directors plans to introduce more mechanisation into the department at a capital cost of Rs. 1,60,000. The effect of this will be to reduce the number of employees to 120, and increasing the output per individual employee by 60%. To provide the necessary incentive to achieve the increased output, the Board intends to offer a 1% increase on the piece work rate of Re. 1 per unit for every 2% increase in average individual output achieved. To sell the increased output, it will be necessary to decrease the selling price by 4%. Calculate the extra weekly contribution resulting from the proposed change and evaluate for the Board’s information, the desirability of introducing the change.

3.64

Page 199: 30510870 Cost Accounting and Financial Management

Labour

Solution 1. Present average output per employee and total future expected output per week

Present average output per employees per week = employeespresent of number Totaloutputpresent Total

= employees 160units 48,000

= 300 units

Total Future expected output per week= Total number of future employees(present output + 60% of present output per employee) = 120 employees (300 units + 60% × 300 units) = 57,600 units 2. Present and proposed piece work rate Present piece work rate = Re 1.00 per unit Proposed piece work rate = Present piece work rate + 30% × Re. 1 = Re. 1.00 + 0.30 P = Rs. 1.30 per unit 3. Present and proposed sale price per unit Present sales price per unit = Rs. 12.50 (Rs. 6,00,000/48,000 units) Proposed sale price per unit = Rs. 12 (Rs. 12.50 – 4% × Rs. 12.50) 4. Present marginal cost (excluding wages) per unit :

= (units) outputaPresent

gespresent wa towards onContributiprofit&expenses Fixedvalue salesPresent −−

= units000,48

000,48.Rs000,40,2.Rs000,00,6.Rs −− = Rs. 6.50 per unit

3.65

Page 200: 30510870 Cost Accounting and Financial Management

Cost Accounting

Statement of extra weekly contribution (Information resulting from the proposed change of mechanisation meant for Board’s evaluation)

Expected sales units 57,600 (Refer to Working note 1) Rs. Rs. Sales value : (A) (57,600 units × Rs. 12) 6,91,200 (Refer to Working note 3) Marginal costs (excluding wages) : (B) (57,600 units × Rs. 6.50) 3,74,400 (Refer to Working note 4) Wages : (C) (57,600 units × Rs. 1.30) 74,880 (Refer to Working note 2) Total marginal cost : (D) = {(B) – (C)} 4,49,280 Marginal contribution : {(A) – (D)} 2,41,920 Less : Present contribution 2,40,000 Increase in contribution (per week) 1,920

Evaluation: Since the mechanisation has resulted in the increase of contribution to the extent of Rs. 1,920 per week, therefore the proposed change should be accepted.

3.10 ABSORPTION OF WAGES 3.10.1 Elements of wages: In common parlance, the term ‘wages’ represents monetary payment which an employee receives at regular intervals for the services rendered. Strictly speaking, however, from the point of view of the employer and the cost to the industry, wages should be taken to include also non-monetary benefits which an employee receives by virtue of employment. Such non-monetary benefits may include: (i) medical facilities; (ii) educational and training facilities; (iii) recreational and sports facilities; (iv) housing and social welfare; and (v) cost of subsidised canteen and co-operative societies. Such benefits are generally given in an industrial establishment. In some cases the provision of benefits is compulsory. Therefore, while computing the wage cost per worker, the monetary

3.66

Page 201: 30510870 Cost Accounting and Financial Management

Labour

value of such non-monetary benefits should also be taken into account. The monetary part of a worker’s remuneration includes the basic wages, dearness allowance, overtime wages, other special allowance, if any, production bonus, employer’s contribution to the provident fund, Employees State Insurance Corporation premium, contribution to pension fund, leave pay, etc. The basic wage is the payment for work done, measured in terms of hours attended or the units produced, as the case may be. The basic wage rate is not normally altered unless there is a fundamental change in the working conditions or methods of manufacture. Dearness allowance is an allowance provided to cover the increase in cost of living from one period to another. This allowance is calculated either as percentage of the basic wage or as a fixed amount for the days worked. In either case, the percentage or the fixed amount is subject to revision whenever the cost of living index rises or falls by a certain figure as agreed to by the employer with the labour union. When permanent rise in the cost of living index occurs, a part of the dearness allowance is often absorbed in the basic wage. Overtime allowance is an allowance paid for the extra hours worked at the rates laid down in the Factories Act. In certain industries, where special allowance for the working conditions, tool maintenance, etc., are paid they are also considered as part of wages. Production bonus is an incentive payment made to workers for efficiency that results in production above the standard. There are different methods of computing incentives. Under the Payment of Bonus Act, a worker is entitled to compulsory bonus of 8.33% wages earned in the relevant year or Rs. 100 (whichever is greater). The bonus may be upto 20% of wages depending upon the quantum of profits calculated as per the Act. Contributions to provident fund and to E.S.I. schemes are compulsory. Under the first mentioned scheme, the employer contributes to the provident fund a sum equal to 8⅓% of the basic wages together with the dearness allowance. Under the E.S.I. scheme, the employer contributes a fixed amount per month per worker calculated on the amount of wages as prescribed by the E.S.I. Act. The payments by the employer under these schemes are payable at the time, wages are paid for the relevant period; these payments form a part of the wages for the period. 3.10.2 Component of wages cost or wages for costing purposes : In addition to wages (including allowances, such as D.A.) that are paid to workers, a firm may have to spend on many other items (such as premium to the Employees State Insurance Corporation or provident fund or bonus). Further, the worker does not spend all the time for which he is paid on productive work. This is because he is entitled to weekly holiday and various type of leave. There is also a certain amount of unavoidable idle time. The question is : to what extent such additional payment or cost in respect of labour can be charged directly to unit of cost as part of direct labour cost? Of course, in the case of indirect labour, all such payments as also the

3.67

Page 202: 30510870 Cost Accounting and Financial Management

Cost Accounting wages paid to them, must be treated as part of overheads. But in the case of direct workers, two alternatives are possible. The additional charges may be treated as overheads. Alternatively, the wage rates being charged to job may be computed by including such payments; automatically then, such payments will be charged to the work done alongwith wages of the worker. (It should be remembered that such wage rate will be only for costing purposes and not for payment to workers). The total of wages and additional payment should be divided by effective hours of work to get such wage rates for costing purposes.

Illustration A worker is paid Rs. 100 per month and a dearness allowance of Rs. 200 p.m. There is a provident fund @ 8⅓% and the employer also contributes the same amount as the employee. The Employees State Insurance Corporation premium is 1½% of wages of which ½% is paid by the employees. It is the firm’s practice to pay 2 months’ wages as bonus each year. The number of working days in a year are 300 of 8 hours each. Out of these the worker is entitled to 15 days leave on full pay. Calculate the wage rate per hour for costing purposes.

Solution Rs.

Wages paid to worker during the year 3,600 *Add Provident Fund @ 8.33% 300 *E.S.I. Premium 1% 36 Bonus at 2 months’ wages 600 Total 4,536

Effective hours per year: 285 × 8 = 2,280 Wage-rate per hour (for costing purpose): Rs. 4,536/2,280 hours = Rs. 1.989

Illustration : Calculate the earnings of A and B from the following particulars for a month and allocate the labour cost to each job X, Y and Z:

A B (i) Basic wages Rs. 100 160 (ii) Dearness allowance (on basic wages) 50% 50% (iii) Contribution to provident fund (on basic wages) 8% 8% (iv) Contribution to employees’ state insurance (on basic wages) 2% 2% (v) Overtime Hours 10

3.68

Page 203: 30510870 Cost Accounting and Financial Management

Labour

The normal working hours for the month are 200. Overtime is paid at double the total of normal wages and dearness allowance. Employer’s contribution to State Insurance and Provident Fund are at equal rates and employees’ contributions. The two workers were employed on jobs X, Y and Z in the following proportions: Jobs

X Y Z Worker A 40% 30% 30% Worker B 50% 20% 30% Overtime was done on job Y.

Solution Statement showing Earnings of Workers A and B

Workers A B Rs. Rs. Basic wages 100 160 Dearness allowance (50% of basic wages) 50 80 Overtime wages (Refer to working note 1) 15 -Gross wages earned 165 240 Less: - Provident fund-8% of basic wages - ESI-2% of basic wage 10 16Net wages paid 155 224

Statement of Labour cost Rs. Rs. Gross wages (excluding overtime) 150 240 Employer’s contribution to P.F. and E.S.I. 10 16Ordinary wages 160 256Labour rate per hour 0.80 1.28 (Rs.160/200) (Rs. 256/200)

3.69

Page 204: 30510870 Cost Accounting and Financial Management

Cost Accounting

Statement showing allocation of wages to jobs Jobs Total Wages X Y Z Rs. Rs. Rs. Rs. Worker A : Ordinary wages: (4:3:3) 160 64 48 48 Overtime 15 — 15 — Worker B : Ordinary wages: (5:2:3) 256 128 51.20 76.8 431 192 114.2 124.8 Working Notes Normal wages are considered as basic wages

Overtime = 2 × 200

.)A.DBasic( + × 10 hours

= 2 × (Rs. 150/200) × 10 hours = Rs.15

Illustration In a factory working six days in a week and eight hours each day, a worker is paid at the rate of Rs. 100 per day basic plus D.A. @ 120% of basic. He is allowed to take 30 minutes off during his hours shift for meals-break and a 10 minutes recess for rest. During a week, his card showed that his time was chargeable to : Job X 15 hrs. Job Y 12 hrs. Job Z 13 hrs. The time not booked was wasted while waiting for a job. In Cost Accounting, how would you allocate the wages of the workers for the week?

3.70

Page 205: 30510870 Cost Accounting and Financial Management

Labour

Solution

Working notes: (i) Total effective hours in a week : [(8 hrs. – (30 mts. + 10 mts.)] × 6 days = 44 hours (ii) Total wages for a week : (Rs. 100 + 120% of Rs. 100) × 6 days = Rs. 1,320 (iii) Wage rate per hour : = Rs. 30 (iv) Time wasted waiting for job (Abnormal idle time): = 44 hrs. – (15 hrs. + 12 hrs. + 13 hrs.) = 4 hrs. Allocation of wages in Cost Accounting Rs.

Allocated to Job X : 15 hours × Rs. 30 = 450 Allocated to Job Y : 12 hours × Rs. 30 = 360 Allocated to Job Z : 13 hours × Rs. 30 = 390 Charged to Costing Profit & Loss A/c : 4 hours × Rs. 30 = 120Total 1,320

3.10.3 Holiday and leave wages : One alternative to account for wages paid on account of paid holiday and leave can be to include them as departmental overheads. In such a case, it is necessary to record such wages separately from “worked for wages”. Such a segregation can be made possible by providing a separate column in the payroll for holiday and leave wages in the same way as there are columns for dearness allowance, provident fund deductions, etc. If, however, a separate or additional column cannot be provided for this purpose it would be necessary to analyse the payroll periodically to ascertain how much of the total payment pertains to “worked for wages” and how much is attributed to leave and holiday wages. Another way could be to inflate the wage rate for costing purposes to include holiday and leave wages. This can be done only in the case of direct workers.

3.71

Page 206: 30510870 Cost Accounting and Financial Management

Cost Accounting Illustration Calculate the labour hour rate of a worker X from the following data : Basic pay Rs. 1,000 p.m. D.A. Rs. 300 p.m. Fringe benefits Rs. 100 p.m. Number of working days in a year 300. 20 days are availed off as holidays on full pay in a year. Assume a day of 8 hours.

Solution

(a) (i) Effective working hour/days in a year 300 Less : Leave days on full pay 20 Effective working days 280 days Total effective working hours (280 days × 8 hours) 2,240 (ii) Total wages paid in a year Rs. Basic pay 12,000 D.A. 3,600 Fringe benefits 1,200 16,800 (iii) Hourly rate : Rs. 16,800/2,240 hours Rs. 7.50

3.10.4 Night shift allowance: In some cases, workers get extra payment if they work at night. Since the extra payment is not for any particular job, therefore such a payment should be treated as part of overheads. 3.10.5 Principles of remuneration: The term ‘remuneration’ has been defined as the reward for labour and service. It is the result of the agreement between the employer and the employee, whereby for a specified work or service rendered by the employee the employer agrees to pay a specified sum of money. Apart from this an employee by virtue of the fact that he is an employee becomes entitled to certain non-monetary benefits. The method of remuneration adopted varies from industry to industry and, in certain cases, even among different units in the same industry. Whatever be the variation, the method of fixing remuneration payable to the various categories of employees has to be based on certain accepted principles. These are:

3.72

Page 207: 30510870 Cost Accounting and Financial Management

Labour

(i) Wage-rates in an industry should be fixed in conformity with the general wage-levels in the geographical area i.e. the rate should be more or less the same for similar efforts and skill. The wage-level in an area would in turn depend upon demand for labour, the availability of labour, the cost of living in the area, the wage levels in neighbouring industrial area, and the capacity of the particular industry to pay.

(ii) Wage-rates should be related to the degree of skill, effort, initiative and responsibility that the employee is expected to assume in respect of the various jobs he may be called upon to perform. There should be generally equal pay for equal work.

(iii) Wage-rates should guarantee a minimum wage regardless of the existence of factors listed under (ii) above, particularly when the working conditions are difficult and dangerous.

(iv) Wage-rates are considered satisfactory only if they enable the workers to maintain a reasonable standard of living.

(v) Separate wage rates should be fixed for different classes of employees since each class expects to maintain a different standard of living; also the education, physical and mental efforts and responsibility required for performing different jobs are not the same.

(vi) It should be possible for worker to increase his earnings through extra effort and by increasing output. If he alone is responsible, he should have the full benefit of the increased productivity. Otherwise, if increased output has resulted from co-operation between management and workers both should share the benefit.

It is important that these basic principles should be recognised in fixing the wage rate of workers; otherwise, there will be dissatisfaction among the employees and, consequently, there will be higher labour turnover. Satisfactory employer-employee relationship is a primary necessity for industrial development and this has to be ensured to a very great degree, by satisfactory schemes of remunerating labour. The aim should be to keep labour cost per unit of output (or service) as low as possible. It is not the same as keeping wages at low levels. There is a definite correlation between wages and productivity; high wages often lead to such an increase in productivity that wages per unit of output fall. However, this rule is also subject to diminishing returns—a point is reached at which any further increase in wage rates does not bring about a corresponding increase in efficiency. But generally, higher wages result in lower cost per unit. Wages affect the national economy through cost of goods produced. If an increase in wages outpaces the corresponding increase in productivity, goods become costlier and cannot compete with those of other countries in the world markets. From the point of view of an expert it is necessary to keep wages in check like other costs. The safe rule is to link up wages with productivity.

3.73

Page 208: 30510870 Cost Accounting and Financial Management

Cost Accounting 3.10.6 Absorption rates of labour cost: Labour cost as stated above include monetary compensation and non-monetary benefits to workers. Monetary benefits include, basic wages, D.A., overtime pay, incentive or production bonus contribution to employee provident fund, House Rent Allowance, Holiday and vacation pay etc. The non-monetary benefits include medical facilities, subsidized canteen services, subsidized housing, education and training facilities. Accounting of monetary and non-monetary benefits to indirect workers does not pose any problems because the total of monetary and non-monetary benefits are treated as overhead and absorbed on the basis of rate per direct labour hour, if overheads are predominantly labour oriented. For direct workers, the ideal method is to charge jobs or units produced by supplying per hour rate calculated as below :

Rate per hour = time idle Normal - hours labourdirect Budgeted

benefitsmonetary -non ofcost and benefitsmonetary estimated of otalT

Another alternative method is to treat the monetary benefits other than basic wages and dearness allowance as well as cost of non-monetary benefits as overheads.

3.11 EFFICIENCY RATING PROCEDURES Efficiency is usually related with performance and may be computed by comparing the time taken with the standard time allotted to perform the given job/task. If the time taken by a worker on a job equals or less than the standard time, then he is rated efficient. In case he takes more time then the standard time he is rated as inefficient. It may be computed as follows :

Efficiency in % = 100taken Time

std. per as allowed Time×

For efficiency rating of employees the following procedures may be followed : 1. Determining standard time/performance standards : The first step is to determine the standard time taken by a worker for performing a particular job/task. The standard time can be determined by using Time & Motion study or Work study techniques. While determining the standard time for a job/task a heterogeneous group of workers is taken and contingency allowances are added for determining standard time. 2. Measuring Actual Performance of workers : For computing efficiency rating it is necessary to develop a procedure for recording the actual performance of workers. The system developed should record the output of each worker along with the time taken by him. 3. Computation of efficiency rating : The efficiency rating of each worker can be computed by using the above mentioned Formula.

3.74

Page 209: 30510870 Cost Accounting and Financial Management

Labour

3.11.1 Need for efficiency rating : 1. As discussed earlier when a firm follows a system of payment by results, the payment

has a direct relationship with the output given by a worker. The firm for making payment to worker is required to ascertain his efficiency level. For instance, under Taylor's differential piece work system the lower rate i.e. 83% of piece rate is given to a worker when his efficiency rating is less than 100% and higher rate viz., 125% of piece rate is offered at efficiency level of either 100% or more. Similarly under Emersion efficiency plans bonus is paid at rising scale at various level of efficiency, ranging from 66.67% to 150%.

2. The efficiency rating also helps the management in preparing labour requirement budget or for preparing manpower requirements. For example, let P Ltd. manufactures two products by using one grade of labour. The following estimates are available :

Product A Product B Budgeted production (units) 3,480 4,000 Std. hrs. allowed per product 5 4

It is further worked out that the efficiency rating (efficiency ratio) for productive hours worked by direct workers in actually manufacturing the production is 80% then the exact standard labour requirement can be worked out as follows :

Product A Product B Total Budgeted production (in units) 3,480 4,000 Std. hours allowed for 17,400 16,000 33,400 budgeted production (3,480 units × 5 hours) (4,000 units × 4 hours)

Since efficiency ratio is given as 80% therefore Std. labour hours required for 100% efficiency

level are ⎟⎠⎞

⎜⎝⎛ ×

80100hours 400,33 = 41,750 hours.

Labour productivity : Productivity is generally determined by the input/output ratio. In the case of labour it is calculated as below :

workdo to taken time Actual workofamount actual doing for time Standard

Labour productivity is an important measure for measuring the efficiency of individual workers. It is an index of efficiency and a sign of effectiveness in the utilisation of resources-men, materials, capital, power and all kinds of services and facilities. It is measured by the output in relation to input. Productivity can be improved by reducing the input for a certain quantity or

3.75

Page 210: 30510870 Cost Accounting and Financial Management

Cost Accounting value of output or by increasing the output from the same given quantity or value of input. Factors for increasing labour productivity : The important factors which must be taken into consideration for increasing labour productivity are as follows: 1. Employing only those workers who possess the right type of skill. 2. Placing a right type of man on the right job. 3. Training young and old workers by providing them the right types of opportunities. 4. Taking appropriate measures to avoid the situation of excess or shortage of labour at the

shop floor. 5. Carrying out work study for the fixation of wage rate, and for the simplification and

standardisation of work. Self Examination Questions Multiple Choice Questions 1. The input-output ratio in case of labour means the ratio of

(a) the value of output to the wages paid. (b) standard time of the production to the actual time paid for. (c) abnormal idle time to normal idle time. (d) number of workers employed to the sanctioned strength.

2. Job specification is (a) the list of operations to be performed for completing the concerned job (b) the requirement in terms of goods to be produced or work to be done. (c) the list of qualities and qualifications which the employees concerned should have

to do the job well. (d) the name of the employees who will be assigned to a job.

3. Job specification is (a) the list of operations to be performed for completing the concerned job (b) the requirement in terms of goods to be produced or work to be done. (c) the list of qualities and qualifications which the employees concerned should have

to do the job well. (d) the name of the employees who will be assigned to a job.

3.76

Page 211: 30510870 Cost Accounting and Financial Management

Labour

4. Direct labour means (a) labour completing the work manually, (b) labour which is recruited directly and not through contractors, (c) permanent labour in the production department, (d) labour which can be conveniently associated with a particular cost unit. 5. Time and motion study is essential for (a) a rational promotion policy, (b) completing a job on time, (c) determining the standard-time and correct method of completing a task, (d) determining prices of products. 6. For reducing the labour cost per unit, which of the following factors is the most

important? (a) low wage rates, (b) higher input-output ratio, (c) strict control and supervision, (d) longer hours of work. 7. Which of the following statements are true ?

(a) Productivity of workers can be improved only if they are supervised closely. (b) It is no use paying higher wages to labour because they would spend their money

on drinking and smoking. (c) A well satisfied team of workers can raise productivity to a large extent. (d) None of the above

8. Labour turnover is measured by, (a) Replacement method. (b) Separation method. (c) Flux method. (d) All of the above.

3.77

Page 212: 30510870 Cost Accounting and Financial Management

Cost Accounting 9. Salary of a foreman should be classified as, (a) Fixed overhead. (b) Variable overhead. (c) Semi fixed or semi variable overhead. (d) None of the above. 10. For reducing the labour cost per unit, which of the following factors is the most

important? (a) Low wage rates. (b) Higher input output ratio. (c) Strict control. (d) Long hours of work.

Answers To Multiple Choice Questions 1.(b);2.(c);3.(a) ;4.(d);5.(c);6.(b);7.(c);8.(d);9.(c);10 (b)

Short Answer Type Questions

1. Describe briefly the functions of the following departments in relation to labour : (a) Personnel department. (b) Engineering department. (c) Cost Accounting department. 2. Distinguish between : (a) Time keeping and Time booking (b) Time study and motion study. 3. Discuss briefly the important factors for the control of labour cost. 4. Discuss briefly the objectives of time keeping. 5. Discuss briefly the various factors necessary for introducing an incentive system.

Long Answer Type Questions

1. What is idle time? Explain the causes leading to idle time and its treatment in Cost Accounts.

3.78

Page 213: 30510870 Cost Accounting and Financial Management

Labour

2. What do you mean by overtime premium? What are the causes of overtime? How would you treat overtime premium in Cost Accounts ?

3. What do you understand by labour turnover? How is it measured? What are its causes? What steps should be taken to check the increasing rate of labour turnover?

4. Define job evaluation and distinguish it from merit rating. Explain the methods and objectives of job evaluation.

5. Explain the factors to be considered in introducing an incentive system.

Numerical Questions

1. Calculate the number of hours worked as overtime by the following workers in a week: Ram Shyam Monday 8 8 Tuesday 7 9 Wednesday 4.5 8 Thursday 8 7 Friday 10 9 Saturday 9 9 46.5 50

2. Three workers A, B and C are put on a common task for which the total remuneration is Rs. 150. A works for 40 hours, B works for 60 hours and C works for 44 hours on the job. The hourly rate is Re. 0.75 of A per hour, B gets Re. 0.80 per hour while C’s remuneration is Re. 0.50 per hour. What should each man get ?

3. A worker is paid @ 50 paise per hour plus a dearness allowance of Rs. 60 per month. The provident fund contribution both by the employee and the worker is 6¼% each. The worker is entitled to 15 days leave with full wages. His normal working per month is 25 days of 8 hours each.

(a) the wages per hour for costing purposes; and (b) the amount to be paid to him for a week in which he puts in 52 hours of work.

3.79

Page 214: 30510870 Cost Accounting and Financial Management

Cost Accounting 4. The following particulars are available to you in respect of a worker: Job No.Time Allowed Time Taken 1844 26 hours 20 hours 1826 30 hours 20 hours Idle time (waiting) 8 hours 48 hours

The basic rate is Rs. 2 per day of 8 hours in addition there is a dearness allowance of Rs. 12 per week of 48 hours. Calculate the wage of the worker on (1) Time Basis (2) Piece Rate Basis (3) Halsey Plan Basis and (4) Rowan Plan Basis.

5. A worker is paid 10% bonus on the hourly rate if he completes his work in the time allotted for it and a further 1% on hourly rate for each 1% in excess of 100% efficiency. His hourly rate is Rs. 5 per hour and he completed a job in 45 hours whereas the time allowed for it was 50 hours. Ascertain the wages earned by this worker.

6. From the following data, calculate the labour turnover rate by applying : (i) Separation method (ii) Accession method (iii) Flux method

Number of workers on the payroll At the beginning of the month 1,800 At the end of the month 2,200

During the month 20 workers left, 80 workers were discharged and 500 workers were recruited. Of these 50 workers were recruited in the vacancies of those separated, while the rest were engaged due to expansion.

7. The company has a suggestion of box scheme and an award equivalent to one and a half months saving in labour cost is passed on to the employee whose suggestion is accepted. Suggestion of an employee to use a Jig for a manufacturing operation of a component is accepted. The cost of the Jig which has a life of one year is Rs. 1,000 and the use of the Jig will reduce the standard time by 8 minutes.

Compute from the following data the amount of award payable to the employee who has given the suggestion.

(i) Number of pieces to be produced in the year : 15,000

3.80

Page 215: 30510870 Cost Accounting and Financial Management

Labour

(ii) Standard time per piece before use of Jig : 80 minutes (iii) Average wage rate of workmen Rs. 160 per day of 8 hours. (iv) Average efficiency of workmen : 80%. 8. The existing incentive system of a certain factory is : Normal working week : 5 days of 9 hours each plus Resorting to overtime of

3 hours for 3 days. Rate of payment : For day work - Rs. 20 per hour. For overtime -

Rs. 30 per hour. Additional bonus payable : Rs. 25 per day if worker is not resorting to overtime.

Rs. 40 per day if worker resorts to overtime. Average output per operative : for 54 hour week, i.e, normal working hours plus 3 hours late sitting for 3 days 120 articles

In order to increase output and eliminate overtime it was decided to switch on to a system of payment by results. The factory considering the introduction of some incentive scheme or to make payment on piece work basis. Assuming that 135 articles are produced in a 45 hour week and the additional bonus under the existing system will be discontinued in the proposed incentive scheme. You are required to calculate :

(i) Weekly earnings; (ii) labour cost per article for an operative under the following systems:

(a) Existing time-rate system (b) Straight piece-work system (c) Rowan system (d) Halsey system

The following information is obtained. Time rate (as usual) : Rs. 20 per hour Basic time allowed : for 15 articles 5 hours Piece work rate : Add 20% to price Premium bonus : Add 50% to time

3.81

Page 216: 30510870 Cost Accounting and Financial Management

Cost Accounting 9. The unit has a strength of 20 workmen worked for 300 working days of 8 hours each with

half an hour break based on the earlier years trend, it is forecast that average absenteeism per workman would be 8 days, in addition to the eligibility of 30 days annual leave. The following details regarding actual working of the unit are available for the year ending on 31st March, 1998.

(i) The factory worked 2 extra days to meet the production targets, but one additional paid holiday had to be declared.

(ii) There was a severe breakdown of a major equipment leading to a loss of 300 man hours.

(iii) Total overtime hours (in addition to 2 extra days worked) amounted to 650 hours. (iv) The actual average absenteeism per workman was 8 days. (v) Basic rate is Rs. 10 per hour and overtime is paid at double rate. You are required

to calculate. (a) Actual working hours of the unit. (b) In Cost Accounting how would you treat the wages of workmen for (ii) & (iii)

above ? 10. A job can be executed either through workman A or B. A takes 32 hours to complete the

job while B finishes it in 30 hours. The standard time to finish the job is 40 hours.

The hourly wage rate is same for both the workers. In addition workman A is entitled to receive bonus according to Halsey plan (50%) sharing while B is paid bonus as per Rowan plan. The works overheads are absorbed on the job at Rs. 7.50 per labour hour worked. The factory cost of the job comes to Rs. 2,600 irrespective of the workman engaged.

Find out the hourly wage rate and cost of raw materials input. Also show cost against each element of cost included in factory cost.

3.82

Page 217: 30510870 Cost Accounting and Financial Management

CHAPTER 4

OVERHEADS

Learning objectives When you have finished studying this chapter, you should be able to ♦ Differentiate between direct costs and overheads. ♦ Understand the meaning of allocation, apportionment and absorption of overheads. ♦ Identify, whether overheads are under absorbed or over absorbed ♦ Understand the accounting and control of administrative, selling and distribution

overheads.

4.1 INTRODUCTION Besides direct expenditure, i.e., expenditure which can be conveniently traced to or identified with any particular unit of production, e.g., direct materials, direct wages and direct expenses, every form of production involves expenses that cannot be conveniently traced to or identified with the articles produced or services provided. Such expenses are incurred for output generally and not for a particular work order e.g., wages paid to watch and ward staff, heating and lighting expenses of factory etc. Expenses of these nature are known as overhead or indirect expenses. Often in a manufacturing concern, overheads exceed direct wages or direct materials and at times even both put together. On this account, it would be a grave mistake to ignore overheads either for the purpose of arriving at the cost of a job or a product or for controlling total expenditure. Overheads also represent expenses that have been incurred in providing certain ancillary facilities or services which facilitate or make possible the carrying out of the production process; by themselves these services are not of any use. For instance, a boiler house produces steam so that machines may run and, without the generation of steam, production would be seriously hampered. But if machines do not run or do not require steam, the boiler house would be useless and the expenses incurred would be a waste. Apart from the overheads incurred in the factory, overheads also arise on account of administration, selling and distribution.

Page 218: 30510870 Cost Accounting and Financial Management

Cost Accounting 4.2 CLASSIFICATION OF OVERHEADS

4.2.1 Classification of overheads by function : Overheads principally are of four types: (i) Factory or Manufacturing Overheads; (ii) Office and Administrative Overheads; (iii) Selling and Distribution Overheads; (iv) Research and Development.

Whether an expense belongs to one class or another depends entirely on the benefit derived from it. For instance, salaries of clerks will be (i) factory or manufacturing expenses, when the clerks concerned work in the factory office; (ii) office and administrative expense when the clerks work in the general office; and (iii) selling and distribution expense when the clerks work in the sales office.

Small concerns may not distinguish between office and selling expenses and still smaller concerns may treat all overhead, of whatever class, together. Big concerns may have even a more detailed classification to be able to exercise better control. A detailed classification of such expenses is given below:

(i) Factory or Manufacturing Expenses:

(a) Stores overheads (expenses connected with purchasing and handling of materials);

(b) Labour overheads (expenses connected with labour); and

(c) Factory administration overheads (expenses connected with administration of the factory).

(ii) Office and Administration Expenses:

(a) Administrative expenses (expenses incurred on managerial personnel - their salaries, costs of facilities provided to them and salaries of their personal staff); and

(b) Office expenses (expenses on the routine office work).

(iii) Selling and Distribution Expenses :

(a) Selling expenses (expenses incurred to persuade customers to purchase the firm’s products and, or engage its services, that is to maintain and expand the market); and

(b) Distribution expenses are those which are incurred to execute orders. One should note that many people use the two terms “selling” and “distribution” as synonymous.

Following are the definitions given by the Institute of Cost and Management Accountants of England.

4.2

Page 219: 30510870 Cost Accounting and Financial Management

Overheads

Production Cost - The cost of the sequence of operations which begins with supplying materials, labour and service and ends with the primary packing of the product. Selling Cost - The cost of seeking to create and stimulate demand sometimes termed (marketing) and of securing orders. Distribution Cost - The cost of the ‘sequence’ of operations which begins with making the packed product available for despatch and ends with making the reconditioned returned empty package, if any, available for re-use. As well as including expenditure incurred in moving articles to central or local storage, distribution cost includes expenditure incurred in moving articles to and from prospective customers as in the case of goods on sale or return basis. In the gas, electricity and water industries ‘Distribution’ means pipes, mains and services which may be regarded as equivalent to packing and transportation. Administration cost - The cost of formulating the policy, directing the organisation and controlling the operations of an undertaking which is not related directly to production, selling, distribution, research or development activity or function. Research and Development Expenses : The Terminology defines research expenses as “the expenses of searching for new or improved products, new application of materials, or new or improved methods.” Similarly, development expenses is defined as “the expenses of the process which begins with the implementation of the decision to produce a new or improved product.” If research is conducted in the methods of production, the research expenses should be charged to the production overhead; while the expenditure becomes a part of the administration overhead if research relates to administration. Similarly, market research expenses are charged to the selling and distribution overhead. Development costs incurred in connection with a particular product should be charged directly to that product. Such expenses are usually treated as “deferred revenue expenses,” and recovered as a cost per unit of the product when production is fully established. General research expenses of a routine nature incurred on new or improved methods of manufacture or the improvement of the existing products should be charged to the general overhead. Even in this case, if the amount involved is substantial it may be treated as a deferred revenue expenditure, and spread over the period during which the benefit would accrue. Expenses on fundamental research, not relating to any specific product, are treated as a part of the administration overhead. Where research proves a failure, the cost associated with it should be excluded from costs and charged to the costing Profit and Loss Account. A list (not exhaustive) of various items under three principal classes of overheads is presented on the next page.

4.3

Page 220: 30510870 Cost Accounting and Financial Management

Factory Expenses Office and Administration Selling and Distribution Expenses Expenses Buildings : Rent, repairs, depreciation and in- Administration : Fees to Selling: Fees to Director who

surance of factory premises, light- Directors’ Salary to General looks after sale and market- ing of factory premises. Manager, Managing Director, ing salary of Sales Manager;

Finance Manager, Chief salesmen and sales office clerks; Machinery : Depreciation, repairs and main- Accountant, Secretary, their commission to agents, advertis-

tenance and insurance of plant immediate personal staff and ing catalogues, price lists. and equipment; power used for other expenses like air condi- samples, show room expenses; machines. tioning of office; entertainment of customers;

Labour : Wages of indirect workers; normal stationery, postage etc., used in idle time (unless wage rates are the office. inflated suitably); Employees’ State Distribution: Finished goods

Insurance premium; Provident godown expenses-salary, rent, Fund contribution, leave pay, insurance lighting etc.

maternity pay; etc. Office : Salaries paid to other packing; carriage outwards; people working in the office; insurance, in transit; Supervision : Salaries to foremen, departmental stationery, postage, etc. light- delivery expenses; superintendents and Works Mana- ing of office, rent, rates, and expenses on receiving and ger; Technical Director’s fees. taxes on office premises. reconditioning, returnable Depreciation, power, insu- empties. Materials : Purchasing and store keeping rance repairs and mainte- expenses, cost of consumable nance of office equipment, etc. stores and supplies, normal losses of materials unless prices are suit- ably inflated, etc. Misc : Factory office telephone, stationery, factory office clerks’ salaries, etc.

Page 221: 30510870 Cost Accounting and Financial Management

Overheads

Expenses that are not taken into account - The undermentioned expenses are usually not included in overheads or, for that matter in cost : (a) Expenses or income of purely financial nature like dividends received, rent received,

cash discount allowed, etc.

(b) Expenses or profits of capital nature like profit or loss on sale of investments, plant and equipment, etc.

(c) Items not representing actual costs but dependent on arbitrary decisions of the management, e.g., an unreasonably high salary to the managing director, providing for depreciation at a rate exceeding the economic rate.

(d) Appropriation of profits for dividends, payment of income tax and transfers to reserves. 4.2.2 Classification of overheads by nature : On a change in the level of activity different expenses behave differently. On this consideration, expenses are classified under the following three categories: (i) Fixed or Constant : These are expenses that are not affected by any variation in the volume of activity, e.g., managerial remuneration, rent, that part of depreciation which is dependent purely on efflux of time, etc. Fixed or constant expenses remain the same from one period to another except when they are deliberately changed, e.g., on increments being granted to staff or additional staff being engaged. (ii) Variable : Expenses that change in proportion to the change in the volume of activity; when output goes up by 10% the variable expenses also go up by 10%. Correspondingly, on a decline of the output, these expenses also decline proportionately e.g., power consumed; consumable stores; repairs and maintenance and depreciation are dependent on the use of assets. Variable expenses are generally constant per unit of output or activity. Suppose variable expenses amount to Rs. 10,000 for a production of 2,000 units i.e., Rs. 5 per unit. When output goes upto 2,200 units, i.e., an increase of 10% the variable expenses amount to Rs. 11,000. i.e., 10,000 plus 10%. The cost per unit will be the same as before. (iii) Semi variable : The expenses that either (a) do not change when there is a small change in the level of activity but change whenever there is a slightly big change. In other words, they change by small steps; or (b) change in the same direction as change in the level of activity but not in the same proportion. An expense for example, may not change if output goes up or comes down by 5% but may change by 3% when there is an increase in production between 5% and 10%. Similarly, another item of expense may change by 1% for every 2% change in activity. Examples of such expenses are : delivery van expenses,

4.5

Page 222: 30510870 Cost Accounting and Financial Management

Cost Accounting telephone charges, depreciation as a whole. Semi-variable expenses usually have two parts—fixed and variable. For instance, the amount of depreciation usually depends on two factors—one on time (fixed) and the other on wear and tear (variable). The two together make depreciation (as a whole) semi-variable. A careful study can make it possible for all semi-variable expenses to split up into two parts. Fundamentally, therefore, there are only two type of expenses—fixed and variable. Graphically the three type of expenses may be shown as below:

One must note that fixed expenses remain unchanged upto the limit of the present capacity. If output goes beyond the capacity limit, fixed expenses will record a jump. Suppose a factory works one shift and produces 10,000 units in the shift. For all levels of output of 10,000 units, fixed expenses will remain unchanged; if the output goes beyond 10,000 units, a second shift will become necessary and this will mean a big increase in fixed expenses such as salary for foremen, lighting etc. Methods of segregating Semi-variable costs into fixed and variable costs – For a detailed understanding please refer to chapter 1. Advantages of Classification of Overheads into Fixed and Variable : The primary objective of segregating semi-variable expenses into fixed and variable is to ascertain marginal costs. Besides this, it has the following advantages also. (a) Controlling expenses : The classification of expenses into fixed and variable components helps in controlling expenses. Fixed costs are generally policy costs, which cannot be easily reduced. They are incurred irrespective of the output and hence are more or less non controllable. Variable expenses vary with the volume of activity and the responsibility for incurring such an expenditure is determined in relation to the output. The management can control these costs by giving proper allowances in accordance with the

4.6

Page 223: 30510870 Cost Accounting and Financial Management

Overheads

output achieved. (b) Preparation of budget estimates : The segregation of overheads into fixed and variable part helps in the preparation of flexible budget. It enables a firm to estimate costs at different levels of activity and make comparison with the actual expenses incurred. Suppose in October, 2005 the output of a factory was 1,000 units and the expenses were:

Rs. Fixed 5,000 Variable 4,000 Semi-variable (40% fixed) 6,000 15,000 In November, 2005 the output was likely to increase to 1,200 units. In that case the budget or estimate of expenses will be :

Rs. Fixed 5,000 Variable 4,800

⎟⎠

⎞⎜⎝

⎛ ×units 1,000

units 1,200Rs.4,000

Semi-variable Fixed, 40% of Rs. 6,000 2,400

Variable : ⎥⎦

⎤⎢⎣

⎡ ×units 1,000

units 1,200Rs.3,600 4,320 6,720

16,520 It would be a mistake to think that with the output going up from 1,000 units to 1,200 units the expenses would increase proportionately to Rs. 18,000. This would be wrong budgeting. (c) Decision making : The segregation of semi variable cost between fixed and variable overhead also helps the management to take many important decisions. For example, decisions regarding the price to be charged during depression or recession or for export market. Likewise, decisions on make or buy, shut down or continue, etc., are also taken after separating fixed costs from variable costs. In fact, when any change is contemplated, say, increase or decrease in production, change in the process of manufacture or distribution, it is necessary to know the total effect on cost (or revenue) and that would be

4.7

Page 224: 30510870 Cost Accounting and Financial Management

Cost Accounting impossible without a correct segregation of fixed and variable costs. The technique of marginal costing, cost volume profit relationship and break-even analysis are all based on such a segregation.

4.3 ACCOUNTING AND CONTROL OF MANUFACTURING OVERHEADS We have already seen that overheads are by nature those costs which cannot be directly related to a product or to any other cost unit. Yet for working out the total cost of a product or a unit of service, the overheads must be included. Thus we have to find out a way by which the overheads can be distributed over the various units of production. One method of working out the distribution of overheads over the various products could be to ascertain the amount of actual overheads and distribute them over the products. This however, creates a problem since the actual amount of overheads can be known only after the financial accounts are closed. If we wait that long, the cost sheets lose their main advantages and utility to the management. All the decisions for which cost sheets are prepared are immediate decisions and cannot be postponed till the actual overheads are known. Therefore, some method has to be found by which overheads can be included in the cost of the products, as soon as prime cost, the cost of raw materials, labour and other direct expenses, is ascertained. One method is to work out pre-determined rates for absorbing overheads. These rates are worked out before an accounting period begins by estimating the amount of overheads and the level of activity in the ensuing period. Thus, as soon as the prime cost of a product or a job is available, the various overheads are charged by these rates. Of course, this implies that the overheads are charged on an estimated basis. Later, when the actual overheads are known, the difference between the overheads charged to the products and actual overheads is worked out and adjusted. Manufacturing Overheads : Generally manufacturing overheads form a substantial portion of the total overheads. It is important, that such overheads should be properly absorbed over the cost of production. The following procedure may be adopted in this regard. The steps given below shows how factory overhead rates are estimated and overheads absorbed on that basis and the last one shows how actuals are compared with the absorbed amount. (Students should carefully note the distinction between the various terms used). 1. Estimation and collection of manufacturing overheads : The first stage is to estimate the amount of overheads, keeping in view the past figures and adjusting them for known future changes. There are four main sources available for the collection of factory overheads viz., (a) Invoices; (b) Stores requisition ; (c) Wage analysis book ; (d) Journal entries. 2. Cost allocation : The term ‘allocation’ implies relating overheads directly to the various departments. The estimated amount of various items of manufacturing overheads

4.8

Page 225: 30510870 Cost Accounting and Financial Management

Overheads

should be allocated to various cost centres or departments. The salary of the works manager cannot be directly allocated to any one department since he looks after the whole factory. It is, therefore, obvious that many overhead items will remain unallocated after this step. 3. Cost apportionment : At this stage, those items of estimated overheads (like the salary of the works manager) which cannot be directly allocated to the various departments and cost centres are apportioned. Apportionment implies “the allotment of proportions of items of cost to cost centres or departments”. It implies that the unallocable expenses are to be spread over the various departments or cost centres on an equitable basis. After this stage, all the overhead costs would have been either allocated to or apportioned over the various departments. 4. Re-apportionment : The next stage is to re-apportion the overhead costs of service departments over production departments. Service departments are those departments which do not directly take part in the production of goods. Such departments provide ancillary services. Examples of such departments are boiler house, canteen, stores, time office, dispensary etc. The overheads of these departments are to be re-apportioned over the production departments since service departments operate primarily for the purpose of providing services to production departments. At this stage, all the factory overheads are collected under production departments. 5. Absorption : The production department overheads are absorbed over production units. The overhead expenses can be absorbed by estimating the overhead expenses and then working out an absorption rate. When overheads are estimated, their absorption is carried out by adopting a pre-determined overhead absorption rate. This rate can be calculated by using any one method as discussed in this chapter at the end. As the actual accounting period begins, each unit of production automatically absorbs a certain amount of factory overheads through pre-determined rates. During the year a certain amount will be absorbed over the various products. This is known as the total amount of absorbed overheads. 6. Treatment of over and under absorption of overheads : After the year end the total amount of actual factory overheads is known. There is bound to be some difference between the actual amount of overheads and the absorbed amount of overheads. The difference has to be adjusted keeping in view of such differences and the reasons therefor. Students will thus see that the whole discussion as above is meant to serve the following two purposes : (a) to charge various products and services with an equitable portion of the total amount

of factory overheads ; and

4.9

Page 226: 30510870 Cost Accounting and Financial Management

Cost Accounting (b) to charge factory overheads immediately as the product or the job is completed

without waiting for the figures of actual factory overheads.

4.4 STEPS FOR THE DISTRIBUTION OF OVERHEADS The various steps for the distribution of overheads have been discussed in detail as below: 4.4.1 Estimation and Collection of Manufacturing Overheads : The amount of factory overheads is required to be estimated. The estimation is usually done with reference to past data adjusted for known future changes. The overhead expenses are usually collected through a system of standing orders. Standing Orders : In every manufacturing business, expenses are incurred on direct materials and direct labour in respect of several jobs or other units of production, manufacture of which is undertaken. The incurring of these expenses is authorised by production orders or work orders. The work order numbers are not ordinarily fixed or permanent. They are generally allotted in a serial order according to the number of manufacturing jobs undertaken by the business. In addition, indirect expenses are in-curred in connection with the rendering of services to the production departments, or to the manufacturing process. The term “Standing Order” denotes sanction for indirect expenses under various heads of expenditure. In large factories, usually the classification of indirect expenditures is combined with a system of Standing Orders (sometimes also referred as Service “Orders”). It is a system under which a number is allotted to each item of expense for the purpose of identification, and the same is continued from year to year. All the indirect expenditure in such a case, is charged to one or the other of the Standing Orders and periodical summaries, giving total of each Standing Order, are prepared for comparison with budgets, as well as for apportioning them among the various departments. The extent of such analysis and the nomenclature adopted are settled by the management according to the needs of the industry. 4.4.2 Allocation of overheads over various Departments or Departmentalisation of Overheads : Most of the manufacturing processes functionally are different and are performed by different departments in the factory. Where such a division of functions had been made, some of the departments should be engaged in actual production of goods, and others in providing services ancillary thereto. At this stage, the factory overheads which can be directly related to the various production or service departments are allocated in this manner. It may, sometime, become necessary to sub-divide a manufacturing organisation into several cost centres, so that a closer distribution of expenses and a more detailed control is practicable.

4.10

Page 227: 30510870 Cost Accounting and Financial Management

Overheads

It is thus obvious that the principal object of setting up cost centres is to collect data, in respect of similar activities more conveniently. This avoids a great deal of cost analysis. When costs are collected by setting up cost centres, several items can be ascertained definitely and the element of estimation is reduced considerably. For instance, the allowance of the normal idle time or the amount to be spent on consumable stores, etc. There are two main type of cost centres - machine or personal - depending on whether the process of manufacture is carried on at a centre by man or machine. For the convenience of recording of expenditure, cost centres are sometimes allotted a code number. Advantages of Departmentalisation : The collection of overheads departmentwise gives rise to the following advantages : (a) Some expenses which relate to the departments will be estimated almost on an exact basis and, to that extent, the accuracy of estimation of overheads will be higher. (b) For the purpose of controlling expenses in a department, it is obviously necessary that the figures in relation to each department should be separately available. It is one of the main principles of control that one should know for each activity how much should have been spent and how much is actually spent. If information about expenses is available only for factory as a whole, it will not be possible to know which department has been over spending. (c) From the point of view of ascertaining the cost of each job, the expenses incurred in the departments through which the job or the product has passed should be known. It is only then that the cost of the job or the product can be charged with the appropriate share of indirect expenses. It is not necessary that a job must pass through all the departments or that the work required in each department should be the same for all jobs. It is, therefore, necessary that only appropriate charge in respect of the work done in the department is made. This can be done only if overheads for each department are known separately. (d) A suitable method of costing can be followed differently for each department e.g., batch costing when a part is manufactured, but single or output costing when the product is assembled. 4.4.3 & 4.4.4 Apportioning overhead expenses over various departments and re-apportioning service department overheads over production department : After the allocable overheads are related to the departments, expenses incurred for several departments have to be apportioned over each department, e.g. rent, power, lighting, insurance and depreciation. For distributing these overheads over different departments benefiting thereby, it is necessary at first to determine the proportion of benefit received by each department and then distribute the total expenditure proportionately on that basis. But the same basis of apportionment cannot be followed for different items of overheads

4.11

Page 228: 30510870 Cost Accounting and Financial Management

Cost Accounting since the benefit of service to a department in each case has to be measured differently. Some of the basis that are generally adopted for the apportionment of expenses are stated below :

Basis Expense items Area or cubic measurement of department Direct labour hours or, where wage rates are more or less uniform, total direct wages of department.

Rent, rates, lighting and building maintenance Supervision

Number of employees in departments Supervision Cost of material used by departments Material handling charges Value of assets Depreciation and insurance Horse power of machines Power

Other basis of apportioning overhead costs : We have considered already that the benefit received by the department generally is the principal criterion on which the costs of service departments or common expenses are apportioned. But other criteria are equally valid. Three of them are mentioned below : (a) Analysis or survey of existing conditions. (b) Ability to pay. (c) Efficiency or incentive. A single concern may have only one criterion under consideration predominantly or may use all (including the service or benefit criterion) for different phases of its activity. Analysis or Survey of existing conditions : At times it may not be possible to determine the advantage of an item of expenses without undertaking an analysis of expenditure. For example, lighting expenses can be distributed over departments only on the basis of the number of light points fixed in each department. Ability to pay : It is a principle of taxation which has been applied in cost accounting as well for distributing the expenditure on the basis of income of the paying department, on a proportionate basis. For example, if a company is selling three different products in a territory, it may decide to distribute the expenses of the sales organisation to the amount of sales of different articles in these territories. This basis, though simple to apply, may be inequitable since the expenditure charged to an article may have no relation to the actual effort involved in selling it. Easy selling lines thus may have to bear the largest proportion of expenses while, on the other hand, these should bear the lowest charge. Efficiency or Incentives : Under this method, the distribution of overheads is made on the

4.12

Page 229: 30510870 Cost Accounting and Financial Management

Overheads

basis of pre-determined levels of production or sales. When distribution of overhead cost is made on this basis and if the level of production exceeds the pre-determined level of production the incidence of overhead cost gets reduced and the total cost per unit of production or of sales, lowered. The opposite is the effect if the assumed levels are not reached. Thus the department whose sales are increasing is able to show a greater profit and thereby is able to earn greater goodwill and appreciation of the management than it would have if the distribution of overheads was made otherwise. Inter-departmental service costs : At first, expenses of all departments are complied without making a distinction between production and service departments. Then the expenses of the service departments are apportioned among the production departments on a suitable basis. This is because ultimately the overheads are to be absorbed over goods produced or jobs completed in the production departments. The re-apportionment of service department expenses over the production departments may be carried out by using any one of the following methods : (i) Direct re-distribution method. (ii) Step method of secondary distribution or non-reciprocal method. (iii) Reciprocal Service method. Direct re-distribution method: Service department costs under this method are apportioned over the production departments only, ignoring the services rendered by one service department to the other. To understand the application of this method go through the illustration which follows. Illustration XL Ltd., has three production departments and four service departments. The expenses for these departments as per Primary Distribution Summary are as follows :

Production Departments : Rs. Rs; A 30,000 B 26,000 C 24,000 80,000 Service Departments : Rs. Rs. Stores 4,000 _____ Time-keeping and Accounts 3,000 Power 1,600 Canteen 1,000 9,600

4.13

Page 230: 30510870 Cost Accounting and Financial Management

Cost Accounting The following information is also available in respect of the production departments :

Dept. A Dept. B Dept. C Horse power of Machine 300 300 200 Number of workers 20 15 15 Value of stores requisition in (Rs.) 2,500 1,500 1,000 Apportion the costs of service departments over the production departments.

Solution Secondary Overhead Distribution Statement

Item of cost Basis of Total Production Depts (as per primary apportionment distribution A B C summary) Rs. Rs. Rs. Rs. Cost as per primary 80,000 30,000 26,000 24,000 distribution summary Stores Value of stores re- quisition : (5:3:2) 4,000 2,000 1,200 800 Time-keeping No. of and Accounts workers (4:3:3) 3,000 1,200 900 900 Power H.P. of M/c’s (3:3:2) 1,600 600 600 400 Canteen No. of workers (4:3:3) 1,000 400 300 300 89,600 34,200 29,000 26,400

(ii) Step Method or Non-reciprocal method : This method gives cognizance to the service rendered by service department to another service department. Therefore, as

4.14

Page 231: 30510870 Cost Accounting and Financial Management

Overheads

compared to previous method, this method is more complicated because a sequence of apportionments has to be selected here. The sequence here begins with the department that renders service to the maximum number of other service departments. In other words the cost of the service department which serves the largest number of other service and production departments, is distributed first. After this, the cost of service department serving the next largest number of departments is apportioned. This process continues till the cost of last service department is apportioned. The cost of last service department is apportioned among production departments only. Some authors are of the view that the cost of service department with largest amount of cost should be distributed first. Refer to the illustration which follows to understand this method.

Illustration Suppose the expenses of two production departments A and B and two service departments X and Y are as under :

Amount Apportionment Basis Rs. Y A B X 2,000 25% 40% 35% Y 1,500 — 40% 60% A 3,000 B 3,200

Solution Summary of Overhead Distribution

Departments X Y A B Rs. Rs. Rs. Rs.

Amount as given above 2,000 1,500 3,000 3,200 Expenses of X Dept. apportioned over Y,A and B Dept. in the ratio (5:8:7) —2,000 500 800 700 Expenses of Y Dept. apportioned over A

4.15

Page 232: 30510870 Cost Accounting and Financial Management

Cost Accounting and B Dept. in the ratio (2:3) - —2,000 800 1,200Total Nil Nil 4,600 5,100 (iii) Reciprocal Service Method : This method recognises the fact that where there are two or more service departments they may render services to each other and, therefore, these inter-departmental services are to be given due weight while re-distributing the expenses of the service departments. The methods available for dealing with reciprocal services are : (a) Simultaneous equation method ;

(b) Repeated distribution method ;

(c) Trial and error method. (a) Simultaneous equation method : According to this method firstly, the costs of service departments are ascertained. These costs are then re-distributed to production departments on the basis of given percentages. (Refer to the following illustration to understand the method) Illustration

Service departments expenses Rs. Boiler House 3,000 Pump Room 600 3,600 The allocation is :

Production Departments Boiler House Pump Room A B Boiler House 60% 35% – 5% Pump Room 10% 40% 50% –

Solution The total expenses of the two service departments will be determined as follows : Let B stand for Boiler House expenses and P for Pump Room expenses.

4.16

Page 233: 30510870 Cost Accounting and Financial Management

Overheads

Then B = 3,000 + 1/2 P P = 600 + 1/20 B Substituting the value of B, P = 600 + 1/20 (3,000 + 1/2 P) = 600 + 150 + 1/40 P = 750 + 1/40 P 40 P = 30,000 + P 39 P = 30,000 P = Rs. 769 (approx.) The total of expenses of the Pump Room are Rs. 769 and that of the Boiler House is Rs. 3,385 i.e., Rs. 3,000 + 1/2 × Rs. 769. The expenses will be allocated to the production departments as under :

Production departments : A B Rs. Rs. Boiler House (60% and 35% of Rs. 3,385) 2,031 1,185 Pump Room (10% and 40% of Rs. 769) 77 307Total 2,108 1,492

The total of expenses apportioned to A and B is Rs. 3,600. (b) Repeated distribution method : Under this method, service departments costs are distributed to other service and production departments on agreed percentages and this process continues to be repeated, till the figures of service departments are either exhausted or reduced to too small a figure. (Refer to the following illustration to understand this method) Illustration PH Ltd., is a manufacturing company having three production departments, ‘A’, ‘B’ and ‘C’ and two service departments ‘X’ and ‘Y’. The following is the budget for December 2005 :-

Total A B C X Y Rs. Rs. Rs. Rs. Rs. Rs. Direct material 1,000 2,000 4,000 2,000 1,000 Direct wages 5,000 2,000 8,000 1,000 2,000

4.17

Page 234: 30510870 Cost Accounting and Financial Management

Cost Accounting Factory rent 4,000 Power 2,500 Depreciation 1,000 Other overheads 9,000 Additional information : Area (Sq. ft.) 500 250 500 250 500 Capital value (Rs. lacs) of assets 20 40 20 10 10 Machine hours 1,000 2,000 4,000 1,000 1,000 Horse power of machines 50 40 20 15 25 A technical assessment of the apportionment of expenses of service departments is as under :

A B C X Y % % % % % Service Dept. ‘X’ 45 15 30 – 10 Service Dept. ‘Y’ 60 35 – 5 – Required : (i) A statement showing distribution of overheads to various departments. (ii) A statement showing re-distribution of service departments expenses to production

departments. (iii) Machine hour rates of the production departments ‘A’, ‘B’ and ‘C’. Solution (i) Overhead Distribution Summary

Basis Total A B C X Y Rs. Rs. Rs. Rs. Rs. Rs. Direct materials Direct – – – – 2,000 1,000 Direct wages Direct – – – – 1,000 2,000 Factory rent Area 4,000 1,000 500 1,000 500 1,000 Power H.P. × M/c Hrs. 2,500 500 800 800 150 250 Depreciation Cap., value 1,000 200 400 200 100 100 Other overheads M/c hrs. 9,000 1,000 2,000 4,000 1,000 1,000 2,700 3,700 6,000 4,750 5,350

4.18

Page 235: 30510870 Cost Accounting and Financial Management

Overheads

(ii) Redistribution of Service Department’s expenses :

A B C X Y

Rs. Rs. Rs. Rs. Rs.

Total overheads 2,700 3,700 6,000 4,750 5,350 Dept. X overhead apportioned in the ratio (45:15:30:—:10) 2,138 712 1,425 –4,750 475 Dept. Y overhead apportioned

in the ratio (60: —35:—:5) 3,495 2,039 − 291 –5,825 Dept. X overhead apportioned in the ratio (45:15:30:—:10) 131 44 87 –291 29 Dept. Y overhead apportioned

in the ratio (60:35:—:5:—) 17 10 − 2 –29 Dept. X overhead apportioned

in the ratio (45:15:30:—:10) 1 − 1 –2 −

8,482 6,505 7,513 − − (iii) Machine hour rate :

Machine hours 1,000 2,000 4,000

Machine hour rate (Rs.) 8.48 3.25 1.88

(Rs. 8,482/1,000 hrs) (Rs. 6,505/2,000 hrs.) (Rs. 7,513/4,000 hrs.) (c) Trial and error method - According to this method the cost of one service Cost Centre is apportioned to another service Cost Centre. The cost of another service centre plus the share received from the first Cost Centre is again apportioned to the first cost centre. This process is repeated till the amount to be apportioned becomes negligible. (Refer to the following illustration to understand this method.)

4.19

Page 236: 30510870 Cost Accounting and Financial Management

Cost Accounting Illustration The ABC Company has the following account balances and distribution of direct charges on 31st March, 2005.

Total Production Depts. Service Depts. Machine Packing Gen. Store & Shop Plant Maintenanace Allocated Overheads : Rs. Rs. Rs. Rs. Rs. Indirect labour 14,650 4,000 3,000 2,000 5,650 Maintenance material 5,020 1,800 700 1,020 1,500 Misc. supplies 1,750 400 1,000 150 200 Superintendent’s salary 4,000 – – 4,000 – Cost & payroll salary 10,000 – – 10,000 – Overheads to be apportioned : Power 8,000 Rent 12,000 Fuel and heat 6,000 Insurance 1,000 Taxes 2,000 Depreciation 1,00,000

1,64,420 6,200 4,700 17,170 7,350 The following data were compiled by means of the factory survey made in the previous year :

Floor Radiator No. of Investment H.P Space Sections Employees Rs. hours Machine Shop 2,000 Sq. ft. 45 20 6,40,000 3,500 Packing 800 ” ” 90 10 2,00,000 500 General Plant 400 ” ” 30 3 10,000 – Store & Maint. 1,600 ” ” 60 5 1,50,000 1,000

4,800 ” ” 225 38 10,00,000 5,000

4.20

Page 237: 30510870 Cost Accounting and Financial Management

Overheads

Expenses charged to the stores and maintenance departments are to be distributed to the other departments by the following percentages : Machine shop 50%; Packing 20%; General Plant 30% ; General Plant overheads is distributed on the basis of number of employees : (a) Prepare an overhead distribution statement with supporting schedules to show computations and basis of distribution including distribution of the service department expenses to producing department. (b) Determine the service department distribution by the method of continued distribution. Carry through 3 cycles. Show all calculations to the nearest rupee. Solution (a) Overhead Distribution Statement Production Departments Service Departments Machine Packing General Stores & Shop Plant Maintenance

Allocated Expenses: Rs. Rs. Rs. Rs. Indirect labour 4,000 3,000 2,000 5,650 Maintenance material 1,800 700 1,020 1,500 Superintendent’s salary − − 4,000 − Misc. supplies 400 1,000 150 200 Cost & payroll salaries − − 10,000 −

Total 6,200 4,700 17,170 7,350 Apportioned expenses (See schedule below) 77,720 25,800 2,830 22,650

Total 83,920 30,500 20,000 30,000 Schedule of Apportioned Expenses

Item Basis Machine Packing General Stores & Shop Plant Maintenance Rs. Rs. Rs. Rs. Power Horse Power Hrs. 5,600 800 – 1,600 Rent Floor Space 5,000 2,000 1,000 4,000 Fuel & Heat Radiator Secs. 1,200 2,400 800 1,600

4.21

Page 238: 30510870 Cost Accounting and Financial Management

Cost Accounting

Insurance Investment 640 200 10 150 Taxes Investment 1,280 400 20 300 Depreciation Investment 64,000 20,000 1,000 15,000

Total 77,720 25,800 2,830 22,650

(b) Distribution of Service Department Expenses Production Departments Service Departments

Machine Packing General Stores & Plant Maintenance

Rs. Rs. Rs. Rs. Total Expense [as per (a)] 83,920 30,500 20,000 30,000 Transfer from Stores & Maintenance 15,000 6,000 9,000 –30,000 Transfer from General Plant 16,571 8,286 –29,000 4,143 Transfer from Stores & Maintenance 2,072 829 1,242 –4,143 Transfer from General Plant 710 355 –1,242 177 Transfer from Stores & Maintenance 88 36 53 –177 Transfer from General Plant 35 18 –53 — Total 1,18,396 46,024 — —

Illustration Modern Manufactures Ltd. have three Production Departments P1, P2, P3 and two Service Departments S1 and S2 details pertaining to which are as under :

P1 P2 P3 S1 S2 Direct wages (Rs.) 3,000 2,000 3,000 1,500 195 Working hours 3,070 4,475 2,419 - - Value of machines (Rs.) 60,000 80,000 1,00,000 5,000 5,000 H.P. of machines 60 30 50 10 - Light points 10 15 20 10 5 Floor space (sq. ft.) 2,000 2,500 3,000 2,000 500

4.22

Page 239: 30510870 Cost Accounting and Financial Management

Overheads

The following figures extracted from the Accounting records are relevant:

Rs. Rent and Rates 5,000 General Lighting 600 Indirect Wages 1,939 Power 1,500 Depreciation on Machines 10,000 Sundries 9,695 The expenses of the Service Departments are allocated as under :

P1 P2 P3 S1 S2

S1 20% 30% 40% - 10%

S2 40% 20% 30% 10% -

Find out the total cost of product X which is processed for manufacture in Departments P1, P2 and P3 for 4, 5 and 3 hours respectively, given that its Direct Material Cost is Rs. 50 and Direct Labour Cost is Rs. 30. Solution

Statement Showing Distribution of Overheads of Modern Manufacturers Ltd.

Particulars Production Depts Service Depts.

Basis Total P1 P2 P3 S1 S2

Rs. Rs. Rs. Rs. Rs. Rs. Rent & Rates Area 5,000 1,000 1,250 1,500 1,000 250 General lighting Light points 600 100 150 200 100 50 Indirect wages Direct wages 1,939 600 400 600 300 39 Power H.P. 1,500 600 300 500 100 − Depreciation of Value of machines machines 10,000 2,400 3,2004,000 200 200 Sundries Direct Wages 9,695 3,000 2,0003,000 1,500 195 ______ _____ _____ _____ _____ ____ 28,734 7,700 7,300 9,800 3,200 734

4.23

Page 240: 30510870 Cost Accounting and Financial Management

Cost Accounting

Redistribution of Service Department’s Expenses Over Production Departments

Production Depts. Service Depts.

Total P1 P2 P3 S1 S2

Rs. Rs. Rs. Rs. Rs. Rs. Total Overheads 28,734 7,700 7,300 9,800 3,200 734 Dept. S1 Overheads apportioned 3,200 640 960 1,280 –3,200 320

in the ratio (20:30:40:—:10) Dept. S2 Overheads apportioned 1,054.00 421.60 210.80 316.20 105.40 –1,054

in the ratio (40:20:30:10:—) Dept. S1 Overheads apportioned 105.40 21.08 31.62 42.16 –105.40 10.54

in the ratio (20:30:40:—:10) Dept. S2 Overheads apportioned 10.54 4.22 2.11 3.16 1.05 –10.54

in the ratio (40:20:30:10:—) Dept. S1 Overheads apportioned 1.05 0.21 0.32 0.42 –1.0 50.10

in the ratio (20:30:40:—:10) Dept. S2 Overheads apportioned 0.10 0.05 0.02 0.03 – −0.10

in the ratio (40:20:30:10:—) Total 8,787.16 8,504.87 11,441.79 Working hours 3,075.00 4,475.00 2,419.00 Rate per hour 2.86 1.90 4.73

(See Working Note 1) Cost of the product ‘X’ Rs. Direct material cost 50.00 Direct labour cost 30.00 Overhead cost 35.13 115.13 Working Note : 1. Working rate per hour for production department

P1 = 075,3

16.787,8.Rs = Rs. 2.86

4.24

Page 241: 30510870 Cost Accounting and Financial Management

Overheads

Similarly production rate for production departments P2 and P3 are Rs. 1.90 and Rs. 4.73.

2. Overhead cost Rs. 2.86 × 4 hrs. + Rs. 1.90 × 5 hrs. + Rs. 4.73 × 3 hrs. = Rs. 11.44 + Rs. 9.50 + Rs. 14.19 = Rs. 35.13 Note

The service departments have only indirect costs which are to be absorbed by production departments. However if the direct wages appearing in the question are assumed to be incurred on the service departments only, which have not been accounted for, by any other activity carried on in the service departments, then total expenses of the service departments including the aforesaid direct wages should be charged in the respective production departments. If this assumption holds good the alternative solution can appear as under.

Alternative Solution

Statement Showing Distribution of Overheads of Modern Manufactures Ltd.

Production Service Departments Departments

Particulars Basis Total P1 P2 P3 S1 S2

Rs. Rs. Rs. Rs. Rs. Rs. Direct wages Actual 1,695 - - - 1,500 195 Rent & rates Area 5,000 1,000 1,250 1,500 1,000 250 General lighting Light points 600 100 150 200 100 50 Indirect wages Direct wages 1,939 600 400 600 300 39 Power H.P. 1,500 600 300 500 100 − Depreciation Value of machines of machines 10,000 2,400 3,200 4,000 200 200 Sundries Direct wages 9,695 3,000 2,000 3,000 1,500 195

30,429 7,700 7,300 9,800 4,700 929

4.25

Page 242: 30510870 Cost Accounting and Financial Management

Cost Accounting

Redistribution of Service Department’s Expenses over Production Departments

Total P1 P2 P3 S1 S2

Rs. Rs. Rs. Rs. Rs. Rs.

Total Overheads 30,429 7,700 7,300 9,800 4,700 929 Dept. S1 Overheads apportioned 4,700 940 1,410 1,880 –4,700 470

in the ratio: (20:30:40:—:10) Dept. S2 overheads apportioned 1,399.00 559.60 279.80 419.70 139.90 –1,399.00

in the ration :(40:20:30:10:—) Dept. S1 overheads apportioned 139.90 27.98 41.97 55.96 –139.90 13.99

in the ratio (20:30:40:—:10) Dept. S2 overheads apportioned 13.99 5.60 2.80 4.20 1.40 –13.99

in the ratio (40:20:30:10:—) Dept. S1 overheads apportioned 1.40 0.28 0.42 0.56 –1.40 0.14

in the ratio (20:30:40:—:10) Dept. S2 overheads apportioned 0.14 0.06 0.03 0.05 –0.14

in the ratio (40:20:30:10:—) Total 9,233.52 9,035.02 12,160.47 Working hours 3,070.00 4,475.00 2,419.00 Working rate per hour 3.00 2.02 5.03 Cost of the Product ‘X’ Rs. P. Direct material cost 50.00 Direct labour cost 30.00 Overhead cost (See working note) 37.19 117.25 Working note : Overhead cost :

Rs. 3 × 4 hrs. + Rs. 2.02 × 5 hrs. + Rs. 5.03 × 3 hrs. = Rs. 12 + Rs. 10.10 + Rs. 15.09 = Rs. 37.19

Illustration : Deccan Manufacturing Ltd., have three departments which are regarded as

4.26

Page 243: 30510870 Cost Accounting and Financial Management

Overheads

production departments. Service departments’ costs are distributed to these production departments using the “Step Ladder Method” of distribution. Estimates of factory overhead costs to be incurred by each department in the forthcoming year are as follows. Data required for distribution is also shown against each department:

Department Factory overhead Direct labour No. of Area in Rs. hours employees sq.m. Production: X 1,93,000 4,000 100 3,000 Y 64,000 3,000 125 1,500 Z 83,000 4,000 85 1,500 Service: P 45,000 1,000 10 500 Q 75,000 5,000 50 1,500 R 1,05,000 6,000 40 1,000 S 30,000 3,000 50 1,000 The overhead costs of the four service departments are distributed in the same

order, viz., P,Q,R and S respectively on the following basis. Department Basis P Number of employees Q Direct labour hours R Area in square metres S Direct labour hours

You are required to : (a) Prepare a schedule showing the distribution of overhead costs of the four

service departments to the three production departments; and (b) Calculate the overhead recovery rate per direct labour hour for each of the three

production departments.

4.27

Page 244: 30510870 Cost Accounting and Financial Management

Cost Accounting Solution (a) Deccan Manufacturing Limited

Schedule Showing the Distribution of Overhead Costs among Departments Service Production

P Q R S X Y Z Rs. Rs. Rs. Rs. Rs. Rs. Rs. Overhead costs 45,000 75,000 1,05,000 30,000 1,93,000 64,000 83,000 Distribution of over- head cost of Dept. ‘P’ (45,000) 5,000 4,000 5,000 10,000 12,500 8,500 Distribution of over- head costs of Dept. ‘Q’ - (80,000) 24,000 12,000 16,000 12,000 16,000 Distribution of over- head cost of Dept. ‘R’ - - (1,33,000) 19,000 57,000 28,500 28,500 Distribution of over- head costs of Dept. ‘S’ - - - (66,000) 24,000 18,000 24,000 Total (A) 3,00,000 1,35,000, 1,60,000

(b) Direct labour hours (B) 4,000 3,000 4,000

Overhead recovery rate per hour )B()A( Rs. 75 Rs. 45 Rs. 40

Illustration A Ltd., manufactures two products A and B. The manufacturing division consists of

two production departments P1 and P2 and two service departments S1 and S2.

Budgeted overhead rates are used in the production departments to absorb factory overheads to the products. The rate of Department P1 is based on direct machine hours, while the rate of Department P2 is based on direct labour hours. In applying overheads, the pre-determined rates are multiplied by actual hours.

For allocating the service department costs to production departments, the basis adopted is as follows : (i) Cost of Department S1 to Department P1 and P2 equally, and

(ii) Cost of Department S2 to Department P1 and P2 in the ratio of 2 : 1 respectively.

4.28

Page 245: 30510870 Cost Accounting and Financial Management

Overheads

The following budgeted and actual data are available: Annual profit plan data : Factory overheads budgeted for the year:

Rs. Rs. Departments P1 25,50,000 S1 6,00,000

P2 21,75,000 S2 4,50,000

Budgeted output in units : Product A 50,000; B 30,000. Budgeted raw-material cost per unit : Product A Rs. 120; Product B Rs. 150. Budgeted time required for production per unit :

Department P1 : Product A : 1.5 machine hours

Product B : 1.0 machine hour Department P2 : Product A : 2 Direct labour hours

Product B : 2.5 Direct labour hours Average wage rates budgeted in Department P2 are :

Product A - Rs. 72 per hour and Product B – Rs. 75 per hour. All materials are used in Department P1 only.

Actual data : (for the month of July, 2005) Units actually produced : Product A : 4,000 units Product B : 3,000 units Actual direct machine hours worked in Department P1 :

On product A 6,100 hours, Product B 4,150 hours. Actual direct labour hours worked in Department P2 :

on product A 8,200 hours, Product B 7,400 hours.

4.29

Page 246: 30510870 Cost Accounting and Financial Management

Cost Accounting

Costs actually incurred: Product A Product B Rs. Rs. Raw materials 4,89,000 4,56,000 Wages 5,91,900 5,52,000 Rs. Rs. Overheads : Department P1 2,31,000 S1 60,000

P2 2,04,000 S2 48,000

You are required to : (i) Compute the pre-determined overhead rate for each production department. (ii) Prepare a performance report for July, 2005 that will reflect the budgeted costs

and actual costs. Solution :

(i) Computation of predetermined overhead rate for each production department from budgeted data

Production Deptts. Service Deptts. P1 P2 S1 S2

Budgeted factory overheads for the year in (Rs.) 25,50,000 21,75,000 6,00,000 4,50,000 Allocation of service department S1’s costs to production departments

P1 and P2 equally in (Rs.) 3,00,000 3,00,000 –6,00,000 –

Allocation of service department S2’s costs to production departments

P1 and P2 in the ratio of 2:1 in (Rs.) 3,00,000 1,50,000 – –4,50,000

Total (Rs.) 31,50,000 26,25,000 Nil Nil Budgeted machine hours in department P1 1,05,000

(Refer to working note 1)

4.30

Page 247: 30510870 Cost Accounting and Financial Management

Overheads

Budgeted labour hours in department P2 1,75,000

(Refer to working note 2) Budgeted machine hour rate (Rs. 31,50,000/1,05,000) Rs. 30 Budgeted labour hour rate (Rs. 26,25,000/1,75,000) Rs. 15

(ii) Performance report for July, 2005 (When 4,000 and 3,000 units of products A and B respectively were actually produced)

Budgeted Actual Rs. Rs. Raw materials used in department P1

A : 4,000 units × Rs. 120 4,80,000 4,89,000 B : 3,000 units × Rs. 150 4,50,000 4,56,000 Direct labour Cost on the basis of labour hours worked in department P2

A : 4,000 units × 2 hrs. × Rs. 72 5,76,000 5,91,900 B : 3,000 units × 2.5 hrs. × Rs. 75 5,62,500 5,52,000 Overhead absorbed on machine hour basis in department P1

A : 4,000 units × 1.5 hrs. × Rs. 30 1,80,000 1,74,400* B : 3,000 units × 1 hr. × Rs. 30 90,000 1,18,649 Overhead absorbed on labour hour basis in department P2

A : 4,000 units × 2 hrs. × Rs. 15 1,20,000 1,31,364 ** B : 3,000 units × 2.5 hrs. × Rs. 15 1,12,500 1,18,548 25,71,000 26,31,861* (Refer to working note 4) ** (Refer to working note 5)

4.31

Page 248: 30510870 Cost Accounting and Financial Management

Cost Accounting Working notes :

Product A Product B Total 1. Budgeted output 50,000 30,000 (in units) Budgeted machine hours 75,000 30,000 1,05,000 in department P1 (50,000×1.5 hrs.) (40,000×1 hr.)

Budgeted labour hours 1,00,000 75,000 1,75,000

in department P2 (50,000×2 hrs.) (30,000×2.5 hrs.)

Product A Product B Total 2. Actual output (in units) 4,000 3,000

Actual machine hours utilised in department P1 6,100 4,150 10,250

Actual labour hours utilised in department P2 8,200 7,400 15,600

3. Computation of actual overhead rates for each production department from actual data

Production Deptts. Service Deptts.

P1 P2 S1 S2

Actual factory overheads for the month of July, 2005 in (Rs.) 2,31,000 2,04,000 60,000 48,000 Allocation of service department S1’s costs in (Rs.) over production

departments P1 and P2 equally 30,000 30,000 –60,000 −

Allocation of service department S2’s costs in (Rs.) over production

departments P1 and P2 in the

ratio 2 : 1 32,000 16,000 − –48,000

Total (Rs.) 2,93,000 2,50,000 Nil Nil

4.32

Page 249: 30510870 Cost Accounting and Financial Management

Overheads

Actual machine hours in department P1 10,250 − − − (Refer to working note 2) Actual labour hours in department P2 − 15,600 − − (Refer to working note 2) Machine hour rate Rs. 28.59 − − − (Rs. 2,93,000/10,250) Labour hour/rate − Rs. 16.02 − − (Rs. 2,50,000/15,600) 4. Actual overheads absorbed (based on machine hours) A : 6,100 hrs × Rs. 28.59 = Rs. 1,74,400 (say) B : 4,150 hrs × Rs. 28.59 = Rs. 1,18,649 (say) 5. Actual overheads absorbed (based on labour hours) A : 8,200 hrs × Rs. 16.02 = Rs. 1,31,364 B : 7,400 hrs × Rs. 16.02 = Rs. 1,18,548 4.4.5 Absorbing overheads over cost units, products, etc.: Collection of the figure of overheads for the factory as a whole or for various departments is not enough. It is clearly necessary to ascertain how much of the overheads is debitable to the cost of the various jobs, products etc. This process is called absorbing the overhead to cost units. We take up below the various implications of this process. However, if only one uniform type of work is done, the task is easy and under such a situation the overhead expenses to be absorbed may be calculated by dividing actual overheads by the number of units of work done or estimated overheads by the estimated output. Normal and pre-determined overhead rates : Various items of overhead expenses generally are not incurred uniformly throughout the accounting period e.g., insurance premium are paid annually, rates and taxes quarterly, etc. The monthly total of overhead expenses for each department or cost centre, therefore, may fluctuate from month to month. As such, monthly totals cannot be regarded as satisfactory. It being necessary to absorb the overhead costs in the cost of production of each lot or batch of production, as soon as its manufacture is complete an estimate of the total amounts of annual overhead expenses must be made in advance. Likewise, an advance estimate of the annual volume of production (in terms of the amount of direct wages, number of direct labour hours, etc.) must be made and, on that basis, a pre-determined overhead rate is calculated.

4.33

Page 250: 30510870 Cost Accounting and Financial Management

Cost Accounting The overhead rate of expenses for absorbing them to production may be estimated on the following three basis. (1) The figure of the previous year or period may be adopted as the overhead rate to be

charged to production in the current year. The assumption is that the value of production as well as overheads will remain constant or that the two will change, proportionately.

(2) The overhead rate for the year may be determined on the basis of estimated expenses and anticipated volume of production activity. For instance, if expenses are estimated at Rs. 10,000 and output at 4,000 units, the overhead rate will be Rs. 250 per unit.

(3) The overhead rate for a year may be fixed on the basis of the normal volume of the business.

If, in the example given above, the normal capacity is 5,000 units, the overhead rate will be Rs. 2. The first method is rather crude and is not likely to yield satisfactory results unless the undertaking is small and output and expenses are fairly constant over the period. In large concerns, conditions are rarely static and, hence, expenses fluctuate from one period to another. Therefore, so far as a large enterprise is concerned, the overhead rates of the past periods may not have much relevance for the purpose of arriving at the cost of production in the current period. The second method is based on the assumption that all expenses shall have to be recovered irrespective of the volume of output. From such an assumption, it follows that, if in any period there is a large idle or unused capacity, the entire amount of the overhead expenses shall have to be absorbed over the reduced volume of the output. A sizeable portion of the overhead expenses is made up of the fixed charges; if those fixed charges have to be included in the costs of the reduced output the incidence of overheads per unit of production would necessarily be high. Similarly, if the volume of output in any period exceeds the normal level, the incidence of fixed overheads per unit will be comparatively lower. The effect of this method is that during periods of falling production (and perhaps of falling prices), the cost of production would be higher and, correspondingly, during a period of rising production and perhaps rising prices), the cost of production would be low. This would be illogical idleness or a level of activity above the normal capacity is abnormal and should not affect costs; the Costing Profit and Loss Account is the place where the effect of such abnormal factors should be shown. It is also possible that when output falls and total cost per unit goes up (if expenses are absorbed over actual output), the firm may demand a price above the market price and may find itself without customers. On this consideration, the most appropriate basis for the computation of

4.34

Page 251: 30510870 Cost Accounting and Financial Management

Overheads

predetermined rate of overhead cost is the normal capacity of production. Estimation on this basis as suggested in the third method, has many advantages over the other two methods. It enables not only the computation of correct costs but also true cost to be recovered. In a competitive economy, the concept of costs is essentially that of normal costs. Normal costs should be the yard stick against which the efficiency or otherwise, of the competing unit should be measured. A cost that is correct need not necessarily be true. True costs are in a sense notional cost and the term “normal” is derived from the concept of normal costs. Hence, the use of normal overhead rates has a two fold advantage. It enables the true as well as correct costs to be calculated; at the same time by highlighting differences between the amount of overhead expenses actually incurred and those absorbed in production it provides useful guidance to the management in taking decisions as regards production and sales. Blanket and departmental overhead rates: Blanket overhead rate refers to the computation of one single overhead rate for the whole factory. It is to be distinguished from the departmental overhead rate which refers to a separate rate for each individual cost centre or department. The use of blanket rate may be proper in certain factories producing only one major product in a continuous process (e.g., chemical factories) or where the work performed in every department is fairly uniform or standardised. Where, however, the product lines are varied or machinery is used to a varying degree in the different departments, that is, where conditions throughout the factory are not uniform, the use of departmental rates is to be preferred. The working condition in the last mentioned case would be such that varying amount of expenses would be continually incurred by the several service departments and hence, the incidence of overhead cost of each department would be different. Since not all products would ordinarily undergo the same type or of different type of operations in different departments, the charging of a single overhead rate in such a case would give misleading results. It may therefore, be concluded that a blanket rate should be applied. (1) where only one major product is being produced. (2) where several products are produced, but

(a) all products pass through all departments; and (b) all products are processed for the same length of time in each department.

Where these conditions do not exist, departmental rates should be used.

4.5 METHODS OF ABSORBING OVERHEADS TO VARIOUS PRODUCTS OR JOBS Before we describe various methods, it would be better to know how to judge whether a method will give good results or not. The method selected for charging overheads to

4.35

Page 252: 30510870 Cost Accounting and Financial Management

Cost Accounting products or jobs should be such as will ensure : (i) that the total amount charged (or recovered) in a period does not differ materially

from the actual expenses incurred in the period. In other words, there should not be any significant over or under recovery of overhead; and

(ii) that the amount charged to individual jobs or products is equitable. In case of factory overhead, this means : (a) that the time spent on completion of each job should be taken into

consideration; (b) that a distinction should be made between jobs done by skilled workers and

those done by unskilled workers. Usually, the latter class of workers needs more supervision, causes greater wear and tear of machines and tools and waste a larger quantity of materials. Hence jobs done by such workers should bear a correspondingly higher burden for overheads; and

(c) that jobs done by manual labour and those done by machines should be distinguished. It stands to reason that no machine expenses should be charged to jobs done by manual labour.

In addition, the methods should (i) be capable of being used conveniently; and (ii) yield uniform result from period to period as far as possible; any change that is

apparent should reflect a change in the underlying situation such as substitution of human labour by machines.

Several methods are commonly employed either individually or jointly for computing the appropriate overhead rate to be employed. The more common of these are : (1) Percentage of direct materials. (2) Percentage of prime cost. (3) Percentage of direct labour cost. (4) Labour hour rate. (5) Machine hour rate. 4.5.1 & 4.5.2 Percentage of direct material and prime cost Suppose for a given period, actual figures are estimated as follows : Rs.

Direct materials 2,00,000

4.36

Page 253: 30510870 Cost Accounting and Financial Management

Overheads

Direct labour 1,00,000 Factory overheads 90,000

The percentage of factory overheads to direct materials will be 45%, to prime cost 30% and to direct labour 90%. If, on a job, material cost is Rs. 10,000 and direct labour is Rs. 7,000 the cost, after absorbing factory overhead, will be as follows :

(i) Rs. 17,000 + 45% Rs. 10,000 or Rs. 21,500, (ii) Rs. 17,000 + 30% Rs. 17,000 or Rs. 22,100, and (iii) Rs. 17,000 + 90% Rs. 7,000 or Rs. 23,300.

One can see how, with a different method, the works cost comes out to be different. Of these methods, the first and second are generally considered to be unsuitable on account of the following reasons : (i) Manufacturing overhead expenses are mostly a function of time i.e., time is the determining factor for the incurrence and application of manufacturing overhead expenses. That they are so would be clear if we recall that overhead expenses, specially manufacturing expenses, can in the ultimate analysis be regarded as expenditure incurred in providing the necessary facilities and service to workers employed in the productive process. The question of facilities and service made available to workers naturally is dependent on the length of time during which workers make use of the facilities. It may, therefore, be said that the job or product on which more time has been spent would entail larger manufacturing expenses than the job requiring less time. The factor is ignored altogether by the first method and largely by the second method. (ii) Overheads are neither related to the prime cost nor to direct material cost except to a very small extent. Thus, if the percentage of material cost is used when there are two jobs requiring the same operational time but using material having varying prices, their manufacturing overhead cost would be different whereas this should not normally be so. The method of absorbing overhead costs on the basis of prime cost also does not take into consideration the time factor. The fact that the amount includes labour cost in addition to material cost does not render the prime cost to be more suitable; infact, the results are liable to be more misleading because of the cumulative error of using both the labour and material cost as the basis of allocation of overhead expenses, on neither of which they are already dependent. (iii) Since material prices are prone to frequent and wide fluctuations, the manufacturing overheads, if based on material cost or prime cost, also would fluctuate violently from period to period. (iv) The skill of the workers involved and whether machines were used or not, are

4.37

Page 254: 30510870 Cost Accounting and Financial Management

Cost Accounting ignored when these methods are used. Percentage of materials cost may, however, be used for the limited purpose of absorbing material handling and store overheads. 4.5.3 Percentage of direct labour cost : This method also fails to give full recognition to the element of the time which is of prime importance in the accounting for and treatment of manufacturing overhead expenses except in so far as the amount of wages is a product of the rate factor multiplied by the time factor. Thus, the time factor is taken into consideration only indirectly or partially in the computation of the overhead percentage rate. This method therefore, cannot be depended upon to produce very accurate results where the same type of work is performed in the same time by different type of workers, skilled and unskilled, with varying rates of pay. Also no distinction is made between jobs done by manual labour and those done by machines. Inspite of the inaccuracies which may arise under this method it is widely used in actual practice because it is simple and does not involve much calculation. If on the other hand, a more scientific method is employed e.g., the direct labour hour rate or the machine hour rate, more complexities in the overhead accounting procedure would be introduced, though the selected method will give proper allowance to time element. Thus, the advantage of elimination of a small error in practice may involve a heavy price on account of introduction of complexities. Advantages: (i) The method is simple and economical to apply. (ii) The time factor is given recognition even if indirectly. (iii) Total expenses recovered will not differ much from the estimated figure since total

wages paid are not likely to fluctuate much. Disadvantages: (i) It gives rise to certain inaccuracies due to the time factor not being given full

importance. (ii) Where machinery is used to some extent in the process of manufacture, an

allowance for such a factor is not made. (iii) It does not provide for varying skills of workers. 4.5.4 Labour hour rate: This method is an improvement on the percentage of direct wage basis, as it fully recognises the significance of the element of time in the incurring and absorption of manufacturing overhead expenses. This method is admirably suited to operations which do not involve any large use of machinery. To calculate labour hour rate, the amount of factory overheads is divided by the total number of direct labour hours.

4.38

Page 255: 30510870 Cost Accounting and Financial Management

Overheads

Suppose factory overheads are estimated at Rs. 90,000 and labour hours at 1,50,000. The overhead absorption rate will be Re. 0.60. If 795 direct labour hours are spent on a job, Rs. 477 will be absorbed as overhead. It can be calculated for each category of workers. 4.5.5 Machine hour rate : By the machine hour rate method, manufacturing overhead expenses are charged to production on the basis of number of hours machines are used on jobs or work orders. There is a basic similarity between the machine hour and the direct labour hour rate method insofar as both are based on the time factor. The choice of one or the other method is conditioned by the actual circumstance of the individual case. In respect of departments or operations in which machines predominate and the operators perform relatively a passive part, the machine hour rate is more appropriate. This is generally the case for operations or processes performed by costly machines, which are automatic or semi-automatic and where operators are essential merely for feeding them rather than for regulating the quantity of the output. In such case, the machine hour rate method alone can be depended on to correctly absorb the manufacturing overhead ex-penses to different items of production. What is needed for computing the machine hour rate is to divide overhead expenses for a specific machine or group of machines for a period by the operating hours of the machine or the group of machines for the period. Usually, the computation is made on the basis of the estimated expenses or the normal expenses for the coming period. Thus the machine hour rate usually is a predetermined rate. It is desirable to work out a rate for each individual machine; where a number of similar machines are working in a group, there may be single rate for the whole group. There are two methods of computing the machine hour rate. According to the first method, only the expenses directly or immediately connected with the operation of the machine are taken into account e.g., power, depreciation, repairs and maintenance, insurance, etc. The rate is calculated by dividing the estimated total of these expenses for a period by the estimated number of operational hours of the machines during the period. It will be obvious, however, that in addition to the expenses stated above there may still be other manufacturing expenses such as supervision charges, shop cleaning and lighting, consumable stores and shop supplies, shop general labour, rent and rates, etc. incurred for the department as a whole and, hence, not charged to any particular machine or group of machines. In order to see that such expenses are not left out of production costs, one should include a portion of such expenses to compute the machine hour rate. Alternatively, the overheads not directly related to machines may be absorbed on the basis of Productive Labour Hour Rate Method or any other suitable method.

4.39

Page 256: 30510870 Cost Accounting and Financial Management

Cost Accounting Illustration A machine costing Rs. 10,000 is expected to run for 10 years. At the end of this period its scrap value is likely to be Rs. 900. Repairs during the whole life of the machine are expected to be Rs. 18,000 and the machine is expected to run 4,380 hours per year on the average. Its electricity consumption is 15 units per hour, the rate per unit being 5 paise. The machine occupies one-fourth of the area of the department and has two points out of a total of ten for lighting. The foreman has to devote about one sixth of his time to the machine. The monthly rent of the department is Rs. 300 and the lighting charges amount to Rs. 80 per month. The foreman is paid a monthly salary of Rs. 960. Find out the machine hour rate, assuming insurance is @ 1% p.a. and the expenses on oil, etc., are Rs. 9 per month.

Solution Fixed expenses per month Rs. Rent (one fourth of the total) 75.00 Lighting (one fifth of the total) 16.00 Foreman’s salary (one sixth of the total) 160.00

Sundry expenses–oil, waste etc. 9.00 Insurance (1% on the value of the machine per year) 8.33 Total constant expenses per month 268.33 Total number of hours per annum 4,380 Total number of hours per month 365 Rs. Rs.

Fixed expenses per hour hrs. 365268.33 Rs.

0.735

Variable expenses per hour : Depreciation : Cost of the machine 10,000 Less: Scrap value 900 9,100 Depreciation per annum 910

Depreciation per hour: hrs. 4,380910

0.208

4.40

Page 257: 30510870 Cost Accounting and Financial Management

Overheads

Repairs for the whole life 18,000

for one hour years 10×4,380Rs.18,000

0.411

Electricity for one hour : 15 units @ 0.05 P 0.750Machine hour rate : 2.104

Illustration Gemini Enterprises undertakes three different jobs A, B and C. All of them require the use of a special machine and also the use of a computer. The computer is hired and the hire charges work out to Rs. 4,20,000 per annum. The expenses regarding the machine are estimated as follows : Rs. Rent for the quarter 17,500 Depreciation per annum 2,00,000 Indirect charges per annum 1,50,000 During the first month of operation the following details were taken from the job register: Job A B C Number of hours the machine was used : (a) Without the use of the computer 600 900 — (b) With the use of the computer 400 600 1,000 You are required to compute the machine hour rate : (a) For the firm as a whole for the month when the computer was used and when the

computer was not used. (b) For the individual jobs A, B and C. Solution Working notes: (i) Total machine hours used 3,500 (600 + 900 + 400 + 600 + 1,000) (ii) Total machine hours without the use of computers 1,500 (600 + 900)

4.41

Page 258: 30510870 Cost Accounting and Financial Management

Cost Accounting (iii) Total machine hours with the use of computer 2,000 (400 + 600 + 1,000) Rs. (iv) Total overheads of the machine per month Rent (Rs. 17,500 ÷ 3 months) 5,833.33 Depreciation (Rs. 2,00,000 ÷ 12 months) 16,666.67 Indirect Charges (Rs. 1,50,000 ÷ 12 months) 12,500.00 Total 35,000.00(v) Computer hire charges for a month = Rs. 35,000 (Rs. 4,20,000 ÷ 12 months) (vi) Overheads for using machines without computer

= hrs. 3,50035,000 Rs.

× 1,500 hrs. = Rs. 15,000

(vii) Overheads for using machines with computer

= hrs. 3,50035,000 Rs.

2,000 hrs. + Rs. 35,000 = Rs. 55,000

(a) Machine hour rate of Gemini Enterprises for the firm as a whole for a month.

(1) When the Computer was used : hours 2,00055,000 Rs.

= Rs. 27.50 per hour

(2) When the computer was note used : hrs. 3,50035,000 Rs.

= Rs. 10 per hour

(b) Machine hour rate for individual job

Rate per hr. Job Rate per hr. A B C

Rs. Hrs. Rs. Hrs. Rs. Hrs. Rs. Overheads Without Computer 10.00 600 6,000 900 9,000 − − With computer 27.50 400 11,000 600 16,500 1,000 27,500 1,000 17,000 1,500 25,500 1,000 27,500 Machine hour rate Rs. 17 Rs. 17 Rs. 27.50

4.42

Page 259: 30510870 Cost Accounting and Financial Management

Overheads

Illustration A machine shop has 8 identical Drilling machines manned by 6 operators. The machine cannot be worked without an operator wholly engaged on it. The original cost of all these machines works out to Rs. 8 lakhs. These particulars are furnished for a 6 months period:

Normal available hours per month 208 Absenteeism (without pay) hours 18 Leave (with pay) hours 20 Normal idle time unavoidable-hours 10 Average rate of wages per worker for 8 hours a day. Rs. 20 Production bonus estimated 15% on wages Value of power consumed Rs. 8,050 Supervision and indirect labour Rs. 3,300 Lighting and electricity Rs. 1,200 These particulars are for a year Repairs and maintenance including consumables 3% of value of machines. Insurance Rs. 40,000 Depreciation 10% of original cost. Other sundry works expenses Rs. 12,000 General management expenses allocated Rs. 54,530. You are required to work out a comprehensive machine hour rate for the machine shop.

Solution Computation of comprehensive machine hour rate of machine shop Rs.

Operator’s wages 17,100 (Refer to working note 2) Production bonus 2,565 (15% on wages) Power consumed 8,050 Supervision and indirect labour 3,300 Lighting and electricity 1,200

4.43

Page 260: 30510870 Cost Accounting and Financial Management

Cost Accounting Repairs and maintenance 12,000 Insurance 20,000 Depreciation 40,000 Sundry works expenses 6,000 General management expenses 27,265 1,37,480

Machine hour rate = operation machines of Hoursshop machine of overheads Total

= hours 5.7600Rs.1,37,48

(Refer to working note 1)

= Rs. 23.87 Working notes. 1. Computation of hours, for which 6 operators are available for 6 months.

Normal available hours p.m. 208 per operator. Less: Absenteeism hours 18 Less: Leave hours 20 Less: Idle time hours 10 48 Utilisable hours p.m. per operator 160 Total utilisable hours for 6 operators and for 6 months are = 160 × 6 × 6 = 5,760 hours As machines cannot be worked without an operator wholly engaged on them therefore, hours for which 6 operators are available for 6 months are the hours for which machines can be used. Hence 5,760 hours represent total machine hours.

2. Computation of operator’s wages

Average rate of wages : hours 820 Rs.

= Rs. 2.50 per hour

4.44

Page 261: 30510870 Cost Accounting and Financial Management

Overheads

Hours per month for which wages are paid to a worker (208 hours – 18 hours) = 190 hours. Total wages paid to 6 operators for 6 months = 190 hours × 6 × 6 × Rs. 2.50 = Rs. 17,100

Advantages of Machine Hour Rate : (1) Where machinery is the main factor of production, it is usually the best method of

charging machine operating expenses to production. (2) The under-absorption of machine overheads would indicate the extent to which the

machines have been idle. (3) It is particularly advantageous where one operator attends to several machines (e.g.

automatic screw manufacturing machine), or where several operators are engaged on the machine e.g. the belt press used in making conveyer belts.

Disadvantages : (1) Additional data concerning the operation time of machines, not otherwise necessary,

must be recorded and maintained. (2) As general department rates for all the machines in a department may be suitable, the

computation of a separate machine hour rate for each machine or group of machines would mean further additional work.

Note: Some people even prefer to add the wages paid to the machine operator in order to get a comprehensive rate of working a machine for one hour.

If all expenses are not allocated to machines, it will be necessary to calculate another rate for charging the general department expenses to production. This second rate can be calculated on the basis of direct labour hours. In effect therefore, both the machine hour and the direct labour hour rate will be applied, though separately.

Illustration Job No. 198 was commenced on October 10, 2005 and completed on November 1, 2005. Materials used were Rs. 600 and labour charged directly to the job was Rs. 400. Other information is as follows: Machine No. 215 used for 40 hours, the machine hour rate being Rs. 3.50. Machine No. 160 used for 30 hours, the machine hour rate being Rs. 4.00. 6 welders worked on the job for five days of 8 hours each : the Direct labour hour per welder is 20P. Expenses not included for calculating the machine hour or direct labour hour rate totalled

4.45

Page 262: 30510870 Cost Accounting and Financial Management

Cost Accounting Rs. 2,000,total direct wages for the period being Rs. 20,000. Ascertain the works costs of job No. 198.

Solution Rs.

Materials 600.00 Direct labour 400.00 1,000.00 Factory overheads : Rs. Machine No. 215 : 40 hours @ Rs. 3.50 140.00 Machine No. 160 : 30 hours @ Rs. 4.00 120.00

2401 hours of welders @ 20 P. per hr. 48.00

General2 10% of wages 40.00 348.00 Works cost 1,348.001. 6×5×8=240 2. Unapportioned expenses Rs. 2,000 which works out at 10% of direct wages.

4.6 TREATMENT OF UNDER-ABSORBED AND OVER-ABSORBED OVERHEADS IN COST ACCOUNTING : Overhead expenses are usually applied to production on the basis of pre-determined rates. Production overheads are to be determined in advance as follows for fixing selling price, quote tender price and to formulate budgets etc.

Pre-determined overhead rate = period the during units of Number Budgeted period the for overheads NormalEstimated/

The actual overhead rate will rarely coincide with the pre-determined overhead rate, due to variation in pre-determined overhead rate and actual overhead rate. Such a variation may arise due to any one of the following situations: (i) Estimated overheads for the period under consideration may remain the same or they

coincide with actual overheads but the number of units produced during the period is either more or less in comparison with budgeted figure. In the former case actual overhead rate will be less and in the latter case, actual overhead rate will be more than the pre-determined overhead rate, hence over-absorption and under-absorption will occur respectively.

(ii) Similarly, if the number of units actually produced during the period remains the

4.46

Page 263: 30510870 Cost Accounting and Financial Management

Overheads

same as budgeted figure but the actual overheads incurred are more or less than the estimated overheads for the period, then a situation of under-absorption or over-absorption will arise respectively.

(iii) If changes occur in different proportion both in the actual overheads and in the number of units produced during the period, then a situation of under or over-absorption (depending upon the situation) will arise.

(iv) If the changes in the numerator (i.e. in actual overheads) and denominator (i.e. in number of units produced) occur uniformly (without changing the proportion between the two) then a situation of neither under nor of over-absorption will arise.

Such over or under-absorption as arrived at under different situations may also be termed as overhead variance. The amount of over-absorption being represented by a credit balance in the account and conversely, the amount of under-absorption being a debit balance. As regards the treatment of such debit or credit balances, the general view is that if the balances are small they should be transferred to the Costing Profit and Loss Account and the cost of individual products should not be increased or reduced as these would be representing normal cost. Where, however the difference is large and due to wrong estimation, it would be desirable to adjust the cost of products manufactured, as otherwise the cost figures would convey a misleading impression. Such adjustments usually take the form of supplementary rates where there is a debit balance in the overhead account and a credit in the other case. Now, the production of any period can be identified in three forms, goods finished and sold, goods finished but held in stock (not yet sold) and semi-finished goods (work in progress). So far as the first category of goods is concerned, it is arguable that the post-mortem of the costs of individual products long after they have been sold may have some academic utility but it is frequently devoid of any practical significance. Therefore, it is suggested that the total variance concerning goods finished and sold should be adjusted by transferring the amount to the Cost of Sale Account, the costs of the individual items of such goods not being affected. As regards the variance pertaining to goods finished and held in stock (i.e. not yet sold), it would be necessary to adjust the value of the stock; similarly the value of work-in-progress should be adjusted. However, over or under recovery of overheads due to abnormal reasons (such as abnormal over or under capacity utilisation) should be transferred to the Costing Profit and Loss Account.

4.47

Page 264: 30510870 Cost Accounting and Financial Management

Cost Accounting

Illustration A light engineering factory fabricates machine parts to customers. The factory commenced fabrication of 12 Nos. machine parts to customers’ specifications and the expenditure incurred on the job for the week ending 21st August, 2005 is given below:

Rs. Rs. Direct materials (all items) 78.00 Direct labour (manual) 20 hours @ Rs. 1.50 per hour 30.00 Machine facilities : Machine No. I : 4 hours @ Rs. 4.50 18.00 Machine No. II : 6 hours @ Rs. 6.50 39.00 57.00 Total 165.00 Overheads @ Rs. 0.80 per hour on 20 manual hours 16.00 Total cost 181.00

The overhead rate of Re. 0.80 per hour is based on 3,000 man hours per week; similarly, the machine hour rates are based on the normal working of Machine Nos. I and II for 40 hours out of 45 hours per week. After the close of each week, the factory levies a supplementary rate for the recovery of full overhead expenses on the basis of actual hours worked during the week. During the week ending 21st August, 2005, the total labour hours worked was 2,400 and Machine Nos. I and II had worked for 30 hours and 32½ hours respectively. Prepare a Cost Sheet for the job for the fabrication of 12 Nos. machine parts duly levying the supplementary rates.

Solution Fabrication of 12 Nos. machine parts (job No......) Date of commencement : 16 August, 2005 Date of Completion. Cost sheet for the week ending, August 21, 2005 :

Rs. Materials 78.00 Labour 20 hours @ Rs. 1.50 30.00 Machine facilities : Rs. Machine No. I : 4 hours @ Rs. 4.50 18.00 Machine No. II : 6 hours @ Rs. 6.50 39.00 57.00

4.48

Page 265: 30510870 Cost Accounting and Financial Management

Overheads

Overheads 20 hours @ 0.80 P. per hour 16.00 181.00 Supplementary Rates Overheads 20 hours @ 20. P. per hour 4.00 Machine facilities : Machine No. I - 4 hours @ Re. 1.50 6.00 Machine No. II - 6 hours @ Re. 1.50 9.00 19.00Cost Rs. 200.00Working notes: Overheads budgeted : 3,000 hours @ 80 P. or Rs. 2,400 Actual hours : 2,400

Actual rate per hour Rs. 2,400/2,400 hours = Re. 1

Supplementary charge Re. 1 less 80 P, or 20 P per hour Machine facilities :

Machine No. I Machine No. II Budgeted (40 × Rs. 4.50) = Rs. 180 (40 × Rs. 6.50) = Rs. 260

Actual number of hours 30 32½ Actual rate per hour Rs. 6 Rs. 8 Supplementary rate per hour (Rs. 6.00 – Rs. 4.50) (Rs. 8.00 – Rs. 6.50)

Illustration In a factory, overheads of a particular department are recovered on the basis of Rs. 5 per machine hour. The total expenses incurred and the actual machine hours for the department for the month of August were Rs. 80,000 and 10,000 hours respectively. Of the amount of Rs. 80,000, Rs. 15,000 became payable due to an award of the Labour Court and Rs. 5,000 was in respect of expenses of the previous year booked in the current month (August). Actual production was 40,000 units, of which 30,000 units were sold. On analysing the reasons, it was found that 60% of the under-absorbed overhead was due to defective planning and the rest was attributed to normal cost increase. How would you treat the under-absorbed overhead in the cost accounts?

4.49

Page 266: 30510870 Cost Accounting and Financial Management

Cost Accounting

Solution Under-absorbed overhead expenses during the month of August Rs. Total expenses incurred in the month of August : 80,000 Less: The amount paid according to labour court award (Assumed to be non-recurring) Rs. 15,000 Expenses of previous year Rs. 5,000 20,000 Net overhead expenses incurred for the month 60,000 Overhead recovered for 10,000 hours @ Rs. 5 per hour 50,000Under-absorbed overheads 10,000

Treatment of under-absorbed overhead in the Cost Accounts It is given in the question that 40,000 units were produced out of which 30,000 units were sold. It is also given that 60% of the under-absorbed overhead was due to defective planning and the rest was attributed to normal cost increase. Rs. 1. 60 percent of under-absorbed overhead is

due to defective planning. This being abnormal, should be debited to Profit and Loss A/c

(60% of Rs. 10,000) 6,000 2. Balance 40 percent of under-absorbed

overhead should be distributed over, Finished Goods and Cost of Sales by supplementary

rate (40% of Rs. 10,000) 4,000 10,000 Rs. 4,000 may be distributed over Finished Goods and Cost of Sales as follows : Finished Goods *Rs. 1,000 Cost of Sales *Rs. 3,000

*Working notes

Under-absorbed overhead : Rs. 4,000 Units produced : 40,000 Rate of under-absorbed overhead recover Re. 0.10 per unit

4.50

Page 267: 30510870 Cost Accounting and Financial Management

Overheads

Amount of under-absorbed overheads charged to finished goods (10,000 × 0.10P) Rs. 1,000 Amount of under-absorbed overheads charged to cost of sales : (30,000 × 0.10P) Rs. 3,000

Illustration In a manufacturing unit, factory overhead was recovered at a pre-determined rate of Rs. 25 per man-day. The total factory overhead expenses incurred and the man-days actually worked were Rs. 41.50 lakhs and 1.5 lakh man-days respectively. Out of the 40,000 units produced during a period, 30,000 were sold. On analysing the reasons, it was found that 60% of the unabsorbed overheads were due to defective planning and the rest were attributable to increase in overhead costs. How would unabsorbed overheads be treated in Cost Accounts ?

Solution Computation of unabsorbed overheads

Man-days worked 1,50,000 Rs. Overhead actually incurred 41,50,000 Less: Overhead absorbed @ Rs. 25 per man-day 37,50,000 (Rs. 25 × 1,50,000) ________ Unabsorbed overheads 4,00,000 Unabsorbed overheads due to defective planning (i.e. 60% of Rs. 4,00,000) 2,40,000Balance of unabsorbed overhead 1,60,000

Treatment of unabsorbed overheads in Cost Accounts (i) The unabsorbed overheads of Rs. 2,40,000 due to defective planning to be treated

as abnormal and therefore be charged to Costing Profit and Loss Account. (ii) The balance unabsorbed overheads of Rs. 1,60,000 be charged to production i.e.,

40,000 units at the supplementary overhead absorption rate i.e., Rs. 4 per unit (Refer to Working Note)

4.51

Page 268: 30510870 Cost Accounting and Financial Management

Cost Accounting

Rs. Charge to Costing Profit and Loss Account as part of the cost of unit sold 1,20,000 (30,000 units @ Rs. 4 p.u.) Add: To closing stock of finished goods 40,000 (10,000 units @ Rs. 4 p.u.) _______ Total 1,60,000

Working note :

Supplementary overhead absorption rate = units 40,0000Rs.1,60,00

= Rs. 4 p.u.

Illustration A factory has three production departments. The policy of the factory is to recover the production overheads of the entire factory by adopting a single blanket rate based on the percentage of total factory overheads to total factory wages. The relevant data for a month are given below :

Department Direct Direct Factory Direct Machine

materials wages overheads labour hours

Rs. Rs. Rs. Hour

Budget

Machining 6,50,000 80,000 3,60,000 20,000 80,000

Assembly 1,70,000 5,50,000 1,40,000 1,00,000 10,000

Packing 1,00,000 70,000 1,25,000 50,000 −

Actuals

Machining 7,80,000 96,000 3,90,000 24,000 96,000

Assembly 1,36,000 2,70,000 84,000 90,000 11,000

Packing 1,20,000 90,000 1,35,000 60,000 −

4.52

Page 269: 30510870 Cost Accounting and Financial Management

Overheads

The details of one of the representative jobs produced during the month are as under: Job No. CW 7083 :

Department Direct Direct Direct Machine materials wages labour hours Rs. Rs. hours Machining 1,200 240 60 180 Assembly 600 360 120 30 Packing 300 60 40 − The factory adds 30% on the factory cost to cover administration and selling overheads and profit. Required : (i) Calculate the overhead absorption rate as per the current policy of the company and

determine the selling price of the Job No. CW 7083. (ii) Suggest any suitable alternative method(s) of absorption of the factory overheads

and calculate the overhead recovery rates based on the method(s) so recommended by you.

(iii) Determine the selling price of Job CW 7083 based on the overhead application rates calculated in (ii) above.

(iv) Calculate the departmentwise and total under or over recovery of overheads based on the company’s current policy and the method(s) recommended by you.

Solution (i) Computation of overhead absorption rate

(as per the current policy of the company) Department Budgeted factory overheads Budgeted direct wages Rs. Rs. Machinery 3,60,000 80,000 Assembly 1,40,000 3,50,000 Packing 1,25,000 70,000Total 6,25,000 5,00,000

Overhead absorption rate = esdirect wag Budgetedoverheadsfactory Budgeted

× 100

4.53

Page 270: 30510870 Cost Accounting and Financial Management

Cost Accounting

= 5,00,000 Rs.6,25,000 Rs.

× 100

= 125% of Direct wages

Selling Price of the Job No. CW-7083

Rs. Direct materials (Rs. 1,200 + Rs. 600 + Rs. 300) 2,100.00 Direct wages (Rs. 240 + Rs. 360 + Rs. 60) 660.00 Overheads (125% × Rs. 660) 825.00Total factory cost 3,585.00 Add: Mark-up (30% × Rs. 3,585) 1,075.50Selling price 4,660.50

(ii) Methods available for absorbing factory overheads and their overhead recovery rates in different departments 1. Machining Department In the machining department, the use of machine time is the predominant factor

of production. Hence machine hour rate should be used to recover overheads in this department. The overhead recovery rate based on machine hours has been calculated as under:—

Machine hour rate = hours machine Budgeted overheadsfactory Budgeted

= hours 80,0003,60,000 Rs.

= Rs. 4.50 per hour

2. Assembly Department In this department direct labour hours is the main factor of production. Hence

direct labour hour rate method should be used to recover overheads in this department. The overheads recovery rate in this case is:

Direct labour hour rate = hours labourdirect Budgeted overheadsfactory Budgeted

4.54

Page 271: 30510870 Cost Accounting and Financial Management

Overheads

= hours 1,00,0001,40,000 Rs.

= Rs. 1.40 per hour

3. Packing Department : Labour is the most important factor of production in this department. Hence

direct labour hour rate method should be used to recover overheads in this department.

The overhead recovery rate in this case comes to: Budgeted factory overhead

Direct labour hour rate = hours labourDirect overheadsfactory Budgeted

= hours 50,0001,25,000 Rs.

= Rs. 2.50 per hour

(iii) Selling Price of Job CW-7083 [based on the overhead application rates calculated in (ii) above]

(Rs.) Direct materials 2,100.00 Direct wages 660.00 Overheads (Refer to Working note) 1,078.00 Factory cost 3,838.00 Add: Mark up (30% of Rs. 3,838) 1,151.40Selling price 4,989.40

Working note :

Overhead Summary Statement Dept. Basis Hours Rate Overheads Rs. Rs. Machining Machine hour 180 4.50 810 Assembly Direct labour hour 120 1.40 168 Packing Direct labour hour 40 2.50 100 Total 1,078

4.55

Page 272: 30510870 Cost Accounting and Financial Management

Cost Accounting

(iv) Departmentwise statement of total under or over recovery of overheads (a) Under current policy

Departments Machining Assembly Packing Total Rs. Rs. Rs. Rs. Direct wages (Actual) 96,000 2,70,000 90,000 Overheads recovered @ 125% of Direct wages: (A) 1,20,000 3,37,500 1,12,500 5,70,000 Actual overheads: (B) 3,90,000 84,000 1,35,000 6,09,000 (Under)/Over recovery of overheads : (A—B) (2,70,000) 2,53,500 (22,500) (39,000)

(b) As per methods suggested Basis of overhead recovery

Machine Direct labour Direct labour Total hours hours hours Rs. Hours worked 96,000 90,000 60,000 Rate/hour (Rs.) 4.50 1.40 2.50 Overhead recovered (Rs.): (A) 4,32,000 1,26,000 1,50,000 7,08,000 Actual overheads (Rs.) : (B) 3,90,000 84,000 1,35,000 6,09,000 (Under)/Over recovery: (A−B) 42,000 42,000 15,000 99,000

Illustration The total overhead expenses of a factory are Rs. 4,46,380. Taking into account the normal working of the factory, overhead was recovered in production at Rs. 1.25 per hour. The actual hours worked were 2,93,104. How would you proceed to close the books of accounts, assuming that besides 7,800 units produced of which 7,000 were sold, there were 200 equivalent units in work-in-progress ? On investigation, it was found that 50% of the unabsorbed overhead was on account of increase in the cost of indirect materials and indirect labour and the remaining 50% was due to factory inefficiency. Also give the profit implication of the method suggested.

4.56

Page 273: 30510870 Cost Accounting and Financial Management

Overheads

Solution Rs. Actual factory overhead expenses incurred 4,46,380 Less : Overheads recovered from production 3,66,380 (2,93,104 hours × Rs. 1.25) Unabsorbed overheads 80,000 Reasons for unabsorbed overheads (i) 50% of the unabsorbed overhead was 40,000 on account of increased in the cost of indirect materials and indirect labour (ii) 50% of the unabsorbed overhead was 40,000 due to factory inefficiency. Treatment of unabsorbed overheads in Cost Accounting 1. Unabsorbed overhead amounting to Rs. 40,000, which were due to increase in the cost

of indirect material and labour should be charged to units produced by using a supplementary rate.

Supplementary rate = units 200) + (7,80040,000 Rs.

= Rs. 5 per unit

The sum of Rs. 40,000 (unabsorbed overhead) should be distributed by using a supplementary rate among cost of sales, finished goods and work-in progress as below :

Rs. Cost of sales 35,000 (7,000 units × Rs. 5) Finished goods 4,000 (800 units × Rs. 5) Work-in progress 1,000 (200 units × Rs. 5) 40,000 The use of cost of sales figure, would reduce the profit for the period by Rs. 35,000 and

will increase the value of stock of finished goods and work-in-progress by Rs. 4,000 and Rs. 1,000 respectively.

4.57

Page 274: 30510870 Cost Accounting and Financial Management

Cost Accounting 2. The balance amount of unabsorbed overheads viz. of Rs. 40,000 due to factory

inefficiency should be charged to Costing Profit & Loss Account, as this is an abnormal loss.

Illustration ABC Ltd. manufactures a single product and absorbs the production overheads at a pre-determined rate of Rs. 10 per machine hour. At the end of financial year 2005-06, it has been found that actual production overheads incurred were Rs. 6,00,000. It included Rs. 45,000 on account of ‘written off’ obsolete stores and Rs. 30,000 being the wages paid for the strike period under an award. The production and sales data for the year 2005-06 is as under :

Production : Finished goods 20,000 units Work-in-progress 8,000 units (50% complete in all respects) Sales : Finished goods 18,000 units The actual machine hours worked during the period were 48,000. It has been found that one-third of the under-absorption of production overheads was due to lack of production planning and the rest was attributable to normal increase in costs. (i) Calculate the amount of under-absorption of production overheads during the year

2005-06; and (ii) Show the accounting treatment of under-absorption of production overheads.

Solution (i) Amount of under-absorption of production overheads during the year 2005-06

Rs. Total production overheads actually incurred 6,00,000 during the year 2005-06 Less : ‘Written off’ obsolete stores Rs. 45,000 Wages paid for strike period Rs. 30,000 75,000 Net production overheads actually incurred : (A) 5,25,000 Production overheads absorbed by 48,000 machine

4.58

Page 275: 30510870 Cost Accounting and Financial Management

Overheads

hours @ Rs. 10 per hour : (B) 4,80,000 Amount of under – absorption of production overheads : [(A) – (B)] 45,000

(ii) Accounting treatment of under absorption of production overheads It is given in the statement of the question that 20,000 units were completely finished

and 8,000 units were 50% complete, one third of the under-absorbed overheads were due to lack of production planning and the rest were attributable to normal increase in costs. Rs. 1. (33 – 1/3% of Rs. 45,000) i.e., Rs. 15,000 of under-absorbed overheads were due to lack of production planning. This being abnormal, should be debited to the Profit and Loss A/c. 15,000 2. Balance (66–2/3% of Rs. 45,000) i.e., Rs. 30,000 of under-absorbed overheads should be distributed over work-in-progress, finished goods and cost of sales by using supplementary rate. 30,000

Total under-absorbed overheads 45,000

Apportionment of unabsorbed overheads of Rs. 30,000 over, work-in progress, finished goods and cost of sales

Equivalent Rs. Completed Units Work-in-Progress 4,000 5,000 (4,000 units × Rs. 1.25) (Refer to working note) Finished goods 2,000 2,500 (2,000 units × Rs. 1.25) Cost of sales 18,000 22,500 (18,000 units × Rs. 1.25) 24,000 30,000 Working Note:

Supplementary rate per unit = 000,24

000,30.Rs = Rs. 1.25

4.7 ACCOUNTING AND CONTROL OF ADMINISTRATIVE OVERHEADS Definition - According to I.C.M.A. Terminology, Administrative overhead is defined as “The sum of those costs of general management and of secretarial accounting and

4.59

Page 276: 30510870 Cost Accounting and Financial Management

Cost Accounting administrative services, which cannot be directly related to the production, marketing, research or development functions of the enterprise.” According to this definition, administrative overhead constitutes the expenses incurred in connection with the formulation of policy directing the organisation and controlling the operations of an undertaking. These overheads are also collected and classified in the same way as the factory overheads. 4.8.1 Accounting of Administrative overheads : There are three distinct methods of accounting of administrative overheads, which are briefly discussed below : (a) Apportioning Administrative Overheads between Production and Sales Departments : According to this method administrative overheads are apportioned over production and sales departments. The reason for the apportionment of overhead expenses over these departments, recognises the fact that administrative overheads are incurred for the benefit of both of these departments. Therefore each department should be charged with the proportionate share of the same. When this method is adopted, administrative overheads lose their identity and get merged with production and selling and distribution overheads. Disadvantages : (1) It is difficult to find suitable bases of administrative overhead apportionment over

production and sales departments. (2) Lot of clerical work is involved in apportioning overheads. (3) It is not justified to apportion total administrative overheads only over production and

sales departments when other equally important department like finance is also there.

(b) Charging to Profit and Loss Account - According to this method administrative overheads are charged to Costing Profit & Loss Account. The reason for charging to Costing Profit & Loss are firstly, the administrative overheads are concerned with the formulation of policies and thus are not directly concerned with either the production or the selling and distribution functions. Secondly, it is difficult to determine a suitable basis for apportioning administrative overheads over production and sales departments. Lastly, these overheads are the fixed costs. In view of these arguments, administrative overheads should be charged to Profit and Loss Account. Disadvantages : (1) Cost of products are understated as administrative overheads are not charged to

costs. (2) The exclusion of administrative overheads from cost of products is against sound

accounting principle.

4.60

Page 277: 30510870 Cost Accounting and Financial Management

Overheads

(c) Treating Administrative Overheads as a separate addition to Cost of Production/Sales : This method considers administration as a separate function like production and sales and, as such costs relating to formulating the policy, directing the organisation and controlling the operations are taken as a separate charge to the cost of the jobs or a product, sold along with the cost of other functions. The basis which are generally used for apportionment are : (i) Works cost (ii) Sales value or quantity (iii) Gross profit on sales (iv) Quantity produced (v) Conversion cost, etc. 4.8.2 Control of Administrative Overheads - Mostly administrative overheads are of fixed nature, and they arise as a result of management policies. These fixed overheads are generally non-controllable. But at the same time these overheads should not be allowed to grow disproportionately. Some degree of control has to be exercised over them. The methods usually adopted for controlling administrative overheads are as follows : (i) Classification and analysis of overheads by administrative departments according to their functions, and a comparison with the accomplished results: According to this method the expenses incurred by each administrative department are collected under standing order numbers for each class of expenditure. These are compared with similar figures of the previous period in relation to accomplishment. Such a comparison will reveal efficiency or inefficiency of the concerned department. However, this method provides only a limited degree of control and comparison does not give useful results if the level of activity is not constant during the periods under comparison. To overcome this difficulty, overhead absorption rates may also be compared from period to period; the extent of over or under absorption will reveal the efficiency or otherwise of the department. It may be possible to compare the cost of a service department with that of similar services obtainable from outside and a decision may be taken whether it is economical to continue the department or entrust the work to outsiders. (ii) Control through Budgets - According to this method, administration budgets (monthly or annually) are prepared for each department. The budgeted figures are compared with actual ones to determine variances. The variances are analysed and responsibility assigned to the concerned department to control these variances. (iii) Control through Standard - Under this method, standards of performance are fixed for each administrative activity, and the actual performance is compared with the standards set. In this way, standards serve not only as yardstick of performance but also facilitate control of costs.

4.61

Page 278: 30510870 Cost Accounting and Financial Management

Cost Accounting Illustration In an engineering company, the factory overheads are recovered on a fixed percentage basis on direct wages and the administrative overheads are absorbed on a fixed percentage basis on factory cost. The company has furnished the following data relating to two jobs undertaken by it in a period:

Job 101 Job 102 Rs. Rs.

Direct materials 54,000 37,500 Direct wages 42,000 30,000 Selling price 1,66,650 1,28,250 Profit percentage on Total Cost 10% 20%

Required : (i) Computation of percentage recovery rates of factory overheads and

administrative overheads. (ii) Calculation of the amount of factory overheads, administrative overheads and

profit for each of the two jobs. (iii) Using the above recovery rates fix the selling price of job 103. The additional

data being:

Direct materials Rs. 24,000 Direct wages Rs. 20,000 Profit percentage on selling price 12-½%

Solution (i) Let factory overhead recovery rate, as percentage of direct wages be F and

administrative overheads recovery rate, as percentage of factory cost be A. Factory Cost of Jobs :

Job 101 = Rs. 96,000 + Rs. 42,000F Job 102 = Rs. 67,500 + Rs. 30,000F

Total Cost of Production of Jobs : Job 101 = (Rs. 96,000 + Rs. 42,000F) + (Rs. 96,000+ Rs. 42,000F)A = Rs. 1,51,500 Job-102 = Rs. 67,500 + Rs. 30,000F) + (Rs. 67,500+ Rs. 30,000F)A = Rs. 1,06,875 (Refer to working note)

4.62

Page 279: 30510870 Cost Accounting and Financial Management

Overheads

On solving above relations: F = 0.60 and A = 0.25 Hence, percentage recovery rates of factory overheads and administrative overheads are 60% and 25% respectively. Working note:

Job 101 Job 102 Total cost of production (Rs.) 1,51,500 1,06,875

profit) of Percentage + (100%price Selling

(Rs. 1,66,650/110%) (Rs. 1,28,250/120%)

(ii) Statement of jobs, showing amount of factory overheads, administrative overheads and profit

Job 101 Job 102 Rs. Rs. Direct materials 54,000 37,500 Direct wages 42,000 30,000Prime cost 96,000 67,500Factory overheads 60% of direct wages 25,200 18,000 Factory cost 1,21,200 85,200Administrative overheads 25% of factory cost 30,300 21,375Total cost 1,51,500 1,06,875 Profit (balancing figure) 15,150 21,375 Selling price 1,66,650 1,28,250

(iii) Selling price of Job 103 Rs. Direct materials 24,000 Direct wages 20,000Prime cost 44,000 Factory overheads (60% of Direct Wages) 12,000

4.63

Page 280: 30510870 Cost Accounting and Financial Management

Cost Accounting

Factory cost 56,000 Administrative overheads 14,000 (25% of factory cost) Total cost 70,000 Profit margin (balancing figure) 10,000

Selling price 87.5%Cost Total

80,000

4.8 ACCOUNTING AND CONTROL OF SELLING AND DISTRIBUTION OVERHEADS Selling cost or overhead expenses are the expenses incurred for the purpose of promoting the marketing and sales of different products. Distribution expenses, on the other hand, are expenses relating to delivery and despatch of goods sold. Examples of selling and distribution expenses have been considered earlier in this booklet. From the definitions it is clear that the two type of expenses represent two distinct type of functions. Some concerns group together these two type of overhead expenses into one composite class, namely, selling and distribution overhead, for the purpose of Cost Accounting. 4.8.1 Accounting of selling and distribution overheads: The collection and accumulation of each expense is made by means of appropriate standing order numbers in the usual way. Where it is decided to apportion a part of the administrative overhead to the selling division the same should also be collected through appropriate standing order numbers. As in the case of administrative overheads, it is not easy to determine an entirely satisfactory basis for computing the overhead rate for absorbing selling overheads. The bases usually adopted are : (a) Sales value of goods; (b) Cost of goods sold; (c) Gross Profit on sales; and (d) Number of orders or units sold. It is considered that the sale value is ordinarily the most logical basis, there being some connection between the amount of sales and the amount of expenses incurred to achieve them. The cost of production, however, is not so satisfactory on basis as it is difficult to conceive of any relationship even remote, between the cost of production of any article and its selling cost. Articles having a high cost of production may require little effort in their sale and vice versa. The basis of gross profit on sales results in a larger share of the selling overhead being applied to goods yielding a large margin of profit and vice versa. The basis therefore follows the principle of ‘ability to pay’, it may not reflect costs or incurred efforts. An estimated amount per unit - The best method for absorbing selling and distributing expenses over various products is to separate fixed expenses from variable expenses. Apportion the fixed expenses according to the benefit derived by each product and thus

4.64

Page 281: 30510870 Cost Accounting and Financial Management

Overheads

ascertaining the fixed expenses per unit. We give below some of the fixed expenses and the basis of apportionment :

Expenses Basis Salaries in the Sales Department and of the sales men.

Estimated time devoted to the sale of various products.

Advertisement Actual amount incurred for each product since these days it is usual to advertise each product separately; common expenses, such as in an exhibition, should be apportioned on the basis of advertisement expenditure on each product.

Show Room expenses Average space occupied by each product. Rent of finished goods godowns and Expenses on own delivery vans

Average quantities delivered during a period.

If a suitable basis for apportioning expenses does not exist it may be apportioned in the proportion of sales of various products. The total of fixed expenses apportioned in this manner, divided by the number of units sold or likely to be sold, will give the fixed expenses per unit. To this should be added the variable expenses which will be different for each product. These expenses are, packaging, freight outwards, insurance in transit, commission payable to salesmen, rebate allowed to customers, etc. All these items will be worked out per unit for each product separately. These items added to fixed expenses per unit will give an estimated amount of the selling and distribution expenses per unit. 4.8.2 Control of Selling & Distribution Overheads - Control of selling and distribution expenses is a difficult task. The reasons for this are as follows : 1. The incidence of selling and distribution overheads depends mainly on external

factors, such as distance of market, extent and nature of competition, terms of sales, etc. which are beyond the control of management.

2. These overheads are dependent upon the customers, behaviour, their liking and disliking, tastes etc. Therefore, as such control over the overheads may result in loss of customers.

3. These expenses being of the nature of policy costs, are not amenable to control. In spite of the above difficulties, the following methods may be used for controlling them. (a) Comparison with past performance - According to this method, selling and

4.65

Page 282: 30510870 Cost Accounting and Financial Management

Cost Accounting distribution overheads are compared with the figures of the previous period. Alternatively, the expenses may be expressed as a percentage of sales, and the percentages may be compared with those of the past period. This method is suitable for small concerns. (b) Budgetary Control - A budget is set up for selling and distribution expenses. The expenses are classified into fixed and variable. If necessary, a flexible budget may be prepared indicating the expenses at different levels of sales. The actual expenses are compared with the budgeted figures and in the case of variances suitable actions are taken. (c) Standard Costing - Under this method standards are set up in relation to the standard sales volume. Standards may be set up for salesmen, territories, products etc. Once the standards are set up, comparison is made between the actuals and standards : variances are enquired into and suitable action taken.

Illustration A company which sells four products, some of them unprofitable, proposes discontinuing the sale of one of them. The following information is available regarding income, costs and activity for the year ended 31st March, 2006.

Products

A B C D Sales (Rs.) 3,00,000 5,00,000 2,50,000 4,50,000 Cost of sales (Rs.) 2,00,000 4,50,000 2,10,000 2,25,000 Area of storage (Sq.ft.) 50,000 40,000 80,000 30,000 Number of parcels sent 1,00,000 1,50,000 75,000 1,75,000 Number of invoices sent 80,000 1,40,000 60,000 1,20,000 Selling and Distribution overheads and the basis of allocation are : Basis of allocation Rs. to products

Fixed Costs Rent & Insurance 30,000 Sq.Ft. Depreciation 10,000 Parcel Salesmen’s salaries & expenses 50,000 Sales Volume Administrative wages and salaries 50,000 No. of invoices Variable Costs : Packing wages & materials 20 paise per parcel

4.66

Page 283: 30510870 Cost Accounting and Financial Management

Overheads

Commission 4% of sales Stationery 10 P. per invoice

You are required to prepare Profit & Loss Statement, showing the percentage of profit or loss to sales for each product.

Solution Statement of Profit or Loss on Various Products during the year ended March 31, 2006. Products Total A B C D Rs. Rs. Rs. Rs. Rs. Sales 15,00,000 3,00,000 5,00,000 2,50,000 4,50,000Variable costs Cost of sales 10,85,000 2,00,000 4,50,000 2,10,000 2,25,000 Commissions 4% of sales 60,000 12,000 20,000 10,000 18,000 Packing wages & materials @ 20 P. per parcel 1,00,000 20,000 30,000 15,000 35,000 Stationery @ 10 P. per invoice 40,000 8,000 14,000 6,000 12,000Total variable costs 12,85,000 2,40,000 5,14,000 2,41,000 2,90,000 Contribution (Sales– variable cost) 2,15,000 60,000 –14,000 9,000 1,60,000 Fixed Costs Rent & Insurance 30,000 7,500 6,000 12,000 4,500 Depreciation 10,000 2,000 3,000 1,500 3,500 Salesmen’s salaries & expenses 60,000 12,000 20,000 10,000 18,000 Administrative wages & salaries 50,000 10,000 17,500 7,500 15,000 Total Fixed costs 1,50,000 31,500 46,500 31,000 41,000 Profit or Loss (Contribution–fixed Costs) 65,000 28,500 –60,500 –22,000 1,19,000 Percentage of profit or Loss on sales 4.3 9.5 –12.1 –8.8 26.4

4.67

Page 284: 30510870 Cost Accounting and Financial Management

Cost Accounting 4.9 CONCEPTS RELATED TO CAPACITY (i) Rated capacity : It refers to the capacity of a machine or a plant as indicated by its

manufacturer. In fact this capacity is the maximum possible productive capacity of a plant. It is also known as installed capacity of a plant. Due to the loss of operating time of a plant it is difficult to achieve this rated capacity. In other words, it is only a theoretical capacity and is therefore, seldom achieved.

(ii) Practical capacity : It is defined as actually utilised capacity of a plant. It is also known as operating capacity. This capacity takes into account loss of time due to repairs, maintenance, minor breakdown, idle time, set up time, normal delays, Sundays and holidays, stock taking etc. Generally, practical capacity is taken between 80 to 90% of the rated capacity. It is also used as a base for determining overhead rates. Practical capacity is also called net capacity or available capacity.

(iii) Normal capacity : It is the capacity of a plant which is expected to be utilised over a long period based on sales expectancy. The determination of this capacity considers the average utilisation of plant capacity during one full business cycle which may extend over 2 to 3 years. It is also known as average capacity and is used to compute overhead recovery rate.

(iv) Capacity based on sales expectancy : It is the capacity of a plant utilised based on sales expectancy.

(v) Actual capacity : It is the capacity actually achieved during a given period. This capacity may lie between practical capacity and capacity based on sales expectancy.

(vi) Idle capacity : It is that part of the capacity of a plant, machine or equipment which cannot be effectively utilised in production. In other words, it is the difference between the practical or normal capacity and capacity utilisation based on expected sales. For example, if the practical capacity of production of a machine is to the tune of 10,000 units in a month, but is used only to produce 8,000 units, because of market demand of the product, then in such a case, 2,000 units will be treated as idle capacity of the machine.

The idle capacity may arise due to lack of product demand, non-availability of raw material, shortage of skilled labour, absenteeism, shortage of power fuel or supplies, seasonal nature of product etc. Idle capacity cost : Costs associated with idle capacity are mostly fixed in nature. These include depreciation, repairs and maintenance charges, insurance premium, rent, rates, management and supervisory costs. These costs remain unabsorbed or unrecovered due

4.68

Page 285: 30510870 Cost Accounting and Financial Management

Overheads

to under-utilisation of plant and service capacity. Idle capacity cost can be calculated as follows :

Idle capacity cost = capacityplant Normalplant to related overhead Aggregate

Treatment of Idle capacity costs : Idle capacity costs can be treated in product costing, in the following ways : (a) If the idle capacity cost is due to unavoidable reasons such as repairs, maintenance,

change over of job etc. a supplementary overhead rate may be used to recover the idle capacity cost. In this case, the costs are charged to the production capacity utilised.

(b) If the idle capacity cost is due to avoidable reasons such as faulty planning, power failure etc.; the cost should be charged to profit and loss account.

(c) If the idle capacity cost is due to seasonal factors, then, the cost should be charged to the cost of production by inflating overhead rates.

(vii) Idle facility : The term ‘facility’ has a wider connotation which may also include prediction capacity. Facilities may be provided by fixed assets such as building space, plants equipment capacity, etc. or by various service functions such as material services, production services, personal services etc. If a firm fails to make full use of the facilities of its disposal, the firm may be said to have idle facilities. Thus idle facilities refer to that part of total facilities which remains unutilised due to any reason such as non-availability of raw material, power, lack of demand etc. In Cost Accounting idle facilities are treated in the same way as those of idle capacity.

4.10 TREATMENT OF CERTAIN ITEMS IN COSTING 4.10.1 Treatment of interest and financial charges : There is controversy whether financial charges, specially interest, should be included in the costs or not. The following arguments are generally advanced in favour of interest to be included in overhead expenses. (1) Computation of total cost is impossible unless interest is taken into account. Interest

is an element of cost and therefore, should be included in cost. This is specially true in business where raw materials in different stages can be used. Thus a timber merchant, if he buys standing trees and seasons the timber himself, would incur a large amount of costs as interest. Another merchant who buys his timber already seasoned would automatically have to pay a higher price; obviously, this price includes interest.

4.69

Page 286: 30510870 Cost Accounting and Financial Management

Cost Accounting (2) Interest is the cost to be paid for the use of capital; capital is also a factor of

production just as labour. Thus, if wages are included in cost of production, why not interest ?

(3) If interest is not included in cost calculation, a number of managerial decisions may be taken wrongly. Thus, where a decision involves replacement of labour with expensive machinery, the question of interest assumes importance, since, if interest is not included, the cost accountant may conclude that machinery is cheaper.

(4) Inclusion of interest also allows comparison of profit on different jobs. Thus, if a job takes 3 months and another takes 6 months the cost of the jobs must include a charge by way of interest before profit can be compared.

(5) In inventory control, interest is an important item to be considered. Where large stocks are kept, the advantage of one time purchase is offset by increase in interest charges.

(6) While submitting tenders for cost plus contracts, etc., interest must be taken into account.

However, many cost accountants argue that interest should not be included in cost accounts since it is not an item of cost and would vary with different methods of financing. Some of the arguments are listed below : (1) Payment of interest depends entirely on the financing policies and financing pattern.

A firm working with proprietor’s capital only will have no interest to pay whereas a firm working with borrowed capital will have to pay a large amount of interest. In reality, whether a firm raises a certain sum of money from the proprietor or borrows from the outside does not make any difference as far as production efficiencies are concerned. If we compare the two firms and include interest as an item of cost, the firm, which works on the proprietor’s capital will show very favourable results. Actually, this is a wrong conclusion.

However, this argument can be met by including a notional amount of interest, irrespective of the fact whether the funds belong to the owners or to the outsiders. Thus, an amount of notional interest may be charged on the total capital whether it is borrowed or not.

(2) Another practical difficulty arises in the calculation of the amount of capital on which interest should be worked out. While the fixed capital is readily ascertainable, working capital keeps on changing. Again, the difficulty becomes pronounced since the working capital would be used by different departments, and allocation of the total interest charges will have to be made over various departments at different points of time.

4.70

Page 287: 30510870 Cost Accounting and Financial Management

Overheads

If notional interest is to be charged, the problem of determining a proper rate of interest also arises. In the money and capital markets, there is a large number of rates depending upon different factors like risk, period of maturity, bank rate, etc.

(3) By including interest on the proprietor’s capital and by taking that figure in the cost of production, we would obviously be including profit since the closing stock will be valued at a higher figure.

It appears that there are practical difficulties in including interest as part of the normal cost. However, excluding it altogether may lead to wrong managerial decisions which is not desirable. It is therefore, suggested that while interest may be excluded from the regular cost sheet, cost calculations for other purposes for decision making should include a proper amount of notional interest where the interest will be material.

4.10.2 Depreciation : Depreciation “is the diminution in the intrinsic value of an asset due to use and/or the lapse of time.” Depreciation is thus the result of two factors viz., the use, and the lapse of time. We know that each fixed asset loses its intrinsic value due to their continuous use and as such the greater the use the higher is the amount of depreciation. The loss in the intrinsic value may also arise even if the asset in question is not in service. In Cost Accounting depreciation is charged to the cost of production. The various reasons for including the depreciation charge in Cost Accounting are as follows : (a) To show a true and fair picture of Balance Sheet. (b) To ascertain the true cost of production. (c) To keep the asset intact by distributing losses in its value over a number of years. (d) To keep the capital intact and to make a provision of the resources for the

replacement of asset in future. (e) To provide for depreciation before distribution of profit as required under the

Companies Act. 4.10.3 Packing expenses : Cost of primary packing necessary for protecting the product or for convenient handling, should become a part of the prime cost. The cost of packing to facilitate the transportation of the product from the factory to the customer should become a part of the distribution cost. If the cost of special packing is at the request of the customer, the same should be charged to the specific work order or the job. The cost of fancy packing necessary to attract customers is an advertising expenditure. Hence, it is to be treated as a selling overhead.

4.71

Page 288: 30510870 Cost Accounting and Financial Management

Cost Accounting 4.10.4 Fringe benefits : These are the additional payments or facilities provided to the workers apart from their salary and direct cost-allowances like house rent, dearness and city compensatory allowances. These benefits are given in the form of overtime, extra shift duty allowance, holiday pay, pension facilities etc. These indirect benefits stand to improve the morale, loyalty and stability of employees towards the organisation. If the amount of fringe benefit is considerably large, it may be recovered as direct charge by means of a supplementary wage or labour rate; otherwise these may be collected as part of production overheads. 4.11.5 Expenses on removal and re-erection of machines : Expenses are sometime incurred on removal and re-erection of machinery in factories. Such expenses may be incurred due to factors like change in the method of production; an addition or alteration in the factory building, change in the flow of production, etc. All such expenses are treated as production overheads. When amount of such expenses is large, it may be spread over a period of time. If such expenses are incurred due to faulty planning or some other abnormal factor, then they may be charged to costing Profit and Loss Account. 4.10.6 Bad debts : There is no unanimity among different authors of Cost Accounting about the treatment of bad debts. One view is that ‘bad debts’ should be excluded from cost. According to this view bad debts are financial losses and therefore, they should not be included in the cost of a particular job or product. According to another view it should form part of selling and distribution overheads, especially when they arise in the normal course of trading. Therefore bad debts should be treated in cost accounting in the same way as any other selling and distribution cost. However extra ordinarily large bad debts should not be included in cost accounts. 4.10.7 Training expenses : Training is an essential input for industrial workers. Training expenses in fact includes wages of workers, costs incurred in running training department, loss arising from the initial lower production, extra spoilage etc. Training expenses of factory workers are treated as part of the cost of production. The training expenses of office; sales or distribution workers should be treated as office; sales or distribution overhead as the case may be. These expenses can be spread over various departments of the concern on the basis of the number of workers on roll. Training expenses would be abnormally high in the case of high labour turnover such expenses should be excluded from costs and charged to the costing profit and loss account. 4.10.8 Canteen expenses : The loss incurred by the firm in running the canteen should be regarded as a production overhead. If the canteen is meant only for factory workers

4.72

Page 289: 30510870 Cost Accounting and Financial Management

Overheads

therefore this loss should be apportioned on the basis of the number of workers employed in each department. If office workers also take advantage of the canteen facility, a suitable share of the loss should be treated as office overhead. 4.10.9 Carriage and cartage expenses : It includes the expenses incurred on the movement (inward and outwards) and transportation of materials and goods. Transportation expenses related to direct material may be included in the cost of direct material and those relating to indirect material (stores) may be treated as factory overheads. Expenses related to the transportation of finished goods may be treated as distribution overhead. 4.10.10 Expenses for welfare activities : All expenses incurred on the welfare activities of employees in a company are part of general overheads. Such expenses should be apportioned between factory, office, selling and distribution overheads on the basis of number of persons involved. 4.10.11 Night shift allowance : Workers in the factories, which operate during night time are paid some extra amount known as ‘night shift allowance’. This extra amount is generally incurred due to the general pressure of work beyond normal capacity level and is treated as production overhead and recovered as such. If this allowance is treated as part of direct wages, the jobs/production carried at night will be costlier than jobs/production performed during the day. However, if additional expenditure on night shift is incurred to meet some specific customer order, such expenditure may be charged directly to the order concerned. If night shifts are run due to abnormal circumstances, the additional expenditure should be charged to the costing profit and loss account.

4.11 Self-Examination Questions

Multiple Choice Questions 1. Director’s remuneration and expenses form a part of : (a) Production overhead (b) Administration overhead (c) Selling overhead (d) Distribution overhead. 2. Salary of a foreman should be classified as a:

(a) fixed overhead (b) ‘variable overhead’ (c) semi-fixed or semi-variable overhead

4.73

Page 290: 30510870 Cost Accounting and Financial Management

Cost Accounting

(d) None of the above. 3. Absorption means :

(a) charging of overheads to cost centres (b) charging of overheads to cost unit (c) charging of overheads to cost centres or cost units (d) none of the above.

4. Which of the following is a service department? (a) Refining department (b) Machining department (c) Receiving department (d) Finishing department.

5. Which method of absorption of factory overheads do you suggest in a concern which produces only one uniform item of product? (a) Percentage of direct wage basis (b) Direct labour hour rate (c) Machine hour rate (d) A rate per unit of output (e) Any of the above.

6. When the amount of under-or-over-absorption is significant, it should be disposed off by (a) Transferring to Costing Profit and Loss A/c (b) The use of supplementary rates (c) Carrying over as a deferred charge to the next accounting year (d) Either of the three.

7. Maximum possible productive capacity of a plant when no operating time is lost is its : (a) Practical capacity (b) Normal capacity (c) Theoretical capacity (d) Capacity based on sales expectancy.

8. When the amount of overhead absorbed is less than the amount of overhead incurred, its is called :

(a) Under-absorption of overhead

4.74

Page 291: 30510870 Cost Accounting and Financial Management

Overheads

(b) Over-absorption of overhead (c) Proper absorption of overhead (d) None of the above.

9. Factory overhead should be absorbed on the basis of : (a) Relationship to cost incurred (b) Direct labour hours (c) Direct labour cost (d) Machine hours.

10. What is the basis for distribution of indirect material cost to various departments? (a) Direct allocation (b) Cost of direct materials consumed (c) Machine hours worked (d) Either of the three.

Answers to Multiple Choice Questions 1.(b); 2.(c); 3.(b); 4.(c); 5.(d); 6.(b); 7.(c); 8.(a); 9.(a); 10(c)

Short Answer Type Questions 1. What do you understand by classification, allocation and apportionment in relation to

overhead expenses. 2. Define Overhead. Explain various classifications of overheads according to functions. 3. How you would treat the following items in cost accounts: (i) Idle capacity costs. (ii) Research and development costs. (iii) Bad debts. (iv) Fringe benefits.

(v) Training expenses. 4. A factory produces three products A, B and C. Which of the following costs are prime

costs and which are overheads? In case of overheads classify them by function; (i) Salary of the accountant. (ii) Temporary labour employed to increase production in order to meet unusual

demand of product C.

4.75

Page 292: 30510870 Cost Accounting and Financial Management

Cost Accounting (iii) Uniforms of sanitary workers. (iv) Raw Material, Godown, Chowkidar salary. (v) Dividends received on investments. (vi) Freight on purchase of raw material. (vii) Remuneration for legal advice. (viii) Consultation fee of advertisement designer. (ix) Rent of godown for storing finished goods. (x) Rent of godown for storing raw materials. (xi) Salary paid to the wife of the managing director. She is designated as whole time

director but does not come to the factory at all. (xii) Loss due to accidental falling of the roof of a section of the factory. (xiii) Primary packing to keep the product crisp. (xiv) Secondary packing with the name of the company, etc. (xv) Packing of boxes of finished product in wooden crates for transportation. (xvi) Income from sale of bags in which raw materials were procured. (xvii) Depreciation of patterns and dies. (xviii) Bad debts. (xix) Cost of stolen materials. (xx) Commission paid to salesman as a percentage of sale price. (xxi) Expenses incurred for repairs and maintenance of water supply lines, sewage pipes

etc. (xxii) Royalties paid on the basis of sales. (xxiii) Amount paid to lawyer for appearing before a Labour Tribunal. (xxiv) Allowance made to customers for late deliveries of goods. (xxv) Carriage Inward. 5. Classify the following costs into fixed, semi-variable and variable costs : (a) Salary of the managing director (b) Salary of foreman (c) Raw material cost (d) Wages of permanent workers (e) Commission to salesman as a percentage of sales

4.76

Page 293: 30510870 Cost Accounting and Financial Management

Overheads

(f) Depreciation of plant (g) Rent of factory (h) Power consumption in the production process (i) Canteen expenses (j) Salary of the store keeper.

Answers 4. Prime cost: ii, vi, xvi, xxv. Factory overheads : - iii, iv, x, xvii, xxi. Administration overheads :- i, vii, xxiii. Selling overheads : - viii, xiv, xviii, xx, xxii. Distribution overheads : -ix, xv. Costing Profit & Loss items : - v, xi, xii, xix, xxiv. 5. Fixed - (a), (b), (g), (i) and (j)

Semi Fixed :- Partly (d), (f). Variable :- (c), partly (d), (e), (h).

Long Answer Type Questions 1. What do you understand by absorption of overheads? Describe the various methods

of absorption of factory overheads. Which of these methods do you consider most scientific and why?

2. Explain the various methods of distribution of Semi-variable expenses into fixed and variable expenses.

3. (a) Explain the different methods of apportionment of service department costs over production departments ?

(b) What are the methods of secondary distribution of overheads? Explain in detail three methods available for dealing with reciprocal services, giving examples for each.

4. What do you mean by under/over absorption of overheads? How does it arise? How it is treated in cost accounts.

Numerical Questions 1. Two service departments, A and B, show expenses of Rs. 5,000 and Rs. 8,000

respectively. 1/10 expenses of department A are chargeable to department B,

4.77

Page 294: 30510870 Cost Accounting and Financial Management

Cost Accounting

whereas 1/4 of the expenses of the latter are chargeable to department A. Ascertain the overheads of the departments to be apportioned to production departments of the two departments.

2. Three machines A, B and C are in use, involving the undermentioned expenditure for a period:

A Rs. 639; B Rs. 697; C Rs. 951. The machines sometimes require the use of a crane also for which Rs. 570 has to be spent. The number of hours for the machines are:

A B C With the use of crane 160 130 480 Without the use of crane 428 577

Calculate the rates for recovery of overheads. 3. The actual figures relating to production for a period in a factory were as follows:

Material used Rs. 5,00,000 Direct labour (Total 1,20,000) Rs. 4,00,000 Factory expenses Rs. 3,00,000 Machine hours totalled 1,00,000

A job requires Rs. 20,000 in material, and 4,000 hours of labour @ Rs. 3 per hour (on the average) of which 2,800 were machine hours. Ascertain the cost of the job using different methods of absorbing overheads.

4. Suppose in the factory mentioned in Q-17, administrative expenses total Rs. 2,50,000 and assume 1/5 of goods produced remained unsold. What is the value that should be put on inventory with alternative treatments of administrative expenses?

5. The products of a factory pass through two departments, though the output emerging from the first department is also saleable. The direct labour in the two processes per period is Rs. 60,000 and Rs. 40,000 and the indirect expenses are Rs. 45,000 and Rs. 40,000. The rate for recovery of the overheads is 85%. Do you think the method followed is proper?

4.78

Page 295: 30510870 Cost Accounting and Financial Management

CHAPTER 5

NON-INTEGRATED ACCOUNTS

Learning objectives When you have finished studying this chapter, you should be able to ♦ Differentiate between integrated and non-integrated systems of accounting. ♦ Write the various journal entries for both integrated and non-integrated systems of

accounting. ♦ Understand the reasons for differences between financial and cost accounts and prepare

a reconciliation statement accordingly.

5.1 INTRODUCTION To operate business operations efficiently and successfully, it is necessary to make use of an appropriate accounting system. Such a system should state in clear terms whether cost and financial transactions should be integrated or kept separately. Where cost and financial accounting records are integrated, the system so evolved is known as integrated or integral accounting. In case cost and financial transactions are kept separately, the system is called Non-Integrated Accounting System.

5.2 NON-INTEGRATED ACCOUNTING SYSTEM It is a system of accounting under which separate ledgers are maintained for cost and financial accounts by Accountants. Under such a system the cost accounts restricts itself to recording only those transactions which relate to the product or service being provided. Hence items of expenses which have a bearing with sales or, production or for that matter any other items which are under the factory management are the ones dealt with in such accounts. This leads to the exclusion of certain expenses like interest and, bad debts and revenue/income from ‘other than the sale of product or service’. A special feature of the non-integrated system of accounts is its ability to deal with notional expenses like rent or interest on capital tied up in the stock. The accounting of notional rent facilitates comparisons amongst factories (some owned and some rented). Similarly, recognition of interest on capital tied up in stock could help make the stores and works managers aware of the money being blocked because of holding stock.

Page 296: 30510870 Cost Accounting and Financial Management

Cost Accounting Non Integrated Accounting Systems contain fewer accounts when compared with financial accounting because of the exclusion of purchases, expenses and also Balance Sheet items like fixed assets, debtors and creditors. Items of accounts which are excluded are represented by an account known as cost ledger control account. The important ledgers to be maintained under non-integrated accounting system in the Cost Accounting department are the following: (a) Cost Ledger - This is the principle ledger of the cost department in which impersonal accounts are recorded. This ledger also contains a Control Account for each subsidiary ledger.

(b) Stores Ledger - It contains an account for each item of stores. The entries in each account maintained in this ledger are made from the invoice, goods received note, material requisitions, material received note etc. Accounts in respect of each item of stores show receipt, issue and balance in physical as well as monetary terms.

(c) Work-in-Progress Ledger - This ledger is also known as job ledger, it contains accounts of unfinished jobs and processes. Each job account is debited with direct and indirect costs related with the job and credited with the amount of finished goods completed and transferred. The balance in a job account represents total balance of job/work-in-progress, as shown by the job account.

(d) Finished Goods Ledger - It contains an account for each item of finished product manufactured or the completed job. If the finished product is transferred to stores, a credit entry is made in the work-in-progress ledger and a corresponding debit entry is made in this ledger.

5.2.1 Principle Accounts : The main accounts which are usually prepared when a separate Cost Ledger is maintained are as follows :

(1) Cost Ledger Control Account - This account is also known as General Ledger Adjustment Account. This account is made to complete double entry. All items of expenditure are credited to this account. Sales are debited to this account and net profit/loss is transferred to this account. The balance in this account represents the net total of all the balances of the impersonal accounts.

(2) Stores Ledger Control Account - Total of material purchases are debited to this account. Whereas issues of material are credited. The balance in this account indicates the total balance of all the individual stores accounts. Abnormal losses or gains if any in this account, are transferred to Costing Profit & Loss Account.

5.2

Page 297: 30510870 Cost Accounting and Financial Management

Non-Integrated Accounts

(3) Work-in-Progress Control Account - This account is debited with the total cost of production, which includes—direct materials, direct labour, direct expenses, production overhead recovered, and is credited with the amount of finished goods completed and transferred. The balance in this account represents total balances of jobs/works-in-progress, as shown by several job accounts.

(4) Finished Goods Control Accounts - This account is debited to the tune of “value of goods” transferred from work-in-progress account, administration costs recovered. This account is credited with the cost of goods sold and Cost of Sales Account is debited. The balance of this account represents the value of goods lying at hand.

(5) Wage Control Account - This account is debited with total wages paid (direct and indirect). Direct wages are further transferred to Work-in-Progress Account and indirect wages to Production Overhead; Administration Overhead or Selling & Distribution Overhead Account, as the case may be. Wages paid for abnormal idle time are transferred to Costing Profit & Loss Account either directly or through Abnormal Loss Account.

(6) Manufacturing/Production/Works Overhead Account - This account is debited with Indirect costs of production such as indirect material, indirect labour, indirect expenses (carriage inward etc.). Overhead recovered is credited to this Account. The difference between overhead incurred and overhead recovered is transferred to Overhead Adjustment Account.

(7) Administrative Overhead Account - This account is debited with overhead incurred and credited with Overhead recovered. The Overhead recovered are debited to Finished Goods Account. The difference between Administrative Overhead incurred and recovered are transferred to Overhead Adjustment Account.

(8) Selling and Distribution Overhead Account - This account is debited with Selling and Distribution Overhead incurred and credited with the recovered Overhead. The difference between incurred and recovered overhead is transferred usually to Overhead Adjustment Account.

(9) Cost of Sales Account - This account is debited with the cost of finished goods transferred from Finished Goods Account for sale as well as with the amount of selling and distribution overhead costs recovered. The balance of this account is ultimately transferred to Sales Account or Costing Profit & Loss Account.

(10) Costing Profit & Loss Account - The net profit or loss in this account is transferred to Cost Ledger Control Account.

5.3

Page 298: 30510870 Cost Accounting and Financial Management

Cost Accounting (11) Overhead Adjustment Account - This account is to be debited for under-recovery of overhead and credited with over-recovery of overhead amount. The net balance in this account is transferred to Costing Profit & Loss Account.

Sometimes, Overhead Adjustment Account is dispensed with and under/over absorbed overheads is directly transferred to Costing Profit & Loss Account from the respective overhead accounts.

5.2.2 Scheme of Entries: The manner in which the Cost Ledger, when maintained on a double entry basis, would operate is illustrated by the following statements of various journal entries as would appear in the cost books. Material: (a) Purchase—Rs. 5,000 (credit or cash) Rs. (i) Dr. Material Control A/c 5,000 Cr. Cost Ledger Control A/c 5,000 (ii) Dr. Stores Ledger Control A/c 5,000 Cr. Material Control A/c 5,000

Note: Sometimes Material Control Account is dispensed with and entries are directly made into Stores Ledger Control A/c, giving a credit to Cost Ledger Control A/c.

(b) Purchases worth Rs. 500 for special job Rs. Dr. Work-in-Progress Ledger Control A/c 500 Cr. Cost Ledger Control A/c 500 (c) Material returned to vendor—Rs. 500 Dr. Cost Ledger Control A/c 500 Cr. Stores Ledger Control A/c 500 (d) (i) Material (Direct) issued to production—Rs. 1,000 Dr. Work-in-Progress Control A/c 1,000 Cr. Stores Ledger Control A/c 1,000 (ii) Material ‘Indirect’ issued to production—Rs. 200 Dr. Manufacturing Overhead A/c 200 Cr. Stores Ledger Control A/c 200

5.4

Page 299: 30510870 Cost Accounting and Financial Management

Non-Integrated Accounts

(e) (i) Material worth Rs. 200 returned from shop to stores: Dr. Stores Ledger Control A/c 200 Cr. Work-in-Progress Control A/c 200 (ii) Material worth Rs. 100 is transferred from Job 1 to Job 2 Dr. Job 2 A/c 100 Cr. Job 1 A/c 100 (f) Material worth Rs. 100 is issued from stores for repairs Dr. Manufacturing Overhead A/c 100 Cr. Stores Ledger Control A/c 100 Labour: (i) Direct wages paid to workers Rs. 1,000 Rs.

(a) Dr. Wage Control A/c 1,000 Cr. Cost Ledger Control A/c 1,000 (b) Dr. WIP A/c 1,000 Cr. Wage Control A/c 1,000 (ii) Indirect wages paid to workers in the production (Rs. 700), administration (Rs. 500),

selling and distribution departments (Rs. 300). (a) Dr. Wage Control A/c 1,500 Cr. Cost Ledger Control A/c 1,500 (b) Dr. Production Overhead A/c 700 Dr. Administrative Overhead A/c 500 Dr. Selling & Dist. Overhead A/c 300 Cr. Wage Control A/c 1,500 Direct Expenses: Rs. 500 for Job No. 12 Rs. Dr. Job No. 12 A/c (WIP Control A/c) 500 Cr. Cost Ledger Control A/c 500

5.5

Page 300: 30510870 Cost Accounting and Financial Management

Cost Accounting Overhead: (i) Overhead expenses incurred Rs. 500

Rs. Rs. Production 150 Administrative 150 Selling and Distribution 200 500 Dr. Production Overhead A/c 150 Dr. Administrative Overhead A/c 150 Dr. Selling & Distribution Overhead A/c 200 Cr. Cost Ledger Control A/c 500 (ii) Carriage Inward—Rs. 100 Dr. Manufacturing Overhead A/c 100 Cr. Cost Ledger Control A/c 100 (iii) Production overhead recovered—Rs. 1,000 Dr. Work-in-Progress Control A/c 1,000 Cr. Production Overhead A/c 1,000 (iv) Administrative Overhead recovered Rs. 500 from finished goods Dr. Finished Goods Ledger Control A/c 500 Cr. Administrative Overhead A/c 500 (v) Selling and Distribution Overhead Rs. 100 recovered from sales Dr. Cost of Sales A/c 100 Cr. Selling & Distribution Overhead A/c 100 The under/over absorbed overheads are transferred to Costing Profit & Loss Account (i) Dr. Production Overhead A/c Cr. Costing Profit & Loss A/c (For over recovery)

5.6

Page 301: 30510870 Cost Accounting and Financial Management

Non-Integrated Accounts

(ii) Dr. Costing Profit & Loss Account Cr. Administrative Overhead A/c (For under recovery) Sales : The entry when sales are effected is as follows: Dr. Cost Ledger Control A/c Cr. Costing Profit & Loss Account Profit/Loss : The entry made in this case (after adjusting over/under absorbed overhead and other items of profit or loss) is as follows: Dr. Costing Profit & Loss A/c Cr. Cost Ledger Control A/c Note : Reverse the above entry in the case of loss.

Illustration : As on 31st March, 2006, the following balances existed in a firm’s Cost Ledger : Dr. Cr. Rs. Rs. Stores Ledger Control A/c 3,01,435 Work-in-Progress Control A/c 1,22,365 Finished Stock Ledger Control A/c 2,51,945 Manufacturing Overhead Control A/c 10,525 Cost Ledger Control A/c _______ 6,65,220 6,75,745 6,75,745During the next three months the following items arose: Rs. Finished product (at cost) 2,10,835 Manufacturing overhead incurred 91,510 Raw materials purchased 1,23,000 Rs. Factory Wages 50,530 Indirect Labour 21,665 Cost of Sales 1,85,890 Material issued to production 1,27,315 Sales returned at Cost 5,380

5.7

Page 302: 30510870 Cost Accounting and Financial Management

Cost Accounting Material returned to suppliers 2,900 Manufacturing overhead charged to production 77,200 You are required to pass the Journal Entries; write up the accounts and schedule the balances, stating what each balance represents. Solution: Journal entries are as follows: Dr. Cr. Rs. Rs. 1.Finished stock ledger Control A/c Dr. 2,10,835

To Work-in-Progress Control A/c 2,10,835 2.Manufacturing Overhead Control A/c Dr. 91,510 To Cost Ledger Control A/c 91,510 3.Stores Ledger Control A/c Dr. 1,23,000 To Cost Ledger Control A/c 1,23,000 4.(i) Wage Control A/c Dr. 72,195 To Cost Ledger Control A/c 72,195 (ii) Work-in-progress Control A/c Dr. 50,530 To Wage Control A/c 50,530 (iii) Manufacturing Overhead Control A/c Dr. 21,665 To Wage Control A/c 21,665 5.Cost of Sales A/c Dr. 1,85,890 To Finished Stock Ledger A/c 1,85,890 6.Work-in-Progress Control A/c Dr. 1,27,315 To Stores Ledger Control A/c 1,27,315 7.Finished Stock Ledger Control A/c Dr. 5,380 To Cost of Sales A/c 5,380 8.Cost Ledger Control A/c Dr. 2,900 To Stores Ledger Control A/c 2,900

5.8

Page 303: 30510870 Cost Accounting and Financial Management

Non-Integrated Accounts

9.Work-in-Progress Control A/c Dr. 77,200 To Manufacturing Overhead Control A/c 77,200

COST LEDGER Cost Ledger Control Account

Rs. Rs. To Stores Ledger Control A/c (return) 2,900 By Balance b/d 6,65,220 ” Balance c/d 9,49,025 ” Manufacturing Overhead Control A/c 91,510 ” Stores Ledger Control A/c 1,23,000 _______ ” Wage Control A/c 72,195 9,51,925 9,51,925

Stores Ledger Control Account Rs. Rs. To Balance b/d 3,01,435 By Work-in-Progress ” Cost Ledger Control A/c 1,23,000 ” Control A/c 1,27,315 ” Cost Ledger Control A/c 2,900 _______ ” Balance c/d 2,94,220 4,24,435 4,24,435

Work-in-Progress Control Account Rs. Rs. To Balance b/d 1,22,365 By Finished Stock ” Wage Control A/c 50,530 Ledger Control A/c 2,10,835 ” Stores Ledger Control A/c 1,27,315 ” Balance c/d 1,66,575 Manufacturing Overhead Control A/c 77,200 _______ 3,77,410 3,77,410

5.9

Page 304: 30510870 Cost Accounting and Financial Management

Cost Accounting

Finished Stock Ledger Control Account Rs. Rs. To Balance b/d 2,51,945 By Cost of Sales A/c 1,85,890 Work-in-Progress ” Balance c/d 2,82,270 Control A/c 2,10,835 ” Cost of Sales A/c (return at cost) 5,380 _______ 4,68,160 4,68,160

Manufacturing Overhead Control Account Rs. Rs. To Cost Ledger Control A/c 91,510 By Balance b/d 10,525 ” Wage Control A/c 21,665 ” Work-in-ProgressControl A/c 77,200 ” Balance c/d (under recovered) 25,450 1,13,175 1,13,175

Wage Control Account Rs. Rs. To Cost Ledger Control A/c 72,195 By Work-in-Progress Control A/c 50,530 ” Manufacturing Overhead ______ Control A/c 21,665 72,195 72,195

Cost of Sales Account Rs. Rs. To Finished Stock Ledger Control A/c 1,85,890 By Finished Stock Ledger Control A/c (Return) 5,380 _______ ” Balance c/d 1,80,510 1,85,890 1,85,890

Trial Balance

Dr. Cr. Rs. Rs. Stores Ledger Control A/c 2,94,220 Work-in-Progress Control A/c 1,66,575 Finished Stock Ledger Control A/c 2,82,270

5.10

Page 305: 30510870 Cost Accounting and Financial Management

Non-Integrated Accounts

Manufacturing Overhead Control A/c 25,450 Cost of Sales A/c 1,80,510 Cost Ledger Control A/c _______ 9,49,025 9,49,025 9,49,025

Illustration : The following figures are extracted from the Trial Balance of Gogetter Co. on 30th September, 2005: Rs. Rs. Inventories: Finished Stock 80,000 Raw Materials 1,40,000 Work-in-Progress 2,00,000 Office Appliances 17,400 Plant & Machinery 4,60,500 Building 2,00,000 Sales 7,68,000 Sales Return and Rebates 14,000 Materials Purchased 3,20,000 Freight incurred on Materials 16,000 Purchase Returns 4,800 Direct Labour 1,60,000 Indirect Labour 18,000 Factory Supervision 10,000 Repairs and Upkeep Factory 14,000 Heat, Light and Power 65,000 Rates and Taxes 6,300 Miscellaneous Factory Expenses 18,700 Sales Commission 33,600 Sales Travelling 11,000 Sales Promotion 22,500 Distribution Deptt.—Salaries and Expenses 18,000 Office Salaries and Expenses 8,600 Interest on Borrowed Funds 2,000 Further details are available as follows:

5.11

Page 306: 30510870 Cost Accounting and Financial Management

Cost Accounting (i) Closing Inventories: Finished Goods 1,15,000 Raw Materials 1,80,000 Work-in-Progress 1,92,000 (ii) Accrued expenses on: Direct Labour 8,000 Indirect Labour 1,200 Interest on Borrwed Funds 2,000 (iii) Depreciation to be provided on: Office Appliances 5% Plant and Machinery 10% Buildings 4% (iv) Distribution of the following costs: Heat, Light and Power to Factory, Office and Distribution in the ratio 8 : 1 : 1. Rates and Taxes two-thirds to Factory and one-third to Office. Depreciation on Buildings to Factory, Office and Selling in the ratio 8 : 1 : 1. With the help of the above information, you are required to prepare a condensed Profit and Loss Statement of Gogetter Co. for the year ended 30th September, 1998 along with supporting schedules of: (i) Cost of Sales. (ii) Selling and Distribution Expenses. (iii) Administration Expenses.

Solution: Profit and Loss Statement of Gogetter Company

for the year ended 30th September, 2005 Rs. Rs. Gross Sales 7,68,000 Less: Returns 14,000 7,54,000 Less: Cost of Sales Refer to Schedule (i) 7,14,020 Net Operating Profit: 39,980 Less: Interest on Borrowed funds 4,000 Net Profit 35,980

5.12

Page 307: 30510870 Cost Accounting and Financial Management

Non-Integrated Accounts

(i) Schedule of Cost of Sales Rs. Rs. Raw Material 1,40,000 (Inventory op. balance) Add: Material Purchased 3,20,000 Add: Freight on Material 16,000 Less: Purchase Returns 4,800 3,31,200 4,71,200 Less: Closing Raw Material Inventories 1,80,000 Material used in production 2,91,200 Direct Labour 1,68,000 Factory Overheads Indirect Labour 19,200 Factory Supervision 10,000 Repairs and Factory Upkeep 14,000 Heat, Light and Power 52,000 Rates and Taxes 4,200 Miscellaneous Factory Expenses 18,700 Depreciation of Plant 46,050 Depreciation of Buildings 6,400 1,70,550 Gross Works Cost 6,29,750 Add: Opening Work-in-Process inventory 2,00,000 8,29,750 Less: Closing Work-in-Process inventory 1,92,000 Works Cost 6,37,750 Add: Administration Expenses [See Schedule (iii)] 18,870 Total Cost of output 6,56,620 Add: Opening Finished Goods inventory 80,000 7,36,620 Less: Closing Finished Goods inventory 1,15,000 Cost of Production of goods sold 6,21,620 Add: Selling and Distribution Expenses [See Schedule (ii)] 92,400 Cost of Sales 7,14,020

5.13

Page 308: 30510870 Cost Accounting and Financial Management

Cost Accounting (ii) Schedule of Selling and Distribution Expenses Rs. Sales Commission 33,600 Sales Travelling 11,000 Sales Promotion 22,500 Distribution Deptt.—Salaries and Expenses 18,000 Heat, Light and Power 6,500 Depreciation of Buildings 800 92,400 (iii) Schedule of Administration Expenses Rs. Office Salaries and Expenses 8,600 Depreciation of Office Appliances 870 Depreciation of Buildings 800 Heat, Light and Power 6,500 Rates and Taxes 2,100 18,870

Illustration : In the absence of the Chief Accountant, you have been asked to prepare a month’s cost accounts for a company which operates a batch costing system fully integrated with the financial accounts. The following relevant information is provided to you: Rs. Balances at the beginning of the month: Stores Ledger Control Account 25,000 Work-in-Progress Control Account 20,000 Finished Goods Control Account 35,000 Prepaid Production Overheads brought forward from previous month 3,000 Transactions during the month: Materials Purchased 75,000 Materials Issued: Rs. To Production 30,000 To Factory Maintenance 4,000 34,000 Materials transferred between batches 5,000

5.14

Page 309: 30510870 Cost Accounting and Financial Management

Non-Integrated Accounts

Total wages paid: Rs. To Direct workers 25,000 To Indirect workers 5,000 30,000 Direct wages charged to batches 20,000 Recorded non-productive time of direct workers 5,000 Selling and Distribution Overheads Incurred 6,000 Other Production Overheads Incurred 12,000 Sales 1,00,000 Cost of Finished Goods Sold 80,000 Cost of Goods completed and transferred into finished goods during the month 65,000 Physical value of work-in-progress at the end of the month 40,000 The production overhead absorption rate is 150% of direct wages charged to work-in-progress. Required: Prepare the following accounts for the month: (a) Stores Ledger Control Account.

(b) Work-in-Progress Control Account.

(c) Finished Goods Control Account.

(d) Production Overhead Control Account.

(e) Profit and Loss Account.

Solution: (a) Stores Ledger Control Account Rs. Rs. To Balance b/d 25,000 By Work in Progress Control A/c 30,000 ” Creditors (or bank) 75,000 ” Production Overhead Control A/c 4,000 _______ ” Balance c/d 66,000 1,00,000 1,00,000

5.15

Page 310: 30510870 Cost Accounting and Financial Management

Cost Accounting

(b) Work-in-Progress Control Account Rs. Rs. To Balance b/d 20,000 By Finished Goods Control A/c 65,000 ” Store Ledger Control A/c 30,000 ” Balance c/d (Physical value) 40,000 ” Wages Control A/c 20,000 ” Production Overhead Control A/c 30,000 (150% of direct wages) ” Profit & Loss A/c (Stock Gains) 5,000 _______ 1,05,000 1,05,000

(c) Finished Goods Control Account Rs. Rs. To Balance b/d 35,000 By Cost of Goods A/c 80,000 ” Work-in-Progress Control A/c 65,000 or Profit & Loss A/c _______ By Balance c/d 20,000 1,00,000 1,00,000

(d) Production Overhead Control Account Rs. Rs. To Balance b/d (Prepaid amount) 3,000 By Work-in-Progress Control A/c30,000 ” Stores Ledger Control A/c 4,000 (150% of direct wages) ” Wages Control A/c: Direct Workers 5,000 Indirect Workers 5,000 10,000 ” Bank 12,000 ” Profit & Loss A/c* 1,000 (Over absorption, balancing figure) ______ ______ 30,000 30,000 * Alternatively the over absorbed overhead may be carried forward.

5.16

Page 311: 30510870 Cost Accounting and Financial Management

Non-Integrated Accounts

(e) Profit & Loss Account Rs. Rs. To Finished goods By Sales A/c 1,00,000 Control A/c ” Production Overhead or Control A/c 1,000 Cost of goods sold A/c 80,000 ” Work-in-Progress Control A/c ” Selling & Distribution Overhead A/c 6,000 (Stock gain) 5,000 ” Balance c/d 20,000 _______ 1,06,000 1,06,000 Notes: (1) Materials transferred between batches will not affect the Control Accounts.

(2) Non-production time of direct workers is a production overhead and therefore will not be charged to work-in-progress control A/c.

(3) Production overheads absorbed in work-in-progress Control A/c will then equal Rs. 30,000 (150% of Rs. 20,000).

(4) In the work-in-progress Control A/c the excess physical value of stock is taken resulting in stock gain. Stock gain is transferred to Profit & Loss A/c.

Illustration: From the following details show the necessary accounts in the Cost Ledger Materials Work-in- Finished Progress Stock Rs. Rs. Rs. Opening Balance 8,000 5,000 10,000 Closing Balance 11,000 9,000 12,000 Transactions during the period: Rs. Materials purchased 25,000 Wages paid 10,000 (including Rs. 2,000 indirect) Overheads incurred 8,000 Overheads absorbed 9,000 Sales 50,000

5.17

Page 312: 30510870 Cost Accounting and Financial Management

Cost Accounting Solution:

Cost Ledger General Ledger Adjustment Account

Dr. Cr.

Rs. Rs. To Cost of Sales A/c 50,000 By Balance b/d 23,000 To balance c/d 32,000 By Stores Led. Control A/c 25,000 By Wages Control A/c 10,000 By Overheads A/c 8,000 ______ By Costing P & L A/c (Profit) 16,000 82,000 82,000

Stores Ledger Control Account Dr. Cr. Rs. Rs. To Balance b/d 8,000 By Work-in-Progress A/c 22,000 To Gen. Led. Adj. A/c 25,000 By Balance c/d 11,000 33,000 33,000

Work-in-Progress Account Dr. Cr. Rs. Rs. To Balance b/d 5,000 By Finished stock 35,000 To Stores Led. Control A/c 22,000 (Balancing figure) To Wages Control A/c 8,000 By Balance c/d 9,000 To Overheads A/c 9,000 ______ 44,000 44,000

Finished Stock Account Dr. Cr. Rs. Rs. To Balance b/d 10,000 By Cost of Sales A/c 33,000 To W.I.P. A/c 35,000 (Balancing figure) ______ By Balance c/d 12,000 45,000 45,000

5.18

Page 313: 30510870 Cost Accounting and Financial Management

Non-Integrated Accounts

Wages Control Account Dr. Cr. Rs. Rs. To Gen. Led. Adj. A/c 10,000 By Work-in-Progress A/c 8,000 ______ By Overheads A/c 2,000 10,000 10,000

Overheads Account Dr. Cr. Rs. Rs. To General Ledger Adjustment A/c 8,000 By W.I.P. A/c 9,000 To Wages Control A/c 2,000 By Costing P & L A/c 1,000 10,000 10,000

Cost of Sales Account Dr. Cr.

Rs. Rs. To Finished Stock A/c 33,000 By Costing P & L A/c 33,000 33,000 33,000

Costing P & L Account Dr. Cr.

Rs. Rs. To Cost of Sales A/c 33,000 By General Ledger To Overheads (Under absorbed) 1,000 Adjustment A/c (Sales A/c) 50,000 To General Ledger Adjustment A/c (Profit) 16,000 ______ 50,000 50,000Illustration : On 31st March, 2006 the following balances were extracted from the books of the Supreme Manufacturing Company: Dr. Cr. Rs. Rs. Stores Ledger Control A/c 35,000 Work-in-Progress Control A/c 38,000

5.19

Page 314: 30510870 Cost Accounting and Financial Management

Cost Accounting Finished Goods Control A/c 25,000 Cost Ledger Control A/c ______ 98,000 98,000 98,000The following transactions took place in April 2006 : Rs. Raw Materials : Purchased 95,000 Returned to suppliers 3,000 Issued to Production 98,000 Returned to stores 3,000 Productive wages 40,000 Indirect labour 25,000 Factory overhead expenses incurred 50,000 Selling and Administrative expenses 40,000 Cost of finished goods transferred to warehouse 2,13,000 Cost of Goods sold 2,10,000 Sales 3,00,000 Factory overheads are applied to production at 150% of direct wages, any under/over absorbed overhead being carried forward for adjustment in the subsequent months. All administrative and selling expenses are treated as period costs and charged off to the Profit and Loss Account of the month in which they are incurred. Show the following Accounts: (a) Cost Ledger Control A/c

(b) Stores Ledger Control A/c

(c) Work-in-Progress Control A/c

(d) Finished Goods Stock Control A/c

(e) Factory Overhead Control A/c

(f) Costing Profit and Loss A/c

(g) Trial Balance as at 30th April, 2006.

5.20

Page 315: 30510870 Cost Accounting and Financial Management

Non-Integrated Accounts

Solution: (a) Cost Ledger Control A/c Dr. Cr. Rs. Rs. To Costing Profit & By Balance b/d 98,000 Loss A/c (Sales) 3,00,000 ” Stores Ledger Control A/c 95,000 ” Stores Ledger Control A/c 3,000 ” Wages Control A/c 65,000 ” Balance c/d 95,000 (Productive wages + Indirect wages) ” Factory Overhead Control A/c 50,000 ” Selling & Admn. Overhead Expenses 40,000 _______ ” Costing Profit & Loss A/c 50,000 3,98,000 3,98,000

(b) Stores Ledger Control A/c Dr. Cr. Rs. Rs. To Balance b/d 35,000 By Cost Ledger Control A/c 3,000 ” Cost Ledger Control A/c 95,000 ” Work-in-Progress Control A/c 98,000 ” Work-in-Progress Control A/c 3,000 ” Balance c/d 32,000 1,33,000 1,33,000

(c) Work-in-Progress Control A/c Dr. Cr. Rs. Rs. To Balance b/d 38,000 By Stores Ledger Control A/c 3,000 ” Stores Ledger Control A/c 98,000 ” Finished Goods A/c 2,13,000 ” Wages Control A/c 40,000 ” Balance c/d 20,000 ” Factory Overhead control A/c 60,000 _______ 2,36,000 2,36,000

(d) Finished Goods Control A/c Dr. Cr. Rs. Rs. To Balance b/d 25,000 By Cost of goods sold A/c 2,10,000 ” Work in Progress Control A/c 2,13,000 ” Balance c/d 28,000 2,38,000 2,38,000

5.21

Page 316: 30510870 Cost Accounting and Financial Management

Cost Accounting (e) Factory Overhead Control A/c Dr. Cr. Rs. Rs. To Wage Control A/c (Indirect Labour) 25,000 By Work-in-Progress A/c 60,000 ” Cost Ledger Control A/c 50,000 ” Balance c/d 15,000 75,000 75,000

(f) Costing Profit and Loss A/c Dr. Cr. Rs. Rs. To Cost of Goods Sold A/c 2,10,000 By Cost Ledger Control A/c ” Selling and Admn. Overhead A/c 40,000 (Sales) 3,00,000 ” Cost Ledger Control A/c (Costing profit) 50,000 _______ 3,00,000 3,00,000

(g) Trial Balance (as at 30th April, 2006) Dr. Cr. Rs. Rs. To Stores Ledger Control A/c 32,000 ” Work-in-Progress Control A/c 20,000 ” Finished Goods Control A/c 28,000 ” Factory Overhead Control A/c 15,000 ” Cost Ledger Control A/c ______ 95,000 95,000 95,000 Working Notes: (1) Wages Control A/c Dr. Cr. Rs. Rs. To Cost Ledger Control A/c 65,000 By Work-in-Progress Control A/c 40,000 ______ By Factory Overhead Control A/c 25,000 65,000 65,000

5.22

Page 317: 30510870 Cost Accounting and Financial Management

Non-Integrated Accounts

(2) Cost of Goods Sold A/c Dr. Cr. Rs. Rs. To Finished Goods Control A/c 2,10,000 By Costing Profit & Loss A/c 2,10,000 2,10,000 2,10,000

(3) Selling & Administrative Expenses A/c Dr. Cr. Rs. Rs. To Cost Ledger Control A/c 40,000 By Costing Profit & Loss A/c 40,000 40,000 40,000

Illustration:

Acme Manufacturing Co. Ltd. opens the costing records, with the balances as on 1st July, 2005 as follows : Rs. Rs. Material control A/c 1,24,000 Work-in-progress A/c 62,500 Finished Goods A/c 1,24,000 Production Overheads A/c 8,400 Administration Overhead 12,000 Selling and Distribution Overhead A/c 6,250 General Ledger Control A/c _______ 3,13,150 3,25,150 3,25,150

The following are the transactions for the quarter ended 30th September 2005 : Rs. Materials purchased 4,80,100 Materials issued to jobs 4,77,400 Materials to works maintenance 41,200 Materials to administration office 3,400 Materials to selling department 7,200 Wages direct 1,49,300 Wages indirect 65,000 Transportation for incoming materials 8,400

5.23

Page 318: 30510870 Cost Accounting and Financial Management

Cost Accounting Production overheads 2,42,250 Absorbed overheads production 3,59,100 Administration overheads 74,000 Administration allocation to production 52,900 Administration allocation to sales 14,800 Sales overheads 64,200 Sales overheads absorbed 82,000 Finished goods produced 9,58,400 Finished goods sold 9,77,300 Sales Realisation 14,43,000 Make up the various accounts as you envisage in the Cost Ledger and prepare a Trial Balance as at 30th September, 2005. Solution:

Acme Manufacturing Co. Ltd. Cost Ledger

Material Control A/c Dr. Cr. Rs. Rs. To Balance b/d 1,24,000 By Work-in-progress 4,77,400 To General ledger 4,80,100 By Production overheads A/c 41,200 control A/c (Purchases) By Administration overhead A/c 3,400 By Selling and distribution 7,200 overhead A/c _______ By Balance c/d 74,900 6,04,100 6,04,100To Balance b/d 74,900

Wages Control A/c Dr. Cr. Rs. Rs. To General ledger control A/c 2,14,300 By Work-in-progress 1,49,300 (Rs. 1,49,300 + Rs. 65,000) ________ ” Production overheads A/c 65,000 2,14,300 2,14,300

5.24

Page 319: 30510870 Cost Accounting and Financial Management

Non-Integrated Accounts

Production Overhead A/c Dr. Cr. Rs. Rs. To Balance b/d 8,400 By Work-in-progress A/c 3,59,100 To General Ledger control A/c: ” Balance c/d 6,150 Transportation 8,400 Production overheads 2,42,250 2,50,650 To Wages control A/c 65,000 To Material control A/c 41,200 _______ 3,65,250 3,65,250To Balance b/d 6,150

Administration Overhead A/c Dr. Cr. Rs. Rs. To General ledger control A/c 74,000 By Balance b/d 12,000 To Material control A/c 3,400 ” Work-in-progress A/c 52,900 To Balance c/d 2,300 ” Cost of sales A/c 14,800 79,700 79,700 By Balance b/d 2,300

Selling and Distribution Overhead A/c Dr. Cr. Rs. Rs. To Balance b/d 6,250 By Cost of Sales A/c 82,000 To General ledger control A/c 64,200 To Material control A/c 7,200 To Balance c/d 4,350 ______ 82,000 82,000 By balance b/d 4,350

5.25

Page 320: 30510870 Cost Accounting and Financial Management

Cost Accounting

Work-in-Progress A/c Dr. Cr. Rs. Rs. To Balance b/d 62,500 By Finished Goods A/c 9,58,400 ” Material control A/c 4,77,400 ” Balance c/d 1,42,800 ” Wages control A/c 1,49,300 ” Production overheads A/c 3,59,100 ” Administration overhead A/c 52,900 ________ 11,01,200 11,01,200To Balance b/d 1,42,800

Finished Goods A/c Dr. Cr. Rs. Rs. To Balance c/d 1,24,000 By Cost of sales A/c 9,77,300 ” Work-in-progress 9,58,400 By Balance c/d 1,05,100 10,82,400 10,82,400To Balance b/d 1,05,100

Cost of Sales A/c Dr. Cr. Rs. Rs. To Finished goods A/c 9,77,300 By Costing profit & loss A/c 10,74,100 ” Administration overheads A/c 14,800 ” Selling & distribution overheads A/c 82,000 ________ 10,74,100 10,74,100

General Ledger Control A/c Dr. Cr. Rs. Rs. To Costing profit and loss A/c 14,43,000 By Balance b/d 3,13,150 To Balance c/d 3,22,300 ” Material control A/c 4,80,100 ” Wages Control A/c 2,14,300 ” Production overhead A/c 2,50,650

5.26

Page 321: 30510870 Cost Accounting and Financial Management

Non-Integrated Accounts

” Administration overhead A/c 74,000 ” Selling and distribution 64,200 overhead A/c ________ ” Costing profit & loss A/c 3,68,900 17,65,300 17,65,300 By Balance b/d 3,22,300

Costing Profit & Loss A/c Dr. Cr. Rs. Rs. To Cost of sales A/c 10,74,100 By General ledger control A/c ” General ledger control A/c 3,68,900 (Sales) 14,43,000 (Profit for the period) ________ ________ 14,43,000 14,43,000

Trial Balance as at 30th September, 2005 Dr. Cr. Rs. Rs. Material control A/c 74,900 Production overhead A/c 6,150 Administration overhead A/c 2,300 Selling and distribution overhead A/c 4,350 Work-in-progress A/c 1,42,800 Finished goods A/c 1,05,100 General ledger control A/c _______ 3,22,300 3,28,950 3,28,950

Illustration: (a) A fire destroyed some accounting records of a company. You have been able to collect the following from the spoilt papers/records and as a result of consultation with accounting staff in respect of January, 2006:

5.27

Page 322: 30510870 Cost Accounting and Financial Management

Cost Accounting (i) Incomplete Ledger Entries:

Raw–Materials A/c

Rs. Rs. Beginning Inventory 32,000

Work-in-Progress A/c

Rs. Rs. Beginning Inventory 9,200 Finished Stock 1,51,000

Creditors A/c

Rs. Rs. Opening Balance 16,400 Closing Balance 19,200

Manufacturing Overheads A/c

Rs. Rs. Amount Spent 29,600

Finished Goods A/c

Rs. Rs. Opening Inventory 24,000 Closing Inventory 30,000

(ii) Additional Information: (1) The cash-book showed that Rs. 89,200 have been paid to creditors for raw-material.

(2) Ending inventory of work-in-progress included material Rs. 5,000 on which 300 direct labour hours have been booked against wages and overheads.

(3) The job card showed that workers have worked for 7,000 hours. The wage rate is Rs. 10 per labour hour.

(4) Overhead recovery rate was Rs. 4 per direct labour hour.

You are required to complete the above accounts in the cost ledger of the company:

5.28

Page 323: 30510870 Cost Accounting and Financial Management

Non-Integrated Accounts

Solution: (a) Dr. Creditors A/c Cr.

Rs. Rs.

To Cash & Bank (1) 89,200 By Balance b/d 16,400 To Balance c/d 19,200 By Purchases (Balancing figure) 92,000 1,08,400 1,08,400

Dr. Work-in-progress A/c Cr.

Rs. Rs.

To Balance b/d 9,200 By Finished stock 1,51,000 To Raw-materials 53,000 By Balance c/d : (Balancing figure) Material (2) : Rs. 5,000 To Wages (3) 70,000 Labour (2) : Rs. 3,000 (7,000 hrs. × Rs. 10) (300 hrs. × 10 hrs.) To Overheads (4) 28,000 Overheads (2) : Rs. 1,200 9,200 (7,000 hrs. × Rs. 4) _______ (300 hrs. × 4 hrs.) _______ 1,60,200 1,60,200 Dr. Raw-materials A/c Cr.

Rs. Rs.

To Balance b/d 32,000 By Work-in-progress 53,000 To Purchases 92,000 (As above) (As above) _______ By Balance c/d 71,000 1,24,000 1,24,000 Dr. Finished Goods A/c Cr.

Rs. Rs.

To Balance b/d 24,000 By Cost of sales 1,45,000 To W.I.P. 1,51,000 (Balancing figure) (As above) _______ By Balance c/d 30,000 1,75,000 1,75,000

5.29

Page 324: 30510870 Cost Accounting and Financial Management

Cost Accounting Dr. Manufacturing Overheads A/c Cr.

Rs. Rs. To Sundries 29,600 By W.I.P. (7000 hrs. × Rs. 4) 28,000 ______ By Under-absorbed Overheads A/c 1,600 29,600 29,600 Illustration : The following incomplete accounts are furnished to you for the month ended 31st October, 2005.

Stores Control Account

1.10.05 To Balance Rs. 54,000

Work in Progress Control Account

1.10.05 To Balance Rs. 6,000

Finished Goods Control Account

1.10.05 To Balance Rs. 75,000

Factory Overheads Control Account

Total debits for October, 2005 Rs. 45,000

Factory Overheads Applied Account

Cost of Goods Sold Account

5.30

Page 325: 30510870 Cost Accounting and Financial Management

Non-Integrated Accounts

Creditors for Purchases Account 1.10.05 by Balance Rs. 30,000 Additional information: (i) The factory overheads are applied by using a budgeted rate based on Direct Labour

Hours. The budget for overheads for 2005 is Rs. 6,75,000 and the budget of direct labour hours is 4,50,000.

(ii) The balance in the account of creditors for purchases on 31.10.05 is Rs. 15,000 and the payments made to creditors in October, 2005 amount to Rs. 1,05,000.

(iii) The finished goods inventory as on 31st October, 2005 is Rs. 66,000.

(iv) The cost of goods sold during the month was Rs. 1,95,000.

(v) On 31st October, 2005 there was only one unfinished job in the factory. The cost records show that Rs. 3,000 (1,200 direct labour hours) of Direct Labour Cost and Rs. 6,000 of Direct Material Cost had been charged.

(vi) A total of 28,200 direct labour hours were worked in October, 2005. All factory workers earn same rate of pay.

(vii) All actual factory overheads incurred in October, 2005 have been posted. You are required to find: (a) Materials purchased during October, 2005.

(b) Cost of goods completed in October, 2005.

(c) Overheads applied to production in October, 2005.

(d) Balance of work in progress on 31st October, 2005.

(e) Direct materials consumed during October, 2005.

(f) Balance of Stores Control Account on 31st October, 2005.

(g) Overabsorbed or underabsorbed overheads for October, 2005.

5.31

Page 326: 30510870 Cost Accounting and Financial Management

Cost Accounting Solution : Working Notes : (i) Overhead recovery rate per direct labour hour : Budgeted factory overheads : Rs. 6,75,000 Budgeted direct labour hours : 4,50,000

Overhead recovery rate =hours labourdirect Budgeted

overheadsfactory Budgeted

= hours 000,50,4000,75,6.Rs

= Rs. 1.50 per direct labour (ii) Direct wage rate per hour : Direct labour cost of WIP : Rs. 3,000 (on 31st October 2005) Direct labour hours of WIP : 1,200 hours

Direct wage rate per hour = WIP on hours labourDirect WIP oncost labourDirect

= hours 1,200Rs.3,000

= Rs.2.50

(iii) Total direct wages charged to production: Total direct labour hours spent on production × Direct wage rate per hour = 28,200 hours × Rs. 2.50 = Rs. 70,500

5.32

Page 327: 30510870 Cost Accounting and Financial Management

Non-Integrated Accounts

(a) Material purchased during October, 2005 Rs.

Payment made to creditors 1,05,000 Add : Closing balance in the account of creditors for purchase 15,000 1,20,000 Less : Opening balance 30,000 Material purchased 90,000

(b) Cost of goods completed in October, 2005 Rs.

The cost of goods sold during the month 1,95,000 Add : Closing finished goods inventory 66,000 2,61,000 Less : Opening finished goods inventory 75,000 Cost of goods completed during the month 1,86,000

(c) Overhead applied to production in October, 2005 = 28,200 hours × Rs. 1.50 = Rs. 42,300

(d) Balance of Work-in-progress on 31st October, 2005 Rs.

Direct material cost 6,000 Direct labour cost 3,000 Overheads (1,200 hours × Rs. 1.50) 1,800 10,800

(e) Direct material consumed during October, 2005 Rs. 75,000 (Refer to following Account)

5.33

Page 328: 30510870 Cost Accounting and Financial Management

Cost Accounting Dr. Work in progress Control A/c Cr.

Rs. Rs. 1.10.05 To Opening Balance 6,000 By Finished goods 1,86,000 ’’ To Direct wages 70,500 (As per (b) above) [Refer to Working Note (iii)] By Balance of WIP 10,800 To Factory Overheads 42,300 (As per (d) above) [As per (c) above]

To Balancing figure 78,000 (Material consumed) _______ _______ 1,96,800 1,96,800

(f) Balance of Stores Control Account on 31st October, 2005 Rs. 66,000 (Refer to following Account)

Dr. Stores Control Account Cr. Rs. Rs. 1.10.05 To Balance 54,000 31.10.05 By W.I.P. Control A/c 78,000 To Creditors A/c 90,000 (As per (e) above) for materials purchased By Balance c/d 66,000 (As per (a) above) _______ (Balancing figure) _______ 1,44,000 1,44,000

(g) Over-absorbed or under-absorbed overheads for October, 2005 : Balance in Factory Overhead Account below showing that Rs. 2,700 is underabsorbed.

Dr. Factory Overhead Account Cr. Rs. Rs. To General Ledger Adj. A/c 45,000 31.10.99 By Factory overhead applied 42,300 (Total debits for Oct. 2005) [Refer to (c) above] _____ Balance (Under-absorbed) 2,700 45,000 45,000

5.3 INTEGRATED (OR INTEGRAL) ACCOUNTING SYSTEM Integrated Accounts is the name given to a system of accounting, whereby cost and financial accounts are kept in the same set of books. Obviously, then there will be no separate sets of books for Costing and Financial records. Integrated accounts provide or meet out fully the

5.34

Page 329: 30510870 Cost Accounting and Financial Management

Non-Integrated Accounts

information requirement for Costing as well as for Financial Accounts. For Costing it provides information useful for ascertaining the Cost of each product, job, process , operation of any other identifiable activity and for carrying necessary analysis. Integrated accounts provide relevant information which is necessary for preparing profit and loss account and the balance sheets as per the requirement of law and also helps in exercising effective control over the liabilities and assets of its business. 5.3.1 Advantages : The main advantages of Integrated Accounts are as follows : (a) The question of reconciling costing profit and financial profit does not arise, as there is

one figure of profit only.

(b) Due to use of one set of books, there is a significant extent of saving in efforts made.

(c) No delay is caused in obtaining information as it is provided from books of original entry.

(d) It is economical also as it is based on the concept of “Centralisation of Accounting function”.

5.3.2 Essential pre-requisites for Integrated Accounts: The essential pre-requisites for integrated accounts include the following steps: 1. The management’s decision about the extent of integration of the two sets of books.

Some concerns find it useful to integrate up to the stage of primary cost or factory cost while other prefer full integration of the entire accounting records.

2. A suitable coding system must be made available so as to serve the accounting purposes of financial and cost accounts.

3. An agreed routine, with regard to the treatment of provision for accruals, prepaid expenses, other adjustment necessary for preparation of interim accounts.

4. Perfect coordination should exist between the staff responsible for the financial and cost aspects of the accounts and an efficient processing of accounting documents should be ensured.

Under this system there is no need for a separate cost ledger. Of course, there will be a number of subsidiary ledgers; in addition to the useful Customers’ Ledger and the Bought Ledger, there will be: (a) Stores Ledger; (b) Stock Ledger and (c) Job Ledger. Also, control accounts are maintained in the financial ledger for each one of these. The nature of these will be the same as discussed above. But there will be no General Ledger Adjustment Account and the entries will be passed in the way these are usually passed in the financial books when expenses are incurred. When these are apportioned or allocated to cost units, the

5.35

Page 330: 30510870 Cost Accounting and Financial Management

Cost Accounting entries are made on similar lines as outlined above for the Cost Ledger. If the illustration given below is to be worked out on integrated account basis, the journal entries would be as follows:

Illustration : Journalise the following transactions assuming that cost and financial transactions are integrated: Rs. Raw materials purchased 2,00,000 Direct materials issued to production 1,50,000 Wages paid (30% indirect) 1,20,000 Wages charged to production 84,000 Manufacturing expenses incurred 84,000 Manufacturing overhead charged to production 92,000 Selling and distribution costs 20,000 Finished products (at cost) 2,00,000 Sales 2,90,000 Closing stock Nil Receipts from debtors 69,000 Payments to creditors 1,10,000 Solution:

Journal Dr. Cr. Rs. Rs. Stores Ledger Control A/c Dr. 2,00,000 To Creditors A/c 2,00,000 (Material Purchased) Work-in-Progress Control A/c Dr. 1,50,000 To Stores Ledger Control A/c 1,50,000 (Materials issued to production) Wages Control A/c Dr. 1,20,000 To Bank A/c 1,20,000 (Wages paid)

5.36

Page 331: 30510870 Cost Accounting and Financial Management

Non-Integrated Accounts

Factory Overhead Control A/c Dr. 36,000 To Wages Control A/c 36,000 (30% of wages paid being indirect charged to overhead) Work-in-Progress Control A/c Dr. 84,000 To Wages Control A/c 84,000 (Direct wages charged to production) Factory Overhead Control A/c Dr. 84,000 To Bank A/c 84,000 (Manufacturing overhead incurred) Work-in-Progress Control A/c Dr. 92,000 To Factory overhead charged A/c 92,000 (Manufacturing overhead charged to production) Selling and Distribution Overhead Control A/c Dr. 20,000 To Bank A/c 20,000 (Selling and distribution costs incurred) Finished Goods Ledger Control A/c Dr. 2,00,000 To Work-in-Progress Control A/c 2,00,000 (Cost of goods finished) Cost of Sales A/c Dr. 2,20,000 To Finished Stock Ledger Control A/c 2,00,000 To Selling and Distribution Control A/c 20,000 (Costs of goods sold) Sundry Debtors A/c Dr. 2,90,000 To Sales A/c 2,90,000 (Finished stock sold) Bank A/c Dr. 69,000 To Sundry Debtors A/c 69,000 (Receipts from debtors) Sundry (Creditors) A/c Dr. 1,10,000

5.37

Page 332: 30510870 Cost Accounting and Financial Management

Cost Accounting To Bank A/c 1,10,000 (Payment made to creditors) Illustration : Dutta Enterprises operates an integral system of accounting. You are required to pass the Journal Entries for the following transactions that took place for the year ended 30th June, 2006. (Narrations are not required.) Rs. Raw materials purchased (50% on Credit) 6,00,000 Materials issued to production 4,00,000 Wages paid (50% Direct) 2,00,000 Wages charged to production 1,00,000 Factory overheads incurred 80,000 Factory overheads charged to production 1,00,000 Selling and distribution overheads incurred 40,000 Finished goods at cost 5,00,000 Sales (50% Credit) 7,50,000 Closing stock Nil Receipts from debtors 2,00,000 Payments to creditors 2,00,000

Solution : Journal Entries under integrated system of accounting for transactions

taking place for the year ended on 30th June, 2006 Dr. Cr. Rs. Rs. Stores Ledger Control A/c Dr. 6,00,000 To Sundry Creditors Account 3,00,000 To Cash or Bank Account 3,00,000 Work-in-Progress Control A/c Dr. 4,00,000 To Stores Ledger Control A/c 4,00,000 Wages Control A/c Dr. 2,00,000 To Cash or Bank Account 2,00,000

5.38

Page 333: 30510870 Cost Accounting and Financial Management

Non-Integrated Accounts

Work-in-Progress Control A/c Dr. 1,00,000 To Wages Control A/c 1,00,000 Factory Overhead Control A/c Dr.1,00,000 To Wages Control A/c 1,00,000 Factory Overhead Control A/c Dr. 80,000 To Cash or Bank A/c 80,000 Work-in-Progress Control A/c Dr.1,00,000 To Factory Overhead Control A/c 1,00,000 Selling and Distribution Overhead Control A/c Dr. 40,000 To Cash or Bank A/c 40,000 Finished Stock Ledger Control A/c Dr.5,00,000 To Work-in-Progress Control A/c 5,00,000 Cost of Sales A/c Dr.5,40,000 To Finished Stock Ledger Control A/c 5,00,000 To Selling and Distribution Control A/c 40,000 Sundry Debtors Account Dr.3,75,000 Cash or Bank Account Dr.3,75,000 To Sales Account 7,50,000 Cash or Bank A/c Dr.2,00,000 To Sundry Debtors A/c 2,00,000 Sundry Creditors A/c Dr.2,00,000 To Cash or Bank A/c 2,00,000

5.39

Page 334: 30510870 Cost Accounting and Financial Management

Cost Accounting Illustration : Bangalore Petrochemicals Co. keeps books on integrated accounting system. The following balances appear in the books as on 1st January, 2005. Dr. Cr. Rs. Rs. Stores control A/c 18,000 Work-in-Progress A/c 17,000 Finished goods A/c 13,000 Bank A/c 10,000 Creditors A/c 8,000 Fixed assets A/c 55,000 Debtors A/c 12,000 Share capital A/c 80,000 Depreciation provision A/c 5,000 Profit and loss A/c _______ 32,000 1,25,000 1,25,000 Transaction for the year ended 31st Dec., 2005 were as given below: Rs. Rs. Wages-direct 87,000 Wages-indirect 5,000 92,000 Purchase of materials (on credit) 1,00,000 Materials issued to production 1,10,000 Materials for repairs 2,000 Goods finished during the year (at cost) 2,15,000 Sales (credit) 3,00,000 Cost of goods sold 2,20,000 Production overhead absorbed 48,000 Production overhead incurred 40,000 Administration overhead incurred 12,000 Selling overhead incurred 14,000 Payments of creditors 1,01,000 Payments of debtors 2,90,000 Depreciation of machinery 1,300 Prepaid rent (included in factory overheads) 300

5.40

Page 335: 30510870 Cost Accounting and Financial Management

Non-Integrated Accounts

Write up accounts in the integrated ledger and prepare a trial balance.

Solution : Stores Control Account

Dr. Cr. 2005 Rs. 2005 Rs. Jan. 1 To Balance b/d 18,000 Dec. 31 ByWork-in-Progress A/c 1,10,000 Dec. 31 ” Creditors A/c 1,00,000 ” Production overheads A/c 2,000 ________ ” Balance c/d 6,000 1,18,000 1,18,0002006 Jan. 1 To Balance b/d 6,000

Wages Control Account

2005 Rs. 2005 Rs. Dec. 31 To Bank A/c 92,000 Dec. 31 By Work-in-Progress A/c 87,000 ______ ” Production overheads A/c 5,000 92,000 92,000

Work-in-Progress A/c

2005 Rs. 2005 Rs. Jan. 1 To Balance b/d 17,000 Dec. 31 By Finished goods A/c 2,15,000 Dec. 31 ” Stores control A/c 1,10,000 ” Balance c/d 47,000 ” Wages control A/c 87,000 ” Production overheads A/c 48,000 _______ 2,62,000 2,62,0002006 Jan. 1 To Balance b/d 47,000

Production Overhead A/c 2005 Rs. 2005 Rs. Dec. 31 To Wages Control A/c 5,000 Dec. 31 By Work-in-Progress A/c 48,000 ” Stores Control A/c 2,000 ” Prepaid Rent A/c 3,00 ” Bank A/c 40,000 ” Depreciation Provision 1,300 ______ 48,300 48,300

5.41

Page 336: 30510870 Cost Accounting and Financial Management

Cost Accounting

Finished Goods A/c 2005 Rs. 2005 Rs. Jan. 1 To Balance b/d 13,000 Dec. 31 By Cost of Sales A/c 2,20,000 Dec. 31 ” Work-in-Progress 2,15,000 ” Balance c/d 20,000 ” Admn. Overhead 12,000 _______ 2,40,000 2,40,0002006 Jan. 1 ” Balance b/d 20,000

Administration Overheads A/c

2005 Rs. 2005 Rs. Dec. 31 To Bank A/c 12,000 Dec. 31 By Finished Goods A/c 12,000 12,000 12,000

Cost of Sales A/c

2005 Rs. 2005 Rs. Dec. 31 To Finished Goods A/c 2,20,000 Dec. 31 By Sales A/c 2,34,000 ” Selling and Dist. Overheads A/c 14,000 _______ 2,34,000 2,34,000

Selling and Distribution Overheads A/c 2005 Rs. 2005 Rs. Dec. 31 To Bank A/c 14,000 Dec. 31 By Cost of Sales A/c 14,000 14,000 14,000

Sales A/c 2005 Rs. 2005 Rs. Dec. 31 To Cost of Sales 2,34,000 Dec. 31 By Debtors A/c ” P & L A/c (Profit) 66,000 (Cr. Sales) 3,00,000 3,00,000 3,00,000

5.42

Page 337: 30510870 Cost Accounting and Financial Management

Non-Integrated Accounts

Prepaid Rent A/c 2005 Rs. 2005 Rs. Dec. 31 To Production Overheads 300 Dec. 31 By Balance c/d 300 300 300 2006 Jan. 1 To Balance b/d 300

Depreciation Provision A/c

2005 Rs. 2005 Rs. Dec. 31 To Balance c/d 6,300 Jan. 1 By Balance b/d 5,000 2005 _____ Dec. 31 ” Production Overhead A/c 1,300 6,300 6,300 2006 Jan.1 By Balance b/d 6,300

Profit and Loss A/c

2005 Rs. 2005 Rs. Dec. 31 To Balance c/d 98,000 Dec. 31 By Sales A/c 66,000 ______ ” Profit b/d (last year) 32,000 98,000 98,000 2006 Jan. 1 By Balance b/d 98,000

Debtors A/c

2005 Rs. 2005 Rs. Jan. 1 To Balance b/d 12,000 Dec. 31 By Bank A/c 2,90,000 Dec. 31 To Sales 3,00,000 By Balance c/d 22,000 3,12,000 3,12,0002006 Jan. 1 To Balance b/d 22,000

5.43

Page 338: 30510870 Cost Accounting and Financial Management

Cost Accounting

Creditors A/c 2005 Rs. 2005 Rs. Dec. 31 To Bank 1,01,000 Jan. 1 By Balance b/d 8,000 To Balance c/d 7,000 Dec. 31 By Stores Control A/c 1,00,000 1,08,000 1,08,0002006 Jan. 1 By Balance b/d 7,000

Bank A/c

2005 Rs. 2005 Rs. Jan. 1 To Balance b/d 10,000 Dec. 31 By Creditors 1,01,000 Dec. 31 ” Debtors 2,90,000 ” Wages Control A/c 92,000 ” Production Overhead A/c 40,000 ” Admn. Overhead A/c 12,000 ” Selling & Distribution Overhead A/c 14,000 _______ ” Balance c/d 41,000 3,00,000 3,00,000 2006 Jan. 1 To Balance b/d 41,000

Fixed Assets A/c

2005 Rs. 2005 Rs. Jan. 1 To Balance b/d 55,000 Dec. 31 By Balance c/d 55,000 55,000 55,0002006 Jan. 1 To Balance b/d 55,000

Share Capital A/c 2005 Rs. 2005 Rs. Dec. 31 To Balance c/d 80,000 Jan. 1 By Balance b/d 80,000 80,000 80,0002006 Jan. 1 By Balance b/d 80,000

5.44

Page 339: 30510870 Cost Accounting and Financial Management

Non-Integrated Accounts

Trial Balance As on 31st December, 2005

Dr. Cr. Rs. Rs. Stores Control A/c 6,000 Work-in-Progress A/c 47,000 Finished Goods A/c 20,000 Bank A/c 41,000 Creditors A/c 7,000 Fixed Assets A/c 55,000 Debtors A/c 22,000 Share Capital A/c 80,000 Depreciation Provision A/c 6,300 Profit and Loss A/c 98,000 Prepaid Rent A/c 300 _______ 1,91,300 1,91,300 Illustration: A company operates on historic job cost accounting system, which is not integrated with the financial accounts. At the beginning of a month, the opening balances in cost ledger were:

Rs. (in lakhs)

Stores Ledger Control Account 80 Work-in-Progress Control Account 20 Finished Goods Control Account 430 Building Construction Account 10 Cost Ledger Control Account 540 During the month, the following transaction took place: Materials − Purchased 40 Issued to production 50 Issued to general maintenance 6 Issued to building construction 4 Wages − Gross wages paid 150

5.45

Page 340: 30510870 Cost Accounting and Financial Management

Cost Accounting Indirect wages 40 For building construction 10 Works Overheads − Actual amount incurred 160 (excluding items shown above) Absorbed in building construction 20 Under absorbed 8 Royalty paid 5 Selling, distribution and administration overheads 25 Sales 450

At the end of the month, the stock of raw material and work-in-progress was Rs. 55 lakhs and Rs. 25 lakhs respectively. The loss arising in the raw material accounts is treated as factory overheads. The building under construction was completed during the month. Company’s gross profit margin is 20% on sales. Prepare the relevant control accounts to record the above transactions in the cost ledger of the company. Solution: Dr. Cost Ledger Control Act Cr.

Rs. Rs.

To Costing P & L A/c 450 By Balance b/d 540 To Stores Ledger Control A/c 55 By Stores Ledger Control A/c 40 To WIP Control A/c 25 By Wages Control A/c 150 To Building Const. A/c 44 By Works Overhead Control A/c 160 To Finished Goods Control A/c 403 By Royalty A/c 5 By Selling, Distribution and Administration Overheads A/c 25 ___ By Costing Profit & Loss A/c 57 977 977

5.46

Page 341: 30510870 Cost Accounting and Financial Management

Non-Integrated Accounts

Dr. Stores Ledger Control A/c Cr.

Rs. Rs.

To Balance b/d 80 By WIP Control A/c 50 To Cost Ledger Control A/c 40 By Works Overhead Control A/c 6 By Building Const. A/c 4 By Closing Balance 4 By Works Overhead Control A/c 5 ___ (Loss) ___ 120 120 Dr. Work-in-Progress Control A/c Cr.

Rs. Rs.

To Balance b/d 20 By Finished Goods Control A/c 333 To Stores Ledger Control A/c 50 By Closing Balance 25 To Wage Control A/c 100 To Works Overhead Control A/c 183 To Royalty A/c 5 ___ 358 358 Dr. Works Overhead Control A/c Cr.

Rs. Rs. To Stores Ledger Control A/c 6 By Building Const. A/c 20 To Wage Control A/c 40 By WIP Control A/c 183 To Cost Ledger Control A/c 160 By Balance (Costing P & L A/c) 8 To Stores Ledger Control A/c (Loss) 5 ___ 211 211 Dr. Wages Control A/c Cr.

Rs. Rs. To Cost Ledger Control A/c 150 By Works Overhead Control A/c 40 By Building Const. A/c 10 ___ By WIP Control A/c 100 150 150

5.47

Page 342: 30510870 Cost Accounting and Financial Management

Cost Accounting Dr. Royalty A/c Cr.

Rs. Rs.

To Cost Ledger Control A/c 5 By WIP Control A/c 5 5 5 Dr. Cost of Goods Sold A/c Cr.

Rs. Rs.

To Finished Goods Control A/c 360 By Cost of Sales A/c 360 360 360 Dr. Selling, Distribution and Administration Overhead A/c Cr.

Rs. Rs.

To Cost Ledger Control A/c 25 By Cost of Sales A/c 25 25 25 Dr. Finished Goods Control A/c Cr.

Rs. Rs.

To Balance b/d 430 By Cost of Goods Sold A/c 360 (Refer to Working Note)

To WIP Control A/c 333 By Balance 403 763 763 Dr. Cost of Sales A/c Cr.

Rs. Rs. To Cost of Goods Sold A/c 360 By Costing P & L A/c 385 To Selling, distribution and administration overhead A/c 25 ___ 385 385

5.48

Page 343: 30510870 Cost Accounting and Financial Management

Non-Integrated Accounts

Dr. Costing P & L A/c Cr. Rs. Rs.

To Cost of Sales A/c 385 By Cost Ledger Control A/c 450 To Works Overhead Control A/c 8 To Cost Ledger Control A/c (Profit) 57 ___ 450 450 Dr. Building Construction A/c Cr.

Rs. Rs. To Balance b/d 10 By Cost Ledger Control A/c 44 To Stores Ledger Control A/c 4 To Wage Control A/c 10 To Works Overhead Control A/c 20 __ 44 44

(Rs. in lakhs)

Trial Balance Dr. Cr. To Stores Ledger Control A/c 55 To WIP Control A/c 25 To Finished Goods Control A/c 403 To Cost Ledger Adjustment A/c ___ 483 483 483

5.4 RECONCILIATION OF COST AND FINANCIAL ACCOUNTS When the cost and financial accounts are kept separately, it is imperative that those should be reconciled, otherwise the cost accounts would not be reliable. In this connection, it is neces-sary to remember that a reconciliation of the two sets of accounts only can be made if both the sets contain sufficient details as would enable the causes of differences to be located. It is, therefore, important that in the financial accounts, the expenses should be analysed in the same way as in the cost accounts. In the text book, there appears a General Ledger Adjustment Account as would appear in the Cost Ledger, students should study the entries therein as well as a discussion that follows to explain the manner in which the details of items included therein could be reconciled with the corresponding items appearing in the financial accounts. They would thus realise that the

5.49

Page 344: 30510870 Cost Accounting and Financial Management

Cost Accounting reconciliation of the balances generally, is possible preparing a Memorandum Reconciliation Account. In this account, the items charged in one set of accounts but not in the other or those charged in excess as compared to that in the other are collected and by adding or subtracting them from the balance of the amount of profit shown by one of the accounts, shown by the other can be reached. The procedure is similar to the one followed for reconciling the balance with a bank that shown by the cash book or the ledger. It is important, however, to know the causes which, generally, give rise to differences in the Cost and Financial Accounts. These are briefly summarised below : 5.4.1 Items included in the financial accounts but not in cost accounts : (a) Matters of pure finance : (i) Interest received on bank deposits. (ii) Interest, dividends, etc. received on investments. (iii) Rents receivable. (iv) Losses on the sales of investments, building etc. (v) Profits made on the sale of fixed assets. (vi) Expenses of the company’s share transfer office, if any. (vii) Transfer fee received. (viii) Remuneration paid to the proprietor in excess of a fair reward for services rendered. (ix) Damages payable at law. (x) Penalties payable at law. (xi) Losses due to scrapping of machinery. (b) Item included in the cost accounts only (notional expenses): (i) Charges in lieu of rent where premises are owned.

(ii) Interest on capital employed in production, but upon which no interest is actually paid if the firm decided to treat interest as part of cost.

(iii) Salary for the proprietor where he works but does not charge a salary. (c) Items whose treatment is different in the two sets of accounts. The objective of cost

accounting is to provide information to management for decision making and control purposes while financial accounting conforms to external reporting requirements. Hence there are chances that certain items are treated differently in the two sets of accounts. For example, LIFO method is not allowed for inventory valuation in India as per the Accounting Standard 2 issued by the Council of the ICAI. However, this method may be

5.50

Page 345: 30510870 Cost Accounting and Financial Management

Non-Integrated Accounts

adopted for cost accounts as it is more suitable for arriving at costs which shall be used as a base for deciding selling prices. Similarly cost accounting may use a different method of depreciation than what is allowed under financial accounting.

(d) Varying basis of valuation: It is another factor which sometimes is responsible for the difference. It is well known that in financial accounts stock are valued either at cost or market price, whichever is lower. But in Cost Accounts, stocks are only valued at cost.

Circumstances where reconciliation statement can be avoided When the Cost and Financial Accounts are integrated - there is no need to have a separate reconciliation statement between the two sets of accounts. Integration means that the same set of accounts fulfill the requirement of both i.e., Cost and Financial Accounts.

Illustration : The following figures are available from the financial records of ABC Manufacturing Co. Ltd. for the year ended 31-3-2006. Rs. Sales (20,000 units) 25,00,000 Materials 10,00,000 Wages 5,00,000 Factory Overheads 4,50,000 Office and administrative Overhead 2,60,000 Selling and distribution Overheads 1,80,000 Finished goods (1,230 units) 1,50,000 Rs. Work-in-Progress : Materials 30,000 Labour 20,000 Factory overheads 20,000 70,000 Goodwill written off 2,00,000 Interest on capital 20,000 In the Costing records, factory overhead is charged at 100% wages, administration overhead 10% of factory cost and selling and distribution overhead at the rate of Rs. 10 per unit sold. Prepare a statement reconciling the profit as per cost records with the profit as per financial records.

5.51

Page 346: 30510870 Cost Accounting and Financial Management

Cost Accounting Solution :

Profit & Loss Account of ABC Manufacturing Co. Ltd. (for the year ended 31-3-2006)

Rs. Rs. To Opening Stock Nil By Sales (20,000 units) 25,00,000 To Materials 10,00,000 ” Closing Stock : To Wages 5,00,000 ” Finished goods (1,230 units) 1,50,000 To Factory Overheads 4,50,000 Work-in-Progress 70,000 To Office & Admn. Overheads 2,60,000 To Selling & Dist. Overheads 1,80,000 To Goodwill written off 2,00,000 To Interest on Capital 20,000 To Profit 1,10,000 ________ 27,20,000 27,20,000

Cost Sheet Rs. Materials 10,00,000 Wages 5,00,000 Direct Expenses Nil Prime Cost 15,00,000 Add : Factory overhead at 100% wages 5,00,000 20,00,000 Less : Closing WIP 70,000 Factory Cost of (20,000 + 1,230) units 19,30,000 Office & Admn. Overhead 10% of Factory cost 1,93,000 21,23,000 Less: Closing Stock of finished goods (1,230 units) 1,23,000 Production Cost of 20,000 units 20,00,000 Selling & Dist. Overhead @ Rs. 10 per unit 2,00,000 Cost of sales of 20,000 units 22,00,000 Sales of 20,000 units 25,00,000 Profit 3,00,000

5.52

Page 347: 30510870 Cost Accounting and Financial Management

Non-Integrated Accounts

Reconciliation Statement Rs. Rs. Profit as per Cost Accounts 3,00,000 Add : Factory overheads over-absorbed (Rs. 5,00,000 – Rs. 4,50,000) 50,000 Selling & Dist. Overhead over-absorbed (Rs. 2,00,000 – Rs. 1,80,000) 20,000 Difference in the valuation of closing stock of finished goods (Rs. 1,50,000 – Rs. 1,23,000) 27,000 97,000 3,97,000 Less: Office & Admn. overhead under-absorbed (Rs. 2,60,000 – Rs. 1,93,000) 67,000 Goodwill written off taken in financial accounts 2,00,000 Interest on capital 20,000 2,87,000 Profit as per financial accounts 1,10,000

Illustration : Following are the figures extracted from the Cost Ledger of a manufacturing unit. Stores : Rs. Opening balance 15,000 Purchases 80,000 Transfer from WIP 40,000 Issue of WIP 80,000 Issue to repairs and maintenance 10,000 Sold as a special case of cost 5,000 Shortage in the year 3,000 Work-in-Progress : Opening inventory 30,000 Direct labour cost charged 30,000 Overhead cost charged 1,20,000 Closing Balance 20,000 Finished Products : Entire output is sold at 10% profit on actual cost from work-in-process. Others : Wages for the period 35,000 Overhead Expenses 1,25,000 Ascertain the profit or loss as per financial account and cost accounts and reconcile them.

5.53

Page 348: 30510870 Cost Accounting and Financial Management

Cost Accounting Solution :

Stores Control A/c

Dr. Cr. Rs. Rs. To Opening Balance 15,000 By WIP Control A/c 80,000 ” Purchases 80,000 ” Overhead Control A/c 10,000 ” WIP Control A/c 40,000 ” GLA A/c 5,000 ” Overhead Control A/c (Shortages) 3,000 _______ ” Balance 37,000 1,35,000 1,35,000

Wage Control A/c

Rs. Rs. To GLA A/c 35,000 By WIP Control A/c 30,000 ______ ” Overhead Control A/c 5,000 35,000 35,000

Overhead Control A/c

Rs. Rs. To Material Control A/c 10,000 By WIP Control A/c 1,20,000 ” Stores Ledger Control A/c 3,000 By Balance c/d 23,000 ” GLA A/c 1,25,000 ” Wage Control A/c 5,000 _______ 1,43,000 1,43,000

WIP Control A/c Rs. Rs. To Stores Control A/c 80,000 By Stores Control A/c 40,000 ” Opening WIP A/c 30,000 ” Closing Balance 20,000 ” Wage Control A/c 30,000 ” Finished goods 2,00,000 ” Overhead Control A/c 1,20,000 _______ 2,60,000 2,60,000

5.54

Page 349: 30510870 Cost Accounting and Financial Management

Non-Integrated Accounts

Rs.

Finished output at cost 2,00,000

Profit at 10% on actual cost from WIP Sales 20,000

2,20,000

Statement of Profit as per Costing Records

Rs. Direct material Cost 40,000 Direct wages 30,000 Prime Cost 70,000 Production Overheads 1,20,000 Works Cost 1,90,000 Add: Opening WIP 30,000 2,20,000 Less: Closing WIP 20,000 Cost of finished goods 2,00,000 Profit (10% of cost) 20,000 Sales 2,20,000

Profit & Loss A/c Rs. Rs. To Material (Op. bal. + By Closing WIP 20,000 Purchases = Sale) 90,000 ” Sale 2,20,000 ” WIP 30,000 ” Balance 37,000 ” Wages 35,000 ” Net loss 3,000 ” Overhead 1,25,000 _______ 2,80,000 2,80,000

Reconciliation Statement Rs. Profit (loss) as per Financial Accounts (3,000) Add: Overheads overabsorbed in Cost A/c 23,000 Net Profit as per Accounts 20,000

5.55

Page 350: 30510870 Cost Accounting and Financial Management

Cost Accounting Illustration: The following figures, have been extracted from the Financial Accounts of a Manufacturing Firm for the first year of its operation: Rs. Direct Material Consumption 50,00,000 Direct Wages 30,00,000 Factory Overhead 16,00,000 Administration Overheads 7,00,000 Selling and Distribution Overheads 9,60,000 Bad Debts 80,000 Preliminary Expenses written off 40,000 Legal Charges 10,000 Dividends Received 1,00,000 Interest Received on Deposits 20,000 Sales (1,20,000 units) 1,20,00,000 Closing Stock : Finished Goods (4,000 units) 3,20,000 Work-in-Progress 2,40,000 The cost accounts for the same period reveal that the direct material consumption was Rs.56,00,000. Factory overhead is recovered at 20% on prime cost. Administration overhead is recovered at Rs. 6 per unit of production. Selling and distribution overheads are recovered at Rs. 8 per unit sold. Prepare the Profit and Loss Accounts both as per financial records and as per cost records. Reconcile the profits as per the two records. Solution:

Profit and Loss Account (As per financial records)

Rs. Rs. To Direct Material 50,00,000 By Sales (1,20,000 units) 1,20,00,000 To Direct Wages 30,00,000 By Closing Stock To Factory Overheads 16,00,000 WIP 2,40,000 ” Gross Profit 29,60,000 Finished Goods (4,000 units) 3,20,000 1,25,60,000 1,25,60,000

5.56

Page 351: 30510870 Cost Accounting and Financial Management

Non-Integrated Accounts

To Administration Overheads 7,00,000 By Gross Profit b/d 29,60,000 ” Selling and Distribution ” Dividend 1,00,000 Overheads 9,60,000 ” Interest 20,000 ” Bad Debts 80,000 ” Preliminary Expenses written off 40,000 ” Legal Charge 10,000 ” Net Profit 12,90,000 ________ 30,80,000 30,80,000

Statement of Cost and Profit

(As per Cost Records) Total Rs.

Direct Material 56,00,000

Direct Wages 30,00,000

Prime Cost 86,00,000

Factory Overhead 17,20,000

1,03,20,000

Less: Closing Stock (WIP) 2,40,000

Works Cost (1,24,000 units) 1,00,80,000

Administration overhead (1,24,000 units @ Rs. 6 p.u.) 7,44,000

Cost of production of (1,24,000 units) 1,08,24,000

Less: Finished Goods (4,000 units @ Rs. 87.29) 3,49,160

Cost of goods sold (1,20,000 units) 1,04,74,840

Selling and Distribution Overhead (1,20,000 @ Rs. 8 p.u.) 9,60,000

Cost of Sales 1,14,34,840

Net profit (Balancing figure) 5,65,160

Sales Revenue 1,20,00,000

5.57

Page 352: 30510870 Cost Accounting and Financial Management

Cost Accounting Statement of Reconciliation of profit as obtained under Cost and Financial Accounts Rs. Rs. Profit as per Cost Records 5,65,160 Add: Excess of Material Consumption 6,00,000 “ Factory Overhead 1,20,000 “ Administration Overhead 44,000 Dividend Received 1,00,000 Interest Received 20,000 8,84,000 14,49,160 Less: Bad debts 80,000 Preliminary expenses written off 40,000 Legal Charges 10,000 Over-valuation of stock in cost book (Rs.3,49,160 – Rs.3,20,000) 29,160 1,59,160 Profit as per Financial Records 12,90,000

Illustration: The following information is available from the financial books of a company having a normal production capacity of 60,000 units for the year ended 31st March, 2006: (i) Sales Rs. 10,00,000 (50,000 units).

(ii) There was no opening and closing stock of finished units. (iii) Direct material and direct wages cost were Rs. 5,00,000 and Rs. 2,50,000 respectively. (iv) Actual factory expenses were Rs. 1,50,000 of which 60% are fixed. (v) Actual administrative expenses were Rs. 45,000 which are completely fixed. (vi) Actual selling and distribution expenses were Rs. 30,000 of which 40% are fixed. (vii) Interest and dividends received Rs. 15,000. You are required to: (a) Find out profit as per financial books for the year ended 31st March,2006; (b) Prepare the cost sheet and ascertain the profit as per cost accounts for the year ended

31st March, 2006 assuming that the indirect expenses are absorbed on the basis of normal production capacity; and

(c) Prepare a statement reconciling profits shown by financial and cost books.

5.58

Page 353: 30510870 Cost Accounting and Financial Management

Non-Integrated Accounts

Solution: Working Note:

Profit & Loss Account (for the year ended 31st March, 2006)

Rs. Rs. To Direct material 5,00,000 By Sales 50,000 units 10,00,000 To Direct wages 2,50,000 By Interest and dividends 15,000 To Actual factory expenses 1,50,000 To Actual administrative expenses 45,000 To Actual selling and distribution expenses 30,000 To Profit 40,000 _______ 10,15,000 10,15,000 (a) Profit as per financial books for the year ended 31st March, 2006 is Rs. 40,000 (Refer to

above Working note). (b) Cost Sheet

(for the year ended 31st March, 2006) Rs. Direct material 5,00,000 Direct wages 2,50,000 Prime cost 7,50,000 Factory expenses: Variable : Rs. 60,000 _______ Fixed : Rs. 75,000 1,35,000 Works cost 8,85,000 Administrative expenses: 37,500 Cost of production 9,22,500 Selling & distribution expenses: Variable : Rs. 18,000 Fixed : Rs. 10,000 28,000 Cost of Sales 9,50,500 Profit 49,500 Sales revenue 10,00,000

5.59

Page 354: 30510870 Cost Accounting and Financial Management

Cost Accounting (c) Statement of Reconciliation

(Reconciling profit shown by Financial and Cost Accounts) Rs. Rs. Profit as per Cost Account 49,500 − Add : Income from interest and dividends 15,000 64,500 Less: Factory expenses under-charged in Cost Accounts 15,000 (Rs. 1,50,000 – Rs. 1,35,000) Administrative expenses under-charged in Cost Accounts 7,500 (Rs. 45,000 – Rs. 37,500) Selling & distribution expenses under—charged in Cost Accounts 2,000 24,500 (Rs. 30,000 – Rs. 28,000) Profit as per Financial Accounts 40,000

Illustration : M/s. H.K. Piano Company showed a net loss of Rs. 4,16,000 as per their financial accounts for the year ended 31st March, 2004. The cost accounts, however, disclosed a net loss of Rs.3,28,000 for the same period. The following information was revealed as a result of scrutiny of the figures of both the sets of books: Rs. (i) Factory overheads under-recovered 6,000 (ii) Administration overheads over-recovered 4,000 (iii) Depreciation charged in financial accounts 1,20,000 (iv) Depreciation recovered in costs 1,30,000 (v) Interest on investment not included in costs 20,000 (vi) Income-tax provided 1,20,000 (vii) Transfer fees (credit in financial books) 2,000 (viii) Stores adjustment (credit in financial books) 2,000 Prepare a Memorandum reconciliation account.

5.60

Page 355: 30510870 Cost Accounting and Financial Management

Non-Integrated Accounts

Solution: Memorandum Reconciliation Account

Dr. Cr. Particulars Rs. Particulars Rs. To Net loss as per costing books 3,28,000 By Administration overhead To Factory overheads over-recovered in costs 4,000 under-recovered in costs 6,000 By Interest on investments To Income-tax not provided in costs 1,20,000 not included in costs 20,000 By Depreciation overcharged in costs 10,000 By Transfer fees in financial books 2,000 By Stores adjustment 2,000 By Net loss as per financial _______ books 4,16,000 4,54,000 4,54,000

5.5 Self-examination questions

Multiple choice questions

1. Separate books of accounts are maintained for costing and financial accounting purposes under, (a) The inter locking system of accounting (b) The integrated system of accounting (c) Both a and b (d) None of the above

2. Under integrated system of accounting, purchase of raw material is debited to which account, (a) Purchase Account (b) Work in progress control account (c) Stores ledger control account/Raw material control account (d) Neither 1,nor 2 nor 3

3. Under integrated system of accounting, issue of raw material is debited to which account, (a) Purchase Account (b) Work in progress control account

5.61

Page 356: 30510870 Cost Accounting and Financial Management

Cost Accounting

(c) Stores ledger control account/Raw material control account (d) Neither 1,nor 2 nor 3

4. Notional costs (a) May be included in interlocking accounts (b) May be included in integrated accounts (c) Cannot be included in Interlocking accounts (d) Neither a nor b nor c

5. In a period Rs 50,000 was incurred on indirect labour. In a Cost Ledger, the double entry will be: Debit Credit (a) Wages control account Overhead control account (b) WIP control account Wages control account (c) Overhead control account Wages control account (d) Wages control account WIP control account

6. In an integrated accounting system the accounting entries for factory overhead absorbed would be: Debit Credit (a) WIP control account Overhead control account (b) Overhead control account WIP control account (c) Overhead control account Cost of sales account (d) Cost of sales account Overhead control account

7. At the end of a financial period, accounting entries for under absorbed overheads would be Debit Credit (a) WIP control account Overhead control account (b) Profit and loss account WIP control account (c) Profit and loss account Overhead control account (d) Overhead control account Profit and loss account

5.62

Page 357: 30510870 Cost Accounting and Financial Management

Non-Integrated Accounts

8. The double entry for factory cost of production in a cost ledger is, Debit Credit (a) Cost of sales account Finished goods control account (b) Finished goods control account WIP control account (c) Costing profit and loss account Finished goods control account (d) WIP control account Finished goods control account

9. In a non integrated system of accounting, the emphasis is on, a. Personal accounts b. real accounts c Nominal accounts d. All of these

10. Which of the following accounts makes the cost ledger self balancing, a. Overhead adjustment account b. Costing P & L account c. Cost ledger control account d. None of the above

Answers to multiple choice questions

1.a; 2.c; 3.b; 4.a; 5.c; 6.a; 7.c; 8.b; 9.c; 10.c;

Short answer type questions

1. What do you understand by Integrated Accounting System? State its advantages and pre-requisites.

2. Discuss the important cost control accounts maintained in a costing system.

3. “Reconciliation of costs and financial accounts in the modern computer age is redundant.” Comment.

Long answer type questions

1. Why is reconcilation of cost and financial accounts necessary? State the possible reasons for difference in profits shown by both the accounts.

2. The following trial balance results from entries in a cost ledger:

5.63

Page 358: 30510870 Cost Accounting and Financial Management

Cost Accounting

Rs. Rs.

(a) General Ledger Adjustment Account 1,15,900

(b) Stores Ledger Account 37,900

(c) Work-in-progress Account 54,300

(d) Finished Goods Account 23,500

(e) Factory Overheads Account 500

(f) Administration Expenses Account _______ 300

Total 1,16,200 1,16,200

Explain that each balance represents in each of the above transactions, and the traditions out of which it has arisen. Show (assumed) details of the Work-in-Progress Account.

3. The following balances are extracted from a company’s ledger as on 31st March. Dr. Cr. Rs. Rs. Raw Material Control Account 50,836 Work-in-progress Control Account 12,745 Finished Stock Control Account 25,980 Nominal Ledger Control Account ______ 89,561 89,561 89,561Further transaction took place during the following quarter as follows: Rs. Factory overhead-allocated to WIP 11,786 Goods finished at cost 36,834 Raw Materials purchased 22,422 Direct Wages — allocated to WIP 8,370 Raw materials — issued to production Cost of goods 41,389 Raw materials credited by supplier 836 Customer’s return (at cost) of goods the finished 2,856 Inventory audit — raw material losses 1,236

You are required to write up the four accounts in the cost ledger.

5.64

Page 359: 30510870 Cost Accounting and Financial Management

CHAPTER 6

METHOD OF COSTING (I) (JOB COSTING, CONTRACT COSTING, BATCH COSTING AND

OPERATING COSTING)

Learning objectives When you have finished studying this chapter, you should be able to ♦ Understand the meaning and distinctive features of Job, Batch, Operating and Contract

Costing.

♦ Understand the accounting procedures to be applied in the above-mentioned different methods of Costing.

6.1 INTRODUCTION Today business and industry needs costing systems to meet their individual requirements. Costing experts believe that it may not be possible to devise a single costing system to fulfill everybody’s needs. They have developed different methods of costing for different industries depending upon the type of manufacture and their nature. Mainly the industries can be grouped into two basic types: (1) Industries doing job work. (2) Industries engaged in mass production of a single product or identical production.

A concern engaged in the execution of specification order is characterised as a firm producing several items distinguishable from one another by respective specifications and other details. Such a concern is thought of involved in performing job works. Production under job work is strictly according to customer’s specifications and each lot, job or production order is unique. Examples of jobs order type of production are : ships building, roads, bridges, manufacture of heavy electrical machinery, machine tools, iron foundries, wood working shops, etc. Here each job or unit of production is treated as a separate identity for the purpose of costing. The methods of costing and for ascertaining cost of each job are known as a job costing. Contract costing and Batch costing.

The continuous or process type of industry is characterised by the continuous production of uniform products according to standard specifications. In such a case the successive

Page 360: 30510870 Cost Accounting and Financial Management

Cost Accounting lots are generally indistinguishable as to size and form and, even if there is some variation in specifications, it is of a minor character. Examples of continuous type of industries are chemical and pharmaceutical products, paper/food products, canning, paints, and varnish oil, rubber, textile etc. Here the methods of Costing used for the purpose of ascertaining costs are: process costing; single costing; operating costing etc.

6.2 JOB COSTING According to this method costs are collected and accumulated according to jobs, contracts, products or work orders. Each job or unit of production is treated as a separate entity for the purpose of costing. Job costing is carried out for the purpose of ascertaining cost of each job and takes into account the cost of materials, labour and overhead etc. The job costing method is also applicable to industries in which production is in batches since batch production basically is of the same character as the job order production, the difference being mainly one in the size of different orders. The method then may also be described as “Batch Costing”. The job costing method of costing may be regarded as the principal method of costing since the basic object and purpose of all costing is to analyse and ascertain cost of each unit of production so that it may be possible to control and regulate cost and to determine the profitability or otherwise of each work order or product line. The basic principles enunciated for the job costing method are, therefore, valid essentially for all types of industry. For example, printing; furniture; hardware; ship-building; heavy machinery; interior decoration, repairs and other similar work. The job costing method essentially involves preparation of a separate cost sheet for each job, disclosing cost of material issued for the job, labour charges incurred (on the basis of bill of material and time cards respectively); when the job is completed, overhead charges are added for ascertaining total expenditure (See carefully the entries made in the specimen page) Job (Account on page alongside the figures are assumed). Job Costing may be employed in the following cases: - When jobs are executed for different customers according to their specifications.

- When no two orders are alike and each order/job needs special treatment.

- Where the work-in-progress differs from period to period on the basis of the number of jobs in hand.

6.2

Page 361: 30510870 Cost Accounting and Financial Management

JOB (OR WORK) ACCOUNT Job No. ———K 576 Name of party : M/s. Slow and Steady Brief Description ———— 10 Special Steel Cupboards Due for completion : August 25, 2005 Commenced on July 10, 2005 —— Completed on : August 19, 2005 MATERIAL LABOUR DIRECT EXPENSES TOTAL

Date Req.Slip Rs. Date Wages Abs- Rs. Date Ref. Rs. Description Rs. No. tract No. 2005 2005 Cash Materials 1,490.00 10/1 275 510.00 15/7 23 315.00 16/8 Voucher 150 31/7 310 530.00 22/7 24 430.00 320 Labour 2,050.00 Direct Expenses 150.00 12/8 405 405.00 29/7 25 240.00 5/8 26 410.00 Prime Cost 3,690.00 Factory Expenses* 1,025.00 12/8 27 340.00 Works Cost 4,715.00 19/8 28 315.00 Admn. Expenses** 943.00 Cost of Total 1,490.00 Total 2,050.00 Total 150 Production 5,658.00

Prepared by ——— Chadha Checked by——— Murthy (If necessary, space may also be provided for including an amount for selling and distribution expenses. A column to record the number of hours can also be provided under the main head.) *Say, at 50% of labour. **Say, at 20% of work cost.

Page 362: 30510870 Cost Accounting and Financial Management

Cost Accounting 6.2.1 Procedure of job Cost Accounting

Accounting for Materials : An essential requirement of job cost accounting is that direct materials and their cost must be traced to and identified with specific job or work order. This segregation of materials cost by jobs or work orders is brought about by the use of separate stores requisitions for each job or work order. Where a bill of material is prepared, it provides the basis for the preparation of these stores requisitions. But when the entire quantity of materials specified in the bill of materials is drawn in one lot or in instalments, the bill itself could be made to serve as a substitute for the stores requisition.

After the materials have been issued and the stores requisitions have been priced, it is usual to enter the value of the stores requisition in a material abstract or analysis book. It serves to analyse and collect the cost of all direct materials according to job or work orders and departmental standing orders or expense code numbers.

From the abstract book, the summary of materials cost of each job is posted to individual job cost sheets or cards in the Work-in-Progress ledger. The postings are usually made weekly or monthly. Similarly, at periodical intervals, from the material abstract books, summary cost of indirect material is posted to different standing orders or expense code numbers in the Overhead Expenses ledger. If any special material has been purchased for a particular job, it is generally the practice to charge such special material direct to the job concerned without passing it through the Stores Ledger, as soon as it is purchased.

If any surplus material is left over in the case of any job, unless it can be immediately and economically used on some other job, the same is returned to the store room with a proper supporting document/stores Debit Note or Shop Credit, and the relevant job account is credited with the value of excess material returned to the store room.

If the surplus material is utilised on some other job, instead of being returned to the store room first, a material transfer note is prepared. The transfer note would show the number of the transfer to job as well as transferee job (or jobs) so that, on that basis, the cost thereof can be adjusted in the Work-in-Progress Ledger.

Accounting for Labour : All direct labour cost must be analysed according to individual jobs or work orders. Similarly, different types of indirect labour cost also must be collected and accumulated under appropriate standing order or expenses code number.

The analysis of labour according to jobs or work orders is, usually, made by means of job time cards or sheets. All direct labour is booked against specific jobs in the job time cards or sheets. All the idle time also is booked against appropriate standing order expense code number either in the job time card for each job or on a separate idle time card for

6.4

Page 363: 30510870 Cost Accounting and Financial Management

Method of Costing (I)

each worker (where the job time card is issued job-wise). The time booked or recorded in the job time and idle time cards is valued at appropriate rates and entered in the labour abstract or analysis book. All direct labour cost is accumulated under relevant job or work order numbers, and the total or the periodical total of each job or work order is then posted to the appropriate job cost card or sheet in Work-in-Progress ledger. The postings are usually made at the end of each week or month.

The abstraction of idle time costs under suitable standing order or expenses code numbers is likewise done and the amounts are posted to the relevant departmental standing order or expense code number in the Overhead Expenses Ledger at periodical intervals. As regards other items of indirect labour cost these are collected from the payrolls books for the purpose of posting against standing order or expenses code numbers in the Overhead Expenses ledger.

Accounting for Overhead : Manufacturing overheads are collected under suitable standing order numbers and selling and distribution overheads against cost accounts numbers. Total overhead expenses so collected are apportioned to service and production depart-ments on some suitable basis. The expenses of service departments are finally transferred to production departments. The total overhead of production departments is then applied to products on some realistic basis, e.g. machine hour; labour hour; percentage of direct wages; percentage of direct materials; etc. It should be remembered that the use of different methods will lead to a different amounts being computed for the works overhead charged to a job hence to different total cost.

The problem of accurately absorbing, in each individual job or work order, the overhead cost of different cost centres or departments involved in the manufacture is difficult under the job costing method. It is because the cost or the expenses thereof cannot be traced to or identified with any particular job or work order. In such circumstances, the best that can be done is to apply a suitable overhead rate to each individual article manufactured or to each production order. This is essentially an arbitrary method.

Price of a job : Price of a job may be arrived by adding the desired percentage of profit to the total cost of the job.

Treatment of spoiled and defective work : Spoiled work is the quantity of production that has been totally rejected and cannot be rectified. Defective work on the other hand refers to production that is not as perfect as the saleable product but is capable of being rectified and brought to the required degree of perfection provided some additional expenditure is incurred.

6.5

Page 364: 30510870 Cost Accounting and Financial Management

Cost Accounting Normally, all the manufacturing operations are not fully successful; they result in turning out a certain amount of defective work. Nonetheless, over a period of time it is possible to work out a normal rate of defectives for each manufacturing process which would represent the number of defective articles which a process shall produce in spite of due care. Defects arise in the following circumstances :

(1) Where a percentage of defective work is allowed in a particular batch as it cannot be avoided.

(2) Where defect is due to bad workmanship.

(3) Where defect is due to the Inspection Department wrongly accepting incoming material of poor quality.

(1) In the first case, when a normal rate of defectives has already been established, if the actual number of defectives is within the normal limit or is near thereto the cost of rectification will be charged to the whole job and spread over the entire output of the batch. If, on the other hand, the number of defective units substantially exceeds the normal, the cost of rectification of the number which exceeds the normal will be written off as a loss in the Costing Profit and Loss Account.

(2) In the second case, when the defective work is due to bad workmanship the cost of rectification will be abnormal cost, i.e., not a legitimate element of the cost. Therefore, the cost of rectification shall be written off as a loss, unless by an ar-rangement, it is to be recovered as a penalty from the workman concerned. It is possible, however that the management did provide for a certain proportion of defectives on account of bad workmanship as an unavoidable feature of production. If that be the case, the cost of rectifying to the extent provided for by the management will be treated as a normal cost and charged to the batch.

(3) In the third case the defect being due to negligence of the Inspection Department, the cost of rectification will be charged to the department and will not be considered as cost of manufacture of the batch. Being an abnormal cost, it will be written off to the Costing Profit and Loss Account.

Illustration

The manufacturing cost of a work order is Rs. 1,000; 8% of the production against that order spoiled and the rejection is estimated to have a realisable value of Rs. 20 only. The normal rate of spoilage is 2%. Record this in the costing journal.

Solution

Actual loss is Rs. 60, i.e. Rs. 80 less Rs. 20 recoverable as materials. Of this net loss of

6.6

Page 365: 30510870 Cost Accounting and Financial Management

Method of Costing (I)

Rs. 15 is normal; Rs. 45 is the abnormal loss to be debited to the Costing Profit and Loss Account. The accounting entries necessary for recording the above facts would be : Rs. Rs. Rs. Materials Control Account Dr. 20 Overhead Control Account Dr. 15 Costing Profit and Loss Control Account Dr. 45 To Work-in-Progress Control Account 80 In the case of defectives being inherent in the manufacturing process, the rectification cost may be charged to the specific jobs in which they have arisen. In case detectives cannot be identified with jobs, the cost of rectification may be treated as factory overheads. Abnormal defectives should be written off to the Costing Profit and Loss Account.

Illustration

A shop floor supervisor of a small factory presented the following cost for Job No. 303, to determine the selling price. Per Unit Rs. Materials 70 Direct wages 18 hours @ Rs. 2.50 45 (Deptt. X 8 hours ; Deptt. Y 6 hours; Deptt. Z 4 hours) Chargeable expenses 5 120 Add : 33-1/3 % for expenses cost 40 160

Analysis of the Profit/Loss Account (for the year 2005)

Rs. Rs. Materials used 1,50,000 Sales less returns 2,50,000 Direct wages: Deptt. X 10,000 Deptt. Y 12,000 Deptt. Z 8,000 30,000

6.7

Page 366: 30510870 Cost Accounting and Financial Management

Cost Accounting Special stores items 4,000 Overheads : Deptt. X 5,000 Deptt. Y 9,000 ______ Deptt. Z 2,000 16,000 Works cost 2,00,000 Gross profit c/d 50,000 _______ 2,50,000 2,50,000 Selling expenses 20,000 Gross profit b/d 50,000 Net profit 30,000 ______ 50,000 50,000It is also noted that average hourly rates for the three Departments X, Y and Z are similar. You are required to: Draw up a job cost sheet. Calculate the entire revised cost using 2005 actual figures as basis. Add 20% to total cost to determine selling price.

Solution Job Cost Sheet

Customer Details ——— Job No._________________ Date of commencement —— Date of completion _________ Particulars Amount Rs. Direct materials 70 Direct wages : Deptt. X Rs. 2.50 × 8 hrs. = Rs. 20.00 Deptt. Y Rs. 2.50 × 6 hrs. = Rs. 15.00 Deptt. Z Rs. 2.50 × 4 hrs. = Rs. 10.00 45 Chargeable expenses 5 Prime cost 120 Overheads :

6.8

Page 367: 30510870 Cost Accounting and Financial Management

Method of Costing (I)

Deptt. X = 000,10.Rs000,5.Rs × 100 = 50% of Rs. 20 = Rs. 10.00

Deptt. Y = 000,12.Rs000,9.Rs × 100 = 75% of Rs. 15 = Rs. 11.25

Deptt. Z = 000,8.Rs000,2.Rs × 100 = 25% of Rs. 10 = Rs. 2.50 23.75

Works cost 143.75

Selling expenses = 000,00,2.Rs

000,20.Rs × 100 = 10% of work cost 14.38

Total cost 158.13 Profit (20% of total cost) 31.63Selling price 189.76

6.3 CONTRACT COSTING

6.3.1 Introduction : A contract usually takes several years to get itself completed. If the profit on such contracts is recorded only after their completion, then wide fluctuations may be noted in the profit figures of contractors from year to year. To avoid these fluctuations in the reported profits and to reflect the revenue in the accounting period during which the activity is undertaken, the profit in respect of each contract in progress is transferred to the profit and loss account of the year by calculating the notional profit. The portion of notional profit to be transferred to the profit and loss account depends on the stage of completion of a contract. To determine such a profit figure the knowledge of various concepts as discussed below is essential in contract costing.

6.3.2 Meaning of Contract Costing : Contract or terminal costing, as it is termed, is one form of application of the principles of job costing. In fact a bigger job is referred to as a contract. Contract costing is usually adopted by building contractors engaged in the task of executing Civil Contracts. Contract costing have the following distinct features :

1. The major part of the work in connection with each contract is ordinarily carried out at the site of the contract.

2. The bulk of the expenses incurred by the contractor are considered as direct.

3. The indirect expenses, mostly consist of office expenses of the yards, stores and works.

6.9

Page 368: 30510870 Cost Accounting and Financial Management

Cost Accounting 4. A separate account is usually maintained for each contract.

5. The number of contracts undertaken by a contractor at a time is not usually very large.

6. The cost unit in contract costing is the contract itself.

6.3.3 Recording of contract costs : Material Cost : All materials supplied from the stores or purchased directly for the contract are debited to the concerned contract account. In the case of transfer of excess material from one contract to other contract, their costs would be adjusted on the basis of material transfer note, signed both by the transferee and the transferor foreman. In case the return of surplus material appears uneconomical on account of high cost of transportation, the same is sold and the concerned contract account is credited with the sale price. Any loss or profit arising there-from is transferred to the Profit and Loss Account. Any theft, or destruction of material by fire represent a loss and as such, the same is transferred to the Profit and Loss Account. If any stores items are used for manufacturing tools, the cost of such stores items are charged to the work expenses account. If the contractee has supplied some materials without affecting the contract price, no accounting entries will be made in the contract account, only a note may be given about it.

Labour Cost : Labour actually employed on the site of the contract is regarded as direct (irrespective of the nature of the task performed) and the wages paid to them are charged to the concerned contract directly or on the basis of a wage analysis sheet (if concurrently a number of contracts are carried on and labourers are required to devote their time on two or more contracts).

Direct Expenses : Direct expenses (if any) are directly charged to the concerned contract.

Indirect Expenses : Indirect expenses (such as expenses of engineers, surveyors, supervisors etc.) may be distributed over several contracts as a percentage of cost of materials, or wages paid or of the prime cost. If however, the contracts are big, the labour hour method may be used for the distribution of expenses.

Plant and Machinery : The value of the plant in a contract may be either debited to contract account and the written down value thereof at the end of the year entered on the credit side for closing the contract account, or only a charge (depreciation) for use of the plant may be debited to the contract account.

Sub-Contract : Sub-contract costs are also debited to the Contract Account.

Extra work : The extra work amount payable by the contractee should be added to the contract price. If extra work is substantial, it is better to treat it as a separate contract. If it

6.10

Page 369: 30510870 Cost Accounting and Financial Management

Method of Costing (I)

is not substantial, expenses incurred should be debited to the contract account as “Cost of Extra work”.

Cost of work certified : All building contractors received payments periodically known as “running payment” on the basis of the architect’s or surveyor’s certificates. But payments are not equal to the value of the work certified, a small percentage of the amount due is retained as security for any defective work which may be discovered later within the guarantee period.

Mathematically :

Cost of work certified = Cost of work to date – (Cost of work uncertified + Material in hand + Plant at site)

The amount retained is called retention money. The full value of the work certified should be credited to the Contract Account and debited to the account of the contract. Since the cash received from him will be less, the balance in his account will be shown as an asset in the balance sheet.

Work uncertified : It represents the cost of the work which has been carried out by the contractor but has not been certified by the contractee’s architect. It is always shown at cost price. The cost of uncertified work may be ascertained as follows : Rs. Total cost to date — Less: Cost of work certified — Material in hand — Plant at site — — Cost of work uncertified — Retention money : A contractor does not receive full payment of the work certified by the surveyor. Contractee retains some amount (say 10% to 20%) to be paid, after sometime, when it is ensured that there is no fault in the work carried out by contractor. If any deficiency or defect is noticed in the work, it is to be rectified by the contractor before the release of the retention money. Retention money provides a safeguard against the risk of loss due to faulty workmanship.

Cash received : It is ascertained by deducting the retention money from the value of work certified i.e., Cash received = Value of work certified – Retention money.

6.11

Page 370: 30510870 Cost Accounting and Financial Management

Cost Accounting Work-in-progress: In Contract Accounts, the value of the work-in-progress consists of (i) the cost of work completed, both certified and uncertified; (ii) the cost of work not yet completed; and (iii) the amount of profit taken as credit. In the Balance Sheet, the work-in-progress is usually shown under two heads, viz., certified and uncertified. The cost of work completed and certified and the profit credited will appear under the head ‘certified’ work-in-progress, while the completed work not yet certified and the cost of labour, material and expenses of work which has not yet reached the stage of completion are shown under the head “uncertified” work-in-progress.

Notional profit : It represents the difference between the value of work certified and cost of work certified. It is determined :

Notional profit = Value of work certified – (Cost of work to date – Cost of work not yet certified)

Estimated profit : It is the excess of the contract price over the estimated total cost of the contract.

6.3.4 Profit/loss on incomplete contracts : To determine the profit to be taken to Profit and Loss Account, in the case of incomplete contracts, the following four situations may arise:

(i) Completion of contract is less than 25 per cent: In this case no profit should be taken to profit and loss account.

(ii) Completion of contract is upto 25 per cent or more than 25 per cent but less than 50 per cent: In this case one-third of the notional profit, reduced in the ratio of cash received to work certified, should be transferred to the Profit and Loss Account. Mathematically:

received Workreceived Cash×Profit Notional×

31

(iii) Completion of contract is upto 50 per cent or more than 50 per cent but less than 90 per cent: In this case, two-third of the notional profit, reduced by proportion of cash received to work certified, is transferred to the Profit and Loss Account. Mathematically :

received Workreceived Cash×Profit Notional×

32

(iv) Completion of contract is upto 90 per cent or more than 90 per cent i.e. it is nearing completion: In this case the profit to be taken to Profit and Loss Account is determined by determining the estimated Profit and using any one of the following formulas :

6.12

Page 371: 30510870 Cost Accounting and Financial Management

Method of Costing (I)

(a) Estimated Profit × priceContract

certified Work

(b) Estimated Profit × priceContract

certified Work ×certified Workreceived Cash

OR

Estimated Profit × priceContract

received Cash

(c) Estimated Profit × cost total Estimateddate to workofCost

(d) Estimated Profit × cost total Estimateddate to workofCost ×

certified Workreceived Cash

(e) Notional Profit × priceContract

certified Work

(This formula may be preferably used in the absence of estimated profit figure). It is preferable to use formula (b) in the absence of specific instructions.

6.3.5 Cost plus Contract : Under Cost plus Contract, the contract price is ascertained by adding a percentage of profit to the total cost of the work. Such type of contracts are entered into when it is not possible to estimate the Contract Cost with reasonable accu-racy due to unstable condition of material, labour services, etc.

Cost plus contracts have the following advantages and disadvantages :

Advantages :

(i) The Contractor is assured of a fixed percentage of profit. There is no risk of incurring any loss on the contract.

(ii) It is useful specially when the work to be done is not definitely fixed at the time of making the estimate.

(iii) Contractee can ensure himself about ‘the cost of the contract’, as he is empowered to examine the books and documents of the contractor to ascertain the veracity of the cost of the contract.

6.13

Page 372: 30510870 Cost Accounting and Financial Management

Cost Accounting Disadvantages - The contractor may not have any inducement to avoid wastages and effect economy in production to reduce cost.

Escalation Clause - If during the period of execution of a contract, the prices of materials, or labour etc., rise beyond a certain limit, the contract price will be increased by an agreed amount. Inclusion of such a clause in a contract deed is called an “Escalation Clause”.

Illustration The following expenses were incurred on a contract : Rs. Material purchased 6,00,000 Material drawn from stores 1,00,000 Wages 2,25,000 Plant issued 75,000 Chargeable expenses 75,000 Apportioned indirect expenses 25,000

The contract was for Rs. 20,00,000 and it commenced on January 1, 2005. The value of the work completed and certified upto 30th November, 2005 was Rs. 13,00,000 of which Rs. 10,40,000 was received in cash, the balance being held back as retention money by the contractee. The value of work completed subsequent to the architect’s certificate but before 31st December, 2005 was Rs. 60,000. There were also lying on the site materials of the value of Rs. 40,000. It was estimated that the value of plant as at 31st December, 2005 was Rs. 30,000. Solution : Dr. Contract Account Cr. Rs. Rs. To Material purchased 6,00,000 By Work-in-progress : ” Stores issued 1,00,000 Work certified 13,00,000 ” Wages 2,25,000 Work uncertified 60,000 ” Plant 75,000 Material unused 40,000 ” Chargeable expenses 75,000 Plant less depreciation 30,000 ” Indirect expenses 25,000 ” Profit and Loss Account, 2/3rds of profit on cash basis 1,76,000*

6.14

Page 373: 30510870 Cost Accounting and Financial Management

Method of Costing (I)

” Work-in-progress balance of profit c/d 1,54,000 ________ 14,30,000 14,30,000” Balance b/d: Work certified 13,00,000 Uncertified 60,000 Material at site 40,000 Plant at site 30,000 14,30,000 Less: Reserve 1,54,000 12,76,000 *Computation of Profit : Rs. Apparent profit 3,30,000 2/3rd of that since 65% of the work is complete 2,20,000 80% of that on cash basis 1,76,000 An alternative method of presentation can be to deduct the balance of profit to be carried down (Rs. 1,54,000 in the above case) from the work certified before it is entered in the contract account. It will be Rs. 11,46,000 in the illustration given above. Of course, the reserve to be so deducted from the work certified will have to be first ascertained by considering the value of the work certified.

Illustration

A contractor prepares his accounts for the year ending 31st December each year. He commenced a contract on 1st April, 2005. The following information relates to the contract as on 31st December, 2005 : Rs. Material issued 2,51,000 Labour charges 5,65,600 Salary to Foreman 81,300 A machine costing Rs. 2,60,000 has been on the site for 146 days, its working life is estimated at 7 years and its final scrap value at Rs. 15,000.

A supervisor, who is paid Rs. 8,000 p.m. has devoted one-half of his time to this contract.

All other expenses and administration charges amount to Rs. 1,36,500.

Material in hand at site costs Rs. 35,400 on 31st December, 2005.

6.15

Page 374: 30510870 Cost Accounting and Financial Management

Cost Accounting The contract price is Rs. 20,00,000. On 31st December, 2005 two-third of the contract was completed. The architect issued certificates covering 50% of the contract price, and the contractor had been paid Rs. 7,50,000 on account.

Prepare Contract A/c and show how much profit or loss should be included in financial accounts to 31st December, 2005.

Solution Dr. Contract Account Cr. Rs. Rs. To Material issued 2,51,000 By Machine 2,46,000 ” Labour charges 5,65,600 (See note 1) ” Foreman salary 81,300 By Material (in hand) 35,400 ” Machine 2,60,000 By Works cost 10,49,000 ” Supervisor’s salary 36,000 (Rs. 8,000 × 9)/2 ” Adm. charges 1,36,500 ________ 13,30,400 13,30,400 To Works cost 10,49,000 By Work certified 10,00,000 To Notional profit 2,13,250 By Work uncertified 2,62,250 ________ (See Note 2) ________ 12,62,250 12,62,250To Profit & Loss A/c 1,06,625 By Notional Profit 2,13,250 To Work-in-Progress 1,06,625 ________ 2,13,250 2,13,250Notes : 1. Machine :

[(Rs. 2,60,000 – Rs. 15,000) ÷ 7] × 365146 = Rs. 14,000

Hence the value of machine after the period of 146 days is Rs. 2,60,000 – Rs. 14,000 = Rs. 2,46,000

6.16

Page 375: 30510870 Cost Accounting and Financial Management

Method of Costing (I)

2. The cost of 66.67% of the contract is Rs. 10,49,000

∴Cost of 100% " " " " 67.66

000,49,10.Rs × 100 = Rs. 15,73,500

∴Cost of 50% of the contract which has been certified by the architect is Rs. 7,86,750. Also the cost of 16.67% of the contract, which has been completed but not certified by the archi-tect is Rs. 2,62,250.

Illustration

M/s. Bansals Construction Company Ltd. took a contract for Rs. 60,00,000 expected to be completed in three years. The following particulars relating to the contract are available:

2004 2005 2006 Rs. Rs. Rs. Materials 6,75,000 10,50,000 9,00,000 Wages 6,20,000 9,00,000 7,50,000 Cartage 30,000 90,000 75,000 Other expenses 30,000 75,000 24,000 Cumulative work certified 13,50,000 45,00,000 60,00,000 Cumulative work uncertified 15,000 75,000 — Plant costing Rs. 3,00,000 was bought at the commencement of the contract. Depreciation was to be charged at 25% per annum, on the written down value method. The contractee pays 75% of the value of work certified as and when certified, and makes the final payment on completion of the contract.

You are required to make a contract account and contractee account as they would appear in each of the three years. Also show how the work-in-progress and other items should appear in the balance sheet.

Solution Dr. Contract Account Cr. Rs. Rs. 2004 2004 To Materials 6,75,000 By Plant at site c/d 2,25,000 To Wages 6,20,000 By work-in-progress c/d : Rs.

6.17

Page 376: 30510870 Cost Accounting and Financial Management

Cost Accounting To Cartage 30,000 Work certified 13,50,000 To Other expenses 30,000 Work uncertified 15,000 13,65,000 To Plant 3,00,000 By Profit & Loss A/c 65,000 ________ (Loss transferred) ________ 16,55,000 16,55,000

2005 Rs. 2005 Rs. To Work-in-progress b/d : By Work-in-progress c/d : Work certified 13,50,000 Work certified 45,00,000 Work uncertified 15,000 13,65,000 Work uncertified 75,000 45,75,000 To Plant b/d 2,25,000 By Plant at site c/d 1,68,750 To Materials 10,50,000 To Wages 9,00,000 To Cartage 90,000 To Other expenses 75,000 To Notional profit c/d 10,38,750 47,43,750 47,43,750 To Profit & loss A/c 5,19,375 By Notional profit b/d 10,38,750 To Work-in-progress c/d 5,19,375 (profit in reserve) (Refer to working note 2) ________ ________ 10,38,750 10,38,750 2006 Rs. 2006 Rs. To Work-in-progress b/d: By Work-in-progress b/d 5,19,375 Work certified 45,00,000 (profit in reserve) Work uncertified 75,000 45,75,000 By Plant at site 1,26,562 By Contractee’s A/c 60,00,000 (contract price) To Plant b/d 1,68,750 To Materials 9,00,000 To Wages 7,50,000 To Cartage 75,000

6.18

Page 377: 30510870 Cost Accounting and Financial Management

Method of Costing (I)

To Other expenses 24,000 To Profit & loss A/c 1,53,187 ________ 66,45,937 66,45,937 Working Notes : 1. In 2004 there is a loss, and so the whole of it will be transferred to the profit and loss

account. 2. In 2005, the contract is 3/4th complete. Hence, the profit to be transferred to the profit

and loss account will be determined as under :

= received Workreceived Cash×Profit Notional×

32

= 45,00,000

00Rs.33,75,0×32 = Rs. 5,19,375

Contractee’s account

2004 Rs. 2004 Rs. To Balance c/d 10,12,500 By Bank 10,12,500 2005 2005 To Balance c/d 33,75,000 By Balance b/d 10,12,500 ________ By Bank 23,62,500* 33,75,000 33,75,000 2006 2006 To Contract A/c 60,00,000 By Balance b/d 33,75,000 (Contract price) ________ By Bank 26,25,000 60,00,000 60,00,000*The total value of work certified at the end of 2005 was Rs. 45,00,000 of that worth Rs. 13,50,000 was certified in 2004. Hence, the cash to be received in 2005 is 75% of Rs. 31,50,000 (Rs. 45,00,000 − Rs. 13,50,000) i.e. Rs. 23,62,500.

Balance sheet (Extract) 2004

Liabilities Rs. Assets Rs. Capital − Plant at site 2,25,000 Less : Loss during the year 65,000 Work-in-progress : Rs. Work certified 13,50,000

6.19

Page 378: 30510870 Cost Accounting and Financial Management

Cost Accounting Work uncertified 15,000 13,65,000 Less: Cash received 10,12,500 3,52,500

Balance sheet (Extract) 2005 Liabilities Rs. Assets Rs. Capital − Plant at site 1,68,750 Add: Profit during the year 5,19,375 Work-in-progress : Rs. Work certified 45,00,000 Work uncertified 75,000 45,75,000 Less: Profit in reserve 5,19,375 40,55,625 Less Cash received 33,75,000 6,80,625

Balance sheet (Extract) 2006 Liabilities Rs. Assets Rs. Capital − Plant at site 1,26,562 Add: Profit during the year 1,53,157

Illustration

Compute a conservative estimate of profit on a contract (which has been 90% complete) from the following particulars. Calculate the proportion of profit to be taken to Profit & Loss Account under various methods and give your recommendation.

Rs. Total expenditure to date 4,50,000 Estimated further expenditure to complete the contract (including contingencies) 25,000 Contract price 6,12,000 Work certified 5,50,800 Work uncertified 34,000 Cash received 4,40,640

6.20

Page 379: 30510870 Cost Accounting and Financial Management

Method of Costing (I)

Solution Computation of notional profit Rs. Value of work certified 5,50,800 Less: Cost of work certified (Rs. 4,50,000 – Rs. 34,000) 4,16,000Notional profit 1,34,800Computation of estimated profit Rs. Contract price 6,12,000 Less: Cost of work to date 4,50,000 Estimated further expenditure to complete the contract 25,000Estimated total cost 4,75,000Estimated profit 1,37,000

Profit to be transferred under various methods

(i) Notional profit × priceContract

certified Work

= Rs. 1,34,800 × 0Rs.6,12,000Rs.5,50,80 = Rs. 1,21,320

(ii) Estimated profit × priceContract

certified Work

= Rs. 1,37,000 × 0Rs.6,12,000Rs.5,50,80 = Rs. 1,23,300

(iii) Estimated profit × priceContract certified Work

× certified Workreceived Cash

= Rs. 1,37,000 × 0Rs.6,12,000Rs.5,50,80

× 0Rs.5,50,800Rs.4,40,64

= Rs. 98,640

(iv) Estimated profit × cost total Estimateddate workofCost

= Rs. 1,37,000 × 0Rs.4,75,000Rs.4,50,00

= Rs. 1,29,790

6.21

Page 380: 30510870 Cost Accounting and Financial Management

Cost Accounting

(v) Estimated profit × cost total Estimateddate workofCost

× certified Workreceived Cash

= Rs. 1,37,000 × 0Rs.4,75,000Rs.4,50,00

× 0Rs.5,50,800Rs.4,40,64

= Rs. 1,03,832

Recommendation : It is recommended that a sum of Rs. 98,640 may be transferred to the profit and loss account. This amount is the least and has been arrived by using the formula (iii) above. According to this formula, profit transferred to the profit and loss account is generally kept the minimum and allows withholding in reserve a larger portion of notional profit to meet future unforeseen expenses and contingencies.

Illustration

A contractor has entered into a long term contract at an agreed price of Rs. 1,75,000 subject to an escalation clause for materials and wages as spelt out in the contract and corresponding actuals are as follows : Standard Actual Materials Qty (tonnes) Rate (Rs.) Qty (tonnes) Rate (Rs.) A 5,000 5 5,050 4.80 B 3,500 8 3,450 7.90 C 2,500 6 2,600 6.60 Labou r Hours Hourly Hours Hourly

Rate (Rs.) Rate (Rs.) X 2,000 7.00 2,100 7.20 Y 2,500 7.50 2,450 7.50 Z 3,000 6.50 3,100 6.60

Reckoning the full actual consumption of material and wages the company has claimed a final price of Rs. 1,77,360. Give your analysis of admissible escalation claim and indicate the final price payable.

6.22

Page 381: 30510870 Cost Accounting and Financial Management

Method of Costing (I)

Solution

Statement showing final claim Standard Standard Actual Rate Variation in Escalation Qty/Hrs. Rate (Rs.) (Rs.) Rate (Rs.) Claim (Rs.) Materials (a) (b) (c) (d) = (c)–(b) (e) =(a) × (d) A 5,000 5.00 4.80 (–) 0.20 (–) 1,000 B 3,500 8.00 7.90 (–) 0.10 (–) 350 C 2,500 6.00 6.60 (+) 0.60 1,500 Materials escalation claim : (P) 150Labour X 2,000 7.00 7.20 (+) 0.20 400 Y 2,500 7.50 7.50 − − Z 3,000 6.50 6.60 (+) 0.10 300 Wages escalation claim : (Q) 700 Final claim: (P) + (Q) 850

Statement showing final price payable Agreed price Rs. 1,75,000 Agreed escalation : Material cost Rs. 150 Labour cost Rs. 700 Rs. 850 Final price payable Rs. 1,75,850 The claim of Rs. 1,77,360 is based on the total increase in cost. This can be verified as shown below:

Statement showing total increase in cost Standard Cost Actual Cost Increase/

Qty/hrs Rate Amount Qty/hrs Rate Amount (Decrease) (Rs.) (Rs.) (Rs.) (Rs.) (a) (b) (c) = (a) × (b) (d) (e) (f) =(d) × (e) (g)=(f) – (c)

I. Materials A 5,000 5.00 25,000 5,050 4.80 24,240 (760) B 3,500 8.00 28,000 3,450 7.90 27,255 (745) C 2,500 6.00 15,000 2,600 6.60 17,160 2,160

68,000 68,655 655

6.23

Page 382: 30510870 Cost Accounting and Financial Management

Cost Accounting II. Labour

X 2,000 7.00 14,000 2,100 7.20 15,120 1,120 Y 2,500 7.50 18,750 2,450 7.50 18,375 (375) Z 3,000 6.50 19,500 3,100 6.60 20,460 960 52,250 53,955 1,705

Total (I + II) 2,360 The final price claimed by the company is contract price Rs. 1,75,000 Add : Increase in cost Rs. 2,360 Rs. 1,77,360

This claim is not admissible because escalation clause covers only that part of increase in cost, which has been caused by inflation.

Note : It is fundamental principle that the contractee would compensate the contractor for the increase in costs which are caused by factors beyond the control of contractor and not for increase in costs which are caused due to inefficiency or wrong estimation.

Illustration :

A contractor commenced a building contract on October 1, 2004. The contract price is Rs. 4,40,000. The following data pertaining to the contract for the year 2005-2006 has been compiled from his books and is as under :

Rs. April 1, 2005 Work-in-progress not certified 55,000 Materials at site 2,000 2005–06 Expenses incurred : Materials issued 1,12,000 Wages paid 1,08,000 Hire of plant 20,000 Other expenses 34,000 March 31, 2006 Materials at site 4,000 Work-in-progress : Not certified 8,000 Work-in-progress : Certified 4,05,000

The cash received represents 80% of work certified. It has been estimated that further costs to complete the contract will be Rs. 23,000 including the materials at site as on

6.24

Page 383: 30510870 Cost Accounting and Financial Management

Method of Costing (I)

March 31, 2006.

Required :

Determine the profit on the contract for the year 2005-06 on prudent basis, which has to be credited to P/L A/c.

Solution Contract Account for the year 2005-06 Dr. Cr. Particulars (Rs.) Particulars (Rs.) 1.4.05 To Work-in-progress 55,000 By Materials at site 4,000 (not certified) To Materials at site 2,000 2005–06 To Materials issued 1,12,000 By Cost of contract c/d (to date) 3,27,000 To Wages paid 1,08,000 To Hire of plant 20,000 To Other expenses 34,000 _______ 3,31,000 3,31,000 31.3.06 To Cost of contract b/d 3,27,000 By Work-certified 4,05,000 (to date) By Work-not certified 8,000 To Profit & loss A/c 66,273 To Profit in reserve 19,727 _______ 4,13,000 4,13,000 Profit for the year 2005-06 = Rs. 4,13,000 – Rs. 3,27,000 = Rs. 86,000 Estimated profit (on the completion of the contract) Rs. Cost of the contract (to date) 3,27,000 Further cost of completing the contract 23,000 Total cost: (A) 3,50,000 Contract price: (B) 4,40,000

6.25

Page 384: 30510870 Cost Accounting and Financial Management

Cost Accounting Estimated profit on the completion of contract: {(A) – (B)} 90,000

Since ⎟⎟⎠

⎞⎜⎜⎝

⎛priceContract

certified Work × 100 = 000,40,4.Rs000,05,4.Rs × 100 = 92.05%

Illustration A construction company under-taking a number of contracts, furnished the following data relating to its uncompleted contracts as on 31st March, 2006 : (Rs. in lacs) Contract Numbers

723 726 729 731 Total Contract Price 23.20 14.40 10.08 28.80 Estimated Costs on completion of contract 20.50 11.52 12.60 21.60 Expenses for the year ended 31.3.06 : Direct Materials 5.22 1.80 1.98 0.80 Direct Wages 2.32 4.32 3.90 2.16 Overheads (Excluding Depreciation) 1.06 2.60 2.62 1.05 Profit Reserve as on 1.4.05 1.50 — — — Plant issued at Cost 5.00 3.50 2.75 3.00 Materials at Site on 1.4.05 0.75 — — — Materials at Site on 31.3.06 0.45 0.20 0.08 0.05 Work Certified till 31.3.05 4.65 — — — Work Certified during the year 2005-06 12.76 13.26 7.56 4.32 Work Uncertified as on 31.3.06 0.84 0.24 0.14 0.18 Progress payments received during the year 9.57 9.0 5.75 3.60

Depreciation @ 20% per annum is to be charged on plant issued. While the Contract No. 723 was carried over from last year, the remaining contracts were started in the 1st week of April, 2005. Required :

(i) Determine the profit/loss in respect of each contract for the year ended 31st March, 2006.

(ii) State the profit/loss to be carried to Profit & Loss A/c for the year ended 31st March, 2006.

6.26

Page 385: 30510870 Cost Accounting and Financial Management

Method of Costing (I)

Solution (i) Statement of Profit/Loss in respect of following contract numbers for the year ended 31st March, 2006 (Rs. in lacs) Contract Numbers

723 726 729 731 A. Contract completion percentage : Work certified : (a) 17.41 13.26 7.56 4.32 Contract price : (b) 23.20 14.40 10.08 28.80 Percentage of completion : [(a) – (b)] 75.04 92.08 75.00 15.00 B. Estimated profit on completion : Contract price : (c) 23.20 14.40 10.08 28.80 Estimated costs on completion : (d) 20.50 11.52 12.60 21.60 Estimated profit (loss) on completion : [(c) – (d)] 2.70 2.88 (2.52) 7.20 C. Profit of the year : Op. stock of materials 0.75 — — — Materials issued 5.22 1.80 1.98 0.80 Direct wages 2.32 4.32 3.90 2.16 Overheads 1.06 2.60 2.62 1.05 Depreciation 1.00 0.70 0.55 0.60

Total : (P) 10.35 9.42 9.05 4.61 Profit in reserve 1.50 — — — Material at site on 31/3/06 0.45 0.20 0.08 0.05 Total : (Q) 1.95 0.20 0.08 0.05 Cost of contract : (R) = [(P) – (Q)] 8.40 9.22 8.97 4.56 Work certified 12.76 13.26 7.56 4.32 Work not certified 0.84 0.24 0.14 0.18 Total : (S) 13.60 13.50 7.70 4.50 Profit (loss) for the year [(R) – (S)] 5.20 4.28 (1.27) (0.06)

6.27

Page 386: 30510870 Cost Accounting and Financial Management

Cost Accounting (ii) Profit to be taken to Profit & Loss Account of the year in respect of respective contract

Contract 723 = 32 × Notional profit × certified Work

received Cash

= 32 × 5.20 ×

76.1257.9 = Rs. 2.60 lacs

= Balance Rs. 2.60 lacs to reserve.

Contract 726 = Estimated total profits on completion × priceContract

certified Work ×

certified Workreceived Cash

= 2.88 ×40.1426.13 ×

26.1300.9 = Rs. 1.80 lacs

= Balance to reserve Rs. 2.48 lacs Contract 729 = Provide for current loss of Rs. 1.27 lacs = Provide for expected loss of Rs. 1.25 lacs Contract 731 = Provide for current loss of Rs. 0.06 lacs

Illustration MNP Construction Ltd. commenced a contract on April 1, 2004. The total contract was for Rs. 17,50,000. It was decided to estimate the total profit and to take to the credit of P/L A/c the proportion of estimated profit on cash basis which work completed bore to the total contract. Actual expenditure in 2004-05 and estimated expenditure in 2005-2006 are given below : 2004-2005 2005-2006 (Actual) (Estimated) Rs. Rs. Materials issued 3,00,000 5,50,000 Labour : Paid 2,00,000 2,50,000 : Outstanding at end 20,000 30,000 Plant purchased 1,50,000 — Expenses : Paid 75,000 1,50,000 : Prepaid at end 15,000 — Plant returns to store (historical cost) 50,000 1,00,000 (on Dec. 31, 2005)

6.28

Page 387: 30510870 Cost Accounting and Financial Management

Method of Costing (I)

Material at site 20,000 50,000 Work certified 8,00,000 Full Work uncertified 25,000 — Cash received 6,00,000 Full The plant is subject to annual depreciation @ 25% of WDV Cost. The contract is likely to be completed on Dec. 31, 2005. Prepare the Contract A/c. Determine the profit on the contract for the year 2004-2005 on prudent basis, which has to be credited to P/L A/c.

Solution MNP Construction Ltd.

Contract Account (1st April, 2004 to 31st March, 2005) Dr. Cr. Particulars Amount Particulars Amount (Rs.) (Rs.) Materials issued 3,00,000 By Plant returned to store 37,500 Labour : Paid 2,00,000 (Refer to working note 1) Outstanding 20,000 2,20,000 By Materials at site 20,000 To Plant purchased 1,50,000 By Work certified 8,00,000 (Refer to working note 4) By Work uncertified 25,000 To Expenses 60,000 By Plant at site 75,000 To Notional profit c/d 2,27,500 (Refer to working note 2) ________ 9,57,500 9,57,500To Profit and Loss A/c 66,321,43 By Notional profit b/d 2,27,500,00 (Refer to working note 5) To Work in Progress A/c 1,61,178,57 (Profit in reserve) ___________ ___________ 2,27,500,00 2,27,500,00

MSP Construction Ltd. Contract Account (1st April, 2004 to 31st December, 2005) (For computing estimated profit) Dr. Cr. Particulars Amount Particulars Amount (Rs.) (Rs.) To Materials issued 8,50,000 By Materials at site 50,000 (Rs. 3,00,000 + Rs. 5,50,000) By Plant returned to store on 37,500

6.29

Page 388: 30510870 Cost Accounting and Financial Management

Cost Accounting To Labour (paid & outstanding) 4,80,000 31st March, 2005 (Rs. 2,20,000 + Rs. 2,30,000 + Rs. 30,000) (Refer to working note 1) To Plant purchased 1,50,000 By Plant returned to store 60,937.50 To Expenses 2,25,000 on 31st December, 2005 (Rs. 60,000 + Rs. 1,65,000) (Refer to working note 3) To Estimated profit 1,93,437.50 By Contractee’s A/c 17,50,000 18,98,437.50 18,98,437.50

Working Notes : 1. Value of the plant returned to store on 31st March, 2005 Rs. Historical cost of the plant returned 50,000 Less : Depreciation @ 25% of WDV cost for 1 year 12,500 Value of the plant returned to store on 31st March, 2005 37,5002. Value of plant at site : Rs. Historical cost of the plant at site 1,00,000 Less : Depreciation @ 25% of WDV cost for 1 year 25,000 Value of the plant at site on 31st March, 2005 75,0003. Value of the plant returned to store on 31st December, 2005 Rs. Value of the plant on 31st March, 2005 75,000.00 Less : Depreciation @ 25% of WDV for a period of 9 months 14,062.50 Value of the plant on 31-12-2005 60,937.50 4. Expenses paid : Total expenses paid 75,000 Less : Prepaid expenses at end 15,000 Expenses paid for the year 2004-2005 60,0005. Profit to be credited to P/L A/c on 31st March, 2005 for the contract likely to be

completed on 31st December, 2005

Estimated profit × certified Workreceived Cash

× priceContract Totalcertified Work

= Rs. 1,93,437.50 × 0Rs.8,00,000Rs.6,00,00

× 00Rs.17,50,00Rs.8,00,00

= Rs. 66,321.43.

6.30

Page 389: 30510870 Cost Accounting and Financial Management

Method of Costing (I)

6.4 BATCH COSTING

6.4.1 Meaning of Batch costing : This is a form of job costing. Under job costing, executed job is used as a cost unit, whereas under batch costing, a lot of similar units which comprises the batch may be used as a cost unit for ascertaining cost. In the case of batch costing separate cost sheets are maintained for each batch of products by assigning a batch number. Cost per unit in a batch is ascertained by dividing the total cost of a batch by number of items produced in that batch. Such a method of Costing is used in the case of pharmaceutical or drug industries, ready-made garments, industries manu-facturing electronic parts of T.V., radio sets etc.

Illustration

A jobbing factory has undertaken to supply 200 pieces of a component per month for the ensuing six months. Every month a batch order is opened against which materials and labour hours are booked at actuals. Overheads are levied at a rate per labour hour. The selling price contracted for is Rs. 8 per piece. From the following data present the cost and profit per piece of each batch order and overall position of the order for 1,200 pieces. Month Batch Material Direct Direct Output cost wages labour hours Rs. Rs. January 210 650 120 240 February 200 640 140 280 March 220 680 150 280 April 180 630 140 270 May 200 700 150 300 June 220 720 160 320 The other details are : Month Chargeable expenses Direct labour Rs. hours January 12,000 4,800 February 10,560 4,400 March 12,000 5,000 April 10,580 4,600 May 13,000 5,000 June 12,000 4,800

6.31

Page 390: 30510870 Cost Accounting and Financial Management

Cost Accounting

Solution

Jan. Feb. March April May June Total Batch output (in units) 210 200 220 180 200 220 1,230 Sale value Rs. 1,680 1,600 1,760 1,440 1,600 1,760 9,840 Material cost Rs. 650 640 680 630 700 720 4,020 Direct wages Rs. 120 140 150 140 150 160 860 Chargeable expenses Rs. 600 672 672 621 780 800 4,145 Total cost Rs. 1,370 1,452 1,502 1,391 1,630 1,680 9,025 Profit per batch Rs. 310 148 258 49 –30 80 815 Total cost per unit Rs. 6.52 7.26 6.83 7.73 8.15 7.64 7.34 Profit per unit Rs. 1.48 0.74 1.17 0.27 –0.15 0.36 0.66 Overall position of the order for 1,200 units Sales value of 1,200 units @ Rs. 8 per unit Rs. 9,600 Total cost of 1,200 units @ Rs. 7.34 per unit Rs. 8,808 Profit Rs. 792 6.4.2 Economic Batch Quantity : In batch costing the most important problem is the determination of optimum size of the batch (how much to produce) or Economic Batch Quantity.

The determination of economic batch quantity involve two types of costs viz., (i) set up cost (or preparation cost) and (ii) carrying cost. With the increase in the batch size, there is an increase in the carrying cost but the set up cost per unit of product is reduced; this situation is reversed when the batch size is reduced. Thus there is one particular batch size for which both set up and carrying costs are minimum. This size is known as economic or optimum batch quantity.

Economic batch quantity can be determined with the help of a table, graph or mathematical formula. The mathematical formula usually used for its determination is as follows :

EBQ =ICDS2

6.32

Page 391: 30510870 Cost Accounting and Financial Management

Method of Costing (I)

Where, D = Annual demand for the product S = Setting up cost per batch C = Carrying cost per unit of production

Note : If the rate of interest (I) and unit cost of production (C) are given, the following formula should be used for determining EBQ.

EBQ = ICDS2

Illustration : Monthly demand for a product 500 units Setting-up cost per batch Rs. 60 Cost of manufacturing per unit Rs. 20 Rate of interest 10% p.a. Determine economic batch quantity.

Solution

EBQ = ICDS2 =

201.060125002

×××× = 600 units.

6.5 FLOW COSTING

It refers to system of costing added in case of manufacturing of machine parts of standard design which later are put together at the assembly time before complete machine is ready.

6.6 OPERATING COSTING

6.6.1 Meaning of Operating Costing: It is a method of ascertaining costs of providing or operating a service. This method of costing is applied by those undertakings which provide services rather than production of commodities. The emphasis under operating costing is on the ascertainment of cost of services rather than on the cost of manufacturing a product. This costing method is usually made use of by transport companies, gas and water works departments, electricity supply companies, canteens, hospitals, theatres, schools etc.

For computing the operating cost, it is necessary to decide first, about the unit for which the cost is to be computed, this may often require the study of some technical and

6.33

Page 392: 30510870 Cost Accounting and Financial Management

Cost Accounting operating data, for finding out the factors which have a bearing on cost. The cost units usually used in the following service undertakings are as below :

Transport service − Passenger km., quintal km., or tonne km.

Supply service − Kw hr., Cubic metre, per kg., per litre.

Hospital − Patient per day, room per day or per bed, per operation etc.

Canteen − Per item, per meal etc.

Cinema − Per ticket. Composite units i.e. tonnes kms., quintal kms. etc. may be computed in two ways. (i) Absolute (weighted average) tonnes-kms., quintal kms. etc. (ii) Commercial (simple average) tonnes-kms., quintal kms. etc. (i) Absolute (weighted average) tonnes-kms. Absolute tonnes-kms., are the sum total of tonnes-kms., arrived at by multiplying various distances by respective load quantities carried. (ii) Commercial (simple average) tonnes-kms. Commercial tonnes-kms., are arrived at by multiplying total distance kms., by average load quantity. Note: To understand the concept of absolute tonnes-kms., and commercial tonnes-kms., students should refer to the following illustration. Illustration A lorry starts with a load of 20 tonnes of goods from station A. It unloads 8 tonnes at station B and rest of goods at station C. It reaches back directly to station A after getting reloaded with 16 tonnes of goods at station C. The distance between A to B, B to C and then from C to A are 80 kms., 120 kms., and 160 kms., respectively. Compute ‘Absolute tonnes-kms.,’ and ‘Commercial tonnes-kms. Solution Absolute tonnes-kms. = 20 tonnes × 80 kms + 12 tonnes × 120 kms + 16 tonnes × 160 kms.

= 5,600 tonnes-kms. Commercial tonnes-kms. = Average load × total kilometres travelled

6.34

Page 393: 30510870 Cost Accounting and Financial Management

Method of Costing (I)

= ⎟⎠⎞

⎜⎝⎛ ++

3161220 tonnes × 360 kms. = 5,760 tonnes-kms.

6.6.2 Preparation of Cost Sheet under Operating Costing: For preparing a cost sheet under operating cost, costs are usually accumulated for a specified period viz., a month, a quarter, or a year etc. All of the accumulated costs should be classified under the following three heads: 1. Fixed costs or standing charges,

2. Variable costs or running charges,

3. Semi-variable costs or maintenance costs.

Note : In the absence of information about semi-variable costs, the costs may be shown under two heads only, i.e., fixed and variable.

Under operating costing, the per unit cost of service may be calculated by dividing the total cost for the period by the total units of service in the period.

Treatment of depreciation and interest - Depreciation if related to effluxion of time, may be treated as fixed. If it is related to the activity level, it may be treated as variable.

If information about interest is explicitly given, it may be treated as fixed cost.

Illustration

You have been given a permit to run a bus on a route 20 Km. long. The bus costs you Rs. 90,000. It has to be insured @ 3% p.a. and the annual tax will be Rs. 1,000. Garage rent is Rs. 100 p.m. Annual repairs will be Rs. 1,000 and the bus is likely to last for 5 years at the end of which the scrap value is likely to be Rs. 6,000.

The driver’s salary will be Rs. 150 p.m. and the conductor’s Rs. 100 together with 10% of the takings as commission (to be shared equally by both). Stationery will cost Rs. 50 p.m. The manager-cum-accountant’s salary will be Rs. 250 p.m.

Diesel and oil be Rs. 25 per hundred kilometres. The bus will make 3 round trips for carrying on the average 40 passengers on each trip. Assuming 15% profit on takings, calculate the bus fare to be charged from each passenger. The bus will work on the average 25 days in a month.

6.35

Page 394: 30510870 Cost Accounting and Financial Management

Cost Accounting Solution

Operating Cost Statement Bus No. : DLP 4179 Carrying capacity : 40 Per annum Per 100 passenger km. Rs. P. Rs. P. 1 2 3 A. Standing Charges Depreciation (90,000 – 6,000) ÷ 5 16,800 Tax 1,000 Insurance 2,700 Stationery 600 Manager’s salary 4,200 _____ Total 25,300 1.756B. Maintenance charges Garage rent 1,200 Repairs 1,000 _____ Total 2,200 0.152C. Operating or running charges Diesel and oil 9,000 Driver’s salary 1,800 Conductor’s salary 1,200 _____ Total 12,000 0.833 Grand Total (A+B+C) 2.741 Loading @ 25/75 0.91 Fare per passenger-km. 3.65 Notes : (1) Number of kms. run in a month : 3 × 2 × 20 × 25 = 3,000

(2) Diesel & Oil : 3,000 × 10025

= Rs. 750

(3) Number of passenger-kms. per month : 3,000 × 40 = 1,20,000 per annum : 1,20,000 × 12 = 14,40,000

6.36

Page 395: 30510870 Cost Accounting and Financial Management

Method of Costing (I)

(4) Loading - If taking is Rs. 100, 10 will have to be given as commission and 15 must remain as profit; the cost must therefore be 75. On 75 the loading must be 25 to make the taking equal to 100.

Illustration

SMC is a public school having five buses each plying in different directions for the transport of its school students. In view of a larger number of students availing of the bus service the buses work two shifts daily both in the morning and in the afternoon. The buses are garaged in the school. The work-load of the students has been so arranged that in the morning the first trip picks up senior students and the second trip plying an hour later picks up the junior students. Similarly in the afternoon the first trip takes the junior students and an hour later the second trip takes the senior students home.

The distance travelled by each bus one way is 8 kms. The school works 25 days in a month and remains closed for vacation in May, June and December. Bus fee, however, is payable by the students for all 12 months in a year.

The details of expenses for a year are as under : Driver’s salary Rs. 450 per month per driver Cleaner’s salary Rs. 350 per month (Salary payable for all 12 months) (one cleaner employed for all the five buses) Licence fee, taxes, etc. Rs. 860 per bus per annum Insurance Rs. 1,000 per bus per annum Repairs & maintenance Rs. 3,500 per bus per annum Purchase price of the bus Rs. 1,50,000 each Life 12 years Scrap value Rs. 30,000 Diesel cost Rs. 2.00 per litre

Each bus gives an average mileage of 4 km. per litre of diesel.

Seating capacity of each bus is 50 students.

The seating capacity is fully occupied during the whole year.

Students picked up and dropped within a range upto 4 kms. of distance from the school

6.37

Page 396: 30510870 Cost Accounting and Financial Management

Cost Accounting are charged half fare and fifty per cent of the students travelling in each trip are in this category. Ignore interest. Since the charges are to be based on average cost you are required to :

(i) Prepare a statement showing the expenses of operating a single bus and the fleet of five buses for a year.

(ii) Work out the average cost per student per month in respect of – (A) students coming from a distance of upto 4 kms. from the school and (B) students coming from a distance beyond 4 kms. from the school.

Solution (i) SMC Public School

Operating Cost Statement Particulars Rate Per Bus Fleet of 5 Per annum buses p.a. Rs. No. Rs. No. Rs.

Driver’s salary 450 p.m. 1 5,400 5 27,000 Cleaner’s salary 350 p.m. 1/5 840 1 4,200 Licence fee, taxes etc. 860 p.a. 860 4,300 Insurance 1,000 p.a. 1,000 5,000 Repairs & maintenance 3,000 p.a. 3,500 17,500 Depreciation 10,000 p.a. 10,000 50,000 Diesel (see Note 1) — 7,200 36,000Total : 28,800 1,44,000Cost per month 2,400 12,000 (ii) No. of students on half fee basis (See note 2) 150 750 (A) Cost per student (half fee) Rs. 16.00 Rs. 16.00 (B) Cost per student (full fee) Rs. 32.00 Rs. 32.00

Working Notes : 1. Calculation of diesel cost per bus : Number of trips of 8 kms. each/day : 8 Distance travelled per day by a bus : 8 × 8 km/trip=64 km.

6.38

Page 397: 30510870 Cost Accounting and Financial Management

Method of Costing (I)

Distance travelled during a month : 64 × 25=1,600 km. Distance travelled p.a. (May, June : 1,600 × 9=14,400 km. and December being vacation) Mileage : 4 Km./litre. Diesel required : 14,400/4=3,600 litres. Cost of diesel : 3600 litres × Rs. 2 per litre = Rs. 7,200 p.a. per bus. 2. Calculation of number of students per bus : Bus capacity 50 students Half fare 50% i.e., 25 students Full fare 50% i.e., 25 students Full fare students as equivalent to half fare students i.e., 50 students Total number of half fare students per trip 75 students Total number of half fare students in two trips. 150 students On full fare basis number of students in two trips. 75 students

Illustration

From the following data pertaining to the year 2005-06 prepare a cost sheet showing the cost of electricity generated per k.w.h. by Chambal Thermal Power Station.

Total units generated 10,00,000 k.m.h. Rs. Operating labour 50,000 Repairs & maintenance 50,000 Lubricants, spares and stores 40,000 Plant supervision 30,000 Administration overheads 20,000

Coal consumed per k.w.h. for the year is 2.5 k.g. @ Re. 0.02 per kg. Depreciation charges @ 5% on capital cost of Rs. 2,00,000.

6.39

Page 398: 30510870 Cost Accounting and Financial Management

Cost Accounting Solution

Power House Cost Statement Total units generated 10,00,000 k.w.h. Per annum Per k.w.h. Rs. Rs. Fixed costs : Plant supervision 30,000 Administration overheads 20,000 Depreciation (5% of Rs. 2,00,000 p.a.) 10,000Total fixed cost: (A) 60,000 0.06 Variable costs Operating labour 50,000 0.05 Lubricating, spares & stores 40,000 0.04 Repairs & maintenance 50,000 0.05 Coal cost (Refer to working note) 50,000 0.05 Total variable cost: (B) 1,90,000 Total cost [(A) + (B)] 2,50,000 0.25

Working Note: Coal cost 10,00,000 k.w.h. × 2.5 kg × 0.02 per kg. = Rs. 50,000 Standard Load - An alternative unit for the distribution of transport cost is the ‘standard’ load. Where the goods to be transported are of varying bulk and weight, the calculation of actual number of tonne-kilometres is not an easy matter. For example, if a business delivers its own products by its own transport, the cost per tonne-kilometres may be most misleading, for an article may have a bulk which is twice that of the other, though of the same weight. In such a case ‘standard load’ is selected as the unit, i.e., the load which a lorry would carry. This would have reference both to bulk and weight and would give an efficient method for distributing the cost of transport over different departments. Thus, if the turnover of various departments is reduced to ‘standard load’ by first calculating their weight and then the bulk of article produced, the costs of distributing the product would be easily ascertained.

This principle also can be extended for associating cost with convenient units of service rendered by an organisation so that management is able to judge whether the organisation is running efficiently and in the manner in which the service requires to be

6.40

Page 399: 30510870 Cost Accounting and Financial Management

Method of Costing (I)

improved or be made more economical. The cost of generation of electricity on the same principle is correlated with units generated and also with units sold; in hospitals the cost of their maintenance is co-related to units of ‘available bed-days’.

6.7 MULTIPLE COSTING

It refers to the method of costing followed by a business wherein a large variety of articles are produced, each differing from the other both in regard to material required and process of manufacture. In such cases, cost of each article is computed separately by using, generally, two or more methods of costing. For instance, for ascertaining the cost of a bicycle, cost of each part will be ascertained by using batch or job costing method and, then the cost of assembling the parts will be ascertained by following the method of single or output costing.

6.9 Self examination questions

Multiple choice questions

(a) The principal factors to be considered in designing a cost system include :

(i) manufacturing process

(ii) desire of management

(iii) nature of business

(iv) all of the above.

(b) The most suitable cost system where the products differ in type of materials and work performed is :

(i) Job Costing

(ii) Process Costing

(iii) Operating Costing

(iv) None of these.

(c) Which of the following statements is true (i) Job cost sheet may be used for estimating profit of jobs. (ii) Job costing cannot be used in conjunction with marginal costing. (iii) In cost plus contracts, the contractor runs a risk of incurring a loss. (iv) Batch costing is a variant of jobs costing.

6.41

Page 400: 30510870 Cost Accounting and Financial Management

Cost Accounting (d) The method of costing used in job order industries is known as

(i) Process costing

(ii) Job costing

(iii) Contract costing

(iv) Marginal costing.

(e) Which of the following statements is true

(i) In job costing method, a cost sheet is prepared for each job.

(ii) A production order is an order received from a customer for particular jobs.

(iii) In contract costing, the contract which is complete up to one fourth of the total contract, one-fourth of the profit should be transferred to Profit & Loss Account.

(iv) In contract costing profit of each contract is computed when the contract is completed.

(f) Which of the following would best describe the characteristics of contract costing:

(a) homogeneous products;

(b) customer driven production;

(c) short period of time between the commencement and completion of the cost unit

(i) (a) and (b) only

(ii) (b) and (c) only

(iii) (a) and (c) only

(iv) (b) only

(g) In job costing which of the following documents are used to record the issue of direct material to a job’

(i) Goods received note

(ii) Material requisition

(iii) Purchase order

(iv) Purchase requisition

6.42

Page 401: 30510870 Cost Accounting and Financial Management

Method of Costing (I)

(h) Which of the following statements is true,

(i) Job cost sheet may be prepared for facilitating routing and scheduling of the job

(ii) Job costing can be suitably used for concerns producing uniformly any specific product

(iii) Job costing cannot be used in companies using standard costing

(iv) Neither (i) nor (ii) nor (Iii)

(i) A lorry capable of carrying 5 tonnes of goods normally carries 80% of the load on the outward journey and 40% of the load on inward jounery. The jounery is 300 kms. for one side. It takes two days to complete the return trip. In a year of 300 days compute the tonnes Kms.

(i) 2,70,000

(ii) 3,00,000

(iii) 3,30,000

(iv) 3,40,000

(j) Which of the following statements is true,

(i) In job costing a cost sheet is prepared for each job

(ii) The concept of economic batch quantity is similar to economic order quantity

(iii) In job costing the cost of each job is arrived at by dividing the total cost incurred by the number of jobs

(iv) Neither (i) nor (ii) nor (iii)

Answers to multiple choice questions

a.(iv); b.(i); c. (iv); d.(ii); e.(i); f.(iv); g.(ii);h.(iv);i.(i); j(i&ii)

Short answer type questions

1. Define the main features of job costing. Mention enterprises which use job costing. 2. Describe briefly the procedure of Cost accumulation under job order costing. 3. Explain briefly what you understand by multiple costing. 4. Explain units of cost used in service undertakings. How are they computed?

6.43

Page 402: 30510870 Cost Accounting and Financial Management

Cost Accounting 5. What are the main features of cost-plus-contracts? Long answer type questions 1. Explain the nature and use of batch costing. Describe the concept of the economical

batch with the help of any suitable example. 2. Why is a portion of profit on uncompleted contracts transferred to the profit and loss

account? How would you determine the amount of profit to be transferred to the profit and loss account?

3. Describe the main features of industrial units using job costing. 4. Describe the essential feature of operating costing and state where it can be usefully

implemented. Numerical questions 1. Raw materials ‘X’ costing Rs. 100 per kg and Y costing Rs. 60 per kilogram are mixed in

equal proportions for making product A. The loss of material in processing works out to 25% of the output. The production expenses are allocated at 50% of direct material cost. The end product is priced with a margin of 331/3% over the total cost. Material Y is not easily available and substitute raw material ‘Z’ has been found for ‘Y’ costing Rs. 50 per kg. It is required to keep the proportion of this substitute material in the mixture as low as possible and at the same time maintain the selling price of the end product at existing levels and ensure the same quantum of profit as at present.

You are required : To Compute what should be the ratio of mix of the raw materials X and Z. 2. Mr. Bansal has a factory. He specialises in the manufacture of small tables of standard

size. He can make 15,000 small tables in a year. The cost per table works out as under for the year and he made and sold 10,000 tables. Rs. Materials 180 Labour 60 Overhead (fixed 90 recovered @ 50% of material cost.) ___ Total cost 330Prices are fixed by adding a margin of 10% of the total cost arrived at the above.

6.44

Page 403: 30510870 Cost Accounting and Financial Management

Method of Costing (I)

In the current year, due to fall in the cost of materials, the total cost worked out as under :- Rs. Materials 120 Labour 60 Overhead recovered @ 50% of material cost 60Total cost 240 Mr. Bansal maintained his standard margin of 10% on the total cost. Sales were at the same level as in the last year. You are asked to

(a) Determine the profit or loss for the current year (b) Compute the price which should have been charged in the current year to yield the

same profit or loss as last year. 3. In a factory in a month 3 new jobs were commenced. The materials and labour used on

them were as follows : Job 1 Job 2 Job 3 Rs. Rs. Rs. Materials 4,000 4,500 2,700 Labour 5,100 8,300 1,400 Works overheads is charged at 60% of labour and office expenses @ 10% of works cost. Jobs 1 and 2 were completed but job 3 was still in progress. Prepare the job Accounts.

4. An expenditure of Rs. 4,85,000 has been incurred on a contract till 31st March, 2006 and value of the work certified is Rs. 5,50,000. The cost of work performed but not yet certified is Rs. 15,000. The profit of Rs. 30,000 had been taken to the credit of Profit & Loss Account till 31st March, 2005. The estimated future expenses are Rs. 1,00,000. The estimated total expenses is to include a provision of 2-1/2 per cent for contingencies. The contract price is Rs. 7,00,000 and the payment received till date is Rs. 5,00,000. Calculate the profit to be taken to the credit of Profit and Loss Account for the year ended on 31st March, 2006.

6.45

Page 404: 30510870 Cost Accounting and Financial Management

Cost Accounting 5. A transport Service Company is running 4 boxes between towns which are 50 kms. apart.

Seating capacity of each bus is 40 passengers. The following particulars were obtained from their books for April 2006, Rs. Wages of Drivers, Conductors and Cleaners 2,400 Salaries of Officer and Supervisory Staff 1,000 Diesel oil and other oil 4,000 Repairs and Maintenance 800 Taxations, Insurance, etc. 1,600 Depreciation 2,600 Interest and other charges 2,000 14,400 Actual passengers carried were 75% of the seating capacity. All the buses run on all the days of the month. Each bus made one round trip per day. Find out the costs per passenger-km.

6.46

Page 405: 30510870 Cost Accounting and Financial Management

CHAPTER 7

METHOD OF COSTING (II) (PROCESS COSTING, OPERATION COSTING, JOINT

PRODUCTS AND BY–PRODUCTS)

Learning objectives

When you have finished studying this chapter, you should be able to

♦ Understand the meaning of Process and Operation costing. ♦ Understand and differentiate between Joint and By products. ♦ Understand the accounting treatment required for normal and abnormal process losses. ♦ Understand the treatment for abnormal gain. ♦ Understand the accounting treatment required for joint products and by products.

7.1 MEANING OF PROCESS COSTING Process Costing is a method of Costing used in industries where the material has to pass through two or more processes for being converted into a final product. It is defined as “a method of Cost Accounting whereby costs are charged to processes or operations and averaged over units produced”. Such type of costing method is useful in the manufacturing of products like steel, soap, chemicals, rubber, vegetable oil, paints, varnish etc. where the production process is continuous and the output of one process becomes the input of the following process till completion. 7.1.1 Basic features : Industries, where process costing can be applied, have normally one or more of the following features :

1. Each plant or factory is divided into a number of processes, cost centres or departments, and each such division is a stage of production or a process.

2. Manufacturing activity is carried on continuously by means of one or more process run sequentially, selectively or parallely.

3. The output of one process becomes the input of another process.

Page 406: 30510870 Cost Accounting and Financial Management

Cost Accounting 4. The end product usually is of like units not distinguishable from one another.

5. It is not possible to trace the identity of any particular lot of output to any lot of input materials. For example, in the sugar industry, it is impossible to trace any lot of sugar bags to a particular lot of sugarcane fed or vice versa.

6. Production of a product may give rise to Joint and/or By-Products. 7.1.2 Costing Procedure : The Cost of each process comprises the cost of : (i) Materials (ii) Labour (iii) Direct expenses, and (iv) Overheads of production. Materials - Materials and supplies which are required for each process are drawn against material requisitions from stores. Each process for which the above drawn materials will be used should be debited with the cost of materials consumed on the basis of the information received from the Cost Accounting department. The finished product of first process generally become the raw materials of second process; under such a situation the account of second process, be debited with the cost of transfer from the first process and the cost of any additional material required under this second process. Labour - Each process account should be debited with the labour cost or wages paid to labour for carrying out the processing activities. Sometimes the wages paid are apportioned over the different processes after selecting appropriate basis. Direct expenses - Each process account should be debited with direct expenses like depreciation, repairs, maintenance, insurance etc. associated with it. Overheads of production - Expenses like rent, power expenses, lighting bills, gas and water bills etc. are known as production overheads. These expenses cannot be allocated to a process. The suitable wayout to recover them is to apportion them over different processes by using suitable basis. Usually, these expenses are estimated in advance and the processes debited with these expenses on a pre-determined basis.

7.2 OPERATION COSTING It is defined as the refinement of process costing. It is concerned with the determination of the cost of each operation rather than the process. In those industries where a process consists of distinct operations, the method of costing applied or used is called operation costing. Operation costing offers better scope for control. It facilitates the computation of unit operation cost at the end of each operation by dividing the total operation cost by total output units.

7.2

Page 407: 30510870 Cost Accounting and Financial Management

Method of Costing (II)

Illustration : From the following data, prepare process accounts indicating the cost of each process and the total cost. The total units that pass through each process were 240 for the period. Process A Process B Process C Rs. Rs. Rs.

Materials 1,500 500 200 Labour 800 2,000 600 Other expenses 260 720 250 Indirect expenses amounting to Rs. 850 may be apportioned on the basis of wages. There was no opening or closing stock.

Solution : Dr. Process ‘A’ Account Cr.

Per unit Total Per unit Total Rs. Rs. Rs. Rs. To Material 6.25 1,500 By Transfer to ” Labour 3.34 800 Process ‘B’ A/c 11.50 2,760 ” Other expenses 1.08 260 ” Indirect expenses 0.83 200 _____ _____ 11.50 2,760 11.50 2,760

Dr. Process ‘B’ Account Cr.

Per unit Total Per unit Total Rs. Rs. Rs. Rs. To be transferred By Transfer to from Process ‘A’ A/c 11.50 2,760 Process ‘C’ A/c 27.00 6,480 ” Materials 2.08 500 ” Labour 8.34 2,000 ” Other expenses 3.00 720 ” Indirect expenses 2.08 500 27.00 6,480 27.00 6,480

7.3

Page 408: 30510870 Cost Accounting and Financial Management

Cost Accounting Dr. Process ‘C’ Account Cr.

Per unit Total Per unit Total Rs. Rs. Rs. Rs. To be transferred By Finished stock from Process ‘B’ A/c 27.00 6,480 A/c transfer 32.00 7,680 ” Materials 0.83 200 ” Labour 2.54 600 ” Other expenses 1.04 250 ” Indirect expenses 0.59 150 32.00 7,680 32.00 7,680

7.3 TREATMENT OF NORMAL PROCESS LOSS, ABNORMAL PROCESS LOSS AND ABNORMAL GAIN Loss of material is inherent during processing operation. The loss of material under different processes arises due to reasons like evaporation or a change in the moisture content etc. Process loss is defined as the loss of material arising during the course of a processing operation and is equal to the difference between the input quantity of the material and its output. There are two types of material losses viz. (i) Normal loss and (ii) Abnormal loss. (i) Normal process loss : It is defined as the loss of material which is inherent in the nature of work. Such a loss can be reasonably anticipated from the nature of the material, nature of operation, the experience and technical data. The cost of normal process loss in practice is absorbed by good units produced under the process. The amount realised by the sale of normal process loss units should be credited to the process account. (ii) Abnormal process loss : It is defined as the loss in excess of the pre-determined loss. This type of loss may occur due to the carelessness of workers, a bad plant design or operation etc. Such a loss cannot obviously be estimated in advance. But it can be kept under control by taking suitable measures. The cost of an abnormal process loss unit is equal to the cost of a good unit. The total cost of abnormal process loss is credited to the process account from which it arise. Cost of abnormal process loss is not treated as a part of the cost of the product. In fact, the total cost of abnormal process loss is debited to costing profit and loss account. Abnormal gain : Sometimes, loss under a process is less than the anticipated normal figure. In other words, the actual production exceeds the expected figures. Under such a situation the difference between actual and expected loss or actual and expected production is known as

7.4

Page 409: 30510870 Cost Accounting and Financial Management

Method of Costing (II)

abnormal gain. So abnormal gain may be defined as unexpected gain in production under normal conditions. The process account under which abnormal gain arises is debited with the abnormal gain. The cost of abnormal gain is computed on the basis of normal production. To be more clear about the above concepts we consider the following illustration.

Illustration : A product passes through three processes. The output of each process is treated as the raw material of the next process to which it is transferred and output of the third process is transferred to finished stock. Ist Process 2nd Process 3rd Process Rs. Rs. Rs. Material issued 40,000 20,000 10,000 Labour 6,000 4,000 1,000 Manufacturing overhead 10,000 10,000 15,000 10,000 units have been issued to the 1st process and after processing, the output of each process is as under : Output Normal Loss Process No. 1 9,750 units 2% Process No. 2 9,400 units 5% Process No. 3 8,000 units 10% No stock of materials or of work-in-progress was left at the end. Calculate the cost of the finished articles. Solution:

Process No. 1 Account Units Rs. Units Rs. To Material 10,000 40,000 By Normal wastage 200 ” Labour 6,000 ” Abnormal wastage 50 286 ” Overhead 10,000 (cost per unit, Rs. 5.714) ” Process No. 2 9,750 55,714 (Transfer of _____ ______ completed units) _____ ______ 10,000 56,000 10,000 56,000

7.5

Page 410: 30510870 Cost Accounting and Financial Management

Cost Accounting Note : The cost of the abnormal wastage : Normal Output = 10,000 units – 200 units = 9,800 units Cost per unit of normal output = Rs. 56,000 ÷ 9,800 units = Rs. 5.714 Cost of 50 units = Rs. 5.714 × 50 = Rs. 286

Process No. 2 Account Units Rs. Units Rs. To Process No. 1 9,750 55,714 By Normal wastage 488 – ” Materials 20,000 (5% of 9,750) ” Labour 4,000 ” Process No. 3 9,400 91,051 ” Overhead 10,000 (cost per unit ” Abnormal gain Rs. 9.686) @ Rs. 9.686 138 1,337 _____ ______ 9,888 91,051 9,888 91,051 Note : The cost per unit is obtained by dividing Rs. 89,714 by 9,262 units, i.e., 9,750 units less 488 units.

Process No. 3 Account Units Rs. Units Rs. To Process No. 2 9,400 91,051 By Normal wastage 940 ” Materials 10,000 ” Abnormal wastage 460 6,364 ” Labour 1,000 (Cost per unit ” Overhead 15,000 Rs. 13.836) _____ _______ ” Finished stock 8.000 1,10,687 9,400 1,17,051 9,400 1,17,051 Note : The calculation of the cost of abnormal wastage : Normal Output = 9,400 units – 940 units = 8,460 units. Cost per unit of normal output = Rs. 1,17,051 ÷ 8,460 = Rs. 13,836 Cost of 460 units is = Rs. 6,364.

7.4 COSTING OF EQUIVALENT PRODUCTION UNITS In the case of process type of industries it is possible to determine the average cost per unit by dividing the total cost incurred during a given period of time by the total number of units

7.6

Page 411: 30510870 Cost Accounting and Financial Management

Method of Costing (II)

produced during the same period. But this is hardly the case in most of the process type industries where manufacturing is a continuous activity. The reason is that the cost incurred in such industries represents the cost of work carried on opening work-in-progress, closing work-in-progress and completed units. Thus to ascertain the cost of each completed unit it is necessary to ascertain the cost of work-in-progress in the beginning and at the end of the process. The valuation of work-in-progress presents a good deal of difficulty because it has units under different stages of completion from those in which work has just begun to those which are only a step short of completion. Work-in-progress can be valued on actual basis, i.e., materials used on the unfinished units and the actual amount of labour expenses involved. However, the degree of accuracy in such a case cannot be satisfactory. An alternative method is based on converting partly finished units into equivalent finished units. Equivalent production means converting the incomplete production units into their equivalent completed units. Under each process, an estimate is made of the percentage completion of work-in-progress with regard to different elements of costs, viz., material, labour and overheads. It is important that the estimate of percentage of completion should be as accurate as possible. The formula for computing equivalent completed units is : Equivalent completed units = {Actual number of units in the process of manufacture} ×

{Percentage of work completed} For instance, if 25% of work has been done on the average of units still under process, then 200 such units will be equal to 50 completed units and the cost of work-in-progress will be equal to the cost of 50 finished units. 7.4.1 Valuation of work-in-progress : For the valuation of work-in-progress following three methods are available :

(1) First-in-First Out (FIFO) method.

(2) Last-in-First Out (LIFO) method.

(3) Average Cost method (or weighted average cost method).

(1) First-in-first-out method - Under this method the units completed and transferred include completed units of opening work-in-progress and subsequently introduced units. Proportionate cost to complete the opening work-in-progress and that to process the completely processed units during the period are derived separately. The cost of opening work-in-progress is added to the proportionate cost incurred on completing the same to get the complete cost of such units. Complete cost of such units plus cost of units completely processed constitute the total cost of units transferred.

7.7

Page 412: 30510870 Cost Accounting and Financial Management

Cost Accounting Illustration : Opening work-in-progress 1,000 units (60% complete) Cost Rs. 1,100. Units introduced during the period 10,000 units; Cost Rs. 19,300. Transferred to next process - 9,000 units. Closing work-in-progress - 800 units (75% complete). Normal loss is estimated at 10% of total input including units in process at the beginning. Scrap realise Re. 1 per unit. Scrapped are 100% complete. Compute equivalent production and cost per equivalent unit. Also evaluate the output. Solution :

FIFO Method Statement of equivalent production and cost per unit

Input Output Equivalent Production Particulars units Particulars units % of work Equivalent done during units current period

Op. work-in-process 1,000 Op. WIP : Completed 1,000 40 400 Units introduced 10,000 Completed 8,000 100 8,000 Normal loss 1,100 - - Closing work- in-process 800 75 600 Abnormal loss 100 100 100 11,000 9,100

Cost of the Process (for the period) Rs. 19,300 Less: Scrap value of normal loss Rs. 1,100Cost per equivalent unit Rs. 18,200 ÷ 9,100 units = Rs. 2

Statement of Evaluation Particulars Equivalent Cost per equi- Amount units valent unit Rs. Rs. 1. Opening WIP completed 400 2.00 800 Add: Cost of opening WIP – – 1,100 Complete Cost of 1,000 units of Op. WIP 1,000 1.90 1,900 2. Completely processed units 8,000 2.00 16,000 3. Abnormal loss 100 2.00 200 4. Closing WIP 600 2.00 1,200

7.8

Page 413: 30510870 Cost Accounting and Financial Management

Method of Costing (II)

(2) Last-in first-out Method - According to this method units lastly entering in the process are the first to be completed. This assumption has a different impact on the costs of the completed units and the closing inventory of work-in-progress. The completed units will be shown at their current cost and the closing inventory of work-in-progress will continue to appear at the cost of the opening inventory of work-in-progress. Illustration : From the following information relating to the month of April 06, calculate the equivalent production units and the value of finished production and work-in-progress, using the LIFO method. Opening work-in-progress on 1st April: 5,000 units; 50% complete. Cost

Rs.

Materials 6,000 Labour 8,000 Overheads 8,000

22,000

Units introduced into the process : 10,000. Cost Rs.

Materials 30,000 Labour 52,500 Overheads 70,000

1,52,500 During the period 7,500 units were completed and transferred to the next process. Closing work-in-progress on 30th April: 7,500 units, 50% complete.

7.9

Page 414: 30510870 Cost Accounting and Financial Management

Cost Accounting Solution : (i) Computation of Equivalent Production Units (LIFO method) Units Particulars Equivalent production Units % of com- Equivalent out pletion units 5,000 Opening Work-in-Process 10,000 Units introduced into the process Units completed and transferred, of the units introduced during the period 7,500 100 7,500 Of the units introduced during the period 2,500 50 1,250 Of the opening work in process 5,000 – – 15,000 15,000 8,750 * Since the units in the opening work in process were already 50% complete; no work has been done on these units during the period.

(ii) Cost per unit of equivalent production = 750,8

000,52,1.Rs =Rs. 17.43

Valuation of finished production and WIP. 1. Finished production : 7,500 × Rs. 17.43 = Rs. 1,30,725

2. Closing WIP: Rs. 22,000 + (1,250 × Rs. 17.43) = Rs. 43,787.50

(3) Average Cost Method - Under this method, the cost of opening work-in-progress and cost of the current period are aggregated and the aggregate cost is divided by output in terms of completed units. The equivalent production in this case consists of work-load already contained in opening work-in-process and work-load of current period. Refer to illustration solved by FIFO method - Under Average Cost Method, the solution will be as follows :

Statement of equivalent Production and Cost per unit

Output Units Equivalent Production Percentage units Transferred to Next Process 9,000 100 9,000 Normal Loss 1,100 – – Abnormal Loss 100 100 100 Closing work-in-process 800 75 600 9,700

7.10

Page 415: 30510870 Cost Accounting and Financial Management

Method of Costing (II)

Costs : Rs. Opening Work-in-Process 1,100 Cost of units introduced 19,300 20,400 Less : Scrap value realised on normal loss 1,100 19,300 Cost per equivalent unit Rs. 19,300 ÷ 9,700 units = Rs. 1.99 (approx.)

Statement of Evaluation

Particulars Equivalent Cost per Amount units equivalent unit (Rs.) Rs. 1. Transferred to next process 9,000 1.99 17,910 2. Abnormal loss 100 1.99 199 3. Closing Work-in-process 600 1.99 1,194 19,303

Illustration : Following information is available regarding process A for the month of February, 2006 : Production Record Rs. Units in process as on 1.2.2006 4,000 (All materials used, 25% complete for labour and overhead) New units introduced 16,000 Units completed 14,000 Units in process as on 28.2.2006 6,000 (All materials used, 33-1/3% complete for labour and overhead) Cost Records Work-in-process as on 1.2.2006 Rs. Materials 6,000 Labour 1,000 Overhead 1,000

8,000

7.11

Page 416: 30510870 Cost Accounting and Financial Management

Cost Accounting Cost during the month Materials 25,600 Labour 15,000 Overhead 15,000

55,600 Presuming that average method of inventory is used, prepare : (i) Statement of equipment production. (ii) Statement showing cost for each element. (iii) Statement of apportionment of cost. (iv) Process cost account for process A.

Solution : (i) Statement of equivalent production (Average cost method) Particulars Materials Labour OverheadInput Output Units % Equivalent % Equivalent % Equivalent (units) Completion units Completion units Completion units 20,000 Completed 14,000 100 14,000 100 14,000 100 14,000 ______ WIP 6,000 100 6,000 33-1/3 2,000 33-1/3 2,00020,000 20,000 20,000 16,000 16,000

(ii) Statement showing cost for each element Particulars Materials Labour Overhead Total Cost of opening work-in-progress (Rs.) 6,000 1,000 1,000 8,000 Cost incurred during the month (Rs.) 25,600 15,000 15,000 55,600Total cost (Rs.) : (A) 31,600 16,000 16,000 63,600 Equivalent units : (B) 20,000 16,000 16,000 Cost per equivalent unit (Rs.) : (C) = (A/B) 1.58 1 1 3.58 (iii) Statement of apportionment of cost Rs. Rs. Value of output transferred : (a) 14,000 units @ Rs. 3.58 50,120 Value of closing work-in-progress : (b) Materials 6,000 units @ 1.58 9,480 Labour 2,000 units @ Re. 1 2,000 Overhead 2,000 units @ Re. 1 2,000 13,480

Total cost : (a + b) 63,600

7.12

Page 417: 30510870 Cost Accounting and Financial Management

Method of Costing (II)

(iv) Process cost account for process A: Process A Cost Account Dr. Cr. Units Rs. Units Rs. To Opening WIP 4,000 8,000 By Completed units 14,000 50,120 To Materials 16,000 25,600 By Closing WIP 6,000 13,480 To Labour 15,000 To Overhead ______ 15,000 ______ ______ 20,000 63,600 20,000 63,600

7.5 INTER-PROCESS PROFITS In some process industries the output of one process is transferred to the next process not at cost but at market value or cost plus a percentage of profit. The difference between cost and the transfer price is known as inter-process profits. The advantages and disadvantages of using inter-process profit, in the case of process type industries are as follows : Advantages : 1. Comparison between the cost of output and its market price at the stage of completion is

facilitated.

2. Each process is made to stand by itself as to the profitability. Disadvantages : 1. The use of inter-process profits involves complication.

2. The system shows profits which are not realised because of stock not sold out.

Illustration A Ltd. produces product ‘AXE’ which passes through two processes before it is completed and transferred to finished stock. The following data relate to October 2005 : Process Particulars I II Finished stock Rs. Rs. Rs. Opening stock 7,500 9,000 22,500 Direct materials 15,000 15,750 Direct wages 11,200 11,250 Factory overheads 10,500 4,500

7.13

Page 418: 30510870 Cost Accounting and Financial Management

Cost Accounting Closing stock 3,700 4,500 11,250 Inter-process profit included in opening stock 1,500 8,250 Output of Process I is transferred to Process II at 25% profit on the transfer price. Output of Process II is transferred to finished stock at 20% profit on the transfer price. Stock in process is valued at prime cost. Finished stock is valued at the price at which it is received from process II. Sales during the period are Rs. 1,40,000. Prepare Process cost accounts and finished goods account showing the profit element at each stage.

Solution

Process I Account Total Cost Profit Total Cost Profit Rs. Rs. Rs. Rs. Rs. Rs. Opening stock 7,500 7,500 – Transfer to Direct materials 15,000 15,000 – Process II A/c 54,000 40,500 13,500 Direct wages 11,200 11,200 – 33,700 33,700 – Less Closing stock 3,700 3,700Prime cost 30,000 30,000 – Overheads 10,500 10,500 – Process cost 40,500 40,500 – Profit 33 1/3 of Total cost 13,500 – 13,500 (see working note 1) ______ ______ _______ ______ _____ _____ 54,000 40,500 13,500 54,000 40,500 13,500

Process II Account

Total Cost Profit Total Cost Profit Rs. Rs. Rs. Rs. Rs. Rs. Opening stock 9,000 7,500 1,500 Transfer Transferred from to finished 1,12,500 75,750 36,750 Process I 54,000 40,500 13,500 stock A/c Direct materials 15,750 15,750 – Direct wages 11,250 11,250 – 90,000 75,000 15,000 Less : Closing stock 4,500 3,750 750

7.14

Page 419: 30510870 Cost Accounting and Financial Management

Method of Costing (II)

Prime cost 85,500 71,250 14,250 Overheads 4,500 4,500 – Process cost 90,000 75,750 14,250 Profit 25% on total cost 22,500 – 22,500 (See working note 2) _______ ______ ______ ________ _______ ______ 1,12,500 75,750 36,750 1,12,500 75,750 36,750

(Finished Stock Account) Total Cost Profit Total Cost Profit Rs. Rs. Rs. Rs. Rs. Rs. Opening stock 22,500 14,250 8,250 Sales 1,40,000 82,500 57,500 Transferred from Process II 1,12,500 75,750 36,750 1,35,000 90,000 45,000 Less : Closing stock 11,250 7,500 3,750 Finished stock : Cost 1,23,750 82,500 41,250Profit 16,250 – 16,250 1,40,000 82,500 57,500 1,40,000 82,500 57,500 Working Notes : Let the transfer price be 100 then profit is 25; i.e. cost price is Rs.75. 1. If cost is Rs. 75 then profit is Rs. 25

If cost is Rs. 40,500 then profit is 7525 × 40,500 = Rs. 13,500

2. If cost is Rs. 80 then profit is Rs. 20

If cost is Rs. 90,000 then profit is 8020 × 90,000 = Rs. 22,500

7.6 JOINT PRODUCTS AND BY-PRODUCTS

7.6.1 Meaning of Joint Products and By-Products : Agricultural product industries, chemical process industries, sugar industries, and extractive industries are some of the industries where two or more products of equal or unequal importance are produced either simultaneously or in the course of processing operation of a main product. In all such industries, the management is faced with the problems such as, valuation of inventory,

7.15

Page 420: 30510870 Cost Accounting and Financial Management

Cost Accounting pricing of product and income determination, problem of taking decision in matters of further processing of by-products and/or joint products after a certain stage etc. In fact the various problems relate to (i) apportionment of common costs incurred for various products and (ii) aspects other than mere apportionment of costs incurred upto the point of separation. Before taking up the above problems, we first define the various necessary concepts. Joint Products - Joint products represent “two or more products separated in the course of the same processing operation usually requiring further processing, each product being in such proportion that no single product can be designated as a major product”. In other words, two or more products of equal importance, produced, simultaneously from the same process, are known as joint products. For example, in the oil industry, gasoline, fuel oil, lubricants, paraffin, coal tar, asphalt and kerosene are all produced from crude petroleum. These are known as joint products. Co-Products - Joint products and co-products are used synonymously in common parlance, but strictly speaking a distinction can be made between two. Co-products may be defined as two or more products which are contemporary but do not emerge necessarily from the same material in the same process. For instance, wheat and gram produced in two separate farms with separate processing of cultivation, are the co-products. Similarly timber boards made from different trees are co-products. By-Products - These are defined as “products recovered from material discarded in a main process, or from the production of some major products, where the material value is to be considered at the time of severance from the main product.” Thus by-products emerges as a result of processing operation of another product or they are produced from the scrap or waste of materials of a process. In short a by-product is a secondary or subsidiary product which emanates as a result of manufacture of the main product. Examples of by-products are molasses in the manufacture of sugar, tar, ammonia and benzole obtained on carbonisation of coal and glycerine obtained in the manufacture of soap. Distinction between Joint-Product and By-Product - The main points of distinction as apparent from the definitions of Joint Products and By-Products are : (i) Joint products are of equal importance whereas by-products are of small economic value. (ii) Joint products are produced simultaneously but the by-products are produced incidentally in addition to the main products.

7.6.2 Apportionment of joint costs : Joint product costs occur in many industries such as : petroleum, oil refinery, meat-making, textiles, dairy, flour mill, saw mill and many other process industries and top management of business concerns require the Accountants to give their opinion for many managerial decisions such as to process

7.16

Page 421: 30510870 Cost Accounting and Financial Management

Method of Costing (II)

further or to sell at split-off stage. To answer this question they require apportionment of joint costs over different products produced. The main problem faced in the case of joint products/by-products is the apportionment of the total cost incurred upto the point of separation of joint products/or by products. For costs incurred after the split off point there is no problem, as these costs can be directly allocated to individual joint products or by-products. Thus the apportionment of joint costs over different products produced involve the following two cases. 1. When two or more products are simultaneously produced and there is by-product.

2. When there are both joint products and by-products.

7.6.3 Method of apportioning joint cost over joint products : Proper apportionment of joint cost over the Joint Products is of considerable importance, as this affects (a) Valuation of closing inventory; (b) Pricing of products; and (c) Profit or loss on the sale of different products. The commonly used methods for apportioning total process costs upto the point of separation over the joint products are as follows : (i) Physical unit method

(ii) Average unit cost method

(iii) Survey method

(iv) Contribution margin method

(v) Market value method : (a) At the point of separation (b) After further processing (c) Net realisable value. (i) Physical unit method - This method is based on the assumption that the joint products are capable of being measured in the same units. Accordingly joint costs here are apportioned on the basis of some physical base, such as weight or measure expressed in gallons, tonnes etc. In other words, the basis used for apportioning joint cost over the joint products is the physical volume of material present in the joint products at the point of separation. Any loss arising during the stage of processing is also apportioned over the products on the same basis. This method cannot be applied if the physical units of the two joint products are different. The main defect of this method is that it gives equal importance and value to all the joint products.

7.17

Page 422: 30510870 Cost Accounting and Financial Management

Cost Accounting Illustration A coke manufacturing company produces the following products by using 5,000 tonnes of coal @ Rs. 15 per tonne into a common process.

Coke 3,500 tonnes Tar 1,200 tonnes Sulphate of ammonia 52 tonnes Benzol 48 tonnes

Apportion the joint cost amongst the products on the basis of the physical unit method.

Solution Products Coke Tar Sulphate Benzol Wastage Total of Ammonia Output (in tonnes) 3,500 1,200 52 48 200 5,000 Wastage (in tonnes) 146 50 2 2 200 (apportioned on the basis of weights) Total weight (in tonnes) 3,646 1,250 54 50 5,000 Joint Cost (in Rs.) @ Rs. 15 per tonne 54,690 18,750 810 750 75,000

Note : 1. Apportionment of wastage of 200 tonnes over the four products is as follows:

Coke :4800200 × 3,500 tonnes = 146 tonnes

Tar : 4800200 × 1,200 tonnes = 50 tonnes

Sulphate of ammonia : 2 tonnes Benzol : 2 tonnes (ii) Average unit cost method - Under this method, total process cost (upto the point of separation) is divided by total units of joint products produced. On division average cost per unit of production is obtained. This is a simple method. The effect of application of this method is that all joint products will have uniform cost per unit. If this method is used as the basis for price fixation, then all the

7.18

Page 423: 30510870 Cost Accounting and Financial Management

Method of Costing (II)

products may have more or less the same price. Under this method customers of high quality items are benefitted as they have to pay less price on their purchase.

Illustration Find out the cost of joint products A, B and C using average unit cost method from the following data : (a) Pre-separation Joint Cost Rs. 60,000.

(b) Production data : Products Units produced A 500 B 200 C 300 1,000

Solution

Average cost per unit =produced Units

costsjoint Total = units 000,1000,60.Rs = Rs. 60

The joint costs apportioned @ Rs. 60 are as follows : Products Units Costs per unit Value A 500 Rs. 60 Rs. 30,000 B 200 Rs. 60 Rs. 12,000 C 300 Rs. 60 Rs. 18,000 Rs. 60,000 (iii) Survey method - This method is also known as point value method. It is based on technical survey of all the factors involved in the production and distribution of products. Under this method joint cost are apportioned over the joint products, on the basis of percentage/point values, assigned to the products according to their relative importance. The percentage or points used for the purpose are usually computed by management with the help of technical advisers. This method is considered to be more equitable than other methods.

(iv) Contribution margin method - According to this method, joint costs are segregated into two parts - variable and fixed. The variable costs are apportioned over the joint products on the basis of units produced (average method) or physical quantities. In case

7.19

Page 424: 30510870 Cost Accounting and Financial Management

Cost Accounting the products are further processed after the point of separation, then all variable cost incurred be added to the variable costs determined earlier. In this way total variable cost is arrived which is deducted from their respective sales values to ascertain their contribution. The fixed costs are then apportioned over the joint products on the basis of the contribution ratios.

Illustration Find out the cost of joint products A and B using contribution margin method from the following data : Sales A : 100 kg @ Rs. 60 per kg B : 120 kg @ Rs. 30 per kg Joint costs Marginal cost Rs. 4,400 Fixed cost Rs. 3,900 Solution The marginal cost (variable cost) of Rs. 4,400 is apportioned over the joint products A and B in the ratio of their physical quantity i.e 100 : 120

Marginal cost for Product A : Rs. 4,400×120100 = Rs. 2,000

Marginal cost for Product B : Rs. 4,400×220120 = Rs. 2,400

The fixed cost of Rs. 3,900 is apportioned over the joint products A and B in the ratio of their contribution margin i.e. 40 : 12 (Refer to working note)

Product A : Rs. 3,900 × 40/52 = Rs. 3,000

Product B : Rs. 3,900 × 12/52 = Rs. 900

7.20

Page 425: 30510870 Cost Accounting and Financial Management

Method of Costing (II)

Working Note : Computation of contribution margin ratio Products Sales revenue Marginal cost Contribution (Rs.) (Rs.) (Rs.) A 6,000 2,000 4,000 B 3,600 2,400 1,200 (Refer to above) Contribution ratio is 40 : 12 (v) Market value method - This is the most popular and convenient method because it makes use of a realistic basis for apportioning joint costs. Under this method joint costs are apportioned after ascertaining “what the traffic can bear”. In other words, the products are made to bear a proportion of the joint cost on the basis of their ability to absorb the same. Market value means weighted market value i.e. units produced × price of a unit of joint product.

(a) Market value at the point of separation - This method is used for the apportionment of joint costs to joint products upto the split off point. It is difficult to apply this method if the market value of the products at the point of separation are not available. It is a useful method where further processing costs are incurred disproportionately.

To determine the apportionment of joint costs over joint products, a factor known as multiplying factor is determined. This multiplying factor on multiplication with the sales values of each joint product gives rise to the proportion of joint cost. For example, a concern incurs a joint cost of Rs. 64,500 in production two products A (200 units), B (200 units) and earns a sales revenue of Rs. 86,000 by selling @ Rs. 170 per unit of product A and B @ Rs. 260 per unit of product B. The multiplying factor in this case is obtained by dividing the total joint cost by total sales revenue and finally multiplying the figure so obtained by 100. The multiplying factor based on the data can be computed as follows :

Multiplying factor : 000,86.Rs500,64.Rs × 100 = 75%

Joint cost apportioned over product A = Sales revenue of product A × 75% = 34,000 × 75% = Rs. 25,500

7.21

Page 426: 30510870 Cost Accounting and Financial Management

Cost Accounting Joint cost apportioned over product B = Sales revenue of product B × 75% = Rs. 52,000 × 75% = Rs. 39,000 Alternatively - This joint cost may be apportioned in the ratio of sales values of different joint products. (b) Market value after processing - Here the basis of apportionment of joint cost is the total sales value of finished products and involves the same principle as discussed in (a) above. Suppose that in the example given in Part (a) above, if sales prices of products A and B after further processing is Rs. 200 and Rs. 300 respectively the joint cost apportioned over Products A and B is as follows : The pre-separation costs of Rs. 64,500 will be apportioned in the ratio of (2 : 3) as follows: Market sales value after further processing Rs. A : 200 units × Rs. 200 = 40,000 B : 200 units × Rs. 300 = 60,000 1,00,000 Joint cost apportioned :

A = Rs. 64,500 × 000,00,1.Rs

000,40.Rs = Rs. 25,800

B = Rs. 64,500 × 000,00,1.Rs

000,60.Rs = Rs. 38,700

The use of this method is unfair where further processing costs after the point of separation are disproportionate or when all the joint products are not subjected to further processing. The net realisable value method which is discussed as below overcomes the shortcoming of this method. (c) Net realisable value method : From the sales value of the joint products (at finished stage) are deducted :

(i) estimated profit margins,

(ii) selling and distribution expenses, if any, and

(iii) post-split off costs.

7.22

Page 427: 30510870 Cost Accounting and Financial Management

Method of Costing (II)

The resultant figure so obtained is known as net realisable value of joint products. Joint costs are apportioned in the ratio of net realisable value. Suppose that in the example given in part (a) above if further processing costs for products A and B are Rs. 4,000 and Rs. 32,000 respectively the Joint cost may be apportioned to products A and B as follows: Products Sales Further Net realisable Joint cost revenue (Rs.) processing value apportioned cost (Rs.) (Rs.) ratio (a) (b) (c)=(a)–(b) A 34,000 4,000 30,000 3/5 B 52,000 32,000 20,000 2/5

Joint cost apportioned over product A = Rs. 64,500 × 3/5 = Rs. 38,700

Joint cost apportioned over product B = Rs. 64,500 × 2/5 = Rs. 25,800

This method is extensively used in many industries.

Illustration : Inorganic Chemicals purchases salt and processes it into more refined products such as Caustic Soda, Chlorine and PVC. In the month of July, Inorganic Chemicals purchased Salt for Rs. 40,000. Conversion of Rs. 60,000 were incurred upto the split off point, at which time two sealable products were produced. Chlorine can be further processed into PVC. The July production and sales information is as follows : Production Sales quantity Selling price (tonnes) (tonnes) (per tonnes) Caustic Soda 1,200 1,200 Rs. 50 Chlorine 800 — — PVC 500 500 Rs. 200 All 800 tonnes of Chlorine were further processed, at an incremental cost of Rs. 20,000 to yield 500 tonnes of PVC. There were no beginning or ending inventories of Caustic Soda, Chlorine or PVC in July.

7.23

Page 428: 30510870 Cost Accounting and Financial Management

Cost Accounting There is active market for Chlorine. Inorganic Chemicals could have sold all its July production of Chlorine at Rs. 75 per tonne. Required : (1) To calculate how joint cost of Rs. 1,00,000 would be apportioned between Caustic Soda

and Chlorine under each of following methods :

(a) Sales value at split off,

(b) Physical measure (method), and

(c) Estimated net realisable value.

(2) Lifetime Swimming Pool Products offers to purchase 800 tonnes of Chlorine in August at Rs. 75 per tonne. This sale of Chlorine would mean that no PVC would be produced in August. How the acceptance of this offer for the month of August would affect operating income ?

Solution : 1. (a) Sales value at split off method Products Sales in Selling price SalesJoint cost tonnes per tonne revenue apportioned* (Rs.) (Rs.) (Rs.) (a) (b) (c)=(a) × (b) Caustic Soda 1,200 50 60,000 50,000 Chlorine 800 75 60,000 50,000 1,20,000 1,00,000

* Apportioned joint cost = value sale Totalcostjoint Total × Sale revenue of each

product

Joint cost apportioned to Caustic Soda = 000,20,1.Rs000,00,1.Rs × Rs. 60,000 = Rs. 50,000

Joint cost apportioned to Chlorine = 000,20,1.Rs000,00,1.Rs × Rs. 60,000 = Rs. 50,000

7.24

Page 429: 30510870 Cost Accounting and Financial Management

Method of Costing (II)

(b) Physical measure method Products Sale in Joint cost (tonnes) apportioned **

Caustic Soda 1,200 60,000 Chlorine 800 40,000 2,000 1,00,000

**Apportioned joint cost = value physical Total

costjoint Total × Physical units of each product

Joint cost apportioned to Caustic Soda = tonnes 2000

000,00,1.Rs × 1,200 tonnes = Rs. 60,000

Joint cost apportioned to chlorine = tonnes 2000

000,00,1.Rs × 800 tonnes = Rs. 40,000

(c) Estimated net realisable value method Products Sale Further Net realisable Apportioned*** revenue processing value Joint cost (Rs.) cost (Rs.) (Rs.) (a) (b) (c) = (a) – (b) Caustic Soda 60,000 – 60,000 42,857 (1,200 tonnes × Rs. 50) Chlorine 1,00,000 20,000 80,000 57,143 (500 tonnes of PVC × Rs. 200) ________ _______ 1,40,000 1,00,000

**Apportioned joint cost = value realisablenet Total

costjoint Total × Net realisable value of each product

Apportioned joint cost for Caustic Soda = 000,40,1.Rs000,00,1.Rs × Rs. 60,000 = Rs. 42,857

Apportioned joint cost for Chlorine = 000,40,1.Rs000,00,1.Rs × Rs. 80,000 = Rs. 57,143

7.25

Page 430: 30510870 Cost Accounting and Financial Management

Cost Accounting 2. Incremental revenue from further processing of Chlorine into PVC Rs. 40,000 (500 tonnes × Rs. 200 – 800 tonnes × Rs. 75) Less : Incremental cost of further processing of Chlorine into PVC Rs. 20,000Incremental operating income from further processing Rs. 20,000

The operating income of Inorganic Chemicals will be reduced by Rs. 20,000 in August if it sells 800 tonnes of Chlorine to Lifetime Swimming Pool Products, instead of further processing of Chlorine into PVC for sale.

Illustration SUNMOON Ltd. produces 2,00,000 : 30,000; 25,000; 20,000 and 75,000 units of its five products A, B, C, D and E respectively in a manufacturing process and sells them at Rs. 17, Rs. 13, Rs. 8, Rs. 10 and Rs. 14 per unit. Except product D remaining products can be further processed and then can be sold at Rs. 25, Rs. 17, Rs. 12 and Rs. 20 per unit in case of A, B, C and E respectively. Raw material costs Rs. 35,90,000 and other manufacturing expenses cost Rs. 5,47,000 in the manufacturing process which are absorbed on the products on the basis of their ‘Net realisable value’. The further processing costs of A, B, C and E are Rs. 12,50,000; Rs. 1,50,000; Rs. 50,000 and Rs. 1,50,000 respectively. Fixed costs are Rs. 4,73,000. You are required to prepare the following in respect of the coming year:

(a) Statement showing income forecast of the company assuming that none of its products are to be further processed.

(b) Statement showing income forecast of the company assuming that products A, B, C and E are to be processed further.

Can you suggest any other production plan whereby the company can maximise its profits ? If yes, then submit a statement showing income forecast arising out of adoption of that plan.

7.26

Page 431: 30510870 Cost Accounting and Financial Management

Method of Costing (II)

Solution Working Note:

Statement showing apportionment of joint costs on net realisable value basis

Products Sales value Post Net realisable Apportioned separation value joint costs (1) (2) (1)–(2)=(3) (4) Rs. Rs. Rs. Rs. A 50,00,000 12,50,000 37,50,000 26,25,000 (2,00,000 units × Rs. 25) B 5,10,000 1,50,000 3,60,000 2,52,000 (30,000 units × Rs. 17) C 3,00,000 50,000 2,50,000 1,75,000 (25,000 units × Rs. 12) D 2,00,000 — 2,00,000 1,40,000 (20,000 units × Rs. 10) E 15,00,000 1,50,000 13,50,000 9,45,000 (75,000 units × Rs. 20) _________ ________ 59,10,000 41,37,000 Total joint cost = Raw material costs + Manufacturing expenses = Rs. 35,90,000 + Rs. 5,47,000 = Rs. 41,37,000

Apportioned joint cost = value realisablenet Total

costjoint Total × Net realisable value of each product

Apportioned joint cost for Product A = 000,10,59.Rs000,37,41.Rs × Rs. 37,50,000 = 26,25,000

Similarly, the approtioned joint cost for products B, C, D and E are Rs. 2,52,000; Rs. 1,75,000; Rs. 1,40,000 and Rs. 9,45,000 respectively.

7.27

Page 432: 30510870 Cost Accounting and Financial Management

Cost Accounting (a) Statement showing income forecast of the company assuming that none of its products are further processed

Products A B C D E Total Rs. Rs. Rs. Rs. Rs. Rs. Sales revenue 34,00,000 3,90,000 2,00,000 2,00,000 10,50,000 52,40,000 (2,00,000 (30,000 (25,000 (20,000 (75,000 units × units × units × units × units × Rs. 17) Rs. 13) Rs. 8) Rs. 10) Rs. 14) Less: Apportioned joint cost 26,25,000 2,52,000 1,75,000 1,40,000 9,45,000 41,37,000

(Refer to working note) Excess of revenue over joint cost of manufacturing 7,75,000 1,38,000 25,000 60,000 1,05,000 11,03,000 Less: Fixed cost 4,73,000 Profit 6,30,000 (b) Statement showing income forecast of the company: assuming that products A, B,

C and E are further processed (Refer to working note) Products A B C D E Total Rs. Rs. Rs. Rs. Rs. Rs. Sales revenue : (X) 50,00,000 5,10,000 3,00,000 2,00,000 15,00,000 75,10,000 Apportioned joint cost : (Y) 26,25,000 2,52,000 1,75,000 1,40,000 9,45,000 41,37,000 Further processing cost : (Z) 12,50,000 1,50,000 50,000 - 1,50,000 16,00,000 Total manufacturing cost : (K)=(Y)+(Z) 38,75,000 4,02,000 2,25,000 1,40,000 10,95,000 57,37,000 Excess of sales revenue over total manufacturing 11,25,000 1,08,000 75,000 60,000 4,05,000 17,73,000 cost : [(X)–(K)] Less: Fixed cost 4,73,000Profit 13,00,000

7.28

Page 433: 30510870 Cost Accounting and Financial Management

Method of Costing (II)

Suggested production plan for maximising profits On comparing the figures of excess of revenue over cost of manufacturing in the above statements one observes that the concern is earning more after further processing of A, C and E products but is loosing a sum of Rs. 30,000 in the case of product B (if it is processed further). Hence the best production plan will be to sell A, C and E after further processing and B and D at the point of split off. The profit statement based on this suggested production plan is as below :

Profit statement based on suggested production plan Products A B C D E Total Rs. Rs. Rs. Rs. Rs. Rs. 1 2 3 4 5 6 7 Sales revenue : (X) 50,00,000 3,90,000 3,00,000 2,00,000 15,00,000 73,90,000 Appointed joint cost : (Y) 26,25,000 2,52,000 1,75,000 1,40,000 9,45,000 41,37,000 Further processing cost : (Z) 12,50,000 - 50,000 - 1,50,000 14,50,000 Total manufacturing cost : (K)=(Y)+(Z) 38,75,000 2,52,000 2,25,000 1,40,000 10,95,000 55,87,000 Excess of sales revenue over manufacturing cost 11,25,000 1,38,000 75,000 60,000 4,05,000 18,03,000 : [(X)–(K)] Less: Fixed cost 4,73,000Profit 13,30,000 Hence the profit of the company has increased by Rs. 30,000. 7.6.4 Methods of apportioning joint cost over by-products : The following methods may be adopted for the accounting of by-products and arriving at the cost of production of the main product :

(a) Market value or value on realisation : The realisation on the disposal of the by-product may be deducted from the total cost of production so as to arrive at the cost of the main product. For example, the amount realised by the sale of molasses in a sugar factory goes to reduce the cost of sugar produced in the factory. When the by-product requires some additional processing and expenses are incurred in making it saleable to the best advantage of the concern, the expenses so incurred should be deducted from the total value realised from the sale of the by-product and only the net

7.29

Page 434: 30510870 Cost Accounting and Financial Management

Cost Accounting realisations should be deducted from the total cost of production to arrive at the cost of production of the main product. Separate accounts should be maintained for collecting additional expenses incurred on : (i) further processing of the by-product, and

(ii) selling, distribution and administration expenses attributable to the by-product.

(b) Standard cost in technical estimates : By-products may be valued at standard costs. The standard may be determined by averaging costs recorded in the past and making technical estimates of the number of units of original raw material going into the main product and the number forming the by-product or by adopting some other consistent basis. This method may be adopted where the by-product is not saleable in the condition in which it emerges or comparative prices of similar products are not available.

(c) Comparative price : Under this method, the value of the by-product is ascertained with reference to the price of a similar or an alternative material. Suppose in a large automobile plant a blast furnace not only produces the steel required for the car bodies but also produces gas which is utilised in the factory. This gas can be valued at the price which would have been paid to a gas company if the factory were to buy it from outside sources.

(d) Re-use basis : In some cases the by-product may be of such a nature that it can be reprocessed in the same process as part of the input of the process. In that case the value put on the by-product should be same as that of the materials introduced into the process. If, however, the by-product can be put into an earlier process only, the value should be the same as for the materials introduced into the process.

7.6.5 Treatment of By-Product Cost in Cost-Accounting : By-product cost can be dealt in cost accounting in the following ways :—

(i) When they are of small total value : When the by-products are of small total value, the amount realised from their sale may be dealt in any one the following two ways :

1. The sales value of the by-products may be credited to the Profit and Loss Account and no credit be given in the Cost Accounts. The credit to the Profit and Loss Account here is treated either as miscellaneous income or as additional sales revenue.

2. The sale proceeds of the by-product may be treated as deductions from the total costs. The sale proceeds in fact should be deducted either from the production cost or from the cost of sales.

7.30

Page 435: 30510870 Cost Accounting and Financial Management

Method of Costing (II)

(ii) When the by-products are of considerable total value : Where by-products are of considerable total value, they may be regarded as joint products rather than as by-products. To determine exact cost of by-products the costs incurred upto the point of separation, should be apportioned over by-products and joint products by using a logical basis. In this case, the joint costs may be divided over joint products and by-products by using relative market values ; physical output method (at the point of split off) or ultimate selling prices (if sold).

(iii) Where they require further processing : In this case, the net realisable value of the by-product at the split-off point may be arrived at by subtracting the further processing cost from the realisable value of by-products. If total sales value of by-products at split-off point is small, it may be treated as per the provisions discussed above under (i). In the contrary case, the amount realised from the sale of by-products will be considerable and thus it may be treated as discussed under (ii). (Students must solve a large number of questions from process costing so as to acquire the required proficiency in the area).

7.7 Self Examination Questions

Multiple choice questions

(i) When FIFO method is used in process costing, the opening stock costs are:

(a) kept separate from the costs of the new period

(b) added to new costs

(c) subtracted from the new costs

(d) averaged with other costs to arrive at total costs. (ii) When average method is used in Process Costing, the opening inventory costs are :

(a) kept separate from the costs of the new period

(b) added to new costs

(c) subtracted from the new costs

(d) averaged with other costs to arrive at total costs. (iii) Popular methods for calculating equivalent production are

(a) FIFO

7.31

Page 436: 30510870 Cost Accounting and Financial Management

Cost Accounting

(b) Average cost

(c) Both 1 and 2

(d) Neither 1 nor 2

(iv) When average method is used in process costing , the opening inventory costs are

(a) Kept separate from the costs of the new period

(b Added to new costs

(c) Subtracted from the new costs

(d) Averaged with other costs to arrive at total costs (v) When FIFO method is used in Process costing , the opening stocks are

(a) Kept separate from the costs of the new period

(b) Added to new costs

(c) Subtracted from the new costs

(d) Averaged with other costs to arrive at total costs

(vi) The output of a process consists of two joint products, Jointpro A and Jointpro B, and a by-product. Jointpro B could go through a further process in order to increase its sales value. To assist management in making the decision whether to carry out further processing, which ONE of the following is relevant ?

(a) The share of the total processing cost which has been allocated to Jointpro B

(b) The sales value of Jointpro A and the by-product

(c) The physical quantities of all three products at separation point

(d) The cost of further processing Jointpro B and the increase in sales value that will result

(vii) When compared with normal spoilage, abnormal spoilage

(a) arises more frequently from factors that are inherent in the manufacturing process.

(b) is generally thought to be more controllable by production management than normal spoilage

(c) is given the same accounting treatment as normal spoilage.

7.32

Page 437: 30510870 Cost Accounting and Financial Management

Method of Costing (II)

(d) is not typically influenced by the "tightness" of production standards.

(viii) Which of the following statements is true,

(a) Process costing is ordinarily applied where all the operations are performed in one department.

(b) Equivalent unit or equivalent production comprises the units completed during the period together with equivalent completed units, represented in the beginning and ending WIP inventories.

(c) Job Costing and process costing cannot be simultaneously used in the same industry.

(d) The cost of abnormal process loss is not included in the cost of the process.

(ix) Which of the following statements is true,

(a) In process costing, ordinarily no distinction is made between direct and indirect materials.

(b) There is no difference between the terms ‘co-products’ and ‘joint products’.

(c) Operating costing is the same as operation costing

(d) Neither a nor b nor c

(x) The main purpose of accounting of joint products, and by-products is to,

(a) Determine profit or loss on each product line

(b) Determine the selling price .

(c) Comply with the statutary requirements

(d) Identify the cost and load it on the main product

Answers to multiple choice questions

i.(a); ii.(c); iii. (c); iv. (c); v. (a); vi.(d); vii. (b); viii. (b); ix.(a); x. (a)

Short answer type questions

1. Define and explain the terms “Joint Products” and “By-products”. Enumerate the methods which may be employed in costing joint products.

2. Explain briefly the methods of accounting for by-products.

3. Distinguish between :

7.33

Page 438: 30510870 Cost Accounting and Financial Management

Cost Accounting

(i) Joint product and By-product

(ii) Abnormal loss and abnormal gain. 4. How would you deal with by-products in costing : (i) Where they are of small total value? (ii) Where they are of considerable total value? (iii) Where they require further processing? 5. Describe briefly the various ways of valuing work-in-progress in process costing.

Long answer type questions

1. Describe the general features of process costing. How is unit cost determined in process costing ?

2. What is meant by operation costing? Is it different from process costing ?

3. What do you understand by the terms, “normal process loss” and “abnormal process loss”? How is the value of abnormal process loss determined?

4. What is meant by abnormal gain in process costing? How is it dealt with in Cost Accounting?

5. What do you understand by the term “inter-process profits”? What is the utility of transferring the output of one process to another process at more than cost?

Numerical questions

1. A product passes through three processes, A, B, and C. The normal wastage of each process is as follows : Process A-3%m Process B-5% and Process C-8%.

The wastage of process A was sold at 0.25 p. per unit, that of process B at 0.50 p. per unit, and that of process C, at Re. 1 per unit. 10,000 units were issued to the Process A in the beginning of October 1998 at a cost of Re. 1 per unit. The other expenses were : Process A Process B Process C Sundry materials Rs. 1,000 Rs. 1,500 Rs. 500 Labour 5,000 8,000 6,500 Direct expenses 1,050 1,188 2,000 Actual output (units) 9,500 9,100 8,100

7.34

Page 439: 30510870 Cost Accounting and Financial Management

Method of Costing (II)

Prepare the Process accounts, assuming that there were no opening or closing stocks. Also give the Abnormal Wastage and the Abnormal Gain Accounts.

2. In a manufacturing unit, raw materials passes through four processes I, II, III & IV and the output of each process is the input of the subsequent process. The loss in the four processes I, II, III & IV are respectively 25%, 20% and 16-2/3% of the input. If the end product at the end of Process IV is 40,000 Kg. what is the quantity of raw material required to be fed at the beginning of Process I and the cost of the same at Rs. 5 per Kg.?

Find out also the effect of increase or decrease in the material cost of the end product for variation of every rupee in the cost of raw material.

3. In a certain period 300 units of main product are produced and 200 units are sold at Rs. 30 per unit. The by-product emerging from the main product is sold at Rs. 600. The total cost of production of 300 units is Rs. 4,500. Calculate the amount of gross profit after crediting by-product value (a) to the cost of production and (b) to cost of sales.

4. The following information is given about two products produced jointly upto a stage. Joint cost (Rs) 40,000 Number of units of Product A 2,000 B 600 Selling price (Rs.) A 30 B 25 Special (separate) expenses (Rs.) A 8,000 B 3,000 Ascertain the profit earned in total and by each product.

5. A factory producing article P also produces a by-product Q which is further processed into a finished product. The cost of manufacture is given below :

Subsequent Cost Joint Costs (Rs.) P (Rs.) Q (Rs.) Material 5,000 3,000 1,500 Labour 3,000 1,400 1,000 Overheads 2,000 600 500 10,000 5,000 3,000 Selling Price Are P : Rs. 16,000, and Q : Rs. 8,000

7.35

Page 440: 30510870 Cost Accounting and Financial Management

Cost Accounting

Estimated profit on selling prices are 25% for P and 20% for Q. Assume that selling and distribution expenses are in proportion of the sale prices. Show how you would apportion joint costs of manufacture and prepare a statement showing cost of production of P and Q.

6. X Ltd. manufactures product A, which yields two by-products B and C. In a period, the amount spent upto the point of separation was Rs. 20,000. Subsequent expenses were : A B C Rs. Rs. Rs. Materials 250 180 60 Direct wages 500 300 120 Overheads 840 300 180Total 1,100 780 360 Gross sale value 15,200 10,000 5,100 It was estimated that profit as a percentage of sales of B and C would be 25% and 15% respectively. Ascertain the profit earned on A.

7.36

Page 441: 30510870 Cost Accounting and Financial Management

CHAPTER 8

STANDARD COSTING

Learning Objectives When you have finished studying this chapter you should be able to; ♦ Understand the meaning of standard cost and variances. ♦ Understand the concept of capacity with reference to a product cost sheet. ♦ Understand the difference between controllable and uncontrollable variances. ♦ Compute variances related to material, labour, overhead and sales. ♦ Understand the reporting pattern which may be adopted for control and decision making

purposes. ♦ Understand the meaning of disposition of variances. ♦ Understand the advantages and disadvantages of standard costing and variance analysis.

8.1 INTRODUCTION Standard costing is defined by the ICMA, London, “as the preparation and use of standard costs, their comparison with actual costs and the analysis of variances to their causes and points of incidence.” Standard costing, thus, is a system of costing which can be used in conjunction with any method of costing, like job costing, process costing, etc. The evolution of standard costing can be traced back to the late nineteenth century when Frederick W. Taylor (1856-1915) and Frank Gilbreth (1869–1924) demonstrated the concept of the time and motion study. Accountants were quick to grab on the fundamental notion that one activity can be performed in one best way. They converted such an estimate of time required to complete a job into money by multiplying it by the prevailing labour rates. Thus the concept of standard labour rate to do a job was established. Material consumption standards were easy to establish as the engineering function would know how much quantity would be required to manufacture one unit of finished goods and this when multiplied by the estimated purchase price would provide for the material cost details. Product manufacturing costs computed then were typically characterised by simplistic assumptions, with the use of ‘blanket’ overhead rates like simple labour overhead recovery bases being the common practice. It may be noted that, during the times of the evolution of standard costing and variance analysis, the classification of overheads as variable and fixed was not prevalent. Expenses then were either

Page 442: 30510870 Cost Accounting and Financial Management

Cost Accounting Direct or Indirect with raw material and the majority of labour being in the former category and the balance known as overheads (refer to chapter 1 for a clear understanding). However, standard costing was quick to imbibe the subsequent introduction of variability and fixed concepts which happened during the 1940s. The successful establishment of estimates or standards relating to the raw material cost, labour cost and the overhead burden provided all information related to product cost. Look at the following estimate of cost prepared in a tabular form;

Estimated Standard Cost in Rs. Capacity : --------- units

Description Total Cost

Total Fixed Cost

Total Variable

Cost

Fixed Cost per

unit

Variable Cost per

unit

Total Cost

per unit Bill of Material Cost xxx ------- xxx ------- xxx xxx Direct Labour xxx ------- xxx ------- xxx xxx Manufacturing overheads

xxx xxx xxx xxx xxx xxx

Marketing and administrative overheads

xxx xxx xxx xxx xxx xxx

Total cost per unit xxx xxx xxx Add desired profit xxx Desired selling price/unit

xxx

The bill of material cost refers to the raw material cost which goes into the making of one unit of finished good. This cost is Direct in nature as it can be traced economically with the cost object i.e., the finished product. The same can be said for direct labour. The other two expenses i.e. manufacturing and marketing/administration are considered as overheads since they do not bear a direct relationship with the cost object. Note that these overheads shall need to be computed considering the capacity utilization which is possible within the given infrastructure. Hence, in case a company produces below its capacity the cost incidence/unit of these overheads shall increase and be more than what has been estimated as standard. In other words the total estimated standard fixed cost (overheads) shall not be fully utilised, thereby leading to an unutilised portion (variance). This variance is known as production volume variance (for further details please refer to the discussion under ‘Types of variances’). Deviations from estimates can also occur in the other estimated costs i.e.

8.2

Page 443: 30510870 Cost Accounting and Financial Management

Standard Costing material cost, labour cost and the estimated selling price. You shall learn more about such variances as you proceed. However, companies, many a times, do not consider the total capacity available as the level of activity to be considered for building a cost estimate. Different types of capacities may be considered. These shall lead to different estimates of Product costs. Why? Let us consider a simple example of a product whose material cost is estimated to be Rs. 10 per unit and the direct labour shall amount to Rs. 5 per unit. Overheads are estimated to be Rs. 1,00,000 per month. These overheads are fixed in nature and can

support a production of 20,000 units. Hence the incidence of overhead cost is20,0001,00,000 Rs =

Rs. 5 per unit and the resultant estimated standard cost of the product is Rs. 20 (the sum of the standard material, standard labour and standard overhead costs). However, in case the capacity considered is only 10,000 units, the estimated /standard cost shall go up to Rs. 25. (Rs. 10, raw material + Rs. 5, labour + Rs. 10, overheads).

8.2 DEFINITION OF STANDARD COST Standard cost is defined “as a pre-determined cost which is calculated from management’s standards of efficient operation and the relevant necessary expenditure. It may be used as a basis for price fixing and for cost control through variance analysis.” The standard cost of a product has been defined by Blocker and Weltmer “as a pre-determined cost based upon engineering specification and representing highly efficient production for quantity standards and forecasts of future market trends for price standards with a fixed amount expressed in dollars for material, labour and overhead for an estimated quantity of production.” It may be seen from this definition that engineering specifications are the basis for quantity standards for materials and time standards for labour while budgets are of importance in determining material price standards, labour rate standards and overhead standards.

8.3 SETTING UP OF STANDARD COST A standard cost by definition is an estimate correlating a technical specification of materials and labour to the prices and wage rates with the addition of overhead for a prescribed level of output. It is thus a measure in quantities, hours and value of the factors of production. There are three main parts of standard costs, viz., (a) direct material, (b) direct labour, and (c) overhead expenses. The compilation of direct material and direct labour standards entails the setting of the following: 8.31 Physical standards: The first step in manufacturing a product is to determine standard quantity of different materials to be used under the manufacturing process; various sub-assemblies, components, other small parts, etc. Besides this the length of time an average worker should take to complete a job. These standards so determined are known as physical

8.3

Page 444: 30510870 Cost Accounting and Financial Management

Cost Accounting standards. The purposes of setting physical standards are: (a) To secure economies in manufacture, and (b) Set selling prices in advance to make it possible to estimate the cost. In printing industry, for example, the standards relating to the printed output are necessary in submitting quotations, for proposed jobs. In manufacturing organisations, the task of setting physical standards is assigned to the industrial engineering department. Physical standards of manufacturing activity refer to (a) specification of products and materials, (b) method of manufacture, (c) equipment to be used. Standards must always be thought of primarily in terms of specific things, as for example, units of time, kilograms of materials, hours of plant capacity, and units of output. The fundamental principle is that if the standards are to be of optimum utility, they should be capable of comparison with actual happenings on a physical unit basis at the primary levels. The fundamental or primary standards determined in terms of units of measurement or physical things need not be changed unless operating methods or products changes. Although it is not the duty of a cost accountant to set physical standards, they are the bases for standard costs. He must be satisfied with regard to the following points: (a) Ensuring that adequate inspection of materials and parts has been carried out. If the inspection does not adequately localise faulty work, the cost accountant may have a hard time while explaining his variance reports. (b) He is notified of every change in physical standards made by engineers so as to enable him to revise his costs as and when such changes are introduced. Material quantity standards: The following procedure is usually followed for setting material quantity standards. (a) Standardisation of products: Detailed specifications, blueprints, norms for normal wastage etc., of products along with their designs are settled. (b) Product classification: Detailed classified list of products to be manufactured are prepared. (c) Standardisation of material: Specifications, quality, etc., of materials to be used in the standard products are settled. (d) Preparation of bill of materials: A bill of material for each product or part showing the symbol or code, description and quantity of each material to be used is prepared. (e) Test runs: Sample or test runs under regulated conditions may be useful in setting quantity standards in a precise manner.

8.4

Page 445: 30510870 Cost Accounting and Financial Management

Standard Costing Labour quantity standards: The following are the steps involved in setting labour quantity standards: (a) Standardisation of product, as explained above. (b) Product classification, as defined earlier. (c) Standardisation of methods: Selection of proper machines to use proper sequence and method of operations. (d) Manufacturing layout: A plan of operation for each product listing the operations to be performed is prepared. (e) Time and motion study is conducted for selecting the best way of completing the job or motions to be performed by workers and the standard time which an average worker will take for each job. (f) The operator is given training to perform the job or operations in the best possible manner. Problems faced while setting physical standards: The problems involved while setting physical standards will vary from industry to industry and may be illustrated as under: (a) A situation may arise where the company is introducing the manufacture of a new line of product. In such cases, it may be necessary to employ workers who have no experience in the job. This creates a problem of setting standard time because it is necessary to make adjustment for the inexperience of workers. (b) Changes in technology may necessitate installation of sophisticated machines. When such machines are installed, the precise estimation of output and standard of efficiency achievable will pose a problem until after a long time when the working conditions are settled. Thus, setting standards for these machines and estimating the standard costs will need considerable amount of work. (c) Often manufacturers go in for product diversification to improve profitability. One of the most important problems that arise with the proposed change in product may be retooling. For example, when an old copper part is to be changed into one made of bronze to suit the new product, special care has to be taken to order new tools which in turn, pose the problem of setting up of standard time in respect of the new tools. (d) Standards of material specifications are established and if the materials are not available as per specifications, the standards may not be achievable. (e) In any engineering factory using punch press operations, the problem of the most economic way of punching discs from metallic rolls arises. The problem actually is how the raw material shall be used with least waste and computing the maximum amount of material that will be allowed for a unit of product. Consider, for example, the most economical way to

8.5

Page 446: 30510870 Cost Accounting and Financial Management

Cost Accounting blank 2-1/6" round diaphragms from a coil of sheet alloy 2" wide. (f) Very often the cost accountant is confronted with the problem of choosing the type of standards to be adopted. For example, the industrial engineer has furnished the standard time for all direct labour operations as under: 1. Standard time attainable by the best operations is 2 hours per unit of product including

allowances for personal fatigue and delay.

2. Attainable good performance for the average trained operator is 2.10 hours per unit of product.

3. Average past performance is 2.60 hours per unit. The problem is, should direct labour standard hour be based on maximum efficiency or attainable good performance or average past performance? If costs are to represent maximum efficiency, the unit cost used in selling price will relatively be low but a high debit variance may arise if the standard efficiency is not achieved. If, however, the standard cost is based on attainable good performance, the variances may tend to be nil. If efficiency is to be gauged, maximum efficiency standard will reflect the off standard performance, there by enabling the departmental head to exercise control. Similar problems as those mentioned above, may also arise in setting of waste standards. For example, the question may arise as to whether only absolutely unavoidable wastage should be provided or the past average level of wastage may be provided. This will again have different impact on the standard cost of production. 8.32 Price or Rate standards: Broadly, the price or rate standards can be set on either of the following bases: (a) Actual average or mean price expected to prevail during the coming period, say one

year; or

(b) Normal prices expected to prevail during a cycle of seasons which may be of a number of years.

Material price standards: Material prices are not altogether within the control of the manufacturer; but the purchasing department, on being apprised of production quantities required, should be able, from its knowledge of current market conditions and trends, to state with reasonable accuracy price for the constituent items. The standards for prices of materials should be based on the following factors, if price fluctuations are small and are not serious. (a) Stock of materials on hand and the prices at which they are held;

8.6

Page 447: 30510870 Cost Accounting and Financial Management

Standard Costing (b) The prices at which orders for future deliveries of materials have already been placed

and

(c) Anticipated fluctuation in price levels. In case there are violent fluctuations in the market price, it may be difficult to determine standard costs of materials; fluctuations in the market price may be of different sorts; prices may be different from month to month, from one season to another or from one year to another. Also, along with prices going up sometimes and coming down at other time, there may be a secular trend which, on the whole, is pushing price upwards or downwards. The nature of difficulties encountered in fixing standard costs of materials will naturally be different in each case. In addition the purchasing policy of the company and the objective to be achieved (from cost accounting) will make a difference. The difficulty in determining the standard cost of material in such a situation can be resolved as follows: (a) In case prices fluctuate from month to month, the average of prices of a year corrected

for the known secular changes and any other expected change can very well serve as the standard price for the next year.

(b) If the fluctuations are seasonal, but the whole year’s requirements are purchased at one time, the weighted average of the likely prices to be paid should be treated as the standard price. But, if buying is also spread over the whole year, the weighted average of the prices for the whole year should be the standard price.

(c) If prices fluctuate from one year to another, a careful estimate of the price likely to prevail next year, based on a statistical study, should be adopted as the standard price.

Wage rate standard: Standard wage rates may be set to cover various grades of labour. In factories ‘where contracts with workers’ unions exist, the rates approved by contract usually become the standard for the period for which the contract applies. Overhead expense standards: In computing the overhead expense standards, consideration should be given to the level of output and the expenses budgeted. A budget showing the level of output to be considered for arriving at overhead expense standards may be based on the practical manufacturing capacity or the average sales capacity or the budgeted capacity to be utilised in the coming year. After having chosen one of the methods of output level, the expenses can be budgeted under different heads under what the management calls good performance for the level of output chosen. These expenses are classified under fixed and variable categories. Thus, the overhead expenses standards are set by computing the optimum level of output for the production departments and thereafter drafting a budget for fixed and variable expenses

8.7

Page 448: 30510870 Cost Accounting and Financial Management

Cost Accounting which will be incurred at this level. If production is seasonal or it fluctuates during the year, a flexible budget may be prepared to facilitate comparison between the target set and the actual expenditure for the period. Standard hour: In industries like coal mining, where the products are homogeneous, the calculation of output is relatively simple. But in concerns manufacturing several products it is difficult to establish a satisfactory basis on which to measure output of the various departments because of differences in volume of individual products, quality, etc. The most satisfactory method of common measure is the use of standard hour. ICMA, London, defines a standard hour as a hypothetical hour, which measures the amount of work which should be performed in one hour. The standard hour is thus a unit of work and not of time. For example, a furniture company producing chairs, desks, tables and cabinets, may show the work content of each type of product measured in standard hours as under: Product Std. Hours per unit Quantity Produced Output in Std. hours Chairs 12 100 1,200 Desk 25 25 625 Tables 24 50 1,200 Cabinets 20 100 2,000

8.4 TYPES OF STANDARDS The accuracy and relevance of an established standard cost depends upon the reliability of the standards set up. In order to compute the standards we must know what degree of accuracy is necessary. There are four different bases or standards which should be considered. These standards have been discussed below: 8.41 Ideal standards: These represent the level of performance attainable when prices for material and labour are most favourable, when the highest output is achieved with the best equipment and layout and when the maximum efficiency in utilisation of resources results in maximum output with minimum cost. These type of standards are criticised on three grounds: (i) Since such standards would be unattainable, no one would take them seriously.

(ii) The variances disclosed would be variances from the ideal standards. These would not, therefore, indicate the extent to which they could have been reasonably and practically avoided.

(iii) There would be no logical method of disposing of these variances. 8.42 Normal standards: These are standards that may be achieved under normal operating conditions. The normal activity has been defined as “the number of standard hours which will

8.8

Page 449: 30510870 Cost Accounting and Financial Management

Standard Costing produce at normal efficiency sufficient goods to meet the average sales demand over a term of years”. These standards are, however, difficult to set because they require a degree of forecasting. The variances thrown out under this system are deviations from normal efficiency, normal sales volume, or normal productive volume. If the actual performance is found to be abnormal, large variances may result and necessitate revision of standards. 8.43 Basic or Bogey standards: These standards are used only when they are likely to remain constant or unaltered over a long period. According to this standard, a base year is chosen for comparison purposes in the same way as statisticians use price indices. Since basic standards do not represent what should be attained in the present period, current standards should also be prepared if basic standards are used. Basic standards are, however, well suited to businesses having a small range of products and long production runs. Basic standards are set, on a long-term basis and are seldom revised. When basic standards are in use, variances are not calculated as the difference between standard and actual cost. Instead, the actual cost is expressed as a percentage of basic cost. The current cost is also similarly expressed and the two percentages are compared to find out how much the actual cost has deviated from the current standard. The percentages are next compared with those of the previous periods to establish the trend of actual and current standard from basic cost. 8.44 Current standards: These standards reflect the management’s anticipation of what actual costs will be for the current period. These are the costs which the business will incur if the anticipated prices are paid for the goods and services and the usage corresponds to that believed to be necessary to produce the planned output. The variances arising from expected standards represent the degree of efficiency in usage of the factors of production, variation in prices paid for materials and services and difference in the volume of production.

8.5 NEED FOR STANDARD COSTS Since standard costs are pre-determined costs computed before the production takes place, they are preferable to actual costs. Moreover, certain conditions resulting from mass production make standard costs necessary and strongly advisable. Some of such conditions are: (a) Historical costs may be too expensive to compute. For example, in a manufacturing concern producing about 1,00,000 parts, divided into various lots, imagine the time and clerical labour involved in arriving at the actual cost lot by lot and then averaging it to determine the cost per unit. (b) The unit costs computed on historical data may vary from day to day and they are of no use to the sales department in setting selling prices. For example, if the historical costs per unit of product in a week are Rs. 1.05, 0.99, 1.27, 1.18, 1.42, 1.56, the selling price cannot be varied from day to day to match the costs.

8.9

Page 450: 30510870 Cost Accounting and Financial Management

Cost Accounting (c) Historical costs are not known until after the completion of a month or even a longer period. But in many cases, to take a decision, the cost of a product has to be calculated even before the production begins. (d) Historical costs may not be adequate for the measurement of efficiency. Standard costs are well suited for measuring operating efficiency because they represent what the costs should be. The management, consequently, knows immediately whether the performance is satisfactory. 8.51 Uses of Standard Costs 1. Use of standard costs is an effective way for planning and controlling costs. 2. Pricing decisions and decisions involving submission of quotations, answering tenders etc., are also facilitated by the use of standard costs. 3. Identification and measurement of variances from standards has been made possible with the use of standard cost, with a view to improve performance or to correct loose standards, if any. 4. Facilitates management by exception.

8.6 THE PROCESS OF STANDARD COSTING Standard costs are pre-determined by using a careful analysis of production methods, physical conditions and price factors. They represent achievable targets and help to build up budgets, gauge performance and obtain product costs. The actual costs will vary from month to month or even from day to day. The basic objective, therefore, of standard costing system is to assist the departmental head by identifying and describing the variances over which he has control. Thus, a set of standards developed under the standard costing system outlines how a task must be accomplished and how much it should cost. As work is done actual costs are recorded and compared with standard cost to determine the variances. The variances, thus arrived at, are analysed further with a view to discovering better ways of adhering to standards or of altering the standards so as to accomplish the objectives. Under this system, the cost is pre-determined for each element, namely, material, labour and overhead and for each line of product manufactured or service rendered. It, therefore, involves: (a) The setting of standards,

(b) Ascertainment of actual costs,

(c) Comparison of actual and standard costs to determine the variance, and

(d) Investigation of variances and taking appropriate action thereon wherever necessary.

8.10

Page 451: 30510870 Cost Accounting and Financial Management

Standard Costing 8.7 TYPES OF VARIANCES: 8.71 Controllable and un-controllable variances: The purpose of the standard costing reports is to investigate the reasons for significant variances so as to identify the problems and take corrective action. Variances are broadly of two types, namely, controllable and uncontrollable. Controllable variances are those which can be controlled by the departmental heads whereas uncontrollable variances are those which are beyond their control. For example, price variance is normally regarded as uncontrollable if the price increase is due to fluctuations of prices in the market. It becomes controllable if the production controller has failed to place orders in time and urgent purchase was made at extra cost. In the former case, no responsibility is attached to any one whereas the departmental head has responsibility for the loss in the latter case. As already explained, not all price variances are uncontrollable. If the uncontrollable variances are of significant nature and are persistent, the standard may need revision. Variances may be broadly classified under the following heads according to the main type of cost. (a) Material – The two basic variances arising during material consumption are material usage and material price variances. The former arises because of variations in the quantity of material actually consumed when compared with what should have been consumed as per the established standards and the latter because of the differences between the planned and the actual material prices paid to the suppliers. Mathematically Material usage variance = Std. price (Std. Qty. – Acutal qty.)

Material price variance = Actual qty. (Std. price – Actual price) (b) Labour – Similar to material usage variance, labour efficiency variance measures the efficiency of labour by identifying the difference between the actual hours worked and the hours which should have been worked as per the established standards. Deviations in the actual rate of pay and the ones estimated are measured by the labour rate variance. Mathematically Labour efficiency variance = Std. rate (Std. time – Actual time)

Labour rate variance = Actual time (Std. rate – Actual rate) (c) Overheads – Normally, for several type of overhead expenses either a single recovery rate or two recovery rates, one representing fixed overheads and the other representing variable overheads, will be prepared. Refer to the ‘Estimated Standard Cost’ table on page 8,2. Overheads have been classified as both fixed and variable thereby giving a standard fixed cost (overhead) per unit and standard variable cost (overhead) per unit. The recovery of the fixed components of the estimated overheads depends upon capacity utilization. In case a

8.11

Page 452: 30510870 Cost Accounting and Financial Management

Cost Accounting company produces less than the projected utilization it shall not be able to recover all the budgeted fixed overheads. This unrecovered portion is known as production volume variance. The other variation is because of variations in actual spending when compared with both estimated fixed and estimated variable overheads. Such a variance is known as Overhead expenses variance. The following detailed discussion shall help you have a clear understanding of these two variances. (1) Production Volume Variance: The term fixed overheads implies that the element of cost does not vary directly in proportion to the output. In other words fixed overheads do not change within a given range of activity. However the unit cost changes even though the fixed overheads are constant in total within the given range of output. So, higher the level of activity, the lower will be the unit cost or vice versa. The management is, therefore, faced with a costing difficulty because it requires a representative rate for charging fixed overheads irrespective of changes in volume of output. For example, if the fixed overheads are Rs. 10,000 and the output varies from 8,000 to 11,000 units, the cost per unit of output would be as under : Fixed Output in Cost per Overheads (Rs.) units unit of output (Rs.) 10,000 8,000 1.25 10,000 9,000 1.11 10,000 10,000 1.00 10,000 11,000 0.91 We have, however, seen that in standard costing, a predetermined rate of overhead recovery is established for costing purposes. This involves the establishment of a predetermine capacity. If we take, for example; 10,000 units as predetermine volume/capacity, the pre-determined rate will be Re.1 per unit. If the factory produces only 8,000 units, there will be a loss due to under-recovery which can be explained in two-ways: (a) The actual cost will be Rs. 10,000 ÷ 8,000 units = Rs. 1.25 per unit whereas the absorbed cost is Re. 1 per hour. Since the cost is more by Re. 0.25 per unit, the total loss is 8,000 units × Re. 0.25 or Rs. 2,000. (b) Since the factory has produced only 8,000 units, the amount of overheads recovered is 8,000 units × Re.1 or Rs. 8,000. Since fixed overheads are constant, the amount which should have been ideally incurred for the department is Rs.10,000. Hence there is a difference of Rs 2,000 between the overheads recovered and the overheads estimated. This variance is known as production volume variance. This shows the cost of failure on the part of the factory

8.12

Page 453: 30510870 Cost Accounting and Financial Management

Standard Costing to produce at the planned activity of 10,000 units. If the company produces 11,000 units, the variance will show the benefits of operating at a level above the budgeted activity. If, however, the factory has produced 10,000 units, there will be no production volume variance because the actual activity equals what was budgeted i.e. the production of 10,000 units. (2) Overhead expenses variance :As discussed above, the Production Volume Variance analyses the unrecovered fixed overheads. Apart from this, there can be variations in the actual spending of both fixed and variable overheads when compared to what was established as a standard. Such variations can be accounted for by analyzing a Overhead expenses variance. The following illustration shows how overhead expense rates are computed and variance analysed. Illustration The overhead expense budget for a factory producing to a capacity of 200 units per month is as follows:

Description of overhead Fixed cost per unit in Rs.

Variable cost per unit in Rs.

Total cost per unit in Rs.

Power and fuel 1,000 500 1,500 Repair and maintenance 500 250 750 Printing and stationary 500 250 750 Other overheads 1,000 500 1,500 Rs. 3,000 Rs. 1,500 4,500

The factory has actually produced only 100 units in a particular month. Details of overheads actually incurred have been provided by the accounts department and are as follows:

Description of overhead Actual cost Power and fuel Rs. 4,00,000 Repair and maintenance Rs. 2,00,000 Printing and stationary Rs. 1,75,000 Other overheads Rs. 3,75,000

You are required to compute the production volume variance and the overhead expenses variance.

8.13

Page 454: 30510870 Cost Accounting and Financial Management

Cost Accounting Solution

Production volume variance Standard fixed overheads per unit : Rs 3,000 (Given) Actual production : 100 units Standard production (capacity) : 200 units Unabsorbed units : 100 units (200 – 100 ) Production volume variance : Rs 3,000 × 100 units = Rs 3,00,000 (Adverse) Overhead expenses variance Standard fixed overheads for actual production : Rs 6,00,000 Standard variable overheads for actual production : Rs 1,500 × 100 units = Rs 1,50,000 Std total overheads for actual production : Rs 7,50,000 Actual overheads : Rs 11,50,000 Overhead expense variance : Rs 4,00,000 (Adverse) Sales variances : Variances which arise due to a change in the actual selling price and the actual quantity of units sold from that what was budgeted are known as sales variances. These variances are computed on the basis of sales value. They provide the sales manager an idea of the effect of various factors affecting sales such as prices, quantity and sales mix on the overall sales value. The sales value variances are more or less similar to material cost variances or labour cost variances. Sales value variance: It is the difference between the budgeted sales and actual sales. The variance can be bifurcated into sales price variance and sales volume variance. (a) Sales price variance : Actual quantity of Sales (Actual price – Budgeted price) or Actual

sales minus actual quantity at budgeted prices.

(b) Sales volume variances: Budgeted price (Actual quantity – Budgeted quantity) or Actual quantity at budgeted price minus budgeted sales.

Computation of variances: Let us now proceed to study with illustrations the method of computation of major variances described earlier. In all the problems illustrated in the

8.14

Page 455: 30510870 Cost Accounting and Financial Management

Standard Costing following pages, ‘F’ means favourable variance and ‘A’ means adverse variance. Direct material variances: Illustration The standard and actual figures of product ‘Z’ are as under: Standard Actual

Material quantity 50 units 45 units Material price per unit Re. 1.00 Re. 0.80

Calculate material cost variances. Solution The variances may be calculated as under: (a) Standard cost = Std. qty × Std. price = 50 units × Re.1.00 = Rs.50 (b) Actual cost = Actual qty. × Actual price = 45 units × Re. 0.80 = Rs. 36 Variances: (i) Price variance = Actual qty (Std. price – Actual price) = 45 units (Re. 1.00 – Re. 0.80) = Rs.9 (F) (ii) Usage variance = Std. price (Std. qty – Actual qty.)

= Re.1 (50 units – 45 units) = Rs.5 (F) (iii) Material cost variance = Standard cost – Actual cost (Total variance) = Rs. 50 – Rs.36 = Rs. 14 (F) Direct labour variances: Illustration The standard and actual figures of a firm are as under Standard time for the job 1,000 hours Standard rate per hour Re. 0.50 Actual time taken 900 hours Actual wages paid Rs. 360 Compute the variances Solution (a) Std. labour cost Rs. (1,000 hours × Re. 0.50) 500 (b) Actual wages paid 360 (c) Actual rate per hour: Rs. 360/900 hours = Re. 0.40

8.15

Page 456: 30510870 Cost Accounting and Financial Management

Cost Accounting Variances (i) Rate variance = Actual time (Std. rate – Actual rate) = 900 hours (Re.0.50 – Re.0.40) = Rs. 90 (F) (ii) Efficiency variance = Std. rate per hr. (Std. time – Actual time) = Re. 0.50 (1,000 hrs. – 900 hrs.) = Rs. 50 (F) (iii) Total labour cost variance = Std. labour cost – Actual labour cost = Rs.140(F) Overhead variances: Illustration XYZ Company has established the following standards for factory overheads. Variable overhead per unit: Rs. 10/- Fixed overheads per month Rs. 1,00,000 Capacity of the plant 20,000 units per month. The actual data for the month are as follows: Actual overheads incurred Rs. 3,00,000 Actual output (units) 15,000 units Required: Calculate overhead variances viz : (i) Production volume variance (ii) Overhead expense variance Solution Un utilised capacity : 20,000 units less 15,000 units = 5,000 units Std fixed overheads per unit = Rs. 5 per unit Production volume variance = 5,000 units × Rs. 5 = Rs. 25,000 (Adverse) Std variable overheads for actual production : Rs. 10 × 15,000 units = Rs. 1,50,000 Std fixed overheads = Rs. 1,00,000 Total overheads on standards for actual production = Rs. 2,50,000 Actual overheads incurred = Rs. 3,00,000 Overhead expense variance = Rs. 50,000 (Adverse)

8.16

Page 457: 30510870 Cost Accounting and Financial Management

Standard Costing Sale variances Illustration Compute the sales variances from the following figures: - Product Budgeted Quantity Actual Actual

Quantity Budgeted Price Quantity Price Price Rs. Rs. Rs.

A 2,000 2.50 2,400 3.00 B 1,500 5.00 1,400 4.50 C 1,000 7.50 1,200 7.00 D 500 10.00 400 10.50

Solution Basic calculation: Product

price Budgeted

price Actual Price

Budgeted quantity

sales

Actual Quantity

Sales

Quantity at budgeted

sales

Actual Sales at

Budgeted Price

Actual Sales

a b c d (e)=a × c f=(a × d g=(b × d)

Rs. Rs. Rs. Rs. Rs.

A 2.50 3.00 2,000 2,400 5,000 6,000 7,200

B 5.00 4.50 1,500 1,400 7,500 7,000 6,300

C 7.50 7.00 1,000 1,200 7,500 9,000 8,400

D 10.00 10.50 500 400 5,000 4,000 4,200

5,000 5,400 25,000 26,000 26,100

Computation of Variances Sales price variance = Actual quantity (Actual price – Budgeted price) = Actual sales – Standard sales = Rs. 26,100 – Rs. 26,000 = Rs. 100(F) Sales volume variance = Budgeted price (Actual quantity – Budgeted quantity) = Std. sales – Budgeted sales = Rs. 26,000 – Rs. 25,000 = Rs. 1,000 (F) Total variance = Actual sales – Budgeted sales = Rs. 26,100 – Rs. 25,000 = Rs.1,100 (F)

8.17

Page 458: 30510870 Cost Accounting and Financial Management

Cost Accounting PREPARATION OF OPERATING STATEMENT UNDER STANDARD COSTING – REPORTING OF VARIANCES Computation of variances and their reporting is not the final step towards the control of various elements of cost. It demands an analysis of variances from the side of the executives, to ascertain the correct reasons for their occurrence. After knowing the exact reasons, it becomes their responsibility to take necessary steps so as to stop the reoccurrence of adverse variances in future. To enhance the utility of such a reporting system it is necessary that such a system of reporting should not only be prompt but should also facilitate the concerned managerial level to take necessary steps. Variance reports should be prepared after keeping in view its ultimate use and its periodicity. Such reports should highlight the essential cost deviations and possibilities for their improvements. In fact the variance reports should give due regard to the following points:- (i) The concerned executives should be informed about what the cost performance should

have been. (ii) How close the actual cost performance is with reference to standard cost performance. (iii) The analysis and causes of variances. (iv) Reporting should be based on the principle of management by exception. (v) The magnitude of variances should also be stated. Standard cost reports: Standard cost reports showing the details of the variances are prepared for control purposes. Two such reports are illustrated below: (a) Standard Costing Profit & Loss statement: A standard costing profit and loss

statement will show the variance of each type under each element of cost department wise and is illustrated as below:

Standard Costing Profit and Loss Report Month ________20________ Particulars Total Dept. A Dept. B Dept. C A. Sales value B. Less: Standard cost of sales C. Standard profit D. Add / Deduct variances Materials : Price Usage Labour : Rate of pay Efficiency

8.18

Page 459: 30510870 Cost Accounting and Financial Management

Standard Costing Overhead : Expense

Production volume

Total

E. Actual profit

The adverse variance may be shown in red or in parenthesis.

The following case study shall help you understand the mechanics of a standard costing system.

STANDARD COSTING AT GLOBAL COMMUNICATION LTD.

ABOUT GLOBAL COMMUNICATIONS

Global Communication Ltd. (GCL), manufacturers of Telephone Exchanges, had been incorporated in the early 90s and has since grown rapidly. Today it is considered as one of the largest Telecom Company in the country. Till only a few months back, being the only manufacturer of such products, GCL was enjoying monopoly and huge Gross Profit Margins to its credit. Costing was hence not thought to be important, except to facilitate statutory Financial Audits. However, the Telecom revolution changed the scenario in no time, with more business houses venturing into Exchange production. Sensing competition, the management had been quick to hire the services of Mr. Ravi Shankar in order to help implement a relevant Costing System for its plant located at Kanpur. The system was to be such, so as to fulfil the requirements of Material and Expenses Control, facilitate Statutory Audit, help in Pricing Decisions and provide for the day-to-day requirements of Excise, etc. Since the variable cost component within the production process was quite high a Standard Costing System was thought off to be most appropriate.

The Manufacturing Process

GCL manufactures two types of Exchanges, viz.

1. Exch 007

2. Exch 009

Both the Exchanges are produced on separate Production Lines; however, the process of manufacture is strikingly similar. Raw Material Cost comprises about 75% of the total Product Cost and is the only identifiable Variable Cost.

8.19

Page 460: 30510870 Cost Accounting and Financial Management

Cost Accounting Production is carried out by two departments – Assembly and Testing. Raw material is issued in lots for 100 units of Finished Goods to the Assembly department. Various Electronic Components are inserted into the Printed Circuit Boards (PCBs) over here. About 40% of the insertions done by the Assembly department are automated, the rest being done by hand. The inserted components are then soldered manually. PCBs are plate shaped metallic sheets having tracks over the surface. These tracks facilitate the electronic connectivity between various components inserted at different parts of the sheets. Thirty-six such populated PCBs are finally assembled in a Rack, which is transferred to the Testing department for quality check and approval. If found OK, the production is said to be complete. In case there is any wastage on the floor (either in Testing or Assembly departments), the material is replaced by the Stores on receipt of a Replacement Slip duly signed by the Assembly or Testing department’s supervisor. The defective component so replaced is handed back to the Stores. However there are no records maintained for such scrap generated, as it does not carry any significant economic value.

The mechanics of the system as explained by Ravi

In due course the system was designed and presented in a meeting attended by almost all senior personnel’s of the Factory. Ravi used an OHP to present a few slides, which have been shown as Exhibits below. The mechanics of the system was described by Ravi as follows:

“A spread sheet for the components of cost for each of the 1800 Raw Materials shall be drawn as shown in Exhibit 1. A standard amount for each of these components of cost for every single raw material shall have to be established in the beginning of the financial year. The sum total of all these estimated cost components shall give ‘Standard Cost of each individual Raw Material’ and would serve as an input for the estimated Raw Material cost (also known as BOM cost i.e., ‘Bill of Material cost’) in the Standard Cost Sheet. There shall be two Cost Sheets pertaining to the two Exchanges being manufactured in the Factory. The Cost Sheets shall be drawn on the capacity available to the company and not the budgeted production since the Company has frequently produced to capacity, especially when nearing the close of the three previous Financial Years.

Hence, projections of Overheads shall be done as if GCL would be attaining 100% capacity utilization. The Bill of Material shall be extended to incorporate other columns as shown in Exhibit 2. This extended version of the BOM shall be known as Direct Material Control Statement (DMCS) and shall be used to calculate net Usage and net Price variances. Two such statements would be prepared, one each for the two Exchanges. The

8.20

Page 461: 30510870 Cost Accounting and Financial Management

Standard Costing Usage and Price Variances shall be calculated by using the Rejection Slips generated in the Production department/lines for identifying actual consumption (in quantity) of Raw Material during a particular month. Since the Rejection Slips shall bear the production Lot Nos., the quantity of Raw Material identified in them shall need to be added to the quantity estimated to be consumed for that particular production Lot No. Actual price of each of the Raw Material consumed shall be provided by the Accounts department and shall be inserted manually in column No.7 of the DMCS. Standard Overheads on actual production for each of the two exchanges shall be arrived at with the help of the projected figures of the respective Cost Sheets (Exhibit 3). Such Standard Overheads on actual Production shall be compared with the factory Trial Balance in order to arrive at the Overhead Expense Variance. Standard Fixed Cost per Unit as projected by the two Cost Sheets shall be utilized to calculate the Production Volume Variance. The Sales Price Variance shall reflect the difference between the actual and the standard Selling Price on actual Sales. Finally, adding all these five variances to the Standard Profit (The difference between the estimated Standard Selling Price and the Estimated Cost per Unit of each of the two Exchanges) shall help arrive at the actual Profit. This Profit shall then be reconciled with the Financial Accounting Profit as shown by the Accounts department. The reporting pattern shall be as shown below.

Period of reporting Type of report

Daily Production volume variance and daily standard profit based on daily production.

Monthly • Statement of profit and loss based on standard profit adjusted with the above mentioned five variances.

• Reconciliation of the costing profit with the financial accounting profit.

• Direct material control statement incorporating usage and material price variances.

8.21

Page 462: 30510870 Cost Accounting and Financial Management

Cost Accounting COMPONENTS OF COST

Exhibit 1

RM FOB/ Basic

Insurance Freight CIF Custom Duties

Inland components of cost e.g. local freight, insurance, etc.

Purchase Cost (7% of landed cost)

Total Standard Cost

RM1 To RM1800 Exhibit 2 DIRECT RAW MATERIAL CONTROL STATEMENT

Std Cost of RM / Ut of Fin Good (1) * (2)

RM Std Qty of RM /ut of FG (1)

Std Cost/ut of RM 1-1800 (2)

Std Qty of RM 1 – 1800 on Actual Prdn (Act Prd * (1)

Actual Qty of RM on Actual Prdn

Actual Price of RM

Usage Variance

Price Variance

RM1 - RM1800 Total Std Cost

Exhibit 3 COST SHEET on Maximum Capacity

Description Total Cost Total Fixed Cost

Total Variable Cost

Fixed Cost/Unit

Variable Cost/Unit

Total Cost/Unit

BOM Cost Salary Other Expenses

Total Factory Cost

8.22

Page 463: 30510870 Cost Accounting and Financial Management

Standard Costing The proposed system: a critical appraisal The presentation being over, the dais was thrown open to suggestions. The system was appreciated by one and all; however the following suggestions were made by the various functional managers. ♦ Purchase cost to be loaded over, as a percentage of the Raw Material cost shall not help

arrive at the exact incidence of cost. According to the Production Manager, “in case I buy a Re. 1/- Resistor and a Rs. 5000/- IC from the same vendor at the same time, will it not be wrong to consider different production overhead burden on the two concerned components?”

♦ The mechanics of calculating the actual quantity of raw material consumed may not give accurate results. Opined the Finance Manager “in case I use the Rejection Slips to understand the extra material consumed, I shall not be sure of the exact figures. This shall necessitate the need of scrap accounting, which at present does not exist”.

♦ The Purchase Manager was sceptical about the mechanics of calculating the raw material purchase price variance. “In case the usage variance is abnormal, a small adverse variance in the purchase price shall be blown out of proportion”.

♦ The Accounts Manager highlighted the need of an Integrated Accounting system rather than the proposed use of the accounting Trial Balance as it existed on date, “without production accounting the control of material is difficult. Year after year we are having problems reconciling Purchases with the Stock. Since it is difficult to physically identify the WIP every month end, the only alternative left is an integrated accounting system with an efficient cycle count (perpetual inventory check)”

Questions 1. The Raw Material Price Variance is calculated on the actual quantity of Raw Material

consumed? Suggest a modification to satisfy the concerns raised by the Purchase Manager.

2. How could an Integrated Accounting system be of help?

ACCOUNTING PROCEDURE FOR STANDARD COST The standard cost operations can be recorded in the books of account. Two important accounting procedures for standard costs are:- (i) Single Plan : The main purpose of standard costing is cost control. To achieve this purpose, the variances should be analysed according to their causes. Analysis should be timely so that much time is not lost in taking corrective action wherever needed. In the partial plan, variances are analysed at the end of period. The single plan system envisages the

8.23

Page 464: 30510870 Cost Accounting and Financial Management

Cost Accounting posting of all items in the debit side of the work-in-progress account at the standard cost leaving the credit side to represent the standard cost of finished production and work-in-progress. This system enables the ascertainment of variances as and when the transaction is posted to work-in-progress account. In other words, the analysis of variances is done from the original documents like invoices, labour sheets, etc., and this method of analysis is known as analysis at source. Since, the single plan system contemplates the analysis of variances at source, the installation of this system requires more planning so that effective documentation at each stage is introduced for proper recording and analysis of variance. Thus for example, the issue of bill of materials to the stores enables the storekeeper to calculate the standard value of materials. If any material is requisitioned beyond the standard, he can mark the same for material usage variance account. In the production department, as and when the finished output is recorded, the standard waste and actual waste can be compared and necessary entries can be made by the shop supervisors for posting the excessive usage to appropriate variance accounts. Scheme of entries: So far as materials are concerned, material price variances are recorded at the time of receipt of the material and the material quantity variances are recorded as far as possible when excess materials are used. The entries will be as illustrated below: 1. Dr. Material Control A/c

Dr. or Cr. Material Price Variance A/c

Cr. Creditors A/c. This entry enables the firm to debit the material control account with the actual purchases at standard cost and credit the creditor’s account at the actual cost of actual prices thereby transferring the variances to price variance account. 2. Dr. Work-in-progress Control A/c

Dr. or Cr. Material Usage Variances A/c

Cr. Material Control A/c This entry charges the work-in-progress control account with the standard cost of standard quantity and credit the material control account at the standard cost of actual issue, the variance being transferred to usage variance account. 3. Dr. Wages Control A/c

Dr./Cr. Labour Rate Variances A/c

Cr. Cash

8.24

Page 465: 30510870 Cost Accounting and Financial Management

Standard Costing This entry is passed to record the wages at standard rate thereby transferring rate variances to the appropriate account. 4. Dr. Work-in-progress Control A/c

Dr. or Cr. Overhead Expense Variances A/c

Cr. Overhead Expense Control A/c. (ii) Partial plan: This system uses current standards in which the inventory will be valued at current standard cost figure. Under this method the work-in-progress account is charged at the actual cost of production for the month and is credited with the standard cost of the month’s production of finished product. The closing balance of work-in-progress is also shown at standard cost. The balance after making the credit entries represent the variance from standard for the month. The analysis of the variance is done after the end of the month. This method is simple in operation because variances are analysed after the end of month but may present difficulties if the firm makes a variety of products. Recapitulation : (1) Current standards are used in both the systems. (2) Under the partial plan, material stocks are carried at actual cost whereas the same are carried out at standard cost under the single plan. (3) The work-in-progress and finished goods are valued at standard cost under both the methods. (4) Computation of variances :

(a) In partial plan, material price variance is computed on material used in finished goods and work-in-progress whereas in single plan it is computed on the material quantity purchased.

(b) The partial plan is suitable where simple analysis of variance is sufficient at the end of the period whereas the single plan is preferred if frequent detailed analysis of variance is desired, as (a) the comparison of actual with standard cost of each operation or operator or (b) the daily reporting of standard cost of excess material used.

DISPOSITION OF VARIANCES There is no unanimity of opinion in regard to disposition of variances. The following are the various methods:– (a) Write off all variances to profit and loss account or cost of sales every month.

8.25

Page 466: 30510870 Cost Accounting and Financial Management

Cost Accounting (b) Distribute the variance prorata to cost of sales, work-in-progress and finished good

stocks. (c) Write off quantity variance to profit and loss account but the price variances may be

spread over cost of sales, work-in-progress and finished goods stocks. The reason behind apportioning price variances to inventories and cost of sales is that they represent cost although they are described as variance.

ADVANTAGES AND CRITICISM OF STANDARD COSTING Advantages of Standard Costing: (i) It serves as a basis for measuring operating performance and cost control. By setting standards, proper classification and determination of variances, is possible. This serves as a signal for prompt corrective action. This system provides for reporting on the principle of exception. The basis of this principle is that only matters which are not proceeding according to plan are reported upon. This enables the managers to concentrate upon essential matters and leave the non-essentials to take care of themselves. By using special forms, any excessive time taken, extra material used or additional services consumed can be brought to light as part of the ordinary routine. In other words, if the variances are negligible, it means that the performance is more or less in accordance with the standards. Significant variances which warrant the attention of the manager are brought to his knowledge.

(ii) It aids price fixing. Standard costing can be used to predict costs. Although actual cost may vary from day to day, standard costs will remain stable over a period of time and, where demand for a product is elastic, this information can be used as a basis for fixing the selling price.

(iii) Introduction of standard costing facilitates evaluation of jobs and introduction of incentives. Job values can be determined by the use of evaluation and scale of wages fixed according to the responsibility involved in each job.

(iv) Standard costing facilitates the estimation of the cost of new products with greater accuracy.

(v) It serves as a basis for inventory valuation. Standard costs are used for inventory valuation because actual costs are not typical and less clerical work is involved in carrying standard value into inventory records than actual value. A further advantage of this procedure is that material stock can be recorded in terms of quantities only.

(vi) Standard costing is also used for the measurement of profits. The question of correct approach of calculating profit is very much related to stock valuation and to the methods of

8.26

Page 467: 30510870 Cost Accounting and Financial Management

Standard Costing dealing with the absorption of fixed overheads. Standard costing will eliminate any variations in profit due to changes in the values of stock holding from period to period and will thus provide a true basis for the measurement of profit.

(vii) Standard costing greatly aids business planning, budgeting and managerial decision making. Standard costs being pre-determined costs, are particularly useful in planning and budgeting.

(viii) Standard costing aids in standardisation of products, operations and processes. Since standards are laid down for each product, its components, materials, operations, processes etc., it improves the overall production efficiency and reduces costs.

(ix) It provides objectives and targets to be achieved by each level of management and defines the responsibilities of departmental managers. Standard costs are pre-determined on the basis of reasonable and achievable level of output. The departmental head, therefore, comes to know what is expected of him and his level of performance in comparison to the targets can be seen from the variance reports. Thus the system serves as an incentive to the departmental head to achieve the targets set by the company.

(x) Standard costing sets a uniform basis for comparison of all elements of costs. Since care is taken in setting standards, the standards become unchanging units of comparison. The standard hour may be used as a basic unit to compare dissimilar products or processes.

(xi) The maximum use of working capital, plant facilities and current assets is assured because wastage of materials and loss due to idle time are closely controlled.

Criticism of Standard Costing: The following is some of the criticism which may be levelled against the standard costing system. The arguments have been suitably answered as stated against each by advocates of the standard costing and hence they do not invalidate the usefulness of the system to business enterprises.

(i) Variation in price: One of the chief problem faced in the operation of the standard costing system is the precise estimation of likely prices or rate to be paid. The variability of prices is so great that even actual prices are not necessarily adequately representative of cost. But the use of sophisticated forecasting techniques should be able to cover the price fluctuation to some extent. Besides this, the system provides for isolating uncontrollable variances arising from variations to be dealt with separately.

(ii) Varying levels of output: If the standard level of output set for pre-determination of standard costs is not achieved, the standard costs are said to be not realised. However, the statement that the capacity utilisation cannot be precisely estimated for absorption of

8.27

Page 468: 30510870 Cost Accounting and Financial Management

Cost Accounting overheads may be true only in some industries of jobbing type. In vast majority of industries, use of forecasting techniques, market research, etc., help to estimate the output with reasonable accuracy and thus the variation is unlikely to be very large. Prime cost will not be affected by such variation and, moreover, variance analysis helps to measure the effects of idle time.

(iii) Changing standard of technology: In case of industries that have frequent technological changes affecting the conditions of production, standard costing may not be suitable. This criticism does not affect the system of standard costing. Cost reduction and cost control is a cardinal feature of standard costing because standards once set do not always remain stable. They have to be revised.

(iv) Attitude of technical people: Technical people are accustomed to think of standards as physical standards and, therefore, they will be misled by standard costs. Since technical people can be educated to adopt themselves to the system through orientation courses, it is not an insurmountable difficulty.

(v) Mix of products: Standard costing presupposes a pre-determined combination of products both in variety and quantity. The mixture of materials used to manufacture the products may vary in the long run but since standard costs are set normally for a short period, such changes can be taken care of by revision of standards.

(vi) Standards may be either too strict or too liberal because they may be based on (a) theoretical maximum efficiency, (b) attainable good performance or (c) average past performance. To overcome this difficulty the management should give thought to the selection of a suitable type of standard. The type of standard most effective in the control of costs is one which represents an attainable level of good performance.

(vii) Standard costs cannot possibly reflect the true value in exchange. If previous historical costs are amended roughly to arrive at estimates for ad hoc purposes, they are not standard costs in the strict sense of the term and hence they cannot also reflect true value in exchange. In arriving at standard costs, however, the economic and technical factors, internal and external, are brought together and analysed to arrive at quantities and prices which reflect optimum operations. The resulting costs, therefore, become realistic measures of the sacrifices involved.

8.28

Page 469: 30510870 Cost Accounting and Financial Management

Standard Costing Self-Examination Questions Multiple choice questions 1. The bookkeeping entries in a standard cost system when the actual price for raw material

is less than the standard price are, (a) Debit raw material control account

Credit material price variance account (b) Debit WIP control account

Credit raw material control account (c) Debit raw material price variance account

Credit raw material control account (d) Debit WIP control account

Credit raw material price variance account 2. A standard which assumes efficient level of operations, but which includes allowance for

factors such as waste and machine downtime is known as an (a) Ideal standard (b) Normal standard (c) Attainable standard (d) Neither a nor b nor c

3. The standard raw material cost for producing one unit of a finished product is Rs. 27. Standard raw material usage for every unit of finished product is 3 kg. If 200 units were produced and Rs. 5,518 was paid for 620 kg. of raw material then the direct material price variance is (a) Rs. 62 (F) (b) Rs. 72(A) (c) Rs. 100(F) (d) Rs. 100(A)

4. The direct material usage variance computed from details of the above question is (a) Rs. 200 (F) (b) Rs. 200(A) (c) Rs. 180(F) (d) Rs. 180(A)

8.29

Page 470: 30510870 Cost Accounting and Financial Management

Cost Accounting 5. If fixed production overheads are under absorbed by Rs. 50,000 and the actual

expenditure was Rs. 55,000 lower than what was budgeted then the fixed production overhead volume variance is (a) Rs. 1,10,000(F) (b) Rs. 1,05,000(A) (c) Rs. 1,10,000(A) (d) NIL

6. The direct material usage variance for last period was Rs. 3,400 adverse. What reasons could have contributed such a variance (a) Output was higher than budgeted (b) The purchase department bought poor quality material (c) The original standard usage was set extremely high (d) An old inefficient machine was causing excess wastage

7. During a period 850 assemblies were made with a nil rate variance and a Rs. 4,400 adverse efficiency variance. If the standard labour hours per assembly are 24 with a Rs. 8 per hour standard labour cost, how many actual labour hours were worked? (a) 19,000 hrs (b) 20,000 hrs (c) 20,440 hrs (d) 20,950 hrs

8. During a period 25,600 labour hours were worked at a standard rate of Rs. 7.50 per hour. The direct labour efficiency variance was Rs. 8,250 (A). How many standard hours were produced? (a) 24,500 (b) 25,000 (c) 24,000 (d) 25,500

9. Standard price of material per kg is Rs. 20, standard usage per unit of production is 5 kg. Actual usage of producing 100 units is 520 kg all of which was purchased @ Rs. 22 per kg. Material price variance is (a) Rs. 1,040 (A)

8.30

Page 471: 30510870 Cost Accounting and Financial Management

Standard Costing

(b) Rs. 2,000 (A) (c) Rs. 400 (A) (d) neither a nor b nor c

10. The actual and standard direct material costs for producing a specified quantity of product are as follows: Actual 51,000 kgs. at Rs. 5.05 Rs. 2,57,550 Standard 50,000 kgs. at Rs. 5.00 Rs. 2,50,000

the direct material price variance is, (a) RS 50 (A) (b) RS. 2,500 (A) (c) Rs 2,550 (A) (d) Rs. 7,550 (A)

Answers to multiple choice questions 1.a. 2.c. 3.a. 4.d. 5.b. 6.b&c 7.d 8.a 9.a 10.c Short answer type questions 1. Explain briefly how standards are compiled for material and labour costs for a product. 2 How are variances disposed off in a standard costing system? Discuss briefly. 3. In group A, the names of variances are given and in group B, examples of the reasons

for cost variances are given. Match them. Group A Group B

A. Material Price Variance. (1) A non-standard mixture used

B. Labour Rate Variance. (2) Changes in basic price of raw materials.

C. Overhead Volume Variance. (3) Poor working conditions.

D. Materials Usage Variance. (4) Using skilled labour in place of unskilled labour.

E. Labour Efficiency Variance. (5) Slackness in production

4. Explain briefly the nature and purpose of material and labour variances.

8.31

Page 472: 30510870 Cost Accounting and Financial Management

Cost Accounting 5. The following are the two journal entries for the transaction noted below. State the plan

to which these entries are applicable.

(a) Standard Clearing A/c Dr.

To Material Control A/c

For charging the actual quantity of material consumed at standard price.

(b) Material Control A/c Dr.

Dr. or Cr. Material Control Variance A/c

To Creditors

(For charging the standard cost of material to material control account there by transferring the price variance to price variance account.)

Long answer type questions

1. Discuss the different type of standards known to you. In case a company, for the purposes of estimating its standard cost of product, switches over to ideal standards from the existing normal standards, would the standard cost of the concerned product increase or decrease? Discuss.

2. What are the basic objectives in the use of standard costs? How can standards be used by management to help control costs?

3. “Calculation of variances in standard costing is not an end in itself, but a means to an end. “Discuss.

4. Standard profits need to be reconciled with material, labour, overhead and sales variances to arrive at actual profits. Discuss.

5. Discuss the various advantages and criticisms levelled against standard costing.

Numerical Questions

1. Compute the material variances from the following data.

Actual quantity consumed 100 Kgs.

Actual price per kg. Rs. 19

Standard price per kg. Rs. 20

Production in standard units is 45 units; one standard unit requires 2 kg. of material.

8.32

Page 473: 30510870 Cost Accounting and Financial Management

Standard Costing 2. The standard time per unit is 2 hours at Re. 1/- per hour. During a period, 500 units are

made and the records showed the actual payment of wages of Rs. 1,800 for 1200 hours worked. Compute the labour cost variances.

3. The following Bill of Material relates to a Product called ‘ABAB’, the maximum capacity per month of which is 200 Units.

Material description Std Quantity Std. Cost

A 1 Kg Rs. 2000 per Kg.

B 10 Nos. Rs. 200 per Unit

C 2 Litres Rs. 50 per litre

Budgeted Fixed Expenses per month equal Rs. 1.5 Lakhs. The budgeted Selling Price of the product is Rs. 6,000. Other variable costs (apart from Raw Material) are budgeted at Rs. 1,000 per Unit. In a particular month 175 Units of this product are produced and sold. The Fixed Costs incurred in the concerned month were Rs. 2 Lakhs whereas the variable cost expenditure was Rs. 900 per Unit. You are required to:

(a) Compute the Standard Cost of the product.

(b) Calculate production volume and variable overhead variances.

4. Suppose that 4 kgs. of material A are required to make one unit of product TS, each kilogram costing Rs. 10. It takes direct labour 5 hours to make one unit of product TS. The labour force is paid Rs. 4.50 per hour.

During the period the following results were recorded.

Material A : 8,200 kgs purchased on credit* Rs. 95,000 Material A: kgs issued to production* 8,200 kgs. Units of product TS produced* 1,600 Direct labour hours worked* 10,000 Cost of direct labour* Rs. 32,000

Required:

Calculate the following variances for the period.

(i) Material price variance

8.33

Page 474: 30510870 Cost Accounting and Financial Management

Cost Accounting

(ii) Material usage variance

(iii) Labour rate variance

(iv) Labour efficiency variance

5. The following Bill of Material relates to ‘1+7 ASCS’, a product manufactured by ABC Ltd.

Raw Material Standard Quantity per Unit of 1+ 7 ASCS (Nos.)

Standard Cost per Unit of raw material (Rs.)

PCB 10 1000

IC 05 900

Relay 15 100

Transformer 10 500

Rack 01 4000

The maximum capacity of the factory manufacturing this product is 200 Units per month. Budgeted Fixed Costs per month are Rs. 30,00,000. Raw material is the only variable cost. Budgeted selling price is Rs. 60,000 per Unit.

From the following actual results of a particular month, you are required to calculate relevant variances and the actual profits made.

Actual production and sales ; 150 Units of ‘1+ 7 ASCS’

Actual Fixed expenses : Rs. 31,00,000

Actual Selling price per Unit : Rs. 59,000.

Opening Stock on floor (raw material)

Closing Stock on floor (raw material)

Issues during the month (raw material)

PCB 100 200 1800 @Rs 1100/Ut IC 50 100 900 @ Rs 1000/Ut Relay 250 250 2250 @Rs 90/Ut Transformer 1000 700 1200 @Rs 500/Ut Rack 100 100 150 @ Rs 4100/Ut

8.34

Page 475: 30510870 Cost Accounting and Financial Management

Standard Costing

8.35

Page 476: 30510870 Cost Accounting and Financial Management

CHAPTER 9

MARGINAL COSTING

Learning objectives When you have finished studying this chapter, you should be able to ♦ Understand the difference between absorption costing and marginal costing ♦ Understand the concept of contribution and contribution to sales ratio. ♦ Understand the method of computation of break-even point, both mathematically and also

with the help of a graph. ♦ Understand the basic limitations of break even analysis

9.1 INTRODUCTION Marginal costing is not a distinct method of costing like job costing, process costing, operating costing, etc., but a special technique used for managerial decision making. Marginal costing is used to provide a basis for the interpretation of cost data to measure the profitability of different products, processes and cost centres in the course of decision making. It can, therefore, be used in conjunction with the different methods of costing such as job costing, process costing, etc., or even with other techniques such as standard costing or budgetary control. In marginal costing, cost ascertainment is made on the basis of the nature of cost. It gives consideration to behaviour of costs. In other words, the technique has developed from a particular conception and expression of the nature and behaviour of costs and their effect upon the profitability of an undertaking. In the orthodox or total cost method, as opposed to marginal costing method, the classification of costs is based on functional basis. Under this method the total cost is the sum total of the cost of direct material, direct labour, direct expenses, manufacturing overheads, administration overheads, selling and distribution overheads. In this system, other things being equal, the total cost per unit will remain constant only when the level of output or mixture is the same from period to period. Since these factors are continually fluctuating, the actual total cost will vary from one period to another. Thus, it is possible for the costing department to say one day that an item costs Rs. 20 and the next day it costs Rs. 18. This situation arises because of changes in volume of output and the

Page 477: 30510870 Cost Accounting and Financial Management

Cost Accounting peculiar behaviour of fixed expenses included in the total cost. Such fluctuating manufacturing activity, and consequently the variations in the total cost from period to period or even from day to day, poses a serious problem to the management in taking sound decisions. Hence, the application of marginal costing has been given wide recognition in the field of decision making.

9.2 THEORY OF MARGINAL COSTING The theory of marginal costing is that in relation to a given volume of output, additional output can normally be obtained at less than proportionate cost because within limits the aggregate of certain items of cost will tend to remain fixed and only the aggregate of the remainder will tend to rise proportionately with increase in output. Conversely, a decrease in the volume of output will normally be accompanied by a less than proportionate fall in the aggregate cost. The theory of marginal costing may therefore be explained in three steps: (i) If the volume of output increases, the average cost per unit will, in the normal

circumstances, be reduced. Conversely, if the output is reduced, the average cost per unit will go up. If the factory produces 1,000 units at a total cost of Rs. 3,000 and if by increasing the output by one unit, the cost goes up to Rs. 3,002, the marginal cost of the additional output is Rs. 2.

(ii) If the increase in output is more than one unit say 20 units, the total increase in cost to produce these units is Rs. 3,045, the average marginal cost is Rs. 2.25 per unit is as under:

units Additionalcost Additional

= units 2045 Rs.

= Rs. 2.25

(iii) The ascertainment of marginal cost is based on the classification and segregation of costs into fixed and variable costs.

9.3 Definitions: In order to appreciate the concept of marginal costing, it is necessary to study the definition of marginal costing and certain other terms associated with this technique. The important terms have been defined as follows: 1. Marginal costing : The ascertainment of marginal cost and of the effect on profit of changes in volume or type of output by differentiating between fixed costs and variable costs. 2. Marginal cost: The amount at any given volume of output by which aggregate variable costs are changed if the volume of output is increased by one unit. In practice this is

9.2

Page 478: 30510870 Cost Accounting and Financial Management

Marginal Costing

measured by the total variable cost attributable to one unit. Marginal cost can precisely be the sum of prime cost and variable overhead. Note: In this context a unit may be a single article, a batch of articles, an order, a stage of production capacity, a process or a department. It relates to the change in output in particular circumstances under consideration. 3. Direct costing: Direct costing is the practice of charging all direct cost to operations, processes or products, leaving all indirect costs to be written off against profits in the period in which they arise. Under direct costing the stocks are valued at direct costs, i.e., costs whether fixed or variable which can be directly attributable to the cost units. In general, the terms marginal costing and direct costing are used as synonymous. However, direct costing differs from marginal costing in that some fixed costs considered direct are charged to operations, processes or products, whereas in marginal costing only variable costs are considered. Marginal costing is mainly concerned with providing of information to management to assist in decision making and for exercising control. Marginal costing is considered to be a technique with a broader meaning than direct costing. Marginal costing is also known as ‘variable costing’ or ‘out of pocket costing’. 4. Differential cost : It may be defined as “the increase or decrease in total cost or the change in specific elements of cost that result from any variation in operations”. It represents an increase or decrease in total cost resulting out of : (a) producing or distributing a few more or few less of the products; (b) a change in the method of production or of distribution; (c) an addition or deletion of a product or a territory; and (d) selection of an additional sales channel. Differential cost, thus includes fixed and semi-variable expenses. It is the difference between the total costs of two alternatives. It is an adhoc cost determined for the purpose of choosing between competing alternatives, each with its own combination of income and costs. 5. Incremental cost : It is defined as, “the additional costs of a change in the level or nature of activity”. As such for all practical purposes there is no difference between incremental cost and differential cost. However, from a conceptual point of view, differential cost refers to both incremental as well as decremental cost. Incremental cost and differential cost calculated from the same data will be the same. In practice, therefore, generally no distinction is made between differential cost and incremental cost. One aspect which is worthy to note is that incremental cost is not the same at all levels. Incremental cost between 50% and 60% level of output may be different from that which is arrived at between 80% and 90% level of output. Differential cost or incremental cost

9.3

Page 479: 30510870 Cost Accounting and Financial Management

Cost Accounting analysis deals with both short-term and long-term problems. This analysis is more useful when various alternatives or various capacity levels are being considered. Differential costs or incremental costs can be easily identified by preparing a flexible budget as shown below: Example

Description Activity Level 50% 60% 70% 80% Units 500 600 700 800 Rs. Rs. Rs. Rs. Variable costs 5,000 6,000 7,000 8,000 Semi-variable costs 1,500 1,600 1,650 1,700 Fixed costs 2,500 2,500 2,500 3,000Total costs 9,000 10,100 11,150 12,700 Differential costs/Incremental costs 1,100 1,050 1,550

6. Contribution : Contribution or the contributory margin is the difference between sales value and the marginal cost. It is obtained by subtracting marginal cost from sales revenue of a given activity. It can also be defined as excess of sales revenue over the variable cost. The difference between sales revenue and marginal/variable cost is considered to be the contribution towards fixed expenses and profit of the entire business. The contribution concept is based on the theory that the profit and fixed expenses of a business is a ‘joint cost’ which cannot be equitably apportioned to different segments of the business. In view of this difficulty the contribution serves as a measure of efficiency of operations of various segments of the business. The contribution forms a fund for fixed expenses and profit as illustrated below:

9.4

Page 480: 30510870 Cost Accounting and Financial Management

Marginal Costing

Product A Product B Product C Selling price Selling price Selling price Less: Marginal cost Less: Marginal cost Less: Marginal cost = Contribution = Contribution = Contribution

Fixed exp

7. Key factor : Key factor or Limiting facperiod limits the activities of an underproducts or services or it may be the shoe.g., labour hours, available plant capaKey Factors or Limiting Factors are: (a) Shortage of raw material. (b) Shortage of labour. (c) Plant capacity available. (d) Sales capacity available. (e) Cash availability.

9.4 ASCERTAINMENT OF MARGINALUnder marginal costing, fixed expensescharged to profit and loss account. In orexpenses as under: 1. Variable expenses : Apart from prime

FUND enses and Profit

tor is a factor which at a particular time or over a taking. It may be the level of demand for the rtage of one or more of the productive resources,

city, raw material’s availability etc. Examples of

COST are treated as period costs and are therefore, der to ascertain the marginal cost, we classify the

costs which are variable, the overhead expenses

9.5

Page 481: 30510870 Cost Accounting and Financial Management

Cost Accounting that change in proportion to the change in the level of activity are also variable expenses. Thus when expenses go up or come down in proportion to a change in the volume of output, such that, with every increase of 20% in output, expenses also go up by 20% or vice versa, these expenses are known as variable expenses. Variable expenses fluctuate in total with fluctuations in the level of output but tend to remain constant per unit of output. Examples of such expenses are raw material, power, commission paid to salesmen as a percentage of sales, etc. 2. Fixed expenses : Fixed expenses or constant expenses are those which do not vary in total with the change in volume of output for a given period of time. Fixed cost per unit of output will, however, fluctuate with changes in the level of production. Examples of such expenses are managerial remuneration, rent, taxes, etc. There may, however, be different levels of fixed costs at different levels of output, as for example, where after certain level of output extra expenditure may be needed. In the case of introduction of additional shift working, fixed expenses will be incurred, say, for the appointment of additional supervisors. Fixed expenses are treated as period costs and are therefore charged to profit and loss account. 3. Semi-variable expenses : These expenses (also known as semi-fixed expenses) do not change within the limits of a small range of activity but may change when the output reaches a new level in the same direction in which the output changes. Such increases or decreases in expenses are not in proportion to output. An example of such an expense is delivery van expense. Semi-variable expenses may remain constant at 50% to 60% level of activity and may increase in total from 60% to 70% level of activity. These expenses can be segregated into fixed and variable by using any one of the method, as given under next heading. Depreciation of plant and machinery depends partly on efflux of time and partly on wear and tear. The former is fixed and the latter is variable. The total cost is arrived at by merging these three type of expenses.

9.5 SEPARATING FIXED AND VARIABLE COSTS – Please Refer to Chapter 1 Uses of segregation of cost Segregation of all expenses into fixed and variable elements is the essence of marginal costing. The primary objective of the classification of expenses into fixed and variable elements is to find out the marginal cost for various types of managerial decisions. A number of such decisions will be discussed later in the chapter-3of this book. The other uses of it are as below: (i) Control of expenses : The classification of expenses helps in controlling expenses. Fixed expenses are said to be sunk costs as these are incurred irrespective of the level of production activity and they are regarded as uncontrollable expenses. Since variable expenses vary with the production they are said to be controllable. By this classification,

9.6

Page 482: 30510870 Cost Accounting and Financial Management

Marginal Costing

therefore, responsibility for incurring variable expenses is determined in relation to activity and hence the management is able to control these expenses. The departmental heads always try to keep these expenses within limits set by the management. (ii) Preparation of budget estimates: This distinction between fixed and variable cost also helps the management to estimate precisely, the budgeted expenses to gauge the actual efficiency of the business, by comparing the actual with budgets. This can be illustrated by means of the following example. Example A firm which produced 1,000 units in June incurred the following expenses. Rs. Variable expenses 2,000 Semi-variable expenses (60% variable) 2,000 Fixed expenses 6,000 Total 10,000If in July, the units are expected to increase to 1,200, it will be wrong to estimate the expenses at Rs. 12,000. The estimate for 1,200 units will have to be prepared as under: Rs. Variable expenses

units 1,000units 1,200 × 2,000 Rs.

2,400

Semi-variable expenses: Variable component

Rs.2,000 ×units 1,000units 1,200

10060

× 1,440

Fixed component Rs. 2,000 ×

10040

800

Fixed expenses 6,000 Total 10,640

If the actual expenses are Rs. 11,000, it is not correct to say that there has been a saving of Rs. 1,000 i.e. (Rs. 12,000 – Rs. 11,000) but that the firm has overspent Rs. 360 i.e., Rs. 11,000 – Rs. 10,640. Thus, managerial control is possible.

9.6 DISTINCTION BETWEEN MARGINAL AND ABSORPTION COSTING The main points of distinction between marginal costing and absorption costing are as

9.7

Page 483: 30510870 Cost Accounting and Financial Management

Cost Accounting below:

Marginal costing Absorption costing 1. Only variable costs are considered for

product costing and inventory valuation.

Both fixed and variable costs are considered for product costing and inventory valuation.

2. Fixed costs are regarded as period costs. The Profitability of different products is judged by their P/V ratio.

Fixed costs are charged to the cost of production. Each product bears a reasonable share of fixed cost and thus the profitability of a product is influenced by the apportionment of fixed costs.

3. Cost data presented highlight the total contribution of each product.

Cost data are presented in conventional pattern. Net profit of each product is determined after subtracting fixed cost along with their variable costs.

4. The difference in the magnitude of opening stock and closing stock does not affect the unit cost of production.

The difference in the magnitude of opening stock and closing stock affects the unit cost of production due to the impact of related fixed cost.

9.61 Presentation of Information: In absorption costing the classification of expenses is based on functional basis whereas in marginal costing it is based on the nature of expenses. In absorption costing, the fixed expenses are distributed over products on absorption costing basis, that is, based on a pre-determined level of output. Since fixed expenses are constant, such a method of recovery will lead to over or under-recovery of expenses depending on the actual output being greater or lesser than the estimate used for recovery. This difficulty will not arise in marginal costing because the contribution is used as a fund for meeting fixed expenses. The presentation of information to management under the two costing techniques is as under: (a) Under Absorption Costing Description Product A Product B Total Rs. Rs. Rs. Sales value Less: Direct material

9.8

Page 484: 30510870 Cost Accounting and Financial Management

Marginal Costing

Direct labour Factory overheads GROSS PROFIT Less: Administration expenses Selling & distribution expenses NET PROFIT (b) Under Marginal Costing Description Product A Product B Total Rs. Rs. Rs. Sales value Less: Direct material Direct labour Variable factory overheads Variable selling & Distribution expenses CONTRIBUTION Less: Fixed factory overheads Administration overheads Fixed selling & distribution expenses NET PROFIT It is evident from the above that under marginal costing technique the contributions of various products are pooled together and the fixed overheads are met out of such total contribution. The total contribution is also known as gross margin. The contribution minus fixed expenses yields net profit. In absorption costing technique cost includes fixed overheads as well. Illustration WONDER LTD. manufactures a single product, ZEST. The following figures relate to ZEST for a one-year period: Activity Level 50% 100% Sales and production (units) 400 800 Rs. lakhs Rs. lakhs Sales 8.00 16.00 Production costs: Variable 3.20 6.40

9.9

Page 485: 30510870 Cost Accounting and Financial Management

Cost Accounting Fixed 1.60 1.60 Selling and administration costs: Variable 1.60 3.20 Fixed 2.40 2.40 The normal level of activity for the year is 800 units. Fixed costs are incurred evenly throughout the year, and actual fixed costs are the same as budgeted. There were no stocks of ZEST at the beginning of the year. In the first quarter, 220 units were produced and 160 units were sold. Required : (a) What would be the fixed production costs absorbed by ZEST if absorption costing is

used?

(b) What would be the under/over-recovery of overheads during the period?

(c) What would be the profit using absorption costing?

(d) What would be the profit using marginal costing?

(e) Why is there a difference between the answers to (c) and (d)? Solution (a) Fixed production costs absorbed: Rs. Budgeted fixed production costs 1,60,000 Budgeted output (normal level of activity 800 units) Therefore, the absorption rate :1,60,000/800 = Rs. 200 per unit During the first quarter, the fixed production cost absorbed by ZEST would be (220 units × Rs. 200) 44,000 (b) Under/over-recovery of overheads during the period: Rs. Actual fixed production overhead 40,000 (1/4 of Rs. 1,60,000) Absorbed fixed production overhead 44,000 Over-recovery of overheads 4,000

9.10

Page 486: 30510870 Cost Accounting and Financial Management

Marginal Costing

(c) Profit for the Quarter (Absorption Costing) Rs. Rs. Sales revenue (160 units × Rs. 2,000) : (A) 3,20,000 Less : Production costs Variable (220 units × Rs. 800) 1,76,000 Fixed overheads absorbed (220 units × Rs. 200) 44,000 Total (220 units × Rs. 1,000) 2,20,000 Less : Closing stock (60 units × Rs. 1,000) 60,000 Production cost of 160 units 1,60,000 Variable sales & admn. costs (160 units × Rs. 400) 64,000 Fixed selling & admn. costs (1/4 of Rs. 2,40,000) 60,000 Total cost of sales of 160 units : (B) 2,84,000 Unadjusted profit : {(A) – (B)} 36,000 Add: Overheads over absorbed 4,000

Actual profit 40,000(d) Profit for the Quarter (Marginal costing) Rs. Rs.

Sales revenue (160 units × Rs. 2,000) : (A) 3,20,000 Variable production costs (220 units × Rs. 800) 1,76,000 Less: Closing stock (60 units × Rs. 800) 48,000 Variable production cost of 160 units 1,28,000 Add: Variable selling & admn. costs 64,000 Total variable cost of sales of 160 units : (B) 1,92,000 Contribution : {(A) – (B)} 1,28,000 Less: Fixed production cost incurred 40,000

Fixed selling & admn. costs incurred 60,000 1,00,000 Actual profit 28,000

9.7 ADVANTAGES AND LIMITATIONS OF MARGINAL COSTING Advantages of Marginal Costing

1. The marginal cost remains constant per unit of output whereas the fixed cost remains constant in total. Since marginal cost per unit is constant from period to period within a short span of time, firm decisions on pricing policy can be taken. If fixed cost is included, the unit cost will change from day to day depending upon the volume of output. This will make decision making task difficult.

9.11

Page 487: 30510870 Cost Accounting and Financial Management

Cost Accounting 2. Overheads are recovered in costing on the basis of pre-determined rates. If fixed

overheads are included on the basis of pre-determined rates, there will be under- recovery of overheads if production is less or if overheads are more. There will be over- recovery of overheads if production is more than the budget or actual expenses are less than the estimate. This creates the problem of treatment of such under or over-recovery of overheads. Marginal costing avoids such under or over recovery of overheads.

3. Advocates of marginal costing argues that under the marginal costing technique, the stock of finished goods and work-in-progress are carried on marginal cost basis and the fixed expenses are written off to profit and loss account as period cost. This shows the true profit of the period.

4. Marginal costing helps in the preparation of break-even analysis which shows the effect of increasing or decreasing production activity on the profitability of the company.

5. Segregation of expenses as fixed and variable helps the management to exercise control over expenditure. The management can compare the actual variable expenses with the budgeted variable expenses and take corrective action through analysis of variances.

6. Marginal costing helps the management in taking a number of business decisions like make or buy, discontinuance of a particular product, replacement of machines, etc.

Limitations of Marginal Costing

1. It is difficult to classify exactly the expenses into fixed and variable category. Most of the expenses are neither totally variable nor wholly fixed. For example, various amenities provided to workers may have no relation either to volume of production or time factor.

2. Contribution of a product itself is not a guide for optimum profitability unless it is linked with the key factor.

3. Sales staff may mistake marginal cost for total cost and sell at a price; which will result in loss or low profits. Hence, sales staff should be cautioned while giving marginal cost.

4. Overheads of fixed nature cannot altogether be excluded particularly in large contracts, while valuing the work-in- progress. In order to show the correct position fixed overheads have to be included in work-in-progress.

5. Some of the assumptions regarding the behaviour of various costs are not necessarily true in a realistic situation. For example, the assumption that fixed cost will remain static throughout is not correct. Fixed cost may change from one period to another. For example salaries bill may go up because of annual increments or due to change in pay

9.12

Page 488: 30510870 Cost Accounting and Financial Management

Marginal Costing

rate etc. The variable costs do not remain constant per unit of output. There may be changes in the prices of raw materials, wage rates etc. after a certain level of output has been reached due to shortage of material, shortage of skilled labour, concessions of bulk purchases etc.

6. Marginal costing ignores time factor and investment. For example, the marginal cost of two jobs may be the same but the time taken for their completion and the cost of machines used may differ. The true cost of a job which takes longer time and uses costlier machine would be higher. This fact is not disclosed by marginal costing.

9.8 MARGINAL COST EQUATION The contribution theory explains the relationship between the variable cost and selling price. It tells us that selling price minus variable cost of the units sold is the contribution towards fixed expenses and profit. If the contribution is equal to fixed expenses, there will be no profit or loss and if it is less than fixed expenses, loss is incurred. Since the variable cost varies in direct proportion to output, therefore if the firm does not produce any unit, the loss will be there to the extent of fixed expenses. These points can be described with the help of following marginal cost equation: S × U – V × U = F + P Where, S = Selling price per unit V = Variable cost per unit U = Units F = Fixed expenses P = Profit

9.9 COST-VOLUME-PROFIT ANALYSIS As the name suggests, cost volume profit (CVP) analysis is the analysis of three variables cost, volume and profit. Such an analysis explores the relationship between costs, revenue, activity levels and the resulting profit. It aims at measuring variations in cost and volume. CVP analysis is based on the following assumptions: 1. Changes in the levels of revenues and costs arise only because of changes in the

number of product (or service) units produced and old – for example, the number of television sets produced and sold by Sony Corporation or the number of packages delivered by Overnight Express. The number of output units is the only revenue driver

9.13

Page 489: 30510870 Cost Accounting and Financial Management

Cost Accounting

and the only cost driver. Just as a cost driver is any factor that affects costs, a revenue driver is a variable, such as volume, that causally affects revenues.

2. Total costs can be separated into two components; a fixed component that does not vary with output level and a variable component that changes with respect to output level. Furthermore, variable costs include both direct variable costs and indirect variable costs of a product. Similarly, fixed costs include both direct fixed costs and indirect fixed costs of a product

3. When represented graphically, the behaviours of total revenues and total costs are linear (meaning they can be represented as a straight line) in relation to output level within a relevant range (and time period).

4. Selling price, variable cost per unit, and total fixed costs (within a relevant range and time period) are known and constant.

5. The analysis either covers a single product or assumes that the proportion of different products when multiple products are sold will remain constant as the level of total units sold changes

6. All revenues and costs can be added, subtracted, and compared without taking into account the time value of money. (Refer to the FM study material for a clear understanding of time value of money).

9.91 THE BREAKEVEN POINT The word contribution has been given its name because of the fact that it literally contributes towards the recovery of fixed costs and the making of profits. The contribution grows along with the sales revenue till the time it just covers the fixed cost. This point where neither profits nor losses have been made is known as a break-even point. This implies that in order to break even the amount of contribution generated should be exactly equal to the fixed costs incurred. Hence, if we know how much contribution is generated from each unit sold we shall have sufficient information for computing the number of units to be sold in order to break even. Mathematically,

Break even point in units = unit per onContributicosts Fixed

Let us consider an example of a company (ABC Ltd) manufacturing a single product, incurring variable costs of Rs 300 per unit and fixed costs of Rs 2, 00,000 per month. If the product sells for Rs 500 per unit, the breakeven point shall be calculated as follows;

9.14

Page 490: 30510870 Cost Accounting and Financial Management

Marginal Costing

Break even point in units = unit per onContributicosts Fixed

= 200 Rs.2,00,000 Rs.

= 1,000 units

9.92 MARGIN OF SAFETY The margin of safety can be defined as the difference between the expected level of sale and the breakeven sales. The larger the margin of safety , the higher are the chances of making profits. In the above example if the forecast sale is 1,700 units per month, the margin of safety can be calculated as follows, Margin of safety = Projected sales – Breakeven sales = 1,700 units – 1,000 units = 700 units or 41 % of sales. The Margin of Safety can also be calculated by identifying the difference between the projected sales and breakeven sales in units multiplied by the contribution per unit. This is possible because, at the breakeven point all the fixed costs are recovered and any further contribution goes into the making of profits.

9.93 CONTRIBUTION TO SALES RATIO(PROFIT VOLUME RATIO) This ratio is usually expressed in percentage. It can be calculated for the product in the example of ABC Ltd. which we have used above as follows, Contribution to sales ratio (C/S ratio) = Rs 200/Rs 500 ×100% = 40% A higher contribution to sales ratio implies that the rate of growth of contribution is faster than that of sales. This is because, once the breakeven point is reached, profits shall grow at a faster rate when compared to a product with a lesser contribution to sales ratio. Assuming that the products variable cost and selling price are constant, the contribution to sales ratio can also be utilised in computing the breakeven point as shown below,

Breakeven point in sales = ratio C/Scosts Fixed

= .402,00,000 Rs.

= Rs. 5,00,000

Rs. 5,00,000 sales equals 1,000 units being sold as has been computed while identifying breakeven point in units as shown above.

9.94 BREAKEVEN CHART A breakeven chart records costs and revenues on the vertical axis and the level of activity on the horizontal axis. The making of the breakeven chart would require you to select

9.15

Page 491: 30510870 Cost Accounting and Financial Management

Cost Accounting appropriate axes. Subsequently, you will need to mark costs/revenues on the Y axis whereas the level of activity shall be traced on the X axis. Lines representing (i) Fixed costs (horizontal line at Rs. 2,00,000 for ABC Ltd), (ii) Total costs at maximum level of activity (joined to the Yaxis where the Fixed cost of Rs. 2,00,000 is marked) and (iii) Revenue at maximum level of activity (joined to the origin) shall be drawn next. The breakeven point is that point where the sales revenue line intersects the total cost line. Other measures like the margin of safety and profit can also be measured from the chart. The breakeven chart for ABC Ltd is drawn below.

Contribution Breakeven chart It is not possible to use a breakeven chart as described above to measure contribution. This is one of its major limitations especially so because contribution analysis is literally the backbone of marginal costing. To overcome such a limitation, accountants frequently resort to the making of a contribution breakeven chart which is based on the same principles as a conventional breakeven chart except for that it shows the variable cost line instead of the fixed cost line. Lines for Total cost and Sales revenue remain the same. The breakeven point and profit can be read off in the same way as with a conventional chart. However it is also possible to read the contribution for any level of activity. Using the same example of ABC Ltd as for the conventional chart, the total variable cost for an output of 1,700 units is 1,700 × Rs. 300 = Rs. 5,10,000. This point can be joined to the origin since the variable cost is nil at zero activity.

9.16

Page 492: 30510870 Cost Accounting and Financial Management

Marginal Costing

The contribution can be read as the difference between the sales revenue line and the variable cost line. Profit-volume chart This is also very similar to a breakeven chart. In this chart the vertical axis represents profits and losses and the horizontal axis is drawn at zero profit or loss. In this chart each level of activity is taken into account and profits marked accordingly. The breakeven point is where this line interacts the horizontal axis. A profit-volume graph for our example (ABC Ltd) will be as follows,

Loss

The loss at a nil activity level is equal to Rs. 2,00,000, i.e. the amount of fixed costs. The second point used to draw the line could be the calculated breakeven point or the calculated profit for sales of 1,700 units.

9.17

Page 493: 30510870 Cost Accounting and Financial Management

Cost Accounting Advantages of the profit-volume chart The biggest advantage of the profit-volume chart is its capability of depicting clearly the effect on profit and breakeven point of any changes in the variables. The following example illustrates this characteristic, Example: A manufacturing company incurs fixed costs of Rs. 3,00,000 per annum. It is a single product company with annual sales budgeted to be 70,000 units at a sales price of Rs. 300 per unit. Variable costs are Rs. 285 per unit. (i) Draw a profit volume graph, and use it to determine the breakeven point.

The company is deliberating upon an increase in the selling price of the product to Rs.350 per unit. This shall be required in order to improve the quality of the product. It is anticipated that despite increase in the selling price the sales volume shall remain unaffected, however, the fixed costs shall increase to Rs. 4,50,000 per annum and the variable costs to Rs. 330 per unit.

(ii) Draw on the same graph as for part (a) a second profit volume graph and give your comments.

Solution: Figure showing changes with a profit-volume chart

9.18

Page 494: 30510870 Cost Accounting and Financial Management

Marginal Costing

Working notes (i) The profit for sales of 70,000 units is Rs. 7,50,000.

Rs.’000 Contribution 70,000 × Rs.(300 – 285) 1050 Fixed costs 300Profit 750

This point is joined to the loss at zero activity, Rs. 3,00,000 i.e., the fixed costs. Working notes (ii) The profit for sales of 70,000 units is Rs. 9,50,000.

Rs.’000 Contribution 70,000 × Rs. (350 – 330) 1400 Fixed costs 450Profit 950

This point is joined to the loss at zero activity, Rs. 4,50,000 i.e., the fixed costs. Comments: It is clear from the graph that there are larger profits available from option (ii). It also shows an increase in the break-even point from 20,000 units to 22,500 units, however, the increase of 2,500 units may not be considered large in view of the projected sales volume. It is also possible to see that for sales volumes above 30,000 units the profit achieved will be higher with option (ii). For sales volumes below 30,000 units option (i) will yield higher profits (or lower losses).

9.96 THE LIMITATIONS OF BREAKEVEN ANALYSIS: The limitations of the practical applicability of breakeven analysis and breakeven charts stem mostly from the assumptions underlying CVP which have been mentioned above. Assumptions like costs behaving in a linear fashion or sales revenue remain constant at different sales levels or the stocks shall remain constant period after period are unrealistic. Similarly, the assumption that the only factor which influences costs is the ‘activity level achieved’ is erroneous because other factors like inflation also have a bearing on costs.

9.19

Page 495: 30510870 Cost Accounting and Financial Management

Cost Accounting 9.97 SELF EXAMINATION QUESTIONS

MULTIPLE CHOICE QUESTIONS

1. A company makes and sells a single product. If the fixed cost incurred in making and selling the product increase:

a. The breakeven point will increase b. The breakeven point will decrease c. The breakeven point will remain the same d. Neither a nor b nor c 2. ABC Ltd manufactures a single product which it sells for Rs. 20 per unit. Fixed costs are

Rs. 60,000 per annum. The contribution to sales ratio is 40 percent. ABC Ltd breakeven point in units is

a. 7,500 b. 8,000 c. 7,000 d. 7,400 3. ABC Ltd sells a single product for Rs. 9 per unit. The variable cost is Rs. 6 per unit and

the fixed cost total Rs. 54,000 per month. In a period when the actual sales were Rs.1,80,000, ABC Ltd. Margin of Safety , in units was

a. 16,000 b. 18,000 c. 2,000 d. 17,000 4. Variable costs are budgeted to be 60% of the sales value whereas the fixed costs are

estimated as 10% of the sales value. If the company increases its selling price by 10% and fixed cost per unit, variable cost per unit and the sales volume remain the same, the effect of the contribution will be

a. An increase by 3% b. An increase by 10% c. A increase by 25% d. A increase by 30%

9.20

Page 496: 30510870 Cost Accounting and Financial Management

Marginal Costing

5. ABC Ltd plans to produce and sell 4,000 units of product C each month, at a selling price of Rs. 18 per unit. The unit cost comprises of Rs. 8 variable cost and Rs. 4 fixed cost. Calculate the monthly margin of safety, as a percentage of planned sales

a. 60% b. 70% c. 65% d. 75% 6. Product X generates a contribution to sales ratio of 30%. Fixed costs directly attributable

to X amount to Rs. 75,000 per month. Calculate the sales revenue required to achieve a monthly profit of Rs. 15,000

a. Rs. 2,00,000 b. Rs. 2,76,000 c. Rs. 3,00,000 d. Rs. 2,50,000 7. Which of the statements about the profit – volume graph are true? a. The profit lines passes through the origin b. Other things being equal, the angle of the profit line becomes steeper when the

selling price increases c. Contribution cannot be read directly from the chart d. Fixed costs are shown as line parallel to the horizontal axis 8. When comparing the profits reported under marginal and absorption costing during a

period when the level of stocks increased

a. Absorption costing profits will be higher and closing stock valuations lower than those under marginal costing

b. Absorption profits will be higher and closing stock valuations higher than those under marginal costing

c. Marginal costing profits will be higher and closing stock valuations lower than those under absorption costing

d. Marginal costing profits will be lower and closing stock valuations higher than those under absorption costing.

9.21

Page 497: 30510870 Cost Accounting and Financial Management

Cost Accounting 9. A company made 17,500 units at a total cost of Rs. 16 each. Three quarters of the cost

were variable and one quarter fixed. 15,000 units were sold at Rs. 25 each. There were no opening stocks. By how much will profit calculated under absorption costing differ from the profit if marginal costing principles were used?

a. Absorption costing profit will be Rs. 22,500 less. b. Absorption costing profit would be Rs. 10,000 greater. c. Absorption costing profit would be Rs. 1,35,000 greater. d. Costing profit would be Rs. 10,000 less. 10. A Rs. 1,30,000 absorption costing profit was made in a particular period which had an

opening and closing stock of 15,000 and 20,000 units respectively. If the fixed overhead absorption rate is Rs. 8 per unit, the marginal costing profit would be

a. Rs. 90,000 b. Rs. 1,30,000 c. Rs. 1,70,000 d. Impossible to calculate Short answer type questions 1. What is a marginal cost? 2. What is contribution? How is it related to profit. 3. What is a limiting or key factor. Give examples. 4. Why is it important to classify costs as fixed and variable. 5. What is a marginal cost equation? Long answer type questions 1. Differentiate between absorption costing and marginal costing. 2. What are the advantages and disadvantages of marginal costing? 3. Critically discuss the assumptions underlying CVP analysis. 4. How is a traditional breakeven chart different from the contribution breakeven chart.

Discuss. 5. What is a profit volume chart? State its advantages. Numerical questions 1. The Ward Company sold 1,00,000 units of its product at Rs. 20 per unit. Variable costs

9.22

Page 498: 30510870 Cost Accounting and Financial Management

Marginal Costing

are Rs. 14 per unit (manufacturing costs of Rs. 11 and selling costs of Rs. 3). Fixed costs are incurred uniformly throughout the year and amount to Rs. 7,92,000 (manufacturing costs of Rs. 500,000 and selling costs of Rs. 292,000). There are no beginning or ending inventories.

Required: Determine the following: a. The break-even point for this product

b. The number of units that must be sold to earn an income of Rs. 60,000 for the year (before income taxes)

c. The number of units that must be sold to earn an after-tax income of Rs. 90,000, assuming a tax-rate of 40 percent.

d. The break-even point for this product after a 10 percent increase in wages and salaries (assuming labour costs are 50 percent of variable costs and 20 percent of fixed costs).

2. ABC Ltd is planning a concert in a remote village in India. The following costs have been estimated,

Rs. Rent of premises 1,300 Advertising 1,000 Printing of tickets 250 Ticket sellers , security 400 Wages of ABC Ltd personnel employed at the concert 600 Fee to artist 1,000

There are no variable costs of staging the concert. The company is considering a selling price for tickets at either Rs. 4 or Rs. 5 each.

Required:

(a) Calculate the number of tickets that must be sold at each price in order to breakeven.

9.23

Page 499: 30510870 Cost Accounting and Financial Management

Cost Accounting

(b) Recalculate, the number of tickets which must be sold at each price in order to breakeven, if the artist agrees to change from a fixed fee of Rs. 1,000 to a fee equal to 25% of the gross sales proceeds.

(c) Calculate the level of ticket sales, for each price, at which the company would be indifferent as between the fixed and percentage fee alternatives.

(d) Comment on the factors which you think the company might consider in choosing between the fixed fee and percentage fee alternative.

9.24

Page 500: 30510870 Cost Accounting and Financial Management

CHAPTER 10

BUDGETS AND BUDGETARY CONTROL

Learning objectives

When you have finished studying this chapter, you should be able to

♦ Understand the objectives and importance of budgeting and budgetary control ♦ Understand the Advantages and disadvantages of budgetary control ♦ Differentiate between various types of budgets. ♦ Understand the process of preparation of budgets

10.1 INTRODUCTION

Budgetary control and Standard costing systems are two essential tools frequently used by business executives for the purpose of planning and control. In the case of budgetary control, the entire exercise starts with the setting up of budgets or targets and ends with the taking of an action, in case the actual figures differed with the budgetary ones.

The Chartered Institute of Management Accountants of England and Wales has defined the terms ‘budget’ and ‘budgetary control’ as follows :

Budget: “A financial and/or quantitative statement, prepared and approved prior to a defined period of time of the policy to be pursued during that period for the purpose of attaining a given objective. It may include income, expenditure and employment of capital”.

Budgets are usually, set up in the light of past experience after taking into account the changes that are expected to occur in the future. It is, therefore, to be expected that actual figures will correspond to the budget unless there is some important change in the conditions. In fact, it must be the constant endeavour of the management to see that actual performance does correspond with the budget concerned. Since budgets assume the optimum efficiency attainable, the system of budgetary control helps to increase efficiency and enables the concern to achieve the targets which are considered attainable.

Page 501: 30510870 Cost Accounting and Financial Management

Cost Accounting

10.2 OBJECTIVES OF BUDGETING

The process of budgeting is initiated with the establishment of specific targets of performance and is followed by executing plans to achieve such desired goals and from time to time comparing actual results with the targets of performances/goals. These targets include both the overall business targets as well as the specific targets for the individual units within the business. Establishing specific targets for future operations is part of the planning function of management, while executing actions to meet the goals is the directing function of management.

10.2.1 Planning : A set of targets/goals is often necessary to guide and focus individual and group actions. For example, students set academic goals, batsmen runs, employees set career goals, and business set financial goals. In the same way, budgeting supports the planning process by requiring all organisational units to establish their targets for the upcoming period. The targets, in turn, motivate individuals and groups to perform at high levels. Using the budget to communicate these expectations throughout the organization has helped many a companies to reduce expenses during a severe business recession.

Planning not only motivates employees to attain goals but also improves overall decision making. During the planning phase of the budget process, all viewpoints are considered, options identified, and cost reduction opportunities assessed. This process may reveal opportunities or threats that were not known prior to the budget planning process.

10.2.2 Directing: Once the budget plans are in place, they can be used to direct and coordinate operations in order to achieve the stated targets. For example, your target to receive “90%” in an exam would result in certain activities, such as reading books, completing assignments, participating in class, and studying for exams. Such actions are fairly easy to direct and coordinate. A business, however, is much more complex and requires more formal direction and coordination. The budget is one way to direct and coordinate business activities and units to achieve stated targets of performance. The budgetary units of an organisation are called responsibility centers. Each responsibility center is led by a manager who has the authority over and responsibility for the unit’s performance.

10.2.3 Controlling: As time passes, the actual performance of an operation can be compared against the planned targets. This provides prompt feedback to employees about their performance. If necessary. employees can use such feedback to adjust their activities in the future. For example, a salesperson may he given a quota to achieve Rs10,00,000 in sales for a particular period. If the actual sales are only Rs 8,75,000, the

* 10.2

Page 502: 30510870 Cost Accounting and Financial Management

Budgets and Budgetary Control

salesperson can use this feedback about underperformance to change sales tactics and improve future sales. Feedback is not only helpful to individuals, but it can also redirect a complete organisation, For example. McDonalds" Corporation recently decided to reverse its growth plans by closing stores and pulling out of a few countries as a result of reporting its first quarterly loss since becoming a public company in 1965.

Comparing actual results to the plan also helps prevent unplanned expenditures. The budget encourages employees to establish their spending priorities. For example, committees in professional Institutes have budgets to support faculty travel to conferences and meetings. The travel budget communicates to the officer the upper limit on travel. Often, desired travel exceeds the budget. Thus, the budget requires the officer to prioritise travel-related opportunities.

10.3 BUDGETARY CONTROL

It can be defined as “the establishment of budgets relating the responsibilities of executives to the requirements of a policy, and the continuous comparison of actual with budgeted results either to secure by individual action the objective of that policy or to provide a base for its revision”.

The salient features of such a system are the following :

(i) Determining the objectives to be achieved, over the budget period, and the policy or policies that might be adopted for the achievement of these ends.

(ii) Determining the variety of activities that should be undertaken for the achievement of the objectives.

(iii) Drawing up a plan or a scheme of operation in respect of each class of activity, in physical as well as monetary terms for the full budget period and its parts.

(iv) Laying out a system of comparison of actual performance by each person, section or department with the relevant budget and determination of causes for the discrepancies, if any.

(v) Ensuring that corrective action will be taken where the plan is not being achieved and, if that be not possible, for the revision of the plan.

In brief, it is a system to assist management in the allocation of responsibility and authority, to provide it with aid for making, estimating and planning for the future and to facilitate the analysis of the variation between estimated and actual performance. In order

10.3

Page 503: 30510870 Cost Accounting and Financial Management

Cost Accounting

that budgetary control may function effectively, it is necessary that the concern should develop proper basis of measurement or standards with which to evaluate the efficiency of operations, i.e., it should have in operation a system of standard costing. Beside this, the organisation of the concern should be so integrated that all lines of authority and responsibility are laid, allocated and defined. This is essential since the system of budgetary control postulates separation of functions and division of responsibilities and thus requires that the organisation shall be planned in such a manner that every one, from the Managing Director down to the Shop Foreman, will have his duties properly defined.

10.3.1 Objectives of Budgetary Control System : The objectives of a system of budgetary control are given below :

(i) Portraying with precision the overall aims of the business and determining targets of performance for each section or department of the business.

(ii) Laying down the responsibilities of each of the executives and other personnel so that every one knows what is expected of him and how he will be judged. Budgetary control is one of the few ways in which an objective assessment of executives or department is possible.

(iii) Providing a basis for the comparison of actual performance with the predetermined targets and investigation of deviation, if any, of actual performance and expenses from the budgeted figures. This naturally helps in adopting corrective measures.

(iv) Ensuring the best use of all available resources to maximise profit or production, subject to the limiting factors. Since budgets cannot be properly drawn up without considering all aspects usually there is good co-ordination when a system of budgetary control operates.

(v) Co-ordinating the various activities of the business, and centralising control and yet enabling management to decentralise responsibility and delegate authority in the overall interest of the business.

(vi) Engendering a spirit of careful forethought, assessment of what is possible and an attempt at it. It leads to dynamism without recklessness. Of course, much depends on the objectives of the firm and the vigour of its management.

(vii) Providing a basis for revision of current and future policies.

(viii) Drawing up long range plans with a fair measure of accuracy.

* 10.4

Page 504: 30510870 Cost Accounting and Financial Management

Budgets and Budgetary Control

(ix) Providing a yardstick against which actual results can be compared.

10.3.2 Working of a Budgetary Control System : The responsibility for successfully introducing and implementing a Budgetary Control System rests with the Budget Committee acting through the Budget Officer. The Budget Committee would be composed of all functional heads and a member from the Board to preside over and guide the deliberations. The main responsibilities of the Budget Officer are :

(i) to assist in the preparation of the various budgets by coordinating the work of the accounts department which is normally responsible to compile the budgets—with the relevant functional departments like Sales, Production, Plant maintenance etc.;

(ii) to forward the budget to the individuals who are responsible to adhere to them, and to guide them in overcoming any practical difficulties in its working;

(iii) to prepare the periodical budget reports for circulation to the individuals concerned;

(iv) to follow-up action to be taken on the budget reports;

(v) to prepare an overall budget working report for discussion at the Budget Committee meetings and to ensure follow-up on the lines of action suggested by the Committee;

(vi) to prepare periodical reports for the Board meeting. Comparing the budgeted Profit and Loss Account and the Balance Sheet with the actual results attained.

It is necessary that every budget should be thoroughly discussed with the functional head before it is finalised. It is the duty of the Budget Officer to see that the periodical budget reports are supplied to the recipients at frequent intervals as far as possible. The efficiency of the Budget Officer, and through him of the Budget Committee, will be judged more by the smooth working of the system and the agreement between the actual figures and the budgeted figures. Budgets are primarily an incentive and a challenge for better performance; it is up to the Budget Officer to see that attention of the different functional heads is drawn to it to face the challenge in a successful manner.

10.3.3 Advantages of Budgetary Control System

1. The use of budgetary control system enables the management of a business concern to conduct its business activities in the efficient manner.

2. It is a powerful instrument used by business houses for the control of their expenditure. It infact provides a yardstick for measuring and evaluating the performance of individuals and their departments.

10.5

Page 505: 30510870 Cost Accounting and Financial Management

Cost Accounting

3. It reveals the deviations to management, from the budgeted figures after making a comparison with actual figures.

4. Effective utilisation of various resources like—men, material, machinery and money—is made possible, as the production is planned after taking them into account.

5. It helps in the review of current trends and framing of future policies.

6. It creates suitable conditions for the implementation of standard costing system in a business organisation.

7. It inculcates the feeling of cost consciousness among workers.

10.3.4 Limitations of Budgetary Control System : The limitations of budgetary control system are as follows :

1. Budgets may or may not be true, as they are based on estimates.

2. Budgets are considered as rigid document.

3. Budgets cannot be executed automatically.

4. Staff co-operation is usually not available during budgetary control exercise.

5. Its implementation is quite expensive.

10.3.5 Components of Budgetary Control System : The policy of a business for a defined period is represented by the master budget the details of which are given in a number of individual budgets called functional budgets. These functional budgets are broadly grouped under the following heads :

(i) Physical budgets - Those budgets which contains information in terms of physical units about sales, production etc. for example, quantity of sales, quantity of production, inventories, and manpower budgets are physical budgets.

(ii) Cost budgets - Budgets which provides cost information in respect of manufacturing, selling, administration etc. for example, manufacturing costs, selling costs, administration cost, and research and development cost budgets are cost budgets.

(iii) Profit budgets - A budget which enables in the ascertainment of profit, for example, sales budget, profit and loss budget, etc.

(iv) Financial budgets - A budget which facilitates in ascertaining the financial position of

* 10.6

Page 506: 30510870 Cost Accounting and Financial Management

Budgets and Budgetary Control

a concern, for example, cash budgets, capital expenditure budget, budgeted balance sheet etc.

10.4 DIFFERENT TYPES OF BUDGETS

Budgets may be classified by : Capacity, coverage they encompass, periods which they cover and conditions on which they are based.

BUDGET

Capacity Coverage Period Conditions Fixed Flexible Functional Master Long Short Basic Current Budgets Budgets Budgets term Budgets term Budgets Budgets Budgets Budgets

Fixed budget - According to Chartered Institute of Management Accountants of England, “a fixed budget, is a budget designed to remain unchanged irrespective of the level of activity actually attained”. A fixed budget shows the expected results of a responsibility center for only one activity level. Once the budget has been determined, it is not changed, even if the activity changes. Fixed budgeting is used by many service companies and for some administrative functions of manufacturing companies, such as purchasing, engi-neering, and accounting. Fixed Budget is used as an effective tool of cost control. In case, the level of activity attained is different from the level of activity for budgeting purposes, the fixed budget becomes ineffective. Such a budget is quite suitable for fixed expenses. It is also known as a static budget.

Flexible budget - Unlike static budgets, flexible budgets show the expected results of a responsibility center for several activity levels. You can think of a flexible budget as a series of static budgets for different levels of activity. Such budgets are especially useful in estimating and controlling factory costs and operating expenses. It is more realistic and practicable because it gives due consideration to cost behaviour at different levels of activity. While preparing a flexible budget the expenses are classified into three categories viz.

(i) Fixed,

(ii) Variable, and

(iii) Semi-variable.

10.7

Page 507: 30510870 Cost Accounting and Financial Management

Cost Accounting

Semi-variable expenses are further segregated into fixed and variable expenses. Flexible budgeting may be resorted to under following situations:

(i) In the case of new business venture due to its typical nature it may be difficult to forecast the demand of a product accurately.

(ii) Where the business is dependent upon the mercy of nature e.g., a person dealing in wool trade may have enough market if temperature goes below the freezing point.

(iii) In the case of labour intensive industry where the production of the concern is dependent upon the availability of labour.

Functional budgets - Budgets which relate to the individual functions in an organisation are known as Functional Budgets. For example, purchase budget; sales budget; production budget; plant-utilisation budget and cash budget.

Master budget - It is a consolidated summary of the various functional budgets. It serves as the basis upon which budgeted P & L A/c and forecasted Balance Sheet are built up.

Long-term budgets - The budgets which are prepared for periods longer than a year are called long-term budgets. Such budgets are helpful in business forecasting and forward planning. Capital expenditure budget and Research and Development budget are exam-ples of long-term budgets.

Short-term budgets - Budgets which are prepared for periods less than a year are known as short-term budgets. Cash budget is an example of short-term budget. Such types of budgets are prepared in cases where a specific action has to be immediately taken to bring any variation under control, as in cash budgets.

Basic budgets - A budget which remains unaltered over a long period of time is called basic budget.

Current budgets - A budget which is established for use over a short period of time and is related to the current conditions is called current budget.

10.5 PREPARATION OF BUDGETS

(a) Definition of objectives - A budget being a plan for the achievement of certain operational objectives, it is desirable that the same are defined precisely. The objectives should be written out; the areas of control demarcated; and items of revenue and expenditure to be covered by the budget stated. This will give a clear understanding of the

* 10.8

Page 508: 30510870 Cost Accounting and Financial Management

Budgets and Budgetary Control

plan and its scope to all those who must cooperate to make it a success.

(b) Location of the key (or budget) factor - There is usually one factor (sometimes there may be more than one) which sets a limit to the total activity. For instance, in India today sometimes non-availability of power does not allow production to increase inspite of heavy demand. Similarly, lack of demand may limit production. Such a factor is known as key factor. For proper budgeting, it must be located and estimated properly.

(c) Appointment of controller - Formulation of a budget usually required whole time services of a senior executive; he must be assisted in this work by a Budget Committee, consisting of all the heads of department along with the Managing Director as the Chairman. The Controller is responsible for co-ordinating and development of budget programmes and preparing the manual of instruction, known as Budget manual. The Budget manual is a schedule, document or booklet which shows, in written forms the budgeting organisation and procedures. The manual should be well written and indexed so that a copy thereof may be given to each departmental head for guidance.

(d) Budget period - The period covered by a budget is known as budget period. There is no general rule governing the selection of the budget period. In practice the Budget Committee determines the length of the budget period suitable for the business. Normally, a calendar year or a period coterminous with the financial year is adopted. The budget period is then sub-divided into shorter periods—it may be months or quarters or such periods as coincide with period of trading activity.

(e) Standard of activity or output - For preparing budgets for the future, past statistics cannot be completely relied upon, for the past usually represents a combination of good and bad factors. Therefore, though results of the past should be studied but these should only be applied when there is a likelihood of similar conditions repeating in the future. Also, while setting the targets for the future, it must be remembered that in a progressive business, the achievement of a year must exceed those of earlier years. Therefore what was good in the past is only fair for the current year.

In budgeting, fixing the budget of sales and of capital expenditure are most important since these budgets determine the extent of development activity. For budgeting sales, one must consider the trend of economic activity of the country, reactions of salesmen, customers and employees, effect of price changes on sales, the provision for advertisement campaign plan capacity etc.

10.9

Page 509: 30510870 Cost Accounting and Financial Management

Cost Accounting

10.5.1 Functional budget - A functional budget is one which is related to function of the business as for example, production budget relating to the manufacturing function. Functional budgets are prepared for each function and they are subsidiary to the master budget of the business. The various types of functional budgets to be prepared will vary according to the size and nature of the business. The various commonly used functional budgets are :

(i) Sales budget

(ii) Production budget

(iii) Plant utilisation budget

(iv) Direct-material usage budget

(v) Direct-material purchase budget

(vi) Direct-labour (personnel) budget

(vii) Factory overhead budget

(viii) Production cost budget

(ix) Ending-inventory budget

(x) Cost-of-goods-sold budget

(xi) Selling and distribution cost budget

(xii) Administration expenses budget

(xiii) Research and development cost budget

(xiv) Capital expenditure budget

(xv) Cash budget

(xvi) Budget summaries/Master budget - Budgeted income statement and Budgeted balance sheet.

The important functional budgets (also known as schedules to master budget) and the master budget are discussed and illustrated below :

Sales budget - Sales forecast is the commencement of budgeting and hence sales budget assumes primary importance. The quantity which can be sold may be the principal budget factor in many business undertakings. In any case in order to chalk out a realistic

* 10.10

Page 510: 30510870 Cost Accounting and Financial Management

Budgets and Budgetary Control

budget programme, there must be an accurate sales forecast. The sales budget indicates for each product (1) the quantity of estimated sales and (2) the expected unit selling price. These data are often reported by regions or by sales representatives.

In estimating the quantity of sales for each product, past sales volumes are often used as a starting point. These amounts are revised for factors that are expected to affect future sales, such as the factors listed below.

♦ backlog of unfilled sales orders ♦ planned advertising and promotion ♦ expected industry and general economic conditions ♦ productive capacity ♦ projected pricing ♦ findings of market research studies ♦ relative product profitability. ♦ competition. Once an estimate of the sales volume is obtained, the expected sales revenue can be determined by multiplying the volume by the expected unit sales price,The sales budget represents the total sales in physical quantities and values for a future budget period. Sales managers are constantly faced with problem like anticipation of customer requirements, new product needs, competitor strategies and various changes in distribution methods or promotional techniques.

The purposes of sales budget is not to attempt to estimate or guess what the actual sales will be, but rather to develop a plan with clearly defined objectives towards which the operational effort is directed in order to attain or exceed the objective. Hence, sales budget is not merely a sales forecast. A budget is a planning and control document which shows what the management intends to accomplish. Thus, the sales budget is active rather than passive. A sales forecast, however, is a projection or estimate of the available customer demand. A forecast reflects the environmental or competitive situation facing the company whereas the sales budget shows how the management intends to react to this environmental and competitive situation. A good budget hinges on aggressive management control rather than on passive acceptance of what the market appears to offer. If the company fails to make this distinction, the budget will remain more a figure-work exercise than a working tool of dynamic management control.

10.11

Page 511: 30510870 Cost Accounting and Financial Management

Cost Accounting

The sales budget may be prepared under the following classification or combination of classifications :

(a) Products or groups of products.

(b) Areas, towns, salesmen and agents.

(c) Types of customers as for example: (i) Government, (ii) Export, (iii) Home sales, (iv) Retail depots.

(d) Period—months, weeks, etc.

The format of a sales budget will be as under :

Last Year Budget Year Northern Southern Central Total Total Region Region Region Qty. Value Qty. Value Qty. Value Qty. Value Qty. Value Product X 1st quarter 2nd quarter 3rd quarter 4th quarter Product Y Total Grand Total

Example of sales budget:

XYZ COMPANY

Sales Budget For the Year Ending March, 20.... Units Selling price Total Per unit (Rs.) (Rs.) Product A 5,000 75 3,75,000 Product B 10,000 80 8,00,000 11,75,000

* 10.12

Page 512: 30510870 Cost Accounting and Financial Management

Budgets and Budgetary Control

Production budget - Production budget shows the production for the budget period based upon :

(a) Sales budget,

(b) Production capacity of the factory,

(c) Planned increase or decrease in finished stocks, and

(d) Policy governing outside purchase.

Production budget is normally stated in units of output. Production should be carefully coordinated with the sales budget to ensure that production and sales are kept in balance during the period. The number of units to be manufactured to meet budgeted sales and inventory needs for each product is set forth in the production budget The production facility available and the sales budget will be compared and coordinated to determine the production budget. If production facilities are not sufficient, consideration may be given to such factors as working overtime, introducing shift working, sub-contracting or purchasing of additional plant and machinery. If, however, the production facilities are surplus, consideration should be given to promote advertising, reduction of prices to increase the sales, sub-contracting of surplus capacity, etc.

One of the conditions to be considered in all the compilation of production budget is the level of stock to be maintained. The level of stocks will depend upon three factors viz. : (a) seasonal industries in which stocks have to be built up during off season to cater to

the peak season,

(b) a steady and uniform level of production to utilise the plant fully and to avoid retrenchment or lay-off of the workers, and

(c) to produce in such a way that minimum stocks are maintained at any time to avoid locking up of funds in inventory.

Production budget can, therefore, show : (a) stabilised production every month, say, the maximum possible production or (b) stabilised minimum quantity of stocks which will reduce inventory costs.

In the case of stabilised production, the production facility will be fully utilised but the inventory carrying costs will vary according to stocks held. In the case of stabilised stocks method, however, the inventory carrying will be the lowest but there may be under-utilisation of capacity.

10.13

Page 513: 30510870 Cost Accounting and Financial Management

Cost Accounting

Example of production budget:

XYZ COMPANY Production budget in units for the year ending March 31, 20....

Products A B Budgeted sales 5,000 10,000 Add : Desired closing stock 500 1,000 Total quantity required 5,500 11,000 Less : Opening stock 1,500 2,000 Units to be produced 4,000 9,000 Plant utilisation budget - Plant utilisation budget represents, in terms of working hours, weight or other convenient units of plant facilities required to carry out the programme laid down in the production budget. The main purposes of this budget are :

(a) To determine the load on each process, cost or groups of machines for the budget period.

(b) To indicate the processes or cost centres which are overloaded so that corrective action may be taken such as : (i) working overtime (ii) sub-contracting (iii) expansion of production facility, etc.

(c) To dovetail the sales production budgets where it is not possible to increase the capacity of any of the overloaded processes.

(d) Where surplus capacity is available in any of the processes, to make effort to boost sales to utilise the surplus capacity.

Direct material usage budget - The steps involved in the compilation of direct materials usage budget are as under :

(i) The quality standards for each item of material have to be specified. In this connection, standardisation of size, quality, colour, etc., may be considered.

(ii) Standard requirement of each item of materials required should also be set. While setting the standard quality consideration should be given to normal loss in process. The standard allowance for normal loss may be given on the basis of past performance, test runs, technical estimates etc.

* 10.14

Page 514: 30510870 Cost Accounting and Financial Management

Budgets and Budgetary Control

(iii) Standard prices for each item of materials should be set after giving consideration to stock and contracts entered into.

After setting standards for quality, quantity and prices, the direct materials budget can be prepared by multiplying each item of material required for the production by the standard price.

Example of direct material usage budget is as under :

XYZ COMPANY Direct material usage in units and Rs.

for the year ending March 31, 20... Direct Materials Type of material Product A Product B Total direct Material Total cost

(4,000 units) (9,000 units) material cost per of material

usage (Units) unit (Rs.) used (Rs.) X (12 units per finished product) 48,000 1,08,000 1,56,000 1.50 2,34,000 Y (4 units per product A & 2 units per product B) 16,000 18,000 34,000 2.50 85,000 Total 3,19,000 Purchase budget - The production budget is the starting point for determining the estimated quantities of direct materials to be purchased. Multiplying these quantities by the expected unit purchase price determines the total cost of direct materials to be purchased.

Two important considerations that govern purchase budgets are as follows:

(a) Economic order quantity.

(b) Re-order point with safety stocks to cover fluctuations in demand.

The direct material purchases budget helps management maintain inventory levels within reasonable limits, For this purpose, the timing of the direct materials purchases should he coordinated between the purchasing and production departments.

10.15

Page 515: 30510870 Cost Accounting and Financial Management

Cost Accounting

An example of material purchase budget is as under :

XYZ Company Direct material purchase budget

for the year ending March 31, 20..... Material X Material Y Total Desired closing stock (units) 3,000 500 Units required for production 1,56,000 34,000 Add : Total needs 1,59,000 34,500 Less: Opening stock (units) 4,000 300 Units to be purchased 1,55,000 34,200 Unit price (Rs.) 1.50 2.50 Purchase cost (Rs.) 2,32,500 85,500 3,18,000 Personnel (or Labour cost) budget - Once sales budget and Production budget are compiled and thereafter plant utilisation budget is settled, detailed amount of the various machine operations involved and services required can be arrived at. This will facilitate preparation of an estimate of different grades of labour required. From this the standard hours required to be worked can be prepared. The total labour complement thus budgeted can be divided into direct and indirect. Standard rates of wages for each grade of labour can be introduced and then the direct and indirect labour cost budget can be prepared.

The advantages of labour budget are the following:

(a) It defines the direct and indirect labour force required.

(b) It enables the personnel department to plan ahead in recruitment and training of workers so that labour turnover can be reduced to the minimum.

(c) It reveals the labour cost to be incurred in the manufacture, to facilitate preparation of manufacturing cost budgets and cash budgets for financing the wage bill.

* 10.16

Page 516: 30510870 Cost Accounting and Financial Management

Budgets and Budgetary Control

Example of direct-labour cost budget:

XYZ COMPANY Direct-labour cost budget

for the year ending March 31, 20...

Units to be Direct labour Total Total budget cost (Rs.) produced hour, per unit hours @ Rs. 2 per hour

Product A 4,000 7 28,000 56,000 Product B 9,000 10 90,000 1,80,000 1,18,000 2,36,000 Production or Factory overhead budget - Production overheads consists of all items such as indirect materials, indirect labour and indirect expenses. Indirect expenses include power, fuel, fringe benefits, depreciation etc. These estimated factory overhead costs necessary for production make up the factory overhead cost budget. This budget usually includes the total estimated cost for each item of factory overhead. The production overhead budget is useful for working out the pre-determined overhead recovery rates. A business may prepare supporting departmental schedules, in which the factory overhead costs are separated into their fixed and variable cost elements. Such schedules enable department managers to direct their attention to those costs for which they are responsible and to evaluate performance

A careful study and determination of the behaviour of different types of costs will be essential in preparation of overhead budget. A few examples are given below to show how the expenses are estimated.

(a) Fixed expenses are policy cost and hence they are based on policy matters.

(b) For estimating indirect labour, work study is resorted to and a flexible estimate of number of indirect workers required for each level of direct workers employed is made—for example, one supervisor for every twenty direct workers.

(c) In regard to the estimate of consumption of indirect materials, the age and condition of the plant and machinery are taken into consideration.

10.17

Page 517: 30510870 Cost Accounting and Financial Management

Cost Accounting

Example of factory overhead budget :

XYZ COMPANY Factory overhead budget for the year ending March 31, 20....

(Anticipated activity of 1,18,000 direct labour hours)

Rs. Rs. Supplies 12,000 Indirect labour 30,000 Cost of fringe benefits 10,000 Power (variable portion) 22,000 Maintenance cost (variable portion) 15,000 Total variable overheads 89,000 Depreciation 10,000 Property taxes 2,000 Property insurance 1,000 Supervision 12,000 Power (Fixed portion) 800 Maintenance (Fixed portion) 3,200 Total fixed overheads 29,000 Total factory overheads 1,18,000 Factory overhead recovery rate is :

hours abourl000,18,1`000,18,1.Rs = Rs.1 per direct labour hour

Production cost budget - Production cost budget covers direct material cost, direct labour cost and manufacturing expenses. After preparing direct material, direct labour and production overhead cost budget, one can prepare production cost budget.

Ending Inventory budget - This budget shows the cost of closing stock of raw materials and finished goods, etc. This information is required to prepare cost-of-goods-sold budget and budgeted financial statements i.e., budgeted income statement and budgeted balance sheet.

* 10.18

Page 518: 30510870 Cost Accounting and Financial Management

Budgets and Budgetary Control

Example of ending inventory budget :

XYZ Company ending-inventory budget March 31, 20....

Units Unit cost Amount Total Rs. Rs. Rs. Direct material X 3,000 1.50 4,500 Y 500 2.50 1,250 5,750 Finished goods A 500 49.00* 24,500 B 1,000 53.00* 53,000 77,500 Total 83,250 * Unit cost of finished goods have been computed as below :

Unit cost Product A Product B of input Units Amount Units Amount

(Rs.) (Rs.) (Rs.) Material X 1.50 12 18.00 12.00 18.00 Material Y 2.50 4 10.00 2.00 5.00 Direct labour 2.00 7 14.00 10.00 20.00 Factory overhead 1.00 7 7.00 10.00 10.00 49.00 53.00 Cost of goods sold budget - This budget covers direct material cost, direct labour cost, manufacturing expenses and cost of ending inventory of finished products. We present below the cost-of-goods-sold budget on the basis of the data taken from the various budgets already illustrated:

XYZ Company cost-of-goods-sold budget for the year ending March 31, 20....

Amount Rs. Direct materials used 3,19,000 Direct labour 2,36,000

10.19

Page 519: 30510870 Cost Accounting and Financial Management

Cost Accounting

Factory overhead 1,18,000 Total manufacturing costs 6,73,000 Add : Finished goods (opening) 1,79,500* 8,52,500 Less : Finished goods (closing) 77,500* Total cost of goods sold 7,75,000

*Assumed given In the above budget if adjustments for opening and closing inventory of finished goods are not shown. The budget will be called production cost budget.

Selling and distribution cost budget - Selling and distribution are indispensable aspects of the profit earning function. At the same time, the pre-determination of these costs is also very difficult.

Selling cost is defined as the cost of seeking to create and stimulate demand and of securing orders. These costs are, therefore, incurred to maintain and increase the level of sales. All expenses connected with advertising, sales promotion, sales office, salesmen, credit collection, market research, after sales service, etc. are generally grouped together to form part of the responsibility of the sales manager. While making a budget, selling costs are divided into fixed and variable. Semi-variable costs should also be separated into variable and fixed elements. The problems faced in the preparation of selling cost budgets are :

(i) heavy expenditure on selling and sales promotion may have to be incurred when the volume of sales is falling off. This will increase the percentage of such costs to total sales, and

(ii) sometimes intensive sales and promotion efforts are called for in one year and the benefit of such efforts accrue in the subsequent years. This makes it difficult to establish a proportion of selling cost to sales.

In spite of these problems, some relationship between selling cost and volume of sales has to be established and it is the duty of the Budget Controller to determine the amount of selling costs to be incurred to achieve the desired level of sales volume. Using the past experience as a guide, consideration should be given to the future trend of sales, possible changes in competition etc., in pre-determination of selling costs.

Distribution cost has been defined as the cost of the sequence of operations which begins

* 10.20

Page 520: 30510870 Cost Accounting and Financial Management

Budgets and Budgetary Control

with making the packet of product available for despatch and ends with making the re-conditioned return of empty package, if any available for re-use. It includes transport cost, storage and warehousing costs, etc.

Preparation of the advertising cost budget is the responsibility of the sales manager or advertisement manager. When preparing the advertisement cost budget consideration should be given to the following factors :

(a) The best method of advertisement must be selected; costs will vary according to the method selected.

(b) The maximum amount to be spent in a period, say one year, has to be decided.

(c) Advertising and sales should be co-ordinated. It means that money should be spent on advertisement only when sufficient quantities of the product advertised are ready for sale.

(d) An effective control over advertisement expenditure should be exercised and the effectiveness of the advertisement should be measured.

The choice of the method of advertising a product is based on the effectiveness of the money spent on advertisement in increasing or maintaining sales. If the output sold increases, the production cost will come down because of the economies of large scale production.

The amount to be spent on advertisement appropriation may be settled on the basis of the following factors:

(i) A percentage on the total sales value of the budget period or on the expected profit may be fixed on the basis of past experience.

(ii) A sum which is expected to be incurred by the competitors may be fixed to be spent during the budget period.

(iii) A fixed sum per unit of output can be fixed and added to cost.

(iv) An amount is fixed on the basis of the ability of the company to spend on advertising.

(v) An advertisement plan is decided upon and the amount to be spent is determined.

Depending upon the nature of the product and the effectiveness of the media of the advertising the company prepares a schedule of various methods of advertisement, to be used for effective sales promotion. The number of advertisements (insertions) are

10.21

Page 521: 30510870 Cost Accounting and Financial Management

Cost Accounting

determined and the cost calculated as per the rates applicable to each of the media selected. This is a sound method.

Example of selling and distribution cost budget:

XYZ Company selling and distribution cost budget for the year ending March 31, 20....

Amount Direct selling expenses Rs. Salesmen’s salaries 14,500 Salesmen’s commission 7,000 Travelling expenses 19,000 40,500 Distribution expenses Warehouse wages 6,000 Warehouse rent, rates, electricity 4,500 Lorry expenses 11,000 21,500 Sales office expenses Salaries 16,000 Rent, rates, electricity 12,000 Depreciation 2,000 Stationery, postage and telephone 12,500 General expenses 3,000 45,500 Advertising Press 4,500 Radio and television 18,500 Shop window displays 4,000 27,000 Total 1,34,500

* 10.22

Page 522: 30510870 Cost Accounting and Financial Management

Budgets and Budgetary Control

Administrative expenses budget - The administrative expenses are mostly policy costs and are, therefore, fixed in nature. The most practical method to follow in preparing estimate of these expenses is to follow the past experience with due regard to anticipated changes either in general policy or the volume of business. To bring such expenses under control, it is necessary to review them frequently and to determine at regular intervals whether or not these expenses continue to be adjusted. Examples of such expenses are : audit fees, depreciation of office equipment, insurance, subscriptions, postage, stationery, telephone, telegrams, office supplies, etc.

XYZ Company administrative expenses budget for the year ending March 31, 20...

Rs. Salaries of clerical staff 28,000 Executives salaries 8,000 Audit fee 600 Depreciation on office equipment 800 Insurance 250 Stationery 1,250 Postage and telegrams 950 Telephones 850 Miscellaneous 5,300Total administrative expenses 46,000

Research and development expense budget - Research is required in order to develop and/or improve products and methods. When research results in definite benefit to the company, development function begins. After development, formal production can commence on commercial scale and then production function starts. Since the areas of research and development cannot be precisely defined, the costs incurred under both the functions are clubbed together as research and development costs. Research and Development (R & D) plays a vital role in maintaining the business. For example, automobile manufacturers, and those who produce drugs, spend considerable sums on R & D to improve the products.

Research may be either pure research or applied research. Pure research increases

10.23

Page 523: 30510870 Cost Accounting and Financial Management

Cost Accounting

knowledge whereas applied research aims at producing definite results like improved methods of production, etc.

Research and development expenses should be controlled carefully and hence a limit on the spending is placed, i.e., the amount to be spent is carefully determined or allocated. The following are the methods of allocation of R & D expenses.

(1) A percentage based on total sales value. This method is good if sales value is steady from year to year.

(2) A percentage based on net profit.

(3) A total sum is estimated on the basis of past experience and future R & D plans and policies.

(4) A sum is fixed on the basis of cash resources available with the company.

(5) All factors which affect the importance of R & D are considered. For example, factors like demand for existing products, competition, economic conditions, etc., are considered carefully and a sum is set as R & D budget.

Capital expenditure budget - The capital expenditure budget represents the planned outlay on fixed assets like land, building, plant and machinery, etc. during the budget period. This budget is subject to strict management control because it entails large amount of expenditure. The budget is prepared to cover a long period of years and it projects the capital costs over the period in which the expenditure is to be incurred and the expected earnings. The preparation of this budget is based on the following considerations :

(i) Overhead on production facilities of certain departments as indicated by the plant utilisation budget.

(ii) Future development plans to increase output by expansion of plant facilities.

(iii) Replacement requests from the concerned departments.

While preparing the capital expenditure budget, consideration should also be given to factors like sales potential to absorb the increased output, possibility of price reductions, increased costs of advertising and sales promotion to absorb increased output, etc.

* 10.24

Page 524: 30510870 Cost Accounting and Financial Management

Budgets and Budgetary Control

The advantages of capital expenditure budget are the following :

(1) It outlines the capital development programme and estimated capital expenditure during the budget period.

(2) It enables the company to establish a system of priorities. When there is a shortage of funds, capital rationing becomes necessary.

(3) It serves as a tool for controlling expenditure.

(4) It provides the amount of expenditure to be incorporated in the future budget summaries for calculation of estimated return on capital employed.

(5) This enables the cash budget to be completed. With other cash commitments capital expenditure commitment should also be considered for the completion of the budget.

(6) It facilitates cost reduction programme, particularly when modernisation and renovation is covered by this budget.

10.5.2 Cash budget - Cash budget represents the cash requirements of the business during the budget period. It is the plan of receipts and payments of cash for the budget period, analysed to show the monthly flow of cash drawn up in such a way that the balance can be forecasted at regular intervals. The cash budget is one of the most important elements of the budgeted balance sheet. Information from the various operating budgets, such as the sales budget, the direct materials purchases budget, and the selling and administrative expenses budget, affects the cash budget. In addition, the capital expenditures budget, dividend policies, and plans for equity or long-term debt financing also affect the cash budget.

10.5.3 Master budget - When all the necessary functional budgets have been prepared, the budget officer will prepare the master budget which may consist of budgeted profit and loss account and budgeted balance sheet. These are in fact the budget summaries. When the master budget is approved by the board of directors, it represents a standard for the achievement of which all the departments will work. On the basis of the various budgets (schedules) prepared earlier in this study, we prepare below budgeted income statement and budgeted balance sheet.

10.25

Page 525: 30510870 Cost Accounting and Financial Management

Cost Accounting

Example of budgeted income statement :

XYZ Company Budgeted Income Statement For the Year Ending March 31, 20....

Amount Rs. Rs. Sales 11,75,000 Less : Cost of goods sold 7,75,000 Gross margin 4,00,000 Less : Selling and distribution expenses 1,36,500 Less : Administrative expenses 46,000 1,82,500 Profit before interest and taxes 2,17,500 Interest expenses (assumed) 50,000 Profit before tax 1,67,500 Income-tax (50% assumed) 83,750 Net profit 83,750

Example of budgeted balance sheet :

XYZ Company Budgeted Balance Sheet March 31, 20....

Rs. Rs. Rs. Share capital 3,50,000 Retained income 1,29,000 4,79,000 Represented by : Plant and machinery 3,40,000 Less : Provision for depreciation 60,000 2,80,000 Raw materials 5,750 Finished goods 77,500 Debtors 1,10,000 Cash 37,750 2,31,000 Less: Creditors 32000 1,99,000 4,79,000

* 10.26

Page 526: 30510870 Cost Accounting and Financial Management

Budgets and Budgetary Control

Note : Information not available in respect of share capital, opening balance of retained earnings, current assets and current liabilities, etc., has been assumed to complete the above balance sheet.

14.4.4 Flexible budget : Definition - A flexible budget is defined as “a budget which, by recognising the difference between fixed, semi-variable and variable costs is designed to change in relation to the level of activity attained”. A fixed budget, on the other hand is a budget which is designed to remain unchanged irrespective of the level of activity actually attained. In a fixed budgetary control, budgets are prepared for one level of activity whereas in a flexible budgetary control system, a series of budgets are prepared one for each of a number of alternative production levels or volumes. Flexible budgets represent the amount of expenses that is reasonably necessary to achieve each level of output specified. In other words, the allowances given under flexible budgetary control system serve as standards of what costs should be at each level of output.

Need : The need for the preparation of the flexible budgets arises in the following circumstances :

(i) seasonal fluctuations in sales and/or production, for example in soft drinks industry;

(ii) a company which keeps on introducing new products or makes changes in the design of its products frequently;

(iii) industries engaged in make-to-order business like ship building;

(iv) an industry which is influenced by changes in fashion; and

(v) general changes in sales.

Distinction between Fixed and Flexible Budget

Fixed Budget Flexible Budget 1. It does not change with actual volume of activity achieved. Thus it is known as rigid or inflexible budget.

It can be recasted on the basis of activity level to be achieved. Thus it is not rigid.

2. It operates on one level of activity and under one set of conditions. It assumes that there will be no change in the prevailing conditions, which is unrealistic.

It consists of various budgets for different levels of activity

3. Here as all costs like - fixed, variable and semi-variable are related to only one

Here analysis of variance provide useful information as each cost is analysed

10.27

Page 527: 30510870 Cost Accounting and Financial Management

Cost Accounting

level of activity so variance analysis does not give useful information.

according to its behaviour.

4. If the budgeted and actual activity levels differ significantly, then the aspects like cost ascertainment and price fixation do not give a correct picture.

Flexible budgeting at different levels of activity, facilitates the ascertainment of cost, fixation of selling price and tendering of quotations.

5. Comparison of actual performance with budgeted targets will be meaningless specially when there is a difference between the two activity levels.

It provides a meaningful basis of comparison of the actual performance with the budgeted targets.

Illustration

A factory which expects to operate 7,000 hours, i.e., at 70% level of activity, furnishes details of expenses as under :

Variable expenses Rs. 1,260 Semi-variable expenses Rs. 1,200 Fixed expenses Rs. 1,800

The semi-variable expenses go up by 10% between 85% and 95% activity and by 20% above 95% activity. Construct a flexible budget for 80, 90 and 100 per cent activities.

Solution

Head of Account Control basis 70% 80% 90% 100% Budgeted hours 7,000 8,000 9,000 10,000 Rs. Rs. Rs. Rs. Variable expenses V 1,260 1,440 1,620 1,800 Semi-variable expenses SV 1,200 1,200 1,320 1,440 Fixed expenses F 1,800 1,800 1,800 1,800

Total expenses 4,260 4,440 4,740 5,040 Recovery rate per hour 0.61 0.55 0.53 0.50 Conclusion :

We notice that the recovery rate at 70% activity is Re. 0.61 per hour. If in a particular month the factory works 8,000 hours, it will be incorrect to estimate the allowance as Rs.

* 10.28

Page 528: 30510870 Cost Accounting and Financial Management

Budgets and Budgetary Control

4,880 @ 0.61. The correct allowance will be Rs. 4,440 as shown in the table. If the actual expenses are Rs. 4,500 for this level of activity, the company has not saved any money but has over-spent by Rs. 60 (Rs. 4,500 – Rs. 4,440).

Illustration

A department of Company X attains sale of Rs. 6,00,000 at 80 per cent of its normal capacity and its expenses are given below :

Administration costs : Rs. Office salaries 90,000 General expenses 2 per cent of sales Depreciation 7,500 Rates and taxes 8,750 Selling costs : Salaries 8 per cent of sales Travelling expenses 2 per cent of sales Sales office expenses 1 per cent of sales General expenses 1 per cent of sales Distribution costs : Wages 15,000 Rent 1 per cent of sales Other expenses 4 per cent of sales Draw up flexible administration, selling and distribution costs budget, operating at 90 per cent, 100 per cent and 110 per cent of normal capacity.

Solution

Flexible Budget of Department....of Company ‘X’

Expenses Basis Level of activity 80% 90% 100% 110% Rs. Rs. Rs. Rs.

(1) (2) (3) (4) (5) (6) Sales 6,00,000 6,75,000 7,50,000 8,25,000

10.29

Page 529: 30510870 Cost Accounting and Financial Management

Cost Accounting

Administration costs : Office salaries Fixed 90,000 90,000 90,000 90,000 General expenses 2% of sales 12,000 13,500 15,000 16,500 Depreciation Fixed 7,500 7,500 7,500 7,500 Rates & taxes Fixed 8,750 8,750 8,750 8,750

Total administration costs 1,18,250 1,19,750 1,21,250 1,22,750 Selling costs : Salaries 8% of sales 48,000 54,000 60,000 66,000 Travelling expenses 2% of sales 12,000 13,500 15,000 16,500 Sales office expenses 1% of sales 6,000 6,750 7,500 8,250 General expenses 1% of sales 6,000 6,750 7,500 8,250

Total selling costs : 72,000 81,000 90,000 99,000 Distribution costs : Wages Fixed 15,000 15,000 15,000 15,000 Rent 1% of sales 6,000 6,750 7,500 8,250 Other expenses 4% of sales 24,000 27,000 30,000 33,000

Total Distribution Cost 45,000 48,750 52,500 56,250

Total Admn., Selling & Dist. costs 2,35,250 2,49,500 2,63,750 2,78,000

Note : In the absence of information it has been assumed that office salaries, depreciation, rates and taxes and wages remain the same at 110% level of activity also. However, in practice some of these costs may change if present capacity is exceeded.

Illustration

Action Plan Manufacturers normally produce 8,000 units of their product in a month, in their Machine Shop. For the month of January, they had planned for a production of 10,000 units. Owing to a sudden cancellation of a contract in the middle of January, they could only produce 6,000 units in January.

Indirect manufacturing costs are carefully planned and monitored in the Machine Shop and the Foreman of the shop is paid a 10% of the savings as bonus when in any month

* 10.30

Page 530: 30510870 Cost Accounting and Financial Management

Budgets and Budgetary Control

the indirect manufacturing cost incurred is less than the budgeted provision.

The Foreman has put in a claim that he should be paid a bonus of Rs. 88.50 for the month of January. The Works Manager wonders how any one can claim a bonus when the Company has lost a sizeable contract. The relevant figures are as under :

Indirect manufacturing Expenses for a Planned for Actuals in costs normal month January January

Rs. Rs. Rs. Salary of foreman 1,000 1,000 1,000 Indirect labour 720 900 600 Indirect material 800 1,000 700 Repairs and maintenance 600 650 600 Power 800 875 740 Tools consumed 320 400 300 Rates and taxes 150 150 150 Depreciation 800 800 800 Insurance 100 100 100 5,290 5,875 4,990 Do you agree with the Works Manager ? Is the Foreman entitled to any bonus for the performance in January ? Substantiate your answer with facts and figures.

Solution

Flexible Budget of “Action Plan Manufacturers” (for the month of January)

Indirect manufacturing Nature of Expenses for Planned Expenses as Actual Difference cost cost a normal expenses for per flexibleexpenses for Increased month January budget forthe month of (decreased) January January Rs. Rs. Rs. Rs. (1) (2) (3) (4) (5) (6) = (5) – (4) Salary of foreman Fixed 1,000 1,000 1,000 1,000 Nil Indirect labour Variable 720 900 540 600 60 (Refer to Working note 1)

10.31

Page 531: 30510870 Cost Accounting and Financial Management

Cost Accounting

Indirect material Variable 800 1,000 600 700 100 (Refer to Working note 2) Repair and maintenance Semi-variable 600 650 550 600 50 (Refer to Working note 3) Power Semi-variable 800 875 725 740 15 (Refer to Working note 4) Tools consumed Variable 320 400 240 300 60 (Refer to Working note 5) Rates and taxes Fixed 150 150 150 150 Nil Depreciation Fixed 800 800 800 800 Nil Insurance Fixed 100 100 100 100 Nil 5,290 5,875 4,705 4,990 285

Conclusion : The above statement of flexible budget clearly shows that the concern’s expenses in the month of January have increased from Rs. 4,705 to Rs. 4,990. Under such circumstances the Foreman of the company is not at all entitled for any performance bonus in January.

Working notes :

1. Indirect labour cost per unit 8,000

720 Rs =0.09P.

Indirect labour for 6,000 units = 6,000 × 0.09P = Rs. 540.

2. Indirect material cost per unit 8,000

800 Rs = 0.10P

Indirect material for 6,000 units = 6,000 × 0.10P = Rs. 600

3. According to high and low point method of segregating semi-variable cost into fixed and variable components, following formulae may be used.

Variable cost of repair and maintenance per unit = leveloutput in Changelevel expense in Change

* 10.32

Page 532: 30510870 Cost Accounting and Financial Management

Budgets and Budgetary Control

= 2,000

600 Rs - 650 Rs

= 0.025 P.

For 8,000 units

Total Variable cost of repair and maintenance = Rs. 200

Fixed repair & maintenance cost = Rs. 400

Hence at 6,000 units output level, total cost of repair and maintenance should be = Rs. 400 + Rs. 0.025 × 6,000 units = Rs. 400 + Rs. 150 = Rs. 550

4. Variable cost of power per unit = units 2,000

800 Rs - 875 Rs = 0.0375

For 8,000 units

Total variable cost of power = Rs. 300

Fixed cost = Rs. 500

Hence, at 6,000 units output level, total cost of power should be

= Rs. 500 + Rs. 0.0375 × 6,000 units

= Rs. 500 + Rs. 225 + Rs. 725

5. Tools consumed cost for 8,000 units = Rs. 320

Hence, tools consumed cost for 6,000 units = (Rs. 320/8,000 units) × 6,000 units

= Rs. 240

10.33

Page 533: 30510870 Cost Accounting and Financial Management

Cost Accounting

Illustration

A single product company estimated its sales for the next year quarterwise as under :

Quarter Sales (Units) I 30,000 II 37,500 III 41,250 IV 45,000 The opening stock of finished goods is 10,000 units and the company expects to maintain the closing stock of finished goods at 16,250 units at the end of the year. The production pattern in each quarter is based on 80% of the sales of the current quarter and 20% of the sales of the next quarter.

The opening stock of raw materials in the beginning of the year is 10,000 kg. and the closing stock at the end of the year is required to be maintained at 5,000 kg. Each unit of finished output requires 2 kg. of raw materials.

The company proposes to purchase the entire annual requirement of raw materials in the first three quarters in the proportion and at the prices given below :

Quarter Purchase of raw materials % to total Price per kg. annual requirement in quantity Rs.

I 30% 2 II 50% 3 III 20% 4

The value of the opening stock of raw materials in the beginning of the year is Rs. 20,000. You are required to present the following for the next year, quarter wise :

(i) Production budget (in units).

(ii) Raw material consumption budget (in quantity).

(iii) Raw material purchase budget (in quantity and value).

(iv) Priced stores ledger card of the raw material using First in First out method.

* 10.34

Page 534: 30510870 Cost Accounting and Financial Management

Budgets and Budgetary Control

Solution

Working Note : Total Annual Production (in units)

Sales in 4 quarters 1,53,750 units Add : Closing balance 16,250 units 1,70,000 units Less : Opening balance 10,000 units Total number of units to be produced in the next year 1,60,000 (i) Production Budget (in units)

Quarters I II III IV Total Units Units Units Units Units Sales 30,000 37,500 41,250 45,000 1,53,750 Production in current quarter 24,000 30,000 33,000 36,000 (80% of the sale of current quarter) Production for next quarter 7,500 8,250 9,000 12,250* (20% of the sale of next quarter) Total production 31,500 38,250 42,000 48,250 1,60,000

* Difference figure.

(ii) Raw material consumption budget in quantity

Quarters Total I II III IV Units to be produced in each quarter: (A) 31,500 38,250 42,000 48,250 1,60,000 Raw material con- sumption p.u. (kg.): (B) 2 2 2 2 Total raw material consumption (Kg.) : (A × B) 63,000 76,500 84,000 96,500 3,20,000

10.35

Page 535: 30510870 Cost Accounting and Financial Management

Cost Accounting

(iii) Raw material purchase budget (in quantity)

Raw material required for production (kg.) 3,20,000 Add : Closing balance of raw material (kg.) 5,000 3,25,000 Less : Opening balance (kg.) 10,000 Material to be purchased (kg.) 3,15,000

Raw material purchase budget (in value)

Quarters % of annual require- Quantity of Rate per Amount ment (Qty.) for pur- raw material kg. chasing raw material to be purchased (kg.) (Rs.) (Rs.) (1) (2) (3) (4) (5) = (3) × (4) I 30 94,500 2 1,89,000 (3,15,000 kg. × 30%) II 50 1,57,500 3 4,72,500 (3,15,000 kg. × 50%) III 20 63,000 4 2,52,000 (3,15,000 kg. × 20%) Total : 3,15,000 9,13,500 (iv) Priced Stores Ledger Card

(of the raw material using FIFO method) Quarters

I II III IV Kg. Rate Value Kg. Rate Value Kg. Rate Value Kg. Rate Value (Rs.) (Rs.) (Rs.) (Rs.) (Rs.) (Rs.) (Rs.) (Rs.) Opening balance :10,000 2 20,000 41,500 2 83,000 1,22,500 3 3,67,500 38,500 3 1,15,500 (A) 63,000 4 2,52,000 Purchases : (B) 94,500 2 1,89,000 1,57,500 3 4,72,500 63,000 4 2,52,000 – – – Consumption : (C) 63,000 2 1,26,000 41,500 2 83,000 84,000 3 2,52,000 38,500 3 1,15,500 35,000 3 1,05,000 58,000 4 2,32,000 Balance : (D) 41,500 2 83,000 1,22,500 3 3,67,500 38,500 3 1,15,500 5,000 4 20,000 (D) = (A) + (B)–(C) 63,000 4 2,52,000

* 10.36

Page 536: 30510870 Cost Accounting and Financial Management

Budgets and Budgetary Control

Illustration

A company is engaged in the manufacture of specialised sub-assemblies required for certain electronic equipments. The company envisages that in the forthcoming month, December, 2006, the sales will take a pattern in the ratio of 3 : 4 : 2 respectively of sub-assemblies, ACB, MCB and DP.

The following is the schedule of components required for manufacture :

Component requirements Sub-assembly Selling price Base board IC08 IC12 IC26 ACB 520 1 8 4 2 MCB 500 1 2 10 6 DP 350 1 2 4 8 Purchase price Rs. 60 20 12 8 The direct labour time and variable overheads required for each of the sub-assemblies are :

Labour hours per sub-assembly Grade A Grade B Variable overheads per sub-assembly Rs. ACB 8 16 36 MCB 6 12 24 DP 4 8 24 Direct wage rate per hour Rs. 5 4 — The labourers work 8 hours a day for 25 days a month.

The opening stocks of sub-assemblies and components for December, 2006 are as under:

Sub-assemblies Components ACB 800 Base Board 1,600 MCB 1,200 IC08 1,200 DP 2,800 IC12 6,000 IC26 4,000

10.37

Page 537: 30510870 Cost Accounting and Financial Management

Cost Accounting

Fixed overheads amount to Rs. 7,57,200 for the month and a monthly profit target of Rs. 12 lacs has been set.

The company is eager for a reduction of closing inventories for December, 2006 of sub-assemblies and components by 10% of quantity as compared to the opening stock. Prepare the following budgets for December 2006 :

(i) Sales budget in quantity and value.

(ii) Production budget in quantity

(iii) Component usage budget in quantity.

(iv) Component purchase budget in quantity and value.

(v) Manpower budget showing the number of workers and the amount of wages payable.

Solution

Working Note :

1. Statement showing contribution :

Sub assemblies ABC MCB DP Total Rs. Rs. Rs. Rs. Selling price per unit (p.u.) : (A) 520 500 350 Marginal Cost p.u. Components Base board 60 60 60 IC08 160 40 40 IC12 48 120 48 IC26 16 48 64 Labour Grade A 40 30 20 Grade B 64 48 32 Variable production overhead 36 24 24 Total marginal cost p.u. : (B) 424 370 288 Contribution p.u. : (C) = (A) – (B) 96 130 62 Sales ratio : (D) 3 4 2 Contribution × Sales ratio : [(E) = (C) × (D)] 288 520 124 932

* 10.38

Page 538: 30510870 Cost Accounting and Financial Management

Budgets and Budgetary Control

2. Desired Contribution for the forthcoming month December, 2006 Rs. Fixed overheads 7,57,200 Desired profit 12,00,000 Desired contribution 19,57,200 3. Sales mix required i.e. number of batches for the forthcoming month December, 2006

Sales mix required =Desired contribution/contribution × Sales ratio =Rs. 19,57,200/932 (Refer to Working notes 1 and 2) = 2,100

Budgets for December, 2006

(i) Sales budget in quantity and value

Sub-assemblies ACB MCB DP Total Sales (quantity) (2,100 × 3:4:2) 6,300 8,400 4,200 (Refer to working note 3) Selling price p.u. (Rs.) 520 500 350 Sales value (Rs.) 32,76,000 42,00,000 14,70,000 89,46,000

(ii) Production budget in quantity

Sub-assemblies ACB MCB DP Sales 6,300 8,400 4,200 Add : Closing stock 720 1,080 2,520 (Opening stock less 10%) ____ ____ ____ Total quantity required 7,020 9,480 6,720 Less : Opening stock 800 1,200 2,800 Production 6,220 8,280 3,920

10.39

Page 539: 30510870 Cost Accounting and Financial Management

Cost Accounting

(iii) Component usage budget in quantity

Sub-assemblies ACB MCB DP Total Production 6,220 8,280 3,920 — Base board (1 each) 6,220 8,280 3,920 18,420 Component IC08 (8 : 2 : 2) 49,760 16,560 7,840 74,160 (6,220 × 8) (8,280 × 2) (3,920 × 2) Component IC12 (4 : 10 : 4) 24,880 82,800 15,680 1,23,360 (6,220 × 4) (8,280 × 10) (3,920 × 4) Component IC26 (2 : 6 : 8) 12,440 49,680 31,360 93,480 (6,220× 2) (8,280 × 6) (3,920 × 8) (iv) Component Purchase budget in quantity and value

Sub-assemblies Base board IC08 IC12 IC26 Total Usage in production 18,420 74,160 1,23,360 93,480 Add : Closing stock 1,440 1,080 5,400 3,600 (Opening stock less 10%) 19,860 75,240 1,28,760 97,080 Less : Opening stock 1,600 1,200 16,000 4,000 Purchase (Quantity) 18,260 74,040 1,22,760 93,080 Purchase price (Rs.) 60 20 12 8 Purchase value (Rs.) 10,95,600 14,80,8 00 14,73,120 7,44,640 47,94,160

(v) Manpower budget showing the number of workers and the amount of wages payable

Direct labour Grade A Grade B Sub- Budgeted Hours per Total Hours per Total Total Assemblies Production Unit Hours Unit Hours ACB 6,220 8 49,760 16 99,520 MCB 8,280 6 49,680 12 99,360 DP 3,920 4 15,680 8 31,360

* 10.40

Page 540: 30510870 Cost Accounting and Financial Management

Budgets and Budgetary Control

(A) Total hours 1,15,120 2,30,240 (B) Hours per man per month 200 200 (C) Number of workers per month : (A/B) 576 1,152 (D) Wage rate per month (Rs.) 1,000 800 (E) Wages payable (Rs.) : (C × D) 5,76,000 9,21,600 14,97,600

14.12 Self-Examination Questions

Multiple choice questions

(i) A budget that gives a summary of all the functional budgets and projected Profit and Loss Account is known as

(a) Capital budget

(b) Flexible budget

(c) Master budget

(d) Discretionary budget

(ii) The fixed-variable cost classification has a special significance in the preparation of

(a) Flexible budget

(b) Master budget

(c) Cash budget

(d) Capital budget

(iii) The basic difference between a fixed budget and a flexible budget is that a fixed budget

(a) includes only fixed costs, and a flexible budget only variable costs

(b) is a budget for a single level of some measures of activity, while a flexible budget consists of several budgets based on different activity levels

(c) is concerned with future acquisition of fixed assets, while a flexible budget is concerned with expenses that vary with sales

(d) cannot be changed after a fiscal period begins, while a flexible budget can be changed after a fiscal period begins

10.41

Page 541: 30510870 Cost Accounting and Financial Management

Cost Accounting

(iv) When preparing a production budget, the quantity to be produced equals:

(a) Sales quantity + opening stock + closing stock

(b) Sales quantity – opening stock + closing stock

(c) Sales quantity – opening stock – closing stock

(d) Sales quantity + opening stock – closing stock

(v) A job requires 2,400 actual labour hours for completion and it is anticipated that there will be 20 percent idle time. If the wage rate is Rs 10 per hour , what is the budgeted labour cost for the job?

(a) Rs 19200

(b) Rs 24,000

(c) Rs 28,800

(d) Rs 30,000

(vi) Of the four costs shown below , which would not be included in the cash budget of an insurance firm?

(a) Depreciation of fixed asset

(b) Commission paid to agents

(c) Office salaries

(d) Capital cost of a new computer

(vii) A master budget comprises

(a) The budgeted profit and loss account

(b) Budgeted cash flow, budgeted profit and loss, budgeted balance sheet

(c) Budgeted cash flow

(d) Entire sets of budgets prepared

(viii) Which of the information below should be contained in a budget manual?

(a) An organisation chart

(b) Timetable for budget preparation

* 10.42

Page 542: 30510870 Cost Accounting and Financial Management

Budgets and Budgetary Control

(c) A list of account codes

(d) all a, b and c

(ix) ABC Ltd cash budget forewarns of a short term surplus. Which of the following would be appropriate action to be taken in such a situation?

(a) Increase debtors and stock to boost sales

(b) Purchase new fixed assets

(c) Repay long term loans

(d) Pay creditors early to obtain a cash discount

(x) PG Ltd. makes a single product and is preparing its material usage budget for next year. Each unit of product requires 2 kg of material, and 5,000 units of product are to be produced next year.

Opening stock of material is budgeted to be 800 kg and PG Ltd. budged to increase material stock at the end of next year by 20%.

The material usage budget for next year is

(a) 8,000 kg

(b) 9,840 kg

(c) 10,000 kg

(d) 10,160 kg.

Answers to multiple choice questions

1. c; 2. a; 3.b; 4.b; 5.d; 6.a; 7.b; 8.d; 9.a&d; 10.c

Short answer type questions

1. Discuss briefly the objectives of budgeting.

2. Define the terms “Budget” and “Budgeting control”.

3. Distinguish between fixed budget and flexible budget.

4. Write short notes on the following,

(a) Capital expenditure budget

10.43

Page 543: 30510870 Cost Accounting and Financial Management

Cost Accounting

(b) Cash budget

5. Distinguish between Functional budgets and master budget.

Long answer type questions

1. Discuss the objectives of introducing a Budgetary Control System in an organisation.

2. Discuss briefly the advantages and limitations of “Budgetary Control System”. 3. (a) List any six functional budgets prepared by a business.

(b) What do you understand by the term sales budget? How it is prepared.

4. “Establishing specific targets for future operations is part of the planning function of management, while executing actions to meet the goals is the directing function of management”. In the context of this statement, discuss the planning directing and controlling functions of management.

5. Discuss the salient features of a budgetary control system.

Numerical questions

1. A company has compiled the following data for preparation of budget for 1998 :

Products A B C Sales per month Units 8,000 4,000 6,000 Rs./Unit Rs./Unit Rs./Unit Selling price 40 80 100 Direct materials 20 48 40 Direct labour : Dept. Rate/Hour Rs. 1. 5.00 5 10 20 2 4.00 8 4 12 Variable overheads 3 3 7 Fixed overheads Rs. 1,50,000 per month

* 10.44

Page 544: 30510870 Cost Accounting and Financial Management

Budgets and Budgetary Control

After the Budget was discussed the following action plan was approved for improving the profitability of the company :

(i) Direct labour in department 1 which is in short supply should be increased by 15,000 hours by spending fixed overheads of Rs. 8,000 per month.

(ii) To boost sales, an advertisement programme should be launched at a cost of Rs. 10,000 per month.

(iii) The selling prices should be reduced by :

A : 2 ½ % B : 8 ¾ % C : 1%

(iv) The sales targets have been increased and the sales department has confirmed that the company will be able to achieve the following quantities of sales :

A : 12,000 Units B : 6,000 Units C : 10,000 Units

Required : (i) Present the original monthly budget for 1998.

(ii) Set an optimal product mix after taking the action plan into consideration and determine its monthly profit.

(iii) In case the requirement of direct labour hours of department 2 in excess of 40,000 hours is to be met by overtime working involving double the normal rate, what will be the effect of so working overtime on the optimum profit calculated by you in (ii) above.

2. P. Ltd. manufactures two products using one type of material and one grade of labour. Shown below an extract from the company’s working papers for the next period’s budget :

Product A Product B Budgeted sales (units) 3,600 4,800 Budgeted material consumption, 5 5 per product (kg.) Budgeted material cost Rs. 12 per kg. Standard hours allowed per product 5 4 Budgeted wage rate Rs. 8 per hour

10.45

Page 545: 30510870 Cost Accounting and Financial Management

Cost Accounting

Overtime premium is 50% and is payable, if a worker works for more than 40 hours a week. There are 90 direct workers.

The target productivity ratio (or efficiency ratio) for the productive hours worked by the direct workers in actually manufacturing the products is 80%; in addition the non-productive down time is budgeted at 20% of the productive hours worked.

There are twelve 5 day weeks in the budget period and it is anticipated that sales and production will occur evenly throughout the whole period.

It is anticipated that stock at the beginning of the period will be :

A 1,020 units: Product B 2,400 units; Raw material 4,300 kgs.

The target closing stock, expressed in terms of anticipated activity during the budget period are 7: Product A 15 days sales; Product B 20 days sales; Raw material 10 days consumption.

Required :

Calculate the material purchases budget and the wages budget for the direct workers, showing the quantities and values, for the next period.

* 10.46

Page 546: 30510870 Cost Accounting and Financial Management

Budgets and Budgetary Control

10.47

Page 547: 30510870 Cost Accounting and Financial Management

PART II

FINANCIAL MANAGEMENT

Page 548: 30510870 Cost Accounting and Financial Management

CHAPTER 1

SCOPE AND OBJECTIVES OF FINANCIAL MANAGEMENT

Learning Objectives After studying this chapter, you will be able to understand ♦ What is financial management and how it evolved? ♦ The objectives of financial management, ♦ Role of a chief financial officer in an organization, and ♦ The relationship of financial management with accounting and other related fields.

1. INTRODUCTION Imagine a scenario where you and your friends decide to set up and manage a small company by the name of Calcutronics Ventures to manufacture and manage your new brand of calculators. Being not only the managers of your company, you are also the owners of the company i.e. the major shareholders. Before you start with business you will have to make certain financial decisions. You will have to decide which assets to buy like premises and machinery. These assets will cost money and the total cost of acquiring them would be your initial investment in the business. Now, a very vital question which arises to your mind is how this investment is to be financed i.e. where do you get the money from to invest in your business? Other questions which need to be answered would be do you have to put your own money only or there are other means of raising money? What is the best way to finance the investment? Who will provide the finance? And how much will it cost to raise the finance? Besides needing the capital to acquire fixed assets like premises and machinery, the business will need capital to run it on day to day basis as well. This capital is known as the working capital, which is needed to purchase the raw materials, pay suppliers, wages, expenses, etc. this leads to another concern regarding how best to finance its day to day operations? The objective will be to ensure that there is always enough cash available to meet company’s operating expenses and that business activities do not suffer due to shortage of cash. Here the focus is on making investment and financing decisions that affect the company in the short term. You will not make any of these decisions without any direction; you would have a goal in mind

Page 549: 30510870 Cost Accounting and Financial Management

Financial Management i.e. to make a return on the investment. As shareholders of the company you would want to be better off financially by undertaking the investment than not. If the business proves successful, it will increase the wealth of the shareholders i.e. yours and your friends and enhance the value of the business. No matter what type of business you choose, you will have to address the questions raised in the above described scenario to understand what financial management is. Thus, financial management is concerned with efficient acquisition and allocation of funds. In operational terms, it is concerned with management of flow of funds and involves decisions relating to procurement of funds, investment of funds in long term and short term assets and distribution of earnings to owners. In other words, focus of financial management is to address three major financial decision areas namely, investment, financing and dividend decisions.

2. MEANING OF FINANCIAL MANAGEMENT Financial management is that managerial activity which is concerned with the planning and controlling of the firm’s financial resources. It is an integrated decision making process concerned with acquiring, financing and managing assets to accomplish the overall goal of a business organisation. It can also be stated as the process of planning decisions in order to maximise the shareholder’s wealth. Financial managers have a major role in cash management, acquisition of funds and in all aspects of raising and allocating capital. As far as business organisations are concerned, the objective of financial management is to maximise the value of business. “Financial management comprises the forecasting, planning, organising, directing, co-ordinating and controlling of all activities relating to acquisition and application of the financial resources of an undertaking in keeping with its financial objective.” This definition of financial management by Raymond Chambers aptly sums up the vital role played by it in any organisation. Another very elaborate definition given by Phillippatus is “Financial Management is concerned with the managerial decisions that result in the acquisition and financing of short term and long term credits for the firm.” As such it deals with the situations that require selection of specific assets (or combination of assets), the selection of specific problem of size and growth of an enterprise. The analysis of these decisions is based on the expected inflows and outflows of funds and their effect on managerial objectives. One more definition of financial management is that “Financial management deals with procurement of funds and their effective utilisation in the business.” Financial management can also be stated as “The management of all the processes associated with the efficient acquisition and deployment of both short and long term financial resources.”

1.2

Page 550: 30510870 Cost Accounting and Financial Management

Scope and Objectives of Financial Management

There are, thus, two basic aspects of financial management viz., procurement of funds and an effective use of these funds to achieve business objectives.

2.1 PROCUREMENT OF FUNDS Since funds can be obtained from different sources therefore their procurement is always considered as a complex problem by business concerns. Funds procured from different sources have different characteristics in terms of risk, cost and control. The funds raised by the issue of equity shares are the best from the risk point of view for the firm, since there is no question of repayment of equity capital except when the firm is under liquidation. From the cost point of view, however, equity capital is usually the most expensive source of funds. This is because the dividend expectations of shareholders are normally higher than prevalent interest rate and also because dividends are an appropriation of profit, not allowed as an expense under the Income Tax Act. Also the issue of new shares to public may dilute the control of the existing shareholders. Debentures as a source of funds are comparatively cheaper than the shares because of their tax advantage. The interest the company pays on a debenture is free of tax, unlike a dividend payment which is made from the taxed profits. However, even when times are hard, interest on debenture loans must be paid whereas dividends need not be. However, debentures entail a high degree of risk since they have to be repaid as per the terms of agreement; also, the interest payment has to be made whether or not the company makes profits. There are thus risk, cost and control considerations which a finance manager must consider while procuring funds. The cost of funds should be at the minimum level for that a proper balancing of risk and control factors must be carried out. Funds can also be procured from banks and financial institutions; they generally provide funds subject to certain restrictive covenants. These covenants restrict the freedom of the borrower to raise loans from other sources. The reform process is also moving in the direction of a closer monitoring of ‘end use’ of resources mobilised through capital markets. Such restrictions are essential for the safety of funds provided by institutions and investors. Besides above there are several other financial instruments used today for raising long term, medium term and short term funds e.g., commercial paper, deep discount bonds etc. The finance manager has to balance the availability of funds and the restrictive provisions tied with such funds resulting in lack of flexibility. In the globalised competitive scenario it is not enough to depend on the available ways of raising finance but resource mobilisation has to be undertaken through innovative ways or financial products which may meet the needs of investors. Multiple option convertible bonds can be sighted as an example. Funds can be raised indigenously as well as from abroad. Foreign Direct Investment (FDI) and Foreign Institutional Investors (FII) are two major routes

1.3

Page 551: 30510870 Cost Accounting and Financial Management

Financial Management for raising funds from foreign sources besides ADR’s (American depository receipts) and GDR’s (Global depository receipts). Obviously, the mechanism of procurement of funds has to be modified in the light of the requirements of foreign investors. Procurement of funds inter alia includes identification of sources of finance, determination of finance mix, raising of funds and division of profits between dividends and retention of profits i.e. internal fund generation.

2.2 EFFECTIVE UTILISATION OF FUNDS The finance manager is also responsible for effective utilisation of funds. He has to point out situations where the funds are being kept idle or where proper use of funds is not being made. All the funds are procured at a certain cost and after entailing a certain amount of risk. If these funds are not utilised in the manner so that they generate an income higher than the cost of procuring them, there is no point in running the business. This is also an important consideration in dividend decision. Hence, it is crucial to employ the funds properly and profitably. The funds are to be invested in the manner so that the company can produce at its optimum level without endangering its financial solvency. Thus, financial implications of each decision to invest in fixed assets are to be properly analysed. For this, the finance manager would be required to possess sound knowledge of techniques of capital budgeting. He must also keep in view the need of adequate working capital and ensure that while the firms enjoy an optimum level of working capital they do not keep too much funds blocked in inventories, book debts and cash, etc.

3. EVOLUTION OF FINANCIAL MANAGEMENT Financial management evolved gradually over the past 50 years. The evolution of financial management is divided into three phases. Financial Management evolved as a separate field of study at the beginning of the century. The three stages of its evolution are: The Traditional Phase: During this phase, financial management was considered necessary only during occasional events such as takeovers, mergers, expansion, liquidation, etc. Also, when taking financial decisions in the organisation, the needs of outsiders (investment bankers, people who lend money to the business and other such people) to the business was kept in mind. The Transitional Phase: During this phase, the day-to-day problems that financial managers faced were given importance. The general problems related to funds analysis, planning and control were given more attention in this phase. The Modern Phase: Modern phase is still going on. The scope of financial management has greatly increased now. It is important to carry out financial analysis for a company. This analysis helps in decision making. During this phase, many theories have been developed regarding efficient markets, capital budgeting, option pricing, valuation models and also in several other important fields in financial management.

1.4

Page 552: 30510870 Cost Accounting and Financial Management

Scope and Objectives of Financial Management

4. IMPORTANCE OF FINANCIAL MANAGEMENT Financial management is all about managing expenditure within a limited budget. It is about allocating money to the necessary items first. If after that, you have some money left, it must be used to pay off the debts. If there is still some money left you can use it as you like. Financial management means management of all matters related to an organisation’s finances. This principle is the same whether it is an organisation, a family, or even a country’s economy. We must balance expenditure and income. Let us understand this better by taking an example of a company, Cotton Textiles Limited. The company earns money by selling textiles. Let us assume that it earns Rs. 10 lakhs every month. This is known as revenue. A company has many expenses. Some of the major expenses of the company can be listed as wages to workers, raw materials for making the textile, electricity and water bills and purchase and repair of machines that are used to manufacture the textile. All these expenses are paid out of the revenues. If the revenues are more than the expenses, then the company will make profits. But, if the expenses are more than revenues, then it will face losses. If it continues like that, eventually, it will lose all its assets. In other words it will lose its property and all that it owns. In that case, even the workers may be asked to leave the company. To avoid this situation, the company has to manage the cash inflows (cash coming into the company) and outflows (various expenses that the company has to meet).

5. SCOPE OF FINANCIAL MANAGEMENT As an integral part of the overall management, financial management is mainly concerned with acquisition and use of funds by an organization. Based on financial management guru Ezra Solomon’s concept of financial management, following aspects are taken up in detail under the study of financial management: (a) Determination of size of the enterprise and determination of rate of growth. (b) Determining the composition of assets of the enterprise. (c) Determining the mix of enterprise’s financing i.e. consideration of level of debt to equity,

etc. (d) Analyse planning and control of financial affairs of the enterprise. The scope of financial management has undergone changes over the years. Until the middle of this century, its scope was limited to procurement of funds under major events in the life of the enterprise such as promotion, expansion, merger, etc. In the modern times, the financial management includes besides procurement of funds, the three different kinds of decisions as well namely, investment, financing and dividend. All the three types of decisions would be dealt in detail during the course of this chapter.

1.5

Page 553: 30510870 Cost Accounting and Financial Management

Financial Management The given figure depicts the overview of the scope and functions of financial management. It also gives the interrelation between the market value, financial decisions and risk return trade off. The financial manager, in a bid to maximize shareholders’ wealth, should strive to maximize returns in relation to the given risk; he should seek courses of actions that avoid unnecessary risks. To ensure maximum return, funds flowing in and out of the firm should be constantly monitored to assure that they are safeguarded and properly utilized.

An Overview of Financial Management

6. OBJECTIVES OF FINANCIAL MANAGEMENT Efficient financial management requires the existence of some objectives or goals because judgement as to whether or not a financial decision is efficient must be made in the light of some objective. Although various objectives are possible but we assume two objectives of financial management for elaborate discussion. These are:

6.1 PROFIT MAXIMISATION It has traditionally been argued that the objective of a company is to earn profit, hence the objective of financial management is also profit maximisation. This implies that the finance manager has to make his decisions in a manner so that the profits of the concern are maximised. Each alternative, therefore, is to be seen as to whether or not it gives maximum profit. However, profit maximisation cannot be the sole objective of a company. It is at best a limited

1.6

Page 554: 30510870 Cost Accounting and Financial Management

Scope and Objectives of Financial Management

objective. If profit is given undue importance, a number of problems can arise. Some of these have been discussed below: (i) The term profit is vague. It does not clarify what exactly it means. It conveys a different meaning to different people. For example, profit may be in short term or long term period; it may be total profit or rate of profit etc. (ii) Profit maximisation has to be attempted with a realisation of risks involved. There is a direct relationship between risk and profit. Many risky propositions yield high profit. Higher the risk, higher is the possibility of profits. If profit maximisation is the only goal, then risk factor is altogether ignored. This implies that finance manager will accept highly risky proposals also, if they give high profits. In practice, however, risk is very important consideration and has to be balanced with the profit objective. (iii) Profit maximisation as an objective does not take into account the time pattern of returns. Proposal A may give a higher amount of profits as compared to proposal B, yet if the returns begin to flow say 10 years later, proposal B may be preferred which may have lower overall profit but the returns flow is more early and quick. (iv) Profit maximisation as an objective is too narrow. It fails to take into account the social considerations as also the obligations to various interests of workers, consumers, society, as well as ethical trade practices. If these factors are ignored, a company cannot survive for long. Profit maximisation at the cost of social and moral obligations is a short sighted policy.

6.2 WEALTH / VALUE MAXIMISATION You must be aware that many companies sell their shares in the stock market. People buy the shares as an investment. It means that they expect these shares to give them some returns. It is the duty of the finance manager to see that the shareholders get good returns on the shares. Hence, the value of the share should increase in the share market. The share value is affected by many things. If a company is able to make good sales and build a good name for itself, in the industry, the company’s share value goes up. If the company makes a risky investment, people may lose confidence in the company and the share value will come down. So, this means that the finance manager has the power to influence decisions regarding finances of the company. The decisions should be such that the share value does not decrease. Thus, wealth or value maximisation is the most important goal of financial management. How do we measure the value/wealth of a firm? According to Van Horne, “Value of a firm is represented by the market price of the company's common stock. The market price of a firm's stock represents the focal judgement of all market participants as to what the value of the particular firm is. It takes into account present and prospective future earnings per share, the timing and risk of these earnings, the dividend policy of the firm and many other factors that

1.7

Page 555: 30510870 Cost Accounting and Financial Management

Financial Management bear upon the market price of the stock. The market price serves as a performance index or report card of the firm's progress. It indicates how well management is doing on behalf of stockholders.”

Why Wealth Maximisation Works? Of course, there are other goals too like: ♦ Achieving a higher growth rate ♦ Attaining a larger market share ♦ Gaining leadership in the market in terms of products and technology ♦ Promoting employee welfare ♦ Increasing customer satisfaction Many companies have several other goals for the welfare of the society, like improving community life, supporting education and research, solving societal problems, etc. But wealth maximisation means that the company is using its resources in a good manner. If the share value is to stay high, the company has to reduce its costs and use the resources properly. If the company follows the goal of wealth maximisation, it means that the company will promote only those policies that will lead to an efficient allocation of resources.

1.8

Page 556: 30510870 Cost Accounting and Financial Management

Scope and Objectives of Financial Management

To achieve wealth maximization, the finance manager has to take careful decision in respect of: 1. Investment decisions: These decisions determine how scarce resources in terms of funds available are committed to projects which can range from acquiring a piece of plant to the acquisition of another company. Funds procured from different sources have to be invested in various kinds of assets. Long term funds are used in a project for various fixed assets and also for current assets. The investment of funds in a project has to be made after careful assessment of the various projects through capital budgeting. A part of long term funds is also to be kept for financing the working capital requirements. Asset management policies are to be laid down regarding various items of current assets. The inventory policy would be determined by the production manager and the finance manager keeping in view the requirement of production and the future price estimates of raw materials and the availability of funds. 2. Financing decisions: These decisions relate to acquiring the optimum finance to meet financial objectives and seeing that fixed and working capital are effectively managed. The financial manager needs to possess a good knowledge of the sources of available funds and their respective costs, and needs to ensure that the company has a sound capital structure, i.e. a proper balance between equity capital and debt. Such managers also need to have a very clear understanding as to the difference between profit and cash flow, bearing in mind that profit is of little avail unless the organisation is adequately supported by cash to pay for assets and sustain the working capital cycle. Financing decisions also call for a good knowledge of evaluation of risk, e.g. excessive debt carried high risk for an organisation’s equity because of the priority rights of the lenders. A major area for risk-related decisions is in overseas trading, where an organisation is vulnerable to currency fluctuations, and the manager must be well aware of the various protective procedures such as hedging (it is a strategy designed to minimise, reduce or cancel out the risk in another investment) available to him. For example, someone who has a shop, takes care of the risk of the goods being destroyed by fire by hedging it via a fire insurance contract. 3. Dividend decisions: These decisions relate to the determination as to how much and how frequently cash can be paid out of the profits of an organisation as income for its owners/shareholders. The owner of any profit-making organization looks for reward for his investment in two ways, the growth of the capital invested and the cash paid out as income; for a sole trader this income would be termed as drawings and for a limited liability company the term is dividends. The dividend decisions thus has two elements – the amount to be paid out and the amount to be retained to support the growth of the organisation, the latter being also a financing decision; the level and regular growth of dividends represent a significant factor in determining a profit-making company’s market value, i.e. the value placed on its shares by the stock market.

1.9

Page 557: 30510870 Cost Accounting and Financial Management

Financial Management All three types of decisions are interrelated, the first two pertaining to any kind of organisation while the third relates only to profit-making organisations, thus it can be seen that financial management is of vital importance at every level of business activity, from a sole trader to the largest multinational corporation. It is instructive to think this point through by taking the case of the sole trader; thus he has to invest capital in a shop, fittings and equipment and in the purchase of stock and sustaining debtors (working capital), he has to have sources of capital to finance his investment such as his own capital and bank borrowings, and he has to make dividend decisions to determine how much can be reasonably withdrawn from the business to ensure that it will remain sufficiently liquid and, if desired, capable of growth.

7. CONFLICTS IN PROFIT VERSUS VALUE MAXIMISATION PRINCIPLE In any company, the management is the decision taking authority. Since the company is a complex organisation comprising of different interested parties, therefore management has a difficult role of reconciling objectives of these parties. In doing so, the management may not always act in the best interest of the shareholders and may pursue its own personal goals. But the management may not be able to exclusively pursue its personal goals due to the constant supervision of the various stakeholders of the company-employees, creditors, customers, government, etc. Since the management would like to survive over the long-run, therefore overall management objective should be directed towards this goal. Every entity associated with the company will evaluate the performance of the management from the fulfilment of its own objective. The survival of the management will be threatened if the objective of any of the entities remains unfulfilled. The wealth maximisation objective is generally in accord with the interests of the various groups such as owners, employees, creditors and society, and thus, it may be consistent with the management objective of survival. However, there may arise a situation where a conflict may arise between the shareholders’ and management’s goals. For example, management may create satisfactory wealth for shareholders than the maximum. Such satisfying behaviour of the management will frustrate the objective of shareholders wealth maximisation as a normative guide to management.

Goal Objective Advantages Disadvantages Profit maximisation

Large amount of profits

(i) Easy to calculate profits (ii) Easy to determine the link between financial decisions and profits.

(i) Emphasizes the short term (ii) Ignores risk or uncertainty (iii) Ignores the timing of returns (iv) Requires immediate resources.

1.10

Page 558: 30510870 Cost Accounting and Financial Management

Scope and Objectives of Financial Management

Shareholders Wealth Maximisation

Highest market value of shares.

(i) Emphasizes the long term (ii) Recognises risk or uncertainty (iii) Recognises the timing of returns (iv) Considers shareholders’ return.

(i) Offers no clear relationship between financial decisions and share price. (ii) Can lead to management anxiety and frustration.

Illustration 1: Profit maximization can be achieved in the short term at the expense of the long term goal, that is, wealth maximisation. For example, a costly investment may experience losses in the short term but yield substantial profits in the long term. Also, a firm that wants to show a short term profit may, for example, postpone major repairs or replacement, although such postponement is likely to hurt its long term profitability. Another example can be taken to understand why wealth maximisation is a preferred objective than profit maximisation. Illustration 2: Profit maximisation does not consider risk or uncertainty, whereas wealth maximisation considers both risk and uncertainty. Suppose there are two products, X and Y, and their projected earnings over the next 5 years are as shown below:

Year Product X Product Y Rs. Rs.

1. 10,000 11,000 2. 10,000 11,000 3. 10,000 11,000 4. 10,000 11,000 5. 10,000 11,000 50,000 55,000

A profit maximization approach would favour product Y over product X. However, if product Y is more risky than product X, then the decision is not as straightforward as the figures seem to indicate. It is important to realize that a trade-off exists between risk and return. Stockholders expect greater returns from investments of higher risk and vice-versa. To choose product Y, stockholders would demand a sufficiently large return to compensate for the comparatively greater level of risk.

1.11

Page 559: 30510870 Cost Accounting and Financial Management

Financial Management 8. ROLE OF CHIEF FINANCIAL OFFICER (CFO) Modern financial management has come a long way from the traditional corporate finance. As the economy is opening up and global resources are being tapped, the opportunities available to finance managers virtually have no limits. Due to the changes in the global environment the chief financial officer needs to have a broader and far-sighted outlook, and must realize that his actions would have far reaching consequences for the firm because they influence the size, profitability, growth, risk and survival of the firm, and as a consequence, affect the overall value of the firm. He must have the flexibility to adapt to the external factors such as economic uncertainty, global competition, technological change, volatility of interest and exchange rates, changes in laws and regulations and ethical concerns. Therefore, in today’s changing environment, the chief financial officer plays a pivotal leadership role in a company’s overall efforts to achieve its goals. He is one of the dynamic members of corporate managerial team. His role, day-by-day, is becoming more and more pervasive and significant in solving the complex managerial problems. The traditional role of the chief financial officer was confined just to raising of funds from a number of sources, but the recent development in the socio-economic and political scenario throughout the world has placed him in a central position in the business organisation. He is now responsible for shaping the fortunes of the enterprise, and is involved in the most vital decision of allocation of capital like mergers, acquisition etc. He like other members of corporate team cannot be averse to the fast developments around him. He has to take note of these changes in order to be relevant and dynamic according to the fast changing circumstances. Therefore a new era has ushered during the recent years in financial management, especially with the development of new financial system, emergence of financial services industry, recent innovations and development of financial tools, techniques, instruments, and products and emphasis on public sector undertakings to be self-supporting and their dependence on capital market for fund requirements, have all changed the role of the chief financial officer. His role, especially, assumes significance in the present day context of liberalization, deregulation and globalisation. To sum it up, the chief financial officer of an organisation plays an important role in the company’s goals, policies, and financial success. His responsibilities include: (a) Financial analysis and planning: Determining the proper amount of funds to employ in the

firm, i.e. designating the size of the firm and its rate of growth. (b) Investment decisions: The efficient allocation of funds to specific assets. (c) Financing and capital structure decisions: Raising funds on favourable terms as possible,

1.12

Page 560: 30510870 Cost Accounting and Financial Management

Scope and Objectives of Financial Management

1.13

i.e., determining the composition of liabilities.

(d) Management of financial resources (such as working capital).

(e) Risk management: Protecting assets.

The figure below shows how the finance function in a large organization may be organized. Typically, the chief financial officer, who may be designated as Vice President (Finance) or Director (Finance), supervises the work of the treasurer and the controller. In turn, these officers are assisted by several specialist managers working under them.

Organisation of Finance Function

9. RELATIONSHIP OF FINANCIAL MANAGEMENT WITH RELATED DISCIPLINES

As an integral part of the overall management, financial management is not a totally independent area. It draws heavily on related disciplines and areas of study namely economics, accounting, production, marketing and quantitative methods. Even though these disciplines are inter-related, there are key differences among them. Some of the relationships are being discussed below:

9.1 FINANCIAL MANAGEMENT AND ACCOUNTING

The relationship between financial management and accounting are closely related to the

Page 561: 30510870 Cost Accounting and Financial Management

Financial Management extent that accounting is an important input in financial decision making. In other words, accounting is a necessary input into the financial management function. Financial accounting generates information relating to operations of the organisation. The outcome of accounting is the financial statements such as balance sheet, income statement, and the statement of changes in financial position. The information contained in these statements and reports helps the financial managers in gauging the past performance and future directions of the organisation. Though financial management and accounting are closely related, still they differ in the treatment of funds and also with regards to decision making. Treatment of Funds: In accounting, the measurement of funds is based on the accrual principle i.e. revenue is recognised at the point of sale and not when collected and expenses are recognised when they are incurred rather than when actually paid. The accrual based accounting data do not reflect fully the financial conditions of the organisation. An organisation which has earned profit (sales less expenses) may said to be profitable in the accounting sense but it may not be able to meet its current obligations due to shortage of liquidity as a result of say, uncollectible receivables. Such an organisation will not survive regardless of its levels of profits. Whereas, the treatment of funds, in financial management is based on cash flows. The revenues are recognised only when cash is actually received (i.e. cash inflow) and expenses are recognised on actual payment (i.e. cash outflow). This is so because the finance manager is concerned with maintaining solvency of the organisation by providing the cash flows necessary to satisfy its obligations and acquiring and financing the assets needed to achieve the goals of the organisation. Thus, cash flow based returns help financial managers to avoid insolvency and achieve desired financial goals. Decision making: The purpose of accounting is to collect and present financial data on the past, present and future operations of the organisation. The financial manager uses these data for financial decision making. It is not that the financial managers cannot collect data or accountants cannot make decisions. But the chief focus of an accountant is to collect data and present the data while the financial manager’s primary responsibility relates to financial planning, controlling and decision making. Thus, in a way it can be stated that financial management begins where accounting ends.

9.2 FINANCIAL MANAGEMENT AND OTHER RELATED DISCIPLINES For its day to day decision making process, financial management also draws on other related disciplines such as marketing, production and quantitative methods apart from accounting. For instance, financial managers should consider the impact of new product development and promotion plans made in marketing area since their plans will require capital outlays and have an impact on the projected cash flows. Likewise, changes in the production process may require capital expenditures which the financial managers must evaluate and finance. Finally, the tools and techniques of analysis developed in the quantitative methods discipline are

1.14

Page 562: 30510870 Cost Accounting and Financial Management

Scope and Objectives of Financial Management

helpful in analyzing complex financial management problems.

Impact of Other Disciplines on Financial Management

The above figure depicts the relationship between financial management and supportive disciplines. The marketing, production and quantitative methods are, thus, only indirectly related to day to day decision making by financial managers and are supportive in nature while accounting is the primary discipline on which the financial manager draws considerably. Even economics can also be considered as one of the major disciplines which help the financial manager to gain knowledge of what goes on in the world outside the business.

Self Examination Questions A. Objective Type Questions 1. If income is more than expenditure, a company will be able to show profits.

(a) True (b) False.

2. Management of all matters related to an organisation’s finances is called: (a) Cash inflows and outflows (b) Allocation of resources

1.15

Page 563: 30510870 Cost Accounting and Financial Management

Financial Management

(c) Financial management (d) Finance.

3. Allocation of resources means paying all expenses on time to avoid interest expenditure. (a) True (b) False.

4. Which of the following is not an element of financial management? (a) Allocation of resources (b) Financial Planning (c) Financial Decision-making (d) Financial control.

5. Financial management is concerned with the actual cash flows of the organisation, while financial accounting is concerned with recording the flow of cash. (a) True (b) False.

6. The most important goal of financial management is: (a) Profit maximisation (b) Matching income and expenditure (c) Using business assets effectively (d) Wealth maximisation.

7. In the traditional phase, the importance of financial management was limited to major events such as mergers and takeovers. (a) True (b) False.

8. To achieve wealth maximization, the finance manager has to take careful decision in respect of: (a) Investment (b) Financing (c) Dividend (d) All the above.

1.16

Page 564: 30510870 Cost Accounting and Financial Management

Scope and Objectives of Financial Management

9. Early in the history of finance, an important issue was: (a) Liquidity (b) Technology (c) Capital structure (d) Financing options.

10. Which of the following are microeconomic variables that help define and explain the discipline of finance? (a) Risk and return (b) Capital structure (c) Inflation (d) All of the above.

Answers to Objective Type Questions 1. (a); 2. (c); 3. (b); 4. (d); 5. (a); 6. (d); 7. (a); 8. (d); 9. (a); 10. (d) B. Short Answer Type Questions 1. Differentiate between the following:

(a) Procurement of funds and Utilization of funds (b) Traditional phase and Modern phase (c) Profit maximization and Value maximization (d) Investment decisions and Dividend decisions (e) Financial management and Financial accounting.

2. Write short notes on the following: (a) Scope of financial management (b) Importance of financial management (c) Financing decisions. C. Long Answer Type Questions 1. What are the two main aspects of the finance function? 2. What are three main considerations in procuring funds?

1.17

Page 565: 30510870 Cost Accounting and Financial Management

Financial Management 3. Explain “Wealth maximisation” and “Profit maximisation” objectives of financial

management. 4. Discuss the role of a chief financial officer. 5. In recent years, there have been a number of environmental, pollution and other

regulations imposed on businesses. In view of these changes, is maximisation of shareholder wealth still a realistic objective?

1.18

Page 566: 30510870 Cost Accounting and Financial Management

CHAPTER 2

TIME VALUE OF MONEY

Learning Objectives After studying this chapter, you will be able to understand ♦ The concept of time value of money; ♦ Techniques of Discounting and Compounding; ♦ Identify the equation for calculating the present value of an annuity and calculation of the

present value of an annuity; and ♦ Identify the equation for calculating the future value of an annuity and calculation of the

future value of an annuity.

1. CONCEPT OF TIME VALUE OF MONEY Most financial transactions involve a series of cash flows - regular or irregular - over a period of time. When evaluating these cash flows the basic concept used is the time value of money. If you are offered the choice between having Rs. 100 today and having Rs. 100 at a future date, you will usually prefer to have Rs. 100 now. If the choice is between paying Rs. 100 now or paying the same Rs. 100 at a future date, you will usually prefer to pay Rs. 100 later. But why is this? Rs. 100 has the same value one year from now also. Actually, although the value is the same, you can do much more with the money if you have it now; over the time you can earn some interest on your money. The time value of money (TVM) is one of the basic concepts of finance. We know that if we deposit money in a bank account we will receive interest. Because of this, we prefer to receive money today rather than the same amount in the future. Money we receive today is more valuable to us than money received in the future by the amount of interest we can earn with the money. This is referred to as the time value of money. The term time value of money can be defined as “The value derived from the use of money over time as a result of investment and reinvestment. This term may refer to either present value or future value calculations. The present value is the value today of an amount that would exist in the future with a stated investment rate called the discount rate.” For example, with a 10% annual discount rate, the present value today of Rs. 110 one year from now is Rs. 100.

Page 567: 30510870 Cost Accounting and Financial Management

Financial Management Considering time value of money is important in decision making, for the purpose of financial decision making expected cash flows are evaluated from the time frame of present time, t0. In finance, we often have a decision making situation wherein cash investment today is evaluated with reference to expected cash flows in future. Say, a firm wants to invest Rs. 1,000 today at t0, its expected cash flows in future are as follows: t1 Rs. 5,000 t2 Rs. 5,000 t3 Rs. 8,000 Should we accept this investment proposal? This needs appreciation that cash flows are at different time frame. These are to be converted into unique time frame, say, with reference to t0. Then we shall have to consider present value of future cash flow: t0 -1,000 Convert into present value of money in terms of t0

t1 Rs. 5,000 t2 Rs. 5,000 t3 Rs. 8,000

We will discuss the technique of computation of time value of money later in this chapter. The reason why there is time value of money is as follows: Opportunity Cost: There are alternative productive uses of money. The cost of any decision includes the cost of the next best opportunity forgone. You can save and invest, get interest and spend. Inflation: It erodes the value of money. Risk: There are always financial and non-financial risks involved. The trade-off between money now and money later depends on, among other things, the rate of interest you can earn by investing. It impacts business finance, consumer finance and government finance. Time value of money results from the concept of interest. Interest rate is the cost of borrowing money as a yearly percentage. For investors, interest rate is the rate earned on an investment as a yearly percentage.

2. SIMPLE INTEREST It may be defined as “Interest calculated as a simple percentage of the original principal amount”. The simple interest ‘I’ on a principal ‘P’ borrowed at the rate of ‘i’ per annum for a

2.2

Page 568: 30510870 Cost Accounting and Financial Management

Time Value of Money

period of ‘t’ years is given by: I = Pit It must be noted that i is represented in decimals and is part of one unit. If the rate of interest is in percent, i can be calculated by dividing it by 100. If we add principal to the interest, we will get the total amount (A).

A= P+ I Illustration1: If you invest Rs 10,000 in a bank at simple interest of 9% per annum, what will be the amount at the end of three years? Solution

Amount, A = P + I = P + Pit = 10,000 + 10,000 ××100

7 3 = 12,100

Illustration 2: Rs. 2,000 is deposited in a bank for two years at simple interest of 6%. How much will be the balance at the end of 2 years? Solution Required balance is given by A = P (1 + it) = 2,000 (1 + 0.06 × 2) = 2,000 × 1.12 = Rs. 2,240. Illustration 3: Find the rate of interest if the amount owed after 6 months is Rs.1,050, borrowed amount being Rs. 1,000.

Solution We know A = P + Pit

21 i 1,000 1,000 1,050 i.e., ××+=

500i 50 or, =

10% 101

50050 i i.e., ===

3. COMPOUND INTEREST Compound interest is the interest that accrues on a deposit or investment that uses compounding which basically means that interest is paid both on previously earned interest and as well as on the principal. In other words, interest due at the end of unit payment period

2.3

Page 569: 30510870 Cost Accounting and Financial Management

Financial Management is added to the principal and interest on the next payment period is computed on the new principal. Naturally, the amount calculated on the basis of compound interest rate is higher than when calculated with the simple rate. The time interval between successive additions of interests is known as conversion (or payment) period. Typical conversion periods are given below:

Conversion Period Description 1 day Compounded daily 1 month Compounded monthly 3 months Compounded quarterly 6 months Compounded semiannually 12 months Compounded annually

at the end of first payment period, A1 = P + Pi = P(1 + i); at the end of second payment period, A2 = A1 + A1 i = A1(1 + i) = P (1 +i)2; at the end of second payment period, A2 = A1 + A1 i = A1(1 + i) = P (1 +i)2; at the end of third payment period, A3 = A2 + A2 i = A2(1 + i) = P (1 +i)3; An = An−1 + A n−1 i = A n−1 (1 + i) = P (1 +i)n;

Thus, the accrued amount An on a principal P after n payment periods at i (in decimal) rate of interest per payment period is given by:

.kr

year per periodspayment of Numberinterest of rate Annual i where

,i) (1 P A nn

==

+=

r. rateinterest annual anat year a times k done is gcompoundin when,kr 1 P

n

⎟⎠⎞

⎜⎝⎛ +=

Computation of An shall be quite simple with a calculator. However, compound interest tables as well as tables for (1+i)n at various rates per annum with (a) annual compounding; (b) monthly compounded and (c) daily compounding are available.

2.4

Page 570: 30510870 Cost Accounting and Financial Management

Time Value of Money

It should be remembered that i and n are with respect to per period, which can be different than a year. For example, annual interest can be payable, on monthly, quarterly or half-yearly basis. This will be clear from the illustrations given.

Graphic View of Compounding

Illustration 4: Determine the compound interest for an investment of Rs 7,500 at 6 % compounded half-yearly. Given that (1+i)n for i = 0.03 and n = 12 is 1.42576.

Solution

03.010026i =×

= , n = 6 × 2 = 12, P = 1,000

Compound Amount = 7,500(1+0.03)12 = 7,500 × 1.42576 = 10,693.20 Compound Interest = 10,693.20 – 7,500 = 3,193.20 Illustration 5: Rs. 2,000 is invested at annual rate of interest of 10%. What is the amount after 2 years if the compounding is done: (a) Annually? (b) Semi annually? (c) Monthly? (d) Daily?

Solution (a) The annual compounding is given by:

2.5

Page 571: 30510870 Cost Accounting and Financial Management

Financial Management

2,000 being P and 0.1 10010 being i 2, being n ,i) (1 P A n

2 =+=

= 2,000 (1.1)2 = 2,000 × 1.21 = Rs. 2,420

(b) For Semiannual compounding, n = 2 × 2 = 4, I = 0.1/2 = 0.05

A4 = 2,000 ( 1 + 0.05)4 = 2,000 × 1.2155 = Rs. 2,431

(c) For monthly compounding, n = 12 × 2 = 24, i = 0.1/12 = 0.00833

A24 = 2,000 (1.00833)24 = 2,000 × 1.22029 = Rs. 2440.58

(d) For daily compounding, n = 365 × 2 = 730, i = 0.1/(365) = 0.00027

A730 = 2,000 (1.00027)730 = 2,000 × 1.22135 = Rs. 2,442.70 Illustration 6: Determine the compound amount and compound interest on Rs. 1,000 at 6% compounded semiannually for 6 years. Given that (1+i)n = 1.42576 for i = 3% and n = 12.

Solution i = (6/2) = 3%, n = 6 × 2 = 12, P = 1,000 Compound amount = P (1 + i)n = 1,000 (1 + 3%)12

= 1,000 × 1.42576 = Rs. 1,425.76 Compound interest = 1,425.76 – 1,000 = Rs. 425.76 Illustration 7: What annual rate of interest compounded annually doubles an investment in 7 years? Given that 21/7 = 1.104090.

Solution If the principal be P, An = 2P Since, An = P(1 + i)n, 2P = P(1 + i)7, Or, 2 = (1 + i)7

Or, 21/7 = 1 + i Or, 1.104090 = 1 + I i.e., I = 0.10409 Required rate of interest = 10.41% Illustration 8: A person opened an account on April, 2005 with a deposit of Rs. 800. The account paid 6% interest compounded quarterly. On October 1, 2005, he closed the account

2.6

Page 572: 30510870 Cost Accounting and Financial Management

Time Value of Money

and added enough additional money to invest in a 6-month Time Deposit for Rs. 1,000 earning 6% compounded monthly. (a) How much additional amount did the person invest on October 1?

(b) What was the maturity value of his Time Deposit on April 1, 2006?

(c) How much total interest was earned?

Given that (1 +i)n is 1.03022500 for %,211 i= n = 2 and is 1.03037751 for i = 6. n and %

21

=

Solution (a) The initial investment earned interests for April – June and July – September quarter, i.e.

for 2 quarters.

In this case, %21 11 800 amount compounded the and 2 n %,

211

46 i

2

⎟⎠⎞

⎜⎝⎛ +====

= 800 × 1.03022500 = Rs. 824.18 The additional amount = Rs. (1,000 – 824.18) = Rs. 175.82

(b) In this case, the Time Deposit earned interest compounded monthly for 2 quarters.

1,000 P 6, n %,21

126 i Here, ====

Required maturity value 1,030.38 Rs. 1.03037751 1,000 %21 1 1,000

6

=×=⎟⎠⎞

⎜⎝⎛ +

(c) Total interest earned = (24.18 + 30.38) = Rs. 54.56

3.1 COMPOUND INTEREST VERSUS SIMPLE INTEREST

The given figure shows graphically the differentiation between compound interest and simple interest. The top two ascending lines show the growth of Rs. 100 invested at simple and compound interest. The longer the funds are invested, the greater the advantage with compound interest. The bottom line shows that Rs. 38.55 must be invested now to obtain Rs. 100 after 10 periods. Conversely, the present value of Rs. 100 to be received after 10 years is Rs. 38.55.

2.7

Page 573: 30510870 Cost Accounting and Financial Management

Financial Management

Compound Interest versus Simple Interest

4. EFFECTIVE RATE OF INTEREST It is the actual equivalent annual rate of interest at which an investment grows in value when interest is credited more often than once a year. If interest is paid m times in a year it can be found by calculating:

Ei = 1mi1

m

−⎟⎠⎞

⎜⎝⎛ +

Illustration 9: If the interest is 10% payable quarterly, find the effective rate of interest. Solution

E = 141.01

4

−⎟⎠⎞

⎜⎝⎛ + = 0.1038 or 10.38%

5. PRESENT VALUE The present value, P, is the amount of money that represents the sum of principal and interest if P is required to be invested now at a certain rate compounded over a number of time periods at a specified rate for each time period.

2.8

Page 574: 30510870 Cost Accounting and Financial Management

Time Value of Money

The present value, P, of the amount An due at the end of n interest period at the rate of i per interest period may be obtained by solving for P, the equation is: An = P(1 + i)n i.e. P = An (1 + i)−n

As mentioned earlier, computation of P may be simple if we make use of either the calculator or the Present Value table showing values of (1+i) −n for various time periods/per annum interest rates. For positive i, the factor (1 + i) −n is always less than 1, indicating thereby, future amount has smaller present value. Illustration 10: What is the present value of Re. 1 to be received after 2 years compounded annually at 10%?

Solution Here An = 1, i = 0.1 Required Present Value = An (1+i) −n

0.83 Re. 0.8264 1.21

1 (1.1)

1 )i1(

A 2nn ====

+=

Thus, Re. 0.83 shall grow to Re. 1 after 2 years at 10% compounded annually. Illustration 11: Find the present value of Rs. 10,000 to be required after 5 years if the interest rate be 9 per cent. Given that (1.09)5 = 1.5386

Solution Here, i = 0.09, n = 5, An = 10,000 Required Present value = An (1 + i) −n

= 10,000 (1.09) −5 = 10,000 × 0.65 = Rs. 6,500.

( )( ) ⎥

⎤⎢⎣

⎡==− 0.65

1.091 1.09 5

5

Illustration 12: What is the present value of Rs. 50,000 to be received after 10 years at 10 per cent compounded annually?

Solution Here n = 10, i = 0.1 P = An (1 + i) −n

= 50,000 (1.1) −10

2.9

Page 575: 30510870 Cost Accounting and Financial Management

Financial Management = 50,000 × 0.385543 = Rs. 19,277.15 Illustration 13: Mr. X has made real estate investment for Rs. 12,000 which he expects will have a maturity value equivalent to interest at 12% compounded monthly for 5 years. If most savings institutions currently pay 8% compounded quarterly on a 5 year term, what is the least amount for which Mr. X should sell his property? Given that (1 + i)n = 1.81669670 for i = 1% and n = 60 and that (1 + i) −n = 0.67297133 for i = 2% and n = 20.

Solution It is a two-part problem. First being determination of maturity value of the investment of Rs. 12,000 and then finding of present value of the obtained maturity value. Maturity value of the investment may be found from An = P (1+i)n,

where P = 12,000, 60. 12 5 n 1%, 1212 i =×===

Now, An = 12,000 (1+1%)60 = 12,000 × 1.81669670 = 21,800.36040000 = Rs. 21,800.36 Thus, maturity value of the investment in real estate = Rs. 21,800.36 The present value, P of the amount An due at the end of n interest periods at the rate of i% interest per period is given by P = An (1 + i) −n

We have in the present case, An = Rs. 21,800.36, 20. 4 5 n 2%, 48 i =×===

Thus, P = 21,800.36 (1+ 2%)−20

= 21,800.36 × 0.67297133 = Rs. 14,671.02 Mr. X should not sell the property for less than Rs. 14,671.02

6. ANNUITY An annuity is a stream of regular periodic payment made or received for a specified period of time. A recurring deposit with the bank is typical example of an annuity. The amount of an annuity, A is the algebraic sum of the payments and the accumulated interest. Thus, if Re. 1 be the periodic payment for an annuity at the interest rate of i per cent per payment period made over n payment periods, the first payment shall accumulate A1 compounded over n−1 time period, the second A2 over n−2 time period, and so on. ∴A1 = 1. (1 + i)n−1, A2 = 1. (1 + i)n−2,………, An−1 = 1. (1 + i)n−n+1 = 1. (1 + i)1, An = 1.(1 + i)0 = 1

2.10

Page 576: 30510870 Cost Accounting and Financial Management

Time Value of Money

The total amount of an annuity after n payment periods, denoted by A(n,i) is therefore given by:

A(n, i) = An + A n−1 +…………..+ A2 + A1

= 1 + (1 + i)1 +…………..+ (1+i) n−2 + (1+i) n−1 {a geometric series with first term 1 and common ratio (1+i)}

i

1 i) (1 1

i) (1 1 i) (1 1

}i) (1 1.{1 nnn −+

=−+−

=+−+−

=

If P be the periodic payments, the amount A of the annuity is given by: A = P. A (n, i)

Or, Amount = ( )i

1i1Pn −+

Table for A (n, i) at different rates of interest may be used conveniently, if available, to workout

problems. The value of expression ( )i

1i1 n −+ can easily be found through financial tables.

Illustration 14: Find the amount of an annuity if payment of Rs. 500 is made annually for 7 years at interest rate of 14% compounded annually. Solution

Here P = 500, n = 7, i = 0.14 A = Rs. 500 × A (7, 0.14) = 500 × 10.7304915 = Rs. 5,365.25 Illustration 15: Rs. 200 is invested at the end of each month in an account paying interest 6% per year compounded monthly. What is the amount of this annuity after 10th payment? Given that (1.005)10 = 1.0511 Solution

We have ( ) ( ) i,ii 1 i n,A

1n−+= being the interest rate (in decimal) per payment period over n

payment period. Here, i = .06/12 = .005, n = 10. Required amount is given by A = P.A (10, .005) = 200 × 10.22 = Rs. 2,044.

2.11

Page 577: 30510870 Cost Accounting and Financial Management

Financial Management

7. PERPETUITY Perpetuity is a stream of payments or a type of annuity that starts payments on a fixed date and such payments continue forever, or perpetually. Often preferred stock which pays a dividend is considered as a form of perpetuity. However, one must assume that the firm does not go bankrupt or is otherwise impeded for making timely payments. The formula for evaluating perpetuity is relatively straight forward. It is simply the expected income stream divided by a discount factor or market rate of interest. It reflects the expected present value of all payments. It is comparable to a perpetual bond. If a preferred issue pays a Rs. 2.00 quarterly dividend and the annual interest rate is 5 percent then one would expect to be willing to pay 2.50/.0125, or Rs. 200 per share. Here, the 5 percent interest rate was adjusted for a simple quarterly disbursement (.05/4 = .0125). Perpetuity is an annuity in which the periodic payments begin on a fixed date and continue indefinitely. Fixed coupon payments on permanently invested (irredeemable) sums of money are prime examples of perpetuities. Scholarships paid perpetually from an endowment fit the definition of perpetuity. The value of the perpetuity is finite because receipts that are anticipated far in the future have extremely low present value (today's value of the future cash flows). Additionally, because the principal is never repaid, there is no present value for the principal. The price of perpetuity is simply the coupon amount over the appropriate discount rate or yield. Perpetuity is an annuity that provides payments indefinitely. A constant stream of identical cash flows with no end. Since this type of annuity is unending, its sum or future value cannot be calculated. Examples of perpetuity can be local governments set aside funds so that it will be available on a regular basis for cultural activities or a children’s charity organisation set up a fund designed to provide a flow of regular payments indefinitely to needy children. Therefore, what happens in perpetuity is that once the initial fund has been established the payments will flow from the fund indefinitely which implies that these payments are nothing more than annual interest payments.

7.1 CALCULATION OF MULTI PERIOD PERPETUITY With perpetuities it is necessary to find a present value based on a series of payments that go on forever. The formula for determining the present value of multi-period perpetuity is as follows:

2.12

Page 578: 30510870 Cost Accounting and Financial Management

Time Value of Money

( ) ( ) ( ) rC

)r1(C

)r1(C.......

r1C

r1C

r1CPV

1nn321 =

+=

+++

++

++

+= ∑

=∞

Where: C = the interest payment each period r = the interest rate per payment period

Illustration 16: Ramesh wants to retire and receive Rs. 3,000 a month. He wants to pass this monthly payment to future generations after his death. He can earn an interest of 8% compounded annually. How much will he need to set aside to achieve his perpetuity goal? Solution C = Rs. 3,000

r = 0.08/12 or 0.00667

Substituting these values in the above formula, we get

00667.0000,3.RsPV =

= Rs. 4,49,775

If he wanted the payments to start today, we must increase the size of the funds to handle the first payment. This is achieved by depositing Rs. 4,52,775 which provides the immediate payment of Rs. 3,000 and leaves Rs. 4,49,775 in the fund to provide the future Rs. 3,000 payments.

7.2 CALCULATION OF GROWING PERPETUITY A stream of cash flows that grows at a constant rate forever is known as growing perpetuity. The formula for determining the present value of growing perpetuity is as follows:

( ) ( ) ( ) grC

1n n)r1(

1n)g1(C)r1()g1(C.......3r1

2)g1(C2r1

)g1(C1r1

CPV−

=∑∞

= +

−+=∞+

∞+++

+

++

+

++

+=

Illustration 17: Assuming that the discount rate is 7% per annum, how much would you pay to receive Rs. 50, growing at 5%, annually, forever? Solution

gr

CPV−

=

2.13

Page 579: 30510870 Cost Accounting and Financial Management

Financial Management

500,205.007.0

50=

−=

8. SINKING FUND It is the fund created for a specified purpose by way of sequence of periodic payments over a time period at a specified interest rate. Size of the sinking fund deposit is computed from A=P.A (n,i), where A is the amount to be saved, P, the periodic payment, n, the payment period.

Illustration 18: How much amount is required to be invested every year so as to accumulate Rs. 3,00,000 at the end of 10 years if the interest is compounded annually at 10%?

Solution

Here, A= 3,00,000 n = 10 i = 0.1

Since, A=P.A (n,i)

3,00,000=P.A(10, 0.1)

= P*15.9374248

Therefore, 62.823,189374248.15

000,00,3P ==

P = Rs. 18,823.62

9. TECHNIQUES OF DISCOUNTING The present value of a sum of money to be received at a future date is determined by discounting the future value at the interest rate that the money could earn over the period. This process is known as Discounting. The figure below shows graphically how the present value interest factor varies in response to changes in interest rate and time. The present value interest factor declines as the interest rate rises and as the length of time increases.

2.14

Page 580: 30510870 Cost Accounting and Financial Management

Time Value of Money

02 4 6 8 10 12

Periods

100

75

50

25

0 percent

6 percent

10 percent

14 percent

PVIF r, n

Graphic View of Discounting

9.1 PRESENT VALUE OF A SINGLE CASH FLOW The present value of a single cash flow is given as:

PV = FVn

n

i11⎟⎠⎞

⎜⎝⎛+

Where, FVn = Future value n years hence r = Rate of interest per annum n = Number of years for which discounting is done.

It can be seen from the above formula that present value of a future money depends upon the three variables i.e. FV, the rate of interest and time period. The published tables for various

combination of n

i11⎟⎠⎞

⎜⎝⎛+

are available.

Illustration 19: Find out the present value of Rs. 2,000 received after in 10 years hence, if discount rate is 8%. Solution

Present value of an amount = FVn

n

i11

⎟⎠⎞

⎜⎝⎛

+

2.15

Page 581: 30510870 Cost Accounting and Financial Management

Financial Management

Now, I = 8% n = 10 years

Present value of an amount = Rs. 2,000 10

08.011

⎟⎠⎞

⎜⎝⎛

+

= Rs. 2,000 (0.463) = Rs. 926 9.2 PRESENT VALUE OF AN ANNUITY Sometimes instead of a single cash flow the cash flows of the same amount is received for a number of years. The present value of an annuity may be expressed as follows :

PVAn = ( ) ( ) ( ) ( )n1n21 i1

Ai1

A...i1

Ai1

A +

++

++

++ −

= ( ) ( ) ( ) ( ) ⎟

⎟⎠

⎞⎜⎜⎝

++

++

++

+ − n1n21 i11

i11...

i11

i11 A

= A ( )( ) ⎟

⎟⎠

⎞⎜⎜⎝

+

−+n

n

i1i1i1

Where, PVAn = Present value of annuity which has duration of n years A = Constant periodic flow i = Discount rate.

Illustration 20: Find out the present value of a 4 year annuity of Rs. 20,000 discounted at 10 per cent. Solution P V = Amount of annuity × Present value (r, n) Now, i = 10%

n = 4 years

PV = Rs. 20,000 ( )

( ) ⎥⎥⎦

⎢⎢⎣

+

−+4

4

1.011.011.01 = Rs. 20,000 × 0.683

= Rs. 13,660

2.16

Page 582: 30510870 Cost Accounting and Financial Management

Time Value of Money

Illustration 21: Rs. 5,000 is paid every year for 10 years to pay off a loan. What is the loan amount if interest rate be 14% per annum compounded annually? Solution V = A, P(n, i) = 5,000 × P (10, 0.14) = 5,000 × 5.21611 = Rs. 26,080.55 Note: The students may, as an exercise, workout the interest amount. Illustration 22: Y bought a TV costing Rs. 13,000 by making a down payment of Rs. 3,000 and agreeing to make equal annual payment for 4 years. How much would be each payment if the interest on unpaid amount be 14% compounded annually? Solution In the present case, present value of the unpaid amount was (13,000 – 3,000) = Rs. 10,000. The periodic payment, A may be found from

3,432.05 Rs. 0.343205 10,000 0.14) (4, P

10,000 i) P(n,

V A =×===

Illustration 23: Z plans to receive an annuity of Rs. 5,000 semi-annually for 10 years after he retires in 18 years. Money is worth 9% compounded semi-annually. (a) How much amount is required to finance the annuity? (b) What amount of single deposit made now would provide the funds for the annuity? (c) How much will Mr. Z receive from the annuity? Solution (a) Let us first find the required present value for the 10 years annuity by using

V = A. P(n, i)

= 5,000 P(20, 4.5%)

= 5,000 × 13.00793654 = Rs. 65,039.68

Since, ( ) ( )20

20

%)5.41(045.1 %5.4 1 4.5% 20, P

+−+

=

413.0079365 0.10852713

1 2.41171402 =−

=

2.17

Page 583: 30510870 Cost Accounting and Financial Management

Financial Management (b) We require the amount of single deposit that matures to Rs. 65,039.68 in 18 years at 9%

compounded semi-annually. We use

An = P(1+ i)n, An = 65,039.68, ?P%,21 4

29 i 36, 2 18 n ====×=

Thus, P = An (1 +i)−n

36

%214 1 65,039.68

⎟⎠⎞

⎜⎝⎛ +=

= 65,039.68 × 0.20502817 = Rs. 13,334.97 (c) Required amount = Rs. 5,000 × 20 = Rs. 1,00,000.

10. TECHNIQUES OF COMPOUNDING The "time value of money" describes the effects of compounding. An amount invested today has more value than the same amount invested at a later date because it can utilize the power of compounding. Compounding is the process by which interest is earned on interest. When a principal amount is invested, interest is earned on the principal during the first period or year. In the second period or year, interest is earned on the original principal plus the interest earned in the first period. Over time, this reinvestment process can help an account grow significantly.

10.1 FUTURE VALUE (FV) OF A SINGLE CASH FLOW The future value of a single cash flow is defined as : FV = PV (1+i)2

Where, FV = Future value n years hence PV = Present value of cash flow today (given) I = Rate of interest per annum n = Number of year for which compounding is done. If any of the variable i.e. PV, I and n varies, the FV also varies. It is very tedious to calculate the value of (1+I)n. The pre-calculated values of (1+I)n for different combinations are published in the form of tables. One may refer to such tables for computation. Otherwise one should use the knowledge of logarithams. Illustration 24: A makes a deposit of Rs. 5,000 in a bank which pays 10% interest compounded annually for 6 years. You are required to find out the amount to be received after 5 years.

2.18

Page 584: 30510870 Cost Accounting and Financial Management

Time Value of Money

Solution FV = PV(1+i)n

Now, PV = Rs. 5,000, i = 10% and n = 6 years ∴ FV = Rs. 5,000 (1 + 10%)6

= Rs. 5,000 × 7.716* = Rs. 38,580 * From table of compounded value of an annuity.

10.2 FUTURE VALUE OF AN ANNUITY An annuity is a series of periodic cash flows (payments or receipts) of equal amount. The premium payments of a life insurance policy, for example, are an annuity. In general terms the future value of an annuity is given as :

FVAn = ( )⎥⎥⎦

⎢⎢⎣

⎡ −+i

1i1An

Where, FVAn = Future value of an annuity which has duration of n years

A = Constant periodic flow i = Interest rate per period n = Duration of the annuity. From the above equation it is clear that the future value of annuity is dependent on three variables i.e. the annual amount, the rate of interest and the time period. If any of these variable changes it will change the future value of the annuity. A published table is available for various combinations of the rate of interest r and the time period n. Illustration 25 : A person is required to pay four equal annual payments of Rs. 5,000 each in his deposit account that pays 8% interest per year. Find out the future value of annuity at the end of 4 years. Solution

FVA = ( )⎟⎟⎠

⎞⎜⎜⎝

⎛ −+i

1i1An

= Rs. 5,000 (4.507) = Rs. 22,535

2.19

Page 585: 30510870 Cost Accounting and Financial Management

Financial Management Self Examination Questions

A. Objective Type Questions 1. Both the future and present value of a sum of money are based on:

(a) Interest rate

(b) Number of time periods

(c) Both a and b

(d) None of the above.

2. An annuity is ___________________.

(a) More than one payment

(b) A series of unequal but consecutive payments

(c) A series of equal and consecutive payments

(d) A series of equal and non-consecutive payments.

3. Time value of money is an important finance concept because:

(a) It takes risk into account

(b) It takes time into account

(c) It takes compound interest into account

(d) All of the above.

4. The concepts of present value and future value are:

(a) Directly related to each other

(b) Not related to each other

(c) Proportionately related to each other

(d) Inversely related to each other.

5. If you have Rs.1000 and you plan to save it for 4 years with an interest rate of 10%, what is the future value of your savings?

(a) Rs.1464.00

2.20

Page 586: 30510870 Cost Accounting and Financial Management

Time Value of Money

(b) Rs.1000.00

(c) Rs.1331.00

(d) Cannot be determined.

6. To increase a given present value, the discount rate should be adjusted:

(a) Upward

(b) Downward

(c) True

(d) False.

7. In three years you are to receive Rs. 5,000. If the interest rate were to suddenly increase, the present value of that future amount to you would:

(a) Fall

(b) Rise

(c) Remain unchanged

(d) Cannot be determined without more information.

Answers to Objective Type Questions

1. (c); 2. (c); 3. (d); 4. (d); 5. (a); 6. (b); 7. (a)

B. Short Answer Type Questions

1. Define the following terms:

(a) Annuity

(b) Perpetuity

(c) Sinking Fund

(d) Simple Interest

(e) Compound Interest.

2. Write short notes on the following:

(a) Time Value of Money

2.21

Page 587: 30510870 Cost Accounting and Financial Management

Financial Management

(b) Effective Rate of Interest

(c) Discounting Techniques

(d) Compounding Techniques.

C. Long Answer Type Questions

1. What is relevance of time value of money in financial decision making?

2. Explain the discounting and compounding techniques of time value of money.

D. Practical Problems

1. A makes a deposit of Rs. 1 0 ,000 in a bank which pays 10% interest compounded annually for 6 years. You are required to find out the amount to be received after 5 years.

2. A person is required to pay four equal annual payments of Rs. 10,000 each in his deposit account that pays 8% interest per year. Find out the future value of annuity at the end of 4 years.

3. Find out the present value of Rs. 4,000 received after in 10 years hence, if discount rate is 8%.

4. Find out the present value of a 4 year annuity of Rs. 10,000 discounted at 10 per cent.

5. If Ramesh wishes to withdraw Rs. 8,000 seven years from now and the interest rate is 12% compounded annually, then how much amount he must deposit today?

6. If a person makes a series of Rs. 5,000 deposits at the end of each of the next 5 years and the interest rate is 12% compounded annually, what will be the future value of these deposits.

7. A company anticipates capital expenditure of Rs. 50,000 for new equipment in 10 years. How much should be deposited annually in a sinking fund earning 10% per year compounded annually to provide for the purchase?

8. A man, aged 35 years intends to invest now at 7% per year compounded semiannually to receive Rs.50,000 at the age of 65 years. How much should be his present investment?

Given that (1 + 0.35/2)−60 = .126934.

9. An investment is made for 4 years at 7% compounded quarterly so as to have a maturity value of Rs.6,000. What is the amount of investment? What is the amount of interest?

2.22

Page 588: 30510870 Cost Accounting and Financial Management

CHAPTER 3

FINANCIAL ANALYSIS AND PLANNING

UNIT – I : APPLICATION OF RATIO ANALYSIS FOR PERFORMANCE EVALUATION, FINANCIAL HEALTH AND DECISION MAKING

Learning Objectives After studying this chapter, you will be able to understand ♦ What is financial analysis and how it helps in decision making?

♦ Learn about the important tools and techniques of financial analysis like ratio analysis.

1.1 INTRODUCTION The basis for financial analysis, planning and decision making is financial information. A business firm prepares its final accounts viz., Balance Sheet and Profit and Loss Account which provide useful financial information for the purpose of decision making. Financial information is needed to predict, compare and evaluate the firm’s earning ability. The former statement viz profit & loss account shows the operating activities of the concern and the latter balance sheet depicts the balance value of the acquired assets and of liabilities at a particular point of time. However, these statements do not disclose all of the necessary and relevant information. For the purpose of obtaining the material and relevant information necessary for ascertaining the financial strengths and weaknesses of an enterprise, it is necessary to analyse the data depicted in the financial statement. The financial manager has certain analytical tools which help in financial analysis and planning. For instance, a cash flow statement is a valuable aid to a financial manager in evaluating the inflows and outflows of cash i.e. sources and applications of cash during particular period. In addition, ratio helps the manager to analyse the past performance of the firm and to make future projections.

1.2 RATIO ANALYSIS Ratio Analysis is a widely used tool of financial analysis. The term ratio in it refers to the relationship expressed in mathematical terms between two individual figures or group of figures connected with each other in some logical manner and are selected from financial statements of the concern. The ratio analysis is based on the fact that a single accounting figure by itself may not communicate any meaningful information but when expressed as a

Page 589: 30510870 Cost Accounting and Financial Management

Financial Management relative to some other figure, it may definitely provide some significant information. The relationship between two or more accounting figures/groups is called a financial ratio. A financial ratio helps to express the relationship between two accounting figures in such a way that users can draw conclusions about the performance, strengths and weaknesses of a firm. Ratio analysis is not just comparing different numbers from the balance sheet, income statement, and cash flow statement. It is comparing the number against previous years, other companies, the industry, or even the economy in general. Ratios look at the relationships between individual values and relate them to how a company has performed in the past, and might perform in the future. All stakeholders within the company need to be able to appreciate how the company is performing. Their understanding of how the firm is performing is enhanced through ratio analysis.

1.3 TYPES OF RATIOS Broadly speaking, the operations and financial position of a firm can be described by studying its short term and long term liquidity position, profitability and its operational activities. Therefore, ratios can be classified into following four broad categories: (i) Liquidity Ratios (ii) Capital Structure/Leverage Ratios (iii) Activity Ratios (iv) Profitability Ratios

1.3.1 LIQUIDITY RATIOS The terms ‘liquidity’ and ‘short-term solvency’ are used synonymously. Liquidity or short-term solvency means ability of the business to pay its short-term liabilities. Inability to pay-off short-term liabilities affects its credibility as well as its credit rating. Continuous default on the part of the business leads to commercial bankruptcy. Eventually such commercial bankruptcy may lead to its sickness and dissolution. Short-term lenders and creditors of a business are very much interested to know its state of liquidity because of their financial stake. Traditionally, two ratios are used to highlight the business ‘liquidity’. These are current ratio and quick ratio. Other ratios include cash ratio, interval measure ratio and net working capital ratio. 1.3.1.1 Current Ratios: The Current Ratio is one of the best known measures of financial strength.

Current Ratio = Current Assets / Current Liabilities

3.2

Page 590: 30510870 Cost Accounting and Financial Management

Financial Analysis and Planning

Where, Current Assets = Inventories + Sundry Debtors + Cash and Bank Balances +

Receivables/ Accruals + Loans and Advances + Disposable Investments

Current Liabilities = Creditors for goods and services + Short-term Loans + Bank Overdraft + Cash Credit + Outstanding Expenses + Provision for Taxation + Proposed Dividend + Unclaimed Dividend

The main question this ratio addresses is: "Does your business have enough current assets to meet the payment schedule of its current debts with a margin of safety for possible losses in current assets?" A generally acceptable current ratio is 2 to 1. But whether or not a specific ratio is satisfactory depends on the nature of the business and the characteristics of its current assets and liabilities. 1.3.1.2 Quick Ratios: The Quick Ratio is sometimes called the "acid-test" ratio and is one of the best measures of liquidity.

Quick Ratio or Acid Test Ratio = Quick Assets/ Quick Liabilities Where, Quick Assets = Current Assets −Inventories Quick Liabilities = Current Liabilities − Bank Overdraft − Cash Credit The Quick Ratio is a much more exacting measure than the Current Ratio. By excluding inventories, it concentrates on the really liquid assets, with value that is fairly certain. It helps answer the question: "If all sales revenues should disappear, could my business meet its current obligations with the readily convertible `quick' funds on hand?" Quick Assets consist of only cash and near cash assets. Inventories are deducted from current assets on the belief that these are not ‘near cash assets’. But in a seller’s market inventories are also near cash assets. Moreover, just like lag in collection of debtors, there is a lag in conversion of inventories into finished goods and sundry debtors. Obviously slow moving inventories are not near cash assets. However, while calculating the quick ratio we have followed the conservatism convention. Quick liabilities are that portion of current liabilities which fall due immediately. Since bank overdraft and cash credit can be used as a source of finance as and when required, it is not included in the calculation of quick liabilities. An acid-test of 1:1 is considered satisfactory unless the majority of "quick assets" are in accounts receivable, and the pattern of accounts receivable collection lags behind the schedule for paying current liabilities.

3.3

Page 591: 30510870 Cost Accounting and Financial Management

Financial Management 1.3.1.3 Cash Ratio/ Absolute Liquidity Ratio: The cash ratio measures the absolute liquidity of the business. This ratio considers only the absolute liquidity available with the firm. This ratio is calculated as:

Ratio CashsLiabilitieCurrent

Securities Marketable Cash=

+

A subsequent innovation in ratio analysis, the Absolute Liquidity Ratio eliminates any unknowns surrounding receivables. The Absolute Liquidity Ratio only tests short-term liquidity in terms of cash and marketable securities. 1.3.1.4 Basic Defense Interval

Taxes)/365 Income Interest Expenses Operating ()Securities Marketable sReceivable (Cash Interval DefenseBasic

++++

=

If for some reason all the company’s revenues were to suddenly cease, the Basic Defense Interval would help determine the number of days the company can cover its cash expenses without the aid of additional financing. 1.3.1.5 Net Working Capital Ratio: Net working capital is more a measure of cash flow than a ratio. The result of this calculation must be a positive number. It is calculated as shown below: Net Working Capital Ratio = Current Assets - Current Liabilities (excluding short-term bank

borrowing) Bankers look at Net Working Capital over time to determine a company's ability to weather financial crises. Loans are often tied to minimum working capital requirements.

1.3.2 CAPITAL STRUCTURE/LEVERAGE RATIOS The capital structure/leverage ratios may be defined as those financial ratios which measure the long term stability and structure of the firm. These ratios indicate the mix of funds provided by owners and lenders and assure the lenders of the long term funds with regard to: (i) Periodic payment of interest during the period of the loan and (ii) Repayment of principal amount on maturity. Therefore leverage ratios are of two types : (a) Capital structure ratios and (b) Coverage ratios.

3.4

Page 592: 30510870 Cost Accounting and Financial Management

Financial Analysis and Planning

1.3.2.1 Capital Structure Ratios: These ratios provide an insight into the financing techniques used by a business and focus, as a consequence, on the long-term solvency position. From the balance sheet one can get only the absolute fund employed and its sources, but only capital structure ratios show the relative weight of different sources. In the balance sheet the student may find shareholders’ fund, loan fund and current liabilities and provisions. These are very often classified as owners’ equities and external equities. “Owners’ Equity” means share capital, both equity share capital and preference share capital and reserves and surplus. ‘External Equity’ means all outside liabilities (inclusive of current liabilities and provisions). Also these are sometimes classified as equity and debt. ‘Equity’ means shareholders fund and ‘Debt’ means long term borrowed fund (so short-term loans, current liabilities and provisions are excluded). As per guidelines for issue of ‘Debentures by Public Limited Company’ debt means term loans, debentures and bonds with an initial maturity period of five years or more, including interest accrued thereon. It also includes all deferred payment liabilities but it does not include short term bank borrowing and advances, unsecured deposits or loans from the public, shareholders and employees, and unsecured loans and deposits from others. It should also include proposed debenture issue. Equity means paid up share capital including preference share capital and reserves. Three popularly used capital structure ratios are: (a) Equity Ratio

Employed Capital Total

Equity rs'Shareholde RatioEquity =

This ratio indicates proportion of owners’ fund to total fund invested in the business. Traditionally, it is believed that higher the proportion of owners’ fund lower is the degree of risk. (b) Debt Ratio

Employed Capital

Debt Total RatioDebt =

Total debt includes short and long term borrowings from financial institutions, debentures/bonds, deferred payment arrangements for buying capital equipments, bank borrowings, public deposits and any other interest bearing loan. Capital employed includes total debt and net worth. This ratio is used to analyse the long-term solvency of a firm. (c) Debt to Equity Ratio

Equity rs'Shareholde

Term Long Preferred Debt RatioEquity toDebt +=

3.5

Page 593: 30510870 Cost Accounting and Financial Management

Financial Management A high ratio here means less protection for creditors. A low ratio, on the other hand, indicates a wider safety cushion (i.e., creditors feel the owner's funds can help absorb possible losses of income and capital). This ratio indicates the proportion of debt fund in relation to equity. This ratio is very often referred in capital structure decision as well as in the legislation dealing with the capital structure decisions (i.e. issue of shares and debentures). Lenders are also very keen to know this ratio since it shows relative weights of debt and equity. Debt equity ratio is the indicator of leverage. According to the traditional school, cost of capital firstly decreases due to the higher dose of leverage, reaches minimum and thereafter increases. So infinite increase in leverage (i.e. debt-equity ratio) is not possible. But according to Modigliani-Miller theory, cost of capital and leverage are independent of each other. But Modigliani-Miller theory is based on certain restrictive assumptions, namely, perfect capital market, homogeneous expectations by the present and prospective investors, presence of homogeneous risk class firms, 100% dividend pay-out, no tax situation, etc. And most of these assumptions are viewed as unrealistic. It is believed that leverage and cost of capital are not unrelated. Presently, there is no norm for maximum debt-equity ratio. Lending institutions generally set their own norms considering the capital intensity and other factors. 1.3.2.2 Coverage Ratios: The coverage ratios measure the firm’s ability to service the fixed liabilities. These ratios establish the relationship between fixed claims and what is normally available out of which these claims are to be paid. The fixed claims consist of: (i) Interest on loans (ii) Preference dividend (iii) Amortisation of principal or repayment of the instalment of loans or redemption of

preference capital on maturity. The following are important coverage ratios : (a) Debt Service Coverage Ratio: Lenders are interested in debt service coverage to judge the firm’s ability to pay off current interest and instalments.

tsInstallmenInterestservicedebt for available EarningsRatio Coverage ServiceDebt

+=

Earning for debt service = Net profit + Non-cash operating expenses like depreciation and other amortizations + Non-operating adjustments like loss on sale of + Fixed assets + Interest on Debt Fund.

3.6

Page 594: 30510870 Cost Accounting and Financial Management

Financial Analysis and Planning

(b) Interest Coverage Ratio : This ratio also known as “times interest earned ratio” indicates the firm’s ability to meet interest (and other fixed-charges) obligations. This ratio is computed as :

Interest

EBIT Ratio CoverageInterest =

Earnings before interest and taxes are used in the numerator of this ratio because the ability to pay interest is not affected by tax burden as interest on debt funds is deductible expense. This ratio indicates the extent to which earnings may fall without causing any embarrassment to the firm regarding the payment of interest charges. A high interest coverage ratio means that an enterprise can easily meet its interest obligations even if earnings before interest and taxes suffer a considerable decline. A lower ratio indicates excessive use of debt or inefficient operations. (c) Preference Dividend Coverage Ratio: This ratio measures the ability of a firm to pay dividend on preference shares which carry a stated rate of return. This ratio is computed as:

liability dividend Preference

EAT Ratio Coverage Dividend eferencePr =

Earnings after tax is considered because unlike debt on which interest is charged on the profit of the firm, the preference dividend is treated as appropriation of profit. This ratio indicates margin of safety available to the preference shareholders. A higher ratio is desirable from preference shareholders point of view. (d) Capital Gearing Ratio: In addition to debt-equity ratio, sometimes capital gearing ratio is also calculated to show the proportion of fixed interest (dividend) bearing capital to funds belonging to equity shareholders.

Losses) Surplus& Reserves Capital Share(Equity Loan) Term Long Debentures Capital Share eference(Pr Ratio Gearing apitalC

−+++

=

For judging long term solvency position, in addition to debt-equity ratio and capital gearing ratio, the following ratios are also used:

(i) Fund Term Long

AssetsFixed

It is expected that fixed assets and core working capital are to be covered by long term fund. In various industries the proportion of fixed assets and current assets are different. So there is no uniform standard of this ratio too. But it should be less than one. If it is more than one, it

3.7

Page 595: 30510870 Cost Accounting and Financial Management

Financial Management means short-term fund has been used to finance fixed assets. Very often many companies resort to such practice during expansion. This may be a temporary arrangement but not a long term remedy.

(ii) AssetsTotal

Fundy Proprietar Ratioy Proprietar =

Proprietary fund includes Equity Share Capital + Preference Share Capital + Reserve & Surplus – Fictitious Assets. Total assets exclude fictitious assets and losses. If one follows standard current ratio 2 : 1 and standard debt-equity ratio 2 : 1, what should be the standard proprietary ratio ? Let Rs. 100 be the total assets of which Rs. 20 be the current assets. Then following standard current ratio Rs. 10 is financed by current liabilities, remaining Rs. 90 is financed by debt and equity. Since following standard debt-equity ratio equity component is 1/3, it is expected that out of Rs. 90, Rs. 30 should come from proprietary fund. If the current assets component increases equity commitment will be reduced and vice- versa.

1.3.3 ACTIVITY RATIO The activity ratios are also called the Turnover ratios or Performance ratios. These ratios are employed to evaluate the efficiency with which the firm manages and utilises its assets. These ratios usually indicate the frequency of sales with respect to its assets. These assets may be capital assets or working capital or average inventory. These ratios are usually calculated with reference to sales/cost of goods sold and are expressed in terms of rate or times. Some of the important activity ratios are as follows: (a) Capital Turnover Ratio

Employed Capital

alesS Ratio Turnover Capital =

This ratio indicates the firm’s ability of generating sales per rupee of long term investment. The higher the ratio, the more efficient the utilisation of owner’s and long-term creditors’ funds. (b) Fixed Assets Turnover Ratio

AssetsCapitalSales Ratio Turnover AssetsFixed =

A high fixed assets turnover ratio indicates efficient utilisation of fixed assets in generating sales. A firm whose plant and machinery are old may show a higher fixed assets turnover ratio than the firm which has purchased them recently.

3.8

Page 596: 30510870 Cost Accounting and Financial Management

Financial Analysis and Planning

(c) Working Capital Turnover

Capital WorkingalesS Turnover Capital Working =

Working Capital Turnover is further segregated into Inventory Turnover, Debtors Turnover, Creditors Turnover. (i) Inventory Turnover Ratio: This ratio also known as stock turnover ratio establishes the relationship between the cost of goods sold during the year and average inventory held during the year. It is calculated as follows:

*Inventory Average

Sales Ratio TurnoverInventory =

2

Stock Closing Stock OpeningInventory Average* +=

Very often inventory turnover is calculated with reference to cost of sales instead of sales. In that case inventory turnover will be calculated as :

Stock AverageSales of Cost

Note : Students are advised to follow this formula for calculating inventory turnover ratio. In the case of inventory of raw material the inventory turnover ratio is calculated using the following formula :

Stock Material Raw Average

Consumed Material Raw

This ratio indicates that how fast inventory is used/sold. A high ratio is good from the view point of liquidity and vice versa. A low ratio would indicate that inventory is not used/ sold/ lost and stays in a shelf or in the warehouse for a long time. (ii) Debtors’ Turnover Ratio: In case firm sells goods on credit, the realization of sales revenue is delayed and the receivables are created. The cash is realised from these receivables later on. The speed with which these receivables are collected affects the liquidity position of the firm. The debtors turnover ratio throws light on the collection and credit policies of the firm. It is calculated as follows:

Receivable AccountsAverageSales

3.9

Page 597: 30510870 Cost Accounting and Financial Management

Financial Management As account receivables pertains only to credit sales, it is often recommended to compute the debtor’s turnover with reference to credit sales instead of total sales. Then the debtor’s turnover would be

Receivable AccountsAverageSales Credit

Note : Students are advised to follow this formula for calculating debtors’ turnover ratio. (iii) Creditors’ Turnover Ratio: This ratio is calculated on the same lines as receivable turnover ratio is calculated. This ratio shows the velocity of debt payment by the firm. It is calculated as follows:

Payable AccountsAveragePurchasesCredit Net AnnualRatio Turnover Creditors =

A low creditor’s turnover ratio reflects liberal credit terms granted by supplies. While a high ratio shows that accounts are settled rapidly.

Payable AccountsAveragePurchases Credit

Debtors’ turnover ratio indicates the average collection period. However, the average collection period can be directly calculated as follows:

SalesCredit Daily AveragesReceivable AccountsAverage

365Sales CreditSalesCredit Daily Average =

Similarly, average payment period can be calculated using :

PurchasesCredit Daily AveragePayable AccountsAverage

In determining the credit policy, debtor’s turnover and average collection period provide a unique guideline. The firm can compare what credit period it receives from the suppliers and what it offers to the customers. Also it can compare the average credit period offered to the customers in the industry to which it belongs.

1.3.4 PROFITABILITY RATIO The profitability ratios measure the profitability or the operational efficiency of the firm. These ratios reflect the final results of business operations. The results of the firm can be evaluated in

3.10

Page 598: 30510870 Cost Accounting and Financial Management

Financial Analysis and Planning

terms of its earnings with reference to a given level of assets or sales or owner’s interest etc. Therefore, the profitability ratios are broadly classified in four categories: (i) Profitability ratios required for analysis from owners’ point of view (ii) Profitability ratios based on assets/investments (iii) Profitability ratios based on sales of the firm (iv) Profitability ratios based on capital market information. 1.3.4.1 Profitability Ratios Required for Analysis from Owner’s Point of View (a) Return on Equity (ROE) : Return on Equity measures the profitability of equity funds invested in the firm. This ratio reveals how profitability of the owners’ funds have been utilised by the firm. This ratio is computed as:

Net worthtaxes afterProfit ROE =

Return on equity is one of the most important indicators of a firm’s profitability and potential growth. Companies that boast a high return on equity with little or no debt are able to grow without large capital expenditures, allowing the owners of the business to withdraw cash and reinvest it elsewhere. Many investors fail to realize, however, that two companies can have the same return on equity, yet one can be a much better business. For that reason, a finance executive at E.I. Du Pont de Nemours and Co., of Wilmington, Delaware, created the DuPont system of financial analysis in 1919. That system is used around the world today and serves as the basis of components that make up return on equity. Composition of Return on Equity using the DuPont Model There are three components in the calculation of return on equity using the traditional DuPont model- the net profit margin, asset turnover, and the equity multiplier. By examining each input individually, the sources of a company's return on equity can be discovered and compared to its competitors. (i) Net Profit Margin: The net profit margin is simply the after-tax profit a company generates for each rupee of revenue. Net profit margins vary across industries, making it important to compare a potential investment against its competitors. Although the general rule-of-thumb is that a higher net profit margin is preferable, it is not uncommon for management to purposely lower the net profit margin in a bid to attract higher sales. Net profit margin = Net Income ÷ Revenue Net profit margin is a safety cushion; the lower the margin, the less room for error. A business with 1% margins has no room for flawed execution. Small miscalculations on management’s part could lead to tremendous losses with little or no warning.

3.11

Page 599: 30510870 Cost Accounting and Financial Management

Financial Management (ii) Asset Turnover: The asset turnover ratio is a measure of how effectively a company converts its assets into sales. It is calculated as follows: Asset Turnover = Revenue ÷ Assets The asset turnover ratio tends to be inversely related to the net profit margin; i.e., the higher the net profit margin, the lower the asset turnover. The result is that the investor can compare companies using different models (low-profit, high-volume vs. high-profit, low-volume) and determine which one is the more attractive business. (iii) Equity Multiplier: It is possible for a company with terrible sales and margins to take on excessive debt and artificially increase its return on equity. The equity multiplier, a measure of financial leverage, allows the investor to see what portion of the return on equity is the result of debt. The equity multiplier is calculated as follows: Equity Multiplier = Assets ÷ Shareholders’ Equity. Calculation of Return on Equity To calculate the return on equity using the DuPont model, simply multiply the three components (net profit margin, asset turnover, and equity multiplier.) Return on Equity = (Net Profit Margin) (Asset Turnover) (Equity Multiplier)

Assets Turnover = Sales ÷ NA

Profit Margin = EBIT ÷ Sales

Financial Leverage (Balance Sheet) = NA ÷ NW

Return on Net Assets (RONA) = EBIT ÷ NA

Financial Leverage (Income) = PAT ÷ E BIT

Return on Equity (ROE) = PAT ÷ NW

Du Pont Chart

3.12

Page 600: 30510870 Cost Accounting and Financial Management

Financial Analysis and Planning

Illustration 1 XYZ Company’s details are as under: Revenue: Rs. 29,261; Net Income: Rs. 4,212 ; Assets: Rs. 27,987; Shareholders’ Equity: Rs. 13,572. Calculate return on equity. Solution Net Profit Margin = Net Income (Rs. 4,212) ÷ Revenue (Rs. 29,261) = 0.1439, or 14.39% Asset Turnover = Revenue (Rs. 29,261) ÷ Assets (Rs. 27,987) = 1.0455 Equity Multiplier = Assets (Rs. 27,987) ÷ Shareholders’ Equity (Rs. 13,572) = 2.0621 Finally, we multiply the three components together to calculate the return on equity: Return on Equity= (0.1439) x (1.0455) x (2.0621) = 0.3102, or 31.02% Analysis: A 31.02% return on equity is good in any industry. Yet, if you were to leave out the equity multiplier to see how much company would earn if it were completely debt-free, you will see that the ROE drops to 15.04%. In other words, for fiscal year 2004, 15.04% of the return on equity was due to profit margins and sales, while 15.96% was due to returns earned on the debt at work in the business. If you found a company at a comparable valuation with the same return on equity yet a higher percentage arose from internally-generated sales, it would be more attractive. (b) Earnings per Share: The profitability of a firm from the point of view of ordinary shareholders can be measured in terms of number of equity shares. This is known as Earnings per share. It is calculated as follows:

goutstandin sharesordinary of Numberholdersequity to availableprofit Net(EPS) share per Earnings =

(c) Dividend per Share: Earnings per share as stated above reflects the profitability of a firm per share; it does not reflect how much profit is paid as dividend and how much is retained by the business. Dividend per share ratio indicates the amount of profit distributed to shareholders per share. It is calculated as:

sharesequity of Numberholders shareequity to ddistribute profits Total share per ividendD =

(d) Price Earning Ratio: The price earning ratio indicates the expectation of equity investors about the earnings of the firm. It relates earnings to market price and is generally taken as a summary measure of growth potential of an investment, risk characteristics, shareholders orientation, corporate image and degree of liquidity. It is calculated as:

3.13

Page 601: 30510870 Cost Accounting and Financial Management

Financial Management

share per Earnings share per pricearket M Ratio EP =

1.3.4.2 Profitability Ratios based on Assets/Investments : (a) Return on Capital Employed/Return on Investment: ROI is the most important ratio of all. It is the percentage of return on funds invested in the business by its owners. In short, this ratio tells the owner whether or not all the effort put into the business has been worthwhile. The ROI is calculated as follows:

100 Employed Capital

Return Employed Capital on Return ×=

Where, Return = Net Profit

± Non-trading adjustments (but not accrual adjustments for amortization of preliminary expenses, goodwill, etc.)

+ Interest on long term debts + Provision for tax – Interest/Dividend from non-trade investments

Capital Employed = Equity Share Capital + Reserve and Surplus + Pref. Share Capital + Debentures and other long term loan – Misc. expenditure and losses – Non-trade Investments.

Intangible assets (assets which have no physical existence like goodwill, patents and trade marks) should be included in the capital employed. But no fictitious asset should be included within capital employed. (b) Return on Investment

100 Employed Capital

Return ROI ×=

= 100 Employed Capital

Sales SalesReturn

××

3.14

Page 602: 30510870 Cost Accounting and Financial Management

Financial Analysis and Planning

Ratio yofitabilitPr 100 SalesReturn

Ratio Turnover Capital Employed Capital

Sales =

So, ROI = Profitability Ratio × Capital Turnover Ratio ROI can be improved either by improving operating profit ratio or capital turnover or by both. (c) Return on Assets (ROA): The profitability ratio is measured in terms of relationship between net profits and assets employed to earn that profit. This ratio measures the profitability of the firm in terms of assets employed in the firm. The ROA may be measured as follows:

or assets total Average

taxes afterprofit Net ROA =

or assets tangible Averagetaxes afterprofit Net =

or assets fixed Average

taxes afterprofit Net =

1.3.4.3 Profitability Ratios based on Sales of Firm (a) Gross Profit Ratio

Profit GrossProfi Gross

This ratio is used to compaclassified suitably into fixed aalso find out P/V ratio.

RatioP/V

Fixed cost remaining same, hOperating profit ratio is also c(b) Operating Profit Ratio

Ope RatioProfit Operating =

100 Sales

Ratiot ×=

re departmental profitability or product profitability. If costs are nd variable elements, then instead of Gross Profit Ratio one can

100 Sales

Cost Variable Sales ×−

=

igher P/V Ratio lowers the break even point. alculated to evaluate operating performance of business.

100 Sales

Profit rating×

3.15

Page 603: 30510870 Cost Accounting and Financial Management

Financial Management Where, Operating Profit = Sales – Cost of Sales. (c) Net Profit Ratio

It measures overall profitability of the business

100 Sales

ProfitNet RatioProfit Net ×=

1.3.4.4 Profitability Ratios based on Capital Market Information Frequently share prices data are punched with the accounting data to generate new set of information. These are (a) Price- Earning Ratio, (b) Yield, (c) Market Value/Book Value per share. (a) Price- Earning Ratio

EPS

Price Share Average Ratio) (P/E Ratio EarningsPrice =−

(Sometimes it is also calculated with reference to closing share price).

EPS

Price Share Closing Ratio P/E =

It indicates the pay back period to the investors or prospective investors. (b) Yield

100 Price Share Average

Dividend Yield ×=

100 Price Share Closing

Dividend ro ×

This ratio indicates return on investment; this may be on average investment or closing investment. Dividend (%) indicates return on paid up value of shares. But yield (%) is the indicator of true return in which share capital is taken at its market value. (c) Market Value/Book Value per Share

SharesEquity of Number worth/ NetPrice Share Average

share per value Bookshare per valueMarket

=

SharesEquity of Number/worth Net

Price Share Closing or

3.16

Page 604: 30510870 Cost Accounting and Financial Management

Financial Analysis and Planning

This ratio indicates market response of the shareholders’ investment. Undoubtedly, higher the ratios better is the shareholders’ position in terms of return and capital gains.

1.4 APPLICATION OF RATIO ANALYSIS IN FINANCIAL DECISION MAKING A popular technique of analysing the performance of a business concern is that of financial ratio analysis. As a tool of financial management, they are of crucial significance. The importance of ratio analysis lies in the fact that it presents facts on a comparative basis and enables drawing of inferences regarding the performance of a firm. Ratio analysis is relevant in assessing the performance of a firm in respect of following aspects:

1.4.1 FINANCIAL RATIOS FOR EVALUATING PERFORMANCE (a) Liquidity Position: With the help of ratio analysis one can draw conclusions regarding liquidity position of a firm. The liquidity position of a firm would be satisfactory if it is able to meet its current obligations when they become due. A firm can be said to have the ability to meet its short-term liabilities if it has sufficient liquid funds to pay the interest on its short maturing debt usually within a year as well the principal. This ability is reflected in the liquidity ratios of a firm. The liquidity ratios are particularly useful in credit analysis by banks and other suppliers of short-term loans. (b) Long-term Solvency: Ratio analysis is equally useful for assessing the long-term financial viability of a firm. This aspect of the financial position of a borrower is of concern to the long term creditors, security analysts and the present and potential owners of a business. The long term solvency is measured by the leverage/capital structure and profitability ratios which focus on earning power and operating efficiency. Ratio analysis reveals the strengths and weaknesses of a firm in this respect. The leverage ratios, for instance, will indicate whether a firm has a reasonable proportion of various sources of finance or whether heavily loaded with debt in which case its solvency is exposed to serious strain. Similarly, the various profitability ratios would reveal whether or not the firm is able to offer adequate return to its owners consistent with the risk involved. (c) Operating Efficiency: Ratio analysis throws light on the degree of efficiency in the management and utilisation of its assets. The various activity ratios measure this kind of operational efficiency. In fact, the solvency of a firm is, in the ultimate analysis, dependent upon the sales revenues generated by the use of its assets – total as well as its components. (d) Overall Profitability : Unlike the outside parties which are interested in one aspect of the financial position of a firm, the management is constantly concerned about the overall profitability of the enterprise. That is, they are concerned about the ability of the firm to meet its short-term as well as long-term obligations to its creditors, to ensure a reasonable return to its owners and secure optimum utilisation of the assets of the firm. This is possible if an integrated view is taken and all the ratios are considered together.

3.17

Page 605: 30510870 Cost Accounting and Financial Management

Financial Management (e) Inter-firm Comparison: Ratio analysis not only throws light on the financial position of a firm but also serves as a stepping stone to remedial measures. This is made possible due to inter-firm comparison/comparison with industry averages. A single figure of particular ratio is meaningless unless it is related to some standard or norm. One of the popular techniques is to compare the ratios of a firm with the industry average. It should be reasonably expected that the performance of a firm should be in broad conformity with that of the industry to which it belongs. An inter-firm comparison would demonstrate the relative position vis-a-vis its competitors. If the results are at variance either with the industry average or with those of the competitors, the firm can seek to identify the probable reasons and, in the light, take remedial measures. Ratios not only perform post mortem of operations, but also serve as barometer for future. Ratios have predictory value and they are very helpful in forecasting and planning the business activities for a future. It helps in budgeting. Conclusions are drawn on the basis of the analysis obtained by using ratio analysis. The decisions affected may be whether to supply goods on credit to a concern, whether bank loans will be made available, etc. (f) Financial Ratios for Budgeting: In this field ratios are able to provide a great deal of assistance, budget is only an estimate of future activity based on past experience, in the making of which the relationship between different spheres of activities are invaluable. It is usually possible to estimate budgeted figures using financial ratios. Ratios also can be made use of for measuring actual performance with budgeted estimates. They indicate directions in which adjustments should be made either in the budget or in performance to bring them closer to each other.

1.5 LIMITATIONS OF FINANCIAL RATIOS The limitations of financial ratios are listed below: (i) Diversified product lines: Many businesses operate a large number of divisions in quite different industries. In such cases ratios calculated on the basis of aggregate data cannot be used for inter-firm comparisons. (ii) Financial data are badly distorted by inflation: Historical cost values may be substantially different from true values. Such distortions of financial data are also carried in the financial ratios. (iii) Seasonal factors may also influence financial data: Illustration 2: A company deals in summer garments. It keeps a high inventory during October - January every year. For the rest of the year its inventory level becomes just 1/4th of the seasonal inventory level.

3.18

Page 606: 30510870 Cost Accounting and Financial Management

Financial Analysis and Planning

So liquidity ratios and inventory ratios will produce biased picture. Year end picture may not be the average picture of the business. Sometimes it is suggested to take monthly average inventory data instead of year end data to eliminate seasonal factors. But for external users it is difficult to get monthly inventory figures. (Even in some cases monthly inventory figures may not be available). (iv) To give a good shape to the popularly used financial ratios (like current ratio, debt- equity ratios, etc.): The business may make some year-end adjustments. Such window dressing can change the character of financial ratios which would be different had there been no such change. (v) Differences in accounting policies and accounting period: It can make the accounting data of two firms non-comparable as also the accounting ratios. (vi) There is no standard set of ratios against which a firm’s ratios can be compared: Some times a firm’s ratios are compared with the industry average. But if a firm desires to be above the average, then industry average becomes a low standard. On the other hand, for a below average firm, industry averages become too high a standard to achieve. (vii) It is very difficult to generalise whether a particular ratio is good or bad: For example, a low current ratio may be said ‘bad’ from the point of view of low liquidity, but a high current ratio may not be ‘good’ as this may result from inefficient working capital management. (viii) Financial ratios are inter-related, not independent: Viewed in isolation one ratio may highlight efficiency. But when considered as a set of ratios they may speak differently. Such interdependence among the ratios can be taken care of through multivariate analysis. Financial ratios provide clues but not conclusions. These are tools only in the hands of experts because there is no standard ready-made interpretation of financial ratios.

1.6 SUMMARY OF RATIOS Another way of categorizing the ratios is being shown to you in a tabular form. A summary of the ratios has been tabulated as under:

1.6.1 PROFITABILITY RATIOS These ratios tell us whether a business is making profits - and if so whether at an acceptable rate. The key ratios are:

Ratio Calculation Comments Gross Profit Margin

[Gross Profit / Revenue] x 100 (expressed as a percentage

This ratio tells us something about the business's ability consistently to control its production costs or to manage the margins it makes on products it buys and sells. Whilst sales value and volumes

3.19

Page 607: 30510870 Cost Accounting and Financial Management

Financial Management

may move up and down significantly, the gross profit margin is usually quite stable (in percentage terms). However, a small increase (or decrease) in profit margin, however caused can produce a substantial change in overall profits.

Operating Profit Margin

[Operating Profit / Revenue] x 100 (expressed as a percentage)

Assuming a constant gross profit margin, the operating profit margin tells us something about a company's ability to control its other operating costs or overheads.

Return on Capital Employed ("ROCE")

Net profit before tax, interest and dividends ("EBIT") / Total Assets (or total assets less current liabilities

ROCE is sometimes referred to as the "primary ratio"; it tells us what returns management has made on the resources made available to them before making any distribution of those returns.

1.6.2 EFFICIENCY RATIOS These ratios give us an insight into how efficiently the business is employing those resources invested in fixed assets and working capital.

Ratio Calculation Comments Sales /Capital Employed

Sales / Capital employed

A measure of total asset utilisation. Helps to answer the question - What sales are being generated by each rupee’s worth of assets invested in the business? Note, when combined with the return on sales, it generates the primary ratio - ROCE.

Sales or Profit / Fixed Assets

Sales or profit / Fixed Assets

This ratio is about fixed asset capacity. A reducing sales or profit being generated from each rupee invested in fixed assets may indicate overcapacity or poorer-performing equipment.

Stock Turnover Cost of Sales / Average Stock Value

Stock turnover helps answer questions such as "Have we got too much money tied up in inventory"?. An increasing stock turnover figure or one which is much larger than the "average" for an industry, may indicate poor stock management.

3.20

Page 608: 30510870 Cost Accounting and Financial Management

Financial Analysis and Planning

Credit Given / "Debtor Days"

(Trade debtors (average, if possible) / (Sales)) x 365

The "debtor days" ratio indicates whether debtors are being allowed excessive credit. A high figure (more than the industry average) may suggest general problems with debt collection or the financial position of major customers.

Credit taken / "Creditor Days"

((Trade creditors + accruals) / (cost of sales + other purchases)) x 365

A similar calculation to that for debtors, giving an insight into whether a business is taking full advantage of trade credit available to it.

1.6.3 LIQUIDITY RATIOS Liquidity ratios indicate how capable a business is of meeting its short-term obligations as they fall due.

Ratio Calculation Comments

Current Ratio Current Assets / Current Liabilities

A simple measure that estimates whether the business can pay debts due within one year from assets that it expects to turn into cash within that year. A ratio of less than one is often a cause for concern, particularly if it persists for any length of time.

Quick Ratio (or "Acid Test"

Cash and near cash assets (short-term investments + trade debtors)

Not all assets can be turned into cash quickly or easily. Some - notably raw materials and other stocks - must first be turned into final product, then sold and the cash collected from debtors. The quick ratio therefore adjusts the current ratio to eliminate all assets that are not already in cash (or "near-cash") form. Once again, a ratio of less than one would start to send out danger signals.

1.6.4 STABILITY RATIOS These ratios concentrate on the long-term health of a business - particularly the effect of the capital/finance structure on the business.

Ratio Calculation Comments

Gearing Borrowing (all long-term debts + normal

Gearing (otherwise known as "leverage") measures the proportion of assets invested in a

3.21

Page 609: 30510870 Cost Accounting and Financial Management

Financial Management

overdraft) / Net Assets (or Shareholders' Funds)

business that are financed by borrowing. In theory, the higher the level of borrowing (gearing) the higher are the risks to a business, since the payment of interest and repayment of debts are not "optional" in the same way as dividends. However, gearing can be a financially sound part of a business's capital structure particularly if the business has strong, predictable cash flows.

Interest cover Operating profit before interest / Interest

This measures the ability of the business to "service" its debt. Are profits sufficient to be able to pay interest and other finance costs?

1.6.5 INVESTOR RATIOS There are several ratios commonly used by investors to assess the performance of a business as an investment.

Ratio Calculation Comments Earnings per share ("EPS") Earnings (profits)

attributable to ordinary shareholders / Weighted average ordinary shares in issue during the year

EPS measures the overall profit generated for each share in existence over a particular period.

Price-Earnings Ratio ("P/E Ratio")

Market price of share / Earnings per share

At any time, the P/E ratio is an indication of how highly the market "rates" or "values" a business. A P/E ratio is best viewed in the context of a sector or market average to get a feel for relative value and stock market pricing.

Dividend Yield (Latest dividend per ordinary share / Current market price of share) x 100

This is known as the "payout ratio". It provides a guide as to the ability of a business to maintain a dividend payment. It also measures the proportion of earnings that are being retained by the business rather than distributed as dividends.

3.22

Page 610: 30510870 Cost Accounting and Financial Management

Financial Analysis and Planning

Illustration 3 In a meeting held at Solan towards the end of 2004, the Directors of M/s HPCL Ltd. have taken a decision to diversify. At present HPCL Ltd. sells all finished goods from its own warehouse. The company issued debentures on 01.01.2005 and purchased fixed assets on the same day. The purchase prices have remained stable during the concerned period. Following information is provided to you:

INCOME STATEMENTS 2004 (Rs.) 2005 (Rs.) Cash Sales 30,000 32,000 Credit Sales 2,70,000 3,00,000 3,42,000 3,74,000 Less: Cost of goods sold

2,36,000 2,98,000

Gross profit 64,000 76,000 Less: Expenses Warehousing 13,000 14,000 Transport 6,000 10,000 Administrative 19,000 19,000 Selling 11,000 14,000 Interest on Debenture 49,000 2,000 59,000Net Profit 15,000 17,000

BALANCE SHEET

2004 (Rs.) 2005 (Rs.) Fixed Assets (Net Block) - 30,000 - 40,000 Debtors 50,000 82,000 Cash at Bank 10,000 7,000 Stock 60,000 94,000 Total Current Assets (CA) 1,20,000 1,83,000 Creditors 50,000 76,000 Total Current Liabilities (CL) 50,000 76,000

3.23

Page 611: 30510870 Cost Accounting and Financial Management

Financial Management Working Capital (CA - CL) 70,000 1,07,000Total Assets 1,00,000 1,47,000Represented by: Share Capital 75,000 75,000 Reserve and Surplus 25,000 42,000 Debentures − 30,000 1,00,000 1,47,000 You are required to calculate the following ratios for the years 2004 and 2005. (i) Gross Profit Ratio (ii) Operating Expenses to Sales Ratio. (iii) Operating Profit Ratio (iv) Capital Turnover Ratio (v) Stock Turnover Ratio (vi) Net Profit to Net Worth Ratio, and (vii) Debtors Collection Period. Ratio relating to capital employed should be based on the capital at the end of the year. Give the reasons for change in the ratios for 2 years. Assume opening stock of Rs. 40,000 for the year 2004. Ignore Taxation. Solution

Computation of Ratios 1. Gross profit ratio 2004 2005 Gross profit/sales

000,00,3100000,64 ×

000,74,3100000,76 ×

21.3% 20.3 2. Operating expense to sales ratio Operating exp / Total sales

000,00,3100000,49 ×

000,74,3100000,57 ×

16.3% 15.2% 3. Operating profit ratio

3.24

Page 612: 30510870 Cost Accounting and Financial Management

Financial Analysis and Planning

Operating profit / Total sales 000,00,3

100000,15 × 000,74,3

100000,19 ×

5% 5.08% 4. Capital turnover ratio Sales / capital employed 3

000,00,1000,00,3

= 54.2000,47,1000,74,3

=

5. Stock turnover ratio COGS / Average stock

000,50000,36,2 =4.7

000,77000,98,2 =3.9

6. Net Profit to Networth Net profit / Networth

000,00,1100000,15 × =15%

000,17,1100000,17 × =14.5%

7. Debtors collection period Average debtors / Average daily sales (Refer to working note) 73.739

000,50 99.936

000,82

67.6 days 87.5 days Working note: Average daily sales = Credit sales / 365

365000,70,2

365000,42,3

Rs.739.73 Rs.936.99 Analysis: The decline in the Gross profit ratio could be either due to a reduction in the selling price or increase in the direct expenses (since the purchase price has remained the same). Similarly there is a decline in the ratio of Operating expenses to sales. However since operating expenses have little bearing with sales, a decline in this ratio cannot be necessarily be interpreted as an increase in operational efficiency. An indepth analysis reveals that the decline in the warehousing and the administrative expenses has been partly set off by an increase in the transport and the selling expenses. The operating profit ratio has remained the same in spite of a decline in the Gross profit margin ratio. In fact the company has not benefited at all in terms of operational performance because of the increased sales. The company has not been able to deploy its capital efficiently. This is indicated by a decline in the Capital turnover from 3 to 2.5 times. In case the capital turnover would have remained at 3 the company would have increased sales and profits by Rs 67,000 and Rs 3,350 respectively.

3.25

Page 613: 30510870 Cost Accounting and Financial Management

Financial Management The decline in the stock turnover ratio implies that the company has increased its investment in stock. Return on Networth has declined indicating that the additional capital employed has failed to increase the volume of sales proportionately. The increase in the Average collection period indicates that the company has become liberal in extending credit on sales. However, there is a corresponding increase in the current assets due to such a policy. It appears as if the decision to expand the business has not shown the desired results. Illustration 4 Following is the abridged Balance Sheet of Alpha Ltd. :-

Liabilities Rs. Assets Rs. Share Capital 1,00,000 Land and Buildings 80,000 Profit and Loss Account 17,000 Plant and Machineries 50,000 Current Liabilities 40,000 Less: Depreciation 15,000 35,000 1,15,000 Stock 21,000 Debtors 20,000 _______ Bank 1,000 42,000Total 1,57,000 Total 1,57,000

With the help of the additional information furnished below, you are required to prepare Trading and Profit & Loss Account and a Balance Sheet as at 31st March, 2005: (i) The company went in for reorganisation of capital structure, with share capital remaining

the same as follows: Share capital 50% Other Shareholders’ funds 15% 5% Debentures 10% Trade Creditors 25%

Debentures were issued on 1st April, interest being paid annually on 31st March. (ii) Land and Buildings remained unchanged. Additional plant and machinery has been

bought and a further Rs. 5,000 depreciation written off. (The total fixed assets then constituted 60% of total gross fixed and current assets.) (iii) Working capital ratio was 8 : 5. (iv) Quick assets ratio was 1 : 1.

3.26

Page 614: 30510870 Cost Accounting and Financial Management

Financial Analysis and Planning

(v) The debtors (four-fifth of the quick assets) to sales ratio revealed a credit period of 2 months. There were no cash sales.

(vi) Return on net worth was 10%. (vii) Gross profit was at the rate of 15% of selling price. (viii) Stock turnover was eight times for the year. Ignore Taxation. Solution Particulars % (Rs.) Share capital 50% 1,00,000 Other shareholders funds 15% 30,000 5% Debentures 10% 20,000 Trade creditors 25% 50,000 Total 100% 2,00,000 Land and Buildings Total liabilities = Total Assets Rs. 2,00,000 = Total Assets Fixed Assets = 60% of total gross fixed assets and current assets = Rs. 2,00,000×60/100 = Rs. 1,20,000

Calculation of additions to Plant & Machinery

Rs. Total fixed assets 1,20,000 Less: Land & Buildings 80,000 Plant and Machinery (after providing depreciation) 40,000 Depreciation on Machinery up to 31-3-2004 15,000 Add: Further depreciation 5,000 Total 20,000 Current assets = Total assets – Fixed assets = Rs. 2,00,000 – Rs. 1,20,000 = Rs. 80,000

3.27

Page 615: 30510870 Cost Accounting and Financial Management

Financial Management Calculation of stock Quick ratio:

= 1sliabilitie Current

stock assets Current=

= 150,000 .Rs

stock 80,000 Rs.=

Rs. 50,000 = Rs. 80,000 – Stock Stock = Rs. 80,000 - Rs. 50,000 = Rs. 30,000 Debtors = 4/5th of quick assets = (Rs. 80,000 – 30,000)× 4/5 = Rs. 40,000 Debtors turnover ratio

= 365Sales Credit

Debtors× = 60 days

= 365SalesCredit

12000,40×

× = 2 months

2 credit sales = 4,80,000 Credit sales = 4,80,000/2 = 2,40,000 Gross profit (15% of sales) Rs. 2,40,000×15/100 = Rs. 36,000 Return on networth (profit after tax) Net worth = Rs. 1,00,000 + Rs. 30,000 = Rs. 1,30,000 Net profit = Rs. 1,30,000×10/100 = Rs. 13,000 Debenture interest = Rs. 20,000×5/100 = Rs. 1,000

3.28

Page 616: 30510870 Cost Accounting and Financial Management

Financial Analysis and Planning

Projected profit and loss account for the year ended 31-3-2005

To cost of goods sold 2,04,000 By sales 2,40,000 To gross profit 36,000 ________ 2,40,000 2,40,000 To debenture interest 1,000 By gross profit 36,000 To administration and other expenses 22,000 To net profit 13,000 ______ 36,000 36,000

Projected Balance Sheet as at 31st March, 2005

Liabilities Rs. Assets Rs. Share capital 1,00,000 Fixed assets Profit and loss A/c 30,000 Land & buildings 80,000 (17,000+13,000) Plant & machinery 60,000 5% Debentures 20,000 Less: Depreciation 20,000 40,000 Current liabilities Current assets Stock 30,000 Trade creditors 50,000 Debtors 40,000 _______ Bank 10,000 80,000 2,00,000 2,00,000

Illustration 5 X Co. has made plans for the next year. It is estimated that the company will employ total assets of Rs. 8,00,000; 50 per cent of the assets being financed by borrowed capital at an interest cost of 8 per cent per year. The direct costs for the year are estimated at Rs. 4,80,000 and all other operating expenses are estimated at Rs. 80,000. the goods will be sold to customers at 150 per cent of the direct costs. Tax rate is assumed to be 50 per cent. You are required to calculate: (i) net profit margin; (ii) return on assets; (iii) asset turnover and (iv) return on owners’ equity.

3.29

Page 617: 30510870 Cost Accounting and Financial Management

Financial Management Solution The net profit is calculated as follows:

Rs. Rs. Sales (150% of Rs. 4,80,000) 7,20,000 Direct costs 4,80,000Gross profit 2,40,000 Operating expenses 80,000 Interest changes (8% of Rs. 4,00,000) 32,000 1,12,000Profit before taxes 1,28,000 Taxes (@ 50%) 64,000Net profit after taxes 64,000

(i) Net profit margin = Sales

taxes afterProfit = 000,20,7.Rs

000,64.Rs = 0.89 or 8.9%

Net profit margin = Sales

T)- (1 EBIT = 000,20,7

)5.1(000,60,1.Rs − = 0.111 or 11.1%

(ii) Return on assets = Assets

T)- (1 EBIT = 000,00,8

)5.1(000,60,1.Rs − = .10 or 10%

(iii) Asset turnover = AssetsSales =

000,00,8.Rs000,20,7.Rs = 0.09 times

(iv) Return on equity = equity Owners'

taxes safterprofit Net = 0Rs.8,00,00 of 50%

000,64.Rs

=0Rs.4,00,00

000,64.Rs = .16 or 16%

Illustration 6 The total sales (all credit) of a firm are Rs. 6,40,000. It has a gross profit margin of 15 per cent and a current ratio of 2.5. The firm’s current liabilities are Rs. 96,000; inventories Rs. 48,000 and cash Rs. 16,000. (a) Determine the average inventory to be carried by the firm, if an inventory turnover of 5 times is expected? (Assume a 360 day year). (b) Determine the average collection period if the opening balance of debtors is intended to be of Rs. 80,000? (Assume a 360 day year).

3.30

Page 618: 30510870 Cost Accounting and Financial Management

Financial Analysis and Planning

Solution

(a) inventory Average

sold goods ofCost turnoverInventory =

Since gross profit margin is 15 per cent, the cost of goods sold should be 85 per cent of the sales.

Cost of goods sold = .85 × Rs. 6,40,000 = Rs. 5,44,000.

Thus, 5 inventory Average

5,44,000 Rs. ==

1,08,000 Rs. 5

5,44,000 Rs. inventory Average ==

(b) 360 salesCredit debtors Average period collection Average ×=

Average debtors = 2

debtors) Closing debtors (Opening +

Closing balance of debtors is found as follows:

Rs. Rs. Current assets (2.5 of current liabilities) 2,40,000 Less: Inventories 48,000 Cash 16,000 64,000 ∴ Debtors 1,76,000

Average debtors = 2

80,000) Rs. 1,76,000 (Rs. +

Rs. 2,56,000 ÷2 = Rs. 1,28,000

days 72 360 6,40,000 Rs.1,28,000 Rs. period collection Average =×=

Illustration 7 Additional information: Profit (after tax at 35 per cent), Rs. 2,70,000; Depreciation, Rs. 60,000; Equity dividend paid, 20 per cent; Market price of equity shares, Rs. 40.

3.31

Page 619: 30510870 Cost Accounting and Financial Management

Financial Management You are required to compute the following, showing the necessary workings: (a) Dividend yield on the equity shares (b) Cover for the preference and equity dividends (c) Earnings per shares (d) Price-earnings ratio. Solution (a) Dividend yield on the equity shares

= cent per 5 100 40 Rs.

10) Rs. (0.20 2 Rs. 100 share per priceMarket

share per Dividend=×

×=×

(b) Dividend coverage ratio

(i) rsshareholde preference to payable Dividend

taxes afterProfit Preference =

times 10 3,00,000) Rs. (0.09 27,000 Rs.

2,70,000 Rs. =×

=

(ii) share per 2 Rs. of ratecurrent at rsshareholdeequity to payable Dividend

dividend share Preference taxes afterProfit Equity −=

times 1.52 2) Rs. shares (80,000 1,60,000 Rs.

27,000 Rs. 2,70,000 Rs. =×

−=

(c) Earnings per equity share

share per 3.04 Rs. 80,000

2,43,00 Rs. goutstandin sharesequity of Number

rsshareholdeequity to available Earnings===

(d) Price-earning (P/E) ratio = times 13.2 4.04 Rs.400 Rs.

share perEquity share per priceMarket ==

Self Examination Questions

A. Objective Type Questions 1. Which of the following assets is not a quick current asset for the purpose of calculating

acid test ratio? (a) Short term bills receivables

3.32

Page 620: 30510870 Cost Accounting and Financial Management

Financial Analysis and Planning

(b) Cash (c) Stock (d) Debtors less provision for bad and doubtful debts.

2. When the current ratio is 2 : 5, and the amount of current liabilities is Rs. 25,000, what is the amount of current assets? (a) Rs. 62,500 (b) Rs. 12,500 (c) Rs. 10,000 (d) None of these.

3. When quick ratio is 1.5 : 1 and the amount of quick assets Rs. 30,000, what is the amount of quick liabilities? (a) Rs. 20,000 (b) Rs. 50,000 (c) Rs. 45,000 (d) Rs. 30,000.

4. When opening stock is Rs. 50,000, closing stock Rs. 60,000, and cost of goods sold Rs. 2,20,000, the stock turnover ratio is (a) 2 times (b) 3 times (c) 4 times (d) 5 times.

5. When net sales for the year are Rs. 2,50,000 and debtors Rs. 50,000, the average collection period is: (a) 60 days (b) 45 days (c) 42 days (d) 72 days.

6. Dividing net sales by average debtors would yield (a) Acid test ratio (b) Return on sales ratio

3.33

Page 621: 30510870 Cost Accounting and Financial Management

Financial Management

(c) Debtors turnover ratio (d) None of these.

7. Given net profit Rs. 150,000, preference dividend Rs. 25,000, taxes Rs. 10,000 and number of equity shares 1,00,000. What is the Earning per Share (EPS)? (a) Rs. 1.50 (b) Rs. 1.25 (c) Rs. 1.15 (d) None of these.

8. When net profit is Rs. 2,25,000, taxes Rs. 25,000 and net worth Rs. 10,00,000 what is the rate of return on shareholders’ equity? (a) 22.5% (b) 20% (c) 25% (d) Cannot be calculated.

9. Accounting information given by a company: Total assets turnover 3 times Net Profit margin 10% Total assets Rs. 1,00,000

The net profit is: (a) Rs. 10,000 (b) Rs. 15,000 (c) Rs. 25,000 (d) Rs. 30,000.

10. Match the following: (1) Test of Liquidity A. ROI (2) Test of Profitability B. Debtors turnover (3) Test of Solvency C. Acid test ratio (4) Test of Activity D. Debt equity ratio

(1) (2) (3) (4)

3.34

Page 622: 30510870 Cost Accounting and Financial Management

Financial Analysis and Planning

(a) (A) (D) (B) (C) (b) (D) (A) (C) (B) (c) (B) (C) (A) (D) (d) (C) (A) (D) (B)

11. Which of the following liabilities are taken into account for acid test ratio? (1) Trade creditors (2) Bank overdraft (3) Bills payable (4) Outstanding expenses (5) Redeemable debentures. (a) 1, 2, 3, 4 and 5 (b) 1, 3 and 4 (c) 1, 2, 3 and 4 (d) 1, 3, 4 and 5.

12. ROI - Return on investment is equal to ................. (a) Net Profit after Tax / Tangible Net Worth (b) Net Profit after Tax / Net Tangible Assets (c) Net Profit after Tax / Paid up Capital (d) Gross Profit / Gross Assets.

13. Balance Sheet of a company indicates that its current ratio is 1.5. Company’s net working capital is Rs. one crore. The Current Assets would amount to (a) Rs. 3 crore (b) Rs.1.5 crore (c) Rs.4 crore (d) Rs. 2.5 crore.

14. Earnings after Interest and Tax is Rs.20 crore, interest is Rs.4 crore, Income Tax is Rs.16 crore. Interest Coverage Ratio would be (a) 10 (b) 9

3.35

Page 623: 30510870 Cost Accounting and Financial Management

Financial Management

(c) 7.5 (d) 5.

15. A firm's equity multiplier is an indication of its __________ position. (a) Liquidity (b) Debt (c) Asset utilization (d) Inventory.

Answers to Objective Type Questions 1. (c); 2. (c); 3. (a); 4. (c); 5. (d); 6. (c); 7. (c); 8. (b); 9. (d); 10. (d); 11. (b); 12. (b); 13. (a); 14. (a); 15. (b) B. Short Answer Type Questions 1. Interpret the liquidity conditions of a business in the following circumstances:

(i) High Current Ratio, High Quick Ratio; (ii) High Current Ratio, Low Quick Ratio; (iii) Low Current Ratio, High Quick Ratio; (iv) Low Current Ratio, Low Quick Ratio.

2. Interpret liquidity conditions of a business in the following circumstances: (i) Ratio Firm Industry Average

Current Ratio 1.7 1.6 Quick Ratio 1.2 0.8

(ii) Ratio Firm Industry Average

Current Ratio 1.2 1.5 Quick Ratio 0.8 0.6

(iii) Ratio Firm Industry Average Current Ratio 2.0 1.1 Quick Ratio 1.5 0.7

3.36

Page 624: 30510870 Cost Accounting and Financial Management

Financial Analysis and Planning

3. (i) High Current and Quick Ratios are accompanied by low absolute cash ratio in SUKA Ltd. What does it imply?

(ii) High Current ratio in POOJA Ltd. is accompanied by low quick and absolute cash ratios. What does it imply? Does it make any difference if current ratio also comes down?

4. What do you understand by the following terms: (a) Earnings per share (b) Dividend per share (c) Activity ratios (d) Leverage ratios (e) Return on Investment.

5. Write short notes on the following: (a) Price Earning ratio (b) Liquidity Ratios (c) Importance of financial analysis (d) Limitations of Financial ratios (e) Use of Financial ratios for Budgeting.

C. Long Answer Type Questions 1. What are the usually followed ratio categories for business data analysis? Are they

overlapping? Mention at least two financial ratios used in each category. 2. PUTA Ltd. maintains very low cash and bank balance whereas PUJA Ltd., its competitor,

maintains high cash balance. Which of the ratios do you use to interpret the cash position of the firms? What would be your interpretation?

3. Can you judge the liquidity of a business undertaking only from the Balance Sheet data? How do you interpret current ratio and quick ratio?

4. How do year-end adjustments affect the liquidity ratios? What precautions are necessary before making liquidity appraisals using current ratio and quick ratio?

5. What are the different ratio measures of profitability? How do you measure profitability of a diversified company?

6. Discuss briefly the need for debt-service coverage ratio. Does it provide sufficient information to the prospective lenders of a firm before entering into a loan agreement?

3.37

Page 625: 30510870 Cost Accounting and Financial Management

Financial Management 7. What are the various turnover ratios? Explain their significance. 8. ‘Increase in stock turnover and debtors’ turnover is not always good’. Do you agree?

Give reasons for your answer. Do you think that such increase causes decline in liquidity and profitability?

9. One way to improve the ROI is to improve the capital turnover ratio. If there is a decline in profitability ratio, how far is it possible to improve ROI by increasing capital turnover ratio?

D. Practical Problems 1. Consider the following cash position ratios.

Particulars AUTO KUTO SUTO Industry Ltd. Ltd. Ltd. Average Absolute cash ratio 0.20 0.25 0.40 0.35 Interval measure (days) 90 80 75 75 Interpret the results

(Hint : Average daily cash operating expenditure of AUTO Ltd. and KUTO Ltd. are relatively lower)

2. Given below are the profitability ratios of XZ Ltd. and the industry averages :

Ratios XZ Ltd. Industry Average Gross Profit (%) 35 32 Operating Profit (%) 27 26 ROI (%) 18 20 Comment on the ratios given above.

3. From the following information determine debt-service coverage ratio.

Debt : 10% Debentures Rs. 5,00,000 12% Term loan Rs. 1,00,000 13.1/2% Term loan Rs. 1,50,000 13% 2nd Debentures Rs. 2,00,000

Fund from operations : Rs. 25,000Amortizations : Preliminary Exp. Rs. 12,000Goodwill w/o Rs. 15,000

3.38

Page 626: 30510870 Cost Accounting and Financial Management

Financial Analysis and Planning

Depreciation Rs. 60,000Provision for tax Rs. 80,000Non-operating Income : Interest/Dividend from Investments Rs. 16,000Profit on sale of fixed assets Rs. 5,000Repayment due : 10% Debentures Rs. 2,00,000 13.1/2% Term loan Rs. 50,000

4. A firm’s average stock holding period is 90 days and average collection period is 60 days. It wants to relax the collection period by 15 days and increase the holding period by 10 days. Its credit sales and cost of goods sold were Rs. 40 lakhs and Rs. 30 lakhs respectively. How much extra working capital does the firm need for this change? Other things remaining same what would be the impact of this policy on ROI if such extra working capital was financed by long-term fund? Assume total sales were Rs. 45 lakhs [Take one year = 360 working days]. Amount (Rs. in lakhs) Fixed Assets 12,650 Inventories 1,250 Sundry Debtors 1,000 Cash and Bank 850 15,750 Less : Current Liabilities 1,500Long-term Capital Employed : 14,250ROI 15% Operating Profit Ratio 20% The company expects 25% increase in sales and 20% increase in operating profit. For this, it plans to relax collection period by 15 days. But it wants to maintain the present rate of inventory turnover and cash/current assets ratio. Assume that there will be no change in fixed assets. Current liabilities are expected to increase by 25%. Find the effect of such changes on ROI.

3.39

Page 627: 30510870 Cost Accounting and Financial Management

Financial Management 5. Given below are the Balance Sheets of PU Ltd. and QU Ltd. as on 31st March, 2006 :

Balance Sheet (Rs. ‘000)

Liabilities PU Ltd. QU Ltd. Assets PU Ltd. QU Ltd.

Share capital Gross Block Equity Shares of Less : Depreciation 812 917 Rs. 10 each 500 400 Investments 100 300 9-1/2% Pref. shares of Current Assets, Loans Rs. 10 each 100 50 and Advances : Reserve and Surplus Inventories 202 52 General Reserve 300 — Sundry Debtors 152 64 P&L A/c 100 50 Cash & Bank 42 32 Secured Loan Deposits 12 42 11% Term Loan 50 620 Advances — 40 10% Debentures 100 100 Unsecured Loan 15% Bank Loan 20 20 18% Short Term Loan 10 15 Current Liabilities & Provisions Sundry Creditors 10 10 Outstanding Expenses 5 2 Provision for Taxation 50 40 Proposed Dividend 75 140 1320 1447 1320 1447

Find the capital structure ratios of the companies. Comment on their overall capital structure. Both the companies are willing to raise 3.2 lakhs rupees by issue of debentures. How do you react if 2: 1 debt-equity ratio norm is to be followed?

3.40

Page 628: 30510870 Cost Accounting and Financial Management

Financial Analysis and Planning

6. Dakshinamurthy Ltd. (In short DAK Ltd.) gives you the following information : (Rs. in lakhs) Sales (75% on credit) 40 Purchases (80% on credit) 16 Cost of production : Material consumed 12 Wages and salaries for production 8 Manufacturing expenses 4 Finished goods— Opening Stock 2 — Completed during the year, 10,000 units — Sold during the year 9,000 units of goods finished during the year and 90% of the

opening stock. Opening Debtors 4 Closing Debtors 2.5 Opening Creditors 1.5 Closing Creditors 2.0 You are asked to find out: (i) Inventory (finished goods) turnover ratio (ii) Average collection and payment periods. Industry average inventory turnover ratio was 8.5, debtors’ turnover was 10 and creditors’ turnover was 6. Interpret the results.

3.41

Page 629: 30510870 Cost Accounting and Financial Management

Financial Management UNIT – II : CASH FLOW ANALYSIS

Learning Objectives After studying this unit you will be able to

♦ Understand the meaning of cash flow statement;

♦ Understand difference between cash flow statement and funds flow statement, their utility and limitations;

♦ Identify the sources and application of cash;

♦ Understand the salient features of AS-3 (Revised); and

♦ Prepare a cash flow statement.

2.1 INTRODUCTION A cash flow statement is a statement which discloses the changes in cash position between the two periods. For example, a balance sheet, shows the balance of cash as on 31.3.2005 at Rs.30,000/- while the cash balance as per its latest balance sheet as on 31.3.2006 was Rs.40,000/-. Thus, there has been an inflow of Rs.10,000/- during a year’s period. The cash flow statement outlines the reasons for such inflows or outflows of cash. The cash flow statement is an important planning tool in the hands of management. This helps the management in formulating plans for immediate future cash needs. A projected cash flow statement or a Cash Budget will help the management in estimating as to how much cash will be available at a particular point of time to meet obligations like payment to trade creditors, repayment of cash loans, dividends, etc. A proper planning of the cash resources will enable the management to make available sufficient cash whenever needed and invest surplus cash, if any in productive and profitable opportunities. The term cash comprises cash on hand, demand deposits with the banks and includes cash equivalents. Due to various limitations of Funds flow statements, the cash flow statement has gained prominence and is used by the management as an important tool of financial analysis, planning and management.

2.2 UTILITY OF CASH FLOW ANALYSIS A cash flow statement is useful for short-term planning. A business enterprise needs sufficient cash to meet its various obligations in the near future such as payment for purchase of fixed assets, payment of debts maturing in the near future, expenses of the business, etc. A historical analysis of the different sources and applications of cash will enable the

3.42

Page 630: 30510870 Cost Accounting and Financial Management

Financial Analysis and Planning

management to make reliable cash flow projections for the immediate future. It may then plan out for investment of surplus or meeting the deficit, if any. Thus, a cash flow analysis is an important financial tool for the management. Its chief advantages are as follows: ♦ Helps in efficient cash management. ♦ Helps in internal financial management. ♦ Discloses the movements of cash. ♦ Discloses the success or failure of cash planning.

2.3 LIMITATIONS OF CASH FLOW ANALYSIS Cash flow analysis is a useful tool of financial analysis. However, it has its own limitations. These limitations are as under: 1. Cash flow statement cannot be equated with the Income Statement. An Income Statement takes into account both cash as well as non-cash items and, therefore, net cash flow does not necessarily mean net income of the business. 2. The cash balance as disclosed by the cash flow statement may not represent the real liquid position of the business since it can be easily influenced by postponing purchases and other payments. 3. Cash flow statement cannot replace the Income Statement or the Funds Flow Statement. Each of them has a separate function to perform. In spite of these limitations it can be said that cash flow statement is a useful supplementary instrument. It discloses the volume as well as the speed at which the cash flows in the different segments of the business. This helps the management in knowing the amount of capital tied up in a particular segment of the business. The technique of cash flow analysis, when used in conjunction with ratio analysis, serves as a barometer in measuring the profitability and financial position of the business. The cash flow statement is prepared in accordance with the provisions contended in AS-3 (Revised) issued by the Council of the Institute of Chartered Accountants of India. Students are advised to read the standard thoroughly to learn various intricacies relating to preparation of cash flow statement. The AS-3 (Revised) while laying down its objectives says that information about the cash flows of an enterprise is useful in providing users of financial statements with a basis to assess the ability of the enterprise to generate cash and cash equivalents and the needs of the enterprise to utilize those cash flows. The economic decisions that are taken by users require an evaluation of the ability of an enterprise to generate cash and cash equivalents and the timing and certainty of their generation.

3.43

Page 631: 30510870 Cost Accounting and Financial Management

Financial Management The Statement deals with the provision of information about the historical changes in cash and cash equivalents of an enterprise by means of a cash flow statement which classifies cash flows during the period from operating, investing and financing activities.

2.4 BENEFITS OF CASH FLOW INFORMATION A cash flow statement, when used in conjunction with the other financial statements, provides information that enables users to evaluate the changes in net assets of an enterprise, its financial structure (including its liquidity and solvency) and its ability to affect the amounts and timing of cash flows in order to adapt to changing circumstances and opportunities. Cash flow information is useful in assessing the ability of the enterprise to generate cash and cash equivalents and enables users to develop models to assess and compare the present value of the future cash flows of different enterprises. It also enhances the comparability of the reporting of operating performance by different enterprises because it eliminates the effects of using different accounting treatments for the same transactions and events. Historical cash flow information is often used as an indicator of the amount, timing and certainty of future cash flows. It is also useful in checking the accuracy of past assessments of future cash flows and in examining the relationship between profitability and net cash flow and the impact of changing prices.

2.5 DEFINITIONS AS-3 (Revised) has defined the following terms as follows: (a) Cash comprises cash on hand and demand deposits with banks. (b) Cash equivalents are short term highly liquid investments that are readily convertible into known amounts of cash and which are subject to an insignificant risk of changes in value. (c) Cash flows are inflows and outflows of cash and cash equivalents. (d) Operating activities are the principal revenue-producing activities of the enterprise and other activities that are not investing or financing activities. Investing activities are the acquisition and disposal of long-term assets and other investments not included in cash equivalents. (e) Financing activities are activities that result in changes in the size and composition of the owners’ capital (including preference share capital in the case of a company) and borrowings of the enterprise.

2.6 CASH AND CASH EQUIVALENTS Cash equivalents are held for the purpose of meeting short-term cash commitments rather than for investment or other purposes. For an investment to qualify as a cash equivalent, it

3.44

Page 632: 30510870 Cost Accounting and Financial Management

Financial Analysis and Planning

must be readily convertible to a known amount of cash and be subject to an insignificant risk of changes in value. Therefore, an investment normally qualifies as a cash equivalent only when it has a short maturity of say, three months or less from the date of acquisition. Investments in shares are excluded from cash equivalents unless they are, in substance, cash equivalents; for example, preference shares of a company acquired shortly before their specified redemption date (provided there is only an insignificant risk of failure of the company to repay the amount at maturity). Cash flows exclude movements between items that constitute cash or cash equivalent because these components are part of the cash management of an enterprise rather than part of its operating, investing and financing activities. Cash management includes the investment of excess cash in cash equivalents.

2.7 PRESENTATION OF CASH FLOW STATEMENT The cash flow statement should report cash flows during the period classified by operating, investing and financing activities. An enterprise presents its cash flows from operating, investment and financing activities in a manner which is most appropriate to its business. Classification by activity provides information that allows users to assess the impact of those activities on the financial position of the enterprise and the amount of its cash and cash equivalents. This information may also be used to evaluate the relationships among those activities. A single transaction may include cash flows that are classified differently. For example, when the instalment paid in respect of a fixed asset acquired on deferred payment basis includes both interest and loan, the interest element is classified under financing activities and the loan element is classified under investing activities. 2.7.1 Operating Activities The amount of cash flows arising from operating activities is a key indicator of the extent to which the operations of the enterprise have generated sufficient cash flows to maintain the operating capability of the enterprise, pay dividends, repay loans and make new investments without recourse to external sources of financing. Information about the specific components of historical operating cash flows is useful, in conjunction with other information, in forecasting future operating cash flows. Cash flows from operating activities are primarily derived from the principal revenue-producing activities of the enterprise. Therefore, they generally result from the transactions and other events that enter into the determination of net profit or loss. Examples of cash flows from operating activities are: (a) Cash receipts from the sale of goods and the rendering of services;

3.45

Page 633: 30510870 Cost Accounting and Financial Management

Financial Management (b) Cash receipts from royalties, fees, commissions and other revenue; (c) Cash payments to suppliers for goods and services; (d) Cash payments to and on behalf of employees; (e) Cash receipts and cash payments of an insurance enterprise for premiums and claims,

annuities and other policy benefits; (f) Cash payments or refunds of income taxes unless they can be specifically identified with

financing and investing activities; and (g) Cash receipts and payments relating to futures contracts, forward contracts, option

contracts and swap contracts when the contracts are held for dealing or trading purposes.

Some transactions, such as the sale of an item of plant, may give rise to a gain or loss which is included in the determination of net profit or loss. However, the cash flows relating to such transactions are cash flows from investing activities. An enterprise may hold securities and loans for dealing or trading purposes, in which case they are similar to inventory acquired specifically for resale. Therefore, cash flows arising from the purchase and sale of dealing or trading securities are classified as operating activities. Similarly, cash advances and loans made by financial enterprises are usually classified as operating activities since they relate to the main revenue-producing activity of that enterprise.

2.7.2 Investing Activities The separate disclosure of cash flows arising from investing activities is important because the cash flows represent the extent to which expenditures have been made for resources intended to generate future income and cash flows. Examples of cash flows arising from investing activities are: (a) Cash payments to acquire fixed assets (including intangibles). These payments include those relating to capitalized research and development costs and self-constructed fixed assets; (b) Cash receipts from disposal of fixed assets (including intangibles); (c) Cash payments to acquire shares, warrants or debt instruments of other enterprises and interests in joint ventures (other than payments for those instruments considered to be cash equivalents and those held for dealing or trading purposes); (d) Cash receipts from disposal of shares, warrants or debt instruments of other enterprises and interests in joint ventures (other than receipts from those instruments considered to be cash equivalents and those held for dealing or trading purposes);

3.46

Page 634: 30510870 Cost Accounting and Financial Management

Financial Analysis and Planning

(e) Cash advances and loans made to third parties (other than advances and loans made by a financial enterprise); (f) Cash receipts from the repayment of advances and loans made to third parties (other than advances and loans of a financial enterprise); (g) Cash payments for futures contracts, forward contracts, option contracts and swap contracts except when the contracts are held for dealing or trading purposes, or the payments are classified as financing activities; and (h) Cash receipts from futures contracts, forward contracts, option contacts and swap contracts except when the contracts are held for dealing or trading purposes, or the receipts are classified as financing activities. When a contract is accounted for as a hedge of an identifiable position, the cash flows of the contract are classified in the same manner as the cash flows of the position being hedged. 2.7.3 Financing Activities The separate disclosure of cash flows arising from financing activities is important because it is useful in predicting claims on future cash flows by providers of funds (both capital and borrowings) to the enterprise. Examples of cash flows arising from financing activities are: (a) Cash proceeds from issuing shares or other similar instruments; (b) Cash proceeds from issuing debentures, loans, notes, bonds and other short or long-term borrowings; and (c) Cash repayments of amounts borrowed. In addition to the general classification of three types of cash flows, AS-3 (Revised) provides for the treatment of the cash flows of certain special items as under: Foreign Currency Cash Flows Cash flows arising from transactions in a foreign currency should be recorded in an enterprises reporting currency. The reporting should be done by applying the exchange rate at the date of cash flow statement. A rate which approximates the actual rate may also be used. For example, weighted average exchange rate for a period may be used for recording foreign currency transactions. The effect of changes in exchange rates on cash and cash equivalents held in foreign currency should be reported as a separate part in the form of reconciliation in order to reconcile cash and cash equivalents at the beginning and end of the period.

3.47

Page 635: 30510870 Cost Accounting and Financial Management

Financial Management Unrealised gains and losses arising from changes in foreign exchange rates are not cash flows. The difference of amount raised due to changes in exchange rate should not be included in operating investing and financing activities. This shall be shown separately in the reconciliation statement. 2.7.4 Extraordinary Items Any cash flows relating to extraordinary items should as far as possible classify them into operating, investing or financing activities and those items should be separately disclosed in the cash flow statement. Some of the examples for extraordinary items is bad debts recovered, claims from insurance companies, winning of a law suit or lottery etc. The above disclosure is in addition to disclosure mentioned in AS-5, ‘Net Profit or Loss for the period, prior period items and changes in accounting policies.’

2.7.5 Interest and Dividends Cash flows from interest and dividends received and paid should each be disclosed separately. The treatment of interest and dividends, received and paid, depends upon the nature of the enterprise i.e., financial enterprises and other enterprises. In case of financial enterprises, cash flows arising from interest paid and interest & Dividends received, should be classified as cash flows from operating activities. In case of other enterprises Cash outflows arising from interest paid on terms loans and debentures should be classified as cash outflows from financing activities. Cash outflows arising from interest paid on working capital loans should be classified as cash outflow from operating activities. Interest and dividends received should be classified as cash inflow from investing activities. Interest and dividends received should be classified as cash inflow from investing activities. Dividend paid on equity and preference share capital should be classified as cash outflow from financing activities. Taxes on Income Cash flows arising from taxes on income should be separately disclosed. It should be classified as cash flows from operating activities unless they can be specifically identified with financing and investing activities.

3.48

Page 636: 30510870 Cost Accounting and Financial Management

Financial Analysis and Planning

When tax cash flows are allocated over more than one class of activity, the total amount of taxes paid is disclosed. 2.7.6 Investments in Subsidiaries, Associates and Joint Ventures Any such investments should be reported in the cash flow statement as investing activity. Any dividends received should also be reported as cash flow from investing activity. 2.7.7 Non-Cash Transactions Investing and financing transactions that do not require the use of cash or cash equivalents should be excluded from a cash flow statement. Such transactions should be disclosed elsewhere in the financial statements in a way that provides all the relevant information about these investing and financing activities. The exclusion of non-cash transactions from the cash flow statement is consistent with the objective of a cash flow statement as these do not involve cash flows in the current period. Examples of non-cash transactions: (a) The acquisition of assets by assuming directly related liabilities. (b) The acquisition of an enterprise by means of issue of shares. (c) Conversion of debt into equity.

2.8 PROCEDURE IN PREPARATION OF CASH FLOW STATEMENT The procedure used for the preparation of cash flow statement is as follows: Calculation of net increase or decrease in cash and cash equivalents accounts: The difference between cash and cash equivalents for the period may be computed by comparing these accounts given in the comparative balance sheets. The results will be cash receipts and payments during the period responsible for the increase or decrease in cash and cash equivalent items. Calculation of the net cash provided or used by operating activities: It is by the analysis of Profit and Loss Account, Comparative Balance Sheet and selected additional information. Calculation of the net cash provided or used by investing and financing activities: All other changes in the Balance sheet items must be analysed taking into account the additional information and effect on cash may be grouped under the investing and financing activities. Preparation of a Cash Flow Statement: It may be prepared by classifying all cash inflows and outflows in terms of operating, investing and financing activities. The net cash flow provided or used in each of these three activities may be highlighted. Ensure that the aggregate of net cash flows from operating, investing and financing activities is equal to net increase or decrease in cash and cash equivalents.

3.49

Page 637: 30510870 Cost Accounting and Financial Management

Financial Management Report any significant investing financing transactions that did not involve cash or cash equivalents in a separate schedule to the Cash Flow Statement. 2.8.1 Reporting of Cash Flow from Operating Activities The purpose for determining the net cash from operating activities is to understand why net profit/loss as reported in the Profit and Loss account must be converted. The financial statements are generally prepared on accrual basis of accounting under which the net income will not indicate the net cash provided by or net loss will not indicate the net cash used in operating activities. In order to calculate the net cash flows in operating activities, it is necessary to replace revenues and expenses with actual receipts and payments in cash. This is done by eliminating the non-cash revenues and/non-cash expenses from the given earned revenues and incurred expenses. There are two methods of converting net profit into net cash flows from operating activities- (i) Direct method, and (ii) Indirect method. (i) Direct Method: Under direct method, cash receipts from operating revenues and cash payments for operating expenses are arranged and presented in the cash flow statement. The difference between cash receipts and cash payments is the net cash flow from operating activities. It is in effect a cash basis Profit and Loss account. In this case, each cash transaction is analysed separately and the total cash receipts and payments for the period is determined. The summarized data for revenue and expenses can be obtained from the financial statements and additional information. We may convert accrual basis of revenue and expenses to equivalent cash receipts and payments. Make sure that a uniform procedure is adopted for converting accrual base items to cash base items. Under direct method, items like depreciation, amortisation of intangible assets, preliminary expenses, debenture discount, etc. are ignored from cash flow statement since the direct method includes only cash transactions and non-cash items are omitted. Likewise, no adjustment is made for loss or gain on the sale of fixed assets and investments. (ii) Indirect Method: In this method the net profit (loss) is used as the base and converts it to net cash provided or used in operating activities. The indirect method adjusts net profit for items that affected net profit but did not affect cash. Non-cash and non-operating charges in the Profit and Loss account are added back to the net profit while non-cash and non-operating credits are deducted to calculate operating profit before working capital changes. It is a partial conversion of accrual basis profit to cash basis profit. Necessary adjustments are made for increase or decrease in current assets and current liabilities to obtain net cash from operating activities.

3.50

Page 638: 30510870 Cost Accounting and Financial Management

Financial Analysis and Planning

2.8.2 Other Disclosure Requirements If any significant cash and cash equivalent balances held by the enterprise are not available for use by it, it should be disclosed in the cash flow statement. For example cash held by the overseas branch which is not available for use by the enterprise due to exchange control regulations or due to other legal restrictions. Any additional information to understand the financial position and liquidity position of an enterprise should be disclosed. For example: The amount of undrawn borrowing facilities that may be available for future operating activities and to settlement of capital commitments, indicating any restrictions on the use of these facilities; and The aggregate amount of cash flows that represent increases in operating capacity separately from those cash flows that are required to maintain operating capacity. A reconciliation of cash and cash equivalents given in its cash flow statement with equivalent items reported in the Balance Sheet. An enterprise should disclose the policy which it adopts in determining the composition of cash and cash equivalent. The effect of any change in the policy for determining components of cash and cash equivalents should be reported in accordance with AS-5, ‘Net Profit or Loss for the period, Prior period items, and Changes in accounting policies’. 2.8.3 Format of Cash Flow Statement AS 3 (Revised) has not provided any specific format for the preparation of cash flow statements, but a general idea can be had from the illustration given in the appendix to the Accounting Standard. There seems to be flexibility in the presentation of cash flow statements. However, a widely accepted format under direct method and indirect method is given below:

Cash Flow Statement (Direct Method) Rs. Cash Flow from Operating Activities Cash receipts from customers xxx Cash paid to suppliers and employees (xxx) Cash generated from operations xxx Income tax paid (xxx) Cash flow before extraordinary items xxx Proceeds from earthquake disaster settlement etc xxx

3.51

Page 639: 30510870 Cost Accounting and Financial Management

Financial Management Net cash from Operating Activities (a) xxx Cash Flows from Investing Activities Purchase of fixed assets (xxx) Proceeds from sale of equipment xxx Interest received xxx Dividend received xxx Net cash from investing Activities (b) xxx Cash Flows from Financing Activities Rs. Proceeds from issuance of share capital xxx Proceeds from long-term borrowings xxx Repayments of long-term borrowings (xxx) Interest paid (xxx) Dividend paid (xxx) Net cash from Financing Activities (c) xxx Net increase (decrease) in Cash and Cash Equivalent (a+b+c) xxx Cash and Cash Equivalents at beginning of period xxx Cash and Cash Equivalent at end of period xxx Cash Flow Statement (Indirect Method) (Rs.) Cash Flow from Operating Activities Net profit before tax and extraordinary items xxx Adjustments for: - Depreciation xxx - Foreign exchange xxx - Investments xxx - Gain or loss on sale of fixed assets (xxx) - Interest/dividend xxx Operating profit before working capital changes xxx Adjustments for: - Trade and other receivables xxx

3.52

Page 640: 30510870 Cost Accounting and Financial Management

Financial Analysis and Planning

- Inventories (xxx) - Trade payable xxx Cash generation from operations xxx - Interest paid (xxx) - Direct Taxes (xxx) Cash before extraordinary items xxx Deferred revenue xxx Net cash from Operating Activities (a) xxx Cash Flow from Investing Activities Purchase of fixed assets (xxx) Sale of fixed assets xxx Purchase of investments xxx Interest received (xxx) Dividend received xxx Loans to subsidiaries xxx Net cash from Investing Activities (b) xxx Cash Flow from Financing Activities Proceeds from issue of share capital xxx Proceeds from long term borrowings xxx Repayment to finance/lease liabilities (xxx) Dividend paid (xxx) Net cash from Financing Activities (c) xxx Net increase (decrease) in Cash and Cash Equivalents (a+b+c) xxx Cash and Cash Equivalents at the beginning of the year xxx Cash and Cash Equivalents at the end of the year xxx

3.53

Page 641: 30510870 Cost Accounting and Financial Management

Financial Management Cash from Operations (Rs.)

Funds from Operations xxx Add: Increase in Current Liabilities (excluding Bank Overdraft) xxx Decrease in Current Assets (excluding cash & bank

balance) xxx xxx

Less: Increase in Current Assets (excluding cash & bank balance)

xxx

Decrease in Current Liabilities

(excluding bank overdraft) xxx xxx

Cash from Operations xxx 2.9 FUNDS FLOW STATEMENT VERSUS CASH FLOW STATEMENT Both funds flow and cash flow statements are used in analysis of past transactions of a business firm. The difference between these two statements is given below: Funds flow statement is based on the accrual accounting system. In case of preparation of cash flow statements all transactions effecting the cash or cash equivalents only is taken into consideration. Funds flow statement analyses the sources and application of funds of long-term nature and the net increase or decrease in long-term funds will be reflected on the working capital of the firm. The cash flow statement will only consider the increase or decrease in current assets and current liabilities in calculating the cash flow of funds from operations. Funds Flow analysis is more useful for long range financial planning. Cash flow analysis is more useful for identifying and correcting the current liquidity problems of the firm. Funds flow statement tallies the funds generated from various sources with various uses to which they are put. Cash flow statement starts with the opening balance of cash and reach to the closing balance of cash by proceeding through sources and uses. The concept and technique of preparing a Cash Flow Statement will be clear with the help of illustrations given in the following pages. Illustration 1: From the following information prepare a Cash Flow Statement according to (a) Direct Method (b) Indirect Method as per AS-3 (Revised). Working notes would from part of your answer

3.54

Page 642: 30510870 Cost Accounting and Financial Management

Financial Analysis and Planning

(1) BALANCE SHEET AS ON 31.12. 2005

(Rs. in ‘000)

2005 2004 Assets Cash on hand and balances with banks 200 25 Short-term investments 670 135 Sundry debtors 1,700 1,200 Interest receivable 100 -- Inventories 900 1,950 Long-term investments 2,500 2,500 Fixed assets at cost 2,180 1,910 Less: Accumulated depreciation (1,450) (1,060) Fixed assets (net) 730 850 Total Assets 6,800 6,660 2005 2004 Liabilities Sundry creditors 150 1,890 Interest payable 230 100 Income taxes payable 400 1,000 Long-term debt 1,110 1,040 Total liabilities 1,890 4,030 Shareholders’ funds Share capital 1,500 1,250 Reserves 3,410 1,380 Total shareholders’ funds 4,910 2,630 Total Liabilities and Shareholders’ funds 6,800 6,660

3.55

Page 643: 30510870 Cost Accounting and Financial Management

Financial Management (2) STATEMENT OF PROFIT AND LOSS for the period ended 31.12. 2005

(Rs. in ‘000)

Sales 30,650 Cost of sales (26,000) Gross profit 4,650 Depreciation (450) Administrative and selling expenses (910) Interest expense (400) Interest income 300 Dividend income 200 Foreign exchange loss (40) Net profit before taxation and extraordinary item 3,350 Extraordinary item- Insurance proceeds from earthquake disaster settlement 180 Net profit after extraordinary item 3,530 Income tax (300) Net Profit 3,230 Additional Information: (Figures in Rs. ‘000). (a) An amount of 250 was raised from the issue of share capital and a further 250 was raised from long-term borrowings. (b) Interest expense was 400 of which 170 was paid during the period. 100 relating to interest expense of the prior period was also paid during the period. (c) Dividends paid were 1,200. (d) Tax deducted at source on dividends received (included in the tax expense of 300 for the year) amounted to 40. (e) During the period, the enterprise acquired fixed assets for 350. The payment was made in cash. (f) Plant with original cost of 80 and accumulated depreciation of 60 was sold for 20. (g) Foreign exchange loss of 40 represents the reduction in the carrying amount of a short-

3.56

Page 644: 30510870 Cost Accounting and Financial Management

Financial Analysis and Planning

term investment in foreign currency designated bonds arising out of a change in exchange rate between the date of acquisition of the investment and the balance sheet date. (h) Sundry debtors and sundry creditors include amounts relating to credit sales and credit purchases only. Solution

CASH FLOW STATEMENT (Direct Method)

(Rs. in ‘000) 2005 Cash flows from operating activities Cash receipts from customers 30,150 Cash paid to suppliers and employees (27,600) Cash generated from operations 2,550 Income taxes paid (860) Cash flow before extraordinary item 1,690 Proceeds from earthquake disaster settlement 180 Net cash from operating activities 1,870 Cash flows from investing activities Purchase of fixed assets (350) Proceeds from sale of equipment 20 Interest received 200 Dividend received 160 Net cash from investing activities 30 Cash Flows from financing activities Proceeds from issuance of share capital 250 Proceeds from long-term borrowings 250 Repayments of long-term borrowings (180) Interest paid (270) Dividend paid (1,200) Net cash used in financing activities (1,150) Net increase in cash and cash equivalents 750 Cash and cash equivalents at beginning of period (See Note 1) 160 Cash and cash equivalents at end of period (See Note 1) 910 Notes to the Cash Flow Statement (Direct & Indirect Method)

3.57

Page 645: 30510870 Cost Accounting and Financial Management

Financial Management 1. Cash and cash equivalents: Cash and cash equivalents consist of cash on hand and balances with banks, and investments in money-market instruments. Cash and cash equivalents included in the cash flow statement comprise the following balance sheet amounts.

2005 2004

Cash on hand and balances with banks 200 25 Short-term investments 670 135

Cash and cash equivalents 870 160 Effects of exchange rate changes 40 --

Cash and cash equivalents as restated 910 160 Cash and cash equivalents at the end of the period include deposits with banks of 100 held by a branch which are not freely permissible to the company because of currency exchange restrictions. The company has undrawn borrowing facilities of 2,000 of which 700 may be used only for future expansion. 2. Total tax paid during the year (including tax deducted at source on dividends received) amounted to 900.

CASH FLOW STATEMENT (Indirect Method)

(Rs. in ‘000)

2005 Cash flows from operating activities Net profit before taxation, and extraordinary item 3,350 Adjustments for: Depreciation 450 Foreign exchange loss 40 Interest income (300) Dividend income (200) Interest expense 400

3.58

Page 646: 30510870 Cost Accounting and Financial Management

Financial Analysis and Planning

Operating profit before working capital changes 3,740 Increase in sundry debtors (500) Decrease in inventories 1,050 Decrease in sundry creditors (1,740) Cash generated from operations 2,550 Income taxes paid (860) Cash flows before extraordinary item 1,690 Proceeds from earthquake disaster settlement 180 Net cash from operating activities 1,870 Cash flows from investing activities Purchase of fixed assets (350) Proceeds from sale of equipment 20 Interest received 200 Dividends received 160 Net cash from investing activities 30 Cash flows from financing activities Proceeds from issuance of share capital 250 Proceeds from long-term borrowings 250 Repayment of long-term borrowings (180) Interest paid (270) Dividends paid (1,200) Net cash used in financing activities (1,150) Net increase in cash and cash equivalents 750 Cash and cash equivalents at beginning of period (See Note 1)

160

Cash and cash equivalents at end of period (See Note 1) 910

3.59

Page 647: 30510870 Cost Accounting and Financial Management

Financial Management Alternative Presentation (Indirect Method) As an alternative, in an indirect method cash flow statement, operating profit before working capital changes is sometimes presented as follows: Revenues excluding investment income 30,650 Operating expenses excluding depreciation (26,910) Operating profit before working capital changes 3,740 Working Notes: The working notes given below do not form part of the cash flow statement. The purpose of these working notes is merely to assist in understanding the manner in which various figures in the cash flow statement have been derived. (Figures are in Rs.’000).

1. Cash receipts from customers Sales 30,650 Add: Sundry debtors at the beginning of the year 1,200 31,850 Less: Sundry debtors at the end of the year 1,700 30,150 2. Cash paid to suppliers and employees Cost of sales 26,000 Administrative & selling expenses 910 26,910 Add: Sundry creditors at the beginning of the year 1,890 Inventories at the end of the year 900 2,790 29,700 Less: Sundry creditors at the end of the year 150 Inventories at the beginning of the year 1,950 2,100 27,600

3.60

Page 648: 30510870 Cost Accounting and Financial Management

Financial Analysis and Planning

3. Income taxes paid (including tax deducted at source from dividends received)

Income tax expense for the year (including tax deducted at source from dividends received

300

Add: Income tax liability at the beginning of the year 1,000 1,300 Less: Income tax liability at the end of the year 400 900 Out of 900, tax deducted at source on dividends received

(amounting to 40), is included in cash flows from investing activities and the balance of 860 is included in cash flows from operating activities.

4. Repayment of long-term borrowings Long-term debt at the beginning of the year 1,040 Add: Long-term borrowings made during the year 250 1,290 Less: Long-term borrowings at the end of the year 1,110 180 5. Interest paid Interest expense for the year 400 Add: Interest payable at the beginning of the year 100 500 Less: Interest payable at the end of the year 230 270

3.61

Page 649: 30510870 Cost Accounting and Financial Management

Financial Management Illustration 2: Swastik Oils Ltd. has furnished the following information for the year ended 31st March, 2006:

(Rs. in lakhs) Net profit 37,500.00 Dividend (including interim dividend paid) 12,000.00 Provision for income-tax 7,500.00 Income-tax paid during the year 6,372.00 Loss on sale of assets (net) 60.00 Book value of assets sold 277.50 Depreciation charged to P&L Account 30,000.00 Profit on sale of investments 150.00 Interest income on investments 41,647.50 Value of investments sold 3,759.00 Interest expenses 15,000.00 Interest paid during the year 15,780.00 Increase in working capital (excluding cash and bank balance) 84,112.50 Purchase of fixed assets 21,840.00 Investments on joint venture 5,775.00 Expenditure on construction work-in-progress 69,480.00 Proceeds from long-term borrowings 38,970.00 Proceeds from short-term borrowings 30,862.50 Opening cash and bank balances 11,032.50 Closing cash and bank balances 2,569.50

You are required to prepare the cash flow statement in accordance with AS-3 for the year ended 31st March, 2006. (Make assumptions wherever necessary).

3.62

Page 650: 30510870 Cost Accounting and Financial Management

Financial Analysis and Planning

Solution SWASTIK OILS LIMITED

Cash Flow Statement for the Year Ended 31st March, 2006

Cash Flows from Operating Activities (Rs. in lakhs) Net profit before taxation (37,500 + 7,500) 45,000.00 Adjustment for: Depreciation charged to P&L A/c 30,000.00 Loss on sale of assets (net) 60.00 Profit on sale of investments (150.00) Interest income on investments (3,759.00) Interest expenses 15,000.00 Operating profit before working capital changes 86,151.00 Increase (change) in working capital (excluding cash and bank

balance) (84,112.50)

Cash generated from operations 2,038.50 Income tax paid (6,372.00) Net cash used in operating activities (A) (4,333.50) (b) Cash Flow from investing Activities Sale of Assets (277.50-60.00) 217.50 Sale of Investments (41,647.50+150) 41,797.50 Interest Income on investments (assumed) 3,759.00 Purchase of fixed assets (21,840.00) Investments in Joint Venture (5,775.00) Expenditure on construction work-in-progress (69,480.00) Net Cash used in investing activities (B) (51,321.00) (c) Cash Flow from Financing Activities Proceeds from long-term borrowings 38,970.00 Proceeds from short-term borrowings 30,862.50 Interest paid (15,780.00) Dividends (including interim dividend paid) (12,000.00)

3.63

Page 651: 30510870 Cost Accounting and Financial Management

Financial Management Net cash from financing activities (C) 42,052.50 Net increase in cash and cash equivalents (A) + (B) + (C) (13,602.00) Cash and cash equivalents at the beginning of the year 11,032.50 Cash and cash equivalents at the end of the year 2,569.50 Self Examination Questions A. Objective Type Questions 1. The term cash includes

(a) Cash and Bank Balances

(b) All the Current Assets

(c) All the Current Liabilities

(d) None of the above. 2. “Cash flow statement reveals the effects of transactions involving movement of cash”.

This statement is (a) Correct

(b) Incorrect

(c) Partially Correct

(d) Irrelevant. 3. The Preparation of Cash flow statement is governed by AS-3 (Revised). This statement

is (a) False

(b) True

(c) Partially true

(d) Cannot say. 4. A cash flow statement is like an income statement

(a) I agree

(b) I disagree

3.64

Page 652: 30510870 Cost Accounting and Financial Management

Financial Analysis and Planning

(c) I cannot say

(d) The statement is ambiguous. 5. Funds flow statement and cash flow statement are one and the same

(a) True

(b) False

(c) I cannot say

(d) The statement is irrelevant. 6. Increase in the amount of bills payable results in

(a) Increase in cash

(b) Decrease in cash

(c) No change in cash

(d) I cannot say. 7. Cash from operations is equal to

(a) Net profit plus increase in outstanding expenses

(b) Net profit plus increase in debtors

(c) Net profit plus increase in stock

(d) None of the above. 8. Cash equivalents are short term highly liquid investments that are readily convertible into

known amounts of cash and which are subject to an insignificant risk of changes in value. (a) I agree

(b) I do not agree

(c) I cannot say

(d) Irrelevant. 9. For an investment to qualify as a cash equivalent, it must be readily convertible to a

known amount of (a) Gold

(b) Cash

3.65

Page 653: 30510870 Cost Accounting and Financial Management

Financial Management

(c) Investment

(d) Real estate. 10. Non-cash transactions

(a) Form part of cash flow statement

(b) Do not form part of cash flow statement

(c) May or may not form part of cash flow statement

(d) I cannot say whether they are part of cash flow statement. Answers to Objective Type Questions 1. (a); 2. (a); 3. (b) 4. (b); 5. (b); 6. (a); 7. (a); 8. (a); 9. (b); 10. (a); B. Long Answer Type Questions 1. What is a cash flow statement? Discuss briefly its major classification. 2. Distinguish between Funds flow statement and Cash flow statement. 3. Outline the Importance of cash flow statement in managing cash flows of a business

organization. 4. Explain the technique of preparing a cash flow statement with imaginary figures. 5. Explain the difference between direct and indirect methods of reporting cash flows from

operating activities as per AS-3 (Revised). C. Practical Problems 1. From the following Summary Cash Account of X Ltd. prepare Cash Flow Statement for

the year ended 31st March, 2006 in accordance with AS-3 (Revised) using the direct method. The Company does not have any cash equivalents.

Summary Cash Account for the year ended 31.3.2006 Balance on 1-4-2005 50 Payment of Suppliers 2,000 Issue of Equity Share 300 Purchase of Fixed Assets 200 Receipts from Customers 2,800 Overhead expense 200 Sale of Fixed Assets 100 Wags and Salaries 100 Taxation 250

3.66

Page 654: 30510870 Cost Accounting and Financial Management

Financial Analysis and Planning

Dividend 50 Repayment of Bank Loan 300 Balance on 31-3-2006 150 3,250 3,250 2. Ms. Jyoti of Star Oils Limited has collected the following information for the preparation of

cash flow statement for the year 2005: (Rs. in lakhs)

Net Profit 25,000 Dividend (including dividend tax) paid 8,535 Provision for Income-tax 5,000 Income-tax paid during the year 4,248 Loss on sale of assets (net) 40 Book value of the assets sold 185 Depreciation charged to Profit & Loss Account 20,000 Amortisation of Capital grant 6 Profit on sale of Investments 100 Carrying amount of Investment sold 27,765 Interest income on investments 2,506 Interest expenses 10,000 Interest paid during the year 10,520 Increase in Working Capital (excluding Cash & Bank balance) 56,075 Purchase of fixed assets 14,560 Investment in joint venture 3,850 Expenditure on construction work-in-progress 34,740 Proceeds from calls in arrear 2 Receipt of grant for capital projects 12

3.67

Page 655: 30510870 Cost Accounting and Financial Management

Financial Management

Proceeds from long-term borrowings 25,980 Proceeds from short-term borrowings 20,575 Opening cash and Bank balance 5,003 Closing Cash and Bank balance 6,988 Required: Prepare the Cash Flow Statement for the year 2005 in accordance with AS-3, Cash Flow Statements issued by the Institute of Chartered Accountants of India. (Make necessary assumption).

3. The summarized Balance Sheets of XYZ Ltd. as at 31st December, 2004 and 2005 are given below:

(Rs.)

Particulars 2004 2005 Liabilities Share capital 4,50,000 4,50,000 General Reserve 3,00,000 3,10,000 Profit and Loss account 56,000 68,000 Creditors 1,68,000 1,34,000 Provision for tax 75,000 10,000 Mortgage loan --- 2,70,000 10,49,000 12,42,000 Assets Fixed assets 4,00,000 3,20,000 Investments 50,000 60,000 Stock 2,40,000 2,10,000 Debtors 2,10,000 4,55,000 Bank 1,49,000 1,97,000 10,49,000 12,42,000

3.68

Page 656: 30510870 Cost Accounting and Financial Management

Financial Analysis and Planning

Additional information: (a) Investments costing Rs.8,000 were sold during the year 2005 for Rs.8,500. (b) Provision for tax made during the year was Rs.9,000. (c) During the year, part of the fixed assets costing Rs.10,000 was sold for Rs.12,000

and the profit was included in profit and loss account. (d) Dividend paid during the year amounted to Rs.40,000. You are required to prepare a Statement of Sources and Uses of cash.

4. The following are the changes in the account balances taken from the Balance Sheets of P Q Ltd. as at the beginning and end of the year:

Changes in Rupees in debit or credit

Equity share capital 30,000 shares of Rs.10 each issued and fully paid 0 Capital reserve (49,200) 8% debentures (50,000) Debenture discount 1,000 Freehold property at cost/revaluation 43,000 Plant and machinery at cost 60,000 Depreciation on plant and machinery (14,400) Debtors 50,000 Stock and work-in-progress 38,500 Creditors (11,800) Net profit for the year (76,500) Dividend paid in respect of earlier year 30,000 Provision for doubtful debts (3,300) Trade investments at cost 47,000 Bank (64,300) 0

You are informed that: (a) Capital reserve as at the end of the year represented realized profits on sale of one

freehold property together with surplus arising on the revaluation of balance of freehold properties.

3.69

Page 657: 30510870 Cost Accounting and Financial Management

Financial Management

(b) During the year plant costing Rs.18,000 against which depreciation provision of Rs.13,500 was lying was sold for Rs.7,000.

(c) During the middle of the year Rs.50,000 debentures were issued for cash at a discount of Rs.1,000.

(d) The net profit for the year was after crediting the profit on sale of plant and charging debenture interest.

You are required to prepare a cash flow statement which will explain, why bank borrowing has increased by Rs.64,300 during the year end. Ignore taxation.

5. Given below are the condensed Balance Sheets of M.M. Kusha Ltd. for two years and the statement of Profit and Loss for one year:

(Rs. lakhs)

As at 31st March 2005 2004 Share Capital In Equity shares of Rs.100 each 150 110 10% Redeemable Preference Shares of Rs.100 each 10 40 Capital Redemption Reserve 10 -- General Reserve 15 10 Profit and Loss Account balance 30 20 8% Debentures with Convertible Option 20 40 Other Term Loans 15 30 250 250 Fixed assets less Depreciation 130 100 Long-Term Investments 40 50 Working Capital 80 100 250 250

Statement of Profit and Loss for the year ended 31st March, 2005

(Rs. lakhs) Sales 600 Less: Cost of sale 400 200 Establishment charges 30

3.70

Page 658: 30510870 Cost Accounting and Financial Management

Financial Analysis and Planning

Selling and distribution expenses 60 Interest expenses 5 Loss on sale of equipment (Book value Rs.40 lakhs) 15 110 90 Interest income 4 Dividend income 2 Foreign exchange gain 10 Damages received for loss of reputation 14 30 120 Depreciation 50 70 Taxation 30 40 Dividends 15 Net Profit carried to Balance Sheet 25

You are informed by the accountant that ledgers relating to debtors, creditors and stock for both the years were seized by the income-tax authorities and it would take at least two months to obtain copies of the same. However, he is able to furnish the following data:

(Rs. lakhs) Particulars 2005 2004 Dividend receivable 2 4 Interest receivable 3 2 Cash on hand and with bank 7 10 Investments maturing within two months 3 2 15 18 Interest payable 4 5 Taxes payable 6 3 10 8 Current ratio 1.5 1.4 Acid test ratio 1.1 0.8

3.71

Page 659: 30510870 Cost Accounting and Financial Management

Financial Management

It is also gathered that debenture-holders owning 50% of the debentures outstanding as on 31-3-2004 exercised the option for conversion into equity shares during the financial year and the same was put through. You are required to prepare a direct method cash flow statement for the financial year, 2005 in accordance with Accounting Standard (AS 3) revised.

3.72

Page 660: 30510870 Cost Accounting and Financial Management

CHAPTER 4

FINANCING DECISIONS

UNIT – I : COST OF CAPITAL

Learning Objectives After studying this unit you will be able to learn

♦ What is cost of capital? ♦ How to measure the cost of each component of capital? ♦ What is weighted average cost of capital (WACC) and marginal cost of capital? and; ♦ How cost of capital is important in financial management?

1.1 INTRODUCTION The financing decision relates to the composition of relative proportion of various sources of finance. The financial management weighs the merits and demerits of different sources of finance while taking the financing decision. A business can be financed from either the shareholders funds or borrowings from outside agencies. The shareholders funds include equity share capital, preference share capital and the accumulated profits whereas borrowings from outsiders include borrowed funds like debentures and loans from financial institutions. The borrowed funds have to be paid back with interest and some amount of risk is involved if the principal and interest is not paid. Equity has no fixed commitment regarding payment of dividends or principal amount and therefore, no risk is involved. It is the decision of the business to decide the ratio of borrowed funds and owned funds. However, most of the companies use a combination of both the shareholders funds and borrowed funds. Whether the companies choose shareholders funds or borrowed funds, each type of fund carries a cost. Borrowed funds involve interest payment whereas equities, as such do not have any fixed obligation but definitely they involve a cost. The cost of equity is the minimum return the shareholders would have received if they had invested elsewhere. Both types of funds incur cost and this is the cost of capital to the company. This means, cost of capital is the minimum return expected by the company. The financing decision is an important managerial decision. It influences the shareholder’s return and risk. As a result, the market value of the share may be affected by the financing decision. Subsequently, whenever funds are to be raised to finance investments, capital

Page 661: 30510870 Cost Accounting and Financial Management

Financial Management structure decision is involved. The process of financing or capital structure decision is depicted in the figure below. A demand for raising funds generates a new capital structure since a decision has to be made as to the quantity and forms of financing. This decision will involve an analysis of the existing capital structure and the factors governing the decision. The company’s policy to retain or distribute earnings affects the shareholders claims. Retention of earnings strengthens the shareholders position. The debt-equity mix of the company is also affected by the new financing decision of the company. The financing mix (debt-equity mix) has implications for the shareholders’ earnings and risk, which in turn, will affect the cost of capital and the market value of the firm.

Financing Decision Process

4.2

Page 662: 30510870 Cost Accounting and Financial Management

Financing Decisions

1.2 DEFINITION OF COST OF CAPITAL Cost of capital maybe defined as the cut off rate for determining estimated future cash proceeds of a project and eventually deciding whether the project is worth undertaking or not. It is also the minimum rate of return that a firm must earn on its investment which will maintain the market value of share at its current level. It can also be stated as the opportunity cost of an investment, i.e. the rate of return that a company would otherwise be able to earn at the same risk level as the investment that has been selected. For example, when an investor purchases stock in a company, he/she expects to see a return on that investment. Since the individual expects to get back more than his/her initial investment, the cost of capital is equal to this return that the investor receives, or the money that the company misses out on by selling its stock. It can also be said as the required return necessary to make a capital budgeting project - such as building a new factory - worthwhile. Cost of capital includes the cost of debt and the cost of equity. The cost of capital determines how a company can raise money maybe through a stock issue, borrowing, or a mix of the two. This is the rate of return that a firm would receive if it invested its money someplace else with similar risk. Another way to think of the cost of capital is as the opportunity cost of funds, since this represents the opportunity cost for investing in assets with the same risk as the firm. When investors are shopping for places in which to invest their funds, they have an opportunity cost. The firm, given its riskiness, must strive to earn the investor’s opportunity cost. If the firm does not achieve the return investors expect (i.e. the investor’s opportunity cost), investors will not invest in the firm’s debt and equity. As a result, the firm’s value (both their debt and equity) will decline. The total capital for a firm is the value of its equity plus the cost of its debt (the cost of debt should be continually updated as the cost of debt changes as a result of interest rate changes). The cost of capital is given as:

Kc= (1-δ)Ke+ δKd

Where, Kc = Weighted cost of capital for the firm δ = Debt to capital ratio, D / (D + E) Ke = Cost of equity

4.3

Page 663: 30510870 Cost Accounting and Financial Management

Financial Management

Kd = After tax cost of debt D = Market value of the firm's debt, including bank loans and leases E = Market value of all equity (including warrants, options, and the equity portion of convertible securities)

1.3 MEASUREMENT OF COST OF CAPITAL The cost of capital is useful in determining a financial plan. A company has to employ a combination of creditor’s and owner’s funds. As more than one type of capital is used in a company, the composite cost of capital can be determined after the cost of each type of funds has been obtained. The first step is, therefore, the calculation of specific cost which is the minimum financial obligation required to secure the use of capital for a particular source. In order to calculate the specific cost of each type of capital, recognition should be given to the explicit and the implicit cost. The explicit cost of any source of capital may be defined as the discount rate that equals that present value of the cash inflows that are incremental to the taking of financing opportunity with the present value of its incremental cash outflows. Implicit cost is the rate of return associated with the best investment opportunity for the firm and its shareholders that will be foregone if the project presently under consideration by the firm was accepted. The explicit cost arises when funds are raised and when funds are used, implicit cost arises. For capital budgeting decisions, cost of capital is nothing but the explicit cost of capital. Now the different components of cost of capital i.e. each source of finance have been discussed in detail.

1.3.1 COST OF DEBT A bond is a long term debt instrument or security. Bonds issued by the government do not have any risk of default. The government honour obligations on its bonds. Bonds of the public sector companies in India are generally secured, but they are not free from the risk of default. The private sector companies also issue bonds, which are also called debentures in India. A company in India can issue secured or unsecured debentures. In the case of a bond or debenture, the rate of interest is generally fixed and known to investors. The principal of a redeemable bond or bond with a maturity is payable after a specified period, called maturity period. The chief characteristics of a bond or debenture are as follows: Face value: Face value is called par value. A bond or debenture is generally issued at a par value of Rs. 100 or Rs. 1,000, and interest is paid on face value.

4.4

Page 664: 30510870 Cost Accounting and Financial Management

Financing Decisions

Interest rate: Interest rate is fixed and known to bondholders or debenture holders. Interest paid on a bond or debenture is tax deductible. The interest rate is also called coupon rate. Coupons are detachable certificates of interest. Maturity: A bond or debenture is generally issued for a specified period of time. It is repaid on maturity. Redemption value: The value that a bondholder or debenture holder will get on maturity is called redemption or maturity value. A bond or debenture may be redeemed at par or at premium (more than par value) or at discount (less than par value). Market value: A bond or debenture may be traded in a stock exchange. The price at which it is currently sold or bought is called the market value of the bond or debenture. Market value may be different from par value or redemption value. 1.3.1.1 Cost of Debentures: The cost of debentures and long term loans is the contractual interest rate adjusted further for the tax liability of the company. When the firm employs debt, it must ensure that common shareholder’s earnings are not diluted. To keep the earnings unchanged, the firm must earn a return equal to the interest rate of debt. If the firm earns less than the interest rate, market share price would be adversely affected. In calculating weighted (average) cost of capital, cost of debt (after tax) should be used. For a company, the higher the interest charges, the lower the amount of tax payable by the company. An illustration will help you in understanding this point. Illustration 1: Consider two companies X and Y:

Company X Company Y Earnings before interest and taxes (EBIT) 100 100 Interest (I) - 40 Profit before tax (PBT) 100 60 Tax (T) 1 35 21 Profit after tax (PAT) 65 39

Assume an effective rate of tax of 35 percent Solution A comparison of the two companies shows that an interest payment of 40 in company Y results in a tax shield of 14 - that is 40 multiplied by 0.35, the corporate tax rate. The important point to remember, while calculating the average cost of capital, is that the post-tax cost of debt must be used and not the pre-tax cost of debt.

4.5

Page 665: 30510870 Cost Accounting and Financial Management

Financial Management 1.3.1.1.1 Cost of Irredeemable Debentures: Cost of debentures not redeemable during the life time of the company.

( )t1NP1K d −=

Where, Kd = Cost of debt after tax I = Annual interest rate NP = Net proceeds of debentures t = Tax rate Suppose a company issues 1,000, 15% debentures of the face value of Rs. 100 each at a discount of Rs. 5. Suppose further, that the under-writing and other costs are Rs. 5,000/- for the total issue. Thus Rs. 90,000 is actually realised, i.e., Rs. 1,00,000 minus Rs. 5,000 as discount and Rs. 5,000 as under-writing expenses. The interest per annum of Rs. 15,000 is therefore the cost of Rs. 90,000, actually received by the company. This is because interest is a charge on profit and every year the company will save Rs. 7,500 as tax, assuming that the income tax rate is 50%. Hence the after tax cost of Rs. 90,000 is Rs. 7,500 which comes to 8.33%. 1.3.1.1.2 Cost of Redeemable Debentures: If the debentures are redeemable after the expiry of a fixed period, the cost of debentures would be:

2NPRV

N/)NPRV()tI(IK d +−+−

=

Where, I = Annual interest payment NP = Net proceeds of debentures RV = Redemption value of debentures t = Tax rate N = Life of debentures. Illustration 2: A company issued 10,000, 10% debentures of Rs. 100 each on 1.4.2006 to be matured on 1.4.2011. If the market price of the debentures is Rs. 80. Compute the cost of debt assuming 35% tax rate.

4.6

Page 666: 30510870 Cost Accounting and Financial Management

Financing Decisions

Solution

2NP RV

NNP RV t) (1 I

K d +

−+−

=

280 100

580 100 .35) (1 10

K d +

⎟⎠⎞

⎜⎝⎛ −

+−=

90

4 .56 +=

= 0.1166 = 0.12 1.3.1.2 Value of Bonds: It is comparatively easy to find out the present value of a bond since its cash flows and the discount rate can be determined easily. If there is no risk of default, then there is no difficulty in calculating the cash flows associated with a bond. The expected cash flows consist of annual interest payments plus repayment of principal. The appropriate capitalisation or discount rate would depend upon the risk of the bond. The risk in holding a government bond is less than the risk associated with a debenture issued by a company. Therefore, a lower discount rate would be applied to the cash flows of the government bond and a higher rate to the cash flows of the company debenture. 1.3.1.2.1 Amortisation of Bond: A bond may be amortised every year i.e. principal is repaid every year rather than at maturity. In such a situation, the principal will go down with annual payments and interest will be computed on the outstanding amount. The cash flows of the bonds will be uneven. The formula for determining the value of a bond or debenture that is amortised every year is as follows:

)k 1(

C......... )k 1(

C )k 1(

CV nd

n2

d

21

d

1B +

+++

++

=

∑+

==

n

1 t t

d

tB )k 1(

C V

Illustration 3: Reserve Bank of India is proposing to sell a 5-year bond of Rs. 5,000 at 8 per cent rate of interest per annum. The bond amount will be amortised equally over its life. What

4.7

Page 667: 30510870 Cost Accounting and Financial Management

Financial Management is the bond’s present value for an investor if he expects a minimum rate of return of 6 per cent? Solution The amount of interest will go on declining as the outstanding amount of bond will be reducing due to amortisation. The amount of interest for five years will be:

First year: Rs. 5,000 × 0.08 = Rs. 400;

Second year: (Rs. 5,000 – Rs. 1,000) × 0.08 = Rs. 320;

Third year: (Rs. 5,000 – Rs. 1,000) × 0.08 = Rs. 240;

Fourth year: (Rs. 5,000 – Rs. 1,000) × 0.08 = Rs. 160; and

Fifth year: (Rs. 5,000– Rs.1,000) × 0.08 = Rs. 108. The outstanding amount of bond will be zero at the end of fifth year. Since Reserve Bank of India will have to return Rs. 1,000 every year, the outflows every year will consist of interest payment and repayment of principal: First year: Rs. 1000 + Rs. 400 = Rs. 1,400; Second year: Rs. 1000 + Rs. 320 = Rs. 1,320; Third year: Rs. 1000 + Rs. 240 = Rs. 1,240; Fourth year: Rs. 1000 + Rs. 160 = Rs. 1,160; and Fifth year: Rs. 1000 + Rs. 108 = Rs. 1,108. Referring to the present value table at the end of the study material, the value of the bond is calculated as follows:

)06.1(

080,1 )06.1(

160,1 )06.1(

240,1 )06.1(

320,1 )06.1(

400,1V 54321B ++++=

= 1,400 × 0.943 + 1,320 × 0.890 + 1,240 × 0.840 + 1,160 × 0.792 + 1,080 × 0.747 = 1,320.20 + 1,174.80 + 1,041.60 + 918.72 + 806.76 = Rs. 5,262.08 1.3.2 COST OF PREFERENCE SHARES The cost of preference share capital is the dividend expected by its holders. Though payment of dividend is not mandatory, non-payment may result in exercise of voting rights by them. The payment of preference dividend is not adjusted for taxes as they are paid after taxes and

4.8

Page 668: 30510870 Cost Accounting and Financial Management

Financing Decisions

is not deductible. The cost of preference share capital is calculated by dividing the fixed dividend per share by the price per preference share. Illustration 4: If Reliance Energy is issuing preferred stock at Rs.100 per share, with a stated dividend of Rs.12, and a floatation cost of 3% then, what is the cost of preference share? Solution

)tcos floatation1( stock preferred of price Marketdividend stock erredefPrK p −

=

%4.12)03.01(100.Rs

12.Rs=

−=

1.3.2.1 Cost of Irredeemable Preference Shares: Cost of irredeemable preference shares

= POPD

Where, PD = Annual preference dividend

PO = Net proceeds in issue of preference shares. Illustration 5: XYZ & Co. issues 2,000 10% preference shares of Rs. 100 each at Rs. 95 each. Calculate the cost of preference shares. Solution

POPD Kp =

( )( )2,000 95

2,000 01 K p ××

=

95

10 =

= 0.1053 1.3.2.2 Cost of Redeemable Preference Shares: If the preference shares are redeemable after the expiry of a fixed period the cost of preference shares would be:

4.9

Page 669: 30510870 Cost Accounting and Financial Management

Financial Management

2NPRV

N/)NPRV(PDK p +−+

=

Where, PD = Annual preference dividend RV = Redemption value of preference shares NP = Net proceeds on issue of preference shares N = Life of preference shares. However, since dividend of preference shares is not allowed as deduction from income for income tax purposes, there is no question of tax advantage in the case of cost of preference shares. It would, thus, be seen that both in the case of debt as well as preference shares, cost of capital is calculated by reference to the obligations incurred and proceeds received. The net proceeds received must be taken into account in working out the cost of capital. Illustration 6: Referring to the earlier question but taking into consideration that if the company proposes to redeem the preference shares at the end of 10th year from the date of issue. Calculate the cost of preference share? Solution

2NPRV

N/)NPRV(PDK p +−+

=

⎟⎠⎞

⎜⎝⎛ +

⎟⎠⎞

⎜⎝⎛ −

+=

295 100

1095 100 10

K p = .107 (approx.)

1.3.3 COST OF EQUITY It may prima facie appear that equity capital does not carry any cost. But this is not true. The market share price is a function of return that equity shareholders expect and get. If the company does not meet their requirements, it will have an adverse effect on the market share price. Also, it is relatively the highest cost of capital. Since expectations of equity holders are high, higher cost is associated with it.

4.10

Page 670: 30510870 Cost Accounting and Financial Management

Financing Decisions

Cost of equity capital is the rate of return which equates the present value of expected dividends with the market share price. The calculation of equity capital cost raises a lot of problems. Its purpose is to enable the corporate manager, to make decisions in the best interest of equity holders. In theory the management strives to maximize the position of equity holders and the effort involves many decisions. Different methods are employed to compute the cost of equity capital. (a) Dividend Price Approach: Here, cost of equity capital is computed by dividing the current dividend by average market price per share. This dividend price ratio expresses the cost of equity capital in relation to what yield the company should pay to attract investors. However, this method cannot be used to calculate cost of equity of units suffering losses.

o

1e P

DK =

Where, Ke = Cost of equity D1 = Annual dividend Po = Market value of equity (ex dividend) This model assumes that dividends are paid at a constant rate to perpetuity. It ignores taxation. (b) Earning/ Price Approach: The advocates of this approach co-relate the earnings of the company with the market price of its share. Accordingly, the cost of ordinary share capital would be based upon the expected rate of earnings of a company. The argument is that each investor expects a certain amount of earnings, whether distributed or not from the company in whose shares he invests. Thus, if an investor expects that the company in which he is going to subscribe for shares should have at least a 20% rate of earning, the cost of ordinary share capital can be construed on this basis. Suppose the company is expected to earn 30% the investor will be

prepared to pay Rs. 150 ⎟⎠⎞

⎜⎝⎛ ×100

2030.Rs for each share of Rs. 100. This approach is similar to

the dividend price approach; only it seeks to nullify the effect of changes in the dividend policy. This approach also does not seem to be a complete answer to the problem of determining the cost of ordinary share since it ignores the factor of capital appreciation or depreciation in the market value of shares. (c) Dividend Price + Growth Approach: Earnings and dividends do not remain constant and the price of equity shares is also directly influenced by the growth rate in dividends.

4.11

Page 671: 30510870 Cost Accounting and Financial Management

Financial Management Where earnings, dividends and equity share price all grow at the same rate, the cost of equity capital may be computed as follows: Ke = (D/P) + G Where, D = Current dividend per share P = Market price per share G = Annual growth rate of earnings of dividend. Illustration 7: A company has paid dividend of Rs. 1 per share (of face value of Rs. 10 each) last year and it is expected to grow @ 10% next year. Calculate the cost of equity if the market price of share is Rs. 55. Solution

G P D K e +=

.10 55

.10) (1 1 ++

=

= .1202 (approx.) (d) Earnings Price + Growth Approach: This approach is an improvement over the earlier methods. But even this method assumes that dividend will increase at the same rate as earnings, and the equity share price is the regulator of this growth as deemed by the investor. However, in actual practice, rate of dividend is recommended by the Board of Directors and shareholders cannot change it. Thus, rate of growth of dividend subsequently depends on director’s attitude. The dividend method should, therefore, be modified by substituting earnings for dividends. So, cost of equity will be given by: Ke = (E/P) + G Where, E = Current earnings per share P = Market share price G = Annual growth rate of earnings. The calculation of ‘G’ (the growth rate) is an important factor in calculating cost of equity capital. The past trend in earnings and dividends may be used as an approximation to predict the future growth rate if the growth rate of dividend is fairly stable in the past.

4.12

Page 672: 30510870 Cost Accounting and Financial Management

Financing Decisions

G = 1.0 (1+G)n where n is the number of years (e) Realized Yield Approach: According to this approach, the average rate of return realized in the past few years is historically regarded as ‘expected return’ in the future. The yield of equity for the year is:

1t

1ttt P

PDY−

−+=

Where, Yt = Yield for the year t Dt = Dividend for share for end of the year t

Pt = Price per share at the end of the year t Pt – 1 = Price per share at the beginning and at the end of the year t Though, this approach provides a single mechanism of calculating cost of equity, it has unrealistic assumptions. If the earnings do not remain stable, this method is not practical. (f) Capital Asset Pricing Model Approach (CAPM): This model describes the linear relationship between risk and return for securities. The risk a security is exposed to are diversifiable and non-diversifiable. The diversifiable risk can be eliminated through a portfolio consisting of large number of well diversified securities. The non-diversifiable risk is assessed in terms of beta coefficient (b or β) through fitting regression equation between return of a security and the return on a market portfolio.

Cost of Equity under CAPM

4.13

Page 673: 30510870 Cost Accounting and Financial Management

Financial Management Thus, the cost of equity capital can be calculated under this approach as: Ke = Rf + b (Rm − Rf) Where, Ke = Cost of equity capital Rf = Rate of return on security b = Beta coefficient Rm = Rate of return on market portfolio

Govt. Bonds

Corp. Debts

Pref.Shares

Equity Shares

0

1

2

3

4

5

6

7

8

9

10

Risk Return relationship of various securities

Therefore, required rate of return = risk free rate + risk premium The idea behind CAPM is that investors need to be compensated in two ways- time value of money and risk. The time value of money is represented by the risk-free rate in the formula and compensates the investors for placing money in any investment over a period of time. The other half of the formula represents risk and calculates the amount of compensation the investor needs for taking on additional risk. This is calculated by taking a risk measure (beta) which compares the returns of the asset to the market over a period of time and compares it to the market premium. The CAPM says that the expected return of a security or a portfolio equals the rate on a risk-

4.14

Page 674: 30510870 Cost Accounting and Financial Management

Financing Decisions

free security plus a risk premium. If this expected return does not meet or beat the required return, then the investment should not be undertaken. The shortcomings of this approach are: (a) Estimation of betas with historical data is unrealistic; and (b) Market imperfections may lead investors to unsystematic risk. Despite these shortcomings, the capital asset pricing approach is useful in calculating cost of equity, even when the firm is suffering losses. The basic factor behind determining the cost of ordinary share capital is to measure the expectation of investors from the ordinary shares of that particular company. Therefore, the whole question of determining the cost of ordinary shares hinges upon the factors which go into the expectations of particular group of investors in a company of a particular risk class. Illustration 8: Calculate the cost of equity capital of H Ltd., whose risk free rate of return equals 10%. The firm’s beta equals 1.75 and the return on the market portfolio equals to 15%. Solution

Ke = Rf + b (Rm − Rf)

Ke = .10 + 1.75 (.15 − .10)

= .10 + 1.75 (.05)

= .1875 1.3.4 COST OF RETAINED EARNINGS Like another source of fund, retained earnings involve cost. It is the opportunity cost of dividends foregone by shareholders. The given figure depicts how a company can either keep or reinvest cash or return it to the shareholders as dividends. (Arrows represent possible cash flows or transfers.) If the cash is reinvested, the opportunity cost is the expected rate of return that shareholders could have obtained by investing in financial assets.

4.15

Page 675: 30510870 Cost Accounting and Financial Management

Financial Management

Cost of Retained Earnings

There are two approaches to measure this opportunity cost. One approach is by using discounted cash flow (DCF) method and the second approach is by using capital asset pricing model.

(a) By DCF : Ks = GPD

0

1 +

Where, D1 = Dividend P0 = Current market price G = Growth rate. (b) By CAPM : Ks = Rf + b (Rm − Rf) Where, Ks = Cost of equity capital Rf = Rate of return on security b = Beta coefficient Rm = Rate of return on market portfolio Illustration 9: ABC Company provides the following details: D0 = Rs. 4.19 P0 Rs. 50 G = 5% Calculate the cost of retained earnings based on DCF method.

4.16

Page 676: 30510870 Cost Accounting and Financial Management

Financing Decisions

Solution

Ks = GPD

0

1 +

= ( ) GP

G1D0

0 ++

= ( ) 05.050.Rs

05.1 19.4 .Rs+

= 0.088 + 0.05 = 13.8% Illustration 10: ABC Company provides the following details: Rf = 7% b = 1.20 RM - Rf = 6% Calculate the cost of retained earnings based on CAPM method. Solution Ks = Rf + b (RM – Rf) = 7% + 1.20 (6%) = 7% + 7.20 Ks = 14.2% 1.3.5 COST OF DEPRECIATION Depreciation provisions may be considered in a similar manner to retained earnings - they have an opportunity cost and represent an increased stake in the firm by its shareholders. However, a distribution of depreciation provisions would produce a capital reduction, probably requiring outstanding debts to be repaid due to the depletion of the capital base, the security against which the debt was obtained. This indicates a proportional combination between the cost of debt repaid and the cost of retained earnings to calculate the cost of capital in the form of depreciation provisions.

1.4 WEIGHTED AVERAGE COST OF CAPITAL (WACC) As you know the capital funding of a company is made up of two components: debt and equity. Lenders and equity holders each expect a certain return on the funds or capital they have provided. The cost of capital is the expected return to equity owners (or shareholders) and to debt holders, so weighted average cost of capital tells the return that both stakeholders -

4.17

Page 677: 30510870 Cost Accounting and Financial Management

Financial Management equity owners and lenders - can expect. WACC, in other words, represents the investors' opportunity cost of taking on the risk of putting money into a company. Since every company has a capital structure i.e. what percentage of debt comes from retained earnings, equity shares, preference shares, and bonds, so by taking a weighted average, it can be seen how much interest the company has to pay for every rupee it borrows. This is the weighted average cost of capital. The weighted average cost of capital for a firm is of use in two major areas: in consideration of the firm's position and in evaluation of proposed changes necessitating a change in the firm's capital. Thus, a weighted average technique may be used in a quasi-marginal way to evaluate a proposed investment project, such as the construction of a new building. Thus, weighted average cost of capital is the weighted average after tax costs of the individual components of firm’s capital structure. That is, the after tax cost of each debt and equity is calculated separately and added together to a single overall cost of capital. K0 = % D(mkt) (Ki) (1 – t) + (% Psmkt) Kp + (Cs mkt) Ke Where, K0 = Overall cost of capital Ki = Before tax cost of debt 1 – t = 1 – Corporate tax rate Kp = Cost of preference capital Ke = Cost of equity % Dmkt = % of debt in capital structure %Psmkt = % of preference share in capital structure % Cs = % of equity share in capital structure. The cost of weighted average method is preferred because the proportions of various sources of funds in the capital structure are different. To be representative, therefore, cost of capital should take into account the relative proportions of different sources of finance. Securities analysts employ WACC all the time when valuing and selecting investments. In discounted cash flow analysis, WACC is used as the discount rate applied to future cash flows for deriving a business's net present value. WACC can be used as a hurdle rate against which to assess return on investment capital performance. It also plays a key role in economic value added (EVA) calculations. Investors use WACC as a tool to decide whether or not to invest. The WACC represents the minimum rate of return at which a company produces value for its investors. Let's say a

4.18

Page 678: 30510870 Cost Accounting and Financial Management

Financing Decisions

company produces a return of 20% and has a WACC of 11%. By contrast, if the company's return is less than WACC, the company is shedding value, which indicates that investors should put their money elsewhere. Therefore, WACC serves as a useful reality check for investors.

1.4.1 CALCULATION OF WACC

Capital Component

Cost Times % of capital structure

Total

Retained Earnings

10% X 25% 2.50%

Common Stocks 11% X 10% 1.10% Preferred Stocks 9% X 15% 1.35% Bonds 6% X 50% 3.00% Total 7.95%

So the WACC of this company is 7.95%. But there are problems in determination of weighted average cost of capital. These mainly relate to computation of equity capital and the assignment of weights to the cost of specific source of financing. Assignment of weights can be possible either on the basis of marginal weighting or historical weighting. The most serious limitation of marginal weighting is that it does not consider the long run implications of firm’s current financing. The validity of the assumption of historical weighting is that choosing between the book value weights and market value weights. While the book value weights may be operationally convenient, the market value basis is theoretically more consistent, sound and a better indicator of firm’s capital structure. The desirable practice is to employ market weights to compute the firm’s cost of capital. This rationale rests on the fact that the cost of capital measures the cost of issuing securities – stocks as well as bonds – to finance projects, and that these securities are issued at market value, not at book value. Illustration 11: Calculate the WACC using the following data by using: (a) Book value weights (b) Market value weights

4.19

Page 679: 30510870 Cost Accounting and Financial Management

Financial Management The capital structure of the company is as under:

Rs. Debentures (Rs. 100 per debenture) 5,00,000 Preference shares (Rs. 100 per share) 5,00,000 Equity shares (Rs. 10 per share) 10,00,000 20,00,000

The market prices of these securities are: Debenture Rs. 105 per debenture Preference Rs. 110 per preference share Equity Rs. 24 each. Additional information:

(1) Rs. 100 per debenture redeemable at par, 10% coupon rate, 4% floatation costs, 10 year maturity.

(2) Rs. 100 per preference share redeemable at par, 5% coupon rate, 2% floatation cost and 10 year maturity.

(3) Equity shares has Rs. 4 floatation cost and market price Rs. 24 per share. The next year expected dividend is Rs. 10 with annual growth of 5%. The firm has practice of paying all earnings in the form of dividend. Corporate tax rate is 50%. Solution Cost of equity

.05 2010 K equity ofCost e +==

= .05 + .05 = .10

296) (100

1096) (100 .5) 10(1

K debt ofCost d +

−+−

==

4.20

Page 680: 30510870 Cost Accounting and Financial Management

Financing Decisions

(approx.) .055 2 196

.4 5=×⎟

⎠⎞

⎜⎝⎛ +

=

2198

102 5

K shares preference ofCost p

⎟⎟⎟⎟

⎜⎜⎜⎜

⎛ +==

q2.5 ⎟⎟⎠

⎞⎜⎜⎝

⎛= = .053 (approx.)

Calculation of WACC using book value weights

Source of capital Book Value Specific cost (K%) Total cost 10% Debentures 5,00,000 .055 27,500 5% Preference shares 5,00,000 .053 26,500 Equity shares 10,00,000 .10 1,00,000 20,00,000 1,54,000

(approx.) 0.077 20,00,000 .Rs1,54,000 Rs. K 0 ==

Calculation of WACC using market value weights

Source of capital Book Value Specific cost (K%) Total cost 10% Debentures 5,25,000 .055 28,875 5% Preference shares 5,50,000 .053 29,150 Equity shares 24,00,000 .10 2,40,000 34,75,000 2,98,025

(approx.) 0.08576 000,75,34.Rs

2,98,025 Rs. K 0 ==

1.5 MARGINAL COST OF CAPITAL The marginal cost of capital may be defined as the cost of raising an additional rupee of capital. Since the capital is raised in substantial amount in practice marginal cost is referred to

4.21

Page 681: 30510870 Cost Accounting and Financial Management

Financial Management as the cost incurred in raising new funds. Marginal cost of capital is derived, when the average cost of capital is calculated using the marginal weights. The marginal weights represent the proportion of funds the firm intends to employ. Thus, the problem of choosing between the book value weights and the market value weights does not arise in the case of marginal cost of capital computation. To calculate the marginal cost of capital, the intended financing proportion should be applied as weights to marginal component costs. The marginal cost of capital should, therefore, be calculated in the composite sense. When a firm raises funds in proportional manner and the component’s cost remains unchanged, there will be no difference between average cost of capital (of the total funds) and the marginal cost of capital. The component costs may remain constant upto certain level of funds raised and then start increasing with amount of funds raised. For example, the cost of debt may remain 7% (after tax) till Rs. 10 lakhs of debt is raised, between Rs. 10 lakhs and Rs. 15 lakhs, the cost may be 8% and so on. Similarly, if the firm has to use the external equity when the retained profits are not sufficient, the cost of equity will be higher because of the floatation costs. When the components cost start rising, the average cost of capital will rise and the marginal cost of capital will however, rise at a faster rate. Illustration 12: ABC Ltd. has the following capital structure which is considered to be optimum as on 31st March, 2006.

Rs.

14% debentures 30,000 11% Preference shares 10,000 Equity (10,000 shares) 1,60,000 2,00,000

The company share has a market price of Rs. 23.60. Next year dividend per share is 50% of year 2006 EPS. The following is the trend of EPS for the preceding 10 years which is expected to continue in future.

Year EPS (Rs.) Year EPS Rs.) 1997 1.00 2002 1.61 1998 1.10 2003 1.77 1999 1.21 2004 1.95 2000 1.33 2005 2.15 2001 1.46 2006 2.36

4.22

Page 682: 30510870 Cost Accounting and Financial Management

Financing Decisions

The company issued new debentures carrying 16% rate of interest and the current market price of debenture is Rs. 96. Preference share Rs. 9.20 (with annual dividend of Rs. 1.1 per share) were also issued. The company is in 50% tax bracket. (A) Calculate after tax:

(i) Cost of new debt (ii) Cost of new preference shares (iii) New equity share (consuming new equity from retained earnings)

(B) Calculate marginal cost of capital when no new shares are issued. (C) How much needs to be spent for capital investment before issuing new shares? 50% of

the 2006 earnings are available as retained earnings for the purpose of capital investment.

(D) What will the marginal cost of capital when the funds exceeds the amount calculated in (C), assuming new equity is issued at Rs. 20 per share?

Solution (A) (i) Cost of new debt

N

t) (1 I K d−

=

.0833 96

)5. (1 16 =−

=

(ii) Cost of new preference shares

OP Kp =

.12 9.21.1 ==

(iii) Cost of new equity shares

G P

D K0

1e +=

10.10 0.1023.601.18 =+= = 0.15

4.23

Page 683: 30510870 Cost Accounting and Financial Management

Financial Management Calculation of D1

D1 = 50% of 2006 EPS = 50% of 2.36 = Rs. 1.18 (B)

Type of Capital Proportion Specific Cost Product (1) (2) (3) (2) × (3) = (4) Debt 0.15 0.0833 0.0125 Preference 0.05 0.12 0.0060 Equity 0.80 0.15 0.1200 Marginal cost of capital 0.1385

(C) The company can spend the following amount:

Retained earnings = (.50) (2.36 × 10,000) = Rs. 11,800

The ordinary equity is 80% of total capital

14,750 Rs. .8011,800 Rs. investment Capital ==

(D) If the company require fund in excess of Rs. 14,750 it will have to issue new shares. The cost of new issue will be

.159 .10 201.18 Rs. K e =+=

The marginal cost of capital will be

Type of Capital Proportion Specific Cost Product (1) (2) (3) (2) × (3) = (4)

Debt 0.15 0.0833 0.0125 Preference 0.05 0.1200 0.0060 Equity (New) 0.80 0.1590 0.1272 0.1457

1.6 CONCLUSION The determination of cost of capital is thus beset with a number of problems in dynamic world

4.24

Page 684: 30510870 Cost Accounting and Financial Management

Financing Decisions

of today. Conditions which are present now may not remain static in future. Therefore, howsoever cost of capital is determined now, it is dependent on certain conditions or situations which are subject to change. Firstly, the firms’ internal structure and character change. For instance, as the firm grows and matures, its business risk may decline resulting in new structure and cost of capital. Secondly, capital market conditions may change, making either debt or equity more favourable than the other. Thirdly, supply and demand for funds may vary from time to time leading to change in cost of different components of capital. Fourthly, the company may experience subtle change in capital structure because of retained earnings unless its growth rate is sufficient to call for employment of debt on a continuous basis. Because of these reasons the firm should periodically re-examine its cost of capital before determining annual capital budget.

4.25

Page 685: 30510870 Cost Accounting and Financial Management

Financial Management UNIT – II : CAPITAL STRUCTURE DECISIONS

Learning Objectives After studying this unit you will be able to learn

♦ What is capital structure? ♦ What are optimal capital structure, and the theories relating to the value of firm; and

♦ Understand EBIT-EPS break even or indifference analysis and construct and interpret an EBIT-EPS chart.

2.1 MEANING OF CAPITAL STRUCTURE Capital structure refers to the mix of a firm’s capitalisation and includes long term sources of funds such as debentures, preference share capital, equity share capital and retained earnings. According to Gerstenberg capital structure is “the make-up of a firm’s capitalisation”. The decisions regarding the forms of financing, their requirements and their relative proportions in total capitalisation are known as capital structure decisions. In arriving at and accomplishing this goal, the finance manager must take extreme care and prudence keeping in mind factors under which the company has to operate as also certain guiding principles of financing. Accordingly, he should choose a pattern of capital which minimises cost of capital and maximises the owners’ return.

2.2 CHOICE OF CAPITAL STRUCTURE A firm has the choice to raise funds for financing its investment proposals from different sources in different proportions. It can: (a) Exclusively use debt, or (b) Exclusively use equity capital, or (c) Exclusively use preference capital, or (d) Use a combination of debt and equity in different proportions, or (e) Use a combination of debt, equity and preference capital in different proportions, or (f) Use a combination of debt and preference capital in different proportions. The choice of the combination of these sources is called capital structure mix. But the question is which of the pattern should the firm choose? Well, while choosing a suitable financing pattern, certain fundamental principles should be kept in mind, which are discussed below:

4.26

Page 686: 30510870 Cost Accounting and Financial Management

Financing Decisions

(a) Cost Principle: According to this principle an ideal pattern or capital structure is one that minimises cost of capital structure and maximises earnings per share (EPS). Debt capital is cheaper than equity capital from the point of its cost and interest being deductible for income tax purpose, where no such deduction is allowed for dividends. Consequently effective rate of interest which the company has to bear would be less than the nominal rate at which debentures are issued. This requires the mix of debt finance with equity finance so as to reduce the aggregate cost of capital. (b) Risk Principle: According to this principle, reliance is placed more on common equity for financing capital requirements than excessive use of debt. Use of more and more debt and preference capital affects share values and in unfavourable situation share prices may consequently drop. There are two risks associated with this principle: (i) Business risk: It is an unavoidable risk because of the environment in which the firm has to operate and business risk is represented by the variability of earnings before interest and tax (EBIT). The variability in turn is influenced by revenues and expenses. Revenues and expenses are affected by demand of firm products, variations in prices and proportion of fixed cost in total cost. (ii) Financial risk: It is a risk associated with the availability of earnings per share caused by use of financial leverage. It is also unavoidable if firm does not use debt in its capital structure. Generally, a firm should neither be exposed to high degree of business risk and low degree of financial risk or vice-versa, so that shareholders do not bear a higher risk. (c) Control Principle: While designing a capital structure, the finance manager may also keep in mind that existing management control and ownership remains undisturbed. Issue of new equity will dilute existing control pattern and also it involves higher cost. Issue of more debt causes no dilution in control, but causes a higher degree of financial risk. This concern over dilution of control is mostly felt in closely-held companies. (d) Flexibility Principle: By flexibility it means that the management chooses such a combination of sources of financing which it finds easier to adjust according to changes in need of funds in future too. In attaining flexibility cost considerations should be kept in mind. If the company is loaded with a debt of 18% and funds are available at 15%, it can return old debt with new debt, at a lesser interest rate. Besides these principles, other factors such as nature of industry, timing of issue and competition in the industry are also being considered. Timing of raising capital should take into account the state of economy and capital market. Industries facing severe competition also resort to more equity than debt. Thus a finance manager in designing a suitable pattern of capital structure must bring about

4.27

Page 687: 30510870 Cost Accounting and Financial Management

Financial Management satisfactory compromise between these militant principles. The compromise can be reached by assigning weights to these principles in terms of various characteristics of the company.

2.3 SIGNIFICANCE OF CAPITAL STRUCTURE The capital structure decisions are so significant in financial management, as they influence debt – equity mix which ultimately affects shareholders return and risk. Since cost of debt is cheaper, firm prefers to borrow rather than to raise from equity. The value of equity depends on earnings per share. So long as return on investment is more than the cost of borrowing, extra borrowing increases the earnings per share. However, beyond a limit, it increases the risk and share price may fall because shareholders may assume that their investment is associated with more risk. But the effect of fall in share price due to heavy load of debt is difficult to measure. Market factors are so highly psychological and complex as they hardly follow these theoretical considerations. However, an appropriate debt -equity mix can be determined empirically within the company taking into consideration the following factors: 2.3.1 Leverages There are two leverages associated with the study of capital structure, namely operating leverage and financial leverage. Operating leverage exists when a firm has a fixed cost that must be defrayed regardless of volume of business. The contrast to the operating leverage, financial leverage refers to mix of debt and equity in the capitalisation of a firm. In order to decide proper financial policy, operating leverage may also be taken into consideration, as the financial leverage is a superstructure built on the operating leverage. The operating profits otherwise known as earnings before interest and tax (EBIT), serves as a fulcrum in defining these two leverages. Financial leverage represents the relationship between firms earnings before interest and tax and earnings available for equity holders. When there is an increase in EBIT, there is a corresponding increase in market price of equity share. However, increased use of debt in the capital structure which proportionately increases EBIT has certain limitations. If debt is employed in greater proportions, marginal cost of debt will also increase and share price may fall down as investors feel it is risky. On the other, in spite of increased risk, market share price may increase because investors speculate future profits. Thus before using financial leverage, its impact on EPS must be weighed. The degree of financial leverage can be found out as:

(EBIT) tax andinterest before Earnings in change Percentage(EPS) share per Earnings in change Percentage

A company having higher operating leverage should be accompanied by a low financial leverage and vice versa, otherwise it will face problems of insolvency and inadequate liquidity.

4.28

Page 688: 30510870 Cost Accounting and Financial Management

Financing Decisions

Thus a combination of both the leverages is a challenging task. 2.3.2 Trading on Equity The term ‘trading on equity’ is derived from the fact that debts are contracted and loans are raised mainly on the basis of equity capital. Those who provide debt have a limited share in the firm’s earnings and hence want to be protected in terms of earnings and values represented by equity capital. Since fixed charges do not vary with the firms earnings before interest and tax, a magnified effect is produced on earnings per share. Whether the leverage is favourable in the sense increase in earnings per share more proportionately to the increased earnings before interest and tax depends on the profitability of investment proposals. If the rate of return on investment exceeds their explicit cost financial leverage is said to be positive. However, the determination of optimal level of debt is a formidable task and is a major policy decision. Determination of optimal level of debt involves equalising between return and risk. Though, there are number of approaches to determine the level of debt, they cannot be considered as satisfactory and as such can serve only as a guideline. Whatever approaches may be followed for determining the optimal level of debt, the objective of maximising share price should be borne in mind. EBIT-EPS analysis is a widely used tool to determine level of debt in a firm. Through this analysis, a comparison can be drawn for various methods of financing by obtaining indifference point. It is a point to the EBIT level at which EPS remain unchanged irrespective of debt level equity mix. For example indifference point for the capital mix (equity share capital and debt) can be determined as follows:

2

2

1

1

E)T1)(IEBIT(

E)T1)(IEBIT( −−=

−−

Where, EBIT = Indifference point E1 = Number of equity shares in Alternative 1 E2 = Number of equity shares in Alternative 2 I1 = Interest charges in Alternative 1 12 = Interest charges in Alternative 2 T = Tax-rate Alternative 1= All equity finance Alternative 2= Debt-equity finance.

4.29

Page 689: 30510870 Cost Accounting and Financial Management

Financial Management The chief deficiency of this method is that it does not take into account the implicit cost associated with debt. The concepts of leverages and EBIT-EPS analysis would be dealt in detail separately for better understanding. 2.3.3 Coverage Ratio The ability of the firm to use debt in the capital structure can also be judged in terms of coverage ratio namely EBIT/Interest. Higher the ratio, greater is the certainty of meeting interest payments. 2.3.4 Cash flow analysis It is a good supporting tool for EBIT-EPS analysis in framing a suitable capital structure. To determine the debt capacity, cash flow under adverse conditions should be examined. A high debt equity ratio is not risky if the company has the ability to generate cash flows. It would, therefore be possible to increase the debt until cash flows equal the risk set out by debt. The main drawback of this approach is that it fails to take into account uncertainty due to technological developments or changes in political climate. These approaches as discussed above do not provide solution to the problem of determining an appropriate level of debt. However, with the information available a range can be determined for an optimum level of debt in the capital structure.

2.4 OPTIMAL CAPITAL STRUCTURE The theory of optimal capital structure deals with the issue of the right mix of debt and equity in the long term capital structure of a firm. This theory states that if a company takes on debt, the value of the firm increases up to a point. Beyond that point if debt continues to increase then the value of the firm will start to decrease. Similarly if the company is unable to repay the debt within the specified period then it will affect the goodwill of the company in the market and may create problems for collecting further debt. Therefore, the company should select its appropriate capital structure with due consideration to the factors mentioned above.

2.5 EBIT-EPS ANALYSIS The basic objective of financial management is to design an appropriate capital structure which can provide the highest earnings per share (EPS) over the firm’s expected range of earnings before interest and taxes (EBIT). EPS measures a firm’s performance for the investors. The level of EBIT varies from year to year and represents the success of a firm’s operations. EBIT-EPS analysis is a vital tool for designing the optimal capital structure of a firm. The objective of this analysis is to find the EBIT level that will equate EPS regardless of the financing plan chosen.

4.30

Page 690: 30510870 Cost Accounting and Financial Management

Financing Decisions

2.5.1 Financial Break-even and Indifference Analysis Financial break-even point is the minimum level of EBIT needed to satisfy all the fixed financial charges i.e. interest and preference dividends. It denotes the level of EBIT for which the firm’s EPS equals zero. If the EBIT is less than the financial breakeven point, then the EPS will be negative but if the expected level of EBIT is more than the breakeven point, then more fixed costs financing instruments can be taken in the capital structure, otherwise, equity would be preferred. EBIT-EPS breakeven analysis is used for determining the appropriate amount of debt a firm might carry. Another method of considering the impact of various financing alternatives on earnings per share is to prepare the EBIT chart or the range of Earnings Chart. This chart shows the likely EPS at various probable EBIT levels. Thus, under one particular alternative, EPS may be Rs. 2 at a given EBIT level. However, the EPS may go down if another alternative of financing is chosen even though the EBIT remains at the same level. At a given EBIT, earnings per share under various alternatives of financing may be plotted. A straight line representing the EPS at various levels of EBIT under the alternative may be drawn. Wherever this line intersects, it is known as break-even point. This point is a useful guide in formulating the capital structure. This is known as EPS equivalency point or indifference point since this shows that, between the two given alternatives of financing (i.e., regardless of leverage in the financial plans), EPS would be the same at the given level of EBIT. The equivalency or indifference point can also be calculated algebraically in the following manner.

2

2

1

1

E)T1)(IEBIT(

E)T1)(IEBIT( −−=

−−

Where, EBIT = Indifference point E1 = Number of equity shares in Alternative 1 E2 = Number of equity shares in Alternative 2 I1 = Interest charges in Alternative 1 12 = Interest charges in Alternative 2 T = Tax-rate Alternative 1= All equity finance Alternative 2= Debt-equity finance.

4.31

Page 691: 30510870 Cost Accounting and Financial Management

Financial Management The indifference point can also be depicted graphically as:

Debt-Equity Indifference Point

Illustration 1: Best of Luck Ltd., a profit making company, has a paid-up capital of Rs. 100 lakhs consisting of 10 lakhs ordinary shares of Rs. 10 each. Currently, it is earning an annual pre-tax profit of Rs. 60 lakhs. The company's shares are listed and are quoted in the range of Rs. 50 to Rs. 80. The management wants to diversify production and has approved a project which will cost Rs. 50 lakhs and which is expected to yield a pre-tax income of Rs. 40 lakhs per annum. To raise this additional capital, the following options are under consideration of the management: (a) To issue equity capital for the entire additional amount. It is expected that the new shares (face value of Rs. 10) can be sold at a premium of Rs. 15. (b) To issue 16% non-convertible debentures of Rs. 100 each for the entire amount. (c) To issue equity capital for Rs. 25 lakhs (face value of Rs. 10) and 16% non-convertible debentures for the balance amount. In this case, the company can issue shares at a premium of Rs. 40 each. You are required to advise the management as to how the additional capital can be raised, keeping in mind that the management wants to maximise the earnings per share to maintain its goodwill. The company is paying income tax at 50%.

4.32

Page 692: 30510870 Cost Accounting and Financial Management

Financing Decisions

Solution Calculation of Earnings per share under the three options:

Particulars Option I Option II Option III (Issue of equity

only) (Issue of

debentures only) (Issue of equity and debentures

equally) (Rs. in lakhs) (Rs in lakhs) (Rs in lakhs) Number of Equity Shares (lakhs):

Existing 10 10 10.00 Now issued 2 - 0.50 Total 12 10 10.50 16% debentures Nil Rs. 50 lakhs Rs. 25 lakhs Estimated total income: From current operations 60 60 60 From new projects 40 40 40 100 100 100 Less: Interest on 16% debentures

- 8 4

Profit before tax 100 92 96 Tax at 50% 50 46 48 Profit after tax 50 46 48 EPS Rs. 4.17 Rs. 4.60 Rs. 4.57 Advise: Option II i.e. issue of 16% debentures is most suitable to maximize the earnings per share.

2.6 COST OF CAPITAL, CAPITAL STRUCTURE AND MARKET PRICE OF SHARE The financial leverage has a magnifying effect on earnings per share, such that for a given level of financial percentage increases with EBIT beyond the point of financial indifference, there will be more than proportionate change in the same direction in the earnings per share. The financing decision of the firm is one of the basic conditions oriented to the achievement of

4.33

Page 693: 30510870 Cost Accounting and Financial Management

Financial Management maximisation for the shareholders wealth. The capital structure should be examined from their view point of its impact on the value of the firm. If the capital structure affects the total value of the firm, a firm should select such a financing mix (a combination of debt and equity) which will maximise the market value of the firm. Such an optimum leverage not only maximises the value of the company and wealth of its owners, but also minimises the cost of capital. As a result, the company is able to increase its economic rate of investment and growth. In theory, capital structure can affect the value of the firm by affecting either its expected earnings or cost of capital or both. While financing mix cannot affect the total earnings, it can affect the share of earnings belonging to the share holders. But financial leverage can largely influence the value of the firm through the cost of capital.

2.7 CAPITAL STRUCTURE THEORIES The following approaches explain the relationship between cost of capital, capital structure and value of the firm: (a) Net income approach (b) Net operating income approach (c) Modigliani-Miller approach (d) Traditional approach. However, the following assumptions are made to understand this relationship. ♦ There are only two kinds of funds used by a firm i.e. debt and equity. ♦ Taxes are not considered. ♦ The payout ratio is 100% ♦ The firm’s total financing remains constant ♦ Business risk is constant over time ♦ The firm has perpetual life. (a) Net Income Approach (NI): According to this approach, capital structure decision is relevant to the value of the firm. An increase in financial leverage will lead to decline in the weighted average cost of capital, while the value of the firm as well as market price of ordinary share will increase. Conversely a decrease in the leverage will cause an increase in the overall cost of capital and a consequent decline in the value as well as market price of equity shares.

4.34

Page 694: 30510870 Cost Accounting and Financial Management

Financing Decisions

From the above diagram, ke and kd are assumed not to change with leverage. As debt increases, it causes weighted average cost of capital to decrease. The value of the firm on the basis of Net Income Approach can be ascertained as follows: V = S + D Where, V = Value of the firm S = Market value of equity

D = Market value of debt

Market value of equity (S) = eK

NI

Where, NI = Earnings available for equ Ke = Equity Capitalisation rate

Under, NI approach, the value of the firmaverage cost of capital is minimum. Thus, thfinancing for minimising the cost of capital. T

ity shareholders

will be maximum at a point where weighted e theory suggests total or maximum possible debt

he overall cost of capital under this approach is :

4.35

Page 695: 30510870 Cost Accounting and Financial Management

Financial Management

firm the of ValueEBITcapital of tcos Overall =

Thus according to this approach, the firm can increase its total value by decreasing its overall cost of capital through increasing the degree of leverage. The significant conclusion of this approach is that it pleads for the firm to employ as much debt as possible to maximise its value. Illustration 2: Rupa Company’s EBIT is Rs. 5,00,000. The company has 10%, 20 lakh debentures. The equity capitalization rate i.e. Ke is 16%. You are required to calculate: (i) Market value of equity and value of firm (ii) Overall cost of capital. Solution (i) Statement showing value of firm

Rs. Net operating income/EBIT 5,00,000 Less: Interest on debentures (10% of Rs. 20,00,000) 2,00,000 Earnings available for equity holders i.e. NI 3,00,000 Equity capitalisation rate (Ke) 16%

Market value of equity ⎟⎠⎞

⎜⎝⎛ ×== 100

16.003,00,000

KNI (S)

e

18,75,000

Market value of debt (D) 20,00,000 Total value of firm V = S + D 38,75,000

(ii) 12.90% 38,75,0005,00,000

firm of ValueEBIT capital ofcost Overall ===

(b) Net Operating Income Approach (NOI): NOI means earnings before interest and tax. According to this approach, capital structure decisions of the firm are irrelevant. Any change in the leverage will not lead to any change in the total value of the firm and the market price of shares, as the overall cost of capital is independent of the degree of leverage. As a result, the division between debt and equity is irrelevant. An increase in the use of debt which is apparently cheaper is offset by an increase in the equity capitalisation rate. This happens because equity investors seek higher compensation as they are opposed to greater risk due to

4.36

Page 696: 30510870 Cost Accounting and Financial Management

Financing Decisions

the existence of fixed return securities in the capital structure.

The above diagram shows that Ko (Overall capitalisation rate) and (debt – capitalisation rate) are constant and Ke (Cost of equity) increases with leverage. Illustration 3: Amita Ltd’s. operating income is Rs. 5,00,000. The firms cost of debt is 10% and currently firm employs Rs. 15,00,000 of debt. The overall cost of capital of the firm is 15%. You are required to determine: (i) Total value of the firm. (ii) Cost of equity. Solution (i) Statement showing value of the firm

Rs. Net operating income/EBIT 5,00,000 Less: Interest on debentures (10% of Rs. 15,00,000) 1,50,000 Earnings available for equityholders 3,50,000 Total cost of capital (K0) (given) 15%

4.37

Page 697: 30510870 Cost Accounting and Financial Management

Financial Management

Value of the firm0.15

5,00,000 k

EBIT V0

==

33,33,333

(ii) Calculation of cost of equity

equity(s) of Value holdersequity for available Earnings Ke =

Rs. Market value of debt (D) 15,00,000 Market value of equity (s) S = V − D = 33,33,333 – 15,00,000 18,33,333

equity of valueMarket ersequityhold for availabe Earnings )(Kequity ofCost e =

Or, equity of valueMarket

debt on paidInterest EBIT −=

18,33,3331,50,000 5,00,000 −

=

19.09% 18,33,333

3,50,000 Rs. ==

⎟⎠⎞

⎜⎝⎛+⎟

⎠⎞

⎜⎝⎛=

VD K

VS K K deo

⎟⎠⎞

⎜⎝⎛−⎟

⎠⎞

⎜⎝⎛=

SD K

SV K K doe

⎟⎠⎞

⎜⎝⎛−⎟

⎠⎞

⎜⎝⎛=

18,33,33315,00,000 0.10

18,33,33333,33,333 0.15

[ ]15,00,000) (0.10 33,33,333) (0.15 18,33,333

1 ×−×=

[ ]1,50,000 5,00,000 18,33,333

1 −= = 19.09%

4.38

Page 698: 30510870 Cost Accounting and Financial Management

Financing Decisions

(c) Modigliani-Miller Approach (MM): The NOI approach is definitional or conceptual and lacks behavioural significance. It does not provide operational justification for irrelevance of capital structure. However, Modigliani-Miller approach provides behavioural justification for constant overall cost of capital and, therefore, total value of the firm. The approach is based on further additional assumptions like: ♦ Capital markets are perfect. All information is freely available and there are no

transaction costs. ♦ All investors are rational. ♦ Firms can be grouped into ‘Equivalent risk classes’ on the basis of their business risk. ♦ Non-existence of corporate taxes. Based on the above assumptions, Modigliani-Miller derived the following three propositions. (i) Total market value of a firm is equal to its expected net operating income dividend by the

discount rate appropriate to its risk class decided by the market. (ii) The expected yield on equity is equal to the risk free rate plus a premium determined as

per the following equation: Kc = Ko + (Ko– Kd) B/S (iii) Average cost of capital is not affected by financial decision.

4.39

Page 699: 30510870 Cost Accounting and Financial Management

Financial Management It is evident from the above diagram that the average cost of the capital (Ko) is a constant and not affected by leverage. The operational justification of Modigliani-Miller hypothesis is explained through the functioning of the arbitrage process and substitution of corporate leverage by personal leverage. Arbitrage refers to buying asset or security at lower price in one market and selling it at higher price in another market. As a result equilibrium is attained in different markets. This is illustrated by taking two identical firms of which one has debt in the capital structure while the other does not. Investors of the firm whose value is higher will sell their shares and instead buy the shares of the firm whose value is lower. They will be able to earn the same return at lower outlay with the same perceived risk or lower risk. They would, therefore, be better off. The value of the levered firm can either be neither greater nor lower than that of an unlevered firm according this approach. The two must be equal. There is neither advantage nor disadvantage in using debt in the firm’s capital structure. Simply stated, the Modigliani Miller approach is based on the thought that no matter how the capital structure of a firm is divided among debt, equity and other claims, there is a conservation of investment value. Since the total investment value of a corporation depends upon its underlying profitability and risk, it is invariant with respect to relative changes in the firm’s financial capitalisation. The approach considers capital structure of a firm as a whole pie divided into equity, debt and other securities. According to MM, since the sum of the parts must equal the whole, therefore, regardless of the financing mix, the total value of the firm stays the same as shown in the figures below:

The shortcoming of this approach is that the arbitrage process as suggested by Modigliani-Miller will fail to work because of imperfections in capital market, existence of transaction cost and

4.40

Page 700: 30510870 Cost Accounting and Financial Management

Financing Decisions

presence of corporate income taxes. However in their 1963 article, they recognised that the value of the firm will increase or cost of capital will decrease where corporate taxes exist. As a result there will be some difference in the earnings of equity and debt-holders in an levered and unlevered firm and value of levered firm will be greater than the value of unlevered firm by an amount equal to amount of debt multiplied by corporate tax rate. Illustration 4: When value of levered firm is more than the value of unlevered firm There are two firms N and M, having same earnings before interest and taxes i.e. EBIT of Rs. 20,000. Firm M is levered company having a debt of Rs. 1,00,000 @ 7% rate of interest. The cost of equity of N company is 10% and of M company is 11.50%. Find out how arbitrage process will be carried on? Solution

Firms N M NOI/EBIT Rs. 20,000 Rs. 20,000 Debt − Rs. 1,00,000 Ke 10% 11.50% Kd − 7%

Value of equity (S) = equity ofCost

Interest NOI −

10%

000,20 SN = = Rs. 2,00,000

11.50%

7,000 20,000S M−

= = Rs. 1,13,043

VN = Rs. 2,00,000 VM = 1,13,043 + 1,00,000 {V = S + D} = Rs. 2,13,043 Assume you have 10% share of levered company. i.e. M. Therefore, investment in 10% of equity of levered company = 10% × 1,13,043 = Rs. 11,304.3 Return will be 10% of (20,000 – 7,000) = Rs. 1,400.

4.41

Page 701: 30510870 Cost Accounting and Financial Management

Financial Management Alternate Strategy will be:

Sell your 10% share of levered firm for Rs. 11,304.3 and borrow 10% of levered firms debt i.e. 10% of Rs. 1,00,000 and invest the money i.e. 10% in unlevered firms stock: Total resources /Money we have = 11,304.3 + 10,000 = 21,304.3 and you invest 10% of 2,00,000 = Rs. 20,000 Surplus cash available with you is = 21,304.3 – 20,000 = Rs. 1,304.3 Your return = 10% EBIT of unlevered firm – Interest to be paid on borrowed funds i.e. = 10% of Rs. 20,000 – 7% of Rs. 10,000 = 2,000 – 700 = Rs. 1,400 i.e. your return is same i.e. Rs. 1,400 which you are getting from ‘N’ company before investing in ‘M’ company. But still you have Rs. 1,304.3 excess money available with you. Hence, you are better off by doing arbitrage. Illustration 5: When value of unlevered firm is more than the value of levered firm There are two firms U and L having same NOI of Rs. 20,000 except that the firm L is a levered firm having a debt of Rs. 1,00,000 @ 7% and cost of equity of U & L are 10% and 18% respectively. Show how arbitrage process will work. Solution

Firms U L NOI/EBIT Rs. 20,000 Rs. 20,000 Debt capital − Rs. 1,00,000 Kd − 7% Ke 10% 18%

Value of equity capital (s) = K

Interest EBIT e

⎟⎟⎠

⎞⎜⎜⎝

⎛ −

=

0.10000,20

0.187,000 20,000 −

= Rs. 2,00,000 Rs. 72,222 Total value of the firm V = S + D

Rs. 2,00,000

Rs. 72,222 + 1,00,000

= Rs. 1,72,222

4.42

Page 702: 30510870 Cost Accounting and Financial Management

Financing Decisions

Assume you have 10% share of unlevered firm i.e. investment of 10% of Rs. 2,00,000 = Rs. 20,000 and Return @ 10% on Rs. 20,000. Investment will be 10% of earnings available for equity i.e. 10% × 20,000 = Rs. 2,000. Alternative strategy:

Sell your share in unlevered firm for Rs. 20,000 and buy 10% share of levered firm’s equity plus debt i.e. 10% equity of levered firm = 7,222 10% debt of levered firm = 10,000 Total investment = 17,222 Your resources are Rs. 20,000 Surplus cash available = Surplus – Investment = 20,000 – 17,222 = Rs. 2,778 Your return on investment is:

7% on debt of Rs. 10,000 700 10% on equity i.e. 10% of earnings available for equity holders i.e. (10% × 13,000) 1,300 Total return 2,000

i.e. in both the cases the return received is Rs. 2,000 and still you have excess cash of Rs. 2,778. Hence, you are better off i.e you will start selling unlevered company shares and buy levered company’s shares thereby pushing down the value of shares of unlevered firm and increasing the value of levered firm till equilibrium is reached. (d) Traditional Approach: This approach favours that as a result of financial leverage up to some point, cost of capital comes down and value of firm increases. However, beyond that point, reverse trends emerge. The principle implication of this approach is that the cost of capital is dependent on the capital structure and there is an optimal capital structure which minimises cost of capital. At the optimal capital structure, the real marginal cost of debt and equity is the same. Before the optimal point, the real marginal cost of debt is less than real marginal cost of equity and beyond this point the real marginal cost of debt is more than real marginal cost of equity.

4.43

Page 703: 30510870 Cost Accounting and Financial Management

Financial Management

The above diagram suggests that cost of capital is a function of leverage. It declines with Kd (debt) and starts rising. This means that there is a range of capital structure in which cost of capital is minimised. The net income approach argues that leverage always affects overall cost of capital and value of the firm. Optimum capital structure occurs at the point where value of the firm is highest and the cost of capital at the lowest. According to net operating income approach capital structure decisions are totally irrelevant. Modigliani-Miller supports the net operating income approach but provides behavioural justification. The traditional approach strikes a balance between these extremes. According to this approach the firm should strive to reach the optimal capital structure and its total valuation through a judicious use of the both debt and equity in capital structure. At the optimal capital structure the overall cost of capital will be minimum and the value of the firm is maximum. It further states that the value of the firm increases with financial leverage upto a certain point. Beyond this point the increase in financial leverage will increase its overall cost of capital and hence the value of firm will decline. This is because the benefits of use of debt may be so large that even after off setting the effect of increase in cost of equity, the overall cost of capital may still go down. However, if financial leverage increases beyond a acceptable limit the risk of debt investor may also increase, consequently cost of debt also starts increasing. The increasing cost of equity owing to increased financial risk and increasing cost of debt makes the overall cost of capital to increase. Illustration 6: Indra company has EBIT of Rs. 1,00,000. The company makes use of debt and equity capital. The firm has 10% debentures of Rs. 5,00,000 and the firm’s equity

4.44

Page 704: 30510870 Cost Accounting and Financial Management

Financing Decisions

capitalization rate is 15%. You are required to compute: (i) Current value of the firm (ii) Overall cost of capital.

Solution (i) Calculation of total value of the firm

Rs. EBIT 1,00,000 Less: Interest (@10% on Rs. 5,00,000) 50,000 Earnings available for equity holders 50,000 Equity capitalization rate i.e. Ke 15%

3,33,333 Rs. 0.15

50,000K

holdersequity for available Earningsholders equity of Valuee

==

=

Value of Debt (given) D 5,00,000 Total value of the firm V = D + S {5,00,000 + 3,33,333) 8,33,333

(ii) ⎟⎠⎞

⎜⎝⎛+⎟

⎠⎞

⎜⎝⎛==

VD K

VS K K capital ofcost Overall deo

⎟⎠

⎞⎜⎝

⎛+⎟⎠

⎞⎜⎝

⎛=8,33,3335,00,000 0.10

8,33,3333,33,333 0.15

[ ]0,0005 ,00005 8,33,333

1 +=

= 12.00%

4.45

Page 705: 30510870 Cost Accounting and Financial Management

Financial Management 2.8 CAPITAL STRUCTURE AND TAXATION The leverage irrelevance theorem of MM is valid if the perfect market assumptions underlying their analysis are satisfied. However, in the face of imperfections characterising the real world capital markets, the capital structure of a firm may affect its valuation. Presence of taxes is a major imperfection in the real world. This section examines the implications of corporate and personal taxes for the capital structure. 2.8.1 Corporate Taxes When taxes are applicable to corporate income, debt financing is advantageous. This is because dividends and retained earnings are not deductible for tax purposes; interest on debt is a tax-deductible expense. As a result, the total income available for both stockholders and debt-holders is greater when debt capital is used. Illustration 7: There are two firms Company A and B having net operating income of Rs. 15,00,000 each. Company B is a levered company whereas Company A is all equity company. Debt employed by Company B is of Rs. 7,00,000 @ 11%. The tax rate applicable to both the companies is 25%. Calculate earnings available for equity and debt for both the firms. Solution Statement of calculation of earnings available to equity holders and debt holders Company A B Net operating income 15,00,000 15,00,000 Less: Interest on Debt (11% of Rs. 7,00,000) − 77,000 Profit before taxes 15,00,000 14,23,000 Less: Tax @ 25% 3,75,000 3,55,750 Profit after tax/Earnings available in equity holders 11,25,000 10,67,250 Total earnings available to equity holders + Debt holders 11,25,000 10,67,250

+ 77,000 =11,44,250

As we can see that the earnings in case of Company B is more than the earnings of Company A because of tax shield available to shareholders of Company B due to the presence of debt structure in Company B. The interest is deducted from EBIT without tax deduction at the corporate level; equity holders also get their income after tax deduction due to which income of both the investors increase to the extent of tax saving on the interest paid i.e. tax shield i.e. 25% × 77,000 = 19,250 i.e. difference in the income of two companies’ earnings i.e. 11,44,250 – 11,25,000 = Rs. 19,250.

4.46

Page 706: 30510870 Cost Accounting and Financial Management

Financing Decisions

UNIT – III : BUSINESS RISK AND FINANCIAL RISK Learning Objectives

After studying this unit, you will be able to

♦ Define, discuss, and quantify “business risk” and “financial risk”;

♦ Define operating and financial leverage and identify causes of both;

♦ Define, calculate, and interpret a firm’s degree of operating, financial, and total leverage;

♦ Calculate a firm’s operating break-even (quantity) point and break-even (sales) point; and

♦ Understand what is involved in determining the appropriate amount of financial leverage for a firm?

3.1 INTRODUCTION

A firm can finance its operations through common and preference shares, with retained earnings, or with debt. Usually a firm uses a combination of these financing instruments.

The proportion of short and long-term debt is considered when analyzing capital structure. Capital structure refers to a firm's debt-to-equity ratio, which provides insight into how risky a company is. Usually a company more heavily financed by debt poses greater risk.

Firms can obtain their long-term financing from either debt or equity or some combination of debt and equity. Capital structure decisions by firms will have an effect on the expected profitability of the firm, the risks facing debt holders and shareholders, the probability of failure, the cost of capital and the market value of the firm.

Risk facing the common shareholders is a function of i.e. is affected by two types of risks, namely business risk and financial risk.

Risk Facing Common Shareholders = f {Business Risk, Financial Risk}

3.1.1 BUSINESS RISK AND FINANCIAL RISK

Business risk refers to the risk associated with the firm's operations. It is the uncertainty about the future operating income (EBIT), i.e. how well can the operating income be predicted? Business risk can be measured by the standard deviation of the Basic Earning Power ratio.

4.47

Page 707: 30510870 Cost Accounting and Financial Management

Financial Management

Financial risk refers to the additional risk placed on the firm's shareholders as a result of debt use i.e. the additional risk a shareholder bears when a company uses debt in addition to equity financing. Companies that issue more debt instruments would have higher financial risk than companies financed mostly or entirely by equity. Financial risk can be measured by ratios such as the firm's financial leverage multiplier, total debt to assets ratio or degree of financial leverage. A company's risk is composed of financial risk, which is linked to debt, and risk, which is often linked to economic climate. If a company is entirely financed by equity, it would pose almost no financial risk, but, it would be susceptible to business risk or changes in the overall economic climate. Leverage refers to the ability of a firm in employing long term funds having a fixed cost, to enhance returns to the owners. In other words, leverage is the amount of debt that a firm uses to finance its assets. The use of various financial instruments or borrowed capital, to increase the potential return of an investment. A firm with a lot of debt in its capital structure is said to be highly levered. A firm with no debt is said to be unlevered.

3.2 DEBT VERSUS EQUITY FINANCING Financing a business through borrowing is cheaper than using equity. This is because: ♦ Lenders require a lower rate of return than ordinary shareholders. Debt financial

securities present a lower risk than shares for the finance providers because they have prior claims on annual income and liquidation.

♦ A profitable business effectively pays less for debt capital than equity for another reason: the debt interest can be offset against pre-tax profits before the calculation of the

4.48

Page 708: 30510870 Cost Accounting and Financial Management

Financing Decisions

corporate tax, thus reducing the tax paid. ♦ Issuing and transaction costs associated with raising and servicing debt are generally

less than for ordinary shares. These are some benefits from financing a firm with debt. Still firms tend to avoid very high gearing levels. One reason is financial distress risk. This could be induced by the requirement to pay interest regardless of the cash flow of the business. If the firm goes through a rough period in its business activities it may have trouble paying its bondholders, bankers and other creditors their entitlement. The relationship between Expected return (Earnings per share) and the level of gearing can be represented as:

Relationship between leverage and risk

Leverage can occur in either the operating or financing portions of the income statement. The effect of leverage is to magnify the effects of changes in sales volume on earnings. Leverage serves to increase both expected return and risk to the firm's stockholders. Leverage helps both the investor and the firm to invest or operate. However, it comes with greater risk. If an investor uses leverage to make an investment and the investment moves against the investor, his or her loss is much greater than it would have been if the investment had not been leveraged - leverage magnifies both gains and losses. In the business world, a company can use leverage to try to generate shareholder wealth, but if it fails to do so, the interest expense and credit risk of default destroys shareholder value. Most companies use debt to finance operations. By doing so, a company increases its leverage because it can invest in business operations without increasing its equity. For example, if a company formed with an investment of Rs. 50 lakhs from investors, the equity in

4.49

Page 709: 30510870 Cost Accounting and Financial Management

Financial Management the company is Rs. 50 lakhs - this is the money the company uses to operate. If the company uses debt financing by borrowing Rs. 200 lakhs, the company now has Rs. 250 lakhs to invest in business operations and more opportunity to increase value for shareholders.

3.3 TYPES OF LEVERAGE The term Leverage in general refers to a relationship between two interrelated variables. In financial analysis it represents the influence of one financial variable over some other related financial variable. These financial variables may be costs, output, sales revenue, Earnings Before Interest and Tax (EBIT), Earning per share (EPS) etc. There are three commonly used measures of leverage in financial analysis. These are: (i) Operating Leverage (ii) Financial Leverage (iii) Combined Leverage

3.3.1 OPERATING LEVERAGE Operating leverage (OL) maybe defined as the employment of an asset with a fixed cost in the hope that sufficient revenue will be generated to cover all the fixed and variable costs. The use of assets for which a company pays a fixed cost is called operating leverage. With fixed costs the percentage change in profits accompanying a change in volume is greater than the percentage change in volume. The higher the turnover of operating assets, the greater will be the revenue in relation to the fixed charge on those assets.

Operating leverage is a function of three factors: (i) Rupee amount of fixed cost,

4.50

Page 710: 30510870 Cost Accounting and Financial Management

Financing Decisions

(ii) Variable contribution margin, and (iii) Volume of sales. Operating leverage is the ratio of net operating income before fixed charges to net operating income after fixed charges. Degree of operating leverage is equal to the percentage increase in the net operating income to the percentage increase in the output.

F)VP(N)VP(NOL−−

−=

Where, OL = Operating leverage N = Number of units sold

P = Selling price per unit V = Variable cost per unit F = Fixed cost

output in increase Percentageincome operatingnet in increase Percentage leverage operating of Degree =

Operating leverage is directly proportional to business risk. More operating leverage leads to more business risk, for then a small sales decline causes a big profit. This can be illustrated graphically as:

4.51

Page 711: 30510870 Cost Accounting and Financial Management

Financial Management Illustration 1: A Company produces and sells 10,000 shirts. The selling price per shirt is Rs. 500. Variable cost is Rs. 200 per shirt and fixed operating cost is Rs. 25,00,000. (a) Calculate operating leverage. (b) If sales are up by 10%, then what is the impact on EBIT? Solution (a) Statement of Profitability

Rs. Sales Revenue (10,000 × 500) 50,00,000 Less: Variable Cost (10,000 × 200) 20,00,000 Contribution 30,00,000 Less: Fixed Cost 25,00,000 EBIT 5,00,000

times 6 lakhs 5lakhs 30

EBITonContributi Leverage Operating ===

(b) sales in %EBIT in % OL

∆∆

=

50,00,0005,00,000

000,00,5/x 6 =

x = 30,000 ∴ ∆EBIT = 30,000/5,00,000

= 6%

3.3.2 FINANCIAL LEVERAGE Financial leverage (FL) maybe defined as ‘the use of funds with a fixed cost in order to increase earnings per share.’ In other words, it is the use of company funds on which it pays a limited return. Financial leverage involves the use of funds obtained at a fixed cost in the hope of increasing the return to common stockholders. Degree of financial leverage is the ratio of the percentage increase in earning per share (EPS) to the percentage increase in earnings before interest and taxes (EBIT).

(EBIT) tax andinterest before earnings in increase Percentage(EPS) share per earning in increase Percentage leverage financial of Degree =

4.52

Page 712: 30510870 Cost Accounting and Financial Management

Financing Decisions

IY

YFL−

=

Or, InterestEBIT

EBITFL−

=

Where, Y = EBIT at a point for which the degree of financial leverage is being calculated I = Amount of interest charges Illustration 2: Suppose there are two firms with the same operating leverage, business risk, and probability distribution of EBIT and only differ with respect to their use of debt (capital structure).

Firm U Firm L No debt Rs. 10,000 of 12% debt Rs. 20,000 in assets Rs. 20,000 in assets 40% tax rate 40% tax rate

Firm U: Unleveraged

Economy Bad Avg. Good Probability 0.25 0.50 0.25 EBIT Rs. 2,000 Rs. 3,000 Rs. 4,000 Interest 0 0 0EBIT Rs. 2000 Rs. 3,000 Rs. 4,000 Taxes (40%) 800 1,200 1,600NI Rs. 1,200 Rs. 1,800 Rs. 2,400

Firm L: Leveraged

Economy Bad Avg. Good Probability 0.25 0.50 0.25 EBIT Rs. 2,000 Rs. 3,000 Rs. 4,000 Interest 1,200 1,200 1,200EBIT Rs. 800 Rs. 1,800 Rs. 2,800

4.53

Page 713: 30510870 Cost Accounting and Financial Management

Financial Management

Taxes (40%) 320 720 1,120NI Rs. 480 Rs.1080 Rs. 1,680*Same as for Firm U.

Ratio comparison between leveraged and unleveraged firms

FIRM U Bad Avg. Good

BEP(=EBIT/TOTAL ASSETS) 10.0% 15.0% 20.0%

ROE(=PAT/NETWORTH) 6.0% 9.0% 12.0%

TIE(INTEREST COVERAGE RATIO (=EBIT/INTEREST)

∞ ∞ ∞

FIRM L Bad Avg. Good

BEP 10.0% 15.0% 20.0%

ROE 4.8% 10.8% 16.8%

TIE 1.67% 2.50% 3.30%

Risk and return for leveraged and unleveraged firms Expected Values:

Firm U Firm L E(BEP) 15.0% 15.0% E(ROE) 9.0% 10.8% E(TIE) ∞ 2.5x

Risk Measures: Firm U Firm L σROE 2.12% 4.24% CVROE 0.24 0.39

4.54

Page 714: 30510870 Cost Accounting and Financial Management

Financing Decisions

Thus, the effect of leverage on profitability and debt coverage can be seen from the above example. For leverage to raise expected ROE, BEP must be greater than kd i.e. BEP > kd because if kd > BEP, then the interest expense will be higher than the operating income produced by debt-financed assets, so leverage will depress income. As debt increases, TIE decreases because EBIT is unaffected by debt, and interest expense increases (Int Exp = kdD). Thus, it can be concluded that the basic earning power (BEP) is unaffected by financial leverage. Firm L has higher expected ROE because BEP > kd and it has much wider ROE (and EPS) swings because of fixed interest charges. Its higher expected return is accompanied by higher risk. 3.3.3 DEGREE OF COMBINED LEVERAGE Combined leverage maybe defined as the potential use of fixed costs, both operating and financial, which magnifies the effect of sales volume change on the earning per share of the firm. Degree of combined leverage (DCL) is the ratio of percentage change in earning per share to the percentage change in sales. It indicates the effect the sales changes will have on EPS.

leverage financial of Degree leverage operating of Degree leverage combined of Degree ×=

DFLDOLDCL ×= Where,

DCL = Degree of combined leverage DOL = Degree of operating leverage DFL = Degree of financial leverage

sales in change PercentageEPS in change Percentage leverage combined of Degree =

Illustration 3: A firm’s details are as under: Sales (@100 per unit) Rs. 24,00,000 Variable Cost 50% Fixed Cost Rs. 10,00,000 It has borrowed Rs. 10,00,000 @ 10% p.a. and its equity share capital is Rs. 10,00,000 (Rs. 100 each)

4.55

Page 715: 30510870 Cost Accounting and Financial Management

Financial Management Calculate: (a) Operating Leverage (b) Financial Leverage (c) Combined Leverage (d) Return on Investment (e) If the sales increases by Rs. 6,00,000; what will the new EBIT? Solution

Rs. Sales 24,00,000 Less: Variable cost 12,00,000 Contribution 12,00,000 Less: Fixed cost 10,00,000 EBIT 2,00,000 Less: Interest 1,00,000 EBT 1,00,000 Less: Tax (50%) 50,000 EAT 50,000 No. of equity shares 10,000 EPS 5

(a) Operating Leverage times 6 2,00,000

000,00,12 ==

(b) Financial Leverage times 2 1,00,000

000,00,2 ==

(c) Combined Leverage = OL × FL = 6 × 2 = 12 times.

(d) R.O.I 5% 100 10,00,000

000,50 =×=

(e) Operating Leverage = 6

.25EBIT∆ 6 =

4.56

Page 716: 30510870 Cost Accounting and Financial Management

Financing Decisions

1.5 4

1 6 EBIT ∆ =×

=

Increase in EBIT = Rs. 2,00,000 × 1.5 = Rs. 3,00,000 New EBIT = 5,00,000 Illustration 4: Betatronics Ltd. has the following balance sheet and income statement information:

Balance Sheet as on March 31st

Liabilities (Rs.) Assets (Rs.) Equity capital (Rs. 10 per share)

8,00,000 Net fixed assets 10,00,000

10% Debt 6,00,000 Current assets 9,00,000 Retained earnings 3,50,000 Current liabilities 1,50,000 19,00,000 19,00,000

Income Statement for the year ending March 31 (Rs.) Sales 3,40,000 Operating expenses (including Rs. 60,000 depreciation)

1,20,000

EBIT 2,20,000 Less: Interest 60,000Earnings before tax 1,60,000 Less: Taxes 56,000Net Earnings (EAT) 1,04,000

(a) Determine the degree of operating, financial and combined leverages at the current sales level, if all operating expenses, other than depreciation, are variable costs.

(b) If total assets remain at the same level, but sales (i) increase by 20 percent and (ii) decrease by 20 percent, what will be the earnings per share at the new sales level?

4.57

Page 717: 30510870 Cost Accounting and Financial Management

Financial Management Solution (a) Calculation of Degree of Operating (DOL), Financial (DFL) and Combined leverages (DCL).

000,20,2.Rs

000,60.Rs000,40,3.RsDOL −=

= 1.27

000,60,1.Rs000,20,2.RsDFL =

= 1.37 DCL = DOL×DFL = 1.27×1.37 = 1.75 (b) Earnings per share at the new sales level

Increase by 20% Decrease by 20% (Rs.) (Rs.) Sales level 4,08,000 2,72,000 Less: Variable expenses 72,000 48,000 Less: Fixed cost 60,000 60,000Earnings before interest and taxes 2,76,000 1,64,000 Less: Interest 60,000 60,000Earnings before taxes 2,16,000 1,04,000 Less: Taxes 75,600 36,400Earnings after taxes (EAT) 1,40,400 67,600 Number of equity shares 80,000 80,000 EPS 1.75 0.84

Working Notes: (i) Variable Costs = Rs. 60,000 (total cost − depreciation) (ii) Variable Costs at: (a) Sales level, Rs. 4,08,000 = Rs. 72,000 (b) Sales level, Rs. 2,72,000 = Rs. 48,000

4.58

Page 718: 30510870 Cost Accounting and Financial Management

Financing Decisions

Illustration 5: Calculate the operating leverage, financial leverage and combined leverage from the following data under Situation I and II and Financial Plan A and B:

Installed Capacity 4,000 units Actual Production and Sales 75% of the Capacity Selling Price Rs. 30 Per Unit Variable Cost Rs. 15 Per Unit

Fixed Cost: Under Situation I Rs. 15,000 Under Situation-II Rs.20,000

Capital Structure:

Financial Plan A B Rs. Rs. Equity 10,000 15,000 Debt (Rate of Interest at 20%) 10,000 5,000 20,000 20,000

Solution Operating Leverage: Situation-I Situation-II Rs. Rs. Sales (s) 90,000 90,000 3000 units @ Rs. 30/- per unit Less: Variable Cost (VC) @ Rs. 15 per unit 45,000 45,000Contribution (C) 45,000 45,000 Less: Fixed Cost (FC) 15,000 20,000Operating Profit (OP) 30,000 25,000(EBIT)

4.59

Page 719: 30510870 Cost Accounting and Financial Management

Financial Management (i) Operating Leverage

OPC = Rs.

30,00045,000 Rs.

25,00045,000

= 1.5 1.8 (ii) Financial Leverages

A (Rs.)

B (Rs.)

Situation 1 Operating Profit (EBIT) 30,000 30,000 Less: Interest on debt 2,000 1,000 PBT 28,000 29,000

Financial Leverage = PBTOP = 04.1

24,00030,000 Rs. 07.1

28,00030,000 Rs. ==

A

(Rs.) B

(Rs.) Situation-II Operating Profit (OP) 25,000 25,000 (EBIT) Less: Interest on debt 2,000 1,000PBT 23,000 24,000

Financial Leverage = PBTOP = 04.1

24,00025,000 Rs. 09.1

23,00025,000 Rs. ==

(iii) Combined Leverages A B (Rs.) (Rs.) (a) Situation I 1.5 x 1.07 =1.6 1.5 x 1.04 = 1.56 (b) Situation II 1.8 x 1.09 =1.96 1.8 x 1.04 =1.87

4.60

Page 720: 30510870 Cost Accounting and Financial Management

Financing Decisions

Self Examination Questions A. Objective Type Questions 1. A firm's cost of capital is the:

(a) Cost of borrowing money (b) Cost of issuing stock (c) Cost of bonds (d) Overall cost of financing to the firm.

2. The cost of debt financing is generally __________ the cost of preferred or common equity financing. (a) Less than (b) More than (c) Equal to (d) Not enough information to tell.

3. The cost of issuing new stock is called: (a) The cost of equity (b) Flotation costs (c) Marginal cost of capital (d) None of the above.

4. The cost of each component of a firm's capital structure multiplied by its weight in the capital structure is called the: (a) Marginal cost of capital (b) The cost of debt (c) Weighted average cost of capital (d) None of the above.

5. When establishing their optimal capital structure, firms should strive to: (a) Minimize the weighted average cost of capital (b) Minimize the amount of debt financing used (c) Maximize the marginal cost of capital (d) None of the above.

4.61

Page 721: 30510870 Cost Accounting and Financial Management

Financial Management 6. The ____________________ is the percentage change in operating income that results

from a percentage change in sales. (a) Degree of financial leverage (b) Breakeven point (c) Degree of operating leverage (d) Degree of combined leverage.

7. If interest expenses for a firm rise, we know that firm has taken on more ______________. (a) Financial leverage (b) Operating leverage (c) Fixed assets (d) None of the above.

8. The ________________ is the percentage change in earnings per share that results from a percentage change in operating income. (a) Degree of combined leverage (b) Degree of financial leverage (c) Breakeven point (d) Degree of operating leverage.

9. Combined leverage is the percentage change in relationship between sales and ____________. (a) Operating income (b) Operating leverage (c) Earnings per share (d) Breakeven point.

10. A highly leveraged firm is __________ risky than its peers. (a) Less (b) More (c) The same (d) None of the above.

4.62

Page 722: 30510870 Cost Accounting and Financial Management

Financing Decisions

11. An advantage of debt financing is: (a) Interest payments are tax deductible (b) The use of debt, up to a point, lowers the firm's cost of capital (c) Does not dilute owner's earnings (d) All of the above.

12. The cost of equity capital is all of the following EXCEPT: (a) The minimum rate that a firm should earn on the equity-financed part of an

investment (b) A return on the equity-financed portion of an investment that, at worst, leaves the

market price of the tock unchanged (c) By far the most difficult component cost to estimate (d) Generally lower than the before-tax cost of debt.

13. In calculating the costs of the individual components of a firm’s financing, the corporate tax rate is important to which of the following component cost formulae?

(a) Common stock (b) Debt (c) Preferred stock (d) None of the above. 14. Market values are often used in computing the weighted average cost of capital because (a) This is the simplest way to do the calculation (b) This is consistent with the goal of maximizing shareholder value (c) This is required in India by the SEBI (d) This is a very common mistake. 15. An EBIT-EPS indifference analysis chart is used for_______ (a) Evaluating the effects of business risk on EPS (b) Examining EPS results for alternative financing plans at varying EBIT levels (c) Determining the impact of a change in sales on EBIT (d) Showing the changes in EPS quality over time.

Answers to Objective Type Questions 1. (d); 2. (a); 3. (b); 4. (c); 5. (a); 6. (c); 7. (a); 8. (b); 9. (c); 10. (b); 11. (d); 12. (d); 13. (b); 14. (b); 15. (b)

4.63

Page 723: 30510870 Cost Accounting and Financial Management

Financial Management B. Short Answer Type Questions 1. Write short notes on:

(a) Weighted average cost of capital (b) Marginal cost of capital (c) Indifference point.

2. Differentiate between the following: (a) Business risk and Financial risk (b) Operating leverage and Financial leverage

(c) Net Income approach and Net Operating Income approach of Capital Structure.

C. Long Answer Type Questions

1. Explain briefly major considerations in capital structure planning. 2. Explain briefly the four approaches for determining the cost of equity shares. 3. Explain briefly, Modigliani and Miller approach on cost of capital. 4. Explain briefly the traditional theory of capital structure. 5. Define Operating Leverage and Financial Leverage. How the two leverages can be

measured? 6. Explain the impact of Financial Leverage on earning per share. 7. What is Combined Leverage? Explain its significance in financial planning of a firm. 8. Explain the advantages of equity financing. 9. What are the advantages of debt financing from the point of company and investors?

D. Practical Problems 1. (a) A company issues Rs. 10,00,000 16% debentures of Rs. 100 each. The company is

in 35% tax bracket. You are required to calculate the cost of debt after tax. If debentures are issued at (i) Par, (ii) 10% discount and (iii) 10% premium.

(b) If brokerage is paid at 2% what will be cost of debentures if issue is at par? 2. A company's share is quoted in market at Rs. 40 currently. A company pays a dividend of

Rs. 2 per share and investors expect a growth rate of 10% per year, compute: (a) The company’s cost of equity capital. (b) If anticipated growth rate is 11% p.a. calculate the indicated market price per share.

4.64

Page 724: 30510870 Cost Accounting and Financial Management

Financing Decisions

(c) If the company’s cost of capital is 16% and anticipated growth rate is 10% p.a., calculate the market price if dividend of Rs. 2 per share is to be maintained.

3. Three companies A, B & C are in the same type of business and hence have similar operating risks. However, the capital structure of each of them is different and the following are the details:

A B C Equity Share capital Rs. 4,00,000 2,50,000 5,00,000[Face value Rs. 10 per share] Market value per share Rs. 15 20 12Dividend per share Rs. 2.70 4 2.88Debentures Rs. Nil 1,00,000 2,50,000[Face value per debenture Rs. 100] Market value per debenture Rs. — 125 80Interest rate — 10% 8%

Assume that the current levels of dividends are generally expected to continue indefinitely and the income tax rate at 50%. You are required to compute the weighted average cost of capital of each company.

4. ZED Limited is presently financed entirely by equity shares. The current market value is Rs. 6,00,000. A dividend Rs. 1,20,000 has just been paid. This level of dividends is expected to be paid indefinitely. The company is thinking of investing in a new project involving a outlay of Rs. 5,00,000 now and is expected to generate net cash receipts of Rs. 1,05,000 per annum indefinitely. The project would be financed by issuing Rs. 5,00,000 debentures at the market interest rate of 18%.

Ignoring tax consideration: (1) Calculate the value of equity shares and the gain made by the shareholders if the

cost of equity rises to 21.6%. (2) Prove that weighted average cost of capital is not affected by gearing.

5. The following figures are made available to you: Net profits for the year 18,00,000 Less: Interest on secured debentures at 15% p.a.

4.65

Page 725: 30510870 Cost Accounting and Financial Management

Financial Management (Debentures were issued 3 months after the commencement of the year) 1,12,500 Profit before tax 16,87,500 Less: Income-tax at 35% and dividend distribution Tax 8,43,750 Profit after tax 8,43,750 Number of equity shares (Rs. 10 each) 1,00,000 Market quotation of equity share Rs. 109.70

The company has accumulated revenue reserves of Rs. 12 lakhs. The company is examining a project calling for an investment obligation of Rs. 10 lakhs. This investment is expected to earn the same rate as funds already employed. You are informed that a debt equity ratio (Debt divided by debt plus equity) higher than 60% will cause the price earning ratio to come down by 25% and the interest rate on additional borrowals will cost company 300 basis points more than on their current borrowal in secured. You are required to advise the company on the probable price of the equity share, if (a) The additional investment were to be raised by way loans; or (b) The additional investment were to be raised by way of equity.

6. The Modern Chemicals Ltd. requires Rs. 25,00,000 for a new plant. This plant is expected to yield earnings before interest and taxes of Rs. 5,00,000. While deciding about the financial plan, the company considers the objective of 2aximising earnings per share. It has three alternatives to finance the project—by raising debt of Rs. 2,50,000 or Rs. 10,00,000 or Rs. 15,00,000 and the balance, in each case, by issuing equity shares. The company's share is currently selling at Rs. 150, but is expected to decline to Rs. 125 in case the funds are borrowed in excess of Rs. 10,00,000. The funds can be borrowed at the rate of 10% upto Rs. 2,50,000, at 15% over Rs. 2,50,000 and upto Rs. 10,00,000 and at 20% over Rs. 10,00,000. The tax rate applicable to the company is 50%. Which form of financing should the company choose ?

7. A company provides the following figures: Rs. Profit 26,00,000

Less: Interest on debentures @ 12% 6,00,000 Profit before tax 20,00,000

4.66

Page 726: 30510870 Cost Accounting and Financial Management

Financing Decisions

Less: Income-tax @ 50% 10,00,000 Profit after tax 10,00,000

Number of equity shares (of Rs. 10 each) 4,00,000 Earning per share (EPS) 2.50 Ruling price in market 25 P/E (Price/Earning) Ratio 10

The company has undistributed reserves of Rs. 60,00,000. The company needs Rs. 20,00,000 for expansion; this amount will earn the same rate as funds already employed. You are informed that a debt equity ratio higher than 35% pulls the PE ratio down to 8 and raises the interest rate on additional amount borrowed at 14%. You are required to ascertain the probable price of the share if- (i) The additional funds are raised as a loans; or (ii) The amount is raised by issuing equity shares.

8. EXE Limited is considering three financing plans. The key information is as follows: (a) Total investment to be raised is Rs. 2,00,000 (b) Plans of Financing Proportion

Plan Equity Debt Preference Shares A 100% — — B 50% 50 — C 50% - 50%

(c) Cost of debt 8%; Cost of preference 8% (d) Tax rate 50%

(Assume no dividend tax) (e) Equity shares of face value of Rs. 10 each will be issued at a premium of Rs. 10 per

share. (g) Expected EBIT is Rs. 80,000 You are required to determine for each plan: (i) Earning per share

4.67

Page 727: 30510870 Cost Accounting and Financial Management

Financial Management

(ii) The Financial break-even point. (iii) Compute the EBIT range among the plans of indifference.

9. Calculate the Operating Leverage from the following data: Sales Rs. 50,000 Debt/Equity 3:1 Interest rate 12% Operating profit Rs. 20,00,000

10. Calculate the Degree of Operating Leverage, Degree of Financial Leverage and the Degree of Combined Leverage for the following firms and interpret the results:

P Q ROutput (unit) 3,00,000 75,000 5,00,000Fixed Costs (Rs.) 3,50,000 7,00,000 5,00,000Unit Variable costs (Rs) 1.00 7.50 0.10Interest expenses (Rs.) 25,000 40,000 –Unit Selling price (Rs.) 3.00 25.00 0.50

11. XYZ Ltd. sells 2000 units @ Rs. 10 per unit. The variable cost of production is Rs. 7 and

Fixed cost is Rs. 1,000. The company raised the required funds by issue of 100, 10% deben- tures @ Rs. 100 each and 2000 equity shares @ Rs. 10 per share. The sales of XYZ Ltd. are expected to increase by 20%. Assume tax rate of company is 50%. You are required to calculate the impact of increase in sales on earning per share.

12. The following figures relate to two companies:

(Rs. in lakhs) P Ltd. Q Ltd.Sales 500 1,000Less : Variable costs 200 300Contribution 300 700Less : Fixed costs 150 400EBIT 150 300Less : Interest 50 100Profit before tax (PBT) 100 200

4.68

Page 728: 30510870 Cost Accounting and Financial Management

Financing Decisions

You are required to : (i) Calculate the operating, financial and combined leverages for the two companies;

and (ii) Comment on the relative risk position of them.

13. The capital structure of ABC Ltd. consist of an ordinary share capital of Rs. 5,00,000 (equity shares of Rs. 100 each at par value) and Rs. 5,00,000 (10% debenture of Rs. 100 each). Sales increased from 50,000 units to 60,000 units, the selling price is Rs. 12 per unit, variable cost amounts to Rs. 8 per unit and fixed expenses amount to Rs. 1,00,000. The income tax rate is assumed to be 50%.

You are required to calculate the following: (a) The percentage increase in earning per share; (b) The degree of financial leverage at 50,000 units and 60,000 units; (c) The degree of operating leverage at 50,000 units and 60,000 units; (d) Comment on the behaviour E.P.S., operating and financial leverage in relation to

increases in sales from 50,000 units to 60,000 units. 14. A firm has sales of Rs. 75,00,000, variable cost of Rs. 42,00,000 and fixed cost of Rs.

6,00,000. It has a debt of Rs. 45,00,000 at 9% and equity of Rs. 55,00,000. (i) What is the firm’s R.O.I.? (ii) Does it have favourable financial leverage? (iii) What are the operating, financial and combined leverage of the firm ? (iv) If the sales drop to Rs. 50,00,000, what will be the new E.B.I.T. ?

15. Calculate the operating leverage, financial leverage and combined leverage from the following data under Situations I and II and Financial Plan A and B : Installed Capacity 4,000 units Actual Production and Sales 75% of the Capacity Selling Price Rs. 30 Per Unit Variable Cost Rs. 15 Per Unit Fixed Cost: Under Situation I Rs. 15,000 Under Situation II Rs. 20,000

4.69

Page 729: 30510870 Cost Accounting and Financial Management

Financial Management

Financial Plan A B Rs. Rs. Equity 10,000 15,000 Debt (Rate of Interest at 20%) 10,000 5,000 20,000 20,000

16. From the following prepare Income statement of Company A, B and C.

Company A B C

Financial Leverage 3 : 1 4 : 1 2 : 1Interest Rs. 200 Rs. 300 Rs. 1,000Operating Leverage 4 : 1 5 : 1 3 : 1Variable cost as a percentage to sales 66 2/3% 75% 50%sales Income tax rate 45% 45% 45%

4.70

Page 730: 30510870 Cost Accounting and Financial Management

CHAPTER 5

TYPES OF FINANCING

Learning Objectives

After studying this chapter, you will be able to

♦ Understand the different sources of finance available to a business; ♦ Differentiate between the various long term, medium term and short term sources of

finance; ♦ Understand the meaning and purpose of Venture Capital financing; ♦ Understand the meaning and purpose of securitisation and debt securitization; ♦ Understand the concept of lease financing; ♦ Understand the financing of export trade by banks; and ♦ Understand the various financial instruments dealt with in the International market.

1. INTRODUCTION

One of the most important consideration for an entrepreneur–company in implementing a new project or undertaking expansion, diversification, modernisation and rehabilitation scheme is ascertaining the cost of project and the means of finance. There are several sources of finance/funds available to any company. An effective appraisal mechanism of various sources of funds available to a company must be instituted in the company to achieve its main objectives. Such a mechanism is required to evaluate risk, tenure and cost of each and every source of fund. The selection of the fund source is dependent on the financial strategy pursued by the company, the leverage planned by the company, the financial conditions prevalent in the economy and the risk profile of both the company as well as the industry in which the company operates. Each and every source of fund has some advantages as well as disadvantages.

2. FINANCIAL NEEDS AND SOURCES OF FINANCE OF A BUSINESS

2.1 Financial Needs of a Business: Business enterprises need funds to meet their different

Page 731: 30510870 Cost Accounting and Financial Management

Financial Management types of requirements. All the financial needs of a business may be grouped into the following three categories:

(i) Long term financial needs: Such needs generally refer to those requirements of funds which are for a period exceeding 5-10 years. All investments in plant, machinery, land, buildings, etc., are considered as long term financial needs. Funds required to finance permanent or hard core working capital should also be procured from long term sources.

(ii) Medium term financial needs: Such requirements refer to those funds which are required for a period exceeding one year but not exceeding 5 years. For example, if a company resorts to extensive publicity and advertisement campaign then such type of expenses may be written off over a period of 3 to 5 years. These are called deferred revenue expenses and funds required for them are classified in the category of medium term financial needs. Sometimes long term requirements, for which long term funds cannot be arranged immediately may be met from medium term sources and thus the demand of medium term financial needs, are generated. As and when the desired long term funds are made available, medium term loans taken earlier may be paid off.

(iii) Short term financial needs: Such type of financial needs arise to finance in current assets such as stock, debtors, cash, etc. Investment in these assets is known as meeting of working capital requirements of the concern. Firms require working capital to employ fixed assets gainfully. The requirement of working capital depends upon a number of factors which may differ from industry to industry and from company to company in the same industry. The main characteristic of short term financial needs is that they arise for a short period of time not exceeding the accounting period. i.e., one year.

The basic principle for meeting the short term financial needs of a concern is that such needs should be met from short term sources, and for medium term financial needs from medium term sources and long term financial needs from long term sources. Accordingly, the method of raising funds is to be decided with reference to the period for which funds are required. Basically, there are two sources of raising funds for any business enterprise. viz., owner’s capital and borrowed capital. The owner’s capital is used for meeting long term financial needs and it primarily comes from share capital and retained earnings. Borrowed capital for all the other types of requirement can be raised from different sources such as debentures, public deposits, loans from financial institutions and commercial banks, etc.

The following section shows at a glance the different sources from where the three aforesaid types of finance can be raised in India.

5.2

Page 732: 30510870 Cost Accounting and Financial Management

Types of Financing

2.2 Sources of Finance of a Business

(i) Long-term

1. Share capital or Equity share

2. Preference shares

3. Retained earnings

4. Debentures/Bonds of different types

5. Loans from financial institutions

6. Loans from State Financial Corporation

7. Loans from commercial banks

8. Venture capital funding

9. Asset securitisation

10. International financing like Euro-issues, Foreign currency loans

(ii) Medium-term

1. Preference shares

2. Debentures/Bonds

3. Public deposits/fixed deposits for duration of three years

4. Commercial banks

5. Financial institutions

6. State financial corporations

7. Lease financing/Hire-Purchase financing

8. External commercial borrowings

9. Euro-issues

10. Foreign Currency bonds

(iii) Short-term

1. Trade credit

2. Accrued expenses and deferred income

5.3

Page 733: 30510870 Cost Accounting and Financial Management

Financial Management 3. Commercial banks

4. Fixed deposits for a period of 1 year or less

5. Advances received from customers

6. Various short-term provisions

It is evident from the above section that funds can be raised from the same source for meeting different types of financial requirements.

2.3 Financial sources of a business can also be classified as follows by using different basis :

1. According to period:

(i) Long term sources

(ii) Medium term sources

(iii) Short term sources

2. According to ownership:

(i) Owners capital or equity capital, retained earnings etc.

(ii) Borrowed capital such as debentures, public deposits, loans etc.

3. According to source of generation:

(i) Internal sources e.g. retained earnings and depreciation funds etc.

(ii) External sources e.g. debentures, loans etc.

However for the sake of convenience, the different sources of funds can also be classified into following categories.

(i) Security financing - financing through shares and debentures.

(ii) Internal financing - financing through retained earning, depreciation.

(iii) Loans financing - this includes both short term and long term loans.

(iv) International financing.

(v) Other sources.

3. LONG TERM SOURCES OF FINANCE

There are different sources of funds available to meet long term financial needs of the

5.4

Page 734: 30510870 Cost Accounting and Financial Management

Types of Financing

business. These sources may be broadly classified into share capital (both equity and preference) and debt (including debentures, long term borrowings or other debt instruments).

In recent times in India, many companies have raised long term finance by offering various instruments to public like deep discount bonds, fully convertible debentures etc. These new instruments have characteristics of both equity and debt and it is difficult to categorised these either as debt or equity.

The different sources of long term finance can now be discussed:

3.1 Owners Capital or Equity Capital : A public limited company may raise funds from promoters or from the investing public by way of owners capital or equity capital by issuing ordinary equity shares. Ordinary equity shares are a source of permanent capital. Ordinary shareholders are owners of the company and they undertake the risks of business. They are entitled to dividends after the income claims of other stakeholders are satisfied. Similarly, in the event of winding up, ordinary shareholders can exercise their claim on assets after the claims of the other suppliers of capital have been met. They elect the directors to run the company and have the optimum control over the management of the company. Since equity shares can be paid off only in the event of liquidation, this source has the least risk involved. This is more so due to the fact that equity shareholders can be paid dividends only when there are distributable profits. However, the cost of ordinary shares is usually the highest. This is due to the fact that such shareholders expect a higher rate of return on their investment as compared to other suppliers of long-term funds. Such behaviour is directly related to the risk undertaken by ordinary shareholders when compared to the providers of other forms of capital e.g. debt. Whereas, an ordinary shareholder shall take responsibility of losses incurred by the company by foregoing dividend or accepting a lesser amount, a debt holder shall be statutorily entitled to get regular payments as per the contract. Hence, when compared to those who have provided loan capital to the company, ordinary shareholders carry a higher amount of risk and so expect a higher return. Further, the dividend payable on shares is an appropriation of profits and not a charge against profits. This means that unlike debt, ordinary equity shares do not provide any tax shield to the company, thereby resulting in a higher cost.

Ordinary share capital also provides a security to other suppliers of funds. Thus, a company having substantial ordinary share capital may find it easier to raise further funds, in view of the fact that share capital provides a security to other suppliers of funds.

The Companies Act, 1956 and SEBI Guidelines for disclosure and investors' protections and the clarifications thereto lay down a number of provisions regarding the issue and management of equity shares capital.

5.5

Page 735: 30510870 Cost Accounting and Financial Management

Financial Management Advantages and disadvantages of raising funds by issue of equity shares are :

(i) It is a permanent source of finance. Since such shares are not redeemable, the company has no liability for cash outflows associated with its redemption.

(ii) Equity capital increases the company’s financial base and thus helps further the borrowing powers of the company.

(iii) The company is not obliged legally to pay dividends. Hence in times of uncertainties or when the company is not performing well, dividend payments can be reduced or even suspended.

(iv) The company can make further issue of share capital by making a right issue.

Apart from the above mentioned advantages, equity capital has some disadvantages to the company when compared with other sources of finance. These are as follows:

(i) The cost of ordinary shares is higher because dividends are not tax deductible and also the floatation costs of such issues are higher.

(ii) Investors find ordinary shares riskier because of uncertain dividend payments and capital gains.

(iii) The issue of new equity shares reduces the earning per share of the existing shareholders until and unless the profits are proportionately increased.

(iv) The issue of new equity shares can also reduce the ownership and control of the existing shareholders.

3.2 Preference Share Capital: These are a special kind of shares; the holders of such shares enjoy priority, both as regards to the payment of a fixed amount of dividend and repayment of capital on winding up of the company.

Long-term funds from preference shares can be raised through a public issue of shares. Such shares are normally cumulative, i.e., the dividend payable in a year of loss gets carried over to the next year till there are adequate profits to pay the cumulative dividends. The rate of dividend on preference shares is normally higher than the rate of interest on debentures, loans etc. Most of preference shares these days carry a stipulation of period and the funds have to be repaid at the end of a stipulated period.

Preference share capital is a hybrid form of financing which imbibes within itself some characteristics of equity capital and some attributes of debt capital. It is similar to equity because preference dividend, like equity dividend is not a tax deductible payment. It

5.6

Page 736: 30510870 Cost Accounting and Financial Management

Types of Financing

resembles debt capital because the rate of preference dividend is fixed. Typically, when preference dividend is skipped it is payable in future because of the cumulative feature associated with most of preference shares.

Cumulative Convertible Preference Shares (CCPs) may also be offered, under which the shares would carry a cumulative dividend of specified limit for a period of say three years after which the shares are converted into equity shares. These shares are attractive for projects with a long gestation period.

Preference share capital may be redeemed at a pre decided future date or at an earlier stage inter alia out of the profits of the company. This enables the promoters to withdraw their capital from the company which is now self-sufficient, and the withdrawn capital may be reinvested in other profitable ventures. It may be mentioned that irredeemable preference shares cannot be issued by any company.

Preference shares have gained importance after the Finance bill 1997 as dividends became tax exempted in the hands of the individual investor and are taxable in the hands of the

company as tax is imposed on distributed profits at a flat rate. At present, a domestic company paying dividend will have to pay dividend distribution tax @ 12.5% plus surcharge of 10% plus an education cess equalling 2% (total 14.025%).

Advantages and disadvantages of raising funds by issue of preference shares are:

(i) No dilution in EPS on enlarged capital base - If equity is issued it reduces EPS, thus affecting the market perception about the company.

(ii) There is leveraging advantage as it bears a fixed charge. Non payment of preference dividends does not force company into liquidity.

(iii) There is no risk of takeover as the preference shareholders do not have voting rights except in case where dividend arrears exist.

(iv) The preference dividends are fixed and pre decided. Hence Preference shareholders do not participate in surplus profits as the ordinary shareholders.

(v) Preference capital can be redeemed after a specified period.

The following are the disadvantages of the preference shares:

(i) One of the major disadvantages of preference shares is that preference dividend is not tax deductible and so does not provide a tax shield to the company. Hence a preference share is costlier to the company than debt e.g. debenture.

5.7

Page 737: 30510870 Cost Accounting and Financial Management

Financial Management (ii) Preference dividends are cumulative in nature. This means that although these dividends may be omitted, they shall need to be paid later. Also, if these dividends are not paid, no dividend can be paid to ordinary shareholders. The non payment of dividend to ordinary shareholders could seriously impair the reputation of the company concerned.

3.3 Retained Earnings: Long-term funds may also be provided by accumulating the profits of the company and by ploughing them back into business. Such funds belong to the ordinary shareholders and increase the net worth of the company. A public limited company must plough back a reasonable amount of profit every year keeping in view the legal requirements in this regard and its own expansion plans. Such funds also entail almost no risk. Further, control of present owners is also not diluted by retaining profits.

3.4 Debentures or Bonds: Loans can be raised from public by issuing debentures or bonds by public limited companies. Debentures are normally issued in different denominations ranging from Rs. 100 to Rs. 1,000 and carry different rates of interest. By issuing debentures, a company can raise long term loans from public. Normally, debentures are issued on the basis of a debenture trust deed which lists the terms and conditions on which the debentures are floated. Debentures are either secured or unsecured.

As compared with preference shares, debentures provide a more convenient mode of long- term funds. The cost of capital raised through debentures is quite low since the interest payable on debentures can be charged as an expense before tax. From the investors' point of view, debentures offer a more attractive prospect than the preference shares since interest on debentures is payable whether or not the company makes profits.

Debentures are thus instruments for raising long-term debt capital. Secured debentures are protected by a charge on the assets of the company. While the secured debentures of a well- established company may be attractive to investors, secured debentures of a new company do not normally evoke same interest in the investing public.

Debentures can be straight debentures or convertible debentures. A convertible debenture is the type which can be converted, either fully or partly, into shares after a specified period of time. Debentures can be divided into the following three categories:

(i) Non convertible debentures – These types of debentures do not have any feature of conversion and are repayable on maturity.

(ii) Fully convertible debentures – Such debentures are converted into equity shares as per the terms of issue in relation to price and the time of conversion. Interest rates on such debentures are generally less than the non convertible debentures because of their carrying the attractive feature of getting themselves converted into shares.

5.8

Page 738: 30510870 Cost Accounting and Financial Management

Types of Financing

(iii) Partly convertible debentures – Those debentures which carry features of a convertible and a non convertible debenture belong to this category. The investor has the advantage of having both the features in one debenture.

Advantages of raising finance by issue of debentures are:

(i) The cost of debentures is much lower than the cost of preference or equity capital as the interest is tax-deductible. Also, investors consider debenture investment safer than equity or preferred investment and, hence, may require a lower return on debenture investment.

(ii) Debenture financing does not result in dilution of control.

(iii) In a period of rising prices, debenture issue is advantageous. The fixed monetary outgo decreases in real terms as the price level increases.

The disadvantages of debenture financing are:

(i) Debenture interest and capital repayment are obligatory payments.

(ii) The protective covenants associated with a debenture issue may be restrictive.

(iii) Debenture financing enhances the financial risk associated with the firm.

(iv) Since debentures need to be paid during maturity, a large amount of cash outflow is needed at that time.

These days many companies are issuing convertible debentures or bonds with a number of schemes/incentives like warrants/options etc. These bonds or debentures are exchangeable at the option of the holder for ordinary shares under specified terms and conditions. Thus for the first few years these securities remain as debentures and later they can be converted into equity shares at a pre-determined conversion price. The issue of convertible debentures has distinct advantages from the point of view of the issuing company. Firstly, such an issue enables the management to raise equity capital indirectly without diluting the equity holding, until the capital raised has started earning an added return to support the additional shares. Secondly, such securities can be issued even when the equity market is not very good. Thirdly, convertible bonds are normally unsecured and, therefore, their issuance may ordinarily not impair the borrowing capacity. These debentures/bonds are issued subject to the SEBI guidelines notified from time to time.

Public issue of debentures and private placement to mutual funds now require that the issue be rated by a credit rating agency like CRISIL (Credit Rating and Information Services of India Ltd.). The credit rating is given after evaluating factors like track record of the company, profitability, debt servicing capacity, credit worthiness and the perceived risk of lending.

5.9

Page 739: 30510870 Cost Accounting and Financial Management

Financial Management 3.5 Loans from Financial Institutions: In India specialised institutions provide long- term financial assistance to industry. Thus, the Industrial Finance Corporation of India, the State Financial Corporations, the Life Insurance Corporation of India, the National Small Industries Corporation Limited, the Industrial Credit and Investment Corporation, the Industrial Development Bank of India, and the Industrial Reconstruction Corporation of India provide term loans to companies. Before a term loan is sanctioned, a company has to satisfy the concerned financial institution regarding the technical, commercial, economic, financial and managerial viability of the project for which the loan is required. Such loans are available at different rates of interest under different schemes of financial institutions and are to be repaid according to a stipulated repayment schedule. The loans in many cases stipulate a number of conditions regarding the management and certain other financial policies of the company.

Term loans represent secured borrowings and at present it is the most important source of finance for new projects. They generally carry a rate of interest inclusive of interest tax, depending on the credit rating of the borrower, the perceived risk of lending and the cost of funds. These loans are generally repayable over a period of 6 to 10 years in annual, semi- annual or quarterly instalments.

Term loans are also provided by banks, State financial/development institutions and all- India term lending financial institutions. Banks and State Financial Corporations normally provide term loans to projects in the small scale sector while for the medium and large industries term loans are provided by State developmental institutions alone or in consortium with banks and State financial corporations. For large scale projects All India financial institutions provide the bulk of term finance either singly or in consortium with other All India financial institutions, State level institutions and/or banks.

After Independence, the institutional set up in India for the provision of medium and long term credit for industry has been broadened. The assistance sanctioned and disbursed by these specialised institutions has increased impressively during the years.. A number of such specialised institutions have been established all over the country.

3.6 Loans from Commercial Banks: The primary role of the commercial banks is to cater to the short term requirements of industry. Of late, however, banks have started taking an interest in term financing of industries in several ways, though the formal term lending is, so far, small and is confined to major banks only.

Term lending by banks has become a controversial issue these days. It has been argued that term loans do not satisfy the canon of liquidity which is a major consideration in all bank

5.10

Page 740: 30510870 Cost Accounting and Financial Management

Types of Financing

operations. According to the traditional values, banks should provide loans only for short periods and for operations which result in the automatic liquidation of such credits over short periods. On the other hand, it is contended that the traditional concept of liquidity requires to be modified. The proceeds of the term loan are generally used for what are broadly known as fixed assets or for expansion in plant capacity. Their repayment is usually scheduled over a long period of time. The liquidity of such loans is said to depend on the anticipated income of the borrowers.

As a matter of fact, a working capital loan is more permanent and long term than a term loan. The reason for making this statement is that a term loan is always repayable on a fixed date and ultimately, a day will come when the account will be totally adjusted. However, in the case of working capital finance, though it is payable on demand, yet in actual practice it is noticed that the account is never adjusted as such; and, if at all the payment is asked back, it is with a clear purpose and intention of refinance being provided at the beginning of the next year or half year. To illustrate this point let us presume that two loans are granted on January 1, 2006 (a) to A; term loan of Rs. 60,000/- for 3 years to be paid back in equal half yearly instalments, and (b) to B : cash-credit limit against hypothecation, etc. of Rs. 60,000.

If we make two separate graphs for the two loans, they may appear to be like the figure shown below.

06 07 08 09 06 07 08 09

Note : It has been presumed that both the concerns are good. Payment of interest has been ignored. It has been presumed that cash credit limit is being enhanced gradually.

The above graphs clearly indicate that at the end of 2009 the term loan would be fully settled whereas the cash credit limit may have been enhanced to over a lakh of rupees. It really amounts to providing finances for long term.

This technique of providing long term finance can be technically called as “rolled over for periods exceeding more than one year”. Therefore, instead of indulging in term financing by the rolled over method, banks can and should extend credit term after a proper appraisal of

5.11

Page 741: 30510870 Cost Accounting and Financial Management

Financial Management applications for terms loans. In fact, as stated above, the degree of liquidity in the provision for regular amortisation of term loans is more than in some of these so called demand loans which are renewed from year to year. Actually, term financing disciplines both the banker and the borrower as long term planning is required to ensure that cash inflows would be adequate to meet the instruments of repayments and allow an active turnover of bank loans. The adoption of the formal term loan lending by commercial banks will not in any way hamper the criteria of liquidity and as a matter of fact, it will introduce flexibility in the operations of the banking system.

The real limitation to the scope of bank activities in this field is that all banks are not well equipped to make appraisal of such loan proposals. Term loan proposals involve an element of risk because of changes in the conditions affecting the borrower. The bank making such a loan, therefore, has to assess the situation to make a proper appraisal. The decision in such cases would depend on various factors affecting the conditions of the industry concerned and the earning potential of the borrower.

Bridge Finance: Bridge finance refers to loans taken by a company normally from commercial banks for a short period, pending disbursement of loans sanctioned by financial institutions. Normally, it takes time for financial institutions to disburse loans to companies. However, once the loans are approved by the term lending institutions, companies, in order not to lose further time in starting their projects, arrange short term loans from commercial banks. Bridge loans are also provided by financial institutions pending the signing of regular term loan agreement, which may be delayed due to non-compliance of conditions stipulated by the institutions while sanctioning the loan. The bridge loans are repaid/ adjusted out of the term loans as and when disbursed by the concerned institutions. Bridge loans are normally secured by hypothecating movable assets, personal guarantees and demand promissory notes. Generally, the rate of interest on bridge finance is higher as com- pared with that on term loans.

4. VENTURE CAPITAL FINANCING

The venture capital financing refers to financing of new high risky venture promoted by qualified entrepreneurs who lack experience and funds to give shape to their ideas. In broad sense, under venture capital financing venture capitalist make investment to purchase equity or debt securities from inexperienced entrepreneurs who undertake highly risky ventures with a potential of success.

4.1 Methods of Venture Capital Financing: In India , Venture Capital financing was first the responsibility of developmental financial institutions such as the Industrial Development Bank of India (IDBI) , the Technical Development and Information Corporation of India(now known

5.12

Page 742: 30510870 Cost Accounting and Financial Management

Types of Financing

as ICICI) and the State Finance Corporations(SFCs). In the year 1988, the Government of India took a policy initiative and announced guidelines for Venture Capital Funds (VCFs). In the same year, a Technology Development Fund (TDF) financed by the levy on all payments for technology imports was established This fund was meant to facilitate the financing of innovative and high risk technology programmes through the IDBI.

The guidelines mentioned above restricted the setting up of Venture Capital Funds by banks and financial institutions only. Subsequently guidelines were issued in the month of September 1995, for overseas investment in Venture Capital in India.

A major development in venture capital financing in India was in the year 1996 when the Securities and Exchange Board of India (SEBI) issued guidelines for venture capital funds to follow. These guidelines described a venture capital fund as a fund established in the form of a company or trust, which raises money through loans, donations, issue of securities or units and makes or proposes to make investments in accordance with the regulations. This move was instrumental in the entry of various foreign venture capital funds to enter India.. The guidelines were further amended in April 2000 with the objective of fuelling the growth of Venture Capital activities in India. A few venture capital companies operate as both investment and fund management companies; others set up funds and function as asset management companies.

It is hoped that the changes in the guidelines for the implementation of venture capital schemes in the country would encourage more funds to be set up to give the required momentum for venture capital investment in India.

Some common methods of venture capital financing are as follows:

(i) Equity financing : The venture capital undertakings generally requires funds for a longer period but may not be able to provide returns to the investors during the initial stages. Therefore, the venture capital finance is generally provided by way of equity share capital. The equity contribution of venture capital firm does not exceed 49% of the total equity capital of venture capital undertakings so that the effective control and ownership remains with the entrepreneur.

(ii) Conditional loan: A conditional loan is repayable in the form of a royalty after the venture is able to generate sales. No interest is paid on such loans. In India venture capital financiers charge royalty ranging between 2 and 15 per cent; actual rate depends on other factors of the venture such as gestation period, cash flow patterns, risk and other factors of the enterprise. Some Venture capital financiers give a choice to the enterprise of paying a high rate of interest (which could be well above 20 per cent) instead of royalty on sales once it becomes

5.13

Page 743: 30510870 Cost Accounting and Financial Management

Financial Management commercially sounds.

(iii) Income note: It is a hybrid security which combines the features of both conventional loan and conditional loan. The entrepreneur has to pay both interest and royalty on sales but at substantially low rates. IDBI's VCF provides funding equal to 80 – 87.50% of the projects cost for commercial application of indigenous technology.

(iv) Participating debenture: Such security carries charges in three phases — in the start up phase no interest is charged, next stage a low rate of interest is charged up to a particular level of operation, after that, a high rate of interest is required to be paid.

5. DEBT SECURITISATION

Securitisation is a financial transaction in which assets are pooled and securities representing interests in the pool are issued. The following example illustrates the process in a conceptual manner:

A finance company has issued a large number of car loans. It desires to raise further cash so as to be in a position to issue more loans. One way to achieve this goal is by selling all the existing loans, however, in the absence of a liquid secondary market for individual car loans, this may not be feasible. Instead, the company pools a large number of these loans and sells interest in the pool to investors. This process helps the company to raise finances and get the loans off its Balance Sheet. .These finances shall help the company disburse further loans. Similarly, the process is beneficial to the investors as it creates a liquid investment in a diversified pool of auto loans, which may be an attractive option to other fixed income instruments. The whole process is carried out in such a way, that the ultimate debtors- the car owners – may not be aware of the transaction. They shall continue making payments the way they were doing before, however, these payments shall reach the new investors instead of the company they (the car owners) had financed their car from.

The example provided above illustrates the general concept of securitisation as understood in common spoken English. Securitisation can take the form of ‘debt securitisation’ in which the underlying pool of assets (debt) is sold to a company or a trust for an immediate cash payment. The company which buys these pool of assets issues securities and utilises the regular cash flows arising out of the underlying pool of assets for servicing such issued securities. Thus securitisation follows a two way process, (1) the sale of an asset or a pool of assets to a company for immediate cash payment and (2) the repackaging and selling the security interests representing claims on incoming cash flows from the asset or pool of assets to third party investors by issuance of tradable securities.

5.14

Page 744: 30510870 Cost Accounting and Financial Management

Types of Financing

The company to which the underlying pool of assets or asset is sold is known as a ‘Special Purpose Vehicle’ (SPV) and the company which sells the underlying pool of assets or asset is known as the originator.

The process of securitisation is generally without recourse i.e. the investor bears the credit risk or risk of default and the issuer is under an obligation to pay to investors only if the cash flows are received by him from the collateral. The issuer however, has a right to legal recourse in the event of default. The risk run by the investor can be further reduced through credit enhancement facilities like insurance, letters of credit and guarantees.

In a simple pass through structure, the investor owns a proportionate share of the asset pool and cash flows when generated are passed on directly to the investor. This is done by issuing pass through certificates. In mortgage or asset backed bonds, the investor has a lien on the underlying asset pool. The SPV accumulates payments from the original borrowers from time to time and makes payments to investors at regular predetermined intervals. The SPV can invest the funds received in short term instruments and improve yield when there is time lag between receipt and payment.

In India, the Reserve Bank of India had issued draft guidelines on securitisation of standard assets in April 2005. These guidelines were applicable to banks, financial institutions and non banking financial companies. The guidelines were suitably modified and brought into effect from February 2006.

5.1 Benefits to the Originator

(i) The assets are shifted off the balance sheet, thus giving the originator recourse to off balance sheet funding.

(ii) It converts illiquid assets to liquid portfolio.

(iii) It facilitates better balance sheet management as assets are transferred off balance sheet facilitating satisfaction of capital adequacy norms.

(iv) The originator's credit rating enhances.

For the investor securitisation opens up new investment avenues. Though the investor bears the credit risk, the securities are tied up to definite assets.

As compared to factoring or bill discounting which largely solve the problems of short term trade financing, securitisation helps to convert a stream of cash receivables into a source of long term finance.

5.15

Page 745: 30510870 Cost Accounting and Financial Management

Financial Management 6. LEASE FINANCING

Leasing is a general contract between the owner and user of the asset over a specified period of time. The asset is purchased initially by the lessor (leasing company) and thereafter leased to the user (lessee company) which pays a specified rent at periodical intervals. Thus, leasing is an alternative to the purchase of an asset out of own or borrowed funds. Moreover, lease finance can be arranged much faster as compared to term loans from financial institutions.

In recent years, leasing has become a popular source of financing in India. From the lessee's point of view, leasing has the attraction of eliminating immediate cash outflow, and the lease rentals can be deducted for computing the total income under the Income tax Act. As against this, buying has the advantages of depreciation allowance (including additional depreciation) and interest on borrowed capital being tax-deductible. Thus, an evaluation of the two alternatives is to be made in order to take a decision. Practical problems for lease financing are covered at Final level in paper of Strategic Financial Management.

7. SHORT TERM SOURCES OF FINANCE

There are various sources available to meet short term needs of finance. The different sources are discussed below:

7.1 Trade Credit: It represents credit granted by suppliers of goods, etc., as an incident of sale. The usual duration of such credit is 15 to 90 days. It generates automatically in the course of business and is common to almost all business operations. It can be in the form of an 'open account' or 'bills payable'. Trade credit is preferred as a source of finance because it is without any explicit cost and till a business is a going concern it keeps on rotating. Another very important characteristic of trade credit is that it enhances automatically with the increase in the volume of business.

7.2. Accrued Expenses and Deferred Income: Accrued expenses represent liabilities which a company has to pay for the services which it has already received. Such expenses arise out of the day to day activities of the company and hence represent a spontaneous source of finance.

Deferred income, on the other hand, reflects the amount of funds received by a company in lieu of goods and services to be provided in the future. Since these receipts increase a company’s liquidity, they are also considered to be an important source of spontaneous finance.

5.16

Page 746: 30510870 Cost Accounting and Financial Management

Types of Financing

7.3 Advances from Customers: Manufacturers and contractors engaged in producing or constructing costly goods involving considerable length of manufacturing or construction time usually demand advance money from their customers at the time of accepting their orders for executing their contracts or supplying the goods. This is a cost free source of finance and really useful.

7.4. Commercial Paper: A Commercial Paper is an unsecured money market instrument issued in the form of a promissory note. The Reserve Bank of India introduced the commercial paper scheme in the year 1989 with a view to enabling highly rated corporate borrowers to diversify their sources of short term borrowings and to provide an additional instrument to investors. Subsequently, in addition to the Corporate, Primary Dealers and All India Financial Institutions have also been allowed to issue Commercial Papers. Commercial Papers can be issued for maturities between 15 days and a maximum up to one year from the date of issue. These can be issued in denominations of Rs 5 lakh or multiples thereof. All eligible issuers are required to get the credit rating from Credit Rating Information Services of India Ltd,(CRISIL), or the Investment Information and Credit Rating Agency of India Ltd (ICRA) or the Credit Analysis and Research Ltd (CARE) or the FITCH Ratings India Pvt Ltd or any such other credit rating agency as is specified by the Reserve Bank of India .Individuals, banking companies, corporate bodies incorporated in India, Non Resident Indians, Foreign Institutional Investors etc are allowed to invest in Commercial Paper, the minimum amount of such investment being Rs 5 lakhs.

7.5 Bank Advances: Banks receive deposits from public for different periods at varying rates of interest. These funds are invested and lent in such a manner that when required, they may be called back. Lending results in gross revenues out of which costs, such as interest on deposits, administrative costs, etc., are met and a reasonable profit is made. A bank's lending policy is not merely profit motivated but has to also keep in mind the socio- economic development of the country.

Bank advances are in the form of loan, overdraft, cash credit and bills purchased/discounted etc. Banks do not sanction advances on a long term basis beyond a small proportion of their demand and time liabilities. Advances are granted against tangible securities such as goods, shares, government promissory notes, Bills etc. In very rare cases, clean advances may also be allowed.

(i) Loans : In a loan account, the entire advance is disbursed at one time either in cash or by transfer to the current account of the borrower. It is a single advance. Except by way of interest and other charges no further adjustments are made in this account. Loan accounts are not running accounts like overdraft and cash credit accounts, repayment under the loan

5.17

Page 747: 30510870 Cost Accounting and Financial Management

Financial Management account may be the full amounts or by way of schedule of repayments agreed upon as in case of term loans. The securities may be shares, government securities, life insurance policies and fixed deposit receipts, etc.

(ii) Overdraft: Under this facility, customers are allowed to withdraw in excess of credit balance standing in their Current Deposit Account. A fixed limit is therefore granted to the borrower within which the borrower is allowed to overdraw his account. Opening of an overdraft account requires that a current account will have to be formally opened. Though overdrafts are repayable on demand, they generally continue for long periods by annual renewals of the limits. This is a convenient arrangement for the borrower as he is in a position to avail of the limit sanctioned, according to his requirements. Interest is charged on daily balances. Since these accounts are operative like cash credit and current accounts, cheque books are provided. As in the case of a loan account the security in an overdraft account may be shares, debentures and Government securities. In special cases, life insurance policies and fixed deposit receipts are also accepted.

(iii) Clean Overdrafts: Request for clean advances are entertained only from parties which are financially sound and reputed for their integrity. The bank has to rely upon the personal security of the borrowers. Therefore, while entertaining proposals for clean advances; banks exercise a good deal of restraint since they have no backing of any tangible security. If the parties are already enjoying secured advance facilities, this may be a point in favour and may be taken into account while screening such proposals. The turnover in the account, satisfactory dealings for considerable period and reputation in the market are some of the factors which the bank will normally see. As a safeguard, banks take guarantees from other persons who are credit worthy before granting this facility. A clean advance is generally granted for a short period and must not be continued for long.

(iv) Cash Credits: Cash Credit is an arrangement under which a customer is allowed an advance up to certain limit against credit granted by bank. Under this arrangement, a customer need not borrow the entire amount of advance at one time; he can only draw to the extent of his requirements and deposit his surplus funds in his account. Interest is not charged on the full amount of the advance but on the amount actually availed of by him. Generally cash credit limits are sanctioned against the security of goods by way of pledge or hypothecation. The borrower can also provide alternative security of goods by way of pledge or hypothecation. Though these accounts are repayable on demand, banks usually do not recall such advances, unless they are compelled to do so by adverse factors. Hypothecation is an equitable charge on movable goods for an amount of debt where neither possession nor ownership is passed on to the creditor. In case of pledge, the borrower delivers the goods to

5.18

Page 748: 30510870 Cost Accounting and Financial Management

Types of Financing

the creditor as security for repayment of debt. Since the banker, as creditor, is in possession of the goods, he is fully secured and in case of emergency he can fall back on the goods for realisation of his advance under proper notice to the borrower.

(v) Advances against goods : Advances against goods occupy an important place in total bank credit. Goods are security have certain distinct advantages. They provide a reliable source of repayment. Advances against them are safe and liquid. Also, there is a quick turnover in goods, as they are in constant demand. So a banker accepts them as security. Generally goods are charged to the bank either by way of pledge or by way of hypothecation. The term 'goods' includes all forms of movables which are offered to the bank as security. They may be agricultural commodities or industrial raw materials or partly finished goods.

For the purpose of calculation of the drawing limits, valuation of the goods is made from time to time. In case of hypothecation advance, an undertaking is obtained from the borrower that the goods are not charged to some other bank. The bank also takes periodical statements of stocks regarding quantity valuation etc.

The Reserve Bank of India issues directives from time to time imposing restrictions on advances against certain commodities. It is obligatory on banks to follow these directives in letter and spirit. The directives also sometimes stipulate changes in the margin.

(vi) Bills Purchased/Discounted : These advances are allowed against the security of bills which may be clean or documentary. Bills are sometimes purchased from approved customers in whose favour limits are sanctioned. Before granting a limit the banker satisfies himself as to the credit worthiness of the drawer. Although the term 'bills purchased' gives the impression that the bank becomes the owner or purchaser of such bills, in actual practice the bank holds the bills only as security for the advance. The bank, in addition to the rights against the parties liable on the bills, can also exercise a pledge’s rights over the goods covered by the documents.

Usance bills maturing at a future date or sight are discounted by the banks for approved parties. When a bill is discounted, the borrower is paid the present worth. The bankers, however, collect the full amounts on maturity. The difference between these two amounts represents earnings of the bankers for the period. This item of income is called 'discount'.

Sometimes, overdraft or cash credit limits are allowed against the security of bills. A suitable margin is usually maintained. Here the bill is not a primary security but only a collateral security. The banker in the case, does not become a party to the bill, but merely collects it as an agent for its customer.

5.19

Page 749: 30510870 Cost Accounting and Financial Management

Financial Management When a banker purchases or discounts a bill, he advances against the bill; he has therefore to be very cautious and grant such facilities only to those customers who are creditworthy and have established a steady relationship with the bank. Credit reports are also compiled on the drawees.

(vii) Advance against documents of title to goods: A document becomes a document of title to goods when its possession is recognised by law or business custom as possession of the goods. These documents include a bill of lading, dock warehouse keeper's certificate, railway receipt, etc. A person in possession of a document to goods can by endorsement or delivery (or both) of document, enable another person to take delivery of the goods in his right. An advance against the pledge of such documents is equivalent to an advance against the pledge of goods themselves.

(viii) Advance against supply of bills: Advances against bills for supply of goods to government or semi-government departments against firm orders after acceptance of tender fall under this category. The other type of bills which also come under this category are bills from contractors for work executed either wholly or partially under firm contracts entered into with the above mentioned Government agencies.

These bills are clean bills without being accompanied by any document of title of goods. But they evidence supply of goods directly to Governmental agencies. Sometimes these bills may be accompanied by inspection notes from representatives of government agencies for having inspected the goods before they are despatched. If bills are without the inspection report, banks like to examine them with the accepted tender or contract for verifying that the goods supplied under the bills strictly conform to the terms and conditions in the acceptance tender.

These supply bills represent debt in favour of suppliers/contractors, for the goods supplied to the government bodies or work executed under contract from the Government bodies. It is this debt that is assigned to the bank by endorsement of supply bills and executing irrevocable power of attorney in favour of the banks for receiving the amount of supply bills from the Government departments. The power of attorney has got to be registered with the Government department concerned. The banks also take separate letter from the suppliers / contractors instructing the Government body to pay the amount of bills direct to the bank.

Supply bills do not enjoy the legal status of negotiable instruments because they are not bills of exchange. The security available to a banker is by way of assignment of debts represented by the supply bills.

5.20

Page 750: 30510870 Cost Accounting and Financial Management

Types of Financing

(ix) Term Loans by banks: Term loans are an instalment credit repayable over a period of time in monthly/quarterly/half-yearly or yearly instalment. Banks grant term loans for small projects falling under priority sector, small scale sector and big units. Banks have now been permitted to sanction term loan for projects as well without association of financial institutions. The banks grant loans for periods which normally range from 3 to 7 years and some- times even more. These loans are granted on the security of fixed assets.

7.6 Financing of Export Trade by Banks: Exports play an important role in accelerating the economic growth of developing countries like India. Of the several factors influencing export growth, credit is a very important factor which enables exporters in efficiently executing their export orders. The commercial banks provide short term export finance mainly by way of pre and post-shipment credit. Export finance is granted in Rupees as well as in foreign currency.

In view of the importance of export credit in maintaining the pace of export growth, RBI has initiated several measures in the recent years to ensure timely and hassle free flow of credit to the export sector. These measures, inter alia, include rationalization and liberalization of export credit interest rates, flexibility in repayment/prepayment of pre-shipment credit, special financial package for large value exporters, export finance for agricultural exports, Gold Card Scheme for exporters etc. Further, banks have been granted freedom by RBI to source funds from abroad without any limit for exclusively for the purpose of granting export credit in foreign currency, which has enabled banks to increase their lending’s under export credit in foreign currency substantially during the last few years.

The advances by commercial banks for export financing are in the form of:

(i) Pre-shipment finance i.e., before shipment of goods.

(ii) Post-shipment finance i.e., after shipment of goods.

7.6.1 Pre-Shipment Finance: This generally takes the form of packing credit facility; packing credit is an advance extended by banks to an exporter for the purpose of buying, manufacturing, processing, packing, shipping goods to overseas buyers. Any exporter, having at hand a firm export order placed with him by his foreign buyer or an irrevocable letter of credit opened in his favour, can approach a bank for availing of packing credit. An advance so taken by an exporter is required to be liquidated within 180 days from the date of its commencement by negotiation of export bills or receipt of export proceeds in an approved manner. Thus packing credit is essentially a short term advance.

Normally, banks insist upon their customers to lodge with them irrevocable letters of credit opened in favour of the customers by the overseas buyers. The letter of credit and firm sale contracts not only serve as evidence of a definite arrangement for realisation of the export

5.21

Page 751: 30510870 Cost Accounting and Financial Management

Financial Management proceeds but also indicate the amount of finance required by the exporter. Packing credit, in the case of customers of long standing, may also be granted against firm contracts entered into by them with overseas buyers.

7.6.1.1 Types of Packing Credit

(a) Clean packing credit : This is an advance made available to an exporter only on production of a firm export order or a letter of credit without exercising any charge or control over raw material or finished goods. It is a clean type of export advance. Each proposal is weighed according to particular requirements of the trade and credit worthiness of the exporter. A suitable margin has to be maintained. Also, Export Credit Guarantee Corporation (ECGC) cover should be obtained by the bank.

(b) Packing credit against hypothecation of goods: Export finance is made available on certain terms and conditions where the exporter has pledge able interest and the goods are hypothecated to the bank as security with stipulated margin. At the time of utilising the advance, the exporter is required to submit, along with the firm export order or letter of credit relative stock statements and thereafter continue submitting them every fortnight and/or whenever there is any movement in stocks.

(c) Packing credit against pledge of goods: Export finance is made available on certain terms and conditions where the exportable finished goods are pledged to the banks with approved clearing agents who will ship the same from time to time as required by the exporter. The possession of the goods so pledged lies with the bank and is kept under its lock and key.

(d) E.C.G.C. guarantee: Any loan given to an exporter for the manufacture, processing, purchasing, or packing of goods meant for export against a firm order qualifies for the packing. credit guarantee issued by Export Credit Guarantee Corporation (ECGC).

(e) Forward exchange contract: Another requirement of packing credit facility is that if the export bill is to be drawn in a foreign currency, the exporter should enter into a forward exchange contact with the bank, thereby avoiding risk involved in a possible change in the rate of exchange.

Documents required : In case of partnership firms, banks usually require the following documents:

(i) Joint and several demand promote signed on behalf of the firm as well as by the partners individually.

(ii) Letter of continuity (signed on behalf of the firm and partners individually).

5.22

Page 752: 30510870 Cost Accounting and Financial Management

Types of Financing

(iii) Letter of Pledge to secure demand cash credit against goods (in case of pledge) OR Agreement of Hypothecation to secure demand cash credit (in case of hypothecation).

(iv) Letter of Authority to operate the account.

(v) Declaration of Partnership.

(In case of sole traders, sole proprietorship declaration).

(vi) Agreement to utilise the monies drawn in terms of contract.

(vii) Letter of Hypothecation (for bills).

In case of limited companies banks usually require the following documents :

(i) Demand Pro-note

(ii) Letter of continuity.

(iii) Agreement of hypothecation or Letter of pledge signed on behalf of the company.

(iv) General guarantee of the directors of the company in their joint and several personal capacities.

(v) Certified copy of the board of directors' resolution.

(vi) Agreement to utilise the monies drawn in terms of contract should bear the seal of the company.

(vii) Letter of Hypothecation (for bills).

7.6.2 Post-shipment Finance: It takes the following forms:

(a) Purchase/discounting of documentary export bills : Finance is provided to exporters by purchasing export bills drawn payable at sight or by discounting usance export bills covering confirmed sales and backed by documents including documents of the title of goods such as bill of lading, post parcel receipts, or air consignment notes.

Documents to be obtained:

(i) Letter of hypothecation covering the goods; and

(ii) General guarantee of directors or partners of the firm as the case may be.

(b) E.C.G.C. Guarantee: Post-shipment finance, given to an exporter by a bank through purchase, negotiation or discount of an export bill against an order, qualifies for post-shipment export credit guarantee. It is necessary, however, that exporters should obtain a shipment or

5.23

Page 753: 30510870 Cost Accounting and Financial Management

Financial Management contracts risk policy of E.C.G.C. Banks insist on the exporters to take a contracts shipments (comprehensive risks) policy covering both political and commercial risks. The Corporation, on acceptance of the policy, will fix credit limits for individual exporters and the Corporation’s liability will be limited to the extent of the limit so fixed for the exporter concerned irrespective of the amount of the policy.

(c) Advance against export bills sent for collection : Finance is provided by banks to exporters by way of advance against export bills forwarded through them for collection, taking into account the creditworthiness of the party, nature of goods exported, usance, standing of drawee, etc. appropriate margin is kept.

Documents to be obtained :

(i) Demand promissory note.

(ii) Letter of continuity.

(iii) Letter of hypothecation covering bills.

(iv) General Guarantee of directors or partners of the firm (as the case may be).

(d) Advance against duty draw backs, cash subsidy, etc.: To finance export losses sustained by exporters, bank advance against duty draw-back, cash subsidy, etc., receivable by them against export performance. Such advances are of clean nature; hence necessary precaution should be exercised.

Conditions : Bank providing finance in this manner see that the relative export bills are either negotiated or forwarded for collection through it so that it is in a position to verify the exporter's claims for duty draw-backs, cash subsidy, etc. 'An advance so availed of by an exporter is required to be liquidated within 180 days from the date of shipment of relative goods.

Documents to be obtained:

(i) Demand promissory note.

(ii) Letter of continuity.

(iii) General Guarantee of directors of partners of the firm as the case may be.

(iv) Undertaking from the borrowers that they will deposit the cheques/payments received from the appropriate authorities immediately with the bank and will not utilise such amounts in any other way.

5.24

Page 754: 30510870 Cost Accounting and Financial Management

Types of Financing

Other facilities extended to exporters:

(i) On behalf of approved exporters, banks establish letters of credit on their overseas or up country suppliers.

(ii) Guarantees for waiver of excise duty, etc. due performance of contracts, bond in lieu of cash security deposit, guarantees for advance payments etc., are also issued by banks to approved clients.

(iii) To approved clients undertaking exports on deferred payment terms, banks also pro- vide finance.

(iv) Banks also endeavour to secure for their exporter-customers status reports of their buyers and trade information on various commodities through their correspondents.

(v) Economic intelligence on various countries is also provided by banks to their ex- porter clients.

7.7 Inter Corporate Deposits: The companies can borrow funds for a short period say 6 months from other companies which have surplus liquidity. The rate of interest on inter corporate deposits varies depending upon the amount involved and time period.

7.8 Certificate of Deposit (CD): The certificate of deposit is a document of title similar to a time deposit receipt issued by a bank except that there is no prescribed interest rate on such funds.

The main advantage of CD is that banker is not required to encash the deposit before maturity period and the investor is assured of liquidity because he can sell the CD in secondary market.

7.9 Public Deposits: Public deposits are very important source of short-term and medium term finances particularly due to credit squeeze by the Reserve Bank of India. A company can accept public deposits subject to the stipulations of Reserve Bank of India from time to time maximum up to 35 per cent of its paid up capital and reserves, from the public and shareholders. These deposits may be accepted for a period of six months to three years. Public deposits are unsecured loans; they should not be used for acquiring fixed assets since they are to be repaid within a period of 3 years. These are mainly used to finance working capital requirements.

8. OTHER SOURCES OF FINANCING

8.1 Seed Capital Assistance: The Seed capital assistance scheme is designed by IDBI for

5.25

Page 755: 30510870 Cost Accounting and Financial Management

Financial Management professionally or technically qualified entrepreneurs and/or persons possessing relevant experience, skills and entrepreneurial traits. All the projects eligible for financial assistance from IDBI, directly or indirectly through refinance are eligible under the scheme. The project cost should not exceed Rs. 2 crores and the maximum assistance under the project will be restricted to 50% of the required promoter's contribution or Rs. 15 lacs whichever is lower.

The Seed Capital Assistance is interest free but carries a service charge of one per cent per annum for the first five years and at increasing rate thereafter. However, IDBI will have the option to charge interest at such rate as may be determined by IDBI on the loan if the financial position and profitability of the company so permits during the currency of the loan. The repayment schedule is fixed depending upon the repaying capacity of the unit with an initial moratorium upto five years.

For projects with a project cost exceeding Rs. 200 lacs, seed capital may be obtained from the Risk Capital and Technology Corporation Ltd. (RCTC) For small projects costing upto Rs. 5 lacs, assistance under the National Equity Fund of the SIDBI may be availed.

8.2 Risk Capital Foundation Scheme: The Risk Capital Foundation Scheme is an autonomous foundation set up and funded by IFCI. It assists promoters of projects costing between Rs 2 crores and Rs 15 crore. The ceiling on the assistance varies between Rs 15 lakhs and Rs 40 lakhs depending on the number of applicant promoters.

8.3 Internal Cash Accruals: Existing profit making companies which undertake an expansion/ diversification programme may be permitted to invest a part of their accumulated reserves or cash profits for creation of capital assets. In such cases, past performance of the company permits the capital expenditure from within the company by way of disinvestment of working/invested funds. In other words, the surplus generated from operations, after meeting all the contractual, statutory and working requirement of funds, is available for further capital expenditure.

8.4 Unsecured Loans: Unsecured loans are typically provided by promoters to meet the promoters' contribution norm. These loans are subordinate to institutional loans. The rate of interest chargeable on these loans should be less than or equal to the rate of interest on institutional loans and interest can be paid only after payment of institutional dues. These loans cannot be repaid without the prior approval of financial institutions. Unsecured loans are considered as part of the equity for the purpose of calculating of debt equity ratio.

8.5 Deferred Payment Guarantee: Many a time suppliers of machinery provide deferred credit facility under which payment for the purchase of machinery can be made over a period of time. The entire cost of the machinery is financed and the company is not required

5.26

Page 756: 30510870 Cost Accounting and Financial Management

Types of Financing

to contribute any amount initially towards acquisition of the machinery. Normally, the supplier of machinery insists that bank guarantee should be furnished by the buyer. Such a facility does not have a moratorium period for repayment. Hence, it is advisable only for an existing profit making company.

8.6 Capital Incentives: The backward area development incentives available often determine the location of a new industrial unit. These incentives usually consist of a lump sum subsidy and exemption from or deferment of sales tax and octroi duty. The quantum of incentives is determined by the degree of backwardness of the location.

The special capital incentive in the form of a lump sum subsidy is a quantum sanctioned by the implementing agency as a percentage of the fixed capital investment subject to an overall ceiling. This amount forms a part of the long-term means of finance for the project. However, it may be mentioned that the viability of the project must not be dependent on the quantum and availability of incentives. Institutions, while appraising the project, assess the viability of the project per se, without considering the impact of incentives on the cash flows and profitability of the project.

Special capital incentives are sanctioned and released to the units only after they have complied with the requirements of the relevant scheme. The requirements may be classified into initial effective steps and final effective steps. The initial effective steps include formation of the firm/company, acquisition of land in the backward area and registration for manufacture of the products. The final effective steps include obtaining clearances under FEMA, capital goods clearance/import licence, conversion of Letter of Intent to Industrial License, tie up of the means of finance, all clearances required for the setting up of the unit, aggregate expenditure incurred for the project should exceed 25% of the project cost and at least 10% of the fixed assets should have been created/acquired site.

The release of special capital incentives by the concerned State Government generally takes one to two years. The promoters therefore find it convenient to avail bridge finance against the capital incentives. Provision for the same should be made in the pre-operative expenses considered in the project cost. Further, as the bridge finance may be available to the extent of 85%, the balance 15% may have to be brought in by the promoters from their own resources.

9. NEW INSTRUMENTS

The new instruments that have been introduced since early 90’s as a source of finance is staggering in their nature and diversity. These new instruments are as follows:

9.1 Deep Discount Bonds: Deep Discount Bonds is a form of zero-interest bonds. These

5.27

Page 757: 30510870 Cost Accounting and Financial Management

Financial Management bonds are sold at a discounted value and on maturity face value is paid to the investors. In such bonds, there is no interest payout during lock in period.

IDBI was the first to issue a deep discount bond in India in January, 1992. The bond of a face value of Rs. 1 lakh was sold for Rs. 2,700 with a maturity period of 25 years. The investor could hold the bond for 25 years or seek redemption at the end of every five years with a specified maturity value as shown below.

Holding Period (years) 5 10 15 20 25Maturity value (Rs.) 5,700 12,000 25,000 50,000 1,00,000

Annual rate of interest (%) 16.12 16.09 15.99 15.71 15.54

The investor can sell the bonds in stock market and realise the difference between face value (Rs. 2,700) and market price as capital gain.

9.2 Secured Premium Notes: Secured Premium Notes is issued along with a detachable warrant and is redeemable after a notified period of say 4 to 7 years. The conversion of detachable warrant into equity shares will have to be done within time period notified by the company.

9.3 Zero interest fully convertible debentures: These are fully convertible debentures which do not carry any interest. The debentures are compulsorily and automatically converted after a specified period of time and holders thereof are entitled to new equity shares of the company at predetermined price. From the point of view of company this kind of instrument is beneficial in the sense that no interest is to be paid on it, if the share price of the company in the market is very high than the investors tends to get equity shares of the company at the lower rate.

9.4 Zero Coupon Bonds: A Zero Coupon Bonds does not carry any interest but it is sold by the issuing company at a discount. The difference between the discounted value and maturing or face value represents the interest to be earned by the investor on such bonds.

9.5 Double Option Bonds: These have also been recently issued by the IDBI. The face value of each bond is Rs. 5,000. The bond carries interest at 15% per annum compounded half yearly from the date of allotment. The bond has maturity period of 10 years. Each bond has two parts in the form of two separate certificates, one for principal of Rs. 5,000 and other for interest (including redemption premium) of Rs. 16,500. Both these certificates are listed on all major stock exchanges. The investor has the facility of selling either one or both parts anytime he likes.

5.28

Page 758: 30510870 Cost Accounting and Financial Management

Types of Financing

9.6 Option Bonds: These are cumulative and non-cumulative bonds where interest is payable on maturity or periodically. Redemption premium is also offered to attract investors. These were recently issued by IDBI, ICICI etc.

9.7 Inflation Bonds: Inflation Bonds are the bonds in which interest rate is adjusted for inflation. Thus, the investor gets interest which is free from the effects of inflation. For example, if the interest rate is 11 per cent and the inflation is 5 per cent, the investor will earn 16 per cent meaning thereby that the investor is protected against inflation.

9.8 Floating Rate Bonds: This as the name suggests is bond where the interest rate is not fixed and is allowed to float depending upon the market conditions. This is an ideal instrument which can be resorted to by the issuer to hedge themselves against the volatility in the interest rates. This has become more popular as a money market instrument and has been successfully issued by financial institutions like IDBI, ICICI etc.

10. INTERNATIONAL FINANCING

The essence of financial management is to raise and utilise the funds raised effectively. There are various avenues for organisations to raise funds either through internal or external sources. The sources of external sources include:

10.1 Commercial Banks: Like domestic loans, commercial banks all over the world extend Foreign Currency (FC) loans also for international operations. These banks also provide to overdraw over and above the loan amount.

10.2 Development Banks: Development banks offer long & medium term loans including FC loans. Many agencies at the national level offer a number of concessions to foreign companies to invest within their country and to finance exports from their countries. E.g. EXIM Bank of USA.

10.3 Discounting of Trade Bills: This is used as a short term financing method. It is used widely in Europe and Asian countries to finance both domestic and international business.

10.4 International Agencies: A number of international agencies have emerged over the years to finance international trade & business. The more notable among them include The Inter- national Finance Corporation (IFC), The International Bank for Reconstruction and Development (IBRD), The Asian Development Bank (ADB), The International Monetary Fund (IMF), etc.

10.5 International Capital Markets: Today, modern organisations including MNC's depend upon sizeable borrowings in Rupees as well as Foreign Currency. In order to cater to the

5.29

Page 759: 30510870 Cost Accounting and Financial Management

Financial Management needs of such organisations, international capital markets have sprung all over the globe such as in London.

In international capital market, the availability of FC is assured under the four main systems viz:

* Euro-currency market

* Export credit facilities

* Bonds issues

* Financial Institutions.

The origin of the Euro-currency market was with the dollar denominated bank deposits & loans in Europe particularly in London. Euro-dollar deposits are dollar denominated time deposits available at foreign branches of US banks & at some foreign banks. Banks based in Europe accept dollar denominated deposits & make dollar denominated deposits to the clients. This forms the backbone of the Euro-currency market all over the globe. In this market, funds are made available as loans through syndicated Euro-credit of instruments such as FRN's. FR certificates of deposits.

10.6 Financial Instruments: Some of the various financial instruments dealt with in the international market are briefly described below:

(a) External Commercial Borrowings(ECB) : ECBs refer to commercial loans (in the form of bank loans , buyers credit, suppliers credit, securitised instruments ( e.g. floating rate notes and fixed rate bonds) availed from non resident lenders with minimum average maturity of 3 years. Borrowers can raise ECBs through internationally recognised sources like (i) international banks, (ii) international capital markets, (iii) multilateral financial institutions such as the IFC, ADB etc, (iv) export credit agencies, (v) suppliers of equipment, (vi) foreign collaborators and (vii) foreign equity holders.

External Commercial Borrowings can be accessed under two routes viz (i) Automatic route and (ii) Approval route. Under the Automatic route there is no need to take the RBI/Government approval whereas such approval is necessary under the Approval route. Company’s registered under the Companies Act and NGOs engaged in micro finance activities are eligible for the Automatic Route where as Financial Institutions and Banks dealing exclusively in infrastructure or export finance and the ones which had participated in the textile and steel sector restructuring packages as approved by the government are required to take the Approval Route.

5.30

Page 760: 30510870 Cost Accounting and Financial Management

Types of Financing

(b) Euro Bonds: Euro bonds are debt instruments which are not denominated in the currency of the country in which they are issued. E.g. a Yen note floated in Germany. Such bonds are generally issued in a bearer form rather than as registered bonds and in such cases they do not contain the investor’s names or the country of their origin. These bonds are an attractive proposition to investors seeking privacy.

(c) Foreign Bonds: These are debt instruments issued by foreign corporations or foreign governments. Such bonds are exposed to default risk, especially the corporate bonds. These bonds are denominated in the currency of the country where they are issued, however, in case these bonds are issued in a currency other than the investors home currency, they are exposed to exchange rate risks. An example of a foreign bond ‘A British firm placing Dollar denominated bonds in USA’.

(d) Fully Hedged Bonds: As mentioned above, in foreign bonds, the risk of currency fluctuations exists. Fully hedged bonds eliminate the risk by selling in forward markets the entire stream of principal and interest payments.

(e) Medium Term Notes: Certain issuers need frequent financing through the Bond route including that of the Euro bond. However it may be costly and ineffective to go in for frequent issues. Instead, investors can follow the MTN programme. Under this programme, several lots of bonds can be issued, all having different features e.g. different coupon rates, different currencies etc. The timing of each lot can be decided keeping in mind the future market opportunities. The entire documentation and various regulatory approvals can be taken at one point of time

(f) Floating Rate Notes: These are issued up to seven years maturity. Interest rates are adjusted to reflect the prevailing exchange rates. They provide cheaper money than foreign loans.

(g) Euro Commercial Papers (ECP): ECPs are short term money market instruments. They are for maturities less than one year. They are usually designated in US Dollars.

(h) Foreign Currency Option: A FC Option is the right to buy or sell, spot, future or forward, a specified foreign currency. It provides a hedge against financial and economic risks.

(i) Foreign Currency Futures: FC Futures are obligations to buy or sell a specified currency in the present for settlement at a future date.

10.7 Euro Issues by Indian Companies : Indian companies are permitted to raise foreign currency resources through issue of ordinary equity shares through Global Depository Receipts(GDRs)/ American Depository Receipts (ADRs) and / or issue of Foreign Currency

5.31

Page 761: 30510870 Cost Accounting and Financial Management

Financial Management Convertible Bonds (FCCB) to foreign investors i.e. institutional investors or individuals (including NRIs) residing abroad . Such investment is treated as Foreign Direct Investment. The government guidelines on these issues are covered under the Foreign Currency Convertible Bonds and Ordinary Shares (Through depositary receipt mechanism) Scheme, 1993 and notifications issued after the implementation of the said scheme. (a) American Depository Deposits (ADR) : These are securities offered by non-US companies who want to list on any of the US exchange. Each ADR represents a certain number of a company's regular shares. ADRs allow US investors to buy shares of these companies without the costs of investing directly in a foreign stock exchange. ADRs are issued by an approved New York bank or trust company against the deposit of the original shares. These are deposited in a custodial account in the US. Such receipts have to be issued in accordance with the provisions stipulated by the SEC. USA which are very stringent. ADRs can be traded either by trading existing ADRs or purchasing the shares in the issuer's home market and having new ADRs created, based upon availability and market conditions. When trading in existing ADRs, the trade is executed on the secondary market on the New York Stock Exchange (NYSE) through Depository Trust Company (DTC) without involvement from foreign brokers or custodians. The process of buying new, issued ADRs goes through US brokers, Helsinki Exchanges and DTC as well as Deutsche Bank. When transactions are made, the ADRs change hands, not the certificates. This eliminates the actual transfer of stock certificates between the US and foreign countries. In a bid to bypass the stringent disclosure norms mandated by the SEC for equity shares, the Indian companies have however, chosen the indirect route to tap the vast American financial market through private debt placement of GDRs listed in London and Luxemberg Stock Exchanges. The Indian companies have preferred the GDRs to ADRs because the US market exposes them to a higher level or responsibility than a European listing in the areas of disclosure, costs, liabilities and timing. The SECs regulations set up to protect the retail investor base are some what more stringent and onerous, even for companies already listed and held by retail investors in their home country. The most onerous aspect of a US listing for the companies is to provide full, half yearly and quarterly accounts in accordance with, or at least reconciled with US GAAPs. (b) Global Depository Receipt (GDRs): These are negotiable certificate held in the bank of one country representing a specific number of shares of a stock traded on the exchange of another country. These financial instruments are used by companies to raise capital in either dollars or Euros. These are mainly traded in European countries and particularly in London. ADRs/GDRs and the Indian Scenario : Indian companies are shedding their reluctance to tap the US markets. Infosys Technologies was the first Indian company to be listed on Nasdaq in 1999. However, the first Indian firm to issue sponsored GDR or ADR was Reliance industries

5.32

Page 762: 30510870 Cost Accounting and Financial Management

Types of Financing

Limited. Beside, these two companies there are several other Indian firms are also listed in the overseas bourses. These are Satyam Computer, Wipro, MTNL, VSNL, State Bank of India, Tata Motors, Dr Reddy's Lab, Ranbaxy, Larsen & Toubro, ITC, ICICI Bank, Hindalco, HDFC Bank and Bajaj Auto.

(c) Indian Depository Receipts (IDRs): The concept of the depository receipt mechanism which is used to raise funds in foreign currency has been applied in the Indian Capital Market through the issue of Indian Depository Receipts (IDRs). IDRs are similar to ADRs/GDRs in the sense that foreign companies can issue IDRs to raise funds from the Indian Capital Market in the same lines as an Indian company uses ADRs/GDRs to raise foreign capital. The IDRs are listed and traded in India in the same way as other Indian securities are traded.

10.8 Other Types of International Issues

(a) Foreign Euro Bonds: In domestic capital markets of various countries the Bonds issues referred to above are known by different names such as Yankee Bonds in the US, Swiss Frances in Switzerland, Samurai Bonds in Tokyo and Bulldogs in UK.

(b) Euro Convertible Bonds: A convertible bond is a debt instrument which gives the holders of the bond an option to convert the bonds into a pre-determined number of equity shares of the company. Usually the price of the equity shares at the time of conversion will have a premium element. These bonds carry a fixed rate of interest and if the issuer company so desires may also include a Call Option (where the issuer company has the option of calling/ buying the bonds for redemption prior to the maturity date) or a Put Option (which gives the holder the option to put/sell his bonds to the issuer company at a pre-determined date and price).

(c) Euro Bonds: Plain Euro Bonds are nothing but debt Instruments. These are not very at- attractive for an investor who desires to have valuable additions to his investments.

(d) Euro Convertible Zero Bonds: These bonds are structured as a convertible bond. No interest is payable on the bonds. But conversion of bonds takes place on maturity at a pre- determined price. Usually there is a five years maturity period and they are treated as a deferred equity issue.

(e) Euro Bonds with Equity Warrants: These bonds carry a coupon rate determined by market rates. The warrants are detachable. Pure bonds are traded at a discount. Fixed In- come Funds Management may like to invest for the purposes of regular income.

5.33

Page 763: 30510870 Cost Accounting and Financial Management

Financial Management Self Examination Questions

A. Objective Type Questions

1. The largest provider of short-term credit for a business is:

(a) Banks (b) Suppliers to the firm (c) Commercial paper (d) None of the above.

2. The number of days until the firm is past due to a supplier is called the:

(a) Discount period (b) Term to credit (c) Payment period (d) None of the above. 3. The principal value of a bond is called the:

(a) The coupon rate (b) The par value (c) The maturity value (d) None of the above.

4. An advantage of debt financing is:

(a) Interest payments are tax deductible (b) The use of debt, up to a point, lowers the firm's cost of capital (c) Does not dilute owner's earnings (d) All of the above.

5. Zero coupon bonds:

(a) Are sold at par (b) Pay no interest payment (c) Are sold at a deep discount

(d) b and c above.

5.34

Page 764: 30510870 Cost Accounting and Financial Management

Types of Financing

6. Commercial paper can be issued by:

(a) Corporate (b) Primary Dealers

(c) All India Financial Institutions

(d) All of the above.

7. Commercial paper can be issued in the following denominations:

(a) Rs 5 Lakhs

(b) Rs 10 Lakhs

(c) Rs 5 Lakhs and multiples there of

(d) Rs 1 Lakh and multiples there of.

8. Commercial paper is a:

(a) Short term source of Finance

(b) Medium term source of Finance

(c) Long term source of Finance

(d) None of the above.

9. Which amongst the following are short term sources of finance?

(a) Accrued expenses

(b) Deferred income

(c) Equity shares

(d) Debentures.

10. In a tax environment, the cost of debenture to the issuing company:

(a) Lower than the cost of equity shares

(b) Higher than the cost of equity shares

(c) Higher than the cost of preference shares

(d) None of the above.

5.35

Page 765: 30510870 Cost Accounting and Financial Management

Financial Management Answers to Objective Type Questions 1. (b); 2. (c); 3. (b); 4. (d); 5. (d); 6. (d); 7. (c); 8. (a); 9. (a) & (b); 10. (a)

B. Short Answer Type Questions

1. Briefly discuss the different sources of long term finance. 2. Write short notes on:

(a) Zero interest fully convertible debentures (b) Deep discount bonds (c) Inflation bonds (d) Debt securitization.

3. Why are debentures considered cheaper than equity as a source of finance? Discuss briefly.

4. What do you understand by the term ‘ploughing back of profits’?

5. Discuss briefly the concept of ‘Seed Capital Assistance’.

C. Long Answer Type Questions 1. Explain the advantages of equity financing.

2. What are the advantages of debt financing from the point of company and investors?

3. What do you mean by venture capital financing and what are the methods of this type of financing?

4. What are the advantages of lease financing?

5. Discuss the various ways in which Indian companies can raise foreign currency.

5.36

Page 766: 30510870 Cost Accounting and Financial Management

CHAPTER 6

INVESTMENT DECISIONS

Learning objectives After studying this chapter, you will be able to ♦ Describe capital budgeting decisions; ♦ Understand the purpose and process of Capital Budgeting; ♦ Appreciate the importance of cash flows and understand the basic principles for

measuring the same; ♦ Evaluate projects using various capital budgeting techniques like PB (Pay Back ), NPV

(Net Present Value), PI (Profitability Index) , IRR (Internal Rate of Return), MIRR (Modified Internal Rate of Return) and ARR (Accounting Rate of Return); and

♦ Understand the advantages and disadvantages of the above mentioned techniques.

1. INTRODUCTION Financing and investment of funds are two crucial financial functions. The investment of funds also termed as capital budgeting requires a number of decisions to be taken in a situation in which funds are invested and benefits are expected over a long period. The term capital budgeting means planning for capital assets. It involves proper project planning and commercial evaluation of projects to know in advance technical feasibility and financial viability of the project. The capital budgeting decision means a decision as to whether or not money should be invested in long-term projects such as the setting up of a factory or installing a machinery or creating additional capacities to manufacture a part which at present may be purchased from outside. It includes a financial analysis of the various proposals regarding capital expenditure to evaluate their impact on the financial condition of the company and to choose the best out of the various alternatives. In any business the commitment of funds in land, buildings, equipment, stock and other types of assets must be carefully made. Once the decision to acquire a fixed asset is taken, it becomes very difficult to reverse that decision. The expenditure on plant and machinery and other long term assets affects operations over a period of years. It becomes a commitment that influences long term prospects and the future earning capacity of the firm.

Page 767: 30510870 Cost Accounting and Financial Management

Financial Management However, Capital Budgeting excludes certain investment decisions, wherein, the benefits of investment proposals cannot be directly quantified. For example, management may be considering a proposal to build a recreation room for employees. The decision in this case will be based on qualitative factors, such as management − employee relations, with less consideration on direct financial returns. However, most investment proposals considered by management will require quantitative estimates of the benefits to be derived from accepting the project. A bad decision can be detrimental to the value of the organisation over a long period of time.

2. PURPOSE OF CAPITAL BUDGETING The capital budgeting decisions are important, crucial and critical business decisions due to following reasons: (i) Substantial expenditure: Capital budgeting decisions involves the investment of substantial amount of funds. It is therefore necessary for a firm to make such decisions after a thoughtful consideration so as to result in the profitable use of its scarce resources. The hasty and incorrect decisions would not only result into huge losses but may also account for the failure of the firm. (ii) Long time period: The capital budgeting decision has its effect over a long period of time. These decisions not only affect the future benefits and costs of the firm but also influence the rate and direction of growth of the firm. (iii) Irreversibility: Most of the investment decisions are irreversible. Once they are taken, the firm may not be in a position to reverse them back. This is because, as it is difficult to find a buyer for the second-hand capital items. (iv) Complex decision: The capital investment decision involves an assessment of future events, which in fact is difficult to predict. Further it is quite difficult to estimate in quantitative terms all the benefits or the costs relating to a particular investment decision.

3. CAPITAL BUDGETING PROCESS The extent to which the capital budgeting process needs to be formalised and systematic procedures established depends on the size of the organisation; number of projects to be considered; direct financial benefit of each project considered by itself; the composition of the firm's existing assets and management's desire to change that composition; timing of expenditures associated with the projects that are finally accepted. (i) Planning: The capital budgeting process begins with the identification of potential investment opportunities. The opportunity then enters the planning phase when the potential effect on the firm's fortunes is assessed and the ability of the management of the firm to exploit the opportunity is determined. Opportunities having little merit are rejected and

6.2

Page 768: 30510870 Cost Accounting and Financial Management

Investment Decisions

promising opportunities are advanced in the form of a proposal to enter the evaluation phase. (ii) Evaluation: This phase involves the determination of proposal and its investments, inflows and outflows. Investment appraisal techniques, ranging from the simple payback method and accounting rate of return to the more sophisticated discounted cash flow techniques, are used to appraise the proposals. The technique selected should be the one that enables the manager to make the best decision in the light of prevailing circumstances. (iii) Selection: Considering the returns and risks associated with the individual projects as well as the cost of capital to the organisation, the organisation will choose among projects so as to maximise shareholders’ wealth. (iv) Implementation: When the final selection has been made, the firm must acquire the necessary funds, purchase the assets, and begin the implementation of the project. (v) Control: The progress of the project is monitored with the aid of feedback reports. These reports will include capital expenditure progress reports, performance reports comparing actual performance against plans set and post completion audits. (vi) Review: When a project terminates, or even before, the organisation should review the entire project to explain its success or failure. This phase may have implication for firms planning and evaluation procedures. Further, the review may produce ideas for new proposals to be undertaken in the future.

4. TYPES OF CAPITAL INVESTMENT DECISIONS There are many ways to classify the capital budgeting decision. Generally capital investment decisions are classified in two ways. One way is to classify them on the basis of firm’s existence. Another way is to classify them on the basis of decision situation. 4.1 On the basis of firm’s existence: The capital budgeting decisions are taken by both newly incorporated firms as well as by existing firms. The new firms may be required to take decision in respect of selection of a plant to be installed. The existing firm may be required to take decisions to meet the requirement of new environment or to face the challenges of competition. These decisions may be classified as follows: (i) Replacement and Modernisation decisions: The replacement and modernisation decisions aim at to improve operating efficiency and to reduce cost. Generally all types of plant and machinery require replacement either because of the economic life of the plant or machinery is over or because it has become technologically outdated. The former decision is known as replacement decisions and later one is known as modernisation decisions. Both replacement and modernisation decisions are called cost reduction decisions. (ii) Expansion decisions: Existing successful firms may experience growth in demand of their product line. If such firms experience shortage or delay in the delivery of their products

6.3

Page 769: 30510870 Cost Accounting and Financial Management

Financial Management due to inadequate production facilities, they may consider proposal to add capacity to existing product line. (iii) Diversification decisions: These decisions require evaluation of proposals to diversify into new product lines, new markets etc. for reducing the risk of failure by dealing in different products or by operating in several markets. Both expansion and diversification decisions are called revenue expansion decisions. 4.2 On the basis of decision situation: The capital budgeting decisions on the basis of decision situation are classified as follows: (i) Mutually exclusive decisions: The decisions are said to be mutually exclusive if two or more alternative proposals are such that the acceptance of one proposal will exclude the acceptance of the other alternative proposals. For instance, a firm may be considering proposal to install a semi-automatic or highly automatic machine. If the firm install a semi-automatic machine it exclude the acceptance of proposal to install highly automatic machine. (ii) Accept-reject decisions: The accept-reject decisions occur when proposals are independent and do not compete with each other. The firm may accept or reject a proposal on the basis of a minimum return on the required investment. All those proposals which give a higher return than certain desired rate of return are accepted and the rest are rejected. (iii) Contingent decisions: The contingent decisions are dependable proposals. The investment in one proposal requires investment in one or more other proposals. For example if a company accepts a proposal to set up a factory in remote area it may have to invest in infrastructure also e.g. building of roads, houses for employees etc.

5. PROJECT CASH FLOWS Project cash flows are defined as the financial costs and benefits associated with a project. The estimation of costs and benefits are made with the help of inputs provided by marketing, production, engineering, costing, purchase, taxation, and other departments. The project cash flow stream consists of cash outflows and cash inflows. The costs are denoted as cash outflows whereas the benefits are denoted as cash inflows. The future costs and benefits associated with each project are as follows: (i) Capital costs (ii) Operating costs (iii) Revenue (iv) Depreciation (v) Residual value An investment decision implies the choice of an objective, an appraisal technique and the

6.4

Page 770: 30510870 Cost Accounting and Financial Management

Investment Decisions

project’s life. The objective and technique must be related to definite period of time. The life of the project may be determined by taking into consideration the following factors: (i) Technological obsolescence (ii) Physical deterioration (iii) A decline in demand for the output of the project. No matter how good a company's maintenance policy, its technological forecasting ability or its demand forecasting ability, uncertainty will always be present because of the difficulty in predicting the duration of a project life. To allow realistic appraisal, the value of cash payment or receipt must be related to the time when the transfer takes place. In particular, it must be recognised that Re. 1 received today is worth more than Re. 1 received at some future date because Re. 1 received today could be earning interest in the intervening period. This is the concept of 'Time Value of Money' (for a detailed understanding of the Time Value of Money please refer to Chapter 2). The process of converting future sums into their present equivalent is known as discounting, which is used to determine the present value of future cash flows.

6. BASIC PRINCIPLES FOR MEASURING PROJECT CASH FLOWS For developing the project cash flows the following principles must be kept in mind: 1. Incremental Principle: The cash flows of a project must be measured in incremental terms. To ascertain a project’s incremental cash flows, one has to look at what happens to the cash flows of the firm 'with the project and without the project', and not before the project and after the project as is sometimes done. The difference between the two reflects the incremental cash flows attributable to the project. Project cash flows for year t = Cash flow for the firm with the project for year t

− Cash flow for the firm without the project for year t.

2. Long Term Funds Principle: A project may be evaluated from various points of view: total funds point of view, long-term funds point of view, and equity point of view. The measurement of cash flows as well as the determination of the discount rate for evaluating the cash flows depends on the point of view adopted. It is generally recommended that a project may be evaluated from the point of view of long-term funds (which are provided by equity stockholders, preference stock holders, debenture holders, and term lending institutions) because the principal focus of such evaluation is normally on the profitability of long-term funds. 3. Exclusion of Financing Costs Principle: When cash flows relating to long-term funds are being defined, financing costs of long-term funds (interest on long-term debt and equity

6.5

Page 771: 30510870 Cost Accounting and Financial Management

Financial Management dividend) should be excluded from the analysis. The question arises why? The weighted average cost of capital used for evaluating the cash flows takes into account the cost of long-term funds. Put differently, the interest and dividend payments are reflected in the weighted average cost of capital. Hence, if interest on long-term debt and dividend on equity capital are deducted in defining the cash flows, the cost of long-term funds will be counted twice. The exclusion of financing costs principle means that: (i) The interest on long-term debt (or interest) is ignored while computing profits and taxes

and; (ii) The expected dividends are deemed irrelevant in cash flow analysis. While dividends pose no difficulty as they come only from profit after taxes, interest needs to be handled properly. Since interest is usually deducted in the process of arriving at profit after tax, an amount equal to interest (1 − tax rate) should be added back to the figure of profit after tax. That is, Profit before interest and tax (1 − tax rate) = (Profit before tax + interest) (1 − tax rate) = (Profit before tax) (1 − tax rate) + (interest) (1 − tax rate) = Profit after tax + interest (1 − tax rate) Thus, whether the tax rate is applied directly to the profit before interest and tax figure or whether the tax − adjusted interest, which is simply interest (1 − tax rate), is added to profit after tax, we get the same result. 4. Post−tax Principle: Tax payments like other payments must be properly deducted in deriving the cash flows. That is, cash flows must be defined in post-tax terms. Illustration 1 ABC Ltd is evaluating the purchase of a new project with a depreciable base of Rs. 1,00,000; expected economic life of 4 years and change in earnings before taxes and depreciation of Rs. 45,000 in year 1, Rs. 30,000 in year 2, Rs. 25,000 in year 3 and Rs. 35,000 in year 4. Assume straight-line depreciation and a 20% tax rate. You are required to compute relevant cash flows.

6.6

Page 772: 30510870 Cost Accounting and Financial Management

Investment Decisions

Solution

Rs. Years

1 2 3 4 Earnings before tax and depreciation

45,000 30,000 25,000 35,000

Less: Depreciation 25,000 25,000 25,000 25,000 Earnings before tax 20,000 5,000 0 10,000 Less: Tax [@20%] 4,000 1,000 0 2,000 16,000 4,000 0 8,000 Add: Depreciation 25,000 25,000 25,000 25,000 Net Cash flow 41,000 29,000 25,000 33,000

Working Note: Depreciation = Rs. 1, 00,000÷4 = Rs. 25,000 Illustration 2 XYZ Ltd is considering a new investment project about which the following information is available. (i) The total outlay on the project will be Rs. 100 lacks. This consists of Rs.60 lacks on plant

and equipment and Rs.40 lacks on gross working capital. The entire outlay will be incurred at the beginning of the project.

(ii) The project will be financed with Rs.40 lacks of equity capital; Rs.30 lacks of long term debt (in the form of debentures); Rs. 20 lacks of short-term bank borrowings, and Rs. 10 lacks of trade credit. This means that Rs.70 lacks of long term finds (equity + long term debt) will be applied towards plant and equipment (Rs. 60 lacks) and working capital margin (Rs.10 lacks) – working capital margin is defined as the contribution of long term funds towards working capital. The interest rate on debentures will be 15 percent and the interest rate on short-term borrowings will be 18 percent.

(iii) The life of the project is expected to be 5 years and the plant and equipment would fetch a salvage value of Rs. 20 lacks. The liquidation value of working capital will be equal to Rs.10 lacks.

(iv) The project will increase the revenues of the firm by Rs. 80 lacks per year. The increase in operating expenses on account of the project will be Rs.35.0 lacks per year. (This

6.7

Page 773: 30510870 Cost Accounting and Financial Management

Financial Management

includes all items of expenses other than depreciation, interest, and taxes). The effective tax rate will be 50 percent.

(v) Plant and equipment will be depreciated at the rate of 331/3 percent per year as per the written down value method. So, the depreciation charges will be :

Rs. (in lacs) First year 20.0 Second year 13.3 Third year 8.9 Fourth year 5.9 Fifth year 4.0 Given the above details, you are required to work out the post-tax, incremental cash flows relating to long-term funds. Solution

Cash Flows for the New Project Rs. (in lacs)

Years

0 1 2 3 4 5

(a) Plant and equipment (60.0)

(b) Working capital margin (10.0)

(c) Revenues 80.0 80.0 80.0 80.0 80.0

(d) Operating Costs 35.0 35.0 35.0 35.0 35.0

(e) Depreciation 20.0 13.33 8.89 5.93 3.95

(f) Interest on short-term bank borrowings 3.6 3.6 3.6 3.6 3.6

(g) Interest on debentures 4.5 4.5 4.5 4.5 4.5

(h) Profit before tax 16.90 23.57 28.01 30.97 32.95

(i) Tax 8.45 11.79 14.01 15.49 16.48

(j) Profit after tax 8.45 11.78 14.00 15.48 16.47

(k) Net salvage value of plant and equipment 20.0

(l) Net recovery working capital margin 10.0

(m) Initial Investment [(a) + (b)] (70.0)

6.8

Page 774: 30510870 Cost Accounting and Financial Management

Investment Decisions

(n) Operating cash inflows

[(j) +(e) + (g) (1–T)] 30.70 27.36 25.14 23.66 22.67

(o) Terminal cash flow

[(k) + (l) ] 30.00

(p) Net cash flow.

[(m) + (n) + (o) ] (70.0) 30.70 27.36 25.14 23.66 52.67

Working Notes (with explanations): (i) The initial investment, occurring at the end of year 0, is Rs. 70 lacs. This represents the

commitment of long-term funds to the project. The operating cash inflow, relating to long-term funds, at the end of year 1 is Rs. 30.7 lacs. That is,

Profit after tax + Depreciation + Interest on debentures (1 – tax ) Rs. 8.45 lacs + Rs.20 lacs + Rs. 4.5 lacs (1 – 0.50) The operating cash inflows for the subsequent years have been calculated similarly.

(ii) The terminal cash flow relating to long-term funds is equal to : Net Salvage value of plant and equipment + Net recovery of working capital margin When the project is terminated, its liquidation value will be equal to: Net Salvage value of plant and equipment + Net recovery of working capital The first component belongs to the suppliers of long-term funds. The second component is applied to repay the current liabilities and recover the working capital margin.

7. CAPITAL BUDGETING TECHNIQUES In order to maximise the return to the shareholders of a company, it is important that the best or most profitable investment projects are selected. Because the results for making a bad long-term investment decision can be both financially and strategically devastating, particular care needs to be taken with investment project selection and evaluation. There are a number of techniques available for appraisal of investment proposals and can be classified as presented below:

6.9

Page 775: 30510870 Cost Accounting and Financial Management

Financial Management

Accounting Rate of Return

Payback Period Net Present Value

Profitability Index

Internal Rate of Return

Discounted Payback

Modified Internal Rate of Return

Time-adjusted or

Discounted Cash Flows

Traditional or

Non-Discounting

CAPITAL BUDGETING TECHNIQUES Organizations may use any or more of capital investment evaluation techniques; some organizations use different methods for different types of projects while others may use multiple methods for evaluating each project. These techniques have been discussed below – net present value, profitability index, internal rate of return, modified internal rate of return, payback period, and accounting (book) rate of return. Payback Period: The payback period of an investment is the length of time required for the cumulative total net cash flows from the investment to equal the total initial cash outlays. At that point in time, the investor has recovered the money invested in the project. As with other methods discussed, the first steps in calculating the payback period are determining the total initial capital investment and the annual expected after-tax net cash flows over the useful life of the investment. When the net cash flows are uniform over the useful life of the project, the number of years in the payback period can be calculated using the following

6.10

Page 776: 30510870 Cost Accounting and Financial Management

Investment Decisions

equation:

flow cashnet tax-after expected Annualinvestment capital initial Total period Payback =

When the annual expected after-tax net cash flows are not uniform, the cumulative cash inflow from operations must be calculated for each year by subtracting cash outlays for operations and taxes from cash inflows and summing the results until the total is equal to the initial capital investment. Advantages: A major advantage of the payback period technique is that it is easy to compute and to understand as it provides a quick estimate of the time needed for the organization to recoup the cash invested. The length of the payback period can also serve as an estimate of a project’s risk; the longer the payback period, the riskier the project as long-term predictions are less reliable. The payback period technique focuses on quick payoffs. In some industries with high obsolescence risk or in situations where an organization is short on cash, short payback periods often become the determining factor for investments. Limitations: The major limitation of the payback period technique is that it ignores the time value of money. As long as the payback periods for two projects are the same, the payback period technique considers them equal as investments, even if one project generates most of its net cash inflows in the early years of the project while the other project generates most of its net cash inflows in the latter years of the payback period. A second limitation of this technique is its failure to consider an investment’s total profitability; it only considers cash flows from the initiation of the project until its payback period and ignores cash flows after the payback period. Lastly, use of the payback period technique may cause organizations to place too much emphasis on short payback periods thereby ignoring the need to invest in long-term projects that would enhance its competitive position. Illustration 3 Suppose a project costs Rs. 20,00,000 and yields annually a profit of Rs. 3,00,000 after depreciation @ 12½% (straight line method) but before tax 50%. The first step would be to calculate the cash inflow from this project. The cash inflow is Rs. 4,00,000 calculated as follows :

Rs. Profit before tax 3,00,000 Less : Tax @ 50% 1,50,000 Profit after tax 1,50,000 Add : Depreciation written off 2,50,000Total cash inflow 4,00,000

While calculating cash inflow, depreciation is added back to profit after tax since it does not

6.11

Page 777: 30510870 Cost Accounting and Financial Management

Financial Management result in cash outflow. The cash generated from a project therefore is equal to profit after tax plus depreciation.

Payback period = 000,00,4

000,00,20 .Rs = 5 Years

Some Accountants calculate payback period after discounting the cash flows by a predetermined rate and the payback period so calculated is called, ‘Discounted payback period’. Payback Reciprocal : As the name indicates it is the reciprocal of payback period. A major drawback of the payback period method of capital budgeting is that it does not indicate any cut off period for the purpose of investment decision. It is, however, argued that the reciprocal of the payback would be a close approximation of the internal rate of return if the life of the project is at least twice the payback period and the project generates equal amount of the annual cash inflows. In practice, the payback reciprocal is a helpful tool for quickly estimating the rate of return of a project provided its life is at least twice the payback period. The payback reciprocal can be calculated as follows:

investment Initial flow in cash annual Average

Illustration 4 Suppose a project requires an initial investment of Rs. 20,000 and it would give annual cash inflow of Rs. 4,000. The useful life of the project is estimated to be 5 years. In this example payback reciprocal will be :

000,20Rs100000,4Rs × = 20%

The above payback reciprocal provides a reasonable approximation of the internal rate of return, i.e. 19% discussed later in this chapter. Accounting (Book) Rate of Return: The accounting rate of return of an investment measures the average annual net income of the project (incremental income) as a percentage of the investment.

Accounting rate of return = Investment

income net annual Average

The numerator is the average annual net income generated by the project over its useful life. The denominator can be either the initial investment or the average investment over the useful life of the project. Some organizations prefer the initial investment because it is objectively determined and is not influenced by either the choice of the depreciation method or the

6.12

Page 778: 30510870 Cost Accounting and Financial Management

Investment Decisions

estimation of the salvage value. Either of these amounts is used in practice but it is important that the same method be used for all investments under consideration. Advantages: The accounting rate of return technique uses readily available data that is routinely generated for financial reports and does not require any special procedures to generate data. This method may also mirror the method used to evaluate performance on the operating results of an investment and management performance. Using the same procedure in both decision-making and performance evaluation ensures consistency. Lastly, the calculation of the accounting rate of return method considers all net incomes over the entire life of the project and provides a measure of the investment’s profitability. Limitations: The accounting rate of return technique, like the payback period technique, ignores the time value of money and considers the value of all cash flows to be equal. Additionally, the technique uses accounting numbers that are dependent on the organization’s choice of accounting procedures, and different accounting procedures, e.g., depreciation methods, can lead to substantially different amounts for an investment’s net income and book values. The method uses net income rather than cash flows; while net income is a useful measure of profitability, the net cash flow is a better measure of an investment’s performance. Furthermore, inclusion of only the book value of the invested asset ignores the fact that a project can require commitments of working capital and other outlays that are not included in the book value of the project. Illustration 5 Suppose a project requiring an investment of Rs. 10,00,000 yields profit after tax and depreciation as follows:

Years Profit after tax and depreciation Rs. 1. 50,000 2. 75,000 3. 1,25,000 4. 1,30,000 5. 80,000

Total 4,60,000Suppose further that at the end of 5 years, the plant and machinery of the project can be sold for Rs. 80,000. In this case the rate of return can be calculated as follows.

profit of years of No. project the in investmentNet

100 Profit Total××

6.13

Page 779: 30510870 Cost Accounting and Financial Management

Financial Management

years 5 9,20,000 Rs100 4,60,000 Rs

×× = 10%

This rate is compared with the rate expected on other projects, had the same funds been invested alternatively in those projects. Sometimes, the management compares this rate with the minimum rate (called-cut off rate) they may have in mind. For example, management may decide that they will not undertake any project which has an average annual yield after tax less than 15%. Any capital expenditure proposal which has an average annual yield of less than 15% will be automatically rejected. Net Present Value Technique: The net present value technique is a discounted cash flow method that considers the time value of money in evaluating capital investments. An investment has cash flows throughout its life, and it is assumed that a rupee of cash flow in the early years of an investment is worth more than a rupee of cash flow in a later year. The net present value method uses a specified discount rate to bring all subsequent net cash inflows after the initial investment to their present values (the time of the initial investment or year 0). Theoretically, the discount rate or desired rate of return on an investment is the rate of return the firm would have earned by investing the same funds in the best available alternative investment that has the same risk. Determining the best alternative opportunity available is difficult in practical terms so rather that using the true opportunity cost, organizations often use an alternative measure for the desired rate of return. An organization may establish a minimum rate of return that all capital projects must meet; this minimum could be based on an industry average or the cost of other investment opportunities. Many organizations choose to use the cost of capital as the desired rate of return; the cost of capital is the cost that an organization has incurred in raising funds or expects to incur in raising the funds needed for an investment. The overall cost of capital of a firm is a proportionate average of the costs of the various components of the firm’s financing. A firm obtains funds by issuing preferred or common stock; borrowing money using various forms of debt such a notes, loans, or bonds; or retaining earnings. The costs to the firm are the returns demanded by debt and equity investors through which the firm raises the funds. The net present value of a project is the amount, in current rupees, the investment earns after yielding the desired rate of return in each period. Net present value = Present value of net cash flow - Total net initial investment The steps to calculating net present value are (1) determine the net cash inflow in each year of the investment, (2) select the desired rate of return, (3) find the discount factor for each year based on the desired rate of return selected, (4) determine the present values of the net

6.14

Page 780: 30510870 Cost Accounting and Financial Management

Investment Decisions

cash flows by multiplying the cash flows by the discount factors, (5) total the amounts for all years in the life of the project, and (6) subtract the total net initial investment. Illustration 6 Compute the net present value for a project with a net investment of Rs. 1, 00,000 and the following cash flows if the company’s cost of capital is 10%? Net cash flows for year one is Rs. 55,000; for year two is Rs. 80,000 and for year three is Rs. 15,000. [PVIF @ 10% for three years are 0.909, 0.826 and 0.751] Solution

Year Net Cash Flows PVIF @ 10% Discounted Cash Flows

1 55,000 0.909 49,995 2 80,000 0.826 66,080 3 15,000 0.751 11,265 1,27,340

Total Discounted Cash Flows 1,27,340 Less: Net Investment 1,00,000 Net Present Value 27,340 Recommendation: Since the net present value of the project is positive, the company should accept the project. Illustration 7 ABC Ltd is a small company that is currently analyzing capital expenditure proposals for the purchase of equipment; the company uses the net present value technique to evaluate projects. The capital budget is limited to 500,000 which ABC Ltd believes is the maximum capital it can raise. The initial investment and projected net cash flows for each project are shown below. The cost of capital of ABC Ltd is 12%. You are required to compute the NPV of the different projects. Project A Project B Project C Project D Initial Investment 200,000 190,000 250,000 210,000 Project Cash Inflows Year 1 50,000 40,000 75,000 75,000 2 50,000 50,000 75,000 75,000 3 50,000 70,000 60,000 60,000 4 50,000 75,000 80,000 40,000 5 50,000 75,000 100,000 20,000

6.15

Page 781: 30510870 Cost Accounting and Financial Management

Financial Management Solution Calculation of net present value: Present

Period value factor Project A Project B Project C Project D 1 0.893 44,650 35,720 66,975 66,975 2 0.797 39,850 39,850 59,775 59,775 3 0.712 35,600 49,840 42,720 42,720 4 0.636 31,800 47,700 50,880 25,440 5 0.567 28,350 42,525 56,700 11,340

Present value of cash inflows 180,250 215,635 277,050 206,250 Less: Initial investment 200,000 190,000 250,000 210,000Net present value (19,750) 25,635 27,050 (3,750)

Advantages (i) NPV method takes into account the time value of money. (ii) The whole stream of cash flows is considered. (iii) The net present value can be seen as the addition to the wealth of share holders.

The criterion of NPV is thus in conformity with basic financial objectives. (iv) The NPV uses the discounted cash flows i.e., expresses cash flows in terms of

current rupees. The NPVs of different projects therefore can be compared. It implies that each project can be evaluated independent of others on its own merit.

Limitations (i) It involves difficult calculations. (ii) The application of this method necessitates forecasting cash flows and the discount

rate. Thus accuracy of NPV depends on accurate estimation of these two factors which may be quite difficult in practice.

(iii) The ranking of projects depends on the discount rate. Let us consider two projects involving an initial outlay of Rs. 25 lakhs each with following inflow :

(Rs in lakhs)

1st year 2nd year Project A 50.0 12.5 Project B 12.5 50.0

6.16

Page 782: 30510870 Cost Accounting and Financial Management

Investment Decisions

At discounted rate of 5% and 10% the NPV of Projects and their rankings at 5% and 10% are as follows: NPV @ 5% Rank NPV @ 10% Rank

Project A 33.94 I 30.78 I Project B 32.25 II 27.66 II

The project ranking is same when the discount rate is changed from 5% to 10%.

However, the impact of the discounting becomes more severe for the later cash flows. Naturally, higher the discount rate, higher would be the impact. In the case of project B the larger cash flows come later in the project life, thus decreasing the present value to a larger extent.

(iv) The decision under NPV method is based on absolute measure. It ignores the difference in initial outflows, size of different proposals etc. while evaluating mutually exclusive projects.

Desirability Factor/Profitability Index: In above Illustration the students may have seen how with the help of discounted cash flow technique, the two alternative proposals for capital expenditure can be compared. In certain cases we have to compare a number of proposals each involving different amounts of cash inflows. One of the methods of comparing such proposals is to workout what is known as the ‘Desirability factor’, or ‘Profitability index’. In general terms a project is acceptable if its profitability index value is greater than 1. Mathematically : The desirability factor is calculated as below :

may) case the (as outflow cash discounted aloutlay/Tot cash Initial flows in cash discounted of Sum

Illustration 8 Suppose we have three projects involving discounted cash outflow of Rs.5,50,000, Rs75,000 and Rs.1,00,20,000 respectively. Suppose further that the sum of discounted cash inflows for these projects are Rs. 6,50,000, Rs. 95,000 and Rs 1,00,30,000 respectively. Calculate the desirability factors for the three projects. Solution The desirability factors for the three projects would be as follows:

1. 000,50,5 .Rs000,50,6 .Rs = 1.18

6.17

Page 783: 30510870 Cost Accounting and Financial Management

Financial Management

2. 000,75 .Rs000,95 .Rs = 1.27

3. 000,20,00,1.Rs000,30,00,1.Rs = 1.001

It would be seen that in absolute terms project 3 gives the highest cash inflows yet its desirability factor is low. This is because the outflow is also very high. The Desirability/ Profitability Index factor helps us in ranking various projects. Advantages The method also uses the concept of time value of money and is a better project evaluation technique than NPV. Limitations Profitability index fails as a guide in resolving capital rationing (discussed later in this chapter) where projects are indivisible. Once a single large project with high NPV is selected, possibility of accepting several small projects which together may have higher NPV than the single project is excluded. Also situations may arise where a project with a lower profitability index selected may generate cash flows in such a way that another project can be taken up one or two years later, the total NPV in such case being more than the one with a project with highest Profitability Index. The Profitability Index approach thus cannot be used indiscriminately but all other type of alternatives of projects will have to be worked out.

8. CAPITAL RATIONING Generally, firms fix up maximum amount that can be invested in capital projects, during a given period of time, say a year. The firm then attempts to select a combination of investment proposals that will be within the specific limits providing maximum profitability and rank them in descending order according to their rate of return; such a situation is of capital rationing. A firm should accept all investment projects with positive NPV, with an objective to maximise the wealth of shareholders. However, there may be resource constraints due to which a firm may have to select from among various projects. Thus there may arise a situation of capital rationing where there may be internal or external constraints on procurement of necessary funds to invest in all investment proposals with positive NPVs. Capital rationing can be experienced due to external factors, mainly imperfections in capital markets which can be attributed to non-availability of market information, investor attitude etc. Internal capital rationing is due to the self-imposed restrictions imposed by management like not to raise additional debt or laying down a specified minimum rate of return on each project.

6.18

Page 784: 30510870 Cost Accounting and Financial Management

Investment Decisions

There are various ways of resorting to capital rationing. For instance, a firm may effect capital rationing through budgets. It may also put up a ceiling when it has been financing investment proposals only by way of retained earnings (ploughing back of profits). Since the amount of capital expenditure in that situation cannot exceed the amount of retained earnings, it is said to be an example of capital rationing. Capital rationing may also be introduced by following the concept of ‘Responsibility Accounting’, whereby management may introduce capital rationing by authorising a particular department to make investment only up to a specified limit, beyond which the investment decisions are to be taken by higher-ups. The selection of project under capital rationing involves two steps: (i) To identify the projects which can be accepted by using the technique of evaluation

discussed above. (ii) To select the combination of projects. In capital rationing it may also be more desirable to accept several small investment proposals than a few large investment proposals so that there may be full utilisation of budgeted amount. This may result in accepting relatively less profitable investment proposals if full utilisation of budget is a primary consideration. Similarly, capital rationing may also mean that the firm foregoes the next most profitable investment following after the budget ceiling even though it is estimated to yield a rate of return much higher than the required rate of return. Thus capital rationing does not always lead to optimum results. The following illustration shows how a firm may resort to capital rationing under situation of resource constraints. Illustration 9 Alpha Limited is considering five capital projects for the years 2000,2001,2002 and 2003. The company is financed by equity entirely and its cost of capital is 12%. The expected cash flows of the projects are as follows :

Year and Cash flows (Rs. ‘000) Project 2000 2001 2002 2003

A (70) 35 35 20 B (40) (30) 45 55 C (50) (60) 70 80 D — (90) 55 65 E (60) 20 40 50

Note : Figures in brackets represent cash outflows.

6.19

Page 785: 30510870 Cost Accounting and Financial Management

Financial Management All projects are divisible i.e. size of investment can be reduced, if necessary in relation to availability of funds. None of the projects can be delayed or undertaken more than once. Calculate which project Alpha Limited should undertake if the capital available for investment is limited to Rs. 1,10,000 in year 2000 and with no limitation in subsequent years. For your analysis, use the following present value factors:

Year 2000 2001 2002 2003 Discounting factor 1.00 0.89 0.80 0.71

Solution Computation of Net Present Value (NPV) & Profitability Index (PI) (Rs. ‘000)

Project Discounted Cash Flows (Refer to working note)

2000 2001 2002 2003 NPV PI

A (70) 31.15 28 14.20 3.35 1.048 B (40) (26.70) 36 39.05 8.35 1.125 C (50) (53.40) 56 56.80 9.40 1.091 D — (80.10) 44 46.15 10.05 1.125 E (60) 17.80 32 35.50 25.30 1.422 Ranking of Projects in descending order of profitability index

Rank 1 2 3 4 5 Projects E D B C A

Selection and Analysis: For Project ‘D’ there is no capital rationing but it satisfies the criterion of required rate of return. Hence Project D may be undertaken. For other projects the requirement is Rs. 2,20,000 in year 2000 whereas the capital available for investment is only Rs. 1,10,000. Based on the ranking, the final selection from other projects which will yield maximum NPV will be:

6.20

Page 786: 30510870 Cost Accounting and Financial Management

Investment Decisions

Project and Rank Amount of Initial Investment

Rs. E(1) 60,000 B(2) 40,000 C(3) 10,000 (Restriction)

Ranking of Projects excluding ‘D’ which is to start in 2001 when there is no limitation on capital availability :

Projects E B C A Rank 1 2 3 4

Working Note : Computation of Discounted Cash flows (Rs. ‘000)

Year 2000 2001 2002 2003Present value factor 1.00 0.89 0.80 0.71Project A Cash flows (70) 35 35 20 Discounted cash flows (70) 31.15 28 14.20B Cash flows (40) (30) 45 55 Discounted cash flows (40) (26.70) 36 39.05C Cash flows (50) (60) 70 80 Discounted cash flows (50) (53.40) 56 56.80D Cash flows — (90) 55 65 Discounted cash flows — (80.10) 44 46.15E Cash flows (60) 20 40 50

Discounted cash flows (60) 17.80 32 35.50

Note : Figures in brackets represent cash outflows. Illustration 10 Venture Ltd. has Rs. 30 lakhs available for investment in capital projects. It has the option of making investment in projects 1,2,3 and 4. Each project is entirely independent and has a useful life of 5 years. The expected present value of cash flows from the projects is as follows:

6.21

Page 787: 30510870 Cost Accounting and Financial Management

Financial Management Projects Initial outlay Present value of cash flowsf

Rs. Rs.

1 8,00,000 10,00,000

2 15,00,000 19,00,000

3 7,00,000 11,40,000

4 13,00,000 20,00,000

Which of the above investment should be undertaken? Assume that the cost of capital is 12% and risk free interest rate is 10% per annum. Given compounded sum of Re. 1 at 10% in 5 years is Rs. 1.611 and discount factor of Re. 1 at 12% rate for 5 years is 0.567. Solution This is a problem on capital rationing. The fund available with the company is Rs. 30 lakhs. The company will adopt those projects which will maximise the NPV.

Statement showing NPV of projects

Project Initial outlay Present value of future cash flow

NPV

Rs Rs (i) (ii) (iii)=(ii) – (i)

1 8,00,000 10,00,000 2,00,000 2 15,00,000 19,00,000 4,00,000 3 7,00,000 11,40,000 4,40,000 4 13,00,000 20,00,000 7,00,000

The NPV of the projects 1,2,3 is Rs. 10,40,000 (with full available amount utilised). The NPV of the projects 1, 3 and 4 is Rs. 40,000 (with Rs. 28 lakhs utilised, leaving Rs. 2,00,000 to be invested elsewhere). Now, Rs. 2,00,000 can be invested for a period of 5 years @ 10%. It is given in the question that the compounded value of Re. 1 @ 10% per annum for 5 years is Rs. 1.611. Therefore for Rs. 2,00,000 invested now for 5 years @ 10% the amount to be received after 5 years will be Rs. 3,22,200 (Rs. 2,00,000 × 1.611).

6.22

Page 788: 30510870 Cost Accounting and Financial Management

Investment Decisions

The amount to be received at the end of 5th years would be Rs. 1,82,687.40 (Rs. 3,22,200 × 0.567). Since, the amount to be received 15,22,687 after 5 years by making investment in projects 1, 3 and 4 & investing balance amount available @ 10% is greater than the amount to be received of Rs. 10,40,000 by investing in projects 1, 2 and 3. It is advisable that the venture Ltd. should make investment in projects 1, 3 and 4. Internal Rate of Return Method: Like the net present value method, the internal rate of return method considers the time value of money, the initial cash investment, and all cash flows from the investment. Unlike the net present value method, the internal rate of return method does not use the desired rate of return but estimates the discount rate that makes the present value of subsequent net cash flows equal to the initial investment. Using this estimated rate of return, the net present value of the investment will be zero. This estimated rate of return is then compared to a criterion rate of return that can be the organization’s desired rate of return, the rate of return from the best alternative investment, or another rate the organization chooses to use for evaluating capital investments. The procedures for computing the internal rate of return vary with the pattern of net cash flows over the useful life of an investment. The first step is to determine the investment’s total net initial cash disbursements and commitments and its net cash inflows in each year of the investment. For an investment with uniform cash flows over its life, the following equation is used: Total initial investment = Annual net cash flow x Annuity discount factor of the discount rate for the number of periods of the investment’s useful life If A is the annuity discount factor, then

A =investment the from flows cashnet (equal) Annual

investment the for scommitment and ntsdisburseme cash initial Total

Once A has been calculated, the discount rate is the interest rate that has the same discount factor as A in the annuity table along the row for the number of periods of the useful life of the investment. This computed discount rate or the internal rate of return will be compared to the criterion rate the organization has selected to assess the investment’s desirability. When the net cash flows are not uniform over the life of the investment, the determination of the discount rate can involve trial and error and interpolation between interest rates. It should be noted that there are several spreadsheet programs available for computing both net present value and internal rate of return that facilitate the capital budgeting process.

6.23

Page 789: 30510870 Cost Accounting and Financial Management

Financial Management Illustration 11 Calculate the internal rate of return of an investment of Rs. 1, 36,000 which yields the following cash inflows: Year Cash Inflows (in Rs.) 1 30,000 2 40,000 3 60,000 4 30,000 5 20,000 Solution Calculation of IRR Since the cash inflow is not uniform, the internal rate of return will have to be calculated by the trial and error method. In order to have an approximate idea about such rate, the ‘Factor’ must be found out. ‘The factor reflects the same relationship of investment and cash inflows as in case of payback calculations’:

CIF =

Where, F = Factor to be located I = Original Investment C = Average Cash inflow per year For the project,

000,36000,36,1 Factor =

= 3.78

The factor thus calculated will be located in the present value of Re.1 received annually for N year’s table corresponding to the estimated useful life of the asset. This would give the expected rate of return to be applied for discounting the cash inflows. In case of the project, the rate comes to 10%.

6.24

Page 790: 30510870 Cost Accounting and Financial Management

Investment Decisions

Year Cash Inflows (Rs.) Discounting factor at 10% Present Value (Rs.) 1 30,000 0.909 27,270 2 40,000 0.826 33,040 3 60,000 0.751 45,060 4 30,000 0.683 20,490 5 20,000 0.621 12,420 Total present value 1,38,280

The present value at 10% comes to Rs. 1,38,280, which is more than the initial investment. Therefore, a higher discount rate is suggested, say, 12%.

Year Cash Inflows (Rs.) Discounting factor at 10% Present Value (Rs.) 1 30,000 0.893 26,790 2 40,000 0.797 31,880 3 60,000 0.712 42,720 4 30,000 0.636 19,080 5 20,000 0.567 11,340 Total present value 1,31,810

The internal rate of return is, thus, more than 10% but less than 12%. The exact rate can be obtained by interpolation:

IRR =

810,31,1.Rs280,38,1.Rs000,36,1.Rs280,38,1.Rs10 ⎥

⎤⎢⎣

⎡⎟⎠⎞

⎜⎝⎛

−−

+x 2

= 10 + ⎟⎠⎞

⎜⎝⎛ 2x

64702280

= 10 + 0.7

IRR = 10.7%

Acceptance Rule The use of IRR, as a criterion to accept capital investment decision involves a comparison of

6.25

Page 791: 30510870 Cost Accounting and Financial Management

Financial Management IRR with the required rate of return known as cut off rate . The project should the accepted if IRR is greater than cut-off rate. If IRR is equal to cut off rate the firm is indifferent. If IRR less than cut off rate the project is rejected. Internal rate of return and mutually exclusive projects Projects are called mutually exclusive, when the selection of one precludes the selection of others e.g. in case a company owns a piece of land which can be put to use for either of the two different projects S or L, then such projects are mutually exclusive to each other i.e. the selection of one project necessarily means the rejection of the other. Refer to the figure below:

0

100

200

300

400

5 107 15

Net Present Value(Rs.)

Project L’s Net Present Value Profile

Cross overrabe = 7%

Project S Net Present value profile5

IRR = 14%

IRR = 12%

Cost of Capital %

As long as the cost of capital is greater than the crossover rate of 7 % , then (1) NPV is

larger than NPV and (2) IRR exceeds IRR L . Hence , if the cut off rate or the cost of capital is greater than 7% , both the methods shall lead to selection of project S. However, if the cost of capital is less than 7% , the NPV method ranks Project L higher , but the IRR method indicates that the Project S is better.

S

L S

As can be seen from the above discussion, mutually exclusive projects can create problems with the IRR technique because IRR is expressed as a percentage and does not take into account the scale of investment or the quantum of money earned. Let us consider another example of two mutually exclusive projects A and B with the following details,

6.26

Page 792: 30510870 Cost Accounting and Financial Management

Investment Decisions

Cash flows Year 0 Year 1 IRR NPV(10%) Project A (Rs 1,00,000) Rs 1,50,000 50% Rs 36,360 Project B (Rs 5,00,000) Rs 6,25,000 25% Rs 68,180

Project A earns a return of 50% which is more than what Project B earns; however the NPV of Project B is greater than that of Project A . Acceptance of Project A means that Project B must be rejected since the two Projects are mutually exclusive. Acceptance of Project A also implies that the total investment will be Rs 4,00,000 less than if Project B had been accepted, Rs 4,00,000 being the difference between the initial investment of the two projects. Assuming that the funds are freely available at 10% , the total capital expenditure of the company should be ideally equal to the sum total of all outflows provided they earn more than 10% along with the chosen project from amongst the mutually exclusive. Hence, in case the smaller of the two Projects i.e. Project A is selected, the implication will be of rejecting the investment of additional funds required by the larger investment. This shall lead to a reduction in the shareholders wealth and thus, such an action shall be against the very basic tenets of Financial Management. In the above mentioned example the larger of the two projects had the lower IRR , but never the less provided for the wealth maximising choice. However, it is not safe to assume that a choice can be made between mutually exclusive projects using IRR in cases where the larger project also happens to have the higher IRR . Consider the following two Projects A and B with their relevant cash flows;

Year A B Rs Rs

0 (9,00,000) (8,00,000) 1 7,00,000 62,500 2 6,00,000 6,00,000 3 4,00,000 6,00,000 4 50,000 6,00,000

6.27

Page 793: 30510870 Cost Accounting and Financial Management

Financial Management In this case Project A is the larger investment and also has t a higher IRR as shown below,

Year (Rs) r=46% PV(Rs) (Rs) R=35% PV(Rs) 0 (9,00,000) 1.0 (9,00,000) (8,00,000) 1.0 (8,00,000) 1 7,00,000 0.6849 4,79, 430 62,500 0.7407 46,294 2 6,00,000 0.4691 2,81,460 6,00,000 0.5487 3,29,220 3 4,00,000 0.3213 1,28,520 6,00,000 0.4064 2,43,840 4 50,000 0.2201 11,005 6,00,000 0.3011 1,80,660 (415) 14 IRR A = 46% IRR B = 35%

However, in case the relevant discounting factor is taken as 5% , the NPV of the two projects provides a different picture as follows;

Project A Project B Year (Rs) r= 5% PV(Rs) (Rs) r= 5% PV(Rs) 0 (9,00,000) 1.0 (9,00,000) (8,00,000) 1.0 (8,00,000) 1 7,00,000 0.9524 6,66,680 62,500 0.9524 59,525 2 6,00,000 0.9070 5,44,200 6,00,000 0.9070 5,44,200 3 4,00,000 0.8638 3,45,520 6,00,000 0.8638 5,18,280 4 50,000 0.8227 41,135 6,00,000 0.8227 4,93,630 NPV 6,97,535 8,15,635

As may be seen from the above, Project B should be the one to be selected even though its IRR is lower than that of Project A. This decision shall need to be taken in spite of the fact that Project A has a larger investment coupled with a higher IRR as compared with Project B. This type of an anomalous situation arises because of the reinvestment assumptions implicit in the two evaluation methods of NPV and IRR. The Reinvestment assumption The Net Present Value technique assumes that all cash flows can be reinvested at the discount rate used for calculating the NPV. This is a logical assumption since the use of the NPV technique implies that all projects which provide a higher return than the discounting factor are accepted. In contrast, IRR technique assumes that all cash flows are reinvested at the projects IRR. This assumption means that projects with heavy cash flows in the early

6.28

Page 794: 30510870 Cost Accounting and Financial Management

Investment Decisions

years will be favoured by this method vis-à-vis projects which have got heavy cash flows in the later years. This implicit reinvestment assumption means that Projects like A , with cash flows concentrated in the earlier years of life will be preferred by the method relative to Projects such as B. Projects with unequal lives Let us consider two mutually exclusive projects ‘A’ and ‘B’ with the following cash flows,

Year 0 1 2 Rs Rs Rs Project A (30,000) 20,000 20,000 Project B (30,000) 37,500

The calculation of NPV and IRR could help us evaluate the two projects; however, it is also possible to equate the life span of the two for decision making purposes. The two projects can be equated in terms of time span by assuming that the company can reinvest in Project ‘B’ at the end of year 1. In such a case the cash flows of Project B will appear to be as follows;

Year 0 1 2 Rs Rs Rs Project ‘B’ (30,000) 37,500 Project B reinvested (30,000) 37,500 Total cash flows (30,000) 7,500 37,500 The NPVs and IRRs of both these projects under both the alternatives are shown below

NPV (r=10%) IRR Rs Cash flows (Project A 2Years) NPV = 4,711 A 22%

Cash flows (Project B 1 Year) NPV = 4,090 B 25%

Adjusted cash flows (Project A 2 Years) NPV = 4,711 A 22%

Adjusted cash flow (Project B 2 years) NPV = 7,810 B 25%

6.29

Page 795: 30510870 Cost Accounting and Financial Management

Financial Management As may be seen from the above analysis, the ranking as per IRR makes Project B superior to Project A, irrespective of the period over which the assumption is made. However, when we consider NPV, the decision shall be favouring Project B; in case the re investment assumption is taken into account. This is diametrically opposite to the decision on the basis of NPV when re investment is not assumed. In that case the NPV of Project A makes it the preferred project. Hence, in case it is possible to re invest as shown in the example above, it is advisable to compare and analyse alternative projects by considering equal lives, however, this process cannot be generalised to be the best practice, as every case shall need to be judged on its own distinctive merits. Comparisons over differing lives are perfectly fine if there is no presumption that the company will be required or shall decide to re invest in similar assets. Multiple Internal Rate of Return: In cases where project cash flows change signs or reverse during the life of a project e.g. an initial cash outflow is followed by cash inflows and subsequently followed by a major cash outflow , there may be more than one IRR. The following graph of discount rate versus NPV may be used as an illustration;

IRR2

NPV

IRR

Discount Role

In such situations if the cost of capital is less than the two IRRs , a decision can be made easily , however otherwise the IRR decision rule may turn out to be misleading as the project should only be invested if the cost of capital is between IRR1 and IRR2. To understand the concept of multiple IRRs it is necessary to understand the implicit re investment assumption in both NPV and IRR techniques. Advantages

6.30

Page 796: 30510870 Cost Accounting and Financial Management

Investment Decisions

(i) This method makes use of the concept of time value of money. (ii) All the cash flows in the project are considered.

6.31

Page 797: 30510870 Cost Accounting and Financial Management

Financial Management (iii) IRR is easier to use as instantaneous understanding of desirability can be determined by

comparing it with the cost of capital (iv) IRR technique helps in achieving the objective of minimisation of shareholders wealth. Limitations

(i) The calculation process is tedious if there are more than one cash outflows interspersed between the cash inflows, there can be multiple IRRs, the interpretation of which is difficult.

(ii) The IRR approach creates a peculiar situation if we compare two projects with different inflow/outflow patterns.

(iii) It is assumed that under this method all the future cash inflows of a proposal are reinvested at a rate equal to the IRR. It is ridiculous to imagine that the same firm has a ability to reinvest the cash flows at a rate equal to IRR.

(iv) If mutually exclusive projects are considered as investment options which have considerably different cash outlays. A project with a larger fund commitment but lower IRR contributes more in terms of absolute NPV and increases the shareholders’ wealth. In such situation decisions based only on IRR criterion may not be correct.

Modified Internal Rate of Return (MIRR): As mentioned earlier, there are several limitations attached with the concept of the conventional Internal Rate of Return. The MIRR addresses some of these deficiencies e.g, it eliminates multiple IRR rates; it addresses the reinvestment rate issue and produces results which are consistent with the Net Present Value method. Under this method , all cash flows , apart from the initial investment , are brought to the terminal value using an appropriate discount rate(usually the Cost of Capital). This results in a single stream of cash inflow in the terminal year. The MIRR is obtained by assuming a single outflow in the zeroth year and the terminal cash in flow as mentioned above. The discount rate which equates the present value of the terminal cash in flow to the zeroth year outflow is called the MIRR. Illustration 12 Using details given in illustration 11, calculate MIRR considering a 8% Cost of Capital .

6.32

Page 798: 30510870 Cost Accounting and Financial Management

Investment Decisions

Solution Year Cash flow

Rs 0 1,36,000 The net cash flows from the investment shall be compounded to the terminal year at 8% as follows,

Year Cash flow @8% reinvestment rate factor Rs. 1 30,000 1.3605 40,8152 40,000 1.2597 50,3883 60,000 1.1664 69,9844 30,000 1.0800 32,4005 20,000 1.0000 20,000

MIRR of the investment based on a single cash in flow of Rs 2,13,587 and a zeroth year cash out flow of Rs 1,36,000 is 9.4% (approximately) Comparison of Net Present Value and Internal Rate of Return Methods Both the net present value and the internal rate of return methods are discounted cash flow methods which mean that they consider the time value of money. The time value of money takes into account that cash received today is worth more than cash received at any future time because cash received today can be invested at a specified interest rate. The time value of money is the opportunity cost (forgone interest) from not having the cash today. Additionally, both these techniques consider all cash flows over the expected useful life of the investment. Because of these two characteristics, discounted cash flow techniques are considered to be the most theoretically sound methods for evaluating capital investments. There are circumstances under which the net present value method and the internal rate of return methods will reach different conclusions. Results may vary significantly when capital investment projects differ in (1) amount of initial investments, (2) net cash flow patterns, or (3) length of useful lives. In addition, these two methods can yield different conclusions in situations with (4) varying costs of capital over the life of a project and (5) multiple investments. To use capital budgeting techniques properly, these situations must be understood. (1) The net present value method will favour a project with a large initial investment because the project is more likely to generate large net cash inflows. Because the internal rate of return method uses percentages to evaluate the relative profitability of an investment, the

6.33

Page 799: 30510870 Cost Accounting and Financial Management

Financial Management amount of the initial investment has no effect on the outcome. Therefore, the internal rate of return method is more appropriate for assessing investments requiring significantly different initial investments. (2) Differences in the timing and amount of net cash inflows affect a project’s internal rate of return. This results from the fact that the internal rate of return method assumes that all net cash inflows from a project earn the same rate of return as the project’s internal rate of return. In contrast, the net present value method assumes that all net cash inflows from an investment earn the desired rate of return used in the calculation. The desired rate of return used by the net present value method is usually the organization’s weighted-average cost of capital, a more conservative and more realistic expectation in most cases. (3) Both methods favour projects with long useful lives as long as a project earns positive net cash inflow during the extended years. As long as the net cash inflow in a year is positive, no matter how small, the net present value increases, and the projects desirability improves. Likewise, the internal rate of return method considers each additional useful year of a project another year that its cumulative net cash inflow will earn a return equal to the project’s internal rate of return. A problem arises when an organization is forgoing more beneficial opportunities to continue a project. For example, there might be uses of space or talent where the organization would earn a higher return than the return from the continuation of the project. (4) As an organization’s financial condition or operating environment changes, its cost of capital could also change. A proper capital budgeting procedure should incorporate changes in the organization’s cost of capital or desired rate of return in evaluating capital investments. The net present value method can accommodate different rates of return over the years by using the appropriate discount rates for the net cash inflow of different periods. The internal rate of return method calculates a single rate that reflects the return of the project under consideration and cannot easily handle situations with varying desired rates of return. (5) The net present value method evaluates investment projects in cash amounts while the internal rate of return method evaluates investment projects in percentages or rates. The net present values from multiple projects can be added to arrive at a single total net present value for all investments while the percentages or rates of return on multiple projects cannot be added to determine an overall rate of return. A combination of projects requires a recalculation of the internal rate of return. Illustration 13 CXC Company is preparing the capital budget for the next fiscal year and has identified the following capital investment projects: Project A: Redesign and modification of an existing product that is current scheduled to be terminated. The enhanced model would be sold for six more years.

6.34

Page 800: 30510870 Cost Accounting and Financial Management

Investment Decisions

Project B: Expansion of a product that has been produced on an experimental basis for the past year. The expected life of the product line is eight years. Project C: Reorganization of the plant’s distribution centre, including the installation of computerized equipment for tracking inventory. Project D: Addition of a new product. In addition to new manufacturing equipment, a significant amount of introductory advertising would be required. If this project is implemented, Project A would not be feasible due to limited capacity. Project E: Automation of the Packaging Department that would result in cost savings over the next six years. Project F: Construction of a building wing to accommodate offices presently located in an area that could be used for manufacturing. This change would not add capacity for new lines but would alleviate crowded conditions that currently exist, making it possible to improve the productivity of two existing product lines that have been unable to meet market demand. The cost of capital for CXC Company is 12%, and it is assumed that any funds not invested in capital projects and any funds released at the end of a project can be invested at this rate. As a benchmark for the accounting (book) rate of return, CXC has traditionally used 10%. Further information about the projects is shown below. Project A Project B Project C Project D Project E Project FCapital Investment 106,000 200,000 140,000 160,000 144,000 130,000 Net Present Value @12% 69,683 23,773 (10,228) 74,374 6,027 69,513 Internal Rate of Return 35% 15% 9% 27% 14% 26% Payback Period 2.2 years 4.5 years 3.9 years 4.3 years 2.9 years 3.3 years Accounting Rate of Return 18% 9% 6% 21% 12% 18% If CXC Company has no budget restrictions for capital expenditures and wishes to maximize stakeholder value, the company would choose, based on the given information, to proceed with Projects A or D (mutually exclusive projects), B, E, and F. All of these projects have a positive net present value and an internal rate of return that is greater that the hurdle rate or cost of capital. Consequently, any one of these projects will enhance stakeholder value. Project C is omitted because it has a negative net present value and the internal rate of return is below the 12% cost of capital. With regard to the mutually exclusive projects, the selection of Project A or Project D is dependent on the valuation technique used for selection. If net present value is the only technique used, CXC Company would select Projects B, D, E, and F with a combined net present value of 173,687, the maximum total available. If either the payback method or the

6.35

Page 801: 30510870 Cost Accounting and Financial Management

Financial Management internal rate of return is used for selection, Projects A, B, E, and F would be chosen as Project A has a considerably shorter payback period than Project D, and Project A also has a higher internal rate of return that Project D. The accounting rate of return for these two projects is quite similar and does not provide much additional information to inform the company’s decision. The deciding factor for CXC Company between Projects A and D could very well be the payback period and the size of the initial investment; with Project A, the company would be putting fewer funds at risk for a shorter period of time. If CXC Company were to use the accounting rate of return as the sole measurement criteria for selecting projects, Project B would not be selected. It is clear from the other measures that Project B will increase stakeholder value and should be implemented if CXC has no budget restrictions; this clearly illustrates the necessity that multiple measures be used when selecting capital investment projects. Rather than an unrestricted budget, let us assume that the CXC capital budget is limited to 4,50,000. In cases where there are budget limitations (referred to as capital rationing), the use of the net present value technique is generally recommended as the highest total net present value of the group of projects that fits within the budget limitation will provide the greatest increase in stakeholder value. The combination of Projects A, B, and F will yield the highest net present value to CXC Company for an investment of 436,000.

Self Examination Questions

A. Objective Type Questions 1. A capital budgeting technique which does not require the computation of cost of capital

for decision making purposes is, (a) Net Present Value method (b) Internal Rate of Return method (c) Modified Internal Rate of Return method (d) Pay back

2. If two alternative proposals are such that the acceptance of one shall exclude the possibility of the acceptance of another then such decision making will lead to, (a) Mutually exclusive decisions (b) Accept reject decisions (c) Contingent decisions (d) None of the above

6.36

Page 802: 30510870 Cost Accounting and Financial Management

Investment Decisions

3. In case a company considers a discounting factor higher than the cost of capital for arriving at present values, the present values of cash inflows will be (a) Less than those computed on the basis of cost of capital (b) More than those computed on the basis of cost of capital (c) Equal to those computed on the basis of the cost of capital (d) None of the above

4. The pay back technique is specially useful during times (a) When the value of money is turbulent (b) When there is no inflation (c) When the economy is growing at a steady rate coupled with minimal inflation. (d) None of the above

5. While evaluating capital investment proposals, time value of money is used in which of the following techniques, (a) Payback method (b) Accounting rate of return (c) Net present value (d) None of the above

6. IRR would favour project proposals which have, (a) Heavy cash inflows in the early stages of the project. (b) Evenly distributed cash inflows throughout the project. (c) Heavy cash inflows at the later stages of the project (d) None of the above.

7. The re investment assumption in the case of the IRR technique assumes that, (a) Cash flows can be re invested at the projects IRR (b) Cash flows can be re invested at the weighted cost of capital (c) Cash flows can be re invested at the marginal cost of capital (d) None of the above

8. Multiple IRRs are obtained when, (a) Cash flows in the early stages of the project exceed cash flows during the later

stages.

6.37

Page 803: 30510870 Cost Accounting and Financial Management

Financial Management

(b) Cash flows reverse their signs during the project (c) Cash flows are un even (d) None of the above.

9. Depreciation is included as a cost in which of the following techniques, (a) Accounting rate of return (b) Net present value (c) Internal rate of return (d) None of the above

10. Management is considering a Rs 1,00,000 investment in a project with a 5 year life and no residual value . If the total income from the project is expected to be Rs 60,000 and recognition is given to the effect of straight line depreciation on the investment, the average rate of return is : (a) 12% (b) 24% (c) 60% (d) 75%

11. Assume cash outflow equals Rs 1,20,000 followed by cash inflows of Rs 25,000 per year for 8 years and a cost of capital of 11%. What is the Net present value? (a) (Rs 38,214) (b) Rs 9,653 (c) Rs 8,653 (d) Rs 38,214

12. What is the Internal rate of return for a project having cash flows of Rs 40,000 per year for 10 years and a cost of Rs 2,26,009? (a) 8% (b) 9% (c) 10% (c) 12%

6.38

Page 804: 30510870 Cost Accounting and Financial Management

Investment Decisions

13. While evaluating investments, the release of working capital at the end of the projects life should be considered as, (a) Cash in flow (b) Cash out flow (c) Having no effect upon the capital budgeting decision (d) None of the above.

14. Capital rationing refers to a situation where, (a) Funds are restricted and the management has to choose from amongst available

alternative investments. (b) Funds are unlimited and the management has to decide how to allocate them to

suitable projects. (c) Very few feasible investment proposals are available with the management. (d) None of the above

15. Capital budgeting is done for (a) Evaluating short term investment decisions. (b) Evaluating medium term investment decisions. (c) Evaluating long term investment decisions. (d) None of the above

Answers to Objective Type Questions 1. (d); 2. (a); 3. (a) ; 4. (a); 5. (c) ; 6. (a); 7. (a); 8. (b); 9. (a); 10. (b); 11. (c); 12. (d); 13. (a); 14. (a); 15. (c).

B. Short Answer Type Questions 1. Define the following terms:

(a) Capital Budgeting (b) Regular payback period and discounted payback period (c) Independent projects and mutually exclusive projects. (d) Internal rate of return method and modified rate of return method. (e) Net Present Value method. (f) Capital rationing.

6.39

Page 805: 30510870 Cost Accounting and Financial Management

Financial Management 2. Why is discounted cash flow a superior method for capital budgeting? 3. “The higher the cut off rate, the more will be the company willing to pay for cost saving

equipment”. Discuss. 4. Does the IRR model make significantly different decisions than the NPV method?

Discuss. 5. Two projects have an identical Net Present Value of Rs 50,000. Are both projects equal

in desirability. 6. What are the basic objections to the use of Average Rate of Return concept for

evaluating projects? 7. Discuss the principal limitations of the cash payback period for evaluating capital

investment proposals. 8. Your boss has suggested that a one year payback means 100 % average returns. Do you

agree? C. Long Answer Type Questions 1. Explain why, if two mutually exclusive projects are being compared, the short term

project might have the higher ranking under NPV criteria if the cost of capital is high, but the long term project will be deemed better if the cost of capital is low. Would changes in the cost of capital ever cause a change in the IRR ranking of two such projects?

2. In what sense is a reinvestment rate assumption embodied in the NPV , IRR and MIRR methods? What is the assumed reinvestment rate of each method?

3. Discuss in detail the ‘Capital Budgeting Process’ 4. What are the various types of Capital Investment decisions known to you? 5. Discuss the basic principles for measuring project cash flows. D. Practical Problems 1. (a) You are required to calculate the total present value of inflow at rate of discount of

12% of following data. Year end Cash inflows

Rs.

1 2,30,000 2 2,28,000 3 2,78,000

6.40

Page 806: 30510870 Cost Accounting and Financial Management

Investment Decisions

4 2,83,000 5 2,73,000 6 80,000 (Scrap Value)

(b) Considering the data given in the above. Calculate the total present value of inflows

and outflows if the rate of discount is 10% assuming that Rs. 10,00,000 of outflows would be spent as follows:

Beginning of year 1 Rs. 2,50,000

Beginning of year 2 Rs. 2,50,000

Beginning of year 3 Rs. 2,50,000

Beginning of year 4 Rs. 2,50,000

2. Consider the following example of cash flows from two projects.

Total 3,15,000 2,30,000

No. of years Project A Project B1 Nil 40,0002 Nil 50,0003 5,000 1,20,0004 20,000 10,0005 50,000 10,0006 1,50,000 Nil7 50,000 Nil8 40,000 Nil

Both projects cost Rs. 1,50,000 each . You are required to compute the payback period for both projects. Which project will you prefer?

3. A company wants to replace its old machine with a new automatic machine. Two models A and B are available at the same cost of Rs. 5 lakhs each. Salvage value of the old machine is Rs. 1 lakh. The utilities of the existing machine can be used if the company purchases model A. Additional cost of utilities to be purchased in that case are Rs. 1 lakh. If the company purchases model B then all the existing utilities will have to be replaced with new utilities costing Rs. 2 lakhs. The salvage value of the old utilities will

6.41

Page 807: 30510870 Cost Accounting and Financial Management

Financial Management

be Rs. 0.20 lakhs. The earnings after taxation are expected to be as follows :

(Cash inflows)

Year/Model A B P.V. Factor Rs. Rs. @ 15%1. 1,00,000 2,00,000 0.872. 1,50,000 2,10,000 0.763. 1,80,000 1,80,000 0.664. 2,00,000 1,70,000 0.575. 1,70,000 40,000 0.50Salvage value at the end of Year 5

50,000 60,000

The targeted return on capital is 15% you are required to : (i) Compute, for the two machines separately, net present value, discounted payback

period and desirability factor and (ii) Advice which of the machines is to be selected ?

4. Suppose the management of a concern is considering two projects both involving Rs. 10,00,000 each and having profits after tax and depreciation as follows:

Year A B Rs. Rs.

1 50,000 Nil

2 75,000 Nil3 1,25,000 Nil4 1,30,000 2,30,0005 80,000 2,30,000

Total 4,60,000 4,60,000 Suppose further that both projects can be sold for Rs. 80,000 each after 5 years. You are required to compute the annual rate of return of both the projects. Will you consider both projects to be equal?

6.42

Page 808: 30510870 Cost Accounting and Financial Management

Investment Decisions

5. A product is currently manufactured on a plant which is not fully depreciated for tax purposes and has a book value of Rs., 60,000 (it was bought for Rs. 1,20,000 six years ago). The cost of the product is as under:

Unit cost Rs.

Direct costs 24.00 Indirect labour 8.00 Other variable overheads 16.00 Fixed overheads 16.00

Rs. 64.00

10,000 units are normally produced. It is expected that the old machine can be used, indefinitely into the future, after suitable repairs, estimated to cost Rs. 40,000 annually, are carried out. A manufacturer of machinery is offering a new machine with the latest technology at Rs.3,00,000 after trading off the old plant for Rs. 30,000. The projected cost of the product will then be :

Per unit Rs.

Direct costs 14.00 Indirect labour 12.00 Other variable overheads 12.00 Fixed overheads 20.00

58.00 The fixed overheads are allocations from other departments pls the depreciation of plant and machinery. The old machine can be sold for Rs. 40,000 in the open market. The new machine is expected to last for 10 years at the end of which, its salvage value will be Rs. 20,000. Rate of corporate taxation is 50%. For tax purposes, the cost of the new machine and that of the old one may be depreciated in 10 years. The minimum rate of return expected is 10%. It is also anticipated that in future the demand for the products will stay to 10,000 units. Advise whether the new machine can be purchased ignore capital gains taxes. Present value of Re. 1 at 10% for years 1-10 are : .909, .826, .751, .683, .621, .564, .513, .467, .424 and .383 respectively.

6.43

Page 809: 30510870 Cost Accounting and Financial Management

Financial Management 6. Modern Enterprises Ltd. is considering the purchase of a new computer system for its

Research and Development Division, which would cost Rs. 35 lakhs. The operation and maintenance costs (excluding depreciation) are expected to be Rs. 7 lakhs per annum. It is estimated that the useful life of the system would be 6 years, at the end of which the disposal value is expected to be Rs. 1 lakh. The tangible benefits expected from the system in the form of reduction in design and drafts-menship costs would be Rs. 12 lakhs per annum. Besides, the disposal of used drawing, office equipment and furniture, initially, is anticipated to net Rs. 9 lakhs. Capital expenditure in research and development would attract 100% write-off for tax purpose. The gains arising from disposal of used assets may be considered tax-free. The company’s effective tax rate is 50%. The average cost of capital to the company is 12%. The present value factors at 12% discount rate are :

Year PVF 1 0.892 2 0.797 3 0.711 4 0.635 5 0.567 6 0.506

After appropriate analysis of cash flows, please advise the company of the financial viability of the proposal.

7. A sole trader installs plant and machinery in rented premises for the production of luxury article, the demand for which is expected to last only 5 years. The total capital put in by the sole trader is as under:

Plant and Machinery Rs. 2,70,500 Working Capital Rs. 40,000

Rs. 3,10,500

The working capital will be fully realised at the end of the 5th year. The scrap value of the plant expected to be realised at the end of the 5th year is only Rs. 5,500. The trader’s earnings are expected to be as under :

6.44

Page 810: 30510870 Cost Accounting and Financial Management

Investment Decisions

Years Cash profits after depreciation and tax Tax payable

Rs. Rs.

1 90,000 20,000

2 1,30,000 30,000

3 1,70,000 40,000

4 1,16,000 26,000

5 19,500 5,000

Present value factors of various rates of interest are given below :

Years 11% 12% 13% 14% 15%

0.9009 0.8929 0.8850 0.8770 0.8696

2 0.8116 0.7972 0.7831 0.7695 0.7561

3 0.7312 0.7118 0.6931 0.6750 0.6575

4 0.6587 0.6355 0.6133 0.5921 0.5718

5 0.5935 0.5674 0.5428 0.5194 0.4972

You are required to compute the present value of cash flows discounted at the various

rates of interests given above and state the return from the project. (3,34,172; 3,25,996; 3,18,128; 3,10,543; 3,03,251 : Yield 14%)

8. The Alpha Co. Ltd, is considering the purchase of a new machine. Two alternative machines (A & B) have been suggested, each costing Rs. 4,00,000. Earnings after taxation but before depreciation are expected to be as follows :

6.45

Page 811: 30510870 Cost Accounting and Financial Management

Financial Management

Year Cash Flows Machine A Machine B Rs. Rs.

1 40,000 1,20,0002 1,20,000 1,60,0003 1,60,000 2,00,0004 2,40,000 1,20,0005 1,60,000 80,000Total 7,20,000 6,80,000

The company has a target rate return on capital @ 10 percent and on this basis, you are

required : (a) Compare profitability of the machines and state which alternative you consider

financially preferable, (b) Compute the pay back period for each project and, (c) Compute annual rate of return for each project. (Present value of machine B is higher than that of machine A; Payback period

machine A – 3 years 4 months, machine B 2 years 7.2 months ; Annual return machine A – 16%, machine B – 14%)

9. Company X is forced to choose between two machines A and B. The two machines are designed differently, but have identical capacity and do exactly the same job. Machine A costs Rs. 1,50,000 and will last for 3 years. It costs Rs. 40,000 per year to run. Machine B is an ‘economy’ model costing only Rs. 1,00,000, but will last only for 2 years, and costs Rs. 60,000 per year to run. These are real cash flows. The costs are forecasted in rupees of constant purchasing power. Ignore tax. Opportunity cost of capital is 10 percent. Which machine company X should buy ?

10. S Engineering Company is considering replacing or repairing a particular machine, which has just broken down. Last year this machine costed Rs. 20,000 to run and maintain. These costs have been increasing in real terms in recent years with the age of the machine. A further useful life of 5 years is expected, if immediate repairs of Rs. 19,000 are carried out. If the machine is not repaired it can be sold immediately to realise about Rs. 5,000 (Ignore loss/gain on such disposal)

6.46

Page 812: 30510870 Cost Accounting and Financial Management

Investment Decisions

Alternatively, the company can buy a new machine for Rs. 49,000 with an expected life of 10 years with no salvage value after providing depreciation on straight line basis. In this case, running and maintenance costs will reduce to Rs. 14,000 each year and are not expected to increase much in real terms for a few years at least. S Engineering Company regard a normal return of 10% p.a. after tax as a minimum requirement on any new investment. Considering capital budgeting techniques, which alternative will you choose? Take corporate tax rate of 50% and assume that depreciation on straight line basis will be accepted for tax purposes also. Given cumulative present value of Re. 1 p.a. at 10% for 5 years Rs. 3.791 and for 10 years Rs. 6.145.

6.47

Page 813: 30510870 Cost Accounting and Financial Management

CHAPTER 7

MANAGEMENT OF WORKING CAPITAL

UNIT – I : MEANING, CONCEPT AND POLICIES OF WORKING CAPITAL

Learning Objectives After studying this chapter, you will be able to understand ♦ The meaning and the significance of working capital management; ♦ The concept of operating cycle and the estimation of working capital needs; ♦ The need for investing in current assets; ♦ The need for managing current assets and current liabilities; and ♦ Financing of working capital.

1.1 INTRODUCTION Decisions relating to working capital and short term financing are referred to as Working Capital Management. These involve managing the relationship between a firm’s short-term assets and its short-term liabilities. The goal of working capital management is to ensure that the firm is able to continue its operations and that it has sufficient cash flow to satisfy both maturing short-term debt and upcoming operational expenses.

1.2 MEANING AND CONCEPT OF WORKING CAPITAL There are two concepts of working capital - gross and net. Gross working capital refers to the firm’s investment in current assets. Current assets are those assets which can be converted into cash within an accounting year. Net working capital refers to the difference between current assets and current liabilities. Current liabilities are those claims of outsiders which are expected to mature for payment within an accounting year. Current Assets include: Stocks of raw materials, Work-in-progress, Finished goods, Trade debtors, Prepayments, Cash balances etc. Current Liabilities include: Trade creditors, Accruals, Taxation payable, Bills Payables, Outstanding expenses, Dividends payable, short term loans.

Page 814: 30510870 Cost Accounting and Financial Management

Financial Management Working capital is also known as operating capital. A most important value, it represents the amount of day-to-day operating liquidity available to a business. A company can be endowed with assets and profitability, but short of liquidity if these assets cannot readily be converted into cash. A positive working capital means that the company is able to payoff its short-term liabilities. A negative working capital means that the company currently is unable to meet its short-term liabilities. From the point of view of time, the term working capital can be divided into two categories viz., Permanent and temporary. Permanent working capital refers to the hard core working capital. It is that minimum level of investment in the current assets that is carried by the business at all times to carry out minimum level of its activities. Temporary working capital refers to that part of total working capital, which is required by a business over and above permanent working capital. It is also called variable working capital. Since the volume of temporary working capital keeps on fluctuating from time to time according to the business activities it may be financed from short-term sources. The following diagrams shows Permanent and Temporary or Fluctuating or variable working capital

7.2

Page 815: 30510870 Cost Accounting and Financial Management

Management of Working Capital

Both kinds of working capital i.e. permanent and fluctuating (temporary) are necessary to facilitate production and sales through the operating cycle. 1.2.1 Importance of Adequate Working Capital: The importance of adequate working capital in commercial undertakings can be judged from the fact that a concern needs funds for its day-to-day running. Adequacy or inadequacy of these funds would determine the efficiency with which the daily business may be carried on. Management of working capital is an essential task of the finance manager. He has to ensure that the amount of working capital available with his concern is neither too large nor too small for its requirements. A large amount of working capital would mean that the company has idle funds. Since funds have a cost, the company has to pay huge amount as interest on such funds. The various studies conducted by the Bureau of Public Enterprises have shown that one of the reason for the poor performance of public sector undertakings in our country has been the large amount of funds locked up in working capital This results in over capitalization. Over capitalization implies that a company has too large funds for its requirements, resulting in a low rate of return a situation which implies a less than optimal use of resources. A firm has, therefore, to be very careful in estimating its working capital requirements. If the firm has inadequate working capital, it is said to be under-capitalised. Such a firm runs the risk of insolvency. This is because, paucity of working capital may lead to a situation where the firm may not be able to meet its liabilities. It is interesting to note that many firms which are otherwise prosperous (having good demand for their products and enjoying profitable marketing conditions) may fail because of lack of liquid resources. If a firm has insufficient working capital and tries to increase sales, it can easily over-stretch the financial resources of the business. This is called overtrading. Early warning signs of over trading include: ♦ Pressure on existing cash. ♦ Exceptional cash generating activities e.g., offering high discounts for early cash payment. ♦ Bank overdraft exceeds authorized limit. ♦ Seeking greater overdrafts or lines of credit. ♦ Part-paying suppliers or other creditors. ♦ Paying bills in cash to secure additional supplies. ♦ Management pre-occupation with surviving rather than managing. ♦ Frequent short-term emergency requests to the bank (to help pay wages, pending receipt

of a cheque).

7.3

Page 816: 30510870 Cost Accounting and Financial Management

Financial Management Every business needs adequate liquid resources in order to maintain day-to-day cash flow. It needs enough cash to pay wages and salaries as they fall due and to pay creditors if it is to keep its workforce engaged and ensure its supplies. Maintaining adequate working capital is not just important in the short-term. Sufficient liquidity must be maintained in order to ensure the survival of the business in the long-term as well. Even a profitable business may fail if it does not have adequate cash flow to meet its liabilities as they fall due. Therefore, when business make investment decisions they must not only consider the financial outlay involved with acquiring the new machine or the new building, etc., but must also take account of the additional current assets that are usually required with any expansion of activity. Increased production leads to hold additional stocks of raw materials and work in progress. Increased sales usually means that the level of debtors will increase. A general increase in the firm’s scale of operations tends to imply a need for greater levels of working capital. A question then arises what is an optimum amount of working capital for a firm? We can say that a firm should neither have too high an amount of working capital nor should the same be too low. It is the job of the finance manager to estimate the requirements of working capital carefully and determine the optimum level of investment in working capital. 1.2.2 Optimum Working Capital: If a company’s current assets do not exceed its current liabilities, then it may run into trouble with creditors that want their money quickly. The working capital ratio, which measures this ability to pay back can be calculated as current assets divided by current liabilities. Current ratio (current assets/current liabilities) (along with acid test ratio to supplement it) has traditionally been considered the best indicator of the working capital situation. It is understood that a current ratio of 2 (two) for a manufacturing firm implies that the firm has an optimum amount of working capital. This is supplemented by Acid Test Ratio (Quick assets/Current liabilities) which should be at least 1 (one). Thus it is considered that there is a comfortable liquidity position if liquid current assets are equal to current liabilities. Bankers, financial institutions, financial analysts, investors and other people interested in financial statements have, for years, considered the current ratio at, ‘two’ and the acid test ratio at, ‘one’ as indicators of a good working capital situation. As a thumb rule, this may be quite adequate. However, it should be remembered that optimum working capital can be determined only with reference to the particular circumstances of a specific situation. Thus, in a company where the inventories are easily saleable and the sundry debtors are as good as liquid cash, the current ratio may be lower than 2 and yet firm may be sound. An optimum working capital ratio is dependent upon the business situation as such and the nature and composition of various current assets. A company having short conversion cycle (from cash to cash) my have a lower current ratio.

7.4

Page 817: 30510870 Cost Accounting and Financial Management

Management of Working Capital

In nutshell, a firm needs to maintain a sound working capital position. It should have adequate working capital to run its business operations. Both excessive as well as inadequate working capital positions are dangerous. Excessive working capital means holding costs and idle funds which earn no profits for the firm. Paucity of working capital not only impairs the firm’s profitability but also results in production interruptions, inefficiencies and sales disruptions. The management should therefore maintain the right amount of working capital continuously. 1.3 MANAGEMENT OF WORKING CAPITAL Working capital management is the functional area of finance that covers all the current accounts of its firm. It is concerned with management of the level of individual current assets and the current liabilities or in other words the management of total working capital. Managing Working Capital is a matter of balance. A firm must have sufficient cash on hand to meet its immediate needs while ensuring that idle cash is invested to the organizations best possible advantage. To avoid the difficulties, it is necessary to have clear and accurate reports on each of the components of working capital and an awareness of the potential impact of likely influences. Sound financial and statistical techniques, supported by judgement should be used to predict the quantum of working capital required at different times. Adequate provisions of working capital mitigates risk. Working capital management entails short-term decisions generally, relating to its next one year period which are “reversible”. Management will use a combination of policies and techniques for the management of working capital. These require managing the current assets – generally cash and cash equivalents, inventories and debtors. There are also a variety of short term financing options which are considered. The various steps in the management of working capital involve: ♦ Cash management – Identify the cash balance which allows for the business to meet day

to day expenses, but reduces cash holding costs. ♦ Inventory management – Identify the level of inventory which allows for uninterrupted

production but reduces the investment in raw materials and hence increases cash flow; The techniques like Just In Time (JIT) and Economic order quantity (EOQ) are used for this.

♦ Debtors management – Identify the appropriate credit policy, i.e., credit terms which will attract customers, such that any impact on cash flows and the cash conversion cycle will be offset by increased revenue and hence Return on Capital (or vice versa). The tools like Discounts and allowances are used for this.

♦ Short term financing – Inventory is ideally financed by credit granted by the supplier; dependent on the cash conversion cycle, it may however, be necessary to utilize a bank loan (or overdraft), or to “convert debtors to cash” through “factoring” in order to finance

7.5

Page 818: 30510870 Cost Accounting and Financial Management

Financial Management

working capital requirements. There are, however, certain constraints in the management of working capital such as: (i) Non-realisation of the importance of working capital. (ii) Continuous inflation in the economy. (iii) The existence of seller’s market or monopoly conditions; and (iv) High profitability. 1.3.1 Determinants of Working Capital: The following factors will generally influence the working capital requirements of the firm: (i) Nature of Business. (ii) Market and demand conditions. (iii) Technology and manufacturing Policies. (iv) Credit Policy of the firm. (v) Availability of credit from suppliers. (vi) Operating efficiency. (vii) Price Level Changes. 1.4 ISSUES IN THE WORKING CAPITAL MANAGEMENT Working capital management entails the control and monitoring of all components of working capital i.e. cash, marketable securities, debtors (receivables) and stocks (inventories) and creditors (payables). The finance manager has to determine the levels and composition of current assets. He has to ensure a right mix of different current assets and that current liabilities are paid in time. There are many aspects of working capital management which makes it important function of financial management. ♦ Time: Working capital management requires much of the finance manager’s time. ♦ Investment: Working capital represents a large portion of the total investment in assets. ♦ Credibility: Working capital management has great significance for all firms but it is very

critical for small firms. ♦ Growth: The need for working capital is directly related to the firm’s growth. It is advisable that the finance manager should take precautionary measures for effective and efficient management of working capital. He has to pay particular attention to the levels of

7.6

Page 819: 30510870 Cost Accounting and Financial Management

Management of Working Capital

current assets and their financing. To decide the levels and financing of current assets, the risk return trade off must be taken into account. 1.4.1 Current Assets to Fixed Assets Ratio: The finance manager is required to determine the optimum level of current assets so that the shareholders value is maximized. A firm needs fixed and current assets to support a particular level of output. However, to support the same level of output, the firm can have different levels of current assets. As the firm’s output and sales increases, the need for current assets also increases. Generally, current assets do not increase in direct proportion to output, current assets may increase at a decreasing rate with output. This relationship is based upon the notion that it takes a greater proportional investment in current assets when only a few units of output are produced than it does later on when the firm can use its current assets more efficiently. The level of the current assets can be measured by creating a relationship between current assets and fixed assets. Dividing current assets by fixed assets gives current assets/fixed assets ratio. Assuming a constant level of fixed assets, a higher current assets/fixed assets ratio indicates a conservative current assets policy and a lower current assets/fixed assets ratio means an aggressive current assets policy assuming all factors to be constant. A conservative policy implies greater liquidity and lower risk whereas an aggressive policy indicates higher risk and poor liquidity. Moderate current assets policy will fall in the middle of conservative and aggressive policies. The current assets policy of most of the firms may fall between these two extreme policies. The following diagram shows alternative current assets policies:

7.7

Page 820: 30510870 Cost Accounting and Financial Management

Financial Management 1.4.2 Liquidity versus Profitability: Risk return trade off − A firm may follow a conservative, aggressive or moderate policy as discussed above. However, these policies involve risk, return trade off. A conservative policy means lower return and risk. While an aggressive policy produces higher return and risk. The two important aims of the working capital management are profitability and solvency. A liquid firm has less risk of insolvency that is, it will hardly experience a cash shortage or a stock out situation. However, there is a cost associated with maintaining a sound liquidity position. However, to have higher profitability the firm may have to sacrifice solvency and maintain a relatively low level of current assets. This will improve firm’s profitability as fewer funds will be tied up in idle current assets, but its solvency would be threatened and exposed to greater risk of cash shortage and stock outs. The following illustration explains the risk-return trade off of various working capital management policies, viz., conservative, aggressive and moderate etc. Illustration 1 A firm has the following data for the year ending 31st March, 2006:

Rs. Sales (1,00,000 @ Rs.20/-) 20,00,000 Earning before Interest and Taxes 2,00,000 Fixed Assets 5,00,000 The three possible current assets holdings of the firm are Rs.5,00,000/-, Rs.4,00,000/- and Rs. 3,00,000. It is assumed that fixed assets level is constant and profits do not vary with current assets levels. The effect of the three alternative current assets policies is as follows: Effect of alternative Working Capital Policies

(Amount in Rs.) Working Capital Policy Conservative Moderate Aggressive Sales 20,00,000 20,00,000 20,00,000 Earnings before Interest and Taxes (EBIT)

2,00,000 2,00,000 2,00,000

Current Assets 5,00,000 4,00,000 3,00,000 Fixed Assets 5,00,000 5,00,000 5,00,000 Total Assets 10,00,000 9,00,000 8,00,000 Return on Total Assets (EBIT/Total Assets)

20% 22.22% 25%

Current Assets/Fixed Assets 1.00 0.80 0.60

7.8

Page 821: 30510870 Cost Accounting and Financial Management

Management of Working Capital

The aforesaid calculations shows that the conservative policy provides greater liquidity (solvency) to the firm, but lower return on total assets. On the other hand, the aggressive policy gives higher return, but low liquidity and thus is very risky. The moderate policy generates return higher than Conservative policy but lower than aggressive policy. This is less risky than Aggressive policy but more risky than conservative policy. In determining the optimum level of current assets, the firm should balance the profitability – Solvency tangle by minimizing total costs. Cost of liquidity and cost of illiquidity. 1.5 ESTIMATING WORKING CAPITAL NEEDS Operating cycle is one of the most reliable method of Computation of Working Capital. However, other methods like ratio of sales and ratio of fixed investment may also be used to determine the Working Capital requirements. These methods are briefly explained as follows: (i) Current assets holding period: To estimate working capital needs based on the

average holding period of current assets and relating them to costs based on the company’s experience in the previous year. This method is essentially based on the Operating Cycle Concept.

(ii) Ratio of sales: To estimate working capital needs as a ratio of sales on the assumption that current assets change with changes in sales.

(iii) Ratio of fixed investments: To estimate Working Capital requirements as a percentage of fixed investments.

A number of factors will, however, be impacting the choice of method of estimating Working Capital. Factors such as seasonal fluctuations, accurate sales forecast, investment cost and variability in sales price would generally be considered. The production cycle and credit and collection policies of the firm will have an impact on Working Capital requirements. Therefore, they should be given due weightage in projecting Working Capital requirements. 1.6 OPERATING OR WORKING CAPITAL CYCLE A useful tool for managing working capital is the operating cycle. The operating cycle analyzes the accounts receivable, inventory and accounts payable cycles in terms of number of days. In other words, accounts receivable are analyzed by the average number of days it takes to collect an account. Inventory is analyzed by the average number of days it takes to turn over the sale of a product (from the point it comes in the store to the point it is converted to cash or an account receivable). Accounts payable are analyzed by the average number of days it takes to pay a supplier invoice. Most businesses cannot finance the operating cycle (accounts receivable days + inventory days) with accounts payable financing alone. Consequently, working capital financing is needed. This shortfall is typically covered by the net profits generated internally or by externally borrowed funds or by a combination of the two.

7.9

Page 822: 30510870 Cost Accounting and Financial Management

Financial Management Most businesses need short-term working capital at some point in their operations. For instance, retailers must find working capital to fund seasonal inventory build-up. But even a business that is not seasonal occasionally experiences peak months when orders are unusually high. This creates a need for working capital to fund the resulting inventory and accounts receivable build-up. Some small businesses have enough cash reserves to fund seasonal working capital needs. However, this is very rare for a new business. If your new venture experiences a need for short-term working capital during its first few years of operation, you will have several potential sources of funding. The important thing is to plan ahead. If you get caught off guard, you might miss out on the one big order. Cash flows in a cycle into, around and out of a business. It is the business’s life blood and every manager’s primary task is to help keep it flowing and to use the cashflow to generate profits. If a business is operating profitably, then it should, in theory, generate cash surpluses. If it doesn’t generate surplus, the business will eventually run out of cash. The faster a business expands, the more cash it will need for working capital and investment. The cheapest and best sources of cash exist as working capital right within business. Good management of working capital will generate cash which will help improve profits and reduce risks. Bear in mind that the cost of providing credit to customers and holding stocks can represent a substantial proportion of a firm’s total profits. There are two elements in the business cycle that absorb cash – Inventory (stocks and work-in-progress) and Receivables (debtors owing you money). The main sources of cash are Payables (your creditors) and Equity and Loans.

Working Capital Cycle

CASH DEBTORS RAW MATERIAL LABOUR OVERHEAD

STOCK WIP

Each component of working capital (namely inventory, receivables and payables) has two dimensions ……TIME ………and MONEY, when it comes to managing working capital then time is money. If you can get money to move faster around the cycle (e.g. collect monies due from debtors more quickly) or reduce the amount of money tied up (e.g. reduce inventory

7.10

Page 823: 30510870 Cost Accounting and Financial Management

Management of Working Capital

levels relative to sales), the business will generate more cash or it will need to borrow less money to fund working capital. As a consequence, you could reduce the cost of bank interest or you will have additional free money available to support additional sales growth or investment. Similarly, if you can negotiate improved terms with suppliers e.g. get longer credit or an increased credit limit, you are effectively creating free finance to help fund future sales.

If you……………… Then …………………. Collect receivables (debtors) faster You release cash from the cycle Collect receivables (debtors) slower Your receivables soak up cash. Get better credit (in terms of duration or amount) from suppliers.

You increase your cash resources.

Shift inventory (stocks) faster You free up cash. Move inventory (stocks) slower. You consume more cash. Working capital cycle indicates the length of time between a company’s paying for materials, entering into stock and receiving the cash from sales of finished goods. It can be determined by adding the number of days required for each stage in the cycle. For example, a company holds raw materials on an average for 60 days, it gets credit from the supplier for 15 days, production process needs 15 days, finished goods are held for 30 days and 30 days credit is extended to debtors. The total of all these, 120 days, i.e., 60 – 15 + 15 + 30 + 30 days is the total working capital cycle. The determination of working capital cycle helps in the forecast, control and management of working capital. It indicates the total time lag and the relative significance of its constituent parts. The duration of working capital cycle may vary depending on the nature of the business. In the form of an equation, the operating cycle process can be expressed as follows: Operating Cycle = R + W + F + D – C Where, R = Raw material storage period W = Work-in-progress holding period F = Finished goods storage period D = Debtors collection period. C = Credit period availed.

7.11

Page 824: 30510870 Cost Accounting and Financial Management

Financial Management

7.12

The various components of operating cycle may be calculated as shown below:

(1) daypernconsumptiomaterialraw ofcost Average

materialrawofstock Averageperiod storage material Raw =

(2) period holdingprogress-in-Work day per production ofcost Average

inventory progress-in-workAverage=

(3) daypersoldgoodsofcostAverage

goodsfinishedofstockAverageperiodstoragegoodsFinished =

(4) day per Sales Credit Average

debts book Averageperiod collection Debtors =

(5) day per purchases credit Average

creditors trade Averageavailed period Credit =

1.6.1 Working Capital Based on Operating Cycle: One of the method for forecasting working capital requirement is based on the concept of operating cycle. The calculation of operating cycle and the formula for estimating working capital on its basis has been demonstrated with the help of following illustration:

Illustration 2

From the following information of XYZ Ltd., you are required to calculate :

(a) Net operating cycle period.

(b) Number of operating cycles in a year.

Rs. (i) Raw material inventory consumed during the year 6,00,000 (ii) Average stock of raw material 50,000 (iii) Work-in-progress inventory 5,00,000 (iv) Average work-in-progress inventory 30,000 (v) Finished goods inventory 8,00,000 (vi) Average finished goods stock held 40,000 (vii) Average collection period from debtors 45 days (viii) Average credit period availed 30 days (ix) No. of days in a year 360 days

Page 825: 30510870 Cost Accounting and Financial Management

Management of Working Capital

The various components of operating cycle may be calculated as shown below:

(1) daypernconsumptiomaterialraw ofcost Average

materialrawofstock Averageperiod storage material Raw =

(2) period holding progress-in-Work day per production ofcost Average

inventory progress-in-workAverage=

(3) daypersoldgoodsofcostAverage

goodsfinishedofstockAverageperiodstoragegoodsFinished =

(4) day per Sales Credit Average

debts book Averageperiod collection Debtors =

(5) day per purchases credit Average

creditors trade Averageavailed period Credit =

1.6.1 Working Capital Based on Operating Cycle: One of the method for forecasting working capital requirement is based on the concept of operating cycle. The calculation of operating cycle and the formula for estimating working capital on its basis has been demonstrated with the help of following illustration: Illustration 2 From the following information of XYZ Ltd., you are required to calculate : (a) Net operating cycle period. (b) Number of operating cycles in a year. Rs. (i) Raw material inventory consumed during the year 6,00,000 (ii) Average stock of raw material 50,000 (iii) Work-in-progress inventory 5,00,000 (iv) Average work-in-progress inventory 30,000 (v) Finished goods inventory 8,00,000 (vi) Average finished goods stock held 40,000 (vii) Average collection period from debtors 45 days (viii) Average credit period availed 30 days (ix) No. of days in a year 360 days

7.13

Page 826: 30510870 Cost Accounting and Financial Management

Financial Management Solution

Calculation of Net Operating Cycle period of XYZ Ltd. Days Raw material storage period: (a) 30

⎟⎟⎠

⎞⎜⎜⎝

⎛ydapernconsumptiomaterialrawofcostAverage

materialrawofstockAverage

(Rs. 50,000 / 1667*) *(Rs. 6,00,000 / 360 days) W.I.P. holding period : (b) 22

⎟⎟⎠

⎞⎜⎜⎝

⎛ −−dayperproductionof costAverage

inventoryprogressinworkAverage

Rs. 30,000 / 1,388)** **(Rs. 5,00,000 / 360 days) Finished goods storage period : (c) 18

⎟⎟⎠

⎞⎜⎜⎝

⎛daypersoldgoodsof costAverage

goodsfinishedofstockAverage

(Rs.40,000 / 2,222)*** ***(Rs. 8,00,000 / 360 days) Debtors collection period: (d) 45 Total operating cycle period: 115 [(a) + (b) + (c) + (d)] Less: Average credit period availed 30 (i) Net operating cycle period 85 (ii) Number of operating cycles in a year 4.2 (360 days / 85 days) The net operating cycle represents the net time gap between investment of cash and its recovery of sales revenue. If depreciation is excluded from expenses in the computation of operating cycle, the net operating cycle also represents the cash conversion cycle. It is the

7.14

Page 827: 30510870 Cost Accounting and Financial Management

Management of Working Capital

net time interval between cash collections from sales of product and cash payments for the resources acquired by the firm. The Finance Manager is required to manage the operating cycle effectively and efficiently. The length of operating cycle is the indicator of performance of management. The net operating cycle represents the time interval for which the firm has to negotiate for Working Capital from its Bankers. It enables to determine accurately the amount of working capital needed for the continuous operation of business activities. The operating cycle calls for proper monitoring of external environment of the business, changes in government policies like taxation, import policies, credit policy of Reserve Bank of India, price trend, technological advancement etc. They have since their own impact on the length of operating cycle. 1.6.2 Estimate of amount of Working Capital based on Current Assets and Current Liabilities The estimate of working capital can be projected if the amount of current assets and current liabilities can be estimated as follows: The various constituents of current assets and current liabilities have a direct bearing on the computation of working capital and the operating cycle. The holding period of various constituents of operating cycle may either contract or expand the net operating cycle period. Shorter the operating cycle period, lower will be the requirement of working capital and vice-versa.

Estimation of Current Assets The estimates of various components of working capital may be made as follows: (i) Raw materials inventory: The funds to be invested in raw materials inventory may be estimated on the basis of production budget, the estimated cost per unit and average holding period of raw material inventory by using the following formula:

)daysin/monthsin(periodholdingmaterialrawAverage

days360/months12unit perunits) (in

materialrawofcostEstimatedproductionEstimated

×

⎪⎪⎭

⎪⎪⎬

⎪⎪⎩

⎪⎪⎨

⎧ ×

Note: 360 days in a year are generally assumed to facilitate calculation. (ii) Work-in-progress inventory: The funds to be invested in work-in-progress can be estimated by the following formula:

7.15

Page 828: 30510870 Cost Accounting and Financial Management

Financial Management

( ))days/months.P.I.WofperiodholdingAverage

days360/months12unitpercostunits) (in

processinworkEstimatedproductionEstimated

×

⎪⎪⎭

⎪⎪⎬

⎪⎪⎩

⎪⎪⎨

⎧ −−×

(iii) Finished Goods: The funds to be invested in finished goods inventory can be estimated with the help of following formula:

)days/months(inventorygoodsfinishedofperiodholdingAverage

days360/months12ondepreciatiexcluding)unitsin(

unitPer(productionofCostproductionEstimated

×

⎪⎪⎭

⎪⎪⎬

⎪⎪⎩

⎪⎪⎨

⎧ ×

(iv) Debtors: Funds to be invested in trade debtors may be estimated with the help of following formula:

ys)(months/daperiodcollectiondebtorsAverage

daysmonths/36012ondepreciatiexcludingunits)in(

unit(PersalesofCostsalescreditEstimated

×

⎪⎪⎭

⎪⎪⎬

⎪⎪⎩

⎪⎪⎨

⎧ ×

(v) Minimum desired Cash and Bank balances to be maintained by the firm has to be added in the current assets for the computation of working capital. Estimation of Current Liabilities Current liabilities generally affect computation of working capital. Hence, the amount of working capital is lowered to the extent of current liabilities (other than bank credit) arising in the normal course of business. The important current liabilities like trade creditors, wages and overheads can be estimated as follows: (i) Trade creditors:

ys)(months/dasuppliersbygrantedperiodCredit

daysmonths/36012unitperunits)(inproduction

tsrequiremenmaterialRawyearlyEstimated

×

⎪⎪⎭

⎪⎪⎬

⎪⎪⎩

⎪⎪⎨

⎧ ×

7.16

Page 829: 30510870 Cost Accounting and Financial Management

Management of Working Capital

(ii) Direct Wages:

ys)(months/dawagesofpaymentinlagtimeAveragex

daysmonths/36012unitperunits)(in

costlabourDirectproductionEstimated

⎪⎪⎭

⎪⎪⎬

⎪⎪⎩

⎪⎪⎨

⎧ ×

(iii) Overheads (other than depreciation and amortization):

)days/months(overheadsofpaymentinlagtimeverageA

days360/months12unitperunits)(inproduction

costOverheadyearlyEstimatd

×

⎪⎪⎭

⎪⎪⎬

⎪⎪⎩

⎪⎪⎨

⎧ ×

Note:The amount of overheads may be separately calculated for different types of overheads. In the case of selling overheads, the relevant item would be sales volume instead of production volume.

The following illustration shows the process of working capital estimation: Illustration 3 On 1st January, the Managing Director of Naureen Ltd. wishes to know the amount of working capital that will be required during the year. From the following information prepare the working capital requirements forecast. Production during the previous year was 60,000 units. It is planned that this level of activity would be maintained during the present year. The expected ratios of the cost to selling prices are Raw materials 60%, Direct wages 10% and Overheads 20%. Raw materials are expected to remain in store for an average of 2 months before issue to production. Each unit is expected to be in process for one month, the raw materials being fed into the pipeline immediately and the labour and overhead costs accruing evenly during the month. Finished goods will stay in the warehouse awaiting dispatch to customers for approximately 3 months. Credit allowed by creditors is 2 months from the date of delivery of raw material. Credit allowed to debtors is 3 months from the date of dispatch. Selling price is Rs.5 per unit. There is a regular production and sales cycle. Wages and overheads are paid on the 1st of each month for the previous month. The company normally keeps cash in hand to the extent of Rs.20,000. Solution Working Notes: 1. Raw material inventory: The cost of materials for the whole year is 60% of the Sales

value.

7.17

Page 830: 30510870 Cost Accounting and Financial Management

Financial Management

Hence it is 60,000 units x Rs.5 x 000,80,1.Rs10060

= . The monthly consumption of raw

material would be Rs.15,000. Raw material requirements would be for two months; hence raw materials in stock would be Rs.30,000.

2. Debtors: The average sales would be Rs.25,000 p.m. Therefore, a sum of Rs.75,000/- would be the amount of sundry debtors.

3. Work in process: (Students may give special attention to this point). It is stated that each unit of production is expected to be in process for one month).

Rs. (a) Raw materials in work-in-process (being one

month’s raw material requirements) 15,000

(b) Labour costs in work-in-process (It is stated that it accrues evenly during the month. Thus, on the first day of each month it would be zero and on the last day of month the work-in-process would include one month’s labour costs. On an average therefore, it would be equivalent to ½ of the month’s labour costs)

1,250

(c) Overheads (For ½ month as explained above) Total work-in-process

_2,500 18,750

4. Finished goods inventory: (3 month’s costs of production)

Raw materials Labour Overheads

45,000 7,500

15,000 67,500

5. Creditors: Suppliers allow a two months’ credit period. Hence, the average amount of creditors would be Rs.30,000 being two months’ purchase of raw materials.

6. Direct Wages payable: The direct wages for the whole year is 60,000 units × Rs.5 x 10% = Rs.30,000. The monthly direct wages would be Rs.2,500 (Rs. 30,000 ÷12). Hence, wages payable would be Rs.2,500.

7. Overheads Payable: The overheads for the whole year is 60,000 units × Rs.5 x 20% = Rs.60,000. The monthly overheads will be Rs.5,000 (Rs.60,000 ÷ 12). Hence overheads payable would be Rs.5,000 p.m.

7.18

Page 831: 30510870 Cost Accounting and Financial Management

Management of Working Capital

Statement of Working Capital required: Current Assets:

Rs. Rs. Raw materials inventory (Refer to working note 1) 30,000 Debtors (Refer to working note 2) 75,000 Working–in-process (Refer to working note 3) 18,750 Finished goods inventory (Refer to working note 4) 67,500 Cash 20,000 2,11,250 Current Liabilities Creditors (Refer to working note 5) 30,000 Direct wages payable (Refer to working note 6) 2,500 Overheads payable (Refer to working note 7) 5,000 37,500Estimated working capital requirements 1,73,750 1.6.3 Working capital requirement estimation based on cash cost: We have already seen that working capital is the difference between current assets and current liabilities. To estimate requirements of working capital, we have to forecast the amount required for each item of current assets and current liabilities. However, in practice another approach may also be useful in estimating working capital requirements. This approach is based on the fact that in the case of current assets, like sundry debtors and finished goods, etc., the exact amount of funds blocked is less than the amount of such current assets. Thus, if we have sundry debtors worth Rs.1 lakh and our cost of production is Rs.75,000, the actual amount of funds blocked in sundry debtors is Rs.75,000 the cost of sundry debtors, the rest (Rs.25,000) is profit. Again some of the cost items also are non-cash costs; depreciation is a non-cash cost item. Suppose out of Rs.75,000, Rs.5,000 is depreciation; then it is obvious that the actual funds blocked in terms of sundry debtors totaling Rs.1 lakh is only Rs.70,000. In other words, Rs.70,000 is the amount of funds required to finance sundry debtors worth Rs.1 lakh. Similarly, in the case of finished goods which are valued at cost, non-cash costs may be excluded to work out the amount of funds blocked. Many experts, therefore, calculate the working capital requirements by working out the cash costs of finished goods and sundry debtors. Under this approach, the debtors are calculated not as a percentage of sales value but as a percentage of cash costs. Similarly, finished goods are valued according to cash costs.

7.19

Page 832: 30510870 Cost Accounting and Financial Management

Financial Management Illustration 4 The following annual figures relate to XYZ Co.,

Rs. Sales (at two months’ credit) 36,00,000 Materials consumed (suppliers extend two months’ credit) 9,00,000 Wages paid (monthly in arrear) 7,20,000 Manufacturing expenses outstanding at the end of the year (Cash expenses are paid one month in arrear)

80,000

Total administrative expenses, paid as above 2,40,000 Sales promotion expenses, paid quarterly in advance 1,20,000 The company sells its products on gross profit of 25% counting depreciation as part of the cost of production. It keeps one months’ stock each of raw materials and finished goods, and a cash balance of Rs.1,00,000. Assuming a 20% safety margin, work out the working capital requirements of the company on cash cost basis. Ignore work-in-process. Solution Statement of Working Capital requirements (cash cost basis)

A. Current Asset Rs Rs . Materials (Rs. 9,00,000 ÷12) 75,000 Finished Goods (Rs. 25,80,000 ÷12) 2,15,000 Debtors (Rs.29,40,000÷6) 4,90,000 Cash 1,00,000 Prepaid expenses (Sales promotion) (Rs. 1,20,000÷4) 30,000 9,10,000 B. Current Liabilities: Creditors for materials (Rs.9,00,000÷6) 1,50,000 Wages outstanding (Rs.7,20,000÷ 12) 60,000 Manufacturing expenses 80,000 Administrative expenses (Rs.2,40,000÷12) 20,000 3,10,000Net working capital (A-B) 6,00,000

7.20

Page 833: 30510870 Cost Accounting and Financial Management

Management of Working Capital

Add safety margin 20% 1,20,000 Total working capital requirements 7,20,000Working Notes:

(i) Computation of Annual Cash cost of Production Rs. Material consumed 9,00,000 Wages 7,20,000 Manufacturing expenses (Rs.80,000 x 12) _9,60,000 Total cash cost of production 25,80,000(ii) Computation of Annual Cash cost of sales: Rs. Cash cost of production as in (i) above 25,80,000 Administrative Expenses 2,40,000 Sales promotion expenses _1,20,000 Total cash cost of sales 29,40,000

Illustration 5 PQ Ltd., a company newly commencing business in 2005 has the undermentioned projected Profit and Loss Account: Rs. Rs. Sales 2,10,000 Cost of goods sold 1,53,000Gross Profit 57,000 Administrative Expenses 14,000 Selling Expenses 13,000 27,000Profit before tax 30,000 Provision for taxation 10,000Profit after tax 20,000The cost of goods sold has been arrived at as under: Materials used 84,000 Wages and manufacturing Expenses 62,500 Depreciation _23,500

1,70,000

Less: Stock of Finished goods (10% of goods produced not yet sold)

__17,000

1,53,000

7.21

Page 834: 30510870 Cost Accounting and Financial Management

Financial Management The figure given above relate only to finished goods and not to work-in-progress. Goods equal to 15% of the year’s production (in terms of physical units) will be in process on the average requiring full materials but only 40% of the other expenses. The company believes in keeping materials equal to two months’ consumption in stock. All expenses will be paid one month in advance. Suppliers of materials will extend 1-1/2 months credit. Sales will be 20% for cash and the rest at two months’ credit. 70% of the Income tax will be paid in advance in quarterly instalments. The company wishes to keep Rs.8,000 in cash. Prepare an estimate of (i) working capital, and (ii) cash cost of working capital. Note: All workings should form part of the answer. Solution (i) Estimate of Working Capital requirements

Current Liabilities Rs. Current Assets Rs. Sundry Creditors: Finished stock: Purchases 14,088 Raw materials 8,400 Provision for taxation 3,000

--------- 17,088

Wages Depreciation

6,250 2,350

17,000

Work-in-progress Balance being working capital required, (say Rs.77,500)

77,543

Materials Wages Depreciation

12,600 3,750

_1,410

17,760 Raw Material 16,100 Sundry Debtors: Materials 10,080 Wages 7,500 Depreciation 2,820 Adm. & Selling

expenses 3,600

Profit 4,000 28,000

7.22

Page 835: 30510870 Cost Accounting and Financial Management

Management of Working Capital

Prepaid Expenses: Wages Admn. & Selling expenses

5,521 2,250

7,771

Cash in hand _8,000 94,631 94,631

(ii) Estimate of Cash Cost of Working Capital Rs.

Working capital as per statement given above 77,543 Less: Profit and Depreciation for which funds are not needed (see working note (vii)

10,580

Cash Cost of working capital required, say Rs.67,000 66,963 Working Notes:

(i) Work-in-progress Rs. 15% of material consumed for finished goods 12,600 15% of 40% of wages and expenses 3,750 15% of 40% of Depreciation _1,410

17,760(ii) Raw materials will be 1/6th of total materials consumed i.e., Rs.84,000 for finished goods

plus Rs.12,600 for work-in-progress, i.e. Rs.16,100. (iii) Sundry Debtors:

80% of two months’ Sales, i.e., Rs.2,10,000 × 000,28.Rs122

10080

Individual items have been computed on that basis. (iv) Creditors for raw materials on the basis of total purchases during the year (Rs.84,000 + Rs.12,600+Rs.16,100) × 1½ /12) = Rs.14,088. (v) Wages paid in advance: (Rs.62,500 + Rs.3,750) × 1/12 = Rs.5,521. (vi) Administrative & Selling expenses paid in advance Rs.27,000 ×1/12 = Rs.2,250/-

7.23

Page 836: 30510870 Cost Accounting and Financial Management

Financial Management (vii) Depreciation and profit included in the cost of current assets:

Rs. Depreciation: Finished goods 2,350 Work-in-progress 1,410 Debtors 2,820 6,580 Profit included in Debtors (including income-tax, i.e., Rs.1,333) 4,000 10,580 Illustration 6 M.A. Limited is commencing a new project for manufacture of a plastic component. The following cost information has been ascertained for annual production of 12,000 units which is the full capacity:

Costs per unit (Rs.) Materials 40 Direct labour and variable expenses 20 Fixed manufacturing expenses 6 Depreciation 10 Fixed administration expenses _4 80 The selling price per unit is expected to be Rs.96 and the selling expenses Rs.5 per unit. 80% of which is variable. In the first two years of operations, production and sales are expected to be as follows:

Year Production (No. of units)

Sales (No.of units)

1 6,000 5,000 2. 9,000 8,500

To assess the working capital requirements, the following additional information is available:

7.24

Page 837: 30510870 Cost Accounting and Financial Management

Management of Working Capital

(a) Stock of materials 2.25 months’ average consumption (b) Work-in-process Nil (c) Debtors 1 month’s average sales. (d) Cash balance Rs.10,000 (e) Creditors for supply of materials 1 month’s average purchase during the year. (f) Creditors for expenses 1 month’s average of all expenses during the

year. Prepare, for the two years: (i) A projected statement of Profit/Loss (Ignoring taxation); and (ii) A projected statement of working capital requirements. Solution (i)

M.A. Limited Projected Statement of Profit / Loss

(Ignoring Taxation)

Year 1 Year 2Production (Units) 6,000 9,000 Sales (Units) 5,000 8,500 Rs. Rs. Sales revenue @ Rs. 96 per unit: (A) 4,80,000 8,16,000Cost of production: Materials @ Rs. 40 per unit 2,40,000 3,60,000 Direct labour and variable expenses @ Rs.20 per unit 1,20,000 1,80,000 Fixed manufacturing expenses (Production Capacity: 12,000 units @ Rs.6) 72,000 72,000 Depreciation (Production Capacity : 12,000 units @ Rs.10) 1,20,000 1,20,000 Fixed administration expenses (Production Capacity : 12,000 units @ Rs.4) 48,000 48,000

7.25

Page 838: 30510870 Cost Accounting and Financial Management

Financial Management Total costs of production _6,00,000 7,80,000 Add: Opening stock of finished goods 1,00,000 (Year 1 : Nil; Year 2 : 1,000 units) Cost of goods available 6,00,000 8,80,000 (Year 1: 6,000 units; Year 2: 10,000 units) Less: Closing stock of finished goods at average cost (year 1: 1000 units, year 2 : 1500 units)

1,00,000 1,32,000

Cost of goods sold 5,00,000 7,48,000 Add: Selling expenses – Variable @ 4 per unit Fixed (12,000 × Re.1)

20,000 12,000

34,000 12,000

Cost of Sales : (B) 5,32,000 7,94,000Profit (+) / Loss (-): (A-B) (-) 52,000 (+) 22,000

Working Notes:

1. Calculation of creditors for supply of materials: Year 1 Year 2 Rs. Rs. Materials consumed during the year 2,40,000 3,60,000 Add: Closing stock (2.25 month’s average consumption) 45,000 67,500 2,85,000 4,27,500 Less: Opening Stock 45,000Purchases during the year 2,85,000 3,82,500 Average purchases per month (Creditors) 23,750 31,875 Year 1

Rs. Year 2

Rs. 2. Creditors for expenses: Total direct labour, manufacturing, administration and selling expenses for the year

2,72,000

3,46,000

Average per month 22,667 28,833

7.26

Page 839: 30510870 Cost Accounting and Financial Management

Management of Working Capital

(ii) Projected statement of working capital requirements Year 1

Rs. Year 2

Rs. Current Assets: Stock of materials (2.25 month’s average consumption) 45,000 67,500 Finished goods 1,00,000 1,32,000 Debtors (1 month’s average sales) 40,000 68,000 Cash _10,000 _10,000 Total Current Assets (A) 1,95,000 2,77,500Current Liabilities: Creditors for supply of materials 23,750 31,875 Refer to working note 1) Creditors for expenses 22,667 28,833(Refer to working note 2) Total Current Liabilities: (B) 46,417 60,708Estimated Working Capital Requirements: (A-B) 1,48,583 2,16,792 1.6.4 Effect of Double Shift Working on Working Capital requirements: Increase in the number of hours of production has an effect on the working capital requirements. The greatest economy in introducing double shift is the greater use of fixed assets-little or marginal funds may be required for additional assets. It is obvious that in double shift working, an increase in stocks will be required as the production rises. However, it is quite possible that the increase may not be proportionate to the rise in production since the minimum level of stocks may not be very much higher. Thus, it is quite likely that the level of stocks may not be required to be doubled as the production goes up two-fold. The amount of materials in process will not change due to double shift working since work started in the first shift will be completed in the second; hence, capital tied up in materials in process will be the same as with single shift working. As such the cost of work-in-process, will not change unless the second shift’s workers are paid at a higher rate. Fixed overheads will remain fixed whereas variable overheads will increase in proportion to the increased production. Semi-variable overheads will increase according to the variable element in them.

7.27

Page 840: 30510870 Cost Accounting and Financial Management

Financial Management However, in examinations the students may increase the amount of stocks of raw materials proportionately unless instructions are to the contrary. Illustration 7 Samreen Enterprises has been operating its manufacturing facilities till 31.3.2006 on a single shift working with the following cost structure:

Per Unit Rs.

Cost of Materials 6.00 Wages (out of which 40% fixed) 5.00 Overheads (out of which 80% fixed) 5.00 Profit 2.00Selling Price 18.00Sales during 2005-06 – Rs.4,32,000. As at 31.3.2006 the company held: Rs. Stock of raw materials (at cost) 36,000 Work-in-progress (valued at prime cost) 22,000 Finished goods (valued at total cost) 72,000 Sundry debtors 1,08,000 In view of increased market demand, it is proposed to double production by working an extra shift. It is expected that a 10% discount will be available from suppliers of raw materials in view of increased volume of business. Selling price will remain the same. The credit period allowed to customers will remain unaltered. Credit availed of from suppliers will continue to remain at the present level i.e., 2 months. Lag in payment of wages and expenses will continue to remain half a month. You are required to assess the additional working capital requirements, if the policy to increase output is implemented.

7.28

Page 841: 30510870 Cost Accounting and Financial Management

Management of Working Capital

Solution Statement of cost at single shift and double shift working

24,000 units 48,000 Units Per Unit Total Per unit Total Rs. Rs. Rs. Rs. Raw materials 6 1,44,000 5.40 2,59,200 Wages - Variable 3 72,000 3.00 1,44,000 Fixed 2 48,000 1.00 48,000 Overheads - Variable 1 24,000 1.00 48,000 Fixed 4 96,000 2.00 96,000Total cost 16 3,84,000 12.40 5,95,200 Profit 2 48,000 5.60 2,68,800 18 4,32,000 18.00 8,64,000 Sales in units 2005-06

units 000,2418.Rs

000,32,4.Rsprice selling Unit

Sales===

Stock of Raw Materials in units on 31.3.2006

= 6

000,36.Rsunit perCost

stock of Value= = 6,000 units

Stock of work-in-progress in units on 31.3.2006

= units000,2)5.Rs6.Rs(

000,22.RsunitperCost

progressinworkofValue=

+=

−−

Stock of finished goods in units 2005-06

.units 500,416.Rs000,72.Rs

unit per Coststock of Value

===

7.29

Page 842: 30510870 Cost Accounting and Financial Management

Financial Management

Comparative Statement of Working Capital Requirement

Single Shift Double Shift Unit Rate

Rs. Amount

Rs. Unit Rate

Rs. Amount

Rs. Current Assets Inventories - Raw Materials 6000 6 36,00 12000 5.40 64,800 Work-in-Progress 2000 11 22,000 2000 9.40 18,800 Finished Goods 4500 16 72,000 9000 12.40 1,11,600 Sundry Debtors 6000 18 1,08,000 12000 18.00 2,16,000Total Current Assets: (A) 2,38,000 4,11,200Current Liabilities Creditors for Materials 4000 6 24,000 8000 5.40 43,200 Creditors for Wages 1000 5 5,000 2000 4.00 8,000 Creditors for Expenses 1000 5 5,000 2000 3.00 _6,000Total Current Liabilities: (B) 34,000 57,200Working Capital: (A) – (B) 2,04,000 3,54,000 Less: Profit included in Debtors 6000 2 _12,000

1,92,00012,000 5.60 _67,200

2,86,800Increase in Working Capital requirement is (Rs.2,86,800 – Rs.1,92,000) or Rs.94,800 Notes: (i) The quantity of material in process will not change due to double shift working since work

started in the first shift will be completed in the second shift. (ii) The valuation of work-in-progress based on prime cost as per the policy of the company

is as under.

Single shift Rs.

Double shift Rs.

Materials 6.00 5.40 Wages – Variable 3.00 3.00 Fixed _2.00 1.00 11.00 9.40

7.30

Page 843: 30510870 Cost Accounting and Financial Management

Management of Working Capital

UNIT – II : TREASURY AND CASH MANAGEMENT

2.1 TREASURY MANAGEMENT: MEANING Tight money, escalating interest rates and economic volatility have called for a specialised skills called Treasury Management. Until recently, no major efforts were made to manage cash. In the wake of the competitive business environment resulting from the liberalization of the economy, there is a pressure to manage cash. The demand for funds for expansions coupled with high interest rates, foreign exchange volatility and the growing volume of financial transactions have necessitated efficient management of money. Treasury management is defined as ‘the corporate handling of all financial matters, the generation of external and internal funds for business, the management of currencies and cash flows and the complex, strategies, policies and procedures of corporate finance.’ The treasury management mainly deals with working capital management and financial risk management. The former constitutes cash management and decides the asset liability mix. Financial risk management includes forex and interest rate management. The key goal of treasury management is planning, organizing and controlling cash assets to satisfy the financial objectives of the organization. The goal may be to maximize the return on the available cash, or minimize interest cost or mobilise as much cash as possible for corporate ventures. Dealing in forex, money and commodity markets involves complex risks of fluctuating exchange rates, interest rates and prices which can affect the profitability of the organization. Treasury managers try to minimize lapses by adopting risk transfer and hedging techniques that suit the internal policies of the organisation. Options, futures and swaps are a few of the major derivative instruments, the Treasury Managers use for hedging their risk.

2.2 FUNCTIONS OF TREASURY DEPARTMENT 1. Cash Management: The efficient collection and payment of cash both inside the organisation and to third parties is the function of the treasury department. The involvement of the department with the details of receivables and payables will be a matter of policy. There may be complete centralization within a group treasury or the treasury may simply advise subsidiaries and divisions on policy matter viz., collection/payment periods, discounts, etc. Any position between these two extremes would be possible. Treasury will normally manage surplus funds in an investment portfolio. Investment policy will consider future needs for liquid funds and acceptable levels of risk as determined by company policy.

7.31

Page 844: 30510870 Cost Accounting and Financial Management

Financial Management 2. Currency Management: The treasury department manages the foreign currency risk exposure of the company. In a large multinational company (MNC) the first step will usually be to set off intra-group indebtedness. The use of matching receipts and payments in the same currency will save transaction costs. Treasury might advise on the currency to be used when invoicing overseas sales. The treasury will manage any net exchange exposures in accordance with company policy. If risks are to be minimized then forward contracts can be used either to buy or sell currency forward. 3. Funding Management: Treasury department is responsible for planning and sourcing the company’s short, medium and long-term cash needs. Treasury department will also participate in the decision on capital structure and forecast future interest and foreign currency rates. 4. Banking: It is important that a company maintains a good relationship with its bankers. Treasury department carry out negotiations with bankers and act as the initial point of contact with them. Short-term finance can come in the form of bank loans or through the sale of commercial paper in the money market. 5. Corporate Finance: Treasury department is involved with both acquisition and divestment activities within the group. In addition it will often have responsibility for investor relations. The latter activity has assumed increased importance in markets where share-price performance is regarded as crucial and may affect the company’s ability to undertake acquisition activity or, if the price falls drastically, render it vulnerable to a hostile bid.

2.3 MANAGEMENT OF CASH Management of cash is an important function of the finance manager. The Finance Manager has to provide adequate cash to each of the units. For the survival of the business it is absolutely essential that there should be adequate cash. It is the duty of finance manger to have liquidity at all parts of the organization while managing cash. On the other hand, he has also to ensure that there are no funds blocked in idle cash. Idle cash resources entail a great deal of cost in terms of interest charges and in terms of opportunities costs. Hence, the question of costs of idle cash must also be kept in mind by the finance manager. A cash management scheme therefore, is a delicate balance between the twin objectives of liquidity and costs. 2.3.1 The Need for Cash: The following are three basic considerations in determining the amount of cash or liquidity as have been outlined by Lord Keynes: ♦ Transaction need: Cash facilitates the meeting of the day-to-day expenses and other debt

payments. Normally, inflows of cash from operations should be sufficient for this purpose. But sometimes this inflow may be temporarily blocked. In such cases, it is only the

7.32

Page 845: 30510870 Cost Accounting and Financial Management

Management of Working Capital

reserve cash balance that can enable the firm to make its payments in time. ♦ Speculative needs: Cash may be held in order to take advantage of profitable

opportunities that may present themselves and which may be lost for want of ready cash/settlement.

♦ Precautionary needs: Cash may be held to act as for providing safety against unexpected events. Safety as is explained by the saying that a man has only three friends an old wife, an old dog and money at bank.

Facets of Cash Management: Cash management is concerned with the managing of (i) Cash flows into and out of the firm; (ii) Cash flows within the firm; and (iii) Cash balances held by the firm at a point of time by financing deficit or investing surplus cash. It is generally represented by a cash management cycle. Sales generates cash which has to be disbursed out. In recent years, a number of innovations have been made in cash management techniques. An obvious aim of the firm these days is to mange its cash affairs in such a way as to maintain a minimum balance of cash and to invest the surplus immediately in profitable investment opportunities. In order to synchronise the cash receipt and payments. A firm need to develop appropriate strategies for cash management viz: (i) Cash Planning: The pattern of cash inflows and outflows should be properly predicted

in advance. Cash budget is a tool to achieve this objective. (ii) Managing the cash flows: The cash inflows should be accelerated, while as far as

possible, the outflows should be decelerated. (iii) Optimum cash level: In deciding about the appropriate level of cash balances, the cost

of idle cash and danger of shortage should be taken into consideration. (iv) Investing surplus cash: The surplus cash should be properly invested to earn profits.

The firm should decide about the division of such cash balance between various alternative short term investment opportunities such as, bank deposits, marketable securities, inter-corporate lending.

The ideal cash management system will depend upon various factors viz., product, organization structure, competition, culture and options available. The task is really complex. The exact nature of a cash management system would depend upon the organizational structure of an enterprise. In a highly centralized organization the system would be such that the central or head office controls the inflows and outflows of cash on a routine and daily basis. In a decentralized form of organisation, where the divisions have compelete

7.33

Page 846: 30510870 Cost Accounting and Financial Management

Financial Management responsibility of conducting their affairs, it may not be possible and advisable for the central office to exercise a detailed control over cash inflows and outflows. 2.3.2 Cash Planning: Cash Planning is a technique to plan and control the use of cash. This protects the financial conditions of the firm by developing a projected cash statement from a forecast of expected cash inflows and outflows for a given period. This may be done periodically either on daily, weekly or monthly basis. The period and frequency of cash planning generally depends upon the size of the firm and philosophy of management. As firms grows and business operations become complex, cash planning becomes inevitable for continuing success. The very first step in this direction is to estimate the requirement of cash. For this purpose cash flow statements and cash budget are required to be prepared. The technique of preparing cash flow and funds flow statements have been discussed in this book. The preparation of cash budget has however, been demonstrated here. 2.3.3 Cash Budget: Cash Budget is the most significant device to plan for and control cash receipts and payments. This represents cash requirements of business during the budget period. One of the significant advantage of cash budget is to determine the net cash inflow or outflow so that the firm is enabled to arrange finances. However, the firm’s decision for appropriate sources of financing should depend upon factors such as cost and risk. Cash Budget helps a firm to manage its cash position. It also helps to utilise funds in better ways. On the basis of cash budget, the firm can decide to invest surplus cash in marketable securities and earn profits. The cash budget is prepared on the basis of receipts and payments method and offers following benefits: (i) It provides a complete picture of all items of expected cash flows. (ii) It is a sound tool of managing daily cash operations. This method, however, suffers from the following limitations: (i) Its reliability is reduced because of the uncertainty of cash forecasts. For example,

collections may be delayed, or unanticipated demands may cause large disbursements. (ii) It fails to highlight the significant movements in the Working Capital items. In order to maintain an optimum cash balance, what is required is (i) a complete and accurate forecast of net cash flows over the planning horizon and (ii) perfect synchronization of cash receipts and disbursements. Thus, implementation of an efficient cash management system starts with the preparation of a plan of firm’s operations for a period in future. This plan will help in preparation of a statement of receipts and disbursements expected at different point of

7.34

Page 847: 30510870 Cost Accounting and Financial Management

Management of Working Capital

time of that period. It will enable the management to pin point the time of excessive cash or shortage of cash. This will also help to find out whether there is any expected surplus cash still unutilized or shortage of cash which is yet to be arranged for. In order to take care of all these considerations, the firm should prepare a cash budget. The following figure highlights the cash surplus and cash shortage position over the period of cash budget for preplanning to take corrective and necessary steps.

2.4 METHODS OF CASH FLOW BUDGETING Cash flow budget is a detailed budget of income and cash expenditures incorporating both revenue and capital items. The cash flow budget should be prepared in the same format in which the actual position is to be presented. The year’s budget is usually phased into shorter periods for control e.g., monthly or quarterly. Cash budget is concerned with liquidity must reflect changes between opening and closing debtor balances and between opening and closing creditor balances as well as focusing attention on other inflows and outflows of cash. The cash budget shows the cash flows arising from the operational budgets and the profit and assets structure. A cash budget can be prepared in the following ways:

7.35

Page 848: 30510870 Cost Accounting and Financial Management

Financial Management 1. Receipts and Payments Method: In this method all the expected receipts and payments for budget period are considered. All the cash inflow and outflow of all functional budgets including capital expenditure budgets are considered. Accruals and adjustments in accounts will not affect the cash flow budget. Anticipated cash inflow is added to the opening balance of cash and all cash payments are deducted from this to arrive at the closing balance of cash. This method is commonly used in business organizations. 2. Adjusted Income Method: In this method the annual cash flows are calculated by adjusting the sales revenues and cost figures for delays in receipts and payments (change in debtors and creditors) and eliminating non-cash items such as depreciation. 3. Adjusted Balance Sheet Method: In this method, the budgeted balance sheet is predicted by expressing each type of asset and short-term liabilities as percentage of the expected sales. The profit is also calculated as a percentage of sales, so that the increase in owners equity can be forecasted. Known adjustments, may be made to long-term liabilities and the balance sheet will then show if additional finance is needed. It is important to note that the capital budget will also be considered in the preparation of cash flow budget because the annual budget may disclose a need for new capital investments and also, the costs and revenues of any new projects coming on stream will need to be incorporated in the short-term budgets. A number of additional financial statements, such as sources and application of funds statement or schedules or loan service payments or capital raising schedules may be produced. The Cash Budget can be prepared for short period or for long period. Cash budget for short period: Preparation of cash budget month by month would require the following estimates: (a) As regards receipts:

1. Receipts from debtors; 2. Cash Sales; and 3. Any other source of receipts of cash (say, dividend from a subsidiary company)

(b) As regards payments: 1. Payments to be made for purchases; 2. Payments to be made for expenses; 3. Payments that are made periodically but not every month;

(i) debenture interest; (ii) income tax paid in advance;

7.36

Page 849: 30510870 Cost Accounting and Financial Management

Management of Working Capital

(iii) sales tax etc. 4. Special payments to be made in a particular month, for example, dividends to

shareholders, redemption of debentures, repayments of loan, payment of assets acquired, etc.

2.4.1 Format of Cash Budget Co. Ltd.

Cash Budget Period………………

Month 1

Month 2

Month 3

Month 12

Receipts: 1. Opening balance 2. Collection from debtors 3. Cash sales 4. Loans from banks 5. Share capital 6. Miscellaneous receipts 7. Other items Total Payments: 1. Payments to creditors 2. Wages 3. Overheads (a) (b) (c) 4. Interest 5. Dividend 6. Corporate tax

7.37

Page 850: 30510870 Cost Accounting and Financial Management

Financial Management 7. Capital expenditure 8. Other items Total Closing balance [Surplus (+)/Shortfall (-)]

Students are required to do good practice in preparing the cash budgets. The following illustration will show how short term cash budgets can be prepared. Illustration 1 Prepare monthly cash budget for six months beginning from April 2006 on the basis of the following information:- (i) Estimated monthly sales are as follows:-

Rs. Rs. January 1,00,000 June 80,000 February 1,20,000 July 1,00,000 March 1,40,000 August 80,000 April 80,000 September 60,000 May 60,000 October 1,00,000 (ii) Wages and salaries are estimated to be payable as follows:-

Rs. Rs.

April 9,000 July 10,000

May 8,000 August 9,000

June 10,000 September 9,000 (iii) Of the sales, 80% is on credit and 20% for cash. 75% of the credit sales are collected

within one month and the balance in two months. There are no bad debt losses. (iv) Purchases amount to 80% of sales and are made and paid for in the month preceding the

sales. (v) The firm has 10% debentures of Rs.1,20,000. Interest on these has to be paid quarterly

in January, April and so on. (vi) The firm is to make an advance payment of tax of Rs.5,000 in July, 2006.

7.38

Page 851: 30510870 Cost Accounting and Financial Management

Management of Working Capital

(vii) The firm had a cash balance of Rs.20,000 on April 1, 2006, which is the minimum desired level of cash balance. Any cash surplus/deficit above/below this level is made up by temporary investments/liquidation of temporary investments or temporary borrowings at the end of each month (interest on these to be ignored).

Solution Workings: Collection from debtors:

(Amount in Rs.)

February March April May June July August September

Total sales 1,20,000 1,40,000 80,000 60,000 80,000 1,00,000 80,000 60,000

Credit sales (80% of total sales)

96,000

1,12,000

64,000

48,000

64,000

80,000

64,000

48,000

Collections:

One month 72,000 84,000 48,000 36,000 48,000 60,000 48,000

Two months 24,000 28,000 16,000 12,000 16,000 20,000

Total collections

1,08,000

76,000

52,000

60,000

76,000

68,000

Monthly Cash Budget for Six months, April to September, 2006 (Amount in Rs.) Receipts:

April May June July August September

Opening balance 20,000 20,000 20,000 20,000 20,000 20,000

Cash sales 16,000 12,000 16,000 20,000 16,000 12,000

Collection from debtors 1,08,000 76,000 52,000 60,000 76,000 68,000

Total cash available (A) 1,44,000 1,08,000 88,000 1,00,000 1,12,000 1,00,000

7.39

Page 852: 30510870 Cost Accounting and Financial Management

Financial Management Payments:

Purchases 48,000 64,000 80,000 64,000 48,000 80,000

Wages & salaries 9,000 8,000 10,000 10,000 9,000 9,000

Interest on debentures 3,000 --- ---- 3,000 --- ----

Tax payment --- --- ---- 5,000 ---- ----

Total payments (B) 60,000 72,000 90,000 82,000 57,000 89,000

Minimum cash balance desired

20,000

20,000

20,000

20,000

20,000

20,000

Total cash needed (C) 80,000 92,000 1,10,000 1,02,000 77,000 1,09,000

Surplus deficit (A-C) 64,000 16,000 (22,000) (2,000) 35,000 (9,000)

Investment/financing Temporary Investments

(64,000)

(16,000)

----

(35,000)

-----

Liquidation of temporary investments or temporary borrowings

----

----

22,000

2,000

----

9,000

Total effect of investment/financing (D)

(64,000)

(16,000)

22,000

2,000

(35,000)

9,000

Closing cash balance (A+D-B)

20,000

20,000

20,000

20,000

20,000

20,000

Illustration 2 From the following information relating to a departmental store, you are required to prepare for the three months ending 31st March, 2006:- (a) Monthwise cash budget on receipts and payments basis; and (b) Statement of Sources and uses of funds for the three months period. It is anticipated that the working capital at 1st January, 2006 will be as follows:-

Rs. in ‘000’s Cash in hand and at bank 545 Short term investments 300 Debtors 2,570

7.40

Page 853: 30510870 Cost Accounting and Financial Management

Management of Working Capital

Stock 1,300 Trade creditors 2,110 Other creditors 200 Dividends payable 485 Tax due 320 Plant 800 Budgeted Profit Statement: Rs.in ‘000’s January February March Sales 2,100 1,800 1,700 Cost of sales 1,635 1,405 1,330 Gross Profit 465 395 370 Administrative, Selling and Distribution Expenses

315

270

255 Net Profit before tax 150 125 115 Budgeted balances at the end of each months: Rs. in ‘000’s 31st Jan. 29th Feb. 31st March Short term investments 700 --- 200 Debtors 2,600 2,500 2,350 Stock 1,200 1,100 1,000 Trade creditors 2,000 1,950 1,900 Other creditors 200 200 200 Dividends payable 485 -- -- Tax due 320 320 320 Plant (depreciation ignored) 800 1,600 1,550 Depreciation amount to Rs.60,000 is included in the budgeted expenditure for each month.

7.41

Page 854: 30510870 Cost Accounting and Financial Management

Financial Management Solution

Workings: Rs. in ‘000’ (1) Payments to creditors: Jan. 2006 Feb.2006 March, 2006 Cost of Sales 1,635 1,405 1,330 Add Closing Stocks 1,200 1,100 1,000 2,835 2,505 2,330 Less: Opening Stocks 1,300 1,200 1,100 Purchases 1,535 1,305 1,230 Add: Trade Creditors, Opening balance 2,110 2,000 1,950 3,645 3,305 3,180 Less: Trade Creditors, closing balance 2,000 1,950 1,900 Payment 1,645 1,355 1,280 (2) Receipts from debtors: Debtors, Opening balances 2,570 2,600 2,500 Add Sales 2,100 1,800 1,700 4,670 4,400 4,200 Less Debtors, closing balance 2,600 2,500 2,350 Receipt 2,070 1,900 1,850

CASH BUDGET (a) 3 months ending 31st March, 2006

(Rs, in 000’s) January, 2006 Feb. 2006 March, 2006 Opening cash balances 545 315 65 Add Receipts: From Debtors 2,070 1,900 1,850 Sale of Investments --- 700 ---- Sale of Plant --- --- 50 Total (A) 2,615 2,915 1,965

7.42

Page 855: 30510870 Cost Accounting and Financial Management

Management of Working Capital

Deduct Payments Creditors 1,645 1,355 1,280 Expenses 255 210 195 Capital Expenditure --- 800 --- Payment of dividend --- 485 --- Purchase of investments 400 --- 200 Total payments (B) 2,300 2,850 1,675 Closing cash balance

(A - B) 315 65 290

(b) Statement of Sources and uses of Funds for the Three Month Period

Ending 31st March, 2006

Sources: Rs.’000 Rs.’000 Funds from operation: Net profit 390 Add Depreciation 180 570 Sale of plant 50 620 Decrease in Working Capital 665 Total 1,285 Uses: Purchase of plant 800 Payment by dividends 485 Total 1,285

Statement of Changes in Working Capital

January,06 March, 06 Increase Decrease Rs.000 Rs.000 Rs.000 Rs.000

Current Assets Cash in hand and at Bank 545 290 255 Short term Investments 300 200 100

7.43

Page 856: 30510870 Cost Accounting and Financial Management

Financial Management Debtors 2,570 2,350 220 Stock 1,300 1,000 300 4,715 3,840 Current Liabilities Trade Creditors 2,110 1,900 210 --- Other Creditors 200 200 --- --- Tax Due 320 320 --- --- 2,630 2,420 Working Capital 2,085 1,420 Decrease 665 665 2,085 2,085 875 875 2.4.2 Cash Budget for long period: Long-range cash forecast often resemble the projected sources and application of funds statement. The following procedure may be adopted to prepare long-range cash forecasts: (i) Take the cash at bank and in the beginning of the year: (ii) Add:

(a) Trading profit (before tax) expected to be earned; (b) Depreciation and other development expenses incurred to be written off; (c) Sale proceeds of assets’; (d) Proceeds of fresh issue of shares or debentures; and (e) Reduction in working capital that is current assets (except cash) less current

liabilities. (iii) Deduct:

(a) Dividends to be paid. (b) Cost of assets to be purchased. (c) Taxes to be paid. (d) Debentures or shares to be redeemed. (e) Increase in working capital.

7.44

Page 857: 30510870 Cost Accounting and Financial Management

Management of Working Capital

Illustration 3 You are given below the Profit & Loss Accounts for two years for a company:

Profit and Loss Account

Year 1 Year 2 Year 1 Year 2

Rs. Rs. Rs. Rs.

To Opening stock 80,00,000 1,00,00,000 By Sales 8,00,00,000 10,00,00,000

To Raw materials 3,00,00,000 4,00,00,000 By Closing stock 1,00,00,000 1,50,00,000

To Stores 1,00,00,000 1,20,00,000 By Misc. Income 10,00,000 10,00,000

To Manufacturing Expenses

1,00,00,000

1,60,00,000

To Other Expenses 1,00,00,000 1,00,00,000

To Depreciation 1,00,00,000 1,00,00,000

To Net Profit 1,30,00,000 1,80,00,000

9,10,00,000 11,60,00,000 9,10,00,000 11,60,00,000 Sales are expected to be Rs.12,00,00,000 in year 3. As a result, other expenses will increase by Rs.50,00,000 besides other charges. Only raw materials are in stock. Assume sales and purchases are in cash terms and the closing stock is expected to go up by the same amount as between year 1 and 2. You may assume that no dividend is being paid. The Company can use 75% of the cash generated to service a loan. How much cash from operations will be available in year 3 for the purpose? Ignore income tax. Solution Projected Profit and Loss Account for the year 3

Year 2 Actual (Rs. in lakhs)

Year 3 Projected

(Rs. in lakhs)

Year 2 Actual (Rs. in lakhs)

Year 3 Projected

(Rs. in lakhs)

To Materials consumed 350 420 By Sales 1,000 1,200

To Stores 120 144 By Misc. Income 10 10

7.45

Page 858: 30510870 Cost Accounting and Financial Management

Financial Management To Mfg. Expenses 160 192

To Other expenses 100 150

To Depreciation 100 100

To Net profit 180 204

1,010 1,210 1,010 1,210 Cash Flow:

(Rs. in lakhs) Profit 204 Add: Depreciation 100 304 Less: Cash required for increase in stock 50 Net cash inflow 254Available for servicing the loan: 75% of Rs.2,54,00,000 or Rs.1,90,50,000 Working Notes: (i) Material consumed in year 2: 35% of sales.

Likely consumption in year 3 : )lakhs(420.Rsor100351200.Rs ×

(ii) Stores are 12% of sales, as in year 2. (iii) Manufacturing expenses are 16% of sales. Note: The above also shows how a projected profit and loss account is prepared. 2.4.3 Managing Cash Collection and Disbursements: The finance manager must control the levels of cash balance at various points in the organization. This task assumes special importance on account of the fact that there is generally a tendency amongst divisional managers to keep cash balance in excess of their needs. Hence, the finance manager must devise a system whereby each division of an organization retains enough cash to meet its day-to-day requirements without having surplus balance on hand. For this, methods have to be employed to: (a) Speed up the mailing time of payments from customers; (b) Reduce the time during which payments received by the firm remain uncollected and

speed up the movement of funds to disbursement banks.

7.46

Page 859: 30510870 Cost Accounting and Financial Management

Management of Working Capital

Having prepared the cash budget, the finance manager should ensure that there does not exists a significant deviation between projected cash flows and actual cash flows. To achieve this cash management efficiency will have to be improved through a proper control of cash collection and disbursement. The twin objectives in managing the cash flows should be to accelerate cash collections as much as possible and to decelerate or delay cash disbursements. 2.4.4 Accelerating Cash Collections: A firm can conserve cash and reduce its requirements for cash balances if it can speed up its cash collections by issuing invoices quickly and taking other necessary steps for cash collection. It can be accelerated by reducing the time lag between a customer pays bill and the cheque is collected and funds become available for the firm’s use. A firm can decentralized collection system known as concentration banking and lock box system to speed up cash collection and reduce float time. (i) Concentration Banking: In concentration banking the company establishes a number of strategic collection centres in different regions instead of a single collection centre at the head office. This system reduces the period between the time a customer mails in his remittances and the time when they become spendable funds with the company. Payments received by the different collection centers are deposited with their respective local banks which in turn transfer all surplus funds to the concentration bank of head office. The concentration bank with which the company has its major bank account is generally located at the headquarters. Concentration banking is one important and popular way of reducing the size of the float. (ii) Lock Box System: Another means to accelerate the flow of funds is a lock box system. While concentration banking, remittances are received by a collection centre and deposited in the bank after processing. The purpose of lock box system is to eliminate the time between the receipt of remittances by the company and deposited in the bank. A lock box arrangement usually is on regional basis which a company chooses according to its billing patterns. Under this arrangement, the company rents the local post-office box and authorizes its bank at each of the locations to pick up remittances in the boxes. Customers are billed with instructions to mail their remittances to the lock boxes. The bank picks up the mail several times a day and deposits the cheques in the company’s account. The cheques may be micro-filmed for record purposes and cleared for collection. The company receives a deposit slip and lists all payments together with any other material in the envelope. This procedure frees the company from handling and depositing the cheques. The main advantage of lock box system is that cheques are deposited with the banks sooner and become collected funds sooner than if they were processed by the company prior to deposit. In other words lag between the time cheques are received by the company and the time they are actually deposited in the bank is eliminated. The main drawback of lock box system is the cost of its operation. The bank provides a number of services in addition to usual clearing of cheques

7.47

Page 860: 30510870 Cost Accounting and Financial Management

Financial Management and requires compensation for them. Since the cost is almost directly proportional to the number of cheques deposited. Lock box arrangements are usually not profitable if the average remittance is small. The appropriate rule for deciding whether or not to use a lock box system or for that matter, concentration banking, is simply to compare the added cost of the most efficient system with the marginal income that can be generated from the released funds. If costs are less than income, the system is profitable, if the system is not profitable, it is not worth undertaking. (iii) Playing the float: Besides accelerating collections, an effective control over payments can also cause faster turnover of cash. This is possible only by making payments on the due date, making excessive use of draft (bill of exchange) instead of cheques. Availability of cash can be maximized by playing the float. In this, a firm estimates accurately the time when the cheques issued will be presented for encashment and thus utilizes the float period to its advantage by issuing more cheques but having in the bank account only so much cash balance as will be sufficient to honour those cheques which are actually expected to be presented on a particular date. 2.4.5 Different Kinds of Float with reference to Management of Cash: The term float is used to refer to the periods that affect cash as it moves through the different stages of the collection process. Four kinds of float with reference to management of cash are: ♦ Billing float: An invoice is the formal document that a seller prepares and sends to the

purchaser as the payment request for goods sold or services provided. The time between the sale and the mailing of the invoice is the billing float.

♦ Mail float: This is the time when a cheque is being processed by post office, messenger service or other means of delivery.

♦ Cheque processing float: This is the time required for the seller to sort, record and deposit the cheque after it has been received by the company.

♦ Banking processing float: This is the time from the deposit of the cheque to the crediting of funds in the sellers account.

2.4.6 Delaying Payments: A firm can increase its net float by speeding up collections. It can also increase the net float by delayed disbursement of funds from the bank by increasing the mail time. A company may make payment to its outstation suppliers by a cheque and send it through mail. The delay in transit and collection of the cheque, will be used to increase the float. 2.4.7 Controlling Disbursements: The effective control of disbursement can also help the firm in conserving cash and reducing the financial requirements. Disbursement arise due to trade credit, which is a spontaneous, source of funds. The firm should make payments using credit terms to the fullest extent.

7.48

Page 861: 30510870 Cost Accounting and Financial Management

Management of Working Capital

2.4.8 Determining The Optimum Cash Balance: A firm should maintain optimum cash balance to cater to the day-to-day operations. It may also carry additional cash as a buffer or safety stock. The amount of cash balance will depend on the risk-return trade off. The firm should maintain optimum - just enough, neither too much nor too little cash balance. This, however, poses a question. How to determine the optimum cash balance if cash flows are predictable and if they are not predictable?

2.5 CASH MANAGEMENT MODELS In recent years several types of mathematical models have been developed which helps to determine the optimum cash balance to be carried by a business organization. The purpose of all these models is to ensure that cash does not remain idle unnecessarily and at the same time the firm is not confronted with a situation of cash shortage. All these models can be put in two categories-inventory type models and stochastic models. Inventory type models have been constructed to aid the finance manager to determine optimum cash balance of his firm. William J. Baumol’s economic order quantity model applies equally to cash management problems under conditions of certainty or where the cash flows are predictable. However, in a situation where the EOQ Model is not applicable, stochastic model of cash management helps in determining the optimum level of cash balance. It happens when the demand for cash is stochastic and not known in advance. 2.5.1 William J. Baumol’s Economic Order Quantity Model, (1952): According to this model, optimum cash level is that level of cash where the carrying costs and transactions costs are the minimum. The carrying costs refers to the cost of holding cash, namely, the interest foregone on marketable securities. The transaction costs refers to the cost involved in getting the marketable securities converted into cash. This happens when the firm falls short of cash and has to sell the securities resulting in clerical, brokerage, registration and other costs. The optimum cash balance according to this model will be that point where these two costs are minimum. The formula for determining optimum cash balance is:

S

PU2C ×=

Where, C = Optimum cash balance U = Annual (or monthly) cash disbursement P = Fixed cost per transaction. S = Opportunity cost of one rupee p.a. (or p.m.)

7.49

Page 862: 30510870 Cost Accounting and Financial Management

Financial Management Illustration 4 A firm maintains a separate account for cash disbursement. Total disbursement are Rs.1,05,000 per month or Rs.12,60,000 per year. Administrative and transaction cost of transferring cash to disbursement account is Rs.20 per transfer. Marketable securities yield is 8% per annum. Determine the optimum cash balance according to William J. Baumol model. Solution

The optimum cash balance C = 100,25.Rs08.0

20.Rs000,60,12.Rs2=

××

The limitation of the Baumol’s model is that it does not allow the cash flows to fluctuate. Firms in practice do not use their cash balance uniformly nor they are able to predict daily cash inflows and outflows. The Miller-Orr (MO) model overcomes this shortcoming and allows for daily cash flow variation. 2.5.2 Miller-Orr Cash Management Model (1966): According to this model the net cash flow is completely stochastic. When changes in cash balance occur randomly the application of control theory serves a useful purpose. The Miller-Orr model is one of such control limit models. This model is designed to determine the time and size of transfers between an investment account and cash account. In this model control limits are set for cash balances. These limits may consist of h as upper limit, z as the return point; and zero as the lower limit. When the cash balance reaches the upper limit, the transfer of cash equal to h – z is invested in marketable securities account. When it touches the lower limit, a transfer from marketable

7.50

Page 863: 30510870 Cost Accounting and Financial Management

Management of Working Capital

securities account to cash account is made. During the period when cash balance stays between (h, z) and (z, 0) i.e. high and low limits no transactions between cash and marketable securities account is made. The high and low limits of cash balance are set up on the basis of fixed cost associated with the securities transactions, the opportunity cost of holding cash and the degree of likely fluctuations in cash balances. These limits satisfy the demands for cash at the lowest possible total costs. The following diagram illustrates the Miller-Orr model.

Return point

Upper control limit

Time Lower control limit

Cas

h B

alan

ce (R

s.)

0

Z

h

The MO Model is more realistic since it allows variations in cash balance within lower and upper limits. The finance manager can set the limits according to the firm’s liquidity requirements i.e., maintaining minimum and maximum cash balance.

2.6 RECENT DEVELOPMENTS IN CASH MANAGEMENT Now-a-days, electronic delivery and payment system are becoming increasingly important because of increased competition and the demand for more efficient and convenient capabilities. A considerable number of transactions and amounts of funds can be moved electronically from one place to another almost instantaneously. Consequently, the opportunities presented are not only beneficial but can create a risk to a business firm. Such threats include internal and external fraud, theft and unauthorized manipulation of financial data. Therefore, we can easily observe the rapid transition from the most basic and traditional principles to now complex strategies dominated by the technology and globalisation, but the

7.51

Page 864: 30510870 Cost Accounting and Financial Management

Financial Management basic goal is same i.e., the efficient utilisation of cash in a way which is consistent with the overall strategic objectives of a business unit. 2.6.1 Electronic Fund Transfer: With the developments which took place in the Information technology, the present banking system is switching over to the computerisation of banks branches to offer efficient banking services and cash management services to their customers. The network will be linked to the different branches, banks. This will help the customers in the following ways: ♦ Instant updation of accounts. ♦ The quick transfer of funds. ♦ Instant information about foreign exchange rates. 2.6.2 Zero Balance Account: For efficient cash management some firms employ an extensive policy of substituting marketable securities for cash by the use of zero balance accounts. Every day the firm totals the cheques presented for payment against the account. The firm transfers the balance amount of cash in the account if any, for buying marketable securities. In case of shortage of cash the firm sells the marketable securities. 2.6.3 Money Market Operations: One of the tasks of ‘treasury function’ of larger companies is the investment of surplus funds in the money market. The chief characteristic of money market banking is one of size. Banks obtain funds by competing in the money market for the deposits by the companies, public authorities, High Networth Investors (HNI), and other banks. Deposits are made for specific periods ranging from overnight to one year, a highly competitive rates which reflect supply and demand on a daily, even hourly basis are quoted. Consequently, the rates can fluctuate quite dramatically, especially for the shorter-term deposits. Surplus funds can thus be invested in money market easily. 2.6.4 Petty Cash Imprest System: For better control on cash, generally the companies use petty cash imprest system wherein the day-to-day petty expenses are estimated taking into account past experience and future needs and generally a week’s requirement of cash will be kept separate for making petty expenses. Again, the next week will commence with the pre-determined balance. This will reduce the strain of the management in managing petty cash expenses and help in the managing cash efficiently. 2.6.5 Management of Temporary Cash Surplus Temporary cash surpluses can be profitably invested in the following: ♦ Short-term deposits in Banks and financial institutions. ♦ Short-term debt market instruments. ♦ Long-term debt instruments.

7.52

Page 865: 30510870 Cost Accounting and Financial Management

Management of Working Capital

♦ Shares of Blue chip listed companies. 2.6.6 Electronic Cash Management System: Most of the cash management systems now-a-days are electronically based, since ‘speed’ is the essense of any cash management system. Electronically, transfer of data as well as funds play a key role in any cash management system. Various elements in the process of cash management are linked through a satellite. Various places that are interlinked may be the place where the instrument is collected, the place where cash is to be transferred in company’s account, the place where the payment is to be transferred etc. Certain networked cash management system may also provide a very limited access to third parties like parties having very regular dealings of receipts and payments with the company etc. A finance company accepting deposits from public through sub-brokers may give a limited access to sub-brokers to verify the collections made through him for determination of his commission among other things. Benefits: Good cash management is a conscious process of knowing: ♦ When, where and how a company’s cash needs will arise. ♦ Knowing what are the best sources of meeting at a short notice additional cash

requirement. ♦ Maintaining good and cordial relations with bankers and other creditors. Scientific cash management results in: ♦ Significant saving in time. ♦ Decrease in interest costs. ♦ Less paper work. ♦ Greater accounting accuracy. ♦ More control over time and funds. ♦ Supports electronic payments. ♦ Faster transfer of funds from one location to another, where required. ♦ Speedy conversion of various instruments into cash. ♦ Making available funds wherever required, whenever required. ♦ Reduction in the amount of ‘idle float’ to the maximum possible extent. ♦ Ensures no idle funds are placed at any place in the organization. ♦ It makes inter-bank balancing of funds much easier.

7.53

Page 866: 30510870 Cost Accounting and Financial Management

Financial Management ♦ It is a true form of centralised ‘Cash Management’. ♦ Produces faster electronic reconciliation. ♦ Allows for detection of book-keeping errors. ♦ Reduces the number of cheques issued. ♦ Earns interest income or reduce interest expense. Even a multinational organization having subsidiaries worldwide, can pool everything internationally so that the company offset the debts with the surplus monies from various subsidiaries. It will result in transformation of treasury function into a profit-centre by optimizing cash and putting it on profitable use. Creative and pro-active cash management solutions can contribute dramatically to a company’s profitability and to its competitive edge. The ultimate purpose of scientific cash management is to ensure solvency, liquidity and profitability of the organization as a whole. 2.6.7 Virtual Banking: The practice of banking has undergone a significant change in the nineties. While banks are striving to strengthen customer base and relationship and move towards relationship banking, customers are increasingly moving away from the confines of traditional branch banking and are seeking the convenience of remote electronic banking services. And even within the broad spectrum of electronic banking the virtual banking has gained prominence Broadly virtual banking denotes the provision of banking and related services through extensive use of information technology without direct recourse to the bank by the customer. The origin of virtual banking in the developed countries can be traced back to the seventies with the installation of Automated Teller Machines (ATMs). Subsequently, driven by the competitive market environment as well as various technological and customer pressures, other types of virtual banking services have grown in prominence throughout the world. The Reserve Bank of India has been taking a number of initiatives, which will facilitate the active involvement of commercial banks in the sophisticated cash management system. One of the pre-requisites to ensure faster and reliable mobility of funds in a country is to have an efficient payment system. Considering the importance of speed in payment system to the economy, the RBI has taken numerous measures since mid Eighties to strengthen the payments mechanism in the country. Introduction of computerized settlement of clearing transactions, use of Magnetic Ink Character Recognition (MICR) technology, provision of inter-city clearing facilities and high value clearing facilities, Electronic Clearing Service Scheme (ECSS), Electronic Funds Transfer (EFT) scheme, Delivery vs. Payment (DVP) for Government securities transactions, setting up of Indian Financial Network (INFINET) are some of the significant developments.

7.54

Page 867: 30510870 Cost Accounting and Financial Management

Management of Working Capital

Introduction of Centralised Funds Management System (CFMS), Securities Services System (SSS), Real Time Gross Settlement System (RTGS) and Structured Financial Messaging System (SFMS) are the other top priority items on the agenda to transform the existing system into a state of the art payment infrastructure in India. The current vision envisaged for the payment systems reforms is one, which contemplates linking up of at least all important bank branches with the domestic payment systems network thereby facilitating cross border connectivity. With the help of the systems already put in place in India and which are coming into being, both banks and corporates can exercise effective control over the cash management. Advantages The advantages of virtual banking services are as follows: ♦ Lower cost of handling a transaction. ♦ The increased speed of response to customer requirements. ♦ The lower cost of operating branch network along with reduced staff costs leads to cost

efficiency. ♦ Virtual banking allows the possibility of improved and a range of services being made

available to the customer rapidly, accurately and at his convenience. The popularity which virtual banking services have won among customers, is due to the speed, convenience and round the clock access they offer.

2.7 CASH MANAGEMENT SERVICES – THE ICICI BANK WAY ICICI Bank offers receivables and payable management solutions to companies under its Cash Management Services. Local cheque collection is offered from more than 630 centres and up country cheques payable at more than 4,500 centres. The cheques can be collected and credited to the centralized account. Companies can plan their expenditure/investments as the credit is provided on a guaranteed day arrangement basis. The vast reach is offered in combination with advanced technology, which enables the companies to receive customized MIS through e-mail and web to take care of reconciliation. Companies can also avail of ICICI Bank’s payment products viz, issue of bulk demand drafts/pay orders, cheque writing, RTGS, ECS and dividend/interest warrants. These products help the companies to reduce administrative cost and improve efficiency. Companies can avoid transit delays and courier costs by using the remote printing (pay orders) facility. (Source: The Economic Times, New Delhi May 2, 2006)

7.55

Page 868: 30510870 Cost Accounting and Financial Management

Financial Management 2.8 MANAGEMENT OF MARKETABLE SECURITIES Management of marketable securities is an integral part of investment of cash as this may serve both the purposes of liquidity and cash, provided choice of investment is made correctly. As the working capital needs are fluctuating, it is possible to park excess funds in some short term securities, which can be liquidated when need for cash is felt. The selection of securities should be guided by three principles. ♦ Safety: Return and risks go hand in hand. As the objective in this investment is ensuring

liquidity, minimum risk is the criterion of selection. ♦ Maturity: Matching of maturity and forecasted cash needs is essential. Prices of long

term securities fluctuate more with changes in interest rates and are therefore, more risky. ♦ Marketability: It refers to the convenience, speed and cost at which a security can be

converted into cash. If the security can be sold quickly without loss of time and price it is highly liquid or marketable.

The choice of marketable securities is mainly limited to Government treasury bills, Deposits with banks and Intercorporate deposits. Units of Unit Trust of India and commercial papers of corporates are other attractive means of parking surplus funds for companies along with deposits with sister concerns or associate companies. Besides this Money Market Mutual Funds (MMMFs) have also emerged as one of the avenues of short-term investment. They focus on short-term marketable securities such as Treasury bills, commercial papers certificate of deposits or call money market. There is a lock in period of 30 days after which the investment may be converted into cash. They offer attractive yields, and are popular with institutional investors and some big companies.

7.56

Page 869: 30510870 Cost Accounting and Financial Management

Management of Working Capital

UNIT – III : MANAGEMENT OF INVENTORY

3.1 INVENTORY MANAGEMENT Inventories constitute a major element of working capital. It is, therefore, important that investment in inventory is property controlled. The objectives of inventory management are, to a great extent, similar to the objectives of cash management. Inventory management covers a large number of problems including fixation of minimum and maximum levels, determining the size of inventory to be carried, deciding about the issues, receipts and inspection procedures, determining the economic order quantity, proper storage facilities, keeping check over obsolescence and ensuring control over movement of inventories. The aspects concerning control over inventories have been discussed in Chapters 1 and 2 of Cost Accounting under Section A of this book.

7.57

Page 870: 30510870 Cost Accounting and Financial Management

Financial Management

UNIT – IV : MANAGEMENT OF RECEIVABLES

4.1 INTRODUCTION A firm needs to offer its goods and services on credit to customers as a Business strategy to boost the sales. This represents a considerable investment of funds so the management of this asset can have significant effect on the profit performance of the company. The basic objective of management of sundry debtors is to optimise the return on investment on this assets known as receivables. Large amounts are tied up in sundry debtors, there are chances of bad debts and there will be cost of collection of debts. On the contrary, if the investment in sundry debtors is low, the sales may be restricted, since the competitors may offer more liberal terms. Therefore, management of sundry debtors is an important issue and requires proper policies and their implementation. While studying management of accounts receivable, we focus on its importance, what determines the investment in it, what are the decision variables involved and how do we determine them. Investment in accounts receivables constitute a substantial portion of a firms assets. Moreover, since cash flows from a sale cannot be invested until the accounts receivable are collected their control warrants added importance, efficient collection will lead to both profitability and liquidity of the firm.

4.2 ROLE TO BE PLAYED BY THE FINANCE MANAGER: It is possible that the finance manager can affect the volume of credit sales and collection period and consequently, the investment in receivables. That is through the changes in credit policy. The term credit policy is used to refer to the combination of three decisions variables: (i) credit standards; (ii) Credit terms; and (iii) Collection efforts. Credit standards refer to the criteria’s to decide the types of customers to whom goods could be sold on credit. Whereas credit terms specify the duration of credit and terms of payments by customers. Collection efforts determine the actual collection period. The lower the collection period, the lower the investment in accounts receivables.

4.3 ASPECTS OF MANAGEMENT OF DEBTORS There are basically three aspects of management of sundry debtors.

7.58

Page 871: 30510870 Cost Accounting and Financial Management

Management of Working Capital

1. Credit policy: The credit policy is to be determined. It involves a trade off between the profits on additional sales that arise due to credit being extended on the one hand and the cost of carrying those debtors and bad debt losses on the other. This seeks to decide credit period, cash discount and other relevant matters. The credit period is generally stated in terms of net days. For example if the firm’s credit terms are “net 50”. It is expected that customers will repay credit obligations not later than 50 days. Further, the cash discount policy of the firm specifies: (a) The rate of cash discount. (b) The cash discount period; and (c) The net credit period. For example, the credit terms may be expressed as “3/15 net 60”. This means that a 3% discount will be granted if the customer pays within 15 days; if he does not avail the offer he must make payment within 60 days. 2. Credit Analysis: This require the finance manager to determine as to how risky it is to advance credit to a particular party. 3. Control of receivable: This requires finance manager to follow up debtors and decide about a suitable credit collection policy. It involves both laying down of credit policies and execution of such policies. There is always cost of maintaining receivables which comprises of following costs: (i) The company requires additional funds as resources are blocked in receivables which

involves a cost in the form of interest (loan funds) or opportunity cost (own funds) (ii) Administrative costs which include record keeping, investigation of credit worthiness etc. (iii) Collection costs. (iv) Defaulting costs.

4.4 FACTORS DETERMINING CREDIT POLICY The credit policy is an important factor determining both the quantity and the quality of accounts receivables. Various factors determine the size of the investment a company makes in accounts receivables. They are, for instance: (i) The effect of credit on the volume of sales; (ii) Credit terms; (iii) Cash discount; (iv) Policies and practices of the firm for selecting credit customers.

7.59

Page 872: 30510870 Cost Accounting and Financial Management

Financial Management (v) Paying practices and habits of the customers. (vi) The firm’s policy and practice of collection. (vii) The degree of operating efficiency in the billing, record keeping and adjustment function,

other costs such as interest, collection costs and bad debts etc., would also have an impact on the size of the investment in receivables. The rising trend in these costs would depress the size of investment in receivables.

The firm may follow a lenient or a stringent credit policy. The firm which follows a lenient credit policy sells on credit to customers on very liberal terms and standards. On the contrary a firm following a stringent credit policy sells on credit on a highly selective basis only to those customers who have proper credit worthiness and who are financially sound. Any increase in accounts receivables that is, additional extension of trade credit not only results in higher sales but also requires additional financing to support the increased investment in accounts receivables. The costs of credit investigations and collection efforts and the chances of bad debts are also increased.

4.5 FACTORS UNDER THE CONTROL OF THE FINANCE MANAGER The finance manager has operating responsibility for the management of the investment in receivables. In addition to his role of supervising the administration of credit, the finance manager is in a particularly strategic position to contribute to top management decisions relating to the best credit policies of the firm. In the beginning, the finance manager plays a very important role in the determination of credit period and deciding the criteria for selection of credit applications. Once it has been done and the management has determined the role of credit in the package of goods and services offered, the finance manager has relatively little impact upon the level of receivables. He may, however, limit the amount of receivables by rejecting occasionally credit applications or he may speed up the conversion of receivables into cash by aggressive collection policy. But these activities have smaller effect upon the level of receivables than the initial and fundamental decision regarding the terms of credit and the overall credit standards laid down by the firm. The basic objective of receivables management should be to maximize return on total investment. The policies which stress short credit terms, stringent credit standards, and aggressive collection policies would, no doubt, reduce the size of investment in receivables and also minimize bad debt losses, but such policies would also restrict sales and profit margins. As a consequence, despite the low investment in receivables, the rate of return on total investment of the firm would be lower than that attainable with higher levels of sales, profits and receivables. The finance manager has to strike a balance between the cost of increased investment in receivables and profits from the higher levels of sales.

7.60

Page 873: 30510870 Cost Accounting and Financial Management

Management of Working Capital

Illustration 1 A trader whose current sales are in the region of Rs.6 lakhs per annum and an average collection period of 30 days wants to pursue a more liberal policy to improve sales. A study made by a management consultant reveals the following information:-

Credit Policy Increase in collection period

Increase in sales Present default anticipated

A 10 days Rs.30,000 1.5% B 20 days Rs.48,000 2% C 30 days Rs.75,000 3% D 45 days Rs.90,000 4%

The selling price per unit is Rs.3. Average cost per unit is Rs.2.25 and variable costs per unit are Rs.2. The current bad debt loss is 1%. Required return on additional investment is 20%. Assume a 360 days year. Which of the above policies would you recommend for adoption? Solution

Evaluation of Credit Policies Part I

Credit Policy Exiting A B C D

Credit Period (Days) 30 40 50 60 75 Expected additional sales (Rs.)

30,000 48,000 75,000 90,000

Contribution of additional sales (one-third of selling price)

10,000 16,000 25,000 30,000

Bad debs (Expected Sales × Default percentage)

6,000 9,450 12,960 20,250 27,600

Additional bad debts -- 3,450 6,960 14,250 21,600

7.61

Page 874: 30510870 Cost Accounting and Financial Management

Financial Management Contribution of additional sales less additional bad debts (A)

__ 6,550 9,040 10,750 8,400

Part II Expected sales (Rs.) 6,00,000 6,30,000 6,48,000 6,75,000 6,90,000 Receivables turnover ratio 12 9 7.2 6 4.8 Average receivables 50,000 70,000 90,000 1,12,500 1,43,750 Investment in receivables (Receivables × Variable cost i.e, two-thirds of sales price i.e. Rs.50,000 × 2/3 = Rs.33,333 and so on)

33,333

46,667

60,000

75,000

95,833 Additional investment in receivables

__

13,334

26,667

41,667

62,500

Required return on additional investment at 20% (B)

__

2,667

5,333

8,333

12,500

Excess of additional contribution over required return on additional investment in receivables (A)-(B)

__

3,883

3,707

2,417

(4,100) The additional contribution over required return on additional investment in receivables is the maximum under Credit Policy A. Hence, Policy A is recommended for adoption followed by B and C. Policy D cannot be adopted because it would result in the reduction of the existing profits. Illustration 2 XYZ Corporation is considering relaxing its present credit policy and is in the process of evaluating two proposed policies. Currently, the firm has annual credit sales of Rs.50 lakhs and accounts receivable turnover ratio of 4 times a year. The current level of loss due to bad debts is Rs.1,50,000. The firm is required to give a return of 25% on the investment in new accounts receivables. The company’s variable costs are 70% of the selling price. Given the following information, which is the better option?

7.62

Page 875: 30510870 Cost Accounting and Financial Management

Management of Working Capital

(Amount in Rs.)

Present Policy

Policy Option I

Policy Option I

Annual credit sales 50,00,000 60,00,000 67,50,000 Accounts receivable turnover ratio 4 times 3 times 2.4 times Bad debt losses 1,50,000 3,00,000 4,50,000 Solution

XYZ CORPORATION Evaluation of Credit Policies

(Amount in Rs.)

Present Policy

Policy Option I

Policy Option II

Annual credit sales 50,00,000 60,00,000 67,50,000 Accounts receivable turnover 4 times 3 times 2.4 times Average collection period 3 months 4 months 5 months Average level of accounts receivable 12,50,000 20,00,000 28,12,500 Marginal increase in investment in receivable less profit margin

___

5,25,000

5,68,750

Marginal increase in sales ___ 10,00,000 7,50,000 Profit on marginal increase in sales (30%) ___ 3,00,000 2,25,000 Marginal increase in bad debt losses ___ 1,50,000 1,50,000Profit on marginal increase in sales less marginal bad debts loss

___ ___

1,50,000

75,000

Required return on marginal investment @ 25% ___ 1,31,250 1,42,188Surplus (loss) after required rate of return ___ 18,750

(67,188)

7.63

Page 876: 30510870 Cost Accounting and Financial Management

Financial Management It is clear from the above that the policy option I has a surplus of Rs.18,750/- whereas option II shows a deficit of Rs.67,188/- on the basis of 25% return. Hence policy option I is better. Illustration 3 As a part of the strategy to increase sales and profits, the sales manager of a company proposes to sell goods to a group of new customers with 10% risk of non-payment. This group would require one and a half months credit and is likely to increase sales by Rs.1,00,000 p.a. Production and Selling expenses amount to 80% of sales and the income-tax rate is 50%. The company’s minimum required rate of return (after tax) is 25%. Should the sales manager’s proposal be accepted? Also find the degree of risk of non-payment that the company should be willing to assume if the required rate of return (after tax) were (i) 30%, (ii) 40% and (iii) 60%. Solution Extension of credit to a group of new customers:

Profitability of additional sales: Rs. Increase in sales per annum 1,00,000 Less Bad debt losses (10%) of sales 10,000Net sales revenue 90,000 Less Production and selling expenses (80% of sales) 80,000Profit before tax 10,000 Less Income tax (50%) 5,000Profit after tax 5,000 Average investment in additional receivables. Period of credit: 1 2

1 months

Receivables turnover: 8112

21=

Average amount of receivables: 500,12.Rs8

000,00,1.Rs=

Average investment in receivables: Rs.12,500 × 80% = Rs.10,000

7.64

Page 877: 30510870 Cost Accounting and Financial Management

Management of Working Capital

The available rate of return: 000,10.Rs000,5.Rs × 100 = 50%

Since the available rate of return is 50%, which is higher than the required rate of return of 25%, the Sales Manager’s proposal should be accepted. (i) Acceptable degree of risk of non-payment if the required rate of return (after tax is 30%) Required amount of profit after tax on investment: Rs.10,000 × 30% = Rs.3,000 Required amount of profit before tax at this level:

000,6.Rs50

100000,3.Rs=

×

Net sales revenue required: Rs.80,000 + Rs.6,000 = Rs.86,000 Acceptable amount of bad debt losses: Rs.1,00,000 – Rs.86,000 = Rs.14,000

Acceptable degree of risk of non-payment:

%14100x000,00,1.Rs

000,14.Rs=

(ii) Acceptable degree of risk of non-payment if the required rate of return (after tax) is 40%: Required amount of profit after tax on investment: Rs.10,000 × 40% = Rs.4,000 Required amount of profit before tax

000,8.Rs50

100x000,4.Rs=

Net sales revenue required: Rs.80,000 + Rs.8,000 = Rs.88,000 Acceptable amount of bad debt losses: Rs.1,00,000 – Rs.88,000 = Rs.12,000 Acceptable degree of risk of non-payment:

7.65

Page 878: 30510870 Cost Accounting and Financial Management

Financial Management

%12100000,00,1000,12.Rs

(iii) Acceptable degree of risk of non-payment of the required rate of return (after tax) is 60%: Required amount of profit after tax on investment: Rs.10,000 × 60% = Rs.6,000 Required amount of profit before tax:

000,12.Rs50

100000,6.Rs=

×

Net sales revenue required: Rs.80,000 + Rs.12,000 = Rs.92,000 Acceptable amount of bad debt losses:

Rs.1,00,000 – Rs.92,000 = Rs.8,000 Acceptable degree of risk of non-payment:

%8100000,00,1.Rs

000,8.Rs=×

Illustration 4 Slow Payers are regular customers of Goods Dealers Ltd., Calcutta and have approached the sellers for extension of a credit facility for enabling them to purchase goods from Goods Dealers Ltd. On an analysis of past performance and on the basis of information supplied, the following pattern of payment schedule emerges in regard to Slow Payers:

Schedule Pattern At the end of 30 days 15% of the bill At the end of 60 days 34% of the bill. At the end of 90 days 30% of the bill. At the end of 100 days 20% of the bill. Non-recovery 1% of the bill.

Slow Payers want to enter into a firm commitment for purchase of goods of Rs.15 lakhs in 2005, deliveries to be made in equal quantities on the first day of each quarter in the calendar year. The price per unit of commodity is Rs.150 on which a profit of Rs.5 per unit is expected to be made. It is anticipated by Goods Dealers Ltd., that taking up of this contract would mean

7.66

Page 879: 30510870 Cost Accounting and Financial Management

Management of Working Capital

an extra recurring expenditure of Rs.5,000 per annum. If the opportunity cost of funds in the hands of Goods Dealers is 24% per annum, would you as the finance manager of the seller recommend the grant of credit to Slow Payers? Workings should form part of your answer. Assume year of 360 days. Solution

Evaluation of Extension of Credit Facility to Slow Payers: (i) Anticipated Return on the Contract Rs. (ii)

Margin return: ⎟⎠⎞

⎜⎝⎛ ×5

150000,00,15.Rs

50,000

Less: Recurring annual costs 5,000 ______

Net anticipated return 45,000(ii) Quarterly sales value of the goods to be delivered on 1st January,

1st April, 1st July nd 1st October: ⎟⎠⎞

⎜⎝⎛

4000,00,15.Rs

3,75,000

(iii) Opportunity Cost (Interest Cost) of Funds to be Locked up: Amount due for each quarter Period

(Days) Products for

each quarter

Rs.56,250 (15% of Rs.3,75,000) 30 days 16,87,500 Rs.1,27,500 (34% of Rs.3,75,000). 60 days 76,50,000 Rs.1,12,500 (30% of Rs.3,75,000) 90 days 1,01,25,000 Rs.75,000 (20% of Rs.3,75,000) 100 days 75,00,000 Rs.3,750 (1% of Rs.3,75,000) Non recovery

(See Note 1)

Total Products 2,69,62,500 Amount of interest cost for the year @ 24%

p.a.:

⎟⎠⎞

⎜⎝⎛ ×× 4

10024

360500,62,69,2

71,900

7.67

Page 880: 30510870 Cost Accounting and Financial Management

Financial Management

(iv) Total Non-recovery of Bad Debts for the year: (Rs.3,750 × 4)

15,000

(v) Profitability of Proposed Grant of Credit Facility:

Net anticipated return from sales 45,000 Less: Interest cost on funds

:Locked up 71,900 …….. Bad debts 15,000 Profits (Loss)

86,900 (41,900)

In the light of the above the finance manager will not recommend the grant of credit facility to Slow Payers as it is not profitable. Note:(i) Interest cost could be calculated on the amount of bad debts also.

(ii) Interest cost could be calculated on the amount of cost of sales instead of sales value.

4.6 FINANCING RECEIVABLES Pledging of accounts receivables and Factoring have emerged as the important sources of financing of accounts receivables now a days. (i) Pledging: This refers to the use of a firm’s receivable to secure a short term loan. A firm’s receivables can be termed as its most liquid assets and this serve as prime collateral for a secured loan. The lender scrutinizes the quality of the accounts receivables, selects acceptable accounts, creates a lien on the collateral and fixes the percentage of financing receivables which ranges around 50 to 90%. The major advantage of pledging accounts receivables is the ease and flexibility it provides to the borrower. Moreover, financing is done regularly. This, however, suffers on account of high cost of financing. (ii) Factoring: Factoring is a new concept in financing of accounts receivables. This refers to out right sale of accounts receivables to a factor or a financial agency. A factor is a firm that acquires the receivables of other firms. The factoring lays down the conditions of the sale in a factoring agreement. The factoring agency bears the right of collection and services the accounts for a fee. Normally, factoring is the arrangement on a non-recourse basis where in the event of default the loss is borne by this factor. However, in a factoring arrangement with recourse, in such situation, the accounts receivables will be turned back to the firm by the factor for resolution. There are a number of financial distributors providing factoring services in India. Some

7.68

Page 881: 30510870 Cost Accounting and Financial Management

Management of Working Capital

commercial banks and other financial agencies provide this service. The biggest advantages of factoring are the immediate conversion of receivables into cash and predicted pattern of cash flows. Financing receivables with the help of factoring can help a company having liquidity without creating a net liability on its financial condition. Besides, factoring is a flexible financial tool providing timely funds, efficient record keepings and effective management of the collection process. This is not considered to be as a loan. There is no debt repayment, no compromise to balance sheet, no long term agreements or delays associated with other methods of raising capital. Factoring allows the firm to use cash for the growth needs of business. Illustration 5 A Factoring firm has credit sales of Rs.360 lakhs and its average collection period is 30 days. The financial controller estimates, bad debt losses are around 2% of credit sales. The firm spends Rs.1,40,000 annually on debtors administration. This cost comprises of telephonic and fax bills along with salaries of staff members. These are the avoidable costs. A Factoring firm has offered to buy the firm’s receivables. The factor will charge 1% commission and will pay an advance against receivables on an interest @15% p.a. after withholding 10% as reserve. What should the firm do? Assume 360 days in a year. Solution

Average level of receivables = Rs.360 lakhs × 36030 = 30 lakhs

Factoring Commission = 1% of Rs.30,00,000 = Rs.30,000 Reserve = 10% of Rs.30,00,000 = Rs.3,00,000 Total (i) = Rs.3,30,000 Thus, the amount available for advance is Average level of receivables Rs.30,00,000 Less: Total (i) from above Rs. 3,30,000 (ii) Rs.26,70,000 Less: Interest @ 15% p.a. for 30 days Rs. 33,375 Net Amount of Advance available. Rs.26,36,625

7.69

Page 882: 30510870 Cost Accounting and Financial Management

Financial Management Evaluation of Factoring Proposal Cost to the Firm

Factoring Commission = Rs.30,00,000 × 000,60,3.Rs30360

=

Interest charges = Rs. 33,375 ×500,60,7.Rs500,00,4.Rs

30360

=

Savings to the firm Rs. Cost of credit administration 1,40,000 Cost of bad-debt losses, 0.02 × 360 lakhs 7,20,000 8,60,000 ∴ The Net benefit to the firm

Rs. Savings to the firm 8,60,000 - Cost to the firm 7,60,500 Net Savings 99,500 Conclusion: Since the savings to the firm exceeds the cost to the firm on account of factoring, ∴The proposal is acceptable.

4.7 INNOVATIONS IN RECEIVABLE MANAGEMENT During the recent years, a number of tools, techniques, practices and measures have been invented to increase effectiveness in accounts receivable management. Following are the major determinants for significant innovations in accounts receivable management and process efficiency. 1. Re-engineering Receivable Process: In some of the organizations real cost reductions

and performance improvements have been achieved by re-engineering in accounts receivable process. Re-engineering is a fundamental re-think and re-design of business processes by incorporating modern business approaches. The nature of accounts receivables is such that decisions made elsewhere in the organization are likely to affect the level of resources that are expended on the management of accounts receivables.

The following aspects provides an opportunity to improve the management of accounts receivables.

7.70

Page 883: 30510870 Cost Accounting and Financial Management

Management of Working Capital

(a) Centralisation: Centralisation of high nature transactions of accounts receivables and payable is one of the practice for better efficiency. This focuses attention on specialized groups for speedy recovery.

(b) Alternative Payment Strategies: Alternative payment strategies in addition to traditional practices, result into efficiencies in the management of accounts receivables. It is observed that payment of accounts outstanding is likely to be quicker where a number of payment alternatives are made available to customers. Besides, this convenient payment methods is a marketing tool that is of benefit in attracting and retaining customers. The following alternative modes of payment may also be used alongwith traditional methods like Cheque Book etc., for making timely payment, added customer service, reducing remittance processing costs and improved cash flows and better debtor turnover. (i) Direct debit: I.e., authorization for the transfer of funds from the purchasers

bank account. (ii) Integrated Voice Response: This system uses human operators and a

computer based system to allow customers to make payment over phone, generally by credit card. This system has proved to be beneficial in the orgnisations processing a large number of payments regularly.

(iii) Collection by a third party: The payment can be collected by an authorized external firm. The payments can be made by cash, cheque, credit card or Electronic fund transfer. Banks may also be acting as collecting agents of their customers and directly depositing the collections in customers bank accounts.

(iv) Lock Box Processing: Under this system an outsourced partner captures cheques and invoice data and transmits the file to the client firm for processing in that firm’s systems.

(v) Payments via Internet. (c) Customer Orientation: Where individual customers or a group of customers have

some strategic importance to the firm a case study approach may be followed to develop good customer relations. A critical study of this group may lead to formation of a strategy for prompt settlement of debt.

2. Evaluation of Risk: Risk evaluation is a major component in the establishment of an effective control mechanism. Once risks have been properly assessed controls can be introduced to either contain the risk to an acceptable level or to eliminate them entirely. This also provides an opportunity for removing inefficient practices. This involves a re-think of processes and questioning the way that tasks are performed. This also opens

7.71

Page 884: 30510870 Cost Accounting and Financial Management

Financial Management

the way for efficiency and effectiveness benefits in the management of accounts receivables.

3. Use of Latest Technology: Technological developments now-a-days provides an opportunity for improvement in accounts receivables process. The major innovations available are the integration of systems used in the management of accounts receivables, the automation and the use of e-commerce. (a) E-commerce refer to the use of computer and electronic telecommunication

technologies, particularly on an inter-organisational level, to support trading in goods and services. It uses technologies such as Electronic Data Inter-change (EDI), Electronic Mail, Electronic Funds Transfer (EFT) and Electronic Catalogue Systems to allow the buyer and seller to transact business by exchange of information between computer application systems.

(b) Accounts Receivable Systems: Now-a-days all the big companies develop and maintain automated receivable management systems. Manual systems of recording the transactions and managing receivables is not only cumbersome but ultimately costly also. These integrated systems automatically update all the accounting records affected by a transaction. For example, if a transaction of credit sale is to be recorded, the system increases the amount the customer owes to the firm, reduces the inventory for the item purchased, and records the sale. This system of a company allows the application and tracking of receivables and collections, using the automated receivables system allows the company to store important information for an unlimited number of customers and transactions, and accommodate efficient processing of customer payments and adjustments.

4. Receivable Collection Practices: The aim of debtors collection should be to reduce, monitor and control the accounts receivable at the same time maintain customer goodwill. The fundamental rule of sound receivable management should be to reduce the time lag between the sale and collection. Any delays that lengthen this span causes receivables to unnecessary build up and increase the risk of bad debts. This is equally true for the delays caused by billing and collection procedures as it is for delays caused by the customer.

The following are major receivable collection procedures and practices: (i) Issue of Invoice. (ii) Open account or open-end credit. (iii) Credit terms or time limits. (iv) Periodic statements. (v) Use of payment incentives and penalties.

7.72

Page 885: 30510870 Cost Accounting and Financial Management

Management of Working Capital

(vi) Record keeping and Continuous Audit. (vii) Export Factoring: Factors provide comprehensive credit management, loss

protection collection services and provision of working capital to the firms exporting internationally.

(viii) Business Process Outsourcing: This refers to a strategic business tool whereby an outside agency takes over the entire responsibility for managing a business process.

5. Use of Financial tools/techniques: The finance manager while managing accounts receivables uses a number of financial tools and techniques. Some of them have been described hereby as follows: (i) Credit analysis: While determining the credit terms, the firm has to evaluate individual customers in respect of their credit worthiness and the possibility of bad debts. For this purpose, the firm has to ascertain credit rating of prospective customers. Credit rating: An important task for the finance manager is to rate the various debtors who seek credit facility. This involves decisions regarding individual parties so as to ascertain how much credit can be extended and for how long. In foreign countries specialized agencies are engaged in the task of providing rating information regarding individual parties. Dun and Broadstreet is one such source. The finance manager has to look into the credit-worthiness of a party and sanction credit limit only after he is convinced that the party is sound. This would involve an analysis of the financial status of the party, its reputation and previous record of meeting commitments. The credit manager here has to employ a number of sources to obtain credit information. The following are the important sources: Trade references; Bank references; Credit bureau reports; Past experience; Published financial statements; and Salesman’s interview and reports. Once the credit-worthiness of a client is ascertained, the next question is to set a limit of the credit. In all such enquiries, the credit manager must be discreet and should always have the interest of high sales in view. (ii) Decision tree analysis of granting credit: The decision whether to grant credit or not is a decision involving costs and benefits. When a customer pays, the seller makes profit but when he fails to pay the amount of cost going into the product is also gone. If the relative chances of recovering the dues can be decided it can form a probability distribution of payment or non-payment. If the chances of recovery are 9 out of 10 then probability of recovery is 0.9 and that of default is 0.1.

7.73

Page 886: 30510870 Cost Accounting and Financial Management

Financial Management

Credit evaluation of a customer shows that the probability of recovery is 0.9 and that of default is 0.1. the revenue from the order is Rs.5 lakhs and cost is Rs.4 lakhs. The decision is whether credit should be granted or not. The analysis is presented in the following diagram.

Rs.1,00,000.00

Rs.4,00,000.00

Grant

Do not grant

The weighted net benefit is Rs.[1,00,000 × 0.9 i.e. 90,000 – 0.1 × 4,00,000 i.e. 40,000] = 50,000. So credit should be granted. (iii) Control of receivables: Another aspect of management of debtors is the control of receivables. Merely setting of standards and framing a credit policy is not sufficient; it is, equally important to control receivables. (iv) Collection policy: Efficient and timely collection of debtors ensure that the bad debt losses are reduced to the minimum and the average collection period is shorter. If a firm spends more resources on collection of debts, it is likely to have smaller bad debts. Thus, a firm must work out the optimum amount that it should spend on collection of debtors. This involves a trade off between the level of expenditure on the one hand and decrease in bad debt losses and investment in debtors on the other. The collection cell of a firm has to work in a manner that it does not create too much resentment amongst the customers. On the other hand, it has to keep the amount of the outstandings in check. Hence, it has to work in a very smoothen manner and diplomatically. It is important that clear-cut procedures regarding credit collection are set up. Such procedures must answer questions like the following: (a) How long should a debtor balance be allowed to exist before collection process is started. (b) What should be the procedure of follow up with defaulting customer? How reminders are to be sent and how should each successive reminder be drafted? (c) Should there be a collection machinery whereby personal calls by company’s representatives are made?

7.74

Page 887: 30510870 Cost Accounting and Financial Management

Management of Working Capital

(d) What should be the procedure for dealing with doubtful accounts? Is legal action to be instituted? How should account be handled?

4.8 MONITORING OF RECEIVABLES (i) Computation of average age of receivables: It involves computation of average

collection period. (ii) Ageing Schedule: When receivables are analysed according to their age, the process is

known as preparing the ageing schedules of receivables. The computation of average age of receivables is a quick and effective method of comparing the liquidity of receivables with the liquidity of receivables in the past and also comparing liquidity of one firm with the liquidity of the other competitive firm. It also helps the firm to predict collection pattern of receivables in future. This comparison can be made periodically. The purpose of classifying receivables by age groups is to have a closer control over the quality of individual accounts. It requires going back to the receivables ledger where the dates of each customer’s purchases and payments are available. The ageing schedule, by indicating a tendency for old accounts to accumulate, provides a useful supplement to average collection period of receivables/sales analysis. Because an analysis of receivables in terms of associated dates of sales enables the firm to recognise the recent increases, and slumps in sales. To ascertain the condition of receivables for control purposes, it may be considered desirable to compare the current ageing schedule with an earlier ageing schedule in the same firm and also to compare this information with the experience of other firms. The following is an illustration of the ageing schedule of receivables:-

Ageing Schedule Age Classes (Days)

As on 30th June, 2005 As on 30th September, 2005

Month of Sale

Balance of Receivables

Percentage to total

Month of Sale

Balance of Receivables

Percentage to total

(Rs.) (Rs.) 1-30 June 41,500 11.9 September 1,00,000 22.7 31-60 May 74,200 21.4 August 2,50,000 56.8 61-90 April 1,85,600 53.4 July 48,000 10.9 91-120 March 35,300 10.2 June 40,000 9.1 121 and more

Earlier 10,800 3.1 Earlier 2,000 0.5

_______ 3,47,400

___ 100

_______ 4,40,000

___ 100

7.75

Page 888: 30510870 Cost Accounting and Financial Management

Financial Management The above ageing schedule shows a substantial improvement in the liquidity of receivables for the quarter ending September, 2005 as compared with the liquidity of receivables for the quarter ending June, 2005. It could be possible due to greater collection efforts of the firm. (iii) Collection Programme:

(a) Monitoring the state of receivables. (b) Intimation to customers when due date approaches. (c) Telegraphic and telephonic advice to customers on the due date. (d) Threat of legal action on overdue A/cs. (e) Legal action on overdue A/cs.

The following diagram shows the relationship between collection expenses and bad debt losses which has to be established as initial increase in collection expenses may have only a small impact on bad debt losses.

7.76

Page 889: 30510870 Cost Accounting and Financial Management

Management of Working Capital

UNIT – V : FINANCING OF WORKING CAPITAL

5.1 INTRODUCTION After determining the amount of working capital required, the next step to be taken by the finance Manager is to arrange the funds. As discussed earlier, it is advisable that the finance manager bifurcates the working capital requirements between the permanent working capital and temporary working capital. The permanent working capital is always needed irrespective of sales fluctuations, hence should be financed by the long-term sources such as debt and equity. On the contrary the temporary working capital may be financed by the short-term sources of finance. The short-term sources of finance, which are generally expected to be matured within the same operating cycle or say within the same accounting year or at the most in next year, finance a major portion of total current assets. This requires a number of decisions to be taken by the finance manager with regard to the Cash Balance and the timing of cash to be maintained, investment in short-term securities, when the payment to creditors is to be made, when and how much funds are to be raised by borrowings. Most of these sources are not often close substitute for one another because each source has unique characteristics, advantages and disadvantages. The present unit, focuses on (i) the different sources of financing working capital requirements as well as recent developments. Broadly speaking, the working capital finance may be classified between the two categories: (i) Spontaneous sources. (ii) Negotiable sources. The finance manager has to be very careful while selecting a particular source, or a combination thereof for financing of working capital. Generally, the following parameters will guide his decisions in this respect: (i) Cost factor (ii) Impact on credit rating (iii) Feasibility (iv) Reliability (v) Restrictions (vi) Hedging approach or matching approach i.e., Financing of assets with the same maturity

as of assets.

7.77

Page 890: 30510870 Cost Accounting and Financial Management

Financial Management The spontaneous sources of finance are those which naturally arise in the course of business operations. Trade credit, credit from employees, credit from suppliers of services, etc. Are some of the examples which may be quoted in this respect. On the other hand the negotiated sources, as the name implies, are those which have to be specifically negotiated with lenders say, commercial banks, financial institutions, general public etc.

5.2 SOURCES OF FINANCE 5.2.1 As outlined above trade credit is a spontaneous source of finance which is normally extended to the purchaser organization by the sellers or services providers. This source of financing working capital is more important since it contributes to about one-third of the total short-term requirements. The dependence on this source is higher due to lesser cost of finance as compared with other sources. Trade credit is guaranteed when a company acquires supplies, merchandise or materials and does not pay immediately. If a buyer is able to get the credit with out completing much formalities, it is termed as ‘open account trade credit.’ On the other hand in the case of “Bills Payable” the purchaser will have to give a written promise to pay the amount of the bill/invoice either on demand or at a fixed future date to the seller or the bearer of the note. Due to its simplicity, easy availability and lesser explicit cost, the dependence on this source is much more in all small or big organizations. Especially, for small enterprises this form of credit is more helpful to small and medium enterprises. The amount of such financing depends on the volume of purchases and the payment timing. Another spontaneous source of short-term financing is the accrued expenses or the outstanding expenses liabilities. The accrued expenses refer to the services availed by the firm, but the payment for which has yet to be made. It is a built in and an automatic source of finance as most of the services like wages, salaries, taxes, duties etc., are paid at the end of the period. The accrued expenses represent an interest free source of finance. There is no explicit or implicit cost associated with the accrued expenses and the firm can ensure liquidity by accruing these expenses. 5.2.2 Inter-corporate Loans and Deposits: Sometime, organizations having surplus funds invest for shot-term period with other organizations. The rate of interest will be higher than the bank rate of interest and depending on the financial soundness of the borrower company. This source of finance reduces dependence on bank financing. 5.2.3 Commercial Papers: Commercial Paper (CP) is an unsecured promissory note issued by a firm to raise funds for a short period. This is an instrument that enables highly rated corporate borrowers for short-term borrowings and provides an additional financial instrument

7.78

Page 891: 30510870 Cost Accounting and Financial Management

Management of Working Capital

to investors with a freely negotiable interest rate. The maturity period ranges from minimum 7 days to less than 1 year. 5.2.4 Commercial Papers in India: Since the CP represents an unsecured borrowing in the money market, the regulation of CP comes under the purview of the Reserve Bank of India which issued guidelines in 1990 on the basis of the recommendations of the Vaghul Working Group. These guidelines were aimed at: (i) Enabling the highly rated corporate borrowers to diversify their sources of short term

borrowings, and (ii) To provide an additional instrument to the short term investors. These guidelines have stipulated certain conditions meant primarily to ensure that only financially strong companies come forward to issue the CP. Subsequently, these guidelines have been modified. The main features of the guidelines relating to issue of CP in India may be summarized as follows: (i) CP should be in the form of usance promissory note negotiable by endorsement and

delivery. It can be issued at such discount to the face value as may be decided by the issuing company. CP is subject to payment of stamp duty.

(ii) The aggregate amount that can be raised by commercial papers is not restricted any longer to the company’s cash credit component of the Maximum Permissible Bank Finance.

(iii) CP is issued in the denomination of Rs.5,00,000, but the maximum lot or investment is Rs.25,00,000 per investor. The secondary market transactions can be of Rs.5,00,000 or multiples thereof. The total amount proposed to be issued should be raised within two weeks from the date on which the proposal is taken on record by the bank.

(iv) CP should be issued for a minimum period of 7 days and a maximum of less than 1 year. No grace period is allowed for repayment and if the maturity date falls on a holiday, then it should be paid on the previous working day. Each issue of CP is treated as a fresh issue.

(v) Commercial papers can be issued by a company whose (i) tangible net worth is not less than Rs.5 crores, (ii) funds based working capital limit is not less than 4 crores, (iii) shares are listed on a stock exchange, (iv) specified credit rating of P2 is obtained from CRISIL or A2 from ICRA, and (v) the current ratio is 1.33:1.

(vi) The issue expenses consisting of dealers fees, credit rating agency fees and other relevant expenses should be borne by the issuing company.

(vii) CP may be issued to any person, banks, companies. The issue of CP to NRIs can only be on a non-repatriable basis and is not transferable.

7.79

Page 892: 30510870 Cost Accounting and Financial Management

Financial Management (viii) CP can be issued up to 100% of the fund based working capital loan limit. The working

capital limit is reduced accordingly on issuance of CP. (ix) Deposits by the issue of CP have been exempted from the provisions of section 58A of

the Companies Act, 1956. Any company proposing to issue CP has to submit an application to the bank which provide working capital limit to it, along with the credit rating of the firm. The issue has to be privately placed within two weeks by the company or through a merchant banker. The initial investor pays the discounted value of the CP to the firm. Thus, CP is issued only through the bank who has sanctioned the working capital limit to the company. It is counted as a part of the total working capital limit and it does not increase the working capital resources of the firm.

Annual Financing Cost = MP360x

SPSPFV−

Where FV = Face value of CP SP = Issue price of CP MP = Maturity period of CP. For example, a CP of the face value of Rs.6,00,000 is issued at Rs.5,80,000 for a maturity period of 120 days. The annual financing cost of the CP is:

Annual Financing Cost = 120360x

000,80,5.Rs000,80,5.Rs000,00,6.Rs −

= 10.34% In the same case, if the maturity period is 180 days, then the annual financing cost is:

Annual Financing Cost = 180360x

000,80,5.Rs000,80,5.Rs000,00,6.Rs −

= 6.90% For the same maturity periods of 120 days and 180 days, if the issue price is taken at Rs.5,60,000, then the annual financing cost comes to 21.42% and 14.28% respectively. So, it can be seen that the cost of CP varies inversely to the issue price as well as the maturity period. 5.2.5 CP as a Source of Financing: From the point of the issuing company, CP provides the following benefits: (a) CP is sold on an unsecured basis and does not contain any restrictive conditions.

7.80

Page 893: 30510870 Cost Accounting and Financial Management

Management of Working Capital

(b) Maturing CP can be repaid by selling new CP and thus can provide a continuous source of funds.

(c) Maturity of CP can be tailored to suit the requirement of the issuing firm. (d) CP can be issued as a source of fund even when money market is tight. (e) Generally, the cost of CP to the issuing firm is lower than the cost of commercial bank

loans. However, CP as a source of financing has its own limitations: (i) Only highly credit rating firms can use it. New and moderately rated firm generally are

not in a position to issue CP. (ii) CP can neither be redeemed before maturity nor can be extended beyond maturity. 5.2.6 Funds Generated from Operations: Funds generated from operations, during an accounting period, increase working capital by an equivalent amount. The two main components of funds generated from operations are profit and depreciation. Working capital will increase by the extent of funds generated from operations. Students may refer to funds flow statement given earlier in this chapter. 5.2.7 Public Deposits: Deposits from the public is one of the important source of finance particularly for well established big companies with huge capital base for short and medium-term. 5.2.8 Bills Discounting: Bill discounting is recognized as an important short term Financial Instrument and it is widely used method of short term financing. In a process of bill discounting, the supplier of goods draws a bill of exchange with direction to the buyer to pay a certain amount of money after a certain period, and gets its acceptance from the buyer or drawee of the bill. 5.2.9 Bill Rediscounting Scheme: The bill rediscounting Scheme was introduced by Reserve Bank of India with effect from 1st November, 1970 in order to extend the use of the bill of exchange as an instrument for providing credit and the creation of a bill market in India with a facility for the rediscounting of eligible bills by banks. Under the bills rediscounting scheme, all licensed scheduled banks are eligible to offer bills of exchange to the Reserve Bank for rediscount. 5.2.10 Factoring: Students may refer to the unit on Receivable Management wherein the concept of factoring has been discussed. Factoring is a method of financing whereby a firm sells its trade debts at a discount to a financial institution. In other words, factoring is a continuous arrangement between a financial institution, (namely the factor) and a firm (namely the client) which sells goods and services to trade customers on credit. As per this arrangement, the factor purchases the client’s trade debts including accounts receivables

7.81

Page 894: 30510870 Cost Accounting and Financial Management

Financial Management either with or without recourse to the client, and thus, exercises control over the credit extended to the customers and administers the sales ledger of his client. To put it in a layman’s language, a factor is an agent who collects the dues of his client for a certain fee. The differences between Factoring and Bills discounting are as follows: (i) Factoring is called as ‘Invoice factoring’ whereas bills discounting is known as “Invoice

discounting”. (ii) In factoring the parties are known as client, factor and debtor whereas in bills discounting

they are known as Drawer, Drawee and Payee. (iii) Factoring is a sort of management of book debts whereas bills discounting is a sort of

borrowing from commercial banks. (iv) For factoring there is no specific Act; whereas in the case of bills discounting, the

Negotiable Instrument Act is applicable.

5.3 WORKING CAPITAL FINANCE FROM BANKS Banks in India today constitute the major suppliers of working capital credit to any business activity. Recently, some term lending financial institutions have also announced schemes for working capital financing. The two committees viz., Tandon Committee and Chore Committee have evolved definite guidelines and parameters in working capital financing, which have laid the foundations for development and innovation in the area. 5.3.1 Instructions on Working Capital Finance by Banks Assessment of Working Capital • Reserve Bank of India has withdrawn the prescription, in regard to assessment of

working capital needs, based on the concept of Maximum Permissible Bank Finance, in April 1997. Banks are now free to evolve, with the approval of their Boards, methods for assessing the working capital requirements of borrowers, within the prudential guidelines and exposure norms prescribed. Banks, however, have to take into account Reserve Bank’s instructions relating to directed credit (such as priority sector, export, etc.), and prohibition of credit (such as bridge finance, rediscounting of bills earlier discounted by NBFCs) while formulating their lending policies.

• With the above liberalizations, all the instructions relating to MPBF issued by RBI from time to time stand withdrawn. Further, various instructions/guidelines issued to banks with objective of ensuring lending discipline in appraisal, sanction, monitoring and utilization of bank finance cease to be mandatory. However, banks have the option of incorporating such of the instructions/guidelines as are considered necessary in their lending policies/procedures.

7.82

Page 895: 30510870 Cost Accounting and Financial Management

Management of Working Capital

5.4 FACTORS DETERMINING CREDIT POLICY The bank credit will generally be in the following forms: • Cash Credit: This facility will be given by the banker to the customers by giving certain

amount of credit facility on continuous basis. The borrower will not be allowed to exceed the limits sanctioned by the bank.

• Bank Overdraft: It is a short-term borrowing facility made available to the companies in case of urgent need of funds. The banks will impose limits on the amount they can lend. When the borrowed funds are no longer required they can quickly and easily be repaid. The banks issue overdrafts with a right to call them in at short notice.

• Bills Discounting: The company which sells goods on credit, will normally draw a bill on the buyer who will accept it and sends it to the seller of goods. The seller, in turn discounts the bill with his banker. The banker will generally earmarks the discounting bill limit.

• Bills Acceptance: To obtain finance under this type of arrangement a company draws a bill of exchange on bank. The bank accepts the bill thereby promising to pay out the amount of the bill at some specified future date.

• Line of Credit: Line of Credit is a commitment by a bank to lend a certain amount of funds on demand specifying the maximum amount.

• Letter of Credit: It is an arrangement by which the issuing bank on the instructions of a customer or on its own behalf undertakes to pay or accept or negotiate or authorizes another bank to do so against stipulated documents subject to compliance with specified terms and conditions.

• Bank Guarantees: Bank guarantee is one of the facilities that the commercial banks extend on behalf of their clients in favour of third parties who will be the beneficiaries of the guarantees.

Self Examination Questions A. Objective Type Questions 1. The credit terms may be expressed as “3/15 net 60”. This means that a 3% discount will

be granted if the customer pays within 15 days, if he does not avail the offer he must make payment within 60 days. (a) I agree with the statement (b) I do not agree with the statement (c) I cannot say.

7.83

Page 896: 30510870 Cost Accounting and Financial Management

Financial Management 2. The term ‘net 50’ implies that the customer will make payment.

(a) Exactly on 50th day (b) Before 50th day (c) Not later than 50th day (iv) None of the above.

3. Trade credit is a source of : (a) Long-term finance (b) Medium term finance (c) Spontaneous source of finance (d) None of the above.

4. The term float is used in (a) Inventory Management (b) Receivable Management (c) Cash Management (d) Marketable securities.

5. William J Baumol’s model of Cash Management determines optimum cash level where the carrying cost and transaction cost are: (a) Maximum (b) Minimum (c) Medium (d) None of the above.

6. In Miller – ORR Model of Cash Management: (a) The lower, upper limit, and return point of Cash Balances are set out (b) Only upper limit and return point are decided (c) Only lower limit and return point are decided (d) None of the above are decided.

7. Working Capital is defined as (a) Excess of current assets over current liabilities (b) Excess of current liabilities over current assets

7.84

Page 897: 30510870 Cost Accounting and Financial Management

Management of Working Capital

(c) Excess of Fixed Assets over long-term liabilities (d) None of the above.

8. Working Capital is also known as “Circulating Capital, fluctuating Capital and revolving capital”. The aforesaid statement is; (a) Correct (b) Incorrect (c) Cannot say.

9. The basic objectives of Working Capital Management are: (a) Optimum utilization of resources for profitability (b) To meet day-to-day current obligations (c) Ensuring marginal return on current assets is always more than cost of capital (d) Select any one of the above statement.

10. The term Gross Working Capital is known as: (a) The investment in current liabilities (b) The investment in long-term liability (c) The investment in current assets (d) None of the above.

11. The term net working capital refers to the difference between the current assets minus current liabilities. (a) The statement is correct (b) The statement is incorrect (c) I cannot say.

12. The term “Core current assets’ was coined by (a) Chore Committee (b) Tandon Committee (c) Jilani Committee (d) None of the above.

13. The concept operating cycle refers to the average time which elapses between the acquisition of raw materials and the final cash realization. This statement is

7.85

Page 898: 30510870 Cost Accounting and Financial Management

Financial Management

(a) Correct (b) Incorrect (c) Partially True (d) I cannot say.

14. As a matter of self-imposed financial discipline can there be a situation of zero working capital now-a-days in some of the professionally managed organizations. (a) Yes (b) No (c) Impossible (d) Cannot say.

15. Over trading arises when a business expands beyond the level of funds available. The statement is (a) Incorrect (b) Correct (c) Partially correct (d) I cannot say.

16. A Conservative Working Capital strategy calls for high levels of current assets in relation to sales. (a) I agree (b) Do not agree (c) I cannot say.

17. The term Working Capital leverage refer to the impact of level of working capital on company’s profitability. This measures the responsiveness of ROCE for changes in current assets. (a) I agree (b) Do not agree (c) The statement is partially true.

18. The term spontaneous source of finance refers to the finance which naturally arise in the course of business operations. The statement is (a) Correct

7.86

Page 899: 30510870 Cost Accounting and Financial Management

Management of Working Capital

(b) Incorrect (c) Partially Correct (d) I cannot say.

19. Under hedging approach to financing of working capital requirements of a firm, each asset in the balance sheet assets side would be offset with a financing instrument of the same approximate maturity. This statement is (a) Incorrect (b) Correct (c) Partially correct (d) I cannot say.

20. Trade credit is a (a) Negotiated source of finance (b) Hybrid source of finance (c) Spontaneous source of finance (d) None of the above.

21. Factoring is a method of financing whereby a firm sells its trade debts at a discount to a financial institution. The statement is (a) Correct (b) Incorrect (c) Partially correct (d) I cannot say.

22. A factoring arrangement can be both with recourse as well as without recourse: (a) True (b) False (c) Partially correct (d) Cannot say.

23. The Bank financing of working capital will generally be in the following form. Cash Credit, Overdraft, bills discounting, bills acceptance, line of credit; Letter of credit and bank guarantee. (a) I agree

7.87

Page 900: 30510870 Cost Accounting and Financial Management

Financial Management

(b) I do not agree (c) I cannot say.

24. When the items of inventory are classified according to value of usage, the technique is known as: (a) XYZ Analysis (b) ABC Analysis (c) DEF Analysis (d) None of the above.

25. When a firm advises its customers to mail their payments to special Post Office collection centers, the system is known as. (a) Concentration banking (b) Lock Box system (c) Playing the float (d) None of the above.

Answer to Objective Type Questions 1. (a); 2.(c); 3. (c); 4. (c); 5. (b); 6. (a); 7. (a); 8. (a); 9. (b); 10. (c); 11. (a); 12. (b); 13. (a); 14. (a); 15. (b); 16. (a); 17. (a); 18. (a); 19. (b); 20. (c); 21. (a); 22. (a); 23. (a); 24. (b); 25. (b). B. Practical Problems 1. Foods Ltd. is presently operating at 60% level producing 36,000 packets of snack foods

and proposes to increase capacity utilization in the coming year by 33�% over the existing level of production. The following data has been supplied: (i) Unit cost structure of the product at current level: Rs.

Raw Material 4 Wages (Variable) 2 Overheads (Variable) 2 Fixed Overheads 1 Profit 3 Selling Price 12

7.88

Page 901: 30510870 Cost Accounting and Financial Management

Management of Working Capital

(ii) Raw materials will remain in stores for 1 month before being issued for production. Material will remain in process for further 1 month. Suppliers grant 3 months credit to the company.

(iii) Finished goods remain in godown for 1 month. (iv) Debtors are allowed credit for 2 months. (v) Lag in wages and overhead payments is 1 month and these expenses accrue

evenly throughout the production cycle. (vi) No increase either in cost of inputs or selling price is envisaged. Prepare a projected profitability statement and the working capital requirement at the new level, assuming that a minimum cash balance of Rs.19,500 has to be maintained.

2. A newly formed company has applied to the commercial bank for the first time for financing its working capital requirements. The following information is available about the projections for the current year: Estimated level of activity: 1,04,000 completed units of production plus 4,000 units of work-in-progress. Based on the above activity estimated cost per unit is:

(Rs. per unit)

Raw material 80 Direct wages 30 Overheads (exclusive of depreciation) 60 Total cost 170 Selling price 200 Raw materials in stock: average 4 weeks consumption, work-in-progress (assume 50% completion stage in respect of conversion cost) (materials issued at the start of the processing).

Finished goods in stock 8,000 units Credit allowed by suppliers Average 4 weeks Credit allowed to debtors/receivables Average 8 weeks Lag in payment of wages Average 1½ weeks Cash at banks (for smooth operation) is expected to be Rs.25,000 Assume that production is carried on evenly throughout the year (52 weeks) and wages and overheads accrue similarly. All sales are on credit basis only. Find out: the net working capital required.

7.89

Page 902: 30510870 Cost Accounting and Financial Management

Financial Management 3. The following is the projected Balance Sheet of Excel Limited as on 31-3-2004. The

company wants to increase the fund-based limits from the Zonal Bank from Rs.100 lakhs to Rs.300 lakhs:

Balance Sheet as on 31-3-2004 (Rs. lakhs)

Liabilities Rs. Assets Rs. Share Capital 100 Fixed Assets 800 Reserves & Surplus 150 Current Assets 1,000 Secured Loans 450 Miscellaneous Expenditure 150 Unsecured Loans 1,050 Current Liabilities 200 1,950 1,950

The following are the other information points to be considered: (1) Secured loans include instalments payable to financial institutions before 31-3-2004

Rs.100 lakhs. (2) Secured loans include working capital facilities expected from Zonal Bank Rs.300

lakhs. (3) Unsecured loans include fixed deposits from public amounting to Rs.400 lakhs out

of which Rs.100 lakhs are due for repayment before 31-3-2004. (4) Unsecured loans include Rs.600 lakhs of zero interest fully convertible debentures

due for conversion on 30-9-2003. (5) Current assets include deferred receivables due for payment after 31-3-2004 Rs.40

lakhs. (6) The company has introduced a voluntary retirement scheme for workers costing

Rs.40 lakhs payable on 31-3-2008 and this amount is included in current liabilities: (i) You are required to calculate from the above information the maximum

permissible bank finance by all the three methods for working capital as per Tandon Committee norms. For your exercise, assume that core current assets constitute 25% of the current assets.

(ii) Also compute the Current Ratio for all the three methods. 4. A company newly commencing business in 2003 has the under mentioned Projected

Profit and Loss Account:

7.90

Page 903: 30510870 Cost Accounting and Financial Management

Management of Working Capital

Sales 42,00,000 Cost of goods sold 30,60,000 Gross Profit 11,40,000 Administrative expenses 2,80,000 Selling expenses 2,60,000 5,40,000 Profit before tax 6,00,000 Provision for taxation 2,00,000 Profit after tax 4,00,000 The cost of goods sold has been arrived at as under (Rs)

Material used 16,80,000 Wages and manufacturing expenses 12,50,000 Depreciation 4,70,000 34,00,000 Less: Stock of finished goods (10% of goods produced not yet sold) 3,40,000 30,60,000 The figures given above relate only to finished goods and not to work-in-progress. Goods equal to 15% of the year’s production (in terms of physical units) will be in process on the average requiring full materials but only 40% of the other expenses. The company believes in keeping material equal to two months consumption in stock. All expenses will be paid one month in arrear. Suppliers of material will extend 1½ month’s credit; Sales will be 20% for cash and the rest at two months’ credit; 90% of the Income-tax will be paid in advance in quarterly instalments. The company wishes to keep Rs.1,00,000 in cash. Prepare an estimate of the requirement of (i) Working Capital; and (ii) Cash Cost of Working Capital.

5. A company is considering its working capital investment and financial policies for the next year. Estimated fixed assets and current liabilities for the next year are Rs.2.60 crore and Rs.2.34 crore respectively. Estimated Sales and EBIT depend on current assets investment, particularly inventories and book-debts. The Financial Controller of the company is examining the following alternative Working Capital Policies:

(Rs. Crores)

Working Capital Policy Investment in Estimated EBIT

7.91

Page 904: 30510870 Cost Accounting and Financial Management

Financial Management

Current Assets Sales Conservative 4.50 12.30 1.23 Moderate 3.90 11.50 1.15 Aggressive 2.60 10.00 1.00 After evaluating the working capital policy, the Financial Controller has advised the adoption of the moderate working capital policy. The company is now examining the use of long-term and short-term borrowings for financing its assets. The company will use Rs.2.50 crore of the equity funds. The corporate tax rate is 35%. The company is considering the following debt alternatives:

Financing Policy Short-term Debt Long-term Debt Conservative 0.54 1.12 Moderate 1.00 0.66 Aggressive 1.50 0.16 Interest rate- Average 12% 16% You are required to calculate the following: (a) Working Capital Investment for each Policy: (a) Net Working Capital position; (b)

Rate of Return; (c) Current ratio. (b) Financing for each policy; (a) Net Working Capital; (b) Rate of Return of

Shareholders equity; (c) Current ratio. 6. The turnover of R Ltd. is Rs.60 lakhs of which 80% is on credit. Debtors are allowed one

month to clear off the dues. A factor is willing to advance 90% of the bills raised on credit for a fee of 2% a month plus a commission of 4% on the total amount of debts. R Ltd. as a result of this arrangement is likely to save Rs.21,600 annually in management costs and avoid bad debts at 1% on the credit sales. A scheduled bank has come forward to make an advance equal to 90% of the debts at an interest rate of 18% p.a. However, its processing fee will be at 2% on the debts. Would you accept factoring or the offer from the bank?

7. A Bank is analyzing the receivables of Jackson Company in order to identify acceptable collateral for a short-term loan. The company’s credit policy is 2/10 net 30. the bank lends 80 per cent on accounts where customers are not currently overdue and where the average payment period does not exceed 10 days past the net period. A schedule of Jackson’s receivables has been prepared. How much will the bank lend on a pledge of receivables, if the bank uses a 10 per cent allowance for cash discount and returns?

Account Amount Days Outstanding Average Payment

7.92

Page 905: 30510870 Cost Accounting and Financial Management

Management of Working Capital

Rs. In days Period historically 74 25,000 15 20 91 9,000 45 60

107 11,500 22 24 108 2,300 9 10 114 18,000 50 45 116 29,000 16 10 123 14,000 27 48

8. A Ltd. has a total sales of Rs.3.2 crores and its average collection period is 90 days. The past experience indicates that bad debt losses are 1.5% on Sales. The expenditure incurred by the firm in administering its receivable collection efforts are Rs.5,00,000. A factor is prepared to buy the firm’s receivables by charging 2% commission. The factor will pay advance on receivables to the firm at an interest rate of 18% p.a. after withholding 10% as reserve. Calculate the effective cost of factoring to the Firm.

9. Explain briefly some of the techniques of inventory control used in manufacturing organization.

10. Ten items kept in inventory by the School of Management Studies at State University are listed below. Which items should be classified as ‘A’ items, ‘B’ items and ‘C’ items? What percentage of items is in each class? What percentage of total annual value is in each class?

Item Annual Usage Value per unit (Rs.) 1 200 40.00 2 100 360.00 3 2,000 0.20 4 400 20.00 5 6,000 0.04 6 1,200 0.80 7 120 100.00 8 2,000 0.70 9 1,000 1.00

10 80 400.00 11 Economic Enterprises require 90,000 units of a certain item annually. The cost per unit

is Rs.3, the cost per purchase order is Rs.300 and the Inventory carrying cost Rs.6 per unit per year.

7.93

Page 906: 30510870 Cost Accounting and Financial Management

Financial Management

(i) What is the Economic Order Quantity. (ii) What should the firm do if the supplier offers discounts as below, viz.,

Order quantity Discounts% 4,500 – 5,999 2 6,000 and above 3

12. The annual demand for an item of raw material is 4,000 units and the purchase price is

expected to be Rs.90 per unit. The incremental cost processing an order is Rs.135 and the cost of storage is estimated to be Rs.12 per unit. What is the optimal order quantity and total relevant cost of this order quantity? Suppose that Rs.135 as estimated to be the incremental cost of processing an order is incorrect and should have been Rs.80. All other estimates are correct. What is the difference in cost on account of this error? Assume at the commencement of the period that a supplier offers 4,000 units at a price of Rs.86. The materials will be delivered immediately and placed in the stores. Assume that the incremental cost of placing the order is zero and original estimate of Rs.135 for placing an order for the economic batch is correct. Should the order be accepted?

13. (a) The following details are available in respect of a firm:

(i) Annual requirement of inventory 40,000 units (ii) Cost per unit (other than carrying and ordering cost) Rs.16 (iii) Carrying costs are likely to be 15% per year (iv) Cost of placing order Rs.480 per order

Determine the economic ordering quantity. (b) The experience of the firm being out of stock is summarized below:

(1) Stock out (No. of units) No. of times 500 1 (1) 400 2 (2) 250 3 (3) 100 4 (4) 50 10 (10) 0 80 (80)

7.94

Page 907: 30510870 Cost Accounting and Financial Management

Management of Working Capital

Figures in brackets indicate percentage of time the firm has been out of stock. (2) Stock out costs are Rs.40 per unit. (3) Carrying cost of inventory per unit is Rs.20. Determine the optimal level of stock out inventory.

(c) A firm has 5 different levels in its inventory. The relevant details are given. Suggest a breakdown of the items into A, B and C

classifications:

Item No. Avg. No. of units inventory Avg. Cost per unit (Rs.) 1 20,000 60 2 10,000 100 3 32,000 11 4 28,000 10 5 60,000 3.40

14. A firm is engaged in the manufacture of two products A and B. Product A uses one unit

of Component P and two units of Components Q. Products B uses two units of Component P, one unit of Component Q and two units of Component R. Component R which is assembled in the factory uses one unit of Component Q. Components P and Q are purchased from the market. The firm has prepared the following forecast for sales and inventory for the next year: Products A B Sales Units 8,000 15,000 Inventories: At the end of the year Units 1,000 2,000 At the beginning of the year Units 3,000 5,000 The production of both the products and the assembling of the component R will be spread out uniformly throughout the year. The firm at present orders its inventory of components P and Q in quantities equivalent to 3 months consumption. The firm has been advised that savings in the provisioning of components can arise by changing over to the ordering system based on economic

7.95

Page 908: 30510870 Cost Accounting and Financial Management

Financial Management

ordering quantities. The firm has compiled the following data relating to the two Components:

(Rs.)

Particulars P Q Component usage per annum 30,000 48,000 Price per unit 2.00 0.80 Order placing costs per order 15.00 15.00 Carrying costs per annum 20% 20% Required: (a) Prepare a budget of production and requirements of components for the next year. (b) Find the economic order quantity. (c) Based on the economic order quantity calculated in (b) above, calculate the savings

arising from switching over to the new ordering system both in terms of cost and reduction in working capital.

15. Radiance Garments Ltd. manufactures readymade garments and sells them on credit basis through a network of dealers. Its present sale is Rs.60 lakh per annum with 20 days credit period. The company is contemplating an increase in the credit period with a view to increasing sales. Present variable costs are 70% of sales and the total fixed costs Rs.8 lakh per annum. The company expects pre-tax return on investment @ 25%. Some other details are given as under:

Proposed Credit Policy Average Collection Period (days) Expected Annual Sales (Rs.lakh)

I 30 65 II 40 70 III 50 74 IV 60 75

Required: When credit policy should the company adopt? Present your answer in a tabular form. Assume 360 days a year. Calculation should be made upto two digits after decimal.

16. H. Ltd. has a present annual sales level of 10,000 units at Rs.300 per unit. The variable cost is Rs.200 per unit and the fixed costs amount to Rs.3,00,000 per annum. The

7.96

Page 909: 30510870 Cost Accounting and Financial Management

Management of Working Capital

present credit period allowed by the company is 1 month. The company is considering a proposal to increase the credit periods to 2 months and 3 months and has made the following estimates:

Existing Proposal Credit Policy 1 month 2 months 3 months Increase in Sales - 15% 30% % of Bad Debts 1% 3% 5% There will be increase in fixed cost by Rs.50,000 on account of increase of sales beyond 25% of present level. The company plans on a pretax return of 20% on investment in receivables. You are required to calculate the most paying credit policy for the company.

17. Star Limited manufacturers of Colour T.V. sets, are considering the liberalization of existing credit terms to three of their large Customers A,B and C. The credit period and likely quantity of TV sets that will be lifted by the customers are as follows:

Quantity Lifted (No. of TV Sets) Credit Period (Days) A B C 0 1,000 1,000 − 30 1,000 1,500 − 60 1,000 2,000 1,000 90 1,000 2,500 1,500 The selling price per TV set is Rs.9,000. The expected contribution is 20% of the selling price. The cost of carrying debtors averages 20% per annum. You are required: (a) Determine the credit period to be allowed to each customer. (Assume 360 day in a

year for calculation purposes). (b) What other problems the company might face in allowing the credit period as

determined in (a) above?

7.97

Page 910: 30510870 Cost Accounting and Financial Management

Financial Management 18. The present credit terms of P Company are 1/10 net 30. Its annual sales are Rs.80

lakhs, its average collection period is 20 days. Its variable costs and average total costs to sales are 0.85 and 0.95 respectively and its cost of capital is 10 per cent. The proportion of sales on which customers currently take discount is 0.5. P Company is considering relaxing its discount terms to 2/10 net 30. Such relaxation is expected to increase sales by Rs.5 lakhs, reduce the average collection period to 14 days and increase the proportion of discount to sales to 0.8. What will be the effect of relaxing the discount policy on company’s profit? Take year as 360 days.

19. The credit manager of XYZ Ltd. is reappraising the company’s policy. The company sells its products on terms of net 30. Cost of goods sold is 85% of sales and fixed costs are further 5% of sales. XYZ classifies its customers on a scale of 1 to 4. During the past five years, the experience was as under: Classification Default as a percentage of

sales Average collection period-in-

days for non-defaulting accounts

1 0 45 2 2 42 3 10 40 4 20 80 The average rate of interest is 15%. What conclusions do you draw about the Company’s Credit Policy? What other factors should be taken into account before changing the present policy? Discuss.

20. Easy Limited specializes in the manufacture of a computer component. The component is currently sold for Rs.1,000 and its variable cost is Rs.800. For the year ended 31-3-2006 the company sold on an average 400 components per month. At present the company grants one month credit to its customers. The company is thinking of extending the same to two months on account of which the following is expected: Increase in Sales 25% Increase in Stock Rs.2,00,000 Increase in Creditors Rs.1,00,000 You are required: To advise the company on whether or not to extend the credit terms if: (a) all customers avail the extended credit period of two months; and

7.98

Page 911: 30510870 Cost Accounting and Financial Management

Management of Working Capital

(b) existing customers do not avail the credit terms but only the new customers avail the same. Assume in this case the entire increase in sales is attributable to the new customers.

21 Star Limited is manufacturer of various electronic gadgets. The annual turnover for the year 2006 was Rs.730 lakhs. The company has a wide network of sales outlets all over the country. The turnover is spread evenly for each of the 50 weeks of the working year. All sales are for credit and sales within the week are also spread evenly over each of the five working days. All invoicing of credit sales is carried out at the Head Office in Bombay. Sales documentation is sent by post daily from each location to the Head Office for the past two years. Delays in preparing and dispatching invoices were noticed. As a result, only some of the invoices were dispatched in the same week and the remainder the following week. An analysis of the delay in invoicing (being the interval between the date of sale and the date of despatch of the invoice indicated the following pattern:

No. of days of delay in invoicing 3 4 5 6 % of weeks sales 20 10 40 30 A further analysis indicated that the debtors take on an average 36 days of credit before paying. This period is measured from the day of despatch of the invoice rather than the date of sale. It is proposed to hire an agency for undertaking the invoicing work at various locations. The agency has assured that the maximum delay would be reduced to three days under the following pattern:

No. of days of delay in invoicing 0 1 2 % of weeks sales 40 40 20 The agency has also offered additionally to monitor the collections which will reduce the credit period to 30 days. Star Limited expects to save Rs.4,000 per month in postage costs. All working funds are borrowed from a local bank at simple interest rate of 20% p.a. The agency has quoted a fee of Rs.2,00,000 p.a. for the invoicing work and Rs.2,50,000 p.a. for monitoring collections and is willing to offer a discount of Rs.50,000 provided both the works are given. You are required to advise Star Limited about the acceptance of agency’s proposal. Working should form part of the answer.

7.99

Page 912: 30510870 Cost Accounting and Financial Management

Financial Management 22. Pollock Co. Pvt. Ltd., which is operating for the last 5 years, has approached Sudershan

Industries for grant of credit limit on account of goods bought from the latter, annexing Balance Sheet and Income Statement for the last 2 years as below:

Pullock Co. Pvt. Ltd. – Balance Sheet (Rs. ‘000)

Liabilities Current year

Last year

Assets Current year

Last year

Share Capital Equity (Rs.10)

600

600

Plant & Equipment (Less Depreciation)

1,500

1,400

Share Premium 400 400 Land 750 750 Retained Earnings 900 700 Total Equity 1,900 1,700 Total Fixed Assets 2,250 2,150 First Mortgage 200 300 Inventories 580 300 Second Mortgage -- 200 Account Receivable 350 200 Bonds 300 300 Marketable Securities 120 120 Long-term Liabilities 500 800 Cash 100 80 Account Payable 300 60 Total Current Assets 1,150 700 Notes Payable 600 220 Secured Liabilities 100 70 Total Current Liabilities

1,000

350

3,400 2,850 3,400 2,850

7.100

Page 913: 30510870 Cost Accounting and Financial Management

Management of Working Capital

Pollock Co. Pvt. Ltd. – Income Statement (Rs.’000)

Particulars Current Year

Last Year

Sales 5,980 5,780 Income from investments 20 6,000 20 5,800 Opening inventory 300 400 Total Manufacturing Costs 4,200 3,200 Ending Inventory (580) 3,920 (300) 3,300 2,080 2,500 General and Admn. Expenses 950 750 Operating Income 1,130 1,750 Interest Expenses 60 62 Earnings before Taxes 1,070 1,688 Income-tax 480 674 Net Income after Taxes 590 1,014 Dividend declared and paid 250

Sudershan Industries has established the following broad guidelines for granting credit limits to its customers: (i) Limit credit limit to 10% of net worth and 20% of the net working capital. (ii) Not to give credit in excess of Rs.1,00,000 to any single customer. You are required to detail the steps required for establishing credit limits to Pollock Co. Pvt. Ltd. In this case what you consider to be reasonable credit limit.

23 The annual cash requirement of A Ltd. is Rs.10 lakhs. The company has marketable securities in lot sizes of Rs.50,000, Rs.1,00,000, Rs.2,00,000, Rs.2,50,000 and Rs.5,00,000. Cost of conversion of marketable securities per lot is Rs.1,000. The company can earn 5% annual yield on its securities. You are required to prepare a table indicating which lot size will have to be sold by the company. Also show that the economic lot size can be obtained by the Baumol Model.

24. JPL has two dates when it receives its cash inflows, i.e., Feb. 15 and Aug. 15. On each of these dates, it expects to receive Rs.15 crore. Cash expenditure are expected to be steady throughout the subsequent 6 month period. Presently, the ROI in marketable

7.101

Page 914: 30510870 Cost Accounting and Financial Management

Financial Management

securities is 8% per annum, and the cost of transfer from securities to cash is Rs.125 each time a transfer occurs. (i) What is the optimal transfer size using the EOQ model? What is the average cash

balance? (ii) What would be your answer to part (i), if the ROI were 12% per annum and the

transfer costs were Rs.75/-? Why do they differ from those in part (i)? 25. Beta Limited has an annual turnover of Rs.84 crores and the same is spread over evenly

each of the 50 weeks of the working year. However, the pattern within each week is that the daily rate of receipts on Mondays and Tuesdays is twice that experienced on the other three days of the week. The cost of banking per day is estimated at Rs.2,500. It is suggested that banking should be done daily or twice a week. Tuesdays and Fridays as compared to the current practice of banking only on Fridays. Beta Limited always operates on bank overdraft and the current rate of interest is 15% per annum. This interest charge is applied by the bank on a simple daily basis. Ignoring taxation, advise Beta Limited the best course of banking. For your exercise, use 360 days a year for computational purposes.

26. The following is the Balance Sheet of Amar Industries Ltd. as at 31st March, 2006: (Rs. lakhs)

Liabilities Capital and Reserves 1,650 12% Debentures 900 Creditors for purchases 600 Creditors for expenses 70 Provision for bonus 30 Provision for tax 100 Proposed dividends 50 3,400 Assets Fixed Assets at cost 1,300 Less: Depreciation 400 900 Sundry debtors 700 Stocks and stores 1,200

7.102

Page 915: 30510870 Cost Accounting and Financial Management

Management of Working Capital

Loans and advances 500 Cash and bank balances 100 3,400

The projected Profit and Loss Account for the first four months in 2006-2007 shows the following:

(Rs.lakhs)

Particulars April May June July Sales 800 800 900 900 Excise duty recoveries 80 80 90 90 (a) 880 880 990 990 Materials: Opening Stock 1200 1200 1260 1320 Add: Purchases 600 660 720 720 Less: Closing Stock 1200 1260 1320 1320 Net 600 600 660 720 Expenses 180 180 200 200 Excise duty 80 84 88 92 (b) 860 864 948 1012 Profi/(Loss)t (a)-(b) 20 16 42 (22)

The following are relevant additional information: (i) 10% of sales are for cash and the balance on 30 days’ credit. (ii) Creditors for purchases are paid in 30 days. (iii) Expenses include:

(a) interest payable at the end of each quarter. (b) depreciation of Rs.10 lakhs per month; (c) provision for bonus to workmen of Rs.5 lakhs per month, payable only in

October, 2006. (d) one-half of rest of the expenses payable in the month following.

7.103

Page 916: 30510870 Cost Accounting and Financial Management

Financial Management

(iv) Rs.250 lakhs of debentures are redeemable by 30th June. (v) Provision for taxation includes Rs.70 lakhs of surplus provision carried forward from

earlier years besides the balance for the year 2005-2006 payable before 30th June, 2006.

(vi) Annual General Meeting is to be held on 31st May, 2006. (vii) Over-draft is permissible interest on which may be ignored. You are required to

prepare the cash budgets for the months of April to July, 2006 on a monthly basis. 27. The following information is available in respect of ABC Ltd:

(1) Materials are purchased and received one month before being used and payment is made to suppliers two months after receipt of materials.

(2) Cash is received from customers three months after finished goods are sold and delivered to them.

(3) No time lag applies to payment of wages and expenses. (4) The following figures apply to recent and future months:

Month Materials received Sales Wages and Expenses January 20,000 30,000 9,500 Febuary 22,000 33,000 10,000 March 24,000 36,000 10,500 April 26,000 39,000 11,000 May 28,000 42,000 11,500 June 30,000 45,000 12,000 July 32,000 48,000 12,500 August 34,000 51,000 13,000

(5) Cash balance at the beginning of April is Rs.10,000. (6) all products are sold immediately they have been made and that materials used and

sums spent on wages and expenses during any particular month relate strictly to the sales made during that month.

Prepare cash flow forecast month by month from April to July, profit and loss forecast for four months (April-July) and a movement of funds statement for the four-month period (April-July).

7.104

Page 917: 30510870 Cost Accounting and Financial Management

Management of Working Capital

28. Fixed overheads excluding bank interest amount to Rs.6,00,000 per annum spread out evenly throughout the year. Sales forecast is as under:

Product July August Sept. Oct. Nov. 2005 L 4,200 4,600 3,600 4,000 4,500 B 2,100 2,300 1,800 2,000 1,900

Production: 75% of each months’ sales will be produced in the month of sale and 25% in the previous months. Sales Pattern: L: -One-third of sales will be on cash basis on which cash discount of 2% is

allowed. -One third will be on documents against payment basis. The documents will be discounted by the bank in the month of sales itself.

-Balance of one-third will be on documents against acceptance basis. The payment under this scheme will be received in the third month. For e.g. for sales made in September, payment will be received in November.

B: 80% of the sales will be against cash to be received in the month of sales and the balance 20% will be received next following month.

Direct Materials: 50% of the direct materials required for each month’s production will be purchased in the previous month and the balance in the month of production itself. The payment will be made in the month next following the purchase. Direct Wages: 80% of the direct wages will be paid in the month of use of direct labour for production and the balance in the next following month. Variable Overheads: 50% to be paid in the month of incurrence and the balance in the next following month. Fixed Overheads: 40% will be paid in the month of incurrence and the other 40% in the next following month. The balance of 20% represents depreciation. The bill discounting charges payable to the bank in the month in which the bills are discounted amount to 50 paise per Rs.100 of bills discounted. A cash balance of Rs.1,00,000 will be maintained on 1st July, 2006. Prepare a cash budget monthwise for July, August and September, 2006.

7.105

Page 918: 30510870 Cost Accounting and Financial Management

Financial Management 29. A new manufacturing company is to be incorporated from January 1, 2003. Its

authorised capital will be Rs.2 crore divided into 20 lakh equity shares of Rs.10 each. It intends to raise capital by issuing equity shares of Rs.1 crore (fully paid) on 1st January. Besides, a loan of Rs.13 lakh @12% per annum will be obtained from a financial institution on 1st January and further borrowings will be made at the same rate of interest on the first day of the month in which borrowing is required. All borrowings will be repaid alongwith interest on the expiry of one year. The company will make payment for the following assets in January:

(Rs. lakhs)

Plant and Machinery 20 Land and Building 40 Furniture 10 Motor Vehicles 10 Stock of Raw Materials 10 The following further details are available: (i) Projected Sales (January-June):

(Rs. lakhs) (Rs. lakhs) January 30 April 40 February 35 May 40 March 35 June 45

(ii) Gross Profit will be 25% on sales. (iii) The company will make credit sales only and these will be collected in the second

month following sales. (iv) Creditors will be paid in the first month following credit purchases. There will be

credit purchases only. (v) The company will keep minimum stock of raw materials of Rs.10 lakhs. (vi) Depreciation will be charged @10% per annum on cost on all fixed assets. (vii) Payment of preliminary expenses of Rs.1 lakh will be made in January.

7.106

Page 919: 30510870 Cost Accounting and Financial Management

Management of Working Capital

(viii) Wages and salaries will be Rs.2 lakhs each month and will be paid on the first day of the next month.

(ix) Administrative expenses of Rs.1 lakh per month will be paid in the month of their incurrence.

Assume no minimum required cash balance. You are required to prepare the monthly cash budget (January-June), the projected Income Statement for the 6 month period and the projected Balance Sheet as on 30th June, 2003.

30. Dyer Ltd. manufactures a variety of products using a standardized process, which takes one month to complete. Each production batch is started at the beginning of a month and is transferred to finished goods at the beginning of the next month. The cost structure, based on current selling price is:

%

Sale Price 100 Variable Costs Raw Materials 30 Other Variable Costs 40 Total Variable Cost – used for Stock Valuation 70 Contribution 30

Activity levels are constant throughout the year and annual sales, all of which are made on credit are Rs.24,00,000. Dyer Ltd. is now planning to increase sales volume by 50% and unit sales price by 10%, such expansion would not alter the fixed costs of Rs.50,000 per month, which includes monthly depreciation of plant of Rs.10,000. Similarly raw material and other variable costs per unit will not alter as a result of the price rise. In order to facilitate the envisaged increases several changes would be required in the long run. The relevant changes are:- (i) The average credit period allowed to customers will increase to 70 days; (ii) Suppliers will continue to be paid on strictly monthly terms; (iii) Raw material stocks held will continue to be sufficient for one month’s production; (iv) Stocks of finished goods held will increase to one month’s output;

7.107

Page 920: 30510870 Cost Accounting and Financial Management

Financial Management

(v) There will be no change in the production period and other variable costs will continue to be paid for in the month of production;

(vi) The current end-of-month working capital position is: (Rs.’000)

Raw Materials 60 WIP 140 Finished Goods 70 270 Debtors 200 470 Creditors 60 Net Working Capital – Excluding Cash 410 Compliance with the long-term changes required by the expansion will be spread over several months. The relevant points concerning the transitional arrangements are: (i) The cash balance anticipated at the end of the May is Rs.80,000. (ii) Upto and including June all sales will be made on one month’s credit. From July all

sales will be on the transitional credit terms which will mean: 60% of sales will take 2 months’ credit. 40% of sales will take 3 months’ credit. (iii) Sale price increase will occur with effect from sales in the month of August. (iv) Production will increase by 50% with effect from the month of July. Raw material

purchases made in June will reflect this. (v) Sales volume will increase by 50% from sales made in October. Required: (a) Show the long-term increase in annual profit and long-term working capital

requirements as a result of the plans for expansion and a price increase. (Costs of financing the extra working capital requirements may be ignored).

(b) Produce a monthly cash forecast for the period from June to December, the first seven months of the transitional period. Prepare also a working capital position at the end of December.

7.108

Page 921: 30510870 Cost Accounting and Financial Management

Management of Working Capital

(c) Using your findings for (a) and (b) above, make brief comments to the management of Dyer Ltd. on the major factors concerning the financial aspects of the expansion which should be brought to their attention.

Assume that there are 360 days in a year and each month contains 30 days.

7.109